You are on page 1of 381

More PDF download : http://www.gksolve.

com

UP K A R ' S

For Bank P.O. & Other Competitive Exams.

By
Dr. N. K. Singh

More PDF download : http://www.gksolve.com

UPKAR PRAKASHAN, AGRA–2


More PDF download : http://www.gksolve.com

© Publishers

Publishers
UPKAR PRAKASHAN
(An ISO 9001 : 2000 Company)

2/11A, Swadeshi Bima Nagar, AGRA–282 002


Phone : 4053333, 2530966, 2531101
Fax : (0562) 4053330, 4031570
E-mail : care@upkar.in, Website : www.upkar.in
Branch Offices :
4845, Ansari Road, Daryaganj, Pirmohani Chowk, 1-8-1/B, R.R. Complex (Near Sundaraiah
New Delhi—110 002 Kadamkuan, Park, Adjacent to Manasa Enclave Gate),
Phone : 011–23251844/66 Patna—800 003 Bagh Lingampally,
Phone : 0612–2673340 Hyderabad—500 044 (A.P.)
Phone : 040–66753330
28, Chowdhury Lane, Shyam B-33, Blunt Square, Kanpur
Bazar, Near Metro Station, Taxi Stand Lane, Mawaiya,
Gate No. 4 Lucknow—226 004 (U.P.)
Kolkata—700004 (W.B.) Phone : 0522–4109080
Phone : 033–25551510

● The publishers have taken all possible precautions in publishing this book, yet if any mistake
has crept in, the publishers shall not be responsible for the same.
● This book or any part thereof may not be reproduced in any form by Photographic,
Mechanical, or any other method, for any use, without written permission from the
Publishers.
● Only the courts at Agra shall have the jurisdiction for any legal dispute.

ISBN : 978-81-7482-627-5
Price : 285·00
(Rs. Two Hundred Eighty Five Only)
Code No. 1613

Printed at : UPKAR PRAKASHAN (Printing Unit) Bye-pass, AGRA


More PDF download : http://www.gksolve.com

Contents

● Average……………………………………………………………….....…………… 3–10
● Time and Distance…………………………………………………………………… 11–17
● Area………………………………….……………………………………………..… 18–31
● Numbers……………………………………………………………………………… 32–45
● Decimal Fraction………………………………………………...…………………… 46–52
● Unitary Method………………………………………………….…………………… 53–54
● Square Root and Cube Root…………………………………………..……………… 55–61
● Simplification………………………………………………………………………… 62–68
● Ratio and Proportion……………………………………………………….………… 69–77
● Logarithm……………………………………………………………………..……… 78–84
● H.C.F. and L.C.M.…………………………………………………………………… 85–91
● Series…………………………………………………………………………….…… 92–105
● Permutation and Combination…………………………………………………..…… 106–114
● Simple Interest…………………………………………………………………..…… 115–122
● Compound Interest…………………………………………………………………… 123–131
● Problems Based on Ages……………………………………………..……………… 132–140
● Percentage………………………………………………………………….………… 141–151
● Profit and Loss………………………………………………………………..……… 152–163
● Time and Work……………………………………………………………….……… 164–174
● Probability………………………………………………………………………….… 175–187
● Partnership…………………………………………………………………………… 188–194
● Volume and Surface Area…………………………………………………………… 195–203
● Races and Games of Skill……………………………….…………………………… 204–210
● Odd Man Out and Series……………………………………………………………… 211–214
● Calendar……………………………………………………………………………… 215–216
● Chain Rule…………………………………………………………………………… 217–222
● Boats and Streams…………………………………………………………………… 223–228
● Trains………………………………………………………………………………… 229–237
More PDF download : http://www.gksolve.com
( iv )

● Pipes and Cisterns……………………………………….…………………………… 238–244


● Alligation or Mixture………………………………………………………………… 245–254
● Stock and Shares………………………………………...…………………………… 255–261
● True Discount………………………………………………………………………… 262–267
● Banker’s Discount…………………………………………………………………… 268–273
● Data Analysis………………………………………………………………………… 274–283
● Tabulation……………………………………………………….…………………… 284–289
● Miscellaneous Exercise-I………………………..…………………………………… 290–303
● Miscellaneous Exercise-II……………….…………………………………………… 304–316
● Miscellaneous Exercise-III…………………………………………………………… 317–330
● Miscellaneous Exercise-IV…………………………………………………………… 331–341
● Miscellaneous Exercise-V…………………………………………………………… 342–348
● Miscellaneous Exercise-VI…………………………………………………………… 349–356
● Miscellaneous Exercise-VII………………………………………..………………… 357–363
● Miscellaneous Exercise-VIII………………………………………………………… 364–376
More PDF download : http://www.gksolve.com

Quantitative
Aptitude Test
More PDF download : http://www.gksolve.com

More PDF download : http://www.gksolve.com

More PDF download : http://www.gksolve.com


More PDF download : http://www.gksolve.com

Average
Formula Q. 4. The average of 11 results ∴ Average price of a pig
is 50. If the average of first six
Average
results is 49 and that of last six is = Rs. ( )
1500
5
= ( Sum of observations
Number of observations ) 52, find the sixth result.
Solution :
= Rs. 300
and sum of all items Sum of 11 results Exercise
= Average × Number of items = (11 × 50) = 550 1. The average of 8 number is 21.
1. If a certain distance is covered Sum of first 6 results If each of the numbers is mul-
by two unequal speed u km/hour and tiplied by 8, the average of the
= (6 × 49) = 294
v km/hour, then, new set of numbers is :
Sum of last 6 results
2 uv (A) 8 (B) 21
Average speed = = (6 × 52) = 312
u+v (C) 29 (D) 168
∴ 6th result
Theorem —If a certain distance is
= (294 + 312 – 550) 2. The average of 50 numbers is
covered at u km/hr and the same
distance is covered at v km/hr, then the = 56. 38. If two numbers, namely 45
average speed during the whole jour- and 55 are discarded, the average
Q. 5. The average age of a
2 uv of the remaining number is :
ney is km/hr. family of 6 members is 22 years. If
u+v the age of the youngest member be (A) 36·5 (B) 37·0
7 years, find the average age of the (C) 37·5 (D) 37·52
Solved Examples family at the birth of the youngest
3. The average score of a cricketer
member.
Q. 1. Find the average of first for 10 matches is 43·9 runs. If
five multiples of 3. Solution : Sum of ages of all
the average for the first six
members
Solution : Average score matches is 53, the average for
= (22 × 6) years
3 (1 + 2 + 3 + 4 + 5) the last four matches is :
= = 132 years
5 (A) 17·15 (B) 29·75
Sum of their age 7 years ago
3 × 15
= ( )5
=9 = (132 – 7 × 6) = 90 years
At that time there were 5 mem-
(C) 30·25 (D) 31
4. The average score of a cricketer
Q. 2. A batsman makes a score bers in 2 matches is 27 and that in 3
of 87 runs in the 17th inning and ∴ Average age at that time others is 32. Then his average
thus increased his average by 3.
Find his average after 17th inning. = ( )
90
5
years = 18 years.
score in 5 matches is :
(A) 11·8 (B) 25
Solution : Let the average after Q. 6. Sandeep covers a journey (C) 29·5 (D) 30
17th inning = x Agra to Delhi by car at an average
then, average after 16th inning speed of 40 km/hr. He returns back 5. The average height of 30 girls
by scooter with an average speed of out of a class of 40 is 160 cms
= (x – 3)
24 km/hr. Find his average speed and that of the remaining girls is
∴ 16 (x – 3) + 87 during the whole journey. 156 cms. The average height of
= 17x Solution : Average speed the whole class is :
⇒ x = (87 – 48) = 39 ⎛ 2 xy ⎞ (A) 158 cms (B) 158·5 cms
=⎜ ⎟ km/hr (C) 159 cms (D) 159·5 cms
Hence, the average after 17th ⎝ x + y⎠
inning = 39. 6. If a, b, c d, e are five consecutive
Q. 3. A cricketer makes 72, 59, = (2 40× 40+ 24× 24) km/hr. odd numbers, their average is :
18, 101 and 7 runs respectively in (A) 5 (a + 4)
= 30 km/hr.
five matches played by him. Find
Q. 7. 10 sheep and 5 pigs were abcd
his average score. (B)
bought for Rs. 6000. If the average 5
Solution : Average score price of a sheep be Rs. 450, find the (C) 5 (a + b + c + d + e)
= (72 + 59 + 18 + 101 + 7
5 ) average price of a pig.
Solution : Total price of 5 pigs
(D) None of these

257 = Rs. [6000 – (10 × 450)] 7. The average age of three boys is
= = 51·4 15 years. If their ages are in the
5 = Rs. 1500

Quantitative Aptitude Test | 3


More PDF download : http://www.gksolve.com

ratio 3 : 5 : 7, the age of the 16. The average salary per head of number of candidates who passed
youngest boy is : all the workers in a workshop is the examination is :
(A) 9 years (B) 15 years Rs. 850. If the average salary per (A) 100 (B) 110
(C) 18 years (D) 21 years head of 7 technicians is Rs. 1000 (C) 120 (D) 150
and the average salary per head
8. The average of three numbers is of the rest is Rs. 780, the total 22. The average expenditure of a
42. The first is twice the second number of workers in the work- man for the first five months is
and the second is twice the third. shop is : Rs. 120 and for the next seven
The difference between the lar- months it is Rs. 130. If he saves
gest and the smallest number is : (A) 18 (B) 20 Rs. 290 in that year, his monthly
(A) 18 (B) 36 (C) 22 (D) 24 average income is :
(C) 54 (D) 72 (A) Rs. 140 (B) Rs. 150
17. The average weight of A, B, C is
45 kg. If the average weight of A (C) Rs. 160 (D) Rs. 170
9. Out of three numbers, the first is
twice the second and is half of and B be 40 kg and that of B and 23. With an average speed of 40 km/
the third. If the average of three C be 43 kg, then the weight of B hr. a train reaches its destination
numbers is 56, the three numbers is : in time. If it goes with an average
in order are : (A) 17 kg (B) 20 kg speed of 35 km/hr, it is late by
(A) 48, 96, 24 (B) 48, 24, 96 15 minutes. The total journey is :
(C) 26 kg (D) 31 kg
(C) 96, 24, 48 (D) 96, 48, 24 (A) 30 km (B) 40 km
18. On a journey across Delhi, a taxi (C) 70 km (D) 80 km
10. The average of first nine multi- average 30 kmph for 60% of the
ples of 3 is : distance, 20 kmph for 20% of it 24. The average salary of 20 workers
(A) 12·0 (B) 12·5 and 10 kmph for the remainder. in an office is Rs. 1900 per
(C) 15·0 (D) 18·5 The average speed for the whole month. If the manager’s salary is
journey is : added, the average salary
11. The average of odd numbers upto becomes Rs. 2000 p.m. What is
100 is : (A) 20 km/hr the manager’s annual salary ?
(A) 51 (B) 50 (B) 22·5 km/hr (A) Rs. 24000
(C) 49·5 (D) 49 (C) 24·625 km/hr (B) Rs. 25200
12. The average of five results is 46 (D) 25 km/hr (C) Rs. 45600
and that of the first four is 45. (D) None of these
19. Average monthly income of a
The fifth result is : 25. In a T. V. factory, an average of
family of 4 earning members
(A) 1 (B) 10 was Rs. 735. One of the earning 60 TVs are produced per day for
(C) 12·5 (D) 50 members died and, therefore, the the first 25 days of the months.
average income came down to A few workers fell ill for the
13. The average of 30 results is 20
Rs. 650. The income of the next five days reducing the daily
and the average of other 20
deceased was : average for the month to 58 sets
results is 30. What is the average per day. The average production
of all the results ? (A) Rs. 692·80 per day for the last 5 days is :
(A) 24 (B) 25 (B) Rs. 820 (A) 45 (B) 48
(C) 48 (D) 50 (C) Rs. 990 (C) 52 (D) 58
14. The average of numbers 0·64204, (D) Rs. 1385 26. The average age of four children
0·64203, 0·64202 and 0·64201 in a family is 12 years. If the
is : 20. The average age of 24 students spacing between their ages is 4
(A) 0·64202 (B) 0·642021 in a class is 10. If the teacher’s years, the age of the youngest
(C) 0·642022 (D) 0·642025 age is included, the average child is :
increases by one. The age of the
15. The average earning of a (A) 6 years (B) 7 years
teacher is :
mechanic for the first-four days (C) 8 years (D) 9 years
(A) 25 years (B) 30 years
of a week is Rs. 18 and for the 27. The average age of four players
last four days is Rs. 22. If he (C) 35 years (D) 40 years is 18·5 years. If the age of the
earns Rs. 20 on the fourth day, coach is also included, the
21. The average of marks obtained
his average earning for the whole average age increases by 20%.
by 120 candidates was 35. If the
week is : The age of the coach is :
average of marks of passed
(A) Rs. 18·95 (B) Rs. 16 candidates was 39 and that of (A) 28 years (B) 31 years
(C) Rs. 20 (D) Rs. 25·71 failed candidates was 15, the (C) 34 years (D) 37 years

Quantitative Aptitude Test | 4


More PDF download : http://www.gksolve.com

28. The average weight of a class of the class. Thereby decreasing the 42. A ship sails out to a mark at the
40 students is 40 kg. If the weight average by 4 years. What was rate of 15 kmph and sails back at
of the teacher be included, the the original strength of class ? the rate of 10 kmph. The average
average weight increases by 500 (A) 10 (B) 11 rate of sailing is :
gms. The weight of the teacher (C) 12 (D) 15 (A) 5 km/hr
is : (B) 12 km/hr
35. The average of 6 observations is
(A) 40·5 kg (B) 60 kg (C) 12·5 km/hr
12. A new seventh observation is
(C) 60·5 kg (D) 62 kg included and the new average is (D) 25 km/hr
29. The average of the daily income decreased by 1. The seventh
43. The average consumption of
of A, B and C is Rs. 60. If B observation is :
petrol for a car for seven months
earns Rs. 20 more than C and A (A) 1 (B) 3 is 110 litres and for next five
earns double of what C earns; (C) 5 (D) 6 months it is 86 litres. The average
what is the daily income of C ?
36. Out of four numbers, the average monthly consumption is :
(A) Rs. 75
of first three is 15 and that of the (A) 96 litres (B) 98 litres
(B) Rs. 60 last three is 16. If the last num- (C) 100 litres (D) 102 litres
(C) Rs. 40 ber is 19, the first is :
(D) None of these 44. The average age of 30 students
(A) 15 (B) 16
in a class is 12 years. The
30. The average weight of 8 men is (C) 18 (D) 19 average age of a group of 5 of
increased by 2 kg when one of 37. The average of 10 numbers is the students is 10 years and that
the men whose weight is 50 kg is calculated as 15. It is discovered of another group of 5 of them is
replaced by a new man. The later on that while calculating 14 years. The average age of the
weight of the new man is— the average one number, namely remaining students is :
(A) 52 kg (B) 58 kg 36 was wrongly read as 26. The (A) 8 years (B) 10 years
(C) 66 kg (D) 68 kg correct average is—
(C) 12 years (D) 14 years
31. The average weight of 8 persons (A) 12·4 (B) 14
(C) 16 (D) 18·6 45. The average temperature of
is increased by 2·5 kg, when one
Monday, Tuesday, Wednesday
of them whose weight is 56 kg is 38. The average of 13 results is 68. and Thursday was 38° and that
replaced by a new man. The The average of first seven is 63 of Tuesday, Wednesday, Thurs-
weight of the new man is— and that of the last seven is 70, day and Friday was 40°. If the
(A) 66 kg (B) 75 kg the seventh result is : temperature on Monday was 30°,
(C) 76 kg (D) 86 kg (A) 47 (B) 65·5 the temperature of Friday was :
32. The average weight of 19 stu- (C) 73·5 (D) 94 (A) 40° (B) 39°
dents is 15 kg. By the admission 39. The average of 25 results is 18, (C) 38° (D) 30°
of a new student the average that of first twelve is 14 and of
weight is reduced to 14·8 kg. last twelve is 17. Thirteenth 46. A shopkeeper earned Rs. 504 in
The weight of the new student result is : 12 days. His average income for
is— (A) 28 (B) 72 the first four days was Rs. 40 a
(A) 10·6 kg (B) 10·8 kg (C) 78 (D) 85 day. His average income for the
remaining days is :
(C) 11 kg (D) 14·9 kg 40. A man goes to a place at the rate
(A) Rs. 40 (B) Rs. 42
of 4 kmph. He comes back on a
33. In a class, there are 20 boys
bicycle at 16 kmph. His average (C) Rs. 43 (D) Rs. 45
whose average age is decreased
speed for the entire journey is : 47. The average of 5 consecutive
by 2 months, when one boy aged
18 years is replaced by a new (A) 5 km/hr (B) 6·4 km/hr numbers is n, if the next two
boy. The age of the new boy is : (C) 8·5 km/hr (D) 10 km/hr numbers are also included, the
(A) 14 years 8 months 41. The average temperature of the average of 7 numbers will :
(B) 15 years first three days is 27°C and that (A) Increase by 2
of the next three is 29°C. If the (B) Increase by 1
(C) 16 years 4 months average of the whole week is
(D) 17 years 10 months (C) Remains the same
28·5°C, the temperature of the
last day of the week is : (D) Increase by 1·4
34. The average age of an adult class
is 40 years. 12 new students with (A) 10·5° C (B) 21° C 48. A man whose bowling average is
an average age of 32 years join (C) 31·5° C (D) 42° C 12·4 takes 5 wickets for 26 runs

Quantitative Aptitude Test | 5


More PDF download : http://www.gksolve.com

and thereby decreases his average the present age of the youngest (C) 17 years 11 months
by 0·4. The number of wickets child if they differ in age by 2 (D) 18 years 3 months
taken by him, before his last years ?
match is : 59. The average age of a family of 6
(A) 1 year members is 22 years. If the age
(A) 85 (B) 78 (B) 2 years of the youngest member be 7
(C) 72 (D) 64 years, the average age of the
(C) 3 years
49. The average weight of three men (D) 5 years family at the birth of the young-
A, B and C is 84 kg. Another est member was :
54. The mean temperature of Mon-
man D joins the group and the (A) 15 years
day to Wednesday was 37°C and
new average becomes 80 kg. If (B) 17 years
of Tuesday to Thursday was
another man E, whose weight is 34°C. If the temperature on (C) 17·5 years
3 kg more than that of D, replace 4
Thursday was th that of Mon- (D) 18 years
A, then the average weight of B, 5
C, D and E becomes 79 kg. The day, what was the temperature 60. 5 years ago, the average of Ram
weight of A is : on Thursday ? and Shyam’s ages was 20 years.
(A) 70 kg (B) 72 kg (A) 34° C Now, the average age of Ram,
(C) 75 kg (D) 80 kg Shyam and Mohan is 30 years.
(B) 35·5° C What will be Mohan’s age 10
50. There were 35 students in a (C) 36° C years hence ?
hostel. If the number of students (D) 36·5° C (A) 45 years
increased by 7, the expenses of
55. Average temperature of first 4 (B) 50 years
the mess were increased by Rs.
42 per day, while the average days of a week is 38·6°C and that (C) 49 years
expenditure per head diminished of the last 4 days is 40·3°C. If (D) 60 years
by Re. 1. The original expendi- the average temperature of the 61. The average height of 40 stu-
ture of the mess was : week be 39·1°C, the temperature dents is 163 cm. On a particular
(A) Rs. 40 (B) Rs. 420 on 4th day is : day, three students A, B, C were
(A) 36·7° C (B) 38·6° C absent and the average of the
(C) Rs. 432 (D) Rs. 442
(C) 39·8° C (D) 41·9° C remaining 37 students was found
51. A batsman has a certain average to be 162 cm. If A, B have equal
runs for 11 innings. In the 12th 56. The average age of 5 members
heights and the height of C be 2
inning he made a score of 90 of a committee is the same as it
cm less than that of A, find the
runs and thereby decreased his was 3 years ago, because an old
height of A :
average by 5. His average after member has been replaced by a
(A) 176 cm (B) 166 cm
12th innings is : new member. The difference
between the ages of old and new (C) 180 cm (D) 186 cm
(A) 127 (B) 145
member is : 62. Out of three numbers, the first is
(C) 150 (D) 217
(A) 2 years (B) 4 years twice the second and is half of
52. The average age of a husband the third. If the average of the
(C) 8 years (D) 15 years
and a wife was 23 years when three numbers is 56, the three
they were married 5 years ago. 57. The average age of A, B, C, D numbers in order are :
The average age of the husband, five years ago was 45 years. By (A) 48, 96, 24
the wife and a child, who was including X, the present average (B) 48, 24, 96
born during the interval is 20 age of all the five is 49 years. (C) 96, 24, 48
years now. How old is the child The present age of X is— (D) 96, 48, 24
now ? (A) 64 years (B) 48 years
63. The average age of A and B is
(A) less than 1 year (C) 45 years (D) 40 years 20 years. If C were to replace A,
(B) 1 year the average would be 19 and if C
58. The average age of 11 players of
(C) 3 years a cricket team is decreased by were to replace B, the average
(D) 4 years two months when two of them would be 21. What are the ages
aged 17 years and 20 years are of A, B and C ?
53. Ten years ago the average age of replaced by two reserves. The (A) 22, 18, 20
a family of 4 members was 24 average age of the reserves is : (B) 18, 22, 20
years. Two children having been
born the average age of the (A) 17 years 1 month (C) 22, 20, 18
family is same to-day. What is (B) 17 years 7 months (D) 18, 20, 22

Quantitative Aptitude Test | 6


More PDF download : http://www.gksolve.com

Answers with Hints


1. (D) Average of new members 10. (C) Average
= (21 × 8) = 168 3 + 6 + 9 + 12 + 15 + 18 + 21 + 24 + 27
=
2. (C) Total of 50 members 9
= (50 × 38) = 1900 3 (1 + 2 + 3 + 4 + 5 + 6 + 7 + 8 + 9)
=
9
Total of 48 members
= [1900 – (45 + 55)] = 1800 = ( )
45
3
= 15
1800
∴ Required average = 11. (B) Sum of odd numbers upto 100
48
= 1 + 3 + 5 + 7 + …… + 95 + 97 + 99
225
= = 37·5 = (1 + 99) + (3 + 97) + (5 + 95) + …… upto 25 terms
6
3. (C) 53 × 6 + x × 4 = 10 × 43·9 = 100 + 100 + 100 + …… upto 25 terms
∴ 4x = 439 – 318 = 2500
⇒ 4x = 121 ∴ x = 30·25
2 × 27 + 3 × 32
∴ Average = ( )
2500
50
= 50

4. (D) Average = 12. (D) Fifth result = (5 × 46 – 4 × 45)


5
150 = (230 – 180) = 50
= = 30
5 13. (A) Total of 50 results = (30 × 20 + 20 × 30)
5. (C) Average height of the whole class = 1200
30 × 160 + 10 × 156 14. (D) Average
= = 159 cms.
40
6. (D) Average
= (
0·64204 + 0·64203 + 0·64202 + 0·64201
4 )
a + (a + 2) + (a + 4) + (a + 6) + (a + 8) 2·5681
= = = 0·642025
5 4
= (a + 4) 15. (C) Total earning for the week
3x + 5x + 7x = Rs. (4 × 18 + 4 × 22 – 20) = Rs. 140
7. (A) = 15
3 ∴ Average earning
⇒ 15x = 15 × 3
⇒ x = 3
= Rs. ( )
140
7
= Rs. 20

16. (C) ˙·˙ 7 × 1000 + x × 780 = (x + 7) × 850


8. (C) Let third number = x
⇒ (850 – 780) x = (7000 – 5950)
Then, second number = 2x
⇒ 70x = 1050
and first number = 4x
∴ x = 15
x + 2x + 4x
∴ = 42 Hence, the total number of workers
3
⇒ 7x = 42 × 3 = (7 + 15) = 22
⇒ x = 18 17. (D) ˙·˙ Weight of (A + B) = (2 × 40) kg = 80 kg
So, (largest) – (smallest) = (4x – x) ⇒ Weight of (B + C) = (2 × 43) kg = 86 kg
⇒ Weight of (A + 2B + C) = (80 + 86) kg = 166 kg
= 3x = 54
⇒ Weight of (A + B + C) = (3 × 45) kg = 135 kg
9. (B) Let second number = x
∴ Weight of B = (166 – 135) kg = 31 kg
Then, first one = 2x and third number = 4x
18. (A) Let total journey = x km
x + 2x + 4x
˙·˙ = 56 Total time taken
3
⇒ 7x = 168 =( 60
100
x× +
1
30 100
20
x× +
1 20
20 100

1
10 ) hrs.
⇒ x = 24
=( ) ( )
x x x x
+ + hrs. = hrs.
So the number are 48, 24, 96. 50 100 50 20

Quantitative Aptitude Test | 7


More PDF download : http://www.gksolve.com

∴ Average speed 29. (C) Let C’s earning = Rs. x.

( 20
)
= x × x km/hr Then,
and
B’s earning = Rs. (x + 20)
A’s earning = Rs. 2x
= 20 km/hr. ˙·˙ 2x + x + 20 + x = 3 × 60
19. (C) Income of the deceased
⇒ 4x = 160
= Rs. (735 × 4 – 650 × 3) = Rs. 990
∴ x = 40
20. (C) Age of the teacher
Hence, the daily earning of C is Rs. 40.
= (25 × 11 – 24 × 10) years = 35 years.
30. (C) Weight increased = (8 × 2) kg = 16 kg
21. (A) Let the number of candidates who passed
Weight of new man = (50 + 16) kg = 66 kg.
= x
31. (C) Total increase = (8 × 2·5) kg = 20 kg
then, ˙·˙ 39 × x + 15 × (120 – x) = 120 × 35
Weight of new man = (56 + 20) kg = 76 kg
⇒ 24x = 4200 – 1800
32. (C) Weight of new student
∴ x = ( ) = 100
2400
24
33.
= (20 × 14·8 – 19 × 15) kg = 11 kg.
(A) Total decrease
22. (B) Total income
= (20 × 2) months
= Rs. (120 × 5 + 130 × 7 + 290) = Rs. 1800
= 3 year 4 months
Average monthly income
∴ Age of the new boy
= Rs. ( )
1800
12
= Rs. 150 = (18 years) – (3 years 4 months)
= 14 years 8 months.
x x 15
23. (C) ˙·˙ – = 34. (C) Let original strength = x
35 40 60
5x 1 Then, 40x + 12 × 32 = (x + 12) × 36
⇒ =
35 × 40 4 ⇒ 40x + 384 = 36x + 432
35 × 40 ⇒ 4x = 48 ∴ x = 12
⇒ x = = 70
4×5
35. (C) Seventh observation = (7 × 11 – 6 × 12)
∴ Total journey = 70 km.
= 5
24. (D) Manager’s salary per month
36. (B) Sum of four numbers = (15 × 3 + 19) = 64
= Rs. (21 × 2000 – 20 × 1900)
Sum of last three numbers = (16 × 3) = 48
= Rs. 4000
∴ First number = (64 – 48) = 16.
∴ Manager’s annual salary
37. (C) Sum of numbers = (10 × 15 – 26 + 36)
= Rs. (4000 × 12) = Rs. 48000.
= 160
25. (B) Production during these 5 days
160
= (30 × 58 – 25 × 60) = 240 ∴ Correct average = = 16.
10
240
∴ Average for last 5 days = = 48 38. (A) Seventh Result = (7 × 63 + 7 × 70 – 13 × 68)
5
= 47.
26. (A) ˙·˙ x + (x + 4) + (x + 8) + (x + 12)
39. (C) Thirteenth Result
= 4 × 12
= (25 × 18 – 12 × 14 – 12 × 17)
⇒ 4x = (48 – 24) = 24
= 78.
∴ x = 6 years.
27. (D) New average ⎛ 2xy ⎞
40. (B) Average speed = ⎜ x + y⎟ km/hr
⎝ ⎠
= (120% of 18·5) = (120 × 18·5) = 22·2

Age of coach
100
= (2 4× +4 16× 16) km/hr
= (5 × 22·2 – 4 × 18·5) = 37 years = 6·4 km/hr.
28. (C) Weight of the teacher 41. (C) 3 × 27 + 3 × 29 + x = 7 × 28·5
= (41 × 40·5 – 40 × 40) kg = 60·5 kg ∴ x = 31·5.

Quantitative Aptitude Test | 8


More PDF download : http://www.gksolve.com

⎛ 2xy ⎞ ∴ B + C + D = (316 – 71) = 245


42. (B) Average = ⎜ x + y⎟ km/hr
⎝ ⎠ So, A = (320 – 245) = 75 kg.
50. (B) Let the original expenditure be Rs. x per day,
= (2 15× 15+ 10× 10) km/hr Then,
x (x + 42)
– = 1
= 12 km/hr. 35 42
110 × 7 + 86 × 5 ⇒ 42x – 35 (x + 42) = 35 × 42
43. (C) Average = ( 12 ) ∴ 7x = 35 × 42 + 35 × 42

= (1200
12 )
= 100 litres. ⇒ x =
2 × 35 × 42
7
= 420

44. (C) Let average age of the remaining students be x. Hence, the original expenditure is Rs. 420
Then, 5 × 10 + 5 × 14 + 20 × x = 30 × 12 51. (B) ˙·˙ 11x + 90 = (x – 5) × 12
⇒ 20x = 360 – 120 ⇒ x = 150
⇒ 20x = 240 ∴ Average after 12th inning
⇒ x = 12. = (150 – 5) = 145.
45. (C) ˙·˙ M + T + W + Th = (4 × 38°) = 152° 52. (D) Age of child
⇒ T + W + Th = (152° – 30°) = 122° = [(20 × 3) – (23 × 2 + 5 × 2)] years
⇒ T + W + Th + F = (4 × 40°) = 160° = 4 years.
⇒ F = (160° – 122°) = 38° 53. (C) ˙·˙ x + x + 2 = (24 × 6) – (24 × 4 + 4 × 10)
46. (C) Let the average for remaining 8 days be Rs. x a = 144 – 136
day. ⇒ 2x + 2 = 8
Then, 4 × 40 + 8 × x = 504
∴ x = 3.
⇒ 8x = 344
54. (C) (M + T + W) = (3 × 37) = 111° …(i)
⇒ x = 43
(T + W + Th) = (3 × 34) = 102° …(ii)
∴ Required average = Rs. 43
Let M = x
47. (B) x + (x + 1) + (x + 2) + (x + 3) + (x + 4)
4
= 5n Then, Th = x
5
⇒ 5x + 10 = 5n
Subtracting (ii) from (i) we get,
⇒ x = (n – 2)
M – Th = 9°
Average of 7 consecutive integers
4
(5x + 10) + (x + 5) + (x + 6) ⇒ x– x = 9°
= 5
7
⇒ x = 45°
7x + 21
= =x+3 ∴ Temperature on Thursday
7
∴ New average = (n – 2 + 3) = n + 1 = (45 × 45°) = 36° C
So, the new average increases by 1.
55. (D) ˙·˙ 4 × 38·6 + 4 × 40·3 – x = 7 × 39·1
48. (A) Let the number of wickets taken before the last
match = x ⇒ x = 41·9

12·4x + 26 ∴ Temperature on 4th day = 41·9° C


Then, = 12 56. (D) Increase during 3 years = (3 × 5) years
x+5
⇒ x = 85 = 15 years
So, the difference between ages of old and new
49. (C) ˙·˙ A + B + C = 3 × 84 = 252;
member is 15 years.
⇒ A + B + C + D = (4 × 80) = 320 57. (C) Present age of X
∴ D = (320 – 252) = 68 = [(49 × 5) – (4 × 45 + 4 × 5)] years = 45 years.
and so E = (68 + 3) = 71 58. (B) Decrease = (11 × 2) months
Now, B + C + D + E = (4 × 79) = 316 = 1 year 10 months

Quantitative Aptitude Test | 9


More PDF download : http://www.gksolve.com

Total age of reserves Then, x + x + (x – 2) = (163 × 40 – 162 × 37)


= (17 + 20) years – (1 year 10 month) ⇒ 3x – 2 = 526
= 35 years 2 months ∴ x = 176 cm
Average age of the reserves 62. (B) Let the numbers be 2x, x and 4x
1
= (35 years 2 months) 2x + x + 4x
2 ˙·˙ Average =
3
= 17 years 7 months.
7x
59. (D) Total present age of the family ⇒ = 56
3
= (6 × 22) = 132 years
3 × 56
Total age of the family 7 years ago ∴ x = = 24
7
= (132 – 7 × 6) = 90 years Hence, the numbers in order are 48, 24 and 96.
At that time, the number of members was 5 63. (A) Let a, b, c are the ages of A, B and C respectively
∴ Average age at that time
a + b = 2 × 20 = 40 …(1)
= ( )90
5
years = 18 years. + b + c = 2 × 19 = 38 …(2)
60. (B) Total age of Ram and Shyam 5 years ago + c + a = 2 × 21 = 42 …(3)
= (2 × 20) = 40 years [Adding all the 3 equalities]
∴ Total age of Ram and Shyam at present + a + b + c = 60 …(4)
= (40 + 5 + 5) = 50 years – (b + c) = – 38
Total age of Ram, Shyam and Mohan now ∴ a = 22
= (3 × 30) = 90 years
– (a + c) = – 42
Mohan’s age now
∴ b = 18
= (90 – 50) years = 40 years
Mohan’s age 10 years hence and c = 20
= (40 + 10) years = 50 years ∴ Age of A = 22 years
61. (A) Let the heights of A, B and C be x cm, x cm and Age of B = 18 years
(x – 2) cm respectively Age of C = 20 years

Quantitative Aptitude Test | 10


More PDF download : http://www.gksolve.com

Time & Distance


Formulae 3 Solution :
Q. 2. Walking of his usual
4 Average speed
Distance
(i) Speed = speed a peon is 10 min. too late to 2 × 70 × 55
Time
Distance
his office. Find his usual time to
cover the distance.
=( 70 + 55 ) km/hr.
(ii) Time =
Speed Solution : = 61·6 km/hr.
(iii) Distance = (Speed × Time) Q. 5. (i) Convert 45 km/hr. into
Let the usual time be x min.
metres/sec.
(iv) If a certain distance is 3 (ii) Convert 6 metres/sec into
covered at x km/hr and the same Time taken at of the usual
4 km/hr.
distance is covered at y km/hr, then
the average speed during whole
journey is—
speed = (43 x) min. Solution :

⎛ 2xy ⎞ ˙·˙
4
x – x = 10 (
(i) 45 km/hr = 45 ×
5
18 ) m/sec
⎜ x + y⎟ km/hr 3
⎝ ⎠ ⇒ 4x – 3x = 30
= 12·5 m/sec

(v) If the speed of a body is


changed in the ratio a : b then the
⇒ x = 30 (
(ii) 6m/sec. = 6 ×
5)
18
km/hr.

ratio of the time taken changes in the Hence, the usual time taken = 21·6 km/hr.
ratio b : a. = 30 min. Exercise
(vi) x km/hr = ( x×
5
18) m/sec Q. 3. A man covers a certain
distance between his house and
1. Laxman has to cover a distance
of 6 km in 45 minutes. If he
office on scooter. Having an covers one half of the distance in
(vii) x metres/sec.
average speed of 30 km/hr. he is 2
= (x × 185) km/hr. late by 10 min. However, with a
speed of 40 km/hr, reaches his
3
rd time. What should be his
speed to cover the remaining
office 5 min. earlier. Find the distance in the remaining time ?
distance between his house and
Examples office.
(A) 12 km/hr. (B) 16 km/hr.
(C) 3 km/hr. (D) 8 km/hr.
Q. 1. A man cycles from A to Solution :
2. A train covers a distance in 50
B a distance of 21 km in 1 hour 40 Let the required distance be x km minutes. If it runs at a speed of
min. The road from A is level for
Time taken to cover x km at 30 48 km per hour on an average.
13 km and then it is uphill to B. The speed at which the train
The man’s average speed on levels
is 15 km/hr. Find his average uphill
km/hour =
x
30 ( )
hrs. must run to reduce the time of
journey to 40 minutes, will be :
speed. Time taken to cover x km at 40 (A) 50 km/hr. (B) 55 km/hr.
Solution :
Let the average uphill speed be x
km/hour =
x
40 ( )
hrs. (C) 60 km/hr. (D) 70 km/hr.
3. Excluding stoppages, the speed
km/hr. Difference between the times of a bus is 54 km/hr. and
13 8 5 1 including stoppages it is 45 km/
Then, + = taken = 15 min. = hr. hr. For how many minutes does
15 x 3 4
the bus stop per hour ?
8 5 13 x x 1
⇒ ˙·˙ – = (A) 9 (B) 10
x = 3 – 15 30 40 4
(C) 12 (D) 20
8 12 ⇒ 4x – 3x = 30
⇒ 4. Two man start together to walk
x = 15 ⇒ x = 30 to a certain destination. One at
8 4 Hence, the required distance is 3·75 km an hour and another at 3
⇒ x= 5 30 km. km an hour. The former arrives
half an hour before the later. The
5×8 Q. 4. Harish covers a certain
⇒ x= distance is—
4 distance by car driving at 70 km/ (A) 9·5 km. (B) 8 km.
= 10 hr. and he returns back at the (C) 7·5 km. (D) 6 km.
starting point riding on a scooter at
∴ Average uphill speed 5. A car covers four successive
55 km/hr. Find his average speed
= 10 km/hr. for the whole journey. three km streches at speeds of 10

Quantitative Aptitude Test | 11


More PDF download : http://www.gksolve.com

km/hr., 20 km/hr, 30 km/hr. and south at 20 km/hr. What time (C) 11·52 km/hr.
60 km/hr. respectively. Its ave- will they take to be 95 km. apart? (D) 12·32 km/hr.
rage speed over this distance is— (A) 4 hrs. 30 min.
20. A speed of 22·5 m/sec. is the
(A) 10 km/hr. (B) 20 km/hr. (B) 4 hrs. 45 min. same as—
(C) 30 km/hr. (D) 25 km/hr. (C) 5 hrs. 16 min. (A) 40·5 km/hr.
6. A and B are two stations. A train (D) 2 hrs. 30 min. (B) 81 km/hr.
goes from A to B at 64 km/hr. 13. A boy goes to school with a (C) 36·8 km/hr.
and returns to A at a slower speed of 3 km/hr. and returns to
speed. If its average speed for (D) 72 km/hr.
the village with a speed of 2
the whole journey is 56 km/hr. at km/hr. If he takes 5 hours in all 21. A speed of 55 m/sec. is the same
what speed did it return ? the distance between the village as—
(A) 48 km/hr. (B) 49·77 km/hr. and the school is— (A) 198 km/hr.
(C) 52 km/hr. (D) 47·46 km/hr. (A) 6 km. (B) 7 km. (B) 11 km/hr.
7. A car completes a certain jour- (C) 8 km. (D) 9 km. 5
(C) 15 km/hr.
ney in 8 hours. It covers half the 18
14. A distance is covered in 2 hours
distance at 40 km/hr. and the rest 45 min. at 4 km/hr. How much (D) 275 km/hr.
at 60 km/hr. The length of the time will be taken to cover it at 22. A speed of 30·6 km/hr. is the
journey is— 16·5 km/hr. ? same as—
(A) 350 km. (B) 420 km. (A) 40 min. (A) 5·1 m/sec.
(C) 384 km. (D) 400 km. (B) 41 min. 15 sec. (B) 8·5 m/sec.
8. Suresh travelled 1200 km by air (C) 45 min. (C) 110·16 m/sec.
which formed (2/5) of his trip. (D) 90 min. (D) 1·7 m/sec.
One-third of the whole trip he
travelled by car and the rest of 15. If a man takes 4 hours to cover a 23. A speed of 36 km/hr. is the same
the journey he performed by distance of 15 km. How much as—
train. The distance travelled by time will be needed to cover 63 (A) 10 m/sec.
train was— km. at the same speed ?
(B) 7·2 m/sec.
(A) 1600 km. (B) 800 km. (A) 12 hrs. 36 min.
(C) 2 m/sec.
(C) 1800 km. (D) 400 km. (B) 16 hrs. 48 min.
(D) 129·6 m/sec.
(C) 16 hrs. 4 min.
9. Rahim covers a certain distance 24. The distance between two
in 14 hrs 40 min. He covers one (D) 15 hrs. 32 min.
stations A and B is 220 km. A
half of the distance by train at 60 16. A train is moving with a speed train leaves A towards B at an
km/hr. and the rest half by road of 92·4 km/hr. How many metres average speed of 80 km/hr. After
at 50 km/hr. The distance will it cover in 10 min. ? half an hour another train leaves
travelled by him is— (A) 1540 (B) 15400 B towards A at an average speed
(A) 960 km. (B) 720 km. (C) 154 (D) 15·4 of 100 km/hr. The distance of
(C) 1000 km. (D) 800 km. the point where the two trains
17. If a man covers 10·2 km. in 3 meet, from A is—
2 hours. The distance covered by
10. A man performs of the total (A) 120 km. (B) 130 km.
15 him in 5 hours is—
9 (C) 140 km. (D) 150 km.
journey by rail, by tonga and (A) 18 km. (B) 15 km.
20 25. A bullock cart has to cover a
(C) 16 km. (D) 17 km.
the remaining 10 km. on foot. distance of 80 km. in 10 hours. If
This total journey is— 18. A man crosses a street 600 m it covers half of the journey in
(A) 15·6 km. (B) 12·8 km. long in 5 minutes. His speed in (3/5) the time, what should be its
(C) 16·4 km. (D) 24 km. km. per hour is— speed to cover the remaining
(A) 7·2 (B) 3·6 distance in time left ?
11. Shard covers two-third of a
(C) 10 (D) 8·4 (A) 8 km/hr.
certain distance at 4 km/hr. and
the remaining at 5 km/hr. If he (B) 20 km/hr.
19. A and B are two towns. Mr.
takes 42 minutes in all, the dis- Faruqui covers the distance from (C) 6·4 km/hr.
tance is— A to B on cycle at 16 km/hr. (D) 10 km/hr.
(A) 2·5 km. (B) 4·6 km. However, he covers the distance
26. Suresh started cycling along the
(C) 4 km. (D) 3 km. from B to A on foot at 9 km/hr.
boundries of a square field from
His average speed during the
12. Two cyclists A and B start from corner point A. After half an
whole journey is—
the same place at the same time. hour, he reached the corner point
One going towards north at 18 (A) 12·5 km/hr. C, diagonally opposite to A. If
km/hr. and other going towards (B) 10·25 km/hr. his speed was 8 km/hr, what is

Quantitative Aptitude Test | 12


More PDF download : http://www.gksolve.com

the area of the field in square 33. The ratio between the rates of The theft is discovered at 2 p.m.
km. ? walking of A and B is 2 : 3. If and the owner sets off in another
(A) 64 the time taken by B to cover a car at 50 km. an hour. He will
(B) 8 certain distance is 36 minutes, overtake the thief at—
the time taken by A to cover that (A) 3·30 p.m. (B) 4 p.m.
(C) 4
much distance is—
(D) Cannot be determined (C) 4·30 p.m. (D) 6 p.m.
(A) 24 min. (B) 54 min.
27. A man goes uphill with an 40. A train leaves Meerut at 6 a.m.
(C) 48 min. (D) 21·6 min. and reaches Delhi at 10 a.m.
average speed of 35 km/hr. and
covers down with an average 34. A man, on tour travels first 160 Another train leaves Delhi at 8
speed of 45 km/hr. The distance km. at 64 km/hr. and the next a.m. and reaches Meerut at 11·30
travelled in both the cases being 160 km at 80 km/hr. The average a.m. At what time do the trains
the same, the average speed for speed for the first 320 km. of the cross one another ?
the entire journey is— tour is— (A) 9·26 a.m. (B) 9 a.m.
3 (A) 35·55 km/hr. (C) 8·36 a.m. (D) 8·56 a.m.
(A) 38 km/hr.
8 (B) 71·11 km/hr. 41. Two trains start at the same time
3 (C) 36 km/hr. from Aligarh and Delhi and
(B) 39 km/hr.
8 (D) 72 km/hr. proceed towards each other at 16
(C) 40 km/hr. km/hr. and 21 km/hr. respecti-
35. A man travels 35 km. partly at 4 vely. When they meet, it is found
(D) None of these km/hr. and at 5 km/hr. If he that one train has travelled 60
28. A man walking at 3 km/hr. covers former distance at 5 km/ km. more than the other. The
crosses a square field diagonally hr. and later distance at 4 km/hr. distance between the two stations
in 2 min. The area of the field he could cover 2 km. more in the is—
is— same time. The time taken to (A) 445 km. (B) 444 km.
cover the whole distance at (C) 440 km. (D) 450 km.
(A) 25 acres (B) 30 acres
original rate is—
(C) 50 acres (D) 60 acres 42. X and Y are two stations 500
(A) 9 hours (B) 7 hours km. apart. A train starts from X
29. The ratio between the rates of 1 and moves towards Y at 20
travelling of A and B is 2 : 3 and (C) 4 hours (D) 8 hours
2 km/hr. Another train starts from
therefore A takes 10 min. more
3 Y at the same time and moves
than the time taken by B to reach 36. By walking at of his usual
4 towards X at 30 km/hr. How far
a destination. If A had walked at
speed, a man reaches his office from X will they cross each
double the speed, he would have
20 minutes later than usual. This other ?
covered the distance in—
usual time is— (A) 200 km. (B) 30 km.
(A) 30 min. (B) 25 min.
(A) 30 minutes (C) 120 km. (D) 40 km.
(C) 15 min. (D) 20 min.
(B) 60 minutes 43. A car travels a distance of 840
30. A certain distance is covered at a
(C) 75 minutes km. at a uniform speed. If the
certain speed. If half of this
(D) 1 hour 30 min. speed of the car is 10 km/hr.
distance is covered in double the more. It takes two hours less to
time, the ratio of the two speed 37. If a train runs at 40 km/hr. It cover the same distance. The
is— reaches its destination late by 11 original speed of the car was :
(A) 4 : 1 (B) 1 : 4 min. but if it runs at 50 km/hr, it (A) 45 km/hr. (B) 50 km/hr.
(C) 2 : 1 (D) 1 : 2 is late by 5 min. only. The correct
(C) 60 km/hr. (D) 75 km/hr.
time for the train to cover its
31. If a boy takes as much time in journey is— 44. A train is running with the speed
running 10 m as a car takes in
(A) 13 min. (B) 15 min. of 45 km/hour. What is its speed
covering 25 m, the distance in metre per second ?
covered by the boy during the (C) 21 min. (D) 19 min.
time the car covers 1 km, is— (A) 12·5
38. Ram travels a certain distance at
(A) 400 m. (B) 40 m. (B) 13·5
3 km/hr. and reaches 15 min.
(C) 250 m. (D) 650 m. late. If he travels at 4 km/hr. he (C) 20·5
reaches 15 min. earlier. The (D) None of these
32. A is twice as fast as B and B is distance he has to travel is—
thrice as fast as C is. The journey 45. If a motor car covers a distance
covered by C in 42 minutes, will (A) 4·5 km. (B) 6 km. of 250 m in 25 seconds, what is
be covered by A in— (C) 7·2 km. (D) 12 km. its speed in kilometre per hour ?
(A) 14 min. (B) 28 min. 39. A thief steals a car at 1·30 p.m. (A) 36·5 (B) 36·8
(C) 63 min. (D) 7 min. and drives it at 45 km. an hour. (C) 37 (D) 36

Quantitative Aptitude Test | 13


More PDF download : http://www.gksolve.com

46. A train leaves Delhi at 5 a.m. (B) 25·50 km. from Q the ratio of the speeds of the cars
and reaches Kanpur at 10 a.m. (C) 25·30 km. from P A and B is 5 : 6 and the speed of
Another train leaves Kanpur at 7 the car B is 90 km. per hour,
(D) Can’t be determined
a.m. and reaches Delhi at 2 p.m. after how long will the two meet
At what time do the two trains 48. Two boys begin together to write each other ?
meet ? out a booklet containing 817 2
lines. The first boy starts with (A) 26 min. (B) 24 min.
(A) 8·45 a.m. 3
first line, writing at the rate of
(B) 3·45 p.m. 200 lines an hour and the second (C) 32 min. (D) 36 min.
(C) 6·45 a.m. boy starts with the last line. He 50. 150 metre long train takes 10
(D) Data inadequate writes line 817 and so on back- seconds to pass a man who is
wards proceeding at the rate of
47. A starts from P to walk to Q a going in the same direction at the
150 lines an hour. At what line
distance of 51·75 kilometeres at speed of 2 km/hr. What is the
will they meet ?
the rate of 3·75 km. an hour. An speed of the train ?
(A) 466th (B) 465th
hour later B starts from Q for P (A) 52 km/hr
and walks at the rate of 4·25 km. (C) 467th (D) 468th
(B) 56 km/hr
an hour. When and where will A 49. Two cars A and B are running
meet B ? (C) 84 km/hr
towards each other from two
(A) 26·25 km. from Q different places 88 km. apart. If (D) Data inadequate

Answers with Hints


1. (A) ˙·˙ Time left = (13 × 4560) hr. ∴ Average speed = {3/512 } km/hr.
12 × 5
= (
3 )
1
= hr. km/hr.
4
Distance left = 3 km. = 20 km/hr.
6. (B) Let the required speed be x km/hr.
∴ Speed required = 3 ÷ ( ) 1
4
km/hr.
Then,
2 × 64 × x
= 56
= 12 km/hr. (64 + x)
⇒ 128x = 64 × 56 + 56x
2. (C) ˙·˙ (
Distance = 48 ×
50
60) km.
∴ x =
64 × 56
= 49·77 km/hr.
= 40 km. 72
7. (C) Let the length of total journey be x km.
Required speed = ( ) 40
40/60
km/hr.
˙·˙
x 1 x 1
· + · = 8
2 40 2 60
40 × 60
= ( ) 40
km/hr.

x
+
80 120
x
= 8
= 60 km/hr. ⇒ 3x + 2x = 1920
3. (B) Due to stoppages, it covers 9 km. less per hour ∴ x = 384 km.
Time taken to cover 9 km. 8. (B) Let the total distance be x km.
= ( 9
54 )
× 60 min = 10 min. Then,
2
5
x = 1200
4. (C) Let the distance be x km. Then 1200 × 5
⇒ x = = 3000 km.
x x 1 2
˙·˙ – = Distance travelled by car
3 3·75 2


3·75x – 3x
3 × 3·75
=
1
2
= ( 1
3 )
× 3000 = 1000 km.
Distance travelled by train
⇒ 1·5x = 3 × 3·75 = [3000 – (1200 + 1000)] km.
3 × 3·75 = 800 km.
∴ x = = 7·5 km.
1·5 9. (D) Let the total distance be x km.
5. (B) Total time taken = ( 3
+
3
+
10 20 30 60
3
+
3
)
hrs. Then,
x 1 x 1
× + ×
2 60 2 50
=
44
3
3 x x 44
= hrs. ⇒ + =
5 120 100 3

Quantitative Aptitude Test | 14


More PDF download : http://www.gksolve.com



5x + 6x = 8800
x = 800
19. (C) Average speed = (2 16× 16+ 9× 9) km/hr.
∴ Required distance = 800 km. = 11·52 km/hr.
10. (D) Let the total journey be x km. Then,
2
x+
9
x + 10 = x
20. (B) (
22·5 m/sec. = 22·5 ×
18
5 ) km/hr.

15 20 = 81 km/hr.
⇒ 8x + 27x + 600 = 60x
⇒ x = 24
21. (A) (
55 m/sec = 55 ×
18
5) km/hr.

∴ Total journey = 24 km. = 198 km/hr.


11. (D) Let total distance be x km.

Then,
2 1 1 1
x· + x· =
42
22. (B) (
30·6 km/hr. = 30·6 ×
5
18 ) m/sec.
3 4 3 5 60 = 8·5 m/sec.
x x 7
⇒ +
6 15
=
10 23. (A) (
36 km/hr. = 36 ×
5
18) m/sec.
⇒ 5x + 2x = 21
= 10 m/sec.
∴ x = 3
24. (A) Let the required distance be x km.
∴ Required distance = 3 km.
x 220 – x 1
12. (D) They are 38 km. apart in 1 hr. Then, – =
80 100 2
∴ They will be 95 km. apart in ( 1
38 )
× 95 hrs. ⇒ 5x – 4 (220 – x) = 200
= 2 hrs. 30 min. ⇒ 9x = 1080
13. (A) Let the required distance be x km. ⇒ x = 120 km.

Then,
x x
+ = 5 ⇒ 2x + 3x = 30 ⇒ x = 6 km
3 2
25. (D) Distance left = ( )1
2
× 80 km. = 40 km.

14. (A) (
Distance = 4 × 2
3
4 ) km. = 4 × ( 11
4 ) km. Time left = [( ) ]
1–
3
5
× 10 hrs .
= 11 km = 4 hours.
11 km Required speed = (40 ÷ 4) km/hr.
∴ Req. time =
(33/2) km/hr
= 10 km/hr.
2 × 11
= hr. 26. (C) Length of two sides of square
33
2
= × 60 minutes
3
( )
= 8×
1
2
km. = 4 km.

= 40 minutes. ∴ Area of the field = (2 × 2) sq. km.


= 4 sq. km.
15. (B) Required time = (4
15 )
× 63 hrs.
27. (B) Average speed = ( 2 × 35 × 45
) km/hr.
= 16 hrs. 48 min. 35 + 45
3
16. (B) (
92·4 kms/hr. = 92·4 ×
5
18 ) m/sec. = 39 km/hr.
8

∴ Req. distance = 92·4 ×


5
18
× 600 metres 28. (C) ( )
Speed = 3 ×
5
18
m/sec.

= 15400 m. = () 5
6
m/sec.
17. (D) Speed = ( )
10·2
3
km/hr. ∴ Distance covered in 2 min.
= (2 × 60) sec.
= 3·4 km/hr.
Distance covered in 5 hours = ( 5
6 )
× 2 × 60 m = 100 m.
= (3·4 × 5) km. = 17 km. ∴ Length of diagonal = 100 m
18. (A) Speed = ( 600
5 × 60) m/sec. 1
Area of the field = × (diagonal)2
2
= ( 600 18
5 × 60 5
× ) km/hr. = ( 1
2 )
× 100 × 100 m2
= 7·2 km/hr. = 5000 m2 = 50 acres.

Quantitative Aptitude Test | 15


More PDF download : http://www.gksolve.com

1 1 Solving these equations, we get


29. (C) Ratio of times taken by A and B = :
2 3 x = 5 and y = 3
Suppose B takes x min. Then A takes (x + 10) min. ∴ Total time taken = (5 + 3) hrs. = 8 hrs.
1 1 3
∴ (x + 10) : x = : 36. (B) At a speed of of the usual speed, the time taken
2 3 4
x + 10 3 4
⇒ = is of the usual time
x 2 3


2x + 20 = 3x
x = 20
∴ ( 4
3 )
of usual time – (Usual time) = 20 min.

∴ Time taken by A = 20 + 10 4 1
⇒ x – x = 20 ⇒ x = 20
3 3
= 30 minutes
⇒ x = 60 min.
If A had walked at double speed
30 37. (D) Let the required time = x min. Then
Req. time = distance covered in (x + 11) min. at 40 km/hr.
2
= 15 minutes. = distance covered in (x + 5) min. at 50 km/hr.
30. (A) Let x km. be covered in y hrs. x + 11 x+5
˙·˙ 40 × = 50 ×
Then, () x
1st speed = y km/hr.

60 60
x = 19 min.

2nd speed =( ) x
2
÷ 2y km/hr.
38. (B) Let the distance be x km.

Then,
x x
– =
30

() =
x
4y
km/hr.

3 4
4x – 3x
=
60
1
x x 1 12 2
∴ Ratio of speed = y : = 1 : = 4 : 1
4y 4 ⇒ x = 6 km.
31. (A) 25 : 10 : : 1000 : x 39. (B) Distance covered by thief in (1/2) hour = 20 km.
10 × 1000 Now, 20 km. is compensated by the owner at a
⇒ x = = 400 m
25 relative speed of 10 km/hr. in 2 hours
32. (D) Let C’s speed = x km/hr. So, he overtakes the thief at 4 p.m.
Then, B’s speed = 3x km/hr. 40. (D) Let the distance between Meerut and Delhi be y
km.
and A’s speed = 6x km/hr.
Average speed of the train leaving Meerut
∴ Ratio of speeds of A, B, C
= 6x : 3x : x = 6 : 3 : 1 ()
=
y
4
km/hr.
1 1 Average speed of the train leaving Delhi
Ratio of times taken = : : 1 or 1 : 2 : 6
6 3
˙·˙ 6 : 1 : : 42 : t ()
=
2y
7
km/hr.
⇒ 6t = 42 Suppose they meet x hrs. after 6 a.m.
⇒ t = 7 min. xy 2y (x – 2)
Then, ˙·˙ + =y
1 1 4 7
33. (B) Ratio of times taken = :
2 3 x 2x – 4
⇒ + = 1
1 1 4 7
˙·˙ : = x : 36
2 3 ⇒ 15x = 44
1 1 44
⇒ × x = × 36 ∴ x = = 2 hrs. 56 min.
3 2 15
∴ x = 54 min. So, the trains meet at 8·56 a.m.
2 × 64 × 80 41. (B) Suppose they meet after x hours.
34. (B) Average speed = ( )
64 + 80
km/hr. Then, 21x – 16x = 60
2 × 64 × 80 ⇒ x = 12
= km/hr.
144 ∴ Required distance = (16 × 12 + 21 × 12) km.
= 71·11 km/hr. = 444 km.
35. (D) Suppose the man covers first distance in x hrs. 42. (A) Suppose they meet x km. from X
and second distance in y hrs. x (500 – x)
Then, 4x + 5y = 35 and 5x + 4y = 37 Then, =
20 30

Quantitative Aptitude Test | 16


More PDF download : http://www.gksolve.com

⇒ 30 x = 10000 – 20x ∴ The two trains will meet at (7 a.m. + 1 hr. 45 min.)
⇒ x = 200 km. = 8·45 a.m.
43. (C) Let the original speed be x km/hr. 47. (B) A has already gone 3·75 km. when B starts of the
840 840 remaining 48 km. A walks 3·75 km. and B walks
Then, x – (x + 10) = 2 4·25 km. in one hour in opposite direction, i.e., they
⇒ 840 (x + 10) – 840x = 2x (x + 10) together pass over (3·75 + 4·25) = 8 km. in one hour.
⇒ x2 + 10x – 4200 = 0 48
Therefore, 48 km. are passed over in = 6 hours.
8
⇒ (x + 70) (x – 60) = 0
Therefore, A meets B in 6 hours after B started and,
∴ x = 60 km/hr. therefore, they meet at a distance of (4·25 × 6) = 25·5
44. (A) Speed of a train in kilometre per hour km. from Q.
= 45 48. (C) Let the two meet at the xth line
Speed of the train in metre per second
From the question,
5
= 45 × x (817 – x)
18 =
= 12·5 200 150
45. (D) Speed of the motor car ⇒ 3x = 4 (817 – x)
250 4 × 817
= m/sec. ⇒ x =
25 7
= 10 m/sec. ∴ x = 466·85
18
= 10 × km/hour i.e., they will meet at the 467th line.
5
= 36 km/hour. 5
49. (C) Speed of the car A = × 90
46. (A) Let the distance between Delhi and Kanpur be x 6
km. = 75 km/hr.
Suppose the train leaving from Delhi is A and the 88
train leaving from Kanpur be B ∴ Reqd. time = × 60
90 + 75
x x
A’s speed = = km/hr. = 32 minutes
10 a.m. – 5 a.m. 5
x x 50. (B) Let the speed of the train be x km/hr.
B’s speed = = km/hr.
2 p.m. – 7 a.m. 7 ˙.˙ Length of the train
Since B starts two hours later than A, the distance 150 3
already covered by A at the time of start of B = 150 m = = km.
1000 20
2x
= km. and time taken to cross the man
5
Remaining distance = 10 seconds
2x 3x 10 1
= x – = km. = = hrs.
5 5 60 × 60 360
Relative speed of approach of two trains
Length of the train
( )
=
x x
+
5 7
=
12x
35
km/hr.
˙.˙ Time =
Speed of the train – Speed of the man

Time taken to cover the remaining distance by both 3


trains. 1 20
∴ =
3 360 (x – 2)
x
5 3 35 7 3
= = × = hrs. ⇒ (x – 2) = × 360
12x 5 12 4 20
35 ⇒ (x – 2) = 54
3
= 1 hrs. x = 54 + 2
4
= 1 hr. 45 min. = 56 km/hr.

Quantitative Aptitude Test | 17


More PDF download : http://www.gksolve.com

Area
Formulae πr2 θ ∴ Area = ⎯
√⎯⎯⎯⎯⎯⎯⎯⎯⎯⎯⎯⎯⎯⎯
s (s – a) (s – b) (s – c)
(iv) Area of sector ACBO =
(1) (i) Area of a rectangle 360
=⎯
√⎯⎯⎯⎯⎯⎯⎯⎯
14 × 5 × 2 × 7 cm2
= (Length × Breadth) (v) Area of sector ACBO
= 14 √
⎯ 5 cm2 = (14 × 2·23) cm2
(ii) Length = (
Area
Breadth
; ) =
1
2 (
× arc AB × r ) = 31·22 cm2
Breadth = (
Area
Length ) Examples
Q. 5. Find the area of a right
angled triangle, whose base is 6·5
(iii) (Diagonal)2 m and hypotenuse 9 m.
Q. 1. Find the area of a
= (Length)2 + (Breadth)2 Solution :
rectangle one of whose sides is 3 m
(2) Area of a square = (Side) 2 and diagonal 5 m. Height = ⎯√⎯⎯⎯⎯⎯⎯⎯
(9) 2 – (6·5)2
1 Solution :
= (Diagonal)2 = √⎯⎯⎯⎯
38·75 = 6·22 m
2
(3) Area of 4 walls of a room Another side = ⎯
√⎯⎯⎯⎯⎯⎯
(5) 2 – (3) 2 m ∴ Area
= 2 × (Length + Breadth) × Height = √⎯⎯16 m = 4 m 1
= × Base × Height
= Perimeter of base × Height 2
∴ Area of the plot
(4) Area of a parallelogram
= (Base × Height)
= (5 × 4) m2
= 20 m2.
= (
1
2 )
× 6·5 × 6·22 m2

(5) Area of a rhombus = 20·215 m2


Q. 2. Find the area of a square
1
= × (Product of diagonal)
2
the length of whose diagonal is 2·4 Exercise
m.
(6) Area of an equilateral triangle 1. The length of a rectangle is
Solution : doubled while its breadth is hal-
⎯√ 3 × (Side)2 1 ved. What is the percentage
= Area = × (diagonal)2
4 2 change in area ?
(7) If a, b, c are the length of the (A) 50
sides of a triangle and
1
=[ 1
2 ]
× (2·4)2 m2 (B) 75
s = (a + b + c) then (C) No change
=(
2 )
2 5·76 2 2
m = 2·88 m (D) None of these
Area of the triangle
Q. 3. Find the area of an 2. The cost of carpenting a room 15
=⎯√⎯⎯⎯⎯⎯⎯⎯⎯⎯⎯⎯⎯⎯⎯
s (s – a) (s – b) (s – c) metres long with a carpet 75 cm
equilateral triangle, each of whose
(8) Area of a triangle sides are 12 m long. wide at 30 paise per metre is Rs.
36. The breadth of the room is—
= (
1
2
× Base × Height ) Solution :
Area of the triangle
(A) 6 metres (B) 8 metres
(9) Area of a trapezium (C) 9 metres (D) 12 metres
1 ⎯√ 3 × (side)2 3. The cost of cultivating a square
= (Sum of parallel sides =
2 4 field at the rate of Rs. 160 per
× Distance between them)
(10) (i) Circumference of a circle = (
1·73
4
× 12 × 12 m2 ) hectare is Rs. 1440. The cost of
putting a fence around it at 75
= 2πr paise per metre is—
= 62·28 m2
(ii) Area of a circle = πr2 (A) Rs. 900 (B) Rs. 1800
Q. 4. Find the area of a triangle
2πrθ whose sides are 9 cm, 12 cm and 7 (C) Rs. 360 (D) Rs. 810
(iii) Arc AB = ,
360 cm. 4. The length of hall is (4/3) times
where ∠ AOB = θ Solution : its breadth. If the area of the hall
A C Let a = 9, b = 12 and c = 7 be 300 square metres, the diffe-
B rence between the length and the
θ 1 breadth is—
r Then, S = (9 + 12 + 7) cm
2 (A) 15 metres
O
= 14 cm (B) 4 metres
∴ (s – a) = 5, (s – b) = 2 (C) 3 metres
Circumference = 2 π r and (s – c) = 7 (D) None of these

Quantitative Aptitude Test | 18


More PDF download : http://www.gksolve.com

5. If the side of a square is doubled, 12. The length and breadth of a rec- 19. If the base of a rectangle is
then the ratio of the resulting tangular piece of land are in the increased by 10% and the area is
square to that of the given square ratio of 5 : 3. The owner spent unchanged, then the correspon-
is— Rs. 3000 for surrounding it from ding altitude must be decreased
(A) 1 : 2 (B) 2 : 1 all the sides at Rs. 7·50 per metre. by—
(C) 3 : 1 (D) 4 : 1 The difference between its length 1
and breadth is— (A) 9 % (B) 10%
11
6. If the side of a square is increased (A) 50 m (B) 100 m
by 25%, then how much per cent 1
(C) 150 m (D) 200 m (C) 11% (D) 11 %
does its area get increased ? 9
(A) 125 (B) 156·25 13. The length of a rectangular plot 20. The length of a rectangle is twice
is twice of its width. If the length its breadth. If its length is de-
(C) 50 (D) 56·25
of a diagonal is 9√⎯ 5 metres, the creased by 5 cm and the breadth
7. A rectangular plot is half as long perimeter of the rectangle is— is increased by 5 cm, the area of
again as it is broad. The area of the rectangle is increased by 75
(A) 27 m
the lawn is (2/3) hectares. The cm2. Therefore, the length of the
length of the plot is— (B) 54 m
rectangle is—
(A) 100 metres (C) 81 m
(A) 20 cm (B) 30 cm
(B) 66·66 metres (D) None of these
(C) 40 cm (D) 50 cm
1 14. The area of a rectangle 144 m
(C) 33 metres 21. The cost of papering the four
3 long is the same as that of a
square having a side 84 m long. walls of a room is Rs. 48. Each
(D) ⎛⎜
100 ⎞ metres one of the length, breadth and
⎟ The width of the rectangle is—
⎝√⎯ 3⎠ (A) 7 m height of another room is double
that of this room. The cost of
8. If the side of a square be (B) 14 m papering the walls of this new
increased by 4 cms. the area (C) 49 m room is—
increases by 60 sq. cms. The side (D) Cannot be determined (A) Rs. 384 (B) Rs. 288
of the square is—
15. The length and breadth of a (C) Rs. 192 (D) Rs. 96
(A) 12 cm playground are 36 m and 21 m
(B) 13 cm respectively. Flagstaffs are 22. Area of four walls of a room is
(C) 14 cm required to be fixed on all along 168 sq. metres. The breadth and
(D) None of these the boundary at a distance of 3 m height of the room are 8 metres
apart. The number of flagstaffs and 6 metres respectively. The
1
9. Area of a square is hectare. will be— length of the room is—
2
The diagonal of the square is— (A) 37 (B) 38 (A) 14 metres (B) 12 metres
(A) 250 metres (C) 39 (D) 40 (C) 6 metres (D) 3·5 metres
(B) 100 metres 16. The length of a plot is four times 23. Area of four walls of a room is
its breadth. A playground mea- 77 square metres. The length and
(C) 50 √⎯ 2 metres suring 1200 square metres occu- breadth of the room are 7·5
(D) 50 metres pies one third of the total area of metres and 3·5 metres respecti-
10. A Verandah 40 metres long 15 the plot. What is the length of vely. The height of the room is—
metres broad is to be paved with the plot in metres ?
(A) 3·5 metres
stones each measuring 6 dm by 5 (A) 20
dm. The number of stones (B) 30 (B) 5·4 metres
required is— (C) 60 (C) 6·77 metres
(A) 1000 (D) None of these (D) 7·7 metres
(B) 2000 17. If the length of diagonal AC of a 24. A rectangle has 15 cm as its
(C) 3000 square ABCD is 5·2 cm. then length and 150 cm2 as its area.
(D) None of these area of the square ABCD is— 1
(A) 15·12 sq. cm Its area is increased to 1 times
11. The ratio between the length and 3
breadth of a rectangular field is (B) 13·52 sq. cm the original area by increasing
5 : 4. If the breadth is 20 metres (C) 12·62 sq. cm only its length. Its new perimeter
less than the length, the peri- (D) 10 sq. cm is—
meter of the field is— 18. One side of a rectangular field is (A) 50 cm (B) 60 cm
(A) 260 m 4 metres and its diagonal is 5 (C) 70 cm (D) 80 cm
(B) 280 m metres. The area of the field is—
25. The length of a rectangular room
(C) 360 m (A) 12 m2 (B) 15 m2
is 4 metres. If it can be parti-
(D) None of these (C) 20 m2 (D) 4√
⎯ 5 m2 tioned into two equal square

Quantitative Aptitude Test | 19


More PDF download : http://www.gksolve.com

rooms. What is the length of of the longest pole that can be 41. If the circumference of a circle is
each partition in metre ? placed in the park ? 352 metre, then its area in m2
(A) 1 (A) 10 metres is—
(B) 2 (B) 12·8 metres (A) 9856 (B) 8956
(C) 4 (C) 13·4 metres (C) 6589 (D) 5986
(D) Data inadequate (D) 18 metres 42. The ratio of the area of a square
26. The length and breadth of a 33. The ratio of the areas of two of side a and equilateral triangle
square are increased by 40% and squares, one having double its of side a is—
30% respectively. The area of diagonal than the other is— (A) 2 : 1 (B) 2 : √
⎯3
the resulting rectangle exceeds (A) 2 : 1 (B) 3 : 1
the area of the square by— (C) 3 : 2 (D) 4 : 1 (C) 4 : 3 (D) 4 : √
⎯3
(A) 42% 43. Area of a square with side x is
34. The area of a rectangle is thrice
(B) 62% equal to the area of a triangle
that of a square. Length of the
(C) 82% rectangle is 40 cm and the with base x. The altitude of the
(D) None of these breadth of the rectangle is (3/2) triangle is—
times that of the side of the x
27. A hall 20 m long and 15 m broad (A) (B) x
is surrounded by a verandah of square. The side of the square in 2
uniform width of 2·5 m. The cost cm is— (C) 2x (D) 4x
of flooring the verandah at the (A) 15 (B) 20
44. In a triangle ABC, BC = 5 cm,
rate of Rs. 3·50 per sq. metre is— (C) 30 (D) 60 AC = 12 cm and AB = 13 cm.
(A) Rs. 500 (B) Rs. 600 35. If the diametre of a circle is The length of the altitude drawn
(C) Rs. 700 (D) Rs. 800 increased by 100%. Its area is from B on AC is—
increased by— (A) 4 cm (B) 5 cm
28. A rectangular lawn 60 metres by
40 metres has two roads each 5 (A) 100% (B) 200% (C) 6 cm (D) 7 cm
metres wide running in the (C) 300% (D) 400%
45. The altitude of an equilateral
middle of it. One parallel to 36. If the radius of a circle be
length and the other parallel to reduced by 50%. Its area is re- triangle of side 2 √
⎯ 3 cm is—
breadth. The cost of gravelling duced by— ⎯ 3 cm
√ 1
the roads at 60 paise per sq. (A) 25% (B) 50% (A) (B) cm
2 2
metre is— (C) 75% (D) 100%
(A) Rs. 300 (B) Rs. 280 ⎯ 3 cm

37. The radius of a circle has been (C) (D) 3 cm
(C) Rs. 285 (D) Rs. 250 4
reduced from 9 cms to 7 cm. The
29. Of the two square fields the area appropriate percentage decrease 46. The length of each side of an
of the one is 1 hectare, while the in area is— equilateral triangle having an
another one is broader by 1%. (A) 31·5% (B) 39·5% area of 4√
⎯ 3 cm2 , is—
The difference in areas is— (C) 34·5% (D) 65·5%
4 ⎯ 3 cm

(A) 101 sq. metres 38. The difference between the cir- (A) cm (B)
⎯3
√ 4
(B) 201 sq. metres cumference and the radius of a
(C) 100 sq. metres circle is 37 cms. The area of the (C) 3 cm (D) 4 cm
(D) 200 sq. metres circle is— 47. The area of an equilateral trian-
(A) 148 sq. cm gle whose side is 8 cms is—
30. If the ratio of the areas of two (B) 111 sq. cm
squares is 9 : 1, the ratio of their (C) 154 sq. cm (A) 64 cm2 (B) 16⎯
√ 3 cm2
perimetres is—
(D) 259 sq. cm (C) 21·3 cm2 (D) 4√
⎯ 3 cm2
(A) 9 : 1 (B) 3 : 1
39. The area of a circular field is 48. The perimeter of an isosceles
(C) 3 : 4 (D) 1 : 3
13·86 hectares. The cost of fen- triangle is equal to 14 cm. The
31. The length of a rectangular field cing it at the rate of 20 paise per lateral side is to the base in the
is increased by 60%. By what metre is— ratio 5 : 4. The area of the
per cent would the width have to (A) Rs. 277·20 triangle is—
be decreased to maintain the (B) Rs. 264 1
same area ? (C) Rs. 324 (A) √ ⎯⎯21 cm2
2
1 (D) Rs. 198 3
(A) 37 % (B) 60% (B) √ ⎯⎯21 cm2
2 40. The area of circle is 38·5 sq. cm. 2
(C) 75% (D) 120% Its circumference is—
(C) ⎯⎯21 cm2

32. A park is 10 metres long and 8 (A) 6·20 cm (B) 11 cm
metres broad. What is the length (C) 22 cm (D) 121 cm (D) 2 √
⎯⎯21 cm2

Quantitative Aptitude Test | 20


More PDF download : http://www.gksolve.com

49. The sides of a triangular board 57. The number of rounds that a 2 2
(A) 28 m
are 13 metres,14 metres and 15 7 7
wheel of diameter m will
metres. The cost of painting it at 11 8
the rate of Rs. 8·75 per m 2 is— make in going 4 km, is— (B) 64 m2
9
(A) Rs. 688·80 (A) 1000 (B) 1500 2
(B) Rs. 735 (C) 1700 (D) 2000 (C) 59 m2
3
(C) Rs. 730·80 58. A wheel makes 1000 revolutions (D) None of these
(D) Rs. 722·50 in covering a distance of 88 km. 66. The area of the largest circle that
50. The base of a rightangled triangle The diameter of the wheel is— can be drawn inside a square of
is 5 metres and hypotenuse is 13 (A) 24 metres (B) 40 metres 14 cm length is—
metres. Its area will be— (C) 28 metres (D) 14 metres (A) 84 m2
(A) 25 m2 59. The diameter of a wheel is 63 (B) 154 cm2
(B) 28 m2 cms. Distance travelled by the (C) 204 cm2
(C) 30 m2 wheel in 100 revolutions is— (D) None of these
(D) None of these (A) 99 metres (B) 198 metres
(C) 63 metres (D) 136 metres 67. If 88 m wire is required to fence
51. If the diagonal of a square is a circular plot of land, then the
doubled, how does the area of 60. A circular road runs rounds a area of the plot is—
the square change ? circular ground. If the difference (A) 526 m2
(A) Becomes four fold between the circumferences of
the outer circle and the inner (B) 556 m2
(B) Becomes three fold
(C) Becomes two fold circle is 66 metres, the width of (C) 616 m2
the road is— (D) None of these
(D) None of the above
(A) 21 metres (B) 10·5 metres
52. The largest possible square is 68. If the circumference of a circle is
(C) 7 metres (D) 5·25 metres
inscribed in a circle of unit increased by 50%, then its area
radius. The area of the square 61. The inner circumference of a will be increased by—
unit is— circular race track, 14 m wide is (A) 50% (B) 100%
(A) 2 (B) π 440 m. Then the radius of the
(C) 125% (D) 225%
outer circle is—
(C) (2 √
⎯ 2) π (D) (4 √ ⎯ 2) π (A) 70 m (B) 56 m 69. If a regular hexagon is inscribed
53. A circle and a square have same (C) 77 m (D) 84 m in a circle of radius, r then its
area. The ratio of the side of the perimeter is—
square and the radius of the circle 62. The ratio of the radii of two
circles is 1 : 3. The ratio of their (A) 3r (B) 6r
is—
areas is— (C) 9r (D) 12r
(A) √⎯ π : 1 (B) 1 : √ ⎯π (A) 1 : 3
(C) 1 : π (D) π : 1 70. The area of the largest triangle
(B) 1 : 6 that can be inscribed in a semi
54. The radius of the wheel of a (C) 1 : 9 circle of radius r cm is—
vehicle is 70 cm. The wheel (D) None of these (A) 2r cm 2 (B) r2 cm2
makes 10 revolutions in 5 se-
conds. The speed of the vehicle 63. The area of a square is 50 sq. 1
is— units. Then the area of the circle (C) 2r2 cm2 (D) r2 cm2
2
(A) 29·46 km/hr. drawn on its diagonal is—
(A) 25 π sq. units 71. Four horses are be tethered at
(B) 31·68 km/hr.
four corners of a square plot of
(C) 36·25 km/hr. (B) 100 π sq. units side 63 metres, so that they just
(D) 32·72 km/hr. (C) 50 π sq. units cannot reach one another. The
55. The radius of a wheel is 1·4 (D) None of these area left ungrazed is—
decimetre. How many times (A) 675·5 m2 (B) 780·6 m2
does it revolve during a journey 64. A circular wire of radius 42 cm
is cut and bent in the form of a (C) 785·8 m2 (D) 850·5 m2
of 0·66 km ?
rectangle whose sides are in the
(A) 375 (B) 750 72. Four circular card board pieces,
ratio of 6 : 5. The smaller side of
(C) 1500 (D) 3000 each of radius 7 cm are placed in
the rectangle is— such a way that each piece
56. A circular disc of area 0·49 π (A) 30 cm (B) 60 cm touches two other pieces. The
square metres rolls down a length
(C) 72 cm (D) 132 cm area of the space enclosed by the
of 1·76 km. The number of revo-
four pieces is—
lutions it makes is— 65. The area of the largest circle that
(A) 300 (B) 400 can be drawn inside a rectangle (A) 21 cm2 (B) 42 cm2
(C) 600 (D) 4000 with sides 7 m by 6 m is— (C) 84 cm2 (D) 168 cm2

Quantitative Aptitude Test | 21


More PDF download : http://www.gksolve.com

73. The length of a rope by which a The area of the circle, excluding 89. A room 5·44 m × 3·74 m is to be
cow must be tethered in order the area of the rectangle is— paved with square tiles. The least
that she may be able to graze an (A) 65·3 m2 (B) 42·4 m2 number of tiles required to cover
area of 9856 sq. m is— (C) 30·6 m2 (D) 39 m2 the floor is—
(A) 56 m (B) 64 m (A) 162 (B) 176
82. The perimeter of a square
(C) 88 m (D) 168 m circumscribed about a circle of (C) 184 (D) 192
74. The radius of a circle is increa- radius r is— 90. Area of smallest square that can
sed so that its circumference (A) 2r (B) 4r circumscribe a circle of area 616
increases by 5%. The area of the cm2 is—
(C) 8r (D) 21πr
circle will increase by— (A) 784 cm2
(A) 10% (B) 10·25% 83. The area of a circle inscribed in (B) 824 cm2
(C) 8·75% (D) 10·5% an equilateral triangle is 462 cm.
(C) 864 cm2
The perimeter of the triangle
75. The circumferences of two is— (D) None of these
concentric circles are 176 m and 91. Radhika runs along the boundary
132 m respectively. What is the (A) 42√
⎯ 3 cms
of a rectangular park at the rate
difference between their radii ? (B) 126 cms of 12 km/hr and completes one
(A) 5 metres (B) 7 metres (C) 72·6 cms full round in 15 minutes. If the
(C) 8 metres (D) 44 metres (D) 168 cms length of the park is 4 times its
84. The area of the circle inscribed breadth, the area of the park is—
76. The diameter of a circle is 105
cm less than the circumference. in an equilateral triangle of side (A) 360000 m2
What is the diameter of the 24 cms is— (B) 36000 m2
circle? (A) 24 π cm2 (B) 36 π cm2 (C) 3600 m2
(A) 44 cm (B) 46 cm (C) 48 π cm2 (D) 18 π cm2 (D) None of these
(C) 48 cm (D) 49 cm 92. The ratio of the corresponding
85. A park is in the form of a square
77. The area of the sector of a circle, one of whose sides is 100 m. The sides of two similar triangles is
whose radius is 12 metres and area of the park excluding the 3 : 4. The ratio of their areas is—
whose angle at the centre is 42 circular lawn in the centre of the (A) 4 : 3 (B) 3 : 4
is— park is 8614 m2 . The radius of (C) 9 : 16 (D) ⎯3:2

(A) 26·4 sq. metres the circular lawn is—
(B) 39·6 sq. metres (A) 21 m 93. If D, E and F are respectively the
midpoints of the sides BC, CA
(C) 52·8 sq. metres (B) 31 m and AB of a Δ ABC and the area
(D) 79·2 sq. metres (C) 41 m of Δ ABC = 36 m2 , then area of
78. The length of minute hand of a (D) None of these Δ DEF is—
wall clock is 7 cms. The area (A) 12 cm2 (B) 9 cm2
swept by the minute hand in 30 86. A rectangular carpet has an area 2
of 60 sq. m. Its diagonal and (C) 18 cm (D) 9 cm2
minutes is—
longer side together equal 5 94. In Δ ABC, side BC = 10 cm and
(A) 147 sq. cm times the shorter side. The length height AD = 4·4 cm. If AC = 11
(B) 210 sq. cm of the carpet is— cm. Then altitude BE equals—
(C) 154 sq. cm (A) 5 m (B) 12 m (A) 5 cm (B) 4 cm
(D) 77 sq. cm (C) 5·6 cm (D) 5·5 cm
(C) 13 m (D) 14·5 m
79. In a circle of radius 21 cm an arc 95. ABCD is a trapezium in which
87. A rectangular carpet has an area
subtends an angle of 72° at the AB | | CD and AB = 2 CD. If its
of 120 sq. m and a perimeter of
centre. The length of the arc is— diagonal intersect each other at
46 m. The length of its diagonal
(A) 13·2 cm (B) 19·8 cm is— O, then ratio of areas of triangles
(C) 21·6 cm (D) 26·4 cm AOB and COD is—
(A) 15 m (B) 16 m
80. The area of sector of a circle of (A) 1 : 2 (B) 2 : 1
(C) 17 m (D) 20 m
radius 5 cm, formed by an arc of (C) 1 : 4 (D) 4 : 1
length 3·5 cms is— 88. If x is the length of a median of 96. The cross section of canal is a
an equilateral triangle then its trapezium in shape. If the canal
(A) 35 sq. cms
area is— is 10 metres wide at the top and
(B) 17·5 sq. cms
(C) 8·75 sq. cms x2 ⎯3
√ 6 metres wide at bottom and the
(A) x2 (B) area of cross section is 640 sq.
(D) 55 sq. cms 2
metres. The depth of canal is—
81. The vertices of rectangle with x2 ⎯3
√ x2 (A) 40 metres
(C) (D)
sides 8 m and 6 m, lie on a circle. 3 2 (B) 80 metres

Quantitative Aptitude Test | 22


More PDF download : http://www.gksolve.com

(C) 160 metres angle between them is 30°. The times the square of the length of
(D) 384 metres area of the parallelogram is— the longer diagonal ?
(A) 12 cm2 4 2
97. The area of a trapezium is 384 (A) (B)
(B) 24 cm2 5 5
sq. cm. If its parallel sides are in
ratio 3 : 5 and the perpendicular (C) 48 cm2 3 1
(C) (D)
distance between them be 12 cm. (D) None of these 4 4
The smaller of parallel sides is— 101. One side of a parallelogram is 14 104. In a rhombus, whose area is 144
(A) 16 cm (B) 24 cm cm. Its distance from the oppo- sq. cm one of its diagonals is
(C) 32 cm (D) 40 cm site side is 16 cm. The area of twice as long as the other. The
the parallelogram is— lengths of its diagonals are—
98. The two parallel sides of a
trapezium are 1 metre and 2 (A) 112 sq. cm (A) 24 cm, 48 cm
metre respectively. The perpen- (B) 224 sq. cm (B) 12 cm, 24 cm
dicular distance between them is (C) 56 π sq. cm (C) 6√⎯ 2 cm 12⎯√ 2 cm
6 metres. The area of the trape- (D) 210 sq. cm (D) 6 cm, 12 cm
zium is—
(A) 18 sq. metres 102. If a square and a rhombus stand 105. The perimeter of a rhombus is 52
(B) 12 sq. metres on the same base, then the ratio metres while its longer diagonal
(C) 9 sq. metres of the areas of the square and the is 24 metres. Its other diagonal
(D) 6 sq. metres rhombus is— is—
(A) Greater than 1 (A) 5 metres (B) 10 metres
99. A parallelogram has sides 60 m
and 40 m and one of its diagonal (B) Equal to 1 (C) 20 metres (D) 28 metres
is 80 m long. Then its area is— 1 106. One side of a rhombus is 10 cms
(C) Equal to
(A) 480 sq. m 2 and one of its diagonals is 12
(B) 320 sq. m 1 cms. The area of the rhombus
(D) Equal to is—
4
(C) 600√ ⎯⎯15 sq. m (A) 120 sq. cm
103. The length of the diagonal of a
(D) 450√ ⎯⎯15 sq. m (B) 96 sq. cm
rhombus is 80% of the length of
100. The adjacent sides of parallelo- the other diagonal. Then the area (C) 80 sq. cm
gram are 6 cm and 4 cm and the of the rhombus is how many (D) 60 sq. cm

Answers with Hints


1. (C) Let length = l and breadth = b ∴ One side = √ ⎯⎯⎯⎯
90000 m
Then, area = lb = 300 m
New length = 2l So, Perimeter = (4 × 300) m = 1200 m
b
and new breadth =
2 ∴ Cost of fencing = Rs. 1200 × ( 75
100 )
∴ ( )
New area = 2l ×
b
2
= lb = Rs. 900
4. (D) Let breadth = x metres.
So, there is no change in area.
2. (A) Length of carpet =
Total Cost
Rate
Then, length =
4
3( )
x metres

4
3600 ˙·˙ x × x = 300
= = 120 m 3
30
3
⇒ x2 = 300 × = 225
(
Area of carpet = 120 ×
75
100 ) m2
⇒ x = 15
4

= 90 m2
Hence, req. difference = [(Length) – (Breadth)]
∴ Area of the room = 90 m2
Hence, breadth of the room = (4
3 )
x–x = x
1
3
=
Area
Length
= ( )
90
15
m
= (1
)
× 15 m
= 6m 3
= 5m
3. (A) Area = ( )
1440
160
hectares 5. (D) Let original length = x metres
= 9 hectares = 90000 m2 New length = (2x) metres

Quantitative Aptitude Test | 23


More PDF download : http://www.gksolve.com

4x2 4 ˙·˙ 16x = 400


∴ Ratio of the area = =
x2 1 ⇒ x = 25
= 4:1 ∴ (Length) – (Breadth)
6. (D) Let area 100 m 2 = (5 × 25 – 3 × 25) m
Then, side = 10 m = (2 × 25) m = 50 m
New side = (125% of 10) 13. (B) Let breadth = x metres,
=( 125
100 )
× 10 Then,

length = 2x metres
Diagonal = ⎯⎯⎯⎯⎯⎯
√ x2 + 4x2 = ⎯
√⎯5x2
= 12·5 m
New area = (12·5 × 12·5) m2 = x√⎯ 5 metres
= (12·5)2 sq. m So, √
⎯5 x = 9 √
⎯5
∴ Increase in area = [12·5)2 – (10) 2 ] m 2 ∴ x = 9
= [(12·5 + 10) (12·5 – 10)] m2 Thus, breadth = 9 m and length = 18 m
= (22·5 × 2·5) m2 ∴ Perimeter = 2 (18 + 9) m = 54 m.
= 56·25 m2 14. (C) Area of the square = (84 × 84) m2
% Increase = 56·25% Area of the rectangle = (84 × 84) m2
7. (A) Let breadth = x metres. 84 × 84
3 ∴ Width = m = 49 m
Then, length = x metres 144
2 15. (B) Perimeter = 2 × (36 + 21) m = 114 m
3 2 114
∴ x × x = × 10000 ∴ Number of Flagstaffs = = 38
2 3 3
4 16. (D) Area of the plot = (3 × 1200) m2 = 3600 m2
⇒ x2 = × 10000
9 Let breadth = x metres,
⇒ x = ( 2
3 )
× 100 m Then
˙·˙
length = 4x metres
4x × x = 3600 m2
∴ Length = ( 3 2
)
× × 100 m = 100 m
2 3 ⇒ x2 = 900 m2
8. (D) Let each side = x cm ⇒ x = 30 m
Then, (x + 4) 2 – x2 = 60 ∴ Length of plot = 4x m
⇒ x + 8x + 16 – x2 = 60
2 = (4 × 30) m
∴ x = 5·5 cm = 120 m

9. (B) Area = ( 1
2 )
× 10000 m2 = 5000 m2 17.
1
(B) Area = × (Diagonal)2 =
2 ( 1
2 )
× 5·2 × 5·2 cm2

1 = 13·52 cm2
∴ × (Diagonal)2 = 5000
2 18. (A) Other side = ⎯
√⎯⎯⎯⎯
52 – 42 = ⎯
√9 = 3 m
⇒ (Diagonal)2 = 10000 19. (A) Let base = b and altitude = h
⇒ Diagonal = 100 m
Then, Area = b × h …(1)
10. (B) Length = (40 × 10) dm = 400 dm,
110 11
Breadth = (15 × 10) dm = 150 dm But New base = ·b= ·b
100 10
Area of Verandah = (400 × 150) dm2
Let New altitude = H
Area of one stone = (6 × 5) dm2
11
∴ Required number of stones Then, ˙·˙ b × H = b·h
10
400 × 150
= ( 6×5 ) = 2000 ⇒ H =
10
11
·h
11. (C) 5x – 4x = 20 m
⇒ x = 20 m ∴ (
Decrease = h – h
10
11 )
∴ Length = (5 × 20) m = 100 m, 1
= h
Breadth = (4 × 20) m = 80 m 11
∴ Perimeter = 2 (100 + 80) m = 360 m. ∴ Required decrease per cent
12. (A) Let length = 5x metres and breadth = 3x metres,
Then, perimeter = 2 × (5x + 3x) m = 16x metres
= ( 1
11
1
)
h × h × 100 %

Total Cost 3000 1


But perimeter = = m = 400 m = 9 %.
Rate 7·50 11

Quantitative Aptitude Test | 24


More PDF download : http://www.gksolve.com

20. (C) Let breadth = l, length = 2l Increase in area = [(140 × 130) – (100 × 100)] m2
∴ Area of rectangle = 2l × l = 8200 m2
= 2l2 …(1) ∴ Required Increase per cent
As per question,
˙·˙ (2l – 5) (l + 5) = 2l2 + 75 cm2
= ( 8200
100 × 100 )
× 100 % = 82%

⇒ 2l2 – 5l + 10l – 25 = 2l2 + 75 27. (C) Area of verandah = [(25 × 20) – (20 × 15)] m2
= 200 m2
⇒ 5l = 75 + 25
∴ Cost of flooring = Rs. (200 × 3·50)
= 100
= Rs. 700
100
∴ l = = 20 28. (C) Area of the roads
5
= (60 × 5 + 40 × 5 – 5 × 5) m2
Hence, length of the rectangle
= 475 m2
= 2l = 2 × 20 = 40 cm.
21. (C) Let the length, breadth and height of the room be
l, b and h respectively
As per question
Cost of 2 (l + b) × h = Rs. 48
∴ Required cost = Cost of 2 (2l + 2b) × 2h ∴ Cost of gravelling = Rs. (475 × 100
60
)
= Cost of 4 [2 (l + b) × h] = Rs. 285
= 4 × Rs. 48 29. (B) Area of the square field = 10000 m2
= Rs. 192.
22. (C) ˙·˙ 2 (l + 8) × 6 = 168 m2 Side of the square = √ ⎯⎯⎯⎯
10000 m = 100 m
Side of another square field
⇒ (l + 8) = 14
= 101 m
∴ l = 14 – 8
∴ Required difference of areas
= 6 metres
= [(101)2 – (100) 2 ] m 2
23. (A) ˙·˙ 2 × (7·5 + 3·5) × h = 77 m2
= [(101 + 100) (101 – 100)] m 2 = 201 m2
77 7
⇒ h = = 30. (B) Let the area of squares be (9x)2 m2 and (x2) m 2
2 × 11 2
h = 3·5 metres. Then, their sides are (3x) m and x metres respectively
∴ Ratio of their perimeters
24. (B) Breadth of the rectangle = ( ) 150
15
cm
=
12x
= 10 cm 4x

New area = ( 4
3 )
× 150 cm2
= 3:1
31. (A) Let length = x and breadth = y
= 200 cm2 Let, New breadth = z
New length = ( ) 200
10
cm Then, New length = ( )160
100
x
= 20 cm 8
= x
New perimeter = 2 (20 + 10) cm 5
= 60 cm 8
∴ x × z = xy
25. (B) Let the width of the room be x metres 5
Then, its area = (4x) m2 5y
or z =
Area of each new square room 8
= (2x) m2 Decrease in breadth
Let the side of each new room
= y metres
( )
= y–
5y
8
3
Then y2 = 2x = y
8
Clearly, 2x is a complete square when x = 2
∴ Decrease per cent
∴ y2 = 4
⇒ y = 2 m. = ( 3
8
1
)
y × y × 100 %
26. (C) Let the side of the square = 100 m 1
= 37 %
New length = 140 m, New breadth = 130 m 2

Quantitative Aptitude Test | 25


More PDF download : http://www.gksolve.com

32. (B) Length of the longest pole Decrease per cent


=⎯√⎯⎯⎯⎯⎯⎯⎯
(10) 2 + (8)2 m = √
⎯⎯⎯
164 m = 12·8 m.
33. (D) Let the diagonal of one square be (2x) cm
= ( 22
7
× 16 × 2 ×
7
22 × 9 × 9 )
× 100 %

= 39·5%
Then, diagonal of another square= x cm
38. (C) ˙·˙ 2πr – r = 37
∴ Area of first square =
1
2 [
× (2x)2 cm2 ] ⇒ ( 2×
22
7 )
– 1 r = 37
= (2x2 ) cm2
37
⇒ r = 37
Area of second square = (12 x ) cm
2 2

7
r = 7
2x2 4 So, area of the circle
∴ Ratio of areas = = =4:1
2 ( )
1 2
x
1
= ( 22
7 )
× 7 × 7 cm2
34. (B) Let the side of the square = x cm
= 154 cm2
Then, breadth of the rectangle 22 2
3 39. (B) ˙·˙ × r = 13·86 × 10000
= x cm 7
2
∴ Area of rectangle ⇒ (
r2 = 13·86 × 10000 ×
7
22)
( 3
= 40 × x cm2
2 ) ∴ r = 210 m

= (60x) cm2 ⇒ Circumference = 2 × ( 22


7 )
× 210 m
˙·˙ 60x = 3x2 = 1320 m
∴ x = 20
Hence, the side of the square
= 20 cm
Cost of fencing = Rs. 1320 ×( 20
100)
= Rs. 264
d 2 πd2
35. (C) Original area = πx
2() =
4 40. (C) ˙·˙
22 2
× r = 38·5
7
New area = π × ( )
2d 2
2
= πd2
⇒ (
r2 = 38·5 × )
7
22
πd2
Increase in area = πd2 –( 4
=
3πd2
4 ) ⇒ r = 3·5 cm
∴Required increase per cent
⎛ 3πd2 4 ⎞
∴ Circumference = 2 × ( 22
7 )
× 3·5 cm

= ⎜ × 2 × 100⎟ % = 22 cm
⎝ 4 πd ⎠ 22
= 300% 41. (A) 2 × × r = 352
7
36. (C) Original area = π × r2
⇒ (
r = 352 ×
7 1
×) = 56 m
(r2) = πr4
2
2 22 2
New area = π ×

Reduction in area = (πr –


πr ∴ Area = ( 22
)
× 56 × 56 m2
4 )
3πr 2 2
2 7
=
4 = 9856 m2
⎛ 3πr2 4 ⎞ a2 4
∴ Reduction per cent = ⎜ 4 × πr2 × 100⎟ % 42. (D) Ratio of areas = = =4:⎯ √3
⎝ ⎠ ⎯ 3 a2 √
√ ⎯3
= 75% 4
37. (B) Original area = ( 22
7 )
× 9 × 9 cm2 43. (C) ˙·˙
1
x2 = × x × h
2
2x2
New area = (227 × 7 × 7) cm 2 ⇒ h = x = 2x

22 1
∴ Decrease = × [(9)2 – (7) 2 ] cm2 44. (B) ˙·˙ s = (13 + 5 + 12) cm = 15 cm
7 2

= (22
7
× 16 × 2 cm2 ) (s – a) = 2 cm, (s – b) = 10 cm and
(s – c) = 3 cm

Quantitative Aptitude Test | 26


More PDF download : http://www.gksolve.com

⇒ Area = ⎯⎯⎯⎯⎯⎯⎯⎯⎯⎯
√ 15 × 2 × 10 × 3 cm2 = 30 cm2 53. (A) ˙·˙ x2 = πr2
1 x
r=√
⇒ × 12 × h = 30 ⇒ ⎯π = ⎯ π : 1.

2
∴ h = 5 cm 54. (B) Circumference = (2 × 227 × 70) cm
√3 × h = √
1 ⎯ 3 × (2 ⎯√ 3)2
45. (D) ˙·˙ ×2⎯ = 440 cm
2 4
∴ h = 3 cm. Distance travelled in 10 revolutions
= 4400 cm = 44 m
⎯ 3 a2 = 4 √
√ ⎯ 3.
46. (D) ˙·˙

4
a2 = 16
∴ Speed =
Distance
Time
= ( )
44
5
m/sec

∴ a = 4 cm
⎡⎢ √
= (44 18
5
×
5 )
km/hr
⎯ 3 × (8)2⎤⎥ cm2 = 16⎯√ 3 cm2 = 31·68 km/hr
47. (B) Req. area = ⎢ ⎥⎦
⎣4 55. (B) r = 0·14 m
48. (D) Let lateral side = (5x) cm and base = (4x) cm
Required number of revolutions
∴ 5x + 5x + 4x = 14 or x = 1
0·66 × 1000 7
So, the sides are 5 cm, 5 cm and 4 cm = ( 2
× ×
1
22 0·14 ) = 750
1
s = (5 + 5 + 4) cm = 7 cm 56. (B) πr2 = 0·49 π ⇒ r = 0·7 m
2
(s – a) = 2 cm Number of revolutions
(s – b) = 2 cm 1·76 × 1000
=
22
and (s – c) = 3 cm 2× × 0·7
7
∴ Required Area = ⎯√⎯⎯⎯⎯⎯⎯⎯⎯
7 × 2 × 2 × 3 cm2 = 400
= 2√⎯⎯21 cm2 57. (D) Number of rounds
1 4 × 1000
49. (B) s = (13 + 14 + 15) = 21, s – a = 8, s – b = 7, = = 2000
2 22 7
×
s–7=6 7 11
58. (C) Distance covered in one revolution
∴ Area to be painted = ⎯√⎯⎯⎯⎯⎯⎯⎯⎯⎯⎯⎯⎯⎯⎯
s (s – a) (s – b) (s – c)
88 × 1000
= √
⎯⎯⎯⎯⎯⎯⎯⎯⎯
21 × 8 × 7 × 6 m2 =
1000
m = 88 m
= 84 m2 ˙·˙ π × d = 88
∴ Cost of painting = Rs. (84 × 8·75) = Rs. 735
22
⇒ × d = 88
50. (C) Altitude = ⎯
√⎯⎯⎯⎯⎯⎯⎯
(13) 2 – (5) 2= ⎯
√⎯⎯
144 = 12 m 7
∴ Area of the triangle = (1
2 )
× 5 × 12 m2 ∴ d = (88 × 227 ) = 28 m
= 30 m2 59. (B) Distance travelled in 100 revolutions
1
51. (A) Ratio of the areas =
1
2
× d2
=
1
4
= 2× ( 22 63
7
×
2
× 100 cm )
× (2d2 )
∴ New area becomes 4 fold.
2 = (2 × 227 × 632 × 100 × 1001 ) m
52. (A) ˙·˙ Diagonal of the square = Diagonal of circle = 198 m
= 2 units 60. (B) ˙·˙ 2π R – 2 πr = 66
⇒ 2π (R – r) = 66
22
⇒ 2× × (R – r) = 66
7

∴ (R – r) = (66 × 227 × 12)


∴ Area = [12 × (2) ] = 2 sq. units
2
=
21
2
= 10·5 m

Quantitative Aptitude Test | 27


More PDF download : http://www.gksolve.com

22 Req. increase per cent 0


61. (D) ˙·˙ 2 × × r = 440
7 ⎛ 5πr2 1 ⎞
= ⎜ 4 × πr2 × 100⎟ %
⇒ r = 440 ×( 7 1
22 2
× )
= 70 m ⎝ ⎠
= 125%
∴ Radius of outer circle
69. (B) Length of each side of hexagon = r
= (70 + 14) m = 84 m
62. (C) Ratio of areas ∴ Its perimeter = 6r
πr2 πr2 1 70. (B) Area of the triangle
= = = =1:9

1
π (3r2 ) 9 πr2 9 = (
1
2 )
× 2r × r cm2
63. (A) ˙·˙ × (Diagonal)2 = 50
2
⇒ (Diagonal) = 10 units
∴ Radius of required circle = 5 units = r2 cm2
Its area = [π × (5)2 ] cm2 = (25 π)2 units
71. (D) Area left ungrazed
64. (B) ˙·˙ Circumference = 2 × ( 22
7 )
× 42 cm = 264 cm
= [63 × 63 – 4 × 14 ×227 ×(632) ] m 2
2

⇒ 2 × (6x + 5x) = 264


99 × 63
= (63 × 63 –
2 )
m 2
⇒ x = 12
Smaller side of rectangle = 5x = 60 cm
= 63 × (63 – ) m = 850·5 m
99 2 2
65. (A) ˙·˙ Radius of circle = 3 m 2

∴ Area of circle = (227 × 3 × 3) m


2 72. (B) Required area enclosed
= [(14 × 14) – 4 × (area of quadrant)] cm2

2 2
= 28 m 90°
7
66. (B) ˙·˙ Radius of circle = 7 m

= (196 – 4 ×227 × 7 × 7 ×360°


90°
) cm 2

= (196 – 154) cm 2 = 42 cm2


∴ Its area = (227 × 7 × 7) cm = 154 cm
2 2
73. (A) ˙·˙
22 2
7
× r = 9856
22
67. (C) ˙·˙ 2 × × r = 88
7

(
⇒ r = 88 ×
7 1
22 2
× )
= 14 m 56 m

∴ Area =
22
7( )
× 14 × 14 m2 = 616 m2

68. (C) Original circumference = 2πr Then r2 = (9856 ×227 ) = 3136


New circumference =
150
100 (
× 2πr = 3πr ) ∴ r = 56 m
74. (B) Let circumference = 100 cm.
˙·˙ 2πR = 3πr Then, ˙·˙ 2πr = 100
3r 100 50
⇒ R = ⇒ r = =
2 2π π
Original area = πr2 New circumference
9r2 9πr2 = 105 cm.
New area = πR2 = π × =
4 4 Then, 2π R = 105

( 9πr2
)5πr2
105
Increase in area = – πr2 = ⇒ R =
4 4 2π

Quantitative Aptitude Test | 28


More PDF download : http://www.gksolve.com

⇒ Original area = (π × 50π ×50π ) = 2500


π
cm 2 Required area = (Area of circle) – (Area of rectangle)

New area = (π ×
= ( 22
7 )
× 5 × 5 – 8 × 6 m2 =
214 2
7
m = 30·6 m2

2π 2π )
105 105
⇒ ×
82. (C) ˙·˙ Side of the square = 2r
11025
= cm2

⇒ Increase in area
= (11025


π )
2500
cm 2 ∴ Perimeter = (4 × 2r) = 8r
22 2
83. (B) ˙·˙ × r = 462
1025 7
= cm2

Required increase per cent ⇒ (
r2 = 462 ×
7
22 )
= 147
π
= ( 1025

×
2500 )
× 100 % = %
41
4
⇒ r = 7√
⎯ 3 cm
= 10·25% ∴ Height of the triangle = 3r = 21 √
⎯ 3 cm
a 2
75. (B) ˙·˙ 2πR – 2π r = (176 – 132)
Now, ˙·˙ a2 = + (3r)2
⇒ 2π (R – r) = 44 4
3a2
⇒ (R – r) =
44 × 7 ⇒
4
= 21 ⎯ (√3 2 )
2 × 22

76. (D) ˙·˙ π d – d = 105 cm


= 7m ⇒ a2 = 1323 ×( 4
3 )
⇒ a = 21 × 2 = 42 cm
⇒ (π – 1) d = 105 cm ∴ Perimeter = 3a = 3 × 42
⇒ ( 22
7 )
– 1 d = 105 cm
1
= 126 cm
⎯ 3 × 24 × 24

84. (C) ˙·˙ × 24 × h =
∴ d =
7
15( )
× 105 cm = 49 cm

2 4
h = 12√
⎯3
77. (C) Area of the sector =
22
7 (
× 12 × 12 ×
42°
360°
m2 ) ⇒ 3r = 12√
⎯3
= 52·8 m2 ∴ r = 4√
⎯ 3 cm
78. (D) Angle swept in 30 min. = 180°

Area swept = (
22
7
×7×7×
180°
360°
cm2) h = 3r
= 77 cm2

79. (D) Arc length =


2πrθ°
360°
= 2× ( 22
7
× 21 ×
72°
360
cm ) Area of the incircle = π × (4√ ⎯ 3)2 cm2
= 48 π cm2
= 26·4 cm
85. (A) ˙·˙ Area of circular lawn = (10000 – 8614) m2
80. (C) Area of sector = (
1
2
× arc length × radius cm2 ) 22 2
= 1386 m2
˙·˙ × r = 1386 m2
= (
1
2
× 3·5 × 5 ) 7

= 8·75 cm2.
⇒ (
r2 = 1386 ×
7
22 )
81. (C) ˙·˙ Diagonal of the rectangle = (63 × 7)
= √
⎯⎯⎯⎯⎯
82 + 6 2 ∴ r = 21 m
= 10 m 86. (B) Let the longer side = l, shorter side = b
and Diagonal = d
8
Then, l·b = 60 …(1)
and d + l = 5b
6
⇒ d = 5b – l …(2)
⇒ 2 2 2
d = 25b + l – 10 l·b
⇒ Radius = 5 m ⇒ l + b2 = 25b2 + l2 – 10 × 60
2

Quantitative Aptitude Test | 29


More PDF download : http://www.gksolve.com

⇒ 24b2 = 600 92. (C) Ratio of areas of similar triangles


∴ b = ⎯⎯25 = 5 m
√ = Ratio of the squares of corresponding sides
60 60 (3x)2 9x2 9
∴ l = b = = 12 m. = = = = 9 : 16
5 (4x)2 16x2 16
87. (C) Let length = a metres and breadth = b metres 93. (B) Area of a triangle formed by joining the mid
Then, 2 (a + b) = 46 1
points of the sides of the triangle is th of area of the
4
⇒ (a + b) = 23 and ab = 120 original triangle.
∴ Diagonal = ⎯√⎯⎯⎯⎯
a2 + b2 = ⎯√⎯⎯⎯⎯⎯⎯⎯⎯⎯
(a + b)2 – 2ab 1 1
94. (B) ˙·˙ × 10 × 4·4 = × 11 × h
= √⎯⎯⎯⎯⎯⎯⎯⎯⎯⎯
(23) 2 – 2 × 120 2 2
10 × 4·4
= √⎯⎯⎯
289 = 17 m ⇒h= = 4 cm
11
a2 3a2 2
88. (C) ˙·˙ a2 = + x2 ⇒ =x 1
4 4 95. (D) Δ AOB = AB × OE
4 2
∴ a2 = x2 1
3 = × 2CD × OE = CD × OE
2
E
A B

D F C
⎯ 3 a2 = ⎯√ 3 × 4 x2

∴ Area = 1
4 4 3 Δ COD = × CD × OF
2
⎯3
√ x2
= Δ AOB CD × OE CD × 2 × OF
3 ∴ = =
Δ COD 1 1
89. (B) Area of the room = (544 × 374) cm2 × CD × OF × CD × OF
2 2
Size of largest square tile = H. C. F. of 544 & 374
= 34 cm 4
= =4:1
Area of 1 tile = (34 × 34) cm2 1
∴ Least number of tiles required 1
96. (B) × (10 + 6) × d = 640
544 × 374 2
= = 176
34 × 34 640 × 2
⇒ d = = 80 m
22 2 16
90. (A) ˙·˙ × r = 616
7 1
97. (B) ˙·˙ (3x + 5x) × 12 = 384
⇒ (
r2 = 616 ×
7
22 )
= 196

2
8x = 64
∴ r = ⎯⎯⎯
√ 196 = 14 cm ∴ x = 8
∴ Smaller side = 3x = 3 × 8 = 24 cm.

98. (C) Area of trapezium = [12 (1 + 2) × 6] m = 9 m


2 2

So diameter = 28 cm 99. (C) AB = 60 m, BC = 40 m and AC = 80 m


∴ Area of square = (28 × 28) cm2 1
= 784 cm2 ∴ s = (60 + 40 + 80) m = 90 m
2
91. (A) Speed = 12 × ( )
5
18
m/sec (s – a) = 30 m, (s – b) = 50 m and (s – c) = 10 m
∴ Area of Δ ABC = ⎯√⎯⎯⎯⎯⎯⎯⎯⎯⎯⎯⎯⎯⎯⎯⎯
[s (s – a) (s – b) (s – c)]
=
10
3 ( )
m/sec
= √ ⎯⎯⎯⎯⎯⎯⎯⎯⎯⎯⎯⎯
90 × 30 × 50 × 10 m2
∴ Perimeter =
10
3 ( )
× 15 × 60 m = 3000 m = 300√ ⎯⎯15 m2
˙·˙ 2 × (x + 4x) = 3000 m ∴ Area of | | gm ABCD = 600√⎯⎯15 m2
⇒ x = 300 m 100. (A) AB = 6 cm; AD = 4 cm and ∠ BAD = 30°
So, length = 1200 m and breadth = 300 m Area of | | gm ABCD = AB × AD × sin 30°
∴ Area = (1200 × 300) m2 = 360000 m2 = 6 × 4 × sin 30° = 12cm2 .

Quantitative Aptitude Test | 30


More PDF download : http://www.gksolve.com

101. (B) Area of | | gm = (14 × 16) cm 2 = 224 cm2 ⇒ x = 12


102. (B) Let ABCD be the square and ABEF be the ∴ Length of diagonals = 12 cm, 24 cm
rhombus.
Then, in right triangles ADF and BCE,
we have AD = BC (sides of a square)
105. (B) Side of the rhombus =
52
4
m = 13 m ( )
and AF = BE (sides of rhombus) AB = 13 m, AC = 24 m.
∴ DF = CE So, AO = 12 m
F C D
D E
C

O
A B
A
[˙.˙ DF2 = AF2 – AD2 and BE2 – BC2 = CE2]
Thus, Δ ADF = Δ BCE B
⇒ Δ ADE + trap ABCF ∴ OB = √ ⎯⎯⎯⎯⎯⎯⎯⎯
AB2 – AO2 = ⎯ √⎯⎯⎯⎯⎯⎯
169 – 144 = 5m
= Δ BCE + trap ABCF
So, BD = 2 × OB = 10 m
∴ Area of sq. ABCD = Area of rhombus ABEF
Hence, other diagonal = 10 m
103. (B) Let length of the longer diagonal = x cm
106. (B) AB = 10 cm, OB = 6 cm, ∠ AOB = 90°
Then, length of other diagonal
∴ OA = ⎯ √⎯⎯⎯⎯⎯⎯⎯
(10) 2 – (6) 2= ⎯
√⎯64 = 8 cm
= ( )80
100
x cm =( )4
5
x cm D

1 4 2 C
Area of rhombus = x × x = x2
2 5 5
O
2
=
5 A
× (Square of the length of longer diagonal) B
1 ˙·˙ AC = 2 × OA = 16 cm and BD = 12 cm
104. (B) ˙·˙ × x × 2x = 144


2
2
x = 144
∴ Area of rhombus = ( 1
2 )
× 16 × 12 cm2 = 96 cm2

Quantitative Aptitude Test | 31


More PDF download : http://www.gksolve.com

Numbers
Numbers—In Hindu Arabic 4. 8888 + 7777 + 666 + ? = 19000 14. The face value of 8 in the
System, we use ten symbols 0, 1, 2, (A) 2719 numeral 458926 is—
3, 4, 5, 6, 7, 8, 9, called digits to
(B) 2679 (A) 8000 (B) 8
represents any number.
(C) 1669 (C) 1000 (D) 458000
Numeral—A group of figures,
representing a number is called a (D) None of these 15. The local value of 7 in the
numeral. numeral 5679032 is—
5. 4003 – ? – 1599 = 716
Representation of a number in (A) 7 (B) 70000
figure is called notation and expre- (A) 1598
(B) 1688 (C) 10000 (D) 5070000
ssing a number in words is called
numeration. (C) 3120 16. ? × 48 = 173 × 240
We use place value system to (D) None of these (A) 545 (B) 685
represent a number. For a given
6. 36000 – 888 = 37000 – ? (C) 865 (D) 495
numeral, we start from the extreme
right as Unit’s place, Ten’s place, (A) 2018 (B) 1848 17. 106 × 106 + 94 × 94 = ?
Hundred place etc. (C) 1888 (D) 1978 (A) 21032 (B) 20032
(C) 23032 (D) 20072
Examples 7. ? – 1046 – 398 – 69 = 999
(A) 2502 (B) 2512 18. 5358 × 51 = ?
Q. 1. ? – 1398 = 2133
(C) 2472 (D) 514 (A) 273358 (B) 273258
Solution : Let x – 1398 = 2133 (C) 273348 (D) 273268
8. ? + 6207 – 9038 = 107
Then x = (2133 + 1398) = 3531
(A) 2724 19. 1307 × 1307 = ?
Q. 2. 7048 – ? = 3999
(B) 2938 (A) 1601249 (B) 1607249
Solution : Let 7048 – x = 3999
(C) 2814 (C) 1701249 (D) 1708249
Then, x = 7048 – 3999 = 3049.
(D) None of these 20. 1299 × 1299 = ?
Q. 3. 9873 + ? = 13200
9. 3571 + ? – 6086 = 115 (A) 1585301 (B) 1684701
Solution : Let 9873 + x = 13200
(C) 1685401 (D) 1687401
Then, x = (13200 – 9873) = 3327 (A) 2400
(B) 2630 21. 1014 × 986 = ?
Q. 4. 78341 – (567 + 2356 +
53172 + 10741) = ? (C) 2515 (A) 998904 (B) 999804
Solution : (C) 998814 (D) 998804
(D) None of these
78341 22. 387 × 387 + 114 × 114 + 2 × 387
10. 36800 – 9999 – 3333 – 66 = ?
567 ⎫⎪ × 114 = ?
⎪ (A) 30200 (B) 27912
2356 (A) 250001 (B) 251001
⎬⎪ (C) 23402 (D) 24102
53172 (C) 260101 (D) 261001

10741 = ⎭ 66836 11. ? – 4799 = 1714 23. 469157 × 9999 = ?
? = 78341 – 66836 (A) 6513 (A) 4586970843
= 11505 (B) 4686970743
(B) 7403
(C) 4691100843
Exercise (C) 3085
(D) 4586870843
(D) None of these
1. 9856 × 156 + 9856 × 844 = 24. 2 ? 63 ÷ 11 = 233
(A) 9856000 (B) 9836500 12. 9501 – ? = 3697 (A) 4 (B) 3
(C) 9794560 (D) 9698350 (A) 13198 (B) 5814 (C) 6 (D) 5
2. 6674 × 625 = ? (C) 5804 (D) 4894 25. 2013 × ?…… 1 = 62403
(A) 4170160 (B) 4072360 13. 66066 + 6606 + 66 + 6 = ? (A) 1 (B) 2
(C) 4171050 (D) 4171250 (A) 258126 (C) 3 (D) 4
3. 9998 × 999 = ? (B) 72744 26. 777777 ÷ 11 = ?
(A) 9997001 (B) 9988002 (C) 72798 (A) 7077 (B) 70707
(C) 9987012 (D) 9898012 (D) None of these (C) 7707 (D) 7007

Quantitative Aptitude Test | 32


More PDF download : http://www.gksolve.com

27. ? × 11 = 555555
(A) 505 (B) 5050
37. (1 – 13) (1 – 14) (1 – 15) 47. Which of the following numbers
is exactly divisible by 99 ?
(A) 3572404 (B) 135792
… (1 – n) = ?
(C) 50505 (D) 5005 1
(C) 913464 (D) 114345
28. 87 × ? = 3393
1 2 48. What least value must be given
(A) 39 (B) 49 (A) n (B) n
to * so that the number 91876
(C) 27 (D) 67 2 (n – 1) 2 * 2 is divisible by 8 ?
(C) (D) (A) 1 (B) 2
29. ? × 147 = 6909 n n (x + 1)
(C) 3 (D) 4
(A) 37 (B) 47 137 × 137 + 137 × 133
(C) 27 (D) 67 + 133 × 133 49. What least value must be given
38. =? to * so that the number 97215
137 × 137 × 137 – 133
30. 356 × 936 – 356 × 836 = ? × 133 × 133 * 6 is divisible by 11 ?
(A) 35600 (B) 34500 (A) 4 (B) 270 (A) 1 (B) 2
(C) 49630 (D) 93600 1 1 (C) 3 (D) 5
(C) (D)
4 270 50. What least value must be given
24 ?
31. = 117 × 117 × 117 – 98 to * so that the number 6135 * 2
18 6
× 98 × 98 is exactly divisible by 9 ?
(A) 12 (B) 10 39. =?
117 × 117 + 117 × 98 (A) 0 (B) 1
(C) 9 (D) 8 + 98 × 98 (C) 2 (D) 3
(A) 215 (B) 311 51. What least number must be added
⎯⎯
√ ? 72
32. = (C) 19 (D) 29 to 1056 to get a number exactly
196 56
divisible by 23 ?
(A) 18 (B) 14 343 × 343 × 343 + 257
× 257 × 257 (A) 21 (B) 25
(C) 324 (D) 212 40. =? (C) 3 (D) 2
343 × 343 – 343 × 257
x 4 + 257 + 257 52. What least number must be
33. If y = , then the value of
5 (A) 8600 (B) 800 subtracted from 13601 to get a
⎛ 4 2y – x⎞ (C) 600 (D) 2600 number exactly divisible by 87 ?
⎜ 7 + 2y + x⎟ is— (A) 49 (B) 23
⎝ ⎠ 41. 42060 ÷ 15 + 5 = ?
(A) 2804 (B) 2809 (C) 29 (D) 31
3 1
(A) (B) 1 (C) 2103 (D) 289 53. The least number of five digits
7 7
? 96 exactly divisible by 456 is—
(C) 1 (D) 2 42. =
54 ? (A) 10142 (B) 10232
34. If ⎯
√⎯3n = 81, then n = ? (A) 72 (B) 27 (C) 10032 (D) 10012
(C) 36 (D) 63
(A) 2 (B) 4 54. The largest number of four digits
(C) 6 (D) 8 ⎯?=4
√ exactly divisible by 88 is—
43. (A) 9768 (B) 8888
19
a 4 3a + 2b (A) 76 (B) 5776 (C) 9988 (D) 9944
35. If b = , then =?
3 3a – 2b (C) 304 (D) 1296 392
55. = 28
(A) 6 (B) 3
44.
4050
= 450 ⎯?

(C) 5 (D) – 1 ⎯?
√ (A) 144 (B) 196
(A) 49 (B) 100 (C) 24 (D) 48
36. When simplified, the product
(C) 81 (D) 9 56. The number (10n – 1) is divisible
( )( )( )
2–
1
3
2–
3
5
2–
5
7 45.
80 ?
=
by 11 for—
(A) All values of n
? 20
( )
… 2–
997
999
is equal to— (A) 40
(C) 800
(B) 400
(D) 1600
(B) Odd values of n
(C) Even values of n
5 (D) n = multiples of 11
(A) 46. Which number should replace
999

(B)
1001 both the asterisks in
*
21 ( ) 57. Which of the following numbers
is prime ?
999

(C)
1001 × ( )
*
189
=1?
(A) 119
(B) 187
3 (A) 21 (B) 63 (C) 247
(D) None of these (C) 3969 (D) 147 (D) None of these

Quantitative Aptitude Test | 33


More PDF download : http://www.gksolve.com

58. The sum of first four prime 5 1 67. When a certain number is multi-
(A) (B) 1
numbers is— 7 7 plied by 13, the product consists
(A) 10 (B) 11 (C) 1 (D) 2 entirely of fives. The smallest
(C) 16 (D) 17 such number is—
64. The largest natural number by (A) 41625 (B) 42515
59. The first prime number is— which the product of three con-
(C) 42735 (D) 42135
(A) 0 (B) 1 secutive even natural numbers is
always divisible, is— 68. What least number must be
(C) 2 (D) 3
(A) 16 (B) 24 subtracted from 1294 so that the
10 110 remainder when divided by 9,
60. = (C) 48 (D) 96
11 ? 11, 13 will leave in each case the
(A) 111 (B) 1100 65. If a = 16 and b = 5, the value of same remainder 6 ?
(C) 121 (D) 100 ⎛ a2 + b2 + ab⎞ (A) 0 (B) 1
⎜ a3 – b3 ⎟ is— (C) 2 (D) 3
⎝ ⎠

⎯⎯⎯⎯
( ) = (1 + ),
27 x 69. In a division sum, the divisor is
61. If 1+ 1
169 13 (A) 12 times the quotient and 5 times
11
then the value of x is— the remainder. If the remainder
1 be 48 then the dividend is—
(A) 1 (B) 3 (B)
19 (A) 240 (B) 576
(C) 5 (D) 7 121
(C) (C) 4800 (D) 4848
62. 62976 ÷ ? = 123 3971
(D) None of these 70. If in a long division sum, the
(A) 412 (B) 502 dividend is 380606 and the
(C) 512 (D) 522 66. 9873 + x = 13200, then x is— successive remainders from the
x 3 (A) 3327 first to the last are 434, 125 and
63. If y = , then the value of (B) 3237 413, then the divisor is—
4
(C) 3337 (A) 451 (B) 843
(6 y–x
+
7 y+x ) equals— (D) None of these (C) 4215 (D) 3372

Answers with Hints


1. (A) Exp. = 9856 × 156 + 9856 × 844 9. (B) Let 3571 + x – 6086 = 115
= 9856 × (156 + 844) Then x = (6086 + 115) – 3571
= 9856 × 1000 = 9856000 = (6201 – 3571) = 2630
2. (D) Exp. = 6674 × 625
10. (C) 9999 36800
6674 × (10)4
= 6674 × (5)4 = 3333 – 13398
24
66740000 + 66 23402
= = 4171250
16 13398
3. (B) Exp. = 9998 × 999
11. (A) Let x – 4799 = 1714
= 9998 × (1000 – 1)
Then x = 4799 + 1714 = 6513.
= 9998 × 1000 – 9998 × 1
= 9998000 – 9998 = 9988002. 12. (C) Let 9501 – x = 3697
4. (C) Let 8888 + 7777 + 666 + x = 19000 Then x = 9501 – 3697
Then 17331 + x = 19000 = 5804
or x = (19000 – 17331) = 1669 13. (B) 66066
6606
5. (B) Let 4003 – x – 1599 = 716
66
Then x = 4003 – 1599 – 716
+6
= 4003 – (1599 + 716)
= (4003 – 2315) = 1688 72744
6. (C) Let 37000 – x = 36000 – 888 14. (B) The face value of 8 in 458926 is 8.
Then x = 37000 – 36000 + 888 15. (B) The local value of 7 in 5679032 is 70000.
= 1888 16. (C) Let x × 48 = 173 × 240
7. (B) Let x – 1046 – 398 – 69 = 999 173 × 240
Then x = = (173 × 5)
Then x = 999 + 1046 + 398 + 69 48
= 2512. = 865.
8. (B) Let x + 6207 – 9038 = 107 17. (D) 106 × 106 + 94 × 94 = (106)2 + (94)2
Then x = (107 + 9038) – (6207) 1 1
= [2 (a2 + b2 )] = [(a + b)2 + (a – b)2]
= 2938 2 2

Quantitative Aptitude Test | 34


More PDF download : http://www.gksolve.com

1 24 x
= [(106 + 94)2 + (106 – 94)2] 31. (D) Let =
2 18 6
1 Then 18x = 24 × 6
= [(200)2 + (12)2] 24 × 6
2 ∴ x = = 8.
1 40144 18
= × [40000 + 144] = = 20072.
⎯⎯

2 2 x 72 9
32. (C) Let = =
18. (B) 5358 × 51 = 5358 × (50 + 1) 196 56 7
x 9 9 81
= 5358 × 50 + 5358 × 1 Then = × =
196 7 7 49
= 267900 + 5358 = 273258.
81 × 196
19. (D) 1307 × 1307 = (1307) 2 So, x = = 324.
49
= (1300 + 7)2 33. (C) Dividing numerator as well as denominator by y,
= (1300) 2 + (7)2 + 2 × 1300 × 7 we get
= 1690000 + 49 + 18200 4 2y – x
Given Exp. = +
= 1708249. 7 2y + x
20. (D) 1299 × 1299 = (1299) 2 x 4
2 –y 2–
= (1300 – 1) 2 4 4 5
= +
= (1300) 2 +(1)2 – 2 × 1300 × 1 7 x= 7+ 4
2+y 2+
= 1690000 + 1 – 2600 = 1687401. 5
4 6 4 3 7
21. (B) 1014 × 986 = (1000 + 14) × (1000 – 14) = + = + = = 1.
7 14 7 7 7
= (1000) 2 – (14) 2
= (1000000 – 196) = 999804. 34. (D) ˙·˙ ⎯⎯3 = 81
√ n

⇒ 3n/2 = 34
22. (B) Given expression n
= a2 + b2 + 2ab ⇒ = 4
2
= (a + b)2 = (387 + 114)2 = (501)2 ⇒ n = 8.
= (500 + 1)2 35. (B) Dividing numerator as well as denominator by b,
= (500) 2 + (1)2 + 2 × 500 × 1 we get
a
= 250000 + 1 + 1000 = 251001.
3a + 2b 3 b + 2
23. (C) 469157 × 9999 = 469157 × (10000 – 1) Given Exp. = = a
3a – 2b
= 4691570000 – 469157 3b–2
= 4691100843. 4
x 3× +2
24. (D) Let = 233 3 4+2
11 = = =3
4 4–2
3× –2
Then x = 233 × 11 = 2563 3
∴ Missing digit is 5. 36. (C) Given Exp.
25. (C) Let 2013 × x = 62403.
62403
( )( )( ) (
= 2–
1
3
2–
3
5
2–
5
7
… 2– )
997
999
Then x = = 31 5 7 9 1001 1001
2013 = × × × …… = ·
3 5 7 999 3
∴ Missing digit is 3. 37. (B) Given Exp.
26. (B) ?=
777777
11
= 70707. ( )( )( ) ( )
= 1–
1
3
1–
1
4
1–
1
5
… 1 –n
1

27. (C) Let x × 11 = 555555 2 3 4 n–1 2


= × × ×…× n =n·
555555 3 4 5
Then x = = 50505. a2 + ab + b2
11 38. (C) Given expression = ,
a3 – b3
28. (A) Let 87 × x = 3393
where a = 137, b = 133
3393 (a2 + ab + b2)
Then x = = 39.
87 =
(a – b) (a2 + ab + b2)
29. (B) Let x × 147 = 6909. 1 1 1
= a–b = =
6909 137 – 133 4
Then, x = = 47.
147 a3 – b3
39. (C) Given expression = 2 ,
30. (A) 356 × 936 – 356 × 836 a + ab + b2
= 356 × (936 – 836) (a – b) (a2 + ab + b2)
where a = 117, b = 98 =
= 356 × 100 = 35600. (a2 + ab + b2)

Quantitative Aptitude Test | 35


More PDF download : http://www.gksolve.com

= (a – b) 392
55. (B) Let = 28
= (117 – 98) = 19. ⎯x

a3 + b3 392
40. (C) Given expression = 2 , Then ⎯ x = 28 = 14

a – ab + b2
(a + b) (a2 – ab + b2 ) ⇒ x = 142 = 196.
where a = 343, b = 257 =
(a2 – ab + b2 ) 56. (C) For even values of n, the number (10n – 1) con-
= (a + b) sists of even numbers of nines and hence it will be
divisible by 11.
= (343 + 257) = 600. 57. (D) 119 is divisible by 7, 187 is divisible by 11, 247
42060 is divisible by 13 and 551 is divisible by 19. So none
41. (B) Given expression = +5
15 of the given numbers is prime.
= 2804 + 5 = 2809. 58. (D) First four prime numbers are 2, 3, 5, 7,
x 96 Their sum = (2 + 3 + 5 + 7) = 17.
42. (A) Let = x , 59. (C) The first prime number is 2.
54 10 110
Then x = 54 × 96
2 60. (C) Let = x ,
11
∴ x = √⎯⎯⎯⎯⎯
54 × 96 = ⎯
√⎯⎯⎯⎯⎯⎯⎯⎯
6 × 9 × 6 × 16 Then 10x = 11 × 110
= 6 × 3 × 4 = 72. 11 × 110
∴ x = = 121.
⎯x = 4
√ 10
43. (B) Let
⎯⎯

19 196 x
61. (A) = 1+
169 13
Then ⎯ x = 19 × 4 = 76
√ x
14
∴ x = 76 × 76 = 5776. ⇒ –1 =
13 13
4050 1 x
44. (C) Let = 450 ⇒ = i.e. x = 1.
⎯x
√ 13 13
4050 62976
Then ⎯ x = 450 = 9
√ 62. (C) Let x = 123
∴ x = (9 × 9) = 81. 62976
80 x Then x = = 512.
45. (A) Let 123
x = 20 6 y–x
Then x2 = 80 × 20 = 1600 63. (C) Given Exp. = +
7 y+x
∴ x = √
⎯⎯⎯⎯
1600 = 40. x
1 –y 1–
3
x x 6 6 4
= +
46. (B) Let × = 1 7 x= 7+ 3
21 189 1+y 1+
Then, x2 = 21 × 189 4
6 1
= 21 × 21 × 9 = + = 1.
∴ x = (21 × 3) = 63. 7 7
64. (C) It is 2 × 4 × 6 = 48.
47. (D) Clearly 114345 is divisible by 9 as well as 11.
So, it is divisible by 99. ⎛ a2 + b2 + ab⎞
65. (A) Given Exp. = ⎜ ⎟
48. (C) By hit and trial we find that 632 is divisible by 8. ⎝ a3 – b3 ⎠
So, * must replaced by 3. 1 1 1
= a – b= = .
49. (C) (9 + 2 + 5 + 6) – (7 + 1 + x) = 14 – x (16 – 5) 11
must be divisible by 11. So x = 3. 66. (A)
67. (C) By trial, we find that the smallest number consis-
50. (B) 6 + 1 + 3 + 5 + x + 2 = 17 + x must be divisible ting entirely of fives and exactly divisible by 13 is
by 9. So x = 1. 555555. On dividing 555555 by 13, we get 42735 as
51. (D) On dividing 1056 by 23, we get 21 as remainder quotient.
∴ Required number to be added = (23 – 21) = 2. ∴ Req. smallest number = 42735.
68. (B) The number when divided by 9, 11, 13 leaving
52. (C) 53. (C) remainder 6 = (l.c.m. of 9, 11, 13) + 6 = 1293.
54. (D) Largest number of four digits = 9999 Required least number = 1294 – 1293 = 1.
88) 9999 (113 69. (D) Let quotient = Q and remainder = R
88 Then, Divisor = 12 Q = 5 R
119 Now, R = 48
⇒ 12 Q = 5 × 48
88
⇒ Q = 20
319 Dividend = Divisor × Quotient + Remainder
264 ∴ Dividend = (20 × 240 + 48)
55 = 4848
∴ Required number = (9999 – 55) = 9944. 70. (B)

Quantitative Aptitude Test | 36


More PDF download : http://www.gksolve.com

Problems Based on Numbers


To Solve the questions based on Solution : 40
˙·˙ x × +y = 1·2 y
numbers, the number is written in Let the number be x 100
algebraic form. For example let the
2 3 1 2
number be x. ˙·˙ of of of x = 24 ⇒ ×x = 1·2 y – y
3 5 4 5
The numerator of a fraction is
⇒ x = 24 × 10 ⇒ 2x = 0·2y × 5 = 1
written in the following form—
= 240 x 1
Three-fourth of a number ∴ =
240 × 30 y 2
3 3x ∴ 30% of 240 =
= x× = 100 or x:y = 1:2
4 4
= 72 Q. 5. The difference between
Two-third of a number
Q. 2. The difference between the middle number of three conse-
2 2x two, two digit numbers is one-third cutive odd numbers and the middle
= x× =
3 3 of one of the numbers. What is number of three consecutive even
definite the sum of these two numbers is 7. What will be the
Double of a number
numbers ? difference between the total of these
= 2x odd numbers and the total of those
Solution : three even numbers ?
Thrice of a number Its answer cannot be found Solution :
= 3x because it is not definite that diffe-
1 Let three consecutive odd num-
20% of a number rence of two number is of the first bers be x, x + 2 and x + 4 and let
3
x × 20 x or second. Hence data is inadequate. three consecutive even numbers be y,
= = y + 2 and y + 4
100 5 Q. 3. The ratio between two
and 120% of number digit number and the sum of these ˙·˙ (x + 2) – (y + 2) = 7
digits is 4 : 1. If the unit digit is ⇒ x–y = 7
120x 6x
= = more than tens digit by 3, what is
100 5 the number ? ∴ Sum of three consecutive odd
numbers.
3 Solution :
30% of of one-third of a = x+x+2+x+4
4
Let the number be (10x + y)
1 3 30 = 3x + 6
number = x× × × 10x + y
3 4 100 4
˙·˙ = and sum of three consecutive
Three consecutive odd numbers (x + y) 1 even numbers.
are x, x + 2 and x + 4 or x – 2, x and ⇒ 10x + y = 4x + 4y = y +y + 2 + y + 4
x + 2, where x is an odd number. ⇒ 6x – 3y = 0 = 3y + 6
Similarly three consecutive even ⇒ 2x – y = 0
numbers are x, (x + 2) and (x + 4) or ∴ Reqd. difference
⇒ y = 2x …(i)
x – 2, x and (x + 2) where x is an even = (3x + 6) – (3y + 6)
number. But y = x+3
= 3x – 3y
⇒ 2x = x + 3
Main Formulae ∴ x = 3 = 3 (x – y)
(i) (x – y) 2 = (x + y)2 – 4xy and y = 2×3=6 = 3 × 7 = 21
(ii) (x + y)2 = (x – y) 2 + 4xy ∴Req. Number
Short Cut Method
(iii) (x + y)2 = x2 + 2xy + y2 = 10x + y
Let three consecutive odd numbers
(iv) (x – y) 2 = x2 – 2xy + y2 = 36. be y – 2, y and y + 2 respectively.
and Q. 4. When 40% of first num- ∴ Total of these odd numbers
1 ber is added to the second number, = 3y
(v) xy = [(x + y 2 ) – (x – y)2] the second number becomes 1·2
4 Let three consecutive even numbers
times of itself. What is the ratio be x – 2, x and x + 2 respectively
between the first and second
Examples numbers ? ∴ Total of these even numbers
Q. 1. Two-third of three-fifth = 3x
Solution :
of one-fourth of a number is 24. ˙·˙ x–y = 7
What is 30 per cent of that Let the first and second numbers ∴ 3x – 3y = 21
number ? be x and y respectively.

Quantitative Aptitude Test | 37


More PDF download : http://www.gksolve.com

Exercise 10. The ratio between two numbers 18. A number whose fifth part
is 3 : 4 and their sum is 420. The increased by 5 is equal to its
1. 11 times a number gives 132. The
greater of the two numbers is— fourth part diminished by 5 is—
number is—
(A) 175 (B) 200 (A) 160 (B) 180
(A) 11
(C) 240 (D) 315 (C) 200 (D) 220
(B) 12
11. The sum of two numbers is 100 19. The product of two numbers is
(C) 13·2
and their difference is 37. The 120. The sum of their squares is
(D) None of these
difference of their squares is— 289. The sum of the two num-
2. A number is as much greater bers is—
(A) 37 (B) 100
than 31 as is less than 55. The (A) 20
number is— (C) 63 (D) 3700 (B) 23
(A) 47 (B) 52 12. The difference of two numbers is (C) 169
(C) 39 (D) 43 1 (D) None of these
8 and th of their sum is 35. The
8
3 20. The sum of squares of two
3. of a number is 19 less than the numbers are—
4 numbers is 80 and the square of
original number. The number (A) 132, 140 (B) 128, 136 their difference is 36. The pro-
is— (C) 124, 132 (D) 136, 144 duct of the two numbers is—
(A) 84 (B) 64 (A) 22 (B) 44
13. The sum of two numbers is 29
(C) 76 (D) 72 and the difference of their (C) 58 (D) 116
1 1 squares is 145. The difference 21. The sum of two numbers is twice
4. of a number subtracted from
4 3 between the numbers is— their difference. If one of the
of the same number gives 12. (A) 13 (B) 5 numbers is 10, the other number
The number is— is—
(C) 8 (D) 11
(A) 144 (B) 120 1
(A) 3 (B) 30
(C) 72 (D) 63 14. The difference of two numbers is 3
5 and the difference of their 1 1
5. If one-fifth of a number decrea- (C) (D) 4
squares is 135. The sum of the 3 4
sed by 5 is 5, then the number numbers is—
is— 22. The sum of two numbers is 15
(A) 27 (B) 25 and sum of their square is 113.
(A) 25 (B) 50
(C) 30 (D) 32 The numbers are—
(C) 60 (D) 75 (A) 4, 11 (B) 5, 10
15. If 1 is added to the denominator
6. 24 is divided into two parts such of fraction, the fraction becomes (C) 6, 9 (D) 7, 8
that 7 times the first part added 1 23. A positive number when decrea-
. If 1 is added to the numerator,
to 5 times the second part makes 2 sed by 4, is equal to 21 times the
146. The first part is— the fraction becomes 1. The frac- reciprocal of the number. The
(A) 11 (B) 13 tion is— number is—
4 5 (A) 3 (B) 5
(C) 16 (D) 17 (A) (B)
7 9 (C) 7 (D) 9
7. A number is 25 more than its 2 10
(C) (D) 4
two-fifth. The number is— 3 11 24. of a certain number is 64. Half
5
(A) 60 (B) 80 of that number is—
4 2
125 125 16. of a number exceeds its by 8.
(C) (D) 5 3 (A) 32 (B) 40
3 7 The number is— (C) 80 (D) 16
8. Three fourth of one-fifth of a (A) 30 25. Three-fourth of number is more
number is 60. The number is— (B) 60 than two-third of the number by
(A) 300 (B) 400 5. The number is—
(C) 90
(A) 72 (B) 60
(C) 450 (D) 1200 (D) None of these
(C) 84 (D) 48
9. The difference between the squa- 17. If one-fourth of one-third of one- 26. The number 62n – 1, where n is
res of two consecutive number half of a number is 15, the any positive integer, is always
is 35. The numbers are— number is— divisible by—
(A) 14, 15 (B) 15, 16 (A) 72 (B) 120 (A) 11
(C) 17, 18 (D) 18, 19 (C) 180 (D) 360 (B) 5

Quantitative Aptitude Test | 38


More PDF download : http://www.gksolve.com

(C) 7 36. The sum of three numbers is 68. 43. A number when divided by 6 is
(D) Both (B) and (C) If the ratio between first and diminished by 40. The number
27. Which of the following numbers second be 2 : 3 and that between is—
is not a square of any natural second and third be 5 : 3, then (A) 72 (B) 84
number ? the second number is— (C) 60 (D) 48
(A) 34692 (B) 4096 (A) 30 (B) 20
44. The sum of seven numbers is
(C) 15129 (D) 15376 (C) 58 (D) 48 235. The average of the first
28. Of the three numbers, the first is 37. Three numbers are in the ratio three is 23 and that of the last
twice the second and is half of 3 : 4 : 5. The sum of the largest three is 42. The fourth number
the third. If the average of three and the smallest equals the sum is—
numbers is 56. The smallest of the third and 52. The smallest (A) 40 (B) 126
number is— number is— (C) 69 (D) 195
(A) 24 (B) 36
(A) 20 (B) 27
(C) 40 (D) 48 45. Divide 50 into two parts so that
(C) 39 (D) 52 the sum of their reciprocals is
29. If 16% of 40% of a number is 8,
the number is— 38. The sum of three consecutive (1/12)—
(A) 200 (B) 225 odd numbers is 57. The middle (A) 20, 30 (B) 24, 26
(C) 125 (D) 320 one is— (C) 28, 22 (D) 36, 14
30. A number exceeds 20% itself by (A) 19 (B) 21 46. Two numbers are such that the
40. The number is— (C) 23 (D) 17 ratio between them is 3 : 5, but if
(A) 50 (B) 60 each is increased by 10, the ratio
39. A certain number of two digits is
(C) 80 (D) 320 between them becomes 5 : 7.
three times the sum of its digits
31. The number x is exactly divisible The numbers are—
and if 45 be added to it, then
by 5 and the remainder obtained (A) 3, 5 (B) 7, 9
digits are reversed. The number
on dividing the number y by 5 is is— (C) 13, 22 (D) 15, 25
1. What remainder will be
(A) 32 (B) 72 47. The ratio between two numbers
obtained when (x + y) is divided
(C) 27 (D) 23 is 2 : 3. If the consequent is 24,
by 5 ?
the antecedent is—
(A) 0 (B) 1 40. If 3 is added to the denominator (A) 36 (B) 16
(C) 2 (D) 3 1
of a fraction. It becomes and if (C) 48 (D) 72
32. What number must be added to 3
3 4 be added to its numerator, it 48. If a number is subtracted from
numerator and denominator of 3
4 becomes , then fraction is— the square of its one half, the
11 4 result is 48. The square root of
to give ?
12 4 3 the number is—
(A) (B)
(A) 5 (B) 6 9 20 (A) 4 (B) 5
(C) 7 (D) 8 7 5 (C) 6 (D) 8
(C) (D)
33. If the unit digit in the product 24 12
75 ? × 49 × 867 × 943 be 1, then 49. There are two numbers such that
41. A fraction becomes 4 when 1 is the sum of twice the first and
the value of ? is— added to both the numerator and thrice the second is 18, while the
(A) 1 (B) 3 denominator and it becomes 7 sum of thrice the first and twice
(C) 7 (D) 9 when 1 is subtracted from both the second is 17. The larger of
34. Three numbers are in the ratio the numerator and denominator. the two is—
4 : 5 : 6 and their average is 25. The numerator of the given frac-
(A) 4 (B) 6
The largest number is— tion is—
(A) 42 (B) 36 (C) 8 (D) 12
(A) 2 (B) 3
(C) 30 (D) 32 50. Of the three numbers, the sum of
(C) 7 (D) 15
35. The sum of three numbers is first two is 45. The sum of the
132. If the first number be twice 42. The sum of squares of two second and the third is 55 and
the second and third number be numbers is 68 and the square of the sum of the third and thrice
one-third of the first, then the their difference is 36. The pro- the first is 90. The third number
second number is— duct of the two numbers is— is—
(A) 32 (B) 36 (A) 16 (B) 32 (A) 20 (B) 25
(C) 48 (D) 60 (C) 58 (D) 104 (C) 30 (D) 35

Quantitative Aptitude Test | 39


More PDF download : http://www.gksolve.com

51. Assuming that A, B and C are 57. If a number is decreased by 4 per cent is first number greater
different single-digit numerical and divided by 6 the result is 9. than the second number ?
value other than what is already What would be the result if 3 is (A) 200
used in the following equation, C subtracted from the number and (B) 150
definitely cannot be ? then it is divided by 5 ? (C) 300
8 A 2 + 3 B 5 + C 4 – 1271. 2 1 (D) Data inadequate
(A) 9 (B) 10
(A) 7 5 5
2 63. If the sum of one-half, one-third
(B) 9 (C) 11 (D) 11 and one-fourth of a number ex-
5
(C) Either (A) or (B) ceeds the number itself by 4,
(D) None of these 58. A number is greater than the what could be the number ?
square of 44 but smaller than the
52. A number when divided by 5 (A) 24
square of 45. If one part of the
leaves a remainder 3. What is the (B) 36
number is the square of 6 and the
remainder when the square of number is a multiple of 5, then (C) 72
the same number is divided by find the number. (D) None of these
5?
(A) 1940 64. The numbers x, y, z are such that
(A) 9 (B) 3
(B) 2080 xy = 96050 and xz = 95625 and y
(C) 1 (D) 4 is greater than z by one. Find out
(C) 1980
2 (D) Cannot be determined the number z.
53. In a question divisor is of the
3 (A) 425 (B) 220
dividend and 2 times the remain- 59. If the two digits of the age of
Mr. Manoj are reversed then the (C) 525 (D) 225
der. If the remainder is 5, find
the dividend. new age so obtained is the age of 65. The ratio of two numbers is 3 : 2.
1 If 10 and the sum of the two
(A) 15 (B) 25 his wife. of the sum o f their
11 numbers are added to their
(C) 18 (D) 24 ages is equal to the difference product. Square of sixteen is
54. How many figures (digits) are between their ages. If Mr. Manoj
obtained. What could be the
required to number a book con- is elder than his wife then find
the difference between their smaller number ?
taining 200 pages ?
ages. (A) 14 (B) 12
(A) 200 (B) 600
(A) Cannot be determined (C) 16 (D) 18
(C) 492 (D) 372
(B) 10 years 1 5
55. The digit in the units place of a 66. of a number is equal to of the
(C) 8 years 5 8
number is equal to the digit in second number. If 35 is added to
(D) 9 years
the tens place of half of that the first number then it becomes
number and the digit in the tens 60. The sum of three consecutive 4 times of second number. What
place of that number is less than numbers is given. What is the is the value of the second num-
the digit in units place of half of difference between first and third ber ?
the number by 1. If the sum of number ? (A) 125 (B) 70
the digits of the number is seven, (A) One (C) 40 (D) 25
then what is the number ? (B) Three 67. In a two digit number, the digital
(A) 52 (C) Either (A) or (B) unit place is 1 more than twice
(B) 16 (D) Two of the digit at ten’s place. If the
(C) 34 61. A number gets reduced to its digit at unit’s and ten’s place be
(D) Data in inadequate one-third, when 48 is subtracted interchanged, then the difference
from it. What is two-third of that between the new number and
56. A two-digit number is seven
number ? original number is less than 1 to
times the sum of its digits. If that of original number. What is
each digit is increased by 2, the (A) 24
the original number ?
number thus obtained is 4 more (B) 72
(A) 52 (B) 73
than six times the sum of its (C) 36
(C) 25 (D) 37
digits. Find the number. (D) None of these
(A) 42 68. If the numerator of a fraction is
62. When any number is divided by increased by 2 and the denomi-
(B) 24 1
12 then dividend becomes of nator is increased by 1, the
(C) 48 4 5
(D) Data adequate the other number. By how much fraction becomes and if the
8

Quantitative Aptitude Test | 40


More PDF download : http://www.gksolve.com

numerator of the same fraction is well as denominator are decrea- and the denominator is increased
increased by 3 and the sed by 1, the fraction becomes by 1, then the fraction becomes
denominator is increased by 1, 4/5. What is the original frac-
3 5
the fraction becomes . What is tion ? . What is the value of the origi-
4 4
the original fraction ? 13 9 nal fraction ?
(A) Data inadequate (A) (B)
16 11 3
2 (A)
(B) 7
7 5 17
4 (C) (D)
(C) 6 21 5
7 (B)
8
3 70. If a fraction’s numerator is
(D) increased by 1 and the denomi- 5
7 (C)
69. If the numerator of a fraction is nator is increased by 2 then the 7
increased by 2 and denominator 2
fraction becomes . But when 6
is increased by 3, the fraction 3 (D)
becomes 7/9 and if numerator as the numerator is increased by 5 7

Answers with Hints


1. (B) Let the number be x. 12. (D) Let the numbers be x and (x + 8).
Then, 11x = 132 1
Then, [x + (x + 8)] = 35
8
⇒ x = 12.
⇒ 2x + 8 = 280
2. (D) Let the number be x.
⇒ 2x = 272
Then, x – 31 = 55 – x ∴ x = 136
⇒ 2x = 55 +31 = 86 So, the numbers are 136 and 144.
∴ x = 43. 13. (B) Let the numbers be a and b.
3. (C) 4. (A) 5. (B) (a2 – b2 ) 145
Then, (a – b) = = = 5.
a+b 29
6. (B) Let the two part be x and (24 – x),
14. (A) Let the numbers be a and b.
Then, 7x + 5 (24 – x) = 146
(a2 – b2 ) 135
⇒ 7x + 120 – 5x = 146 Then, (a + b) = = = 27
(a – b) 5
⇒ 2x = 26 x
15. (C) Let the required fraction be y.
⇒ x = 13
∴ First part = x x 1
Then, =
y+1 2
= 13
⇒ 2x – y = 1
7. (C) 8. (B)
x+1
9. (C) Let the two numbers be x and (x + 1)
y = 1 …(1)
Then, (x + 1) 2 – x2 = 35
⇒ x–y = –1 …(2)
⇒ x2 + 2x + 1 – x2 = 35 Solving,
⇒ 2x = 34 We get x = 2, y = 3
or x = 17 2
∴ The fraction is .
So, the numbers are 17 and 18. 3
10. (C) Let the numbers be 3x and 4x. 16. (B) Let the number be x,
Then, 3x + 4x = 420 4 2
Then, x– x = 8
⇒ 7x = 420 5 3
⇒ x = 60 12x – 10x
⇒ = 8
15
∴ Greater number = 4 × 60 = 240
⇒ 2x = 120
11. (D) Let the numbers be a and b.
∴ x = 60
Then, a + b = 100 and a – b = 37 17. (D) Let the number be x.
∴ a2 – b2 = (a + b) (a – b) 1 1 1
Then, of of of x = 15
= 100 × 37 = 3700 4 3 2

Quantitative Aptitude Test | 41


More PDF download : http://www.gksolve.com

1 25. (B) Let the number be x.


⇒ x = 15
24 3 2
⇒ x = 24 × 15 = 360. Then, x– x = 5
4 3
18. (C) Let the number be x. 9x – 8x
⇒ = 5
x x 12
Then, +5 = –5
5 4 ∴ x = 60.
x x 26. (D) 62 – 1 = 35, which is divisible by both 5 and 7.
⇒ – = 10
4 5 27. (A) The square of a natural number never ends in 2.
5x – 4x 28. (A) Let the second number be x.
⇒ = 10
20 Then, first number = 2x and third number = 4x
⇒ x = 200 2x + x + 4x
∴ = 56
19. (B) Let the numbers be a and b, 3
Then, (a + b)2 = (a2 + b2 ) + 2ab ⇒ 7x = 3 × 56
3 × 56
= 289 + 2 × 120 ∴ x = = 24
7
= 289 + 240 So, the smallest number is 24.
= 529 16 40
29. (C) Let × ×x = 8
∴ a+b = ⎯⎯⎯
√ 529 = 23 100 100
8 × 100 × 100
20. (A) Let the numbers be a and b. Then, x =
16 × 40
Then, a2 + b2 = 80 = 125.
and (a – b) 2 = 36 30. (A) Let the required number be x.
⇒ a2 + b2 – 2ab = 36 20
Then, x– x = 40
⇒ 2ab = (a2 + b2 ) – 36 100
= 80 – 36 = 44 ⇒ 5x – x = 200
∴ Product = ab = 22 ∴ x = 50.
x
21. (B) Let the other number be x. 31. (B) Let = p Let y when divided by 5 give q as
5
Then, 10 + x = 2 (x – 10) quotient and 1 as remainder.
⇒ x = 30 Then, y = 5q + 1
22. (D) Let the numbers be x and (15 – x) Now, x = 5p and y = 5q + 1
Then, x2 + (15 – x)2 = 113 ∴ x + y = 5p + 5q + 1 = 5 (p + q) + 1
⇒ x2 – 15x + 56 = 0 So, required remainder = 1
⇒ (x – 7) (x – 8) = 0 3 + x 11
32. (D) Let = .
∴ x = 8 4 + x 12
or x = 7 Then, 12 (13 + x) = 11 (4 + x)
So, the numbers are 7 and 8. ⇒ x = 44 – 36 = 8.
23. (C) Let the number be x, 33. (D) x × 9 × 7 × 3 = a number with unit digit 1.
21 Clearly, the minimum value of x is 9.
Then x–4 = x
34. (C) Let the numbers be 4x, 5x and 6x
⇒ x2 – 4x – 21 = 0 4x + 5x + 6x
⇒ x2 – 7x + 3x – 21 = 0 Then, = 25
3
⇒ x (x – 7) + 3 (x – 7) = 0 or 15x = 75
⇒ (x – 7) (x + 3) = 0 or x = 5
∴ x = 7 (Neglecting = – 3) Then largest number = 6x = 6 × 5 = 30.
24. (B) Let the number be x. 35. (B) Let second number be 3x, then, first one is 6x
4 and the third one is 2x.
Then, x = 64
5 ˙.˙ 3x + 6x + 2x = 132
64 × 5 ⇒ 11x = 132
⇒ x = = 80
4 ⇒ x = 12
∴ Half of the number = 40 ∴ Second number = 3x = 3 × 12 = 36.

Quantitative Aptitude Test | 42


More PDF download : http://www.gksolve.com

36. (A) Let the 3 numbers are a,b,c x–1


and, = 7
a 2 b 5 y–1
Then, b = 3, c = 3 ⇒ x – 7y = – 6
a 2 × 5 10 Solving these equations we get x = 15, y = 3.
⇒ b = 3 × 5 = 15
42. (A) Let the numbers be a and b.
b 5 × 3 15
and c = 3 × 3= 9 Then, a2 + b2 = 68 …(1)
⇒ a : b : c = 10 : 15 : 9 and (a – b) 2 = 36
Let the numbers be 10x, 15x and 9x. Now, (a – b) 2 = 36 …(2)
Then, 10x + 15x + 9x = 68 ⇒ a2 + b2 – 2ab = 36
⇒ 34x = 68 ⇒ 68 – 2ab = 36
⇒ x = 2 ⇒ 2ab = 32
∴ Second number = 15x = 15 × 2 = 30. ⇒ ab = 16.
37. (C) Let the numbers be 3x, 4x and 5x. 43. (D) Let the required number be x,
Then, 5x + 3x = 4x + 52 x
Then, + 40 = x
⇒ 4x = 52 6
⇒ x = 13 ⇒ x + 240 = 6x
⇒ x = 48.
∴ Then smallest number = 3x = 3 × 13 = 39.
44. (A) (23 × 3 + x + 42 × 3) = 235 ⇒ x = 40
38. (A) Let the required odd integers be x, x + 2 and
∴ Fourth number = 40
x + 4.
45. (A) Let the numbers be x and (50 – x),
Then, x + x + 2 +x + 4 = 57
1 1 1
Then,
⇒ 3x = 51 x + 50 – x = 12
⇒ x = 17 50 – x + x 1
⇒ =
∴ The integers are 17, 19, 21. x (50 – x) 12
39. (C) Let unit digit = x and ten’s digit = y ⇒ x2 – 50x + 600 = 0
3 (x + y) = 10y + x, 10y + x + 45 = 10x + y ⇒ x = 30 or 20
2x – 7y = 0, 9x – 9y = 45 ∴ The numbers are 20, 30.
or x – y = 5 46. (D) Let the numbers be 3x and 5x.
3x + 10 5
Solving these equations, we get Then, =
5x + 10 7
x = 7, y = 2
⇒ 7 (3x + 10) = 5 (5x + 10)
∴ Required number = 27.
⇒ x = 5
x
40. (D) Let the required fraction be y ∴ The numbers are 15 and 25.
47. (B) Let the antecedent be x.
x 1
˙·˙ = x 2
y+3 3 Then, =
24 3
⇒ 3x – y = 3 …(1)
24 × 2
x+4 3 ⇒ x = = 16.
and, 3
y = 4 48. (A) Let the number be x.
⇒ 4x – 3y = – 16 …(2)
Solving these equations, we get
Then, () x 2
2
– x = 48

x = 5, y = 12 x2
⇒ – x = 48
5 4
∴ Required fraction = ⇒ x2 – 4x – 192 = 0
12
x ⇒ (x – 16) (x + 12) = 0
41. (D) Let the required fraction be y . ⇒ x = 16.
x+1 ∴ The square root of the number is 4.
Then, = 4
y+1 49. (A) Let the numbers be x and y,
⇒ x – 4y = 3 Then, 2x + 3y = 18, 3x + 2y = 17

Quantitative Aptitude Test | 43


More PDF download : http://www.gksolve.com

Solving, we get x = 3, y = 4 Solving equations (i) and (ii), we get


∴ Larger number = 4. x = 2 and y = 6
50. (C) Let the numbers be x, y, z. ∴ From equation (A)
Then, x + y = 45, y + z = 55, z + 3x = 90 Number = 10 (y – 1) + x = 52.
Now, y = (45 – x) and z = 55 – y 56. (A) Let the two-digit number be 10x + y
z = 55 – (45 – x) = 10 + x
10x + y = 7 (x + y)
∴ 10 + x + 3x = 90
⇒ x = 2y …(i)
⇒ x = 20
So, third number = z 10 (x + 2) + (y + 2) = 6 (x + y + 4) + 4
= 10 + x = 30. or 10x + y + 22 = 6x + 6y + 28
51. (D) Since A + B + C = 16 ⇒ 4x – 5y = 6 …(ii)
(Possible value of A, B and C are 0, 6, 7, and 9) Solving equations (i) and (ii)
Also A ≠ B, B ≠ C, A ≠ C We get x = 4 and y = 2.
If C = 6, A + B should be 10, which is not possible. 57. (D) Let the number be x.
If C = 9, A + B should be 7, which is also not
possible. x–4
∴ = 9
If C = 0, A + B should be 16, which is also not 6
possible. ⇒ x = 58
52. (D) The number is of the form (5x + 3) where x is an x–3 58 – 3
Again = = 11 .
integer 5 5
(5x + 3) 2 25x2 + 30x + 9 58. (C) Let the number be x.
∴ =
5 5 442 < x < 452 ⇒ 1936 < x < 2025 …(i)
2
25x 30x 5 + 4
= + + From equation (i), the required number will be any
5 5 5 number between 1936 and 2025. Since one part of
∴ The remainder is 4. the number is the square of 6 means one factor is 36
53. (A) According to the question, ∴ L.C.M. of 36 and 5 = 180
2 ∴ Number will be multiple of 180 i.e.,
Divisor = × dividend
3 180 × 11 = 1980 the only value which satisfies the
and Divisor = 2 × remainder equation (i).
2 59. (D) Let the age of Mr. Manoj be (10x + y) yrs.
or × dividend = 2 × 5
3 ∴ His wife’s age = (10y + x) years
2×5×3 1
∴ Dividend = = 15 Then, (10x + y + 10y + x)
2 11
54. (C) Number of one digit pages from = 10x + y – 10y – x
1 to 9 = 9
⇒ x + y = 9x – 9y
Number of two digit pages from
10 to 99 = 90 ⇒ 8x = 10y
Number of three digit pages from x 5

100 to 200 = 101 y = 4
∴ Total number of required figures ∴ x = 5 and y = 4 (because any other multiple of 5
= 9 × 1 + 90 × 2 + 101 × 3 = 492 will make x of two digits).
55. (A) Let 1/2 of the no. = 10x + y ∴ Diff. 10x + y – 10y – x = 9x – 9y = 9 (x – y)
and the no. = 10v + w = 9 (5 – 4) = 9 yrs.
From the given conditions, 60. (D) Let the three consecutive numbers be x, x + 1 and
w = x and v = y – 1 x + 2 respectively
Thus the no. = 10 (y – 1) + x …(A) ∴ Diff. between first and third numbers
∴ 2 (10x + y) = 10 (y – 1) + x = x + 2 – x = 2.
⇒ 8y – 19x = 10 …(i) 61. (D) Let the number be x.
x 2
v+w = 7 Then, x – = 48; x = 48.
3 3
⇒ y–1+x = 7 62. (D) Here neither the remainder nor the dividend nor
∴ x+y = 8 …(ii) the second number is given. So, can’t be determined.

Quantitative Aptitude Test | 44


More PDF download : http://www.gksolve.com

63. (D) Let the number be x ⇒ 8x – 5y = 11 …(i)

∴ (
1 1 1
+ +
2 3 4 )x = (6 +124 + 3) x Again
x+3
y+1
=
3
4
13
= x ⇒ 4x – 3y = 9 …(ii)
12
According to the question, Solving equations, (i) and (ii), we get,
13 x = 3 and y = 7
x–x = 4
12
3
∴ x = 48 ∴ Fraction = .
7
64. (D) xy = 96050 …(i)
69. (C) Let the numerator and denominator be x and y
and xz = 95625 …(ii) respectively.
and y–z = 1 …(iii)
x+2 7
Dividing (i) by (ii) we get Then, =
y+3 9
y 96050 3842 226

z = 95625 = 3825 = 225 …(iv) 9 (x + 2) = 7 (y + 3)
Combining (iii) and (iv) we get z = 225. ⇒ 9x – 7y = 3 …(i)
65. (B) Let the two numbers be 3x and 2x x–1 4
According to the question, Again, =
y–1 5
10 + (3x + 2x) + (3x × 2x) = (16) 2
⇒ 5x – 4y = 1 …(ii)
⇒ 6x2 + 5x – 246 = 0
⇒ 2
6x + 41x – 36x – 246 = 0 Solving (i) and (ii) we get,
⇒ x (6x + 41) – 6 (6x + 41) = 0 x = 5, y = 6
⇒ (6x + 41) (x – 6) = 0
5
– 41 Reqd. fraction =
∴ x = 6 or 6
6
(But – ve value cannot be accepted) x
70. (C) Let the fraction be y
∴ Smaller number = 2x = 2 × 6 = 12.
1 5 x 25 x+1 2
66. (C) x = y ∴ Then, =
5 8 y = 8 … (i) y+2 3
x + 35 = 4y ⇒ 3x + 3 = 2y + 4
25 ⇒ 3x = 2y + 1 …(i)
or y + 35 = 4y
8
x+5 5
∴ y = 40. Also, we have =
y+1 4
67. (D) Let the original number be 10x + y
y = 2x + 1 …(i) ⇒ 4x + 20 = 5y + 5
and (10y + x) – (10x + y) = 10x + y – 1 ⇒ 4x = 5y – 15 …(ii)
⇒ 9y – 9x = 10x + y – 1
From (i) and (ii), we get
⇒ 19x – 8y = 1 …(ii)
Putting the value of (i) in equation (ii) we get 2y + 1 5y – 15
˙·˙ =
3 4
19x – 8 (2x + 1) = 1
⇒ 19x – 16x – 8 = 1 ⇒ 8y + 4 = 15y – 45
⇒ 3x = 9, ⇒ 7y = 49
⇒ x = 3
∴ y = 7
So y = 2×3+1=7
2y + 1 2 × 7 + 1
∴ Original number = 10 × 3 + 7 = 37 and x = = =5
3 3
x
68. (D) Let the original fraction be y ∴ Required original fraction
x+2 5 x 5
Then = = y=
y+1 8 7

Quantitative Aptitude Test | 45


More PDF download : http://www.gksolve.com

Decimal Fraction
Decimal fractions : Fractions in 3 7 2 4. ? ÷ ·0025 = 800
= 0·375, = 0·583, = 0·666,
which denominators are powers of 10 8 12 3 (A) ·2 (B) ·02
are called decimal fractions. 14 16 1 (C) 2000 (D) 2
= 0·736, = 0·64 and = 0·5
1 1 1 19 25 2
, , etc. are respecti- 5. 0·001 ÷ ? = 0·01
10 100 1000 Clearly 0·375 < 0·5 < 0·583 (A) 10 (B) ·1
vely the tenth, the hundredth and the < 0·64 < 0·666 < 0·736
thousandth part of 1. (C) ·01 (D) ·001
3 1 7 16 2 14
7 ∴ < < < < < 6. 0·000033 ÷ 0·11 = ?
is 7 tenth written as ·7 (Called 8 2 12 25 3 19
10 (A) ·003 (B) ·03
Q. 3. (i) 0·001 ÷ ? = 0·1
decimal seven) (C) ·0003 (D) ·3
(ii) ? ÷ ·025 = 40
13 7. 25 ÷ ·0005 = ?
is 13 hundredth, written as Solution :
100 (A) 50
·13 (Called decimal one-three) 0·001
(i) Let x = 0·1 (B) 500
9 (C) 5000
is 9 hundredth, written as
100 0·001 1 (D) None of these
·09 (Called decimal zero-nine) Then, x = = = ·01
0·100 100
8. 12 ÷ 0·09 of 0·3 × 2 = ?
4 x
is 4 thousandth, written as (ii) Let = 40, (A) 0·8
1000 ·025
·004 (Called decimal zero-zero four) (B) 0·08
Then x = 40 × ·025 = 1 (C) 8
and so on.
53·678 = 50 + 3 + ·6 + ·07 + Q. 4. Given that 172 × 38 = (D) None of these
·008. 6536, find 1·72 × ·38.
9. 50·8 ÷ 2540 = ?
Solution : (A) 2 (B) ·2
Examples Sum of decimal places (C) 0·002 (D) 0·02
Q. 1. Express each of the = (2 + 2) = 4 10. 0·0169 ÷ 0·013 = ?
following as a vulgar fraction : ∴ 1·72 × ·38 = ·6536. (A) ·13 (B) ·013
– ⎯ – Q. 5. Subtract :
(i) 0·17, (ii) 0·1254, (iii) 2·536 (C) 1·3 (D) 13
Solution : (i) 16·3629 from 21·003 11. 15·60 × 0·30 = ?
– 17 – 1 16 8 (ii) 8·2967 from 11. (A) 4·68 (B) 0·458
(i) 0·17 = = =
90 90 45 Solution : (C) 0·468 (D) 0·0468
⎯ 1254 – 12 (i) 21·003 12. 3 × 0·3 × 0·03 × 0·003 × 30 = ?
(ii) 0·1254 =
9900 – 16·3629 (A) ·0000243 (B) ·000243
1242 69 4·6401 (C) ·00243 (D) ·0243
= =
9900 550
(ii) 11·0000 13. 16·7 + 12·38 – ? = 10·09
– –
(iii) 2·536 = 2 + 0·536 – 8·2967 (A) 17·89 (B) 18·99
536 – 53 2·7033 (C) 16·98 (D) 20·09
=2+
900 14. 0·6 + 0·66 + 0·066 + 6·606 = ?
483 Exercise
=2+ (A) 6·744 (B) 6·738
900 1. 0·8 × ? = 0·0004 (C) 7·932 (D) 7·388
161 161 (A) ·0005 (B) ·005
=2+ =2 . 15. The H. C. F. of 0·54, 1·8 and 7·2
300 300 (C) ·5 (D) ·00005 is—
Q. 2. Arrange the fractions 2. 0·09 × 0·008 = ? (A) 1·8 (B) ·18
3 7 2 14 16 1
, , , , and in ascending (A) 0·072 (B) 0·0072 (C) ·018 (D) 18
8 12 3 19 25 2
order of magnitude. (C) 0·00072 (D) 0·72 16. What decimal of an hour is a
Solution : 3. ? % of 10·8 = 32·4 second ?
Converting each of the given (A) 3 (B) 30 (A) ·0025 (B) ·0256
fraction into decimal form, we get (C) 300 (D) 0·3 (C) ·00027 (D) ·000126

Quantitative Aptitude Test | 46


More PDF download : http://www.gksolve.com

17. Which of the following fractions 35. 8·32 × 0·999 = ?



√ 5
is the smallest ? 25. If ⎯
√⎯15 = 3·88, the value of (A) 0·831168 (B) 8·31618
3
11 9 is— (C) 8·31168 (D) 8·31668
(A) (B)
13 11 (A) 0·43 (B) 1·89 36. (9·75 × 9·75 – 2 × 9·75 × 5·75
3 5 (C) 1·29 (D) 1·63
(C) (D) + 5·75 × 5·75) = ?
4 7
(A) 13·25 (B) 3·625
18. Which of the following fractions 26. If ⎯
√ 5 = 2·24, then the value of
(C) 4 (D) 16
are in ascending order ? 3√
⎯5
16 11 17 is— 37. 13·065 × 13·065 – 3·065 × 3·065
(A) , , 2√
⎯ 5 – 0·48 =?
19 14 22
(A) 0·168 (B) 1·68 (A) 161·3 (B) 159·5
11 16 17
(B) , , (C) 16·8 (D) 168 (C) 141·6 (D) 100
14 19 22
17 11 16 27. If 12276 ÷ 155 = 79·2 the value ·896 × ·752 + ·896 × ·248
(C) , , of 122·76 ÷ 15·5 is— 38. =?
22 14 19 ·7 × ·034 + ·7 × ·966
16 17 11 (A) 7·092 (B) 7·92 (A) 1·28 (B) 0·976
(D) , , (C) 79·02 (D) 79·2
19 22 14 (C) 12·8 (D) 9·76
·24 × ·35 1 ·356 × ·356 – 2 × ·356 × ·106
19. is equal to— 28. If = ·2689, then the value 39.
·14 × ·15 × ·02 3·718 ·632 × ·632 + 2 × ·632 × ·368
1
(A) 2 (B) 20 of is— + ·106 × ·106
·0003718 =?
(C) 200 (D) 2000 + ·368 × ·368
(A) 2689 (B) 2·689
·35 × ·0015 (A) ·25 (B) ·0765
20. Exp. = written as a (C) 26890 (D) ·2689
·25 × ·07 (C) ·345 (D) ·0625
percentage is— 0·5
29. 0·15 ÷ =? 40. 3·65 × 3·65 + 2·35 × 2·35
(A) 0·3% 15
– 2 × 2·35 × 3·65
(A) 4·5 (B) 45 =?
(B) 3% 1·69
(C) 30% (C) 0·03 (D) 0·45 (A) 1·69 (B) 2·35
(D) None of these 20 + 8 × 0·5 (C) 3·65 (D) 1
30. = 12
20 – ?
21. If 2805 ÷ 2·55 = 1100, then 41.
280·5 ÷ 25·5 is equal to— (A) 8
0·5 × 0·5 × 0·5 + 0·6 × 0·6 × 0·6
(B) 18 =?
(A) 1·01 (B) 1·1 0·5 × 0·5 – 0·3 + 0·6 × 0·6
(C) 2
(C) 0·11 (D) 11 (A) 0·3 (B) 1·1
(D) None of these
(C) 0·1 (D) 0·61
22. The square root of 17·28 ÷ ?
31. = 200 ·125 + ·027
0·324 × 0·081 × 4·624 3·6 × 0·2 42. =?
1·5625 × 0·0289 × 72·9 × 64 ·5 × ·5 – 1·5 + ·09
(A) 120 (B) 1·20
is— (A) ·08 (B) 1
(C) 12 (D) 0·12
(C) ·2 (D) ·8
(A) 24 3420 ?
×7 0·47 × 0·47 × 0·47 – 0·33
(B) 2·4
(C) 0·024
32.
19
=
35
0·01 43. (0·47 × 0·47 + 0·47 × 0·33
(A) × 0·33 × 0·33
(D) None of these 9
18
)
+ 0·33 × 0·33
=?
(B) (A) 0·14 (B) 0·8
⎯√⎯⎯
0·289 7
23. =? (C) 15·51 (D) 1
0·00121 63
(C)
1·04 × 1·04 + 1·04 × 0·04
(A)
170
11
(B)
17
110
5
(D) None of these
44. (1·04 × 1·04 × 1·04 – 0·04
+ 0·04 × 0·04
(C)
17
1100
(D)
17
11
33.
·538 × ·538 – ·462 × ·462
1 – ·924
=? )
× 0·04 × 0·04
=?

(A) ·076 (B) 1·042 (A) 0·10 (B) 0·1


24. If ⎯
√⎯⎯⎯
4096 = 64, then the value of (C) 1 (D) 2 (C) 1 (D) 0·01

⎯⎯⎯⎯
40·96 + ⎯
√⎯⎯⎯⎯
0·4096 + ⎯
√⎯⎯⎯⎯⎯
0·004096 34. 45.
(0·87)3 + (0·13)3
(0·05)2 + (0·41)2 + (0·073)2 (0·87) + (0·13)2 – 0·87 × 0·13
2
+⎯√⎯⎯⎯⎯⎯⎯⎯
0·00004096 is— =? =?
(0·005)2 + (0·041)2 + (0·0073)2
(A) 7·09 (B) 7·1014 (A) 0·1 (B) 10 (A) 0·13 (B) 0·74
(C) 7·1104 (D) 7·12 (C) 100 (D) 1000 (C) 0·87 (D) 1

Quantitative Aptitude Test | 47


More PDF download : http://www.gksolve.com

46. (0·04)3 = ? 11 ·23 – ·023


(C) 4 66. =?
(A) 0·064 (B) 0·0064 90 ·0023 ÷ 23
(C) 0·00064 (D) 0·000064 (D) None of these (A) 0·207 (B) 207
47 58. 1 litre of water weighs 1 kg. How (C) 2070 (D) 0·0207
47. The value of is— many cubic millimetres of water
10000 (3·537 – ·948) 2 + (3·537 + ·948)2
will weigh 0·1 gm ? 67.
(A) ·0047 (B) ·0470 (3·537)2 + (·948)2
(C) ·00047 (D) ·000047 (A) 0·1 (B) 1 =?
(C) 10 (D) 100 (A) 4·485 (B) 2·589
3
48. =? 59. What should be subtracted from (C) 4 (D) 2
0·3 – 3·03
3+ the multiplication of 0·527 and
3 × 0·91 68. (·803 × ·647 + ·803 × ·353) = ?
2·013 to get 1 ?
(A) 1·5 (B) 15 (A) ·803 (B) 1
(A) 0·060851 (B) 2·060851
(C) ·75 (D) 1·75 (C) ·45 (D) 1·450
(C) 0·939085 (D) 1·9339085
{(0·1)0·0001 + 1 } is equal 6·5 × 4·7 + 6·5 × 5·3
2– (0·01)2
49. ⎯ 69. =?
60. 0· 53 = ? 1·3 × 7·9 – 1·3 × 6·9
to— 53 53 (A) 3·9 (B) 39
(A) (B)
(A) 100 (B) 101 100 90 (C) 34·45 (D) 50
(C) 1010 (D) 1101 53 26
(C) (D) 70. The greatest fraction out of
50. The L. C. M. of 3·0, 0·09 and 2·7 99 45 2 5 11 7
is— ⎯ ⎯ ⎯ ⎯ , , and is—
5 6 12 8
61. (· 6 + · 7 + · 8 + · 3 ) = ?
(A) 2·7 (B) ·27 7 11
3 33 (A) (B)
(C) ·027 (D) 27 (A) 2 (B) 2 8 12
10 100
51. G. C. D. of 1·08, ·36 and ·9 is— ⎯ 5 2
2 (C) (D)
(A) ·03 (B) ·9 (C) 2 (D) 2· 33 6 5
3
(C) ·18 (D) ·108 71. The place value of 3 in 0·07359
62. If 1·5x = 0·04 y then the value of
⎯ is—
52. 0·1 36 = ? y–x
is— 3
136 136 y+x (A) 3 (B)
(A) (B) 100
1000 999 730
136 3 (A) 3 3
(C) (D) 77 (C) (D)
990 22 1000 10000
73
⎯ ⎯ (B) 72. (·6 × ·6 × ·6 × ·4 × ·4 × ·4 + 3
77
53. (0·63 + 0·37) = ? × ·6 × ·4 × (·6 + ·4) = ?
⎯ 7·3
(A) 1 (B) 1· 01 (C) (A) 21·736 (B) 2·1736
77
⎯ (D) None of these (C) ·21736 (D) 1
(C) · 101 (D) 1·01
73. (·58 × ·58 × ·58 – ·42 × ·42
⎯ ⎯ 63. If ⎯√⎯⎯⎯⎯⎯⎯⎯⎯⎯
0·05 × 0·5 × a = 0·5 × 0·05
54. (0·3467 + 0·1333) = ? × ·42 – 3 × ·58 × ·42 × ·16) = ?
a
⎯ ×√
⎯ b then b = ? (A) 0·004096 (B) 1·3976
(A) 0·48 (B) 0·48 01
⎯ (C) 0·16 (D) 1
(C) 0·4 8 (D) 0·48 (A) ·0025
(B) ·025 74. By how much is 12% of 24·2
⎯ ⎯ more than 10% of 14·2 ?
55. (3·57 – 2· 14) = ? (C) ·25
⎯ (A) 0·1484 (B) 14·84
(A) 1·43 (B) 1·43 01 (D) None of these (C) 1·484 (D) 2·762
⎯ 64. 2·53 × ·154 is the same as—
(C) 1· 43 (D) 1·43 75. 3·5 + 21 × 1·3 = ?
(A) 253 × ·00154 (A) 7·28
⎯ ⎯ ⎯
56. (2·47 + 3·53 + 0·05) = ? (B) 25·3 × 1·54 (B) 6·13
⎯ (C) 253 × ·0154 (C) 72·8
(A) 6 (B) 6· 06
⎯ (D) 253 × ·0154 (D) None of these
(C) 0·66 (D) 6· 01 ? – 0·11 76. What decimal fraction is 40 ml
– 65. = 1·6
1·6 of a litre ?
57. The value of 4·1 2 is—
11 (A) 2·56 (A) ·4
(A) 4 (B) 1·76 (B) ·04
99
2 (C) 0·267 (C) ·05
(B) 5
9 (D) None of these (D) None of these

Quantitative Aptitude Test | 48


More PDF download : http://www.gksolve.com

Answers with Hints


1. (A) Let 0·8 × x = 0·0004 15. (B) The given numbers are 0·54, 1·80 and 7·20
0·0004 0·004 H. C. F. of 54, 180 and 720 is 18
Then, x = = = ·0005
0·8 8 ∴ H. C. F. of given numbers = 0·18
2. (C) ˙·˙ 9 × 8 = 72 1
16. (C) Required decimal = = ·00027
∴ 0·09 × 0·008 = 0·00072 (Five places of decimal) 60 × 60
3. (C) Let x% of 10·8 = 32·4 11
17. (D) = 0·846,
x 3
Then, × 10·8 = 32·4
100 9
= 0·818,
32·4 × 100 11
or x = = 300
10·8 3
x = 0·75
4. (D) Let = 800 4
·0025
5
Then, x = 800 × ·0025 and = 0·714
7
800 × 25 5
= =2 Clearly, 0·714 is the smallest and hence is
10000 7
0·001 smallest fraction.
5. (B) Let x = 0·01 16
18. (C) = 0·842,
0·001 0·001 19
Then, x = =
0·01 0·010 11
= 0·785
1 14
= = 0·1
10 17
and = 0·772
0·000033 0·0033 22
6. (C) = = ·0003
0·11 11 ∴ 0·772 < 0·785 < 0·842
25 250000 17 11 16
7. (D) = = 50000 or < < .
·0005 5 22 14 19
8. (D) Given expression = 12 ÷ 0·027 × 2 ·24 × ·35 24 × 35 × 100
12 24000 19. (C) = = 200
= ×2= ·14 × ·15 × ·02 14 × 15 × 2
0·027 27 ·35 × ·0015 35 × 15
8000 20. (B) Exp. = =
= ·25 × ·07 25 × 700
9
9. (D)
50·8
=
508
2540 25400 100
=
2
= ·02
= ( 3
100 )
× 100 % = 3%

0·0169 169 13 280·5 2805


10. (C) = = = 1·3 21. (D) =
0·0130 130 10 25·5 255
11. (A) ˙·˙ 1560 × 30 = 46800 2805 1100
= = = 11
2·55 × 100 100
∴ 15·60 × 0·30 = 4·6800
324 × 81 × 4624
= 4·68 (Four places of decimal) 22. (C) Given expression =
15625 × 289 × 729 × 64
12. (C) 3 × 3 × 3 × 3 × 30 = 2430
9
∴ 3 × 0·3 × 0·03 × 0·003 × 30 =
15625
= 0·002430 (Six places of decimal)
3
13. (B) Let 16·7 + 12·38 – x = 10·09 ∴ Its square root = = ·024
125
∴ x = (16·7 + 12·38 – 10·09) = 18·99
⎯√⎯⎯ ⎯⎯⎯

0·289 0·28900
14. (C) 0·6 23. (A) =
0·00121 0·00121
0·66
⎯⎯⎯⎯

⎯⎯28900
√ 28900
0·066 = =
121 ⎯⎯⎯
√ 121
6·606
170
7·932 =
11
Quantitative Aptitude Test | 49
More PDF download : http://www.gksolve.com

24. (C) Given expression 34. (C) Given expression 0


(a2 + b2 + c2)
⎯√⎯4096
100 √ ⎯⎯10000 ⎯⎯⎯
√ 1000000 √⎯⎯⎯⎯
4096 4096 4096 = a 2
= + + +
100000000
( ) ( ) ( )
10
+
b 2
10
+
c
10
2

⎯√⎯⎯⎯
4096 √⎯⎯⎯⎯
4096 √⎯⎯⎯
4096 √⎯⎯⎯⎯
4096
= + + + 100 × (a2 + b2 + c2)
10 100 1000 10000 = = 100
(a2 + b2 + c2)
64 64 64 64
= + + + 35. (C) Given expression = 8·32 × 0·999
10 100 1000 10000
= 6·4 + ·64 + ·064 + ·0064 = 7·1104. = 8·32 × (1 – 0·001) = 8·32 – 8·32 × 0·001
= 8·32 – ·00832 = 8·31168.
25. (C)
⎯√ 53 = √⎯√⎯ 53 × ⎯√⎯√33 = √⎯⎯315 36. (D) Given expression
= (a2 – 2ab + b2 ) where a = 9·75 and b = 5·75
3·88
= = 1·29 = (a – b)2 = (9·75 – 5·75)2 = (4)2 = 16
3
37. (A) Given expression
3√
⎯5 3 × 2·24
26. (B) = = (13·065)2 – (3·065)2
2√
⎯ 5 – 0·48 2 × 2·24 – 0·48
= (13·065 + 3·065) × (13·065 – 3·065)
6·72 6·72
= = = 1·68 = (16·13 × 10) = 161·3
4·48 – 0·48 4
38. (A) Given expression
122·76 12276
27. (B) = ·896 × (·752 + ·248) ·896 × 1
15·50 1550 = =
·7 × (·034 + ·966) ·700 × 1
12276 1 79·2
= × = = 7·92 896
155 10 10 = = 1·28
700
28. (A)
1
·003718
=
10000
3·718
= 10000 ×( 1
3·718 ) 39. (D) Given expression
= 10000 × ·2689 = 2689. (·356)2 – 2 × ·356 × ·106 + (·106)2
=
(·632)2 + 2 × ·632 × ·368 + (·368)2
0·5 15 5
29. (A) ? = 0·15 ÷ = ÷ ⎛ a2 – 2ab + b2 ⎞ (a – b)2
15 100 150 =⎜ ⎟ =
15 150 ⎝ c2 + 2cd + d2 ⎠ (c + d) 2
= × = 4·5
100 5 (·356 – ·106) 2
=
20 + 8 × 0·5 (·632 + ·368)2
30. (B) Let = 12.
20 – x = (·25)2 = ·0625
Then, 24 = 12 (20 – x) 40. (D) Given expression
∴ 12x = 216 (3·65)2 + (2·35)2 – 2 × 3·65 × 2·35
=
⇒ x = 18 1·69
a2 + b2 – 2ab
17·28 ÷ x = , where a = 3·65 and b = 2·35
31. (D) Let = 200 1·69
3·6 × 0·2
(a – b)2 (3·65 – 2·35) 2 (1·3)2 1·69
17·28 = = = = =1
Then, = 200 × 3·6 × 0·2 1·69 1·69 1·69 1·69
x
41. (B) Given expression
17·28 1728
∴ x= = = 0·12. (0·5)3 + (0·6)3
200 × 3·6 × 0·2 200 × 36 × 2 =
(0·5) – 0·5 × 0·6 + (0·6)2
2
3420 x
32. (D) Let = ×7 ⎛ a3 + b3 ⎞
19 0·01 =⎜ ⎟
3420 0·01 180 9 ⎝ a2 – ab + b2 ⎠
Then, x = × = =
19 7 700 35 = (a + b)
33. (C) Given expression = (0·5 + 0·6)
(·538)2 – (·462)2 (·538 + ·462) (·538 – ·462) = 1·1.
= =
·076 ·076 42. (D) Given expression
·076 (0·5)3 + (0·3)3
= =1 =
·076 (0·5) – 0·5 × 0·3 + (0·3)2
2

Quantitative Aptitude Test | 50


More PDF download : http://www.gksolve.com

a3 + b3 (a + b) (a2 – ab + b2 ) ⎯ ⎯
= = 54. (B) Given Exp. = 0·34 67 + 0·1333
a2 – ab + b 2 (a2 – ab + b2 )
3467 – 34 1333 – 13
= (a + b) = (0·5 + 0·3) = 0·8 = +
9900 9900
43. (A) Given expression 3433 + 1320 4753
= =
(0·47)3 – (0·33)3 9900 9900
=
(0·47) + 0·47 × 0·33 + (0·33)2
2
4801 – 48 ⎯
= = 0·4801.
(a3 – b3 ) (a – b) (a2 + ab + b2) 9900
= 2 =
(a + ab + b )2 (a2 + ab + b2) ⎯ ⎯ 57 14
55. (C) Given Exp. 3· 57 – 2· 14 = 3 + –2–
= (a – b) = (0·47 – 0·33) = 0·14. 99 99
44. (C) Given expression 57 14
= 1+ –
99 99
(1·04)2 + 1·4 × 0·04 + (0·04)2
= 43 ⎯
(1·04)3 – (0·04)3 = 1+ = 1·43
99
a2 + ab + b2 (a2 + ab + b2)
= 3 3 = ⎯ ⎯ ⎯
a –b (a – b) (a2 + ab + b2) 56. (B) Given Exp. = 2· 47 + 3·53 + 0·05
1 1 47 53 05
= a – b= =1 = 5+ + +
1·04 – 0·04 99 99 99
45. (D) Given expression 105 06
= 5+ =5+1
a3 + b3 99 99
= 2 where = 0·87 and b = 0·13
a + b2 – ab 06 ⎯
= 6 = 6·06.
(a + b) (a2 + b2 – ab) 99
= = (a + b) – –
(a2 + b2 – ab) 12 – 1 11
57. (C) 4·12 = 4 + 0·12 = 4 + =4
= (0·87 + 0·13) = 1 90 90
46. (D) (0·04) 3 = 0·04 × 0·04 × 0·04 = ·000064. 58. (D) 1000 gm is the weight of = 1000 cu. cm of water
47 1 gm is the weight of 1 cu. cm. of water
47. (A) = ·0047
10000 = 1000 cu. mm.
48. (A) Given expression 1 1000
gm is the weight of = cu. mm of water
3 3 3 10 10
= = =
2·73 273 3–1 = 100 cu. mm of water
3– 3–
3 × 0·91 3 × 91 59. (A) ˙·˙ 0·527 × 2·013 = 1·060851
3 Hence, the required number = 0·060851.
= = 1·5
2 ⎯ 53
60. (C) 0·53 = .
49. (A) Given expression = (
0·01 – 0·0001
0·0001
+1 ) 99

= (
·0099
)
+ 1 = (99 + 1)
61.
– – – –
(C) · 6 + ·7 + ·8 + ·3 = ( 6 7 8 3
+ + +
9 9 9 9 )
·0001 24 8 2
= 100 = = =2 .
9 3 3
50. (D) The given numbers are 3·00, 0·09 and 2·70 x 0·04 4 2
62. (B) y = = =
L. C. M. of 300, 9 and 270 is 2700 1·5 150 75
∴ Required L. C. M. = 27·00 = 27 x 2
1 –y 1 –
51. (C) The given numbers are 1·08, ·36 and 0·9 y–x 75
∴ Given Exp. =
y+x x= 2
G. C. D. of 108, 36 and 90 is 18 1+y 1+
75
∴ Required G. C. D. = 0·18

52.
⎯ 136 – 1 135 3
(D) 0·136 =
990
= =
990 22
=
73/75
77/75
=( )
73 75
×
75 77
=
73
77
⎯ ⎯ 63. (B) ⎯⎯⎯⎯⎯⎯⎯⎯
√ ·05 × ·5 × a = ·5 × ·05 × √
⎯b
53. (B) Given Exp. = 0· 63 + 0·37
63 37 ⇒ ⎯⎯⎯⎯⎯⎯
√ ·025 × a = 0·25 × √⎯b
= +
99 99 ⇒ 0·025a = ·025 × ·025 × b
100 1 ⎯ a ·025 × ·025
= = 1 = 1·01 ∴ b = = ·025
99 99 ·025

Quantitative Aptitude Test | 51


More PDF download : http://www.gksolve.com

64. (A) Clearly 2·53 × ·154 is the same as 253 × ·00154 71. (C) The place value of 3 in the given decimal fraction
as both contain same number of decimal places. 3
x – 0·11 is ·003 i.e., .
1000
65. (D) Let = 1·6
1·6
72. (D) Given expression
Then, x – 0·11 = 1·6 × 1·6 = 2·56
∴ x = 2·56 + 0·11 = 2·67. = (·6)3 + (·4)3 + 3 × ·6 × ·4 × (·6 + ·4)
·207 ·207
66. (C) Given expression = = = a3 + b3 + 3ab (a + b) = (a + b)3
·0023 ·0001
23 = (·6 + ·4)3 = 1 3 = 1.
·2070
= = 2070. 73. (A) Given expression
·0001
(a – b) 2 + (a + b)2 = (·58)3 – (·42)3 – 3 × ·58 × ·42 × (·58 – ·42)
67. (D) Given expression =
(a2 + b2 )
= a3 – b3 – 3ab (a – b) = (a – b)3
(where a = 3·537 and b = 0·948)
2 (a2 + b2 ) (where a = ·58 and b = ·42)
= =2
(a2 + b2 )
= (·58 – ·42)3 = ·16 × ·16 × ·16 = 0·004096
68. (A) Given expression = ·803 × (·647 + ·353)
= ·803 × 1 = ·803
6·5 × (4·7 + 5·3)
74. (C) Required difference = (100
12
× 24·2 –
10
100
× 14·2)
69. (D) Given expression =
13 × (7·9 – 6·9) = 2·904 – 1·42 = 1·484.
6·5 × 10
= = 50.
1·3 × 1 75. (D) Given Expression = 3·5 + 21 × 1·3
2 5 11 = 3·5 + 27·3 = 30·8.
70. (B) = 0·4, = 0·833, = 0·916
5 6 12
40
7 76. (B) Required fraction =
and = 0·875 1000
8
11 4
Clearly, the greatest fraction is 0·916, i.e. . = = ·04
12 100

Quantitative Aptitude Test | 52


More PDF download : http://www.gksolve.com

Unitary Method
The method in which the value Solution : ∴ Work of 8 men and 7 women
of a unit is first found is called the 5 dozens = 12 × 5 = 60 = Work of (14 + 7)
Unitary Method. Threfore, in solving
and, 4 dozens = 12 × 4 = 48 = Work of 21 women
any question by this method the value
of 1 unit is first found and then the ˙.˙ Cost of one chair ˙.˙ 7 women complete the work
value of require units is to be found. = Rs. 341 in = 60 days
The following points are to be ∴ Cost of 60 chairs ⇒ 1 woman will complete the
kept in mind while solving the ques- work in = 60 × 7 days
= 341 × 60
tions by this method : ∴ 21 women will complete the
= Rs. 20460
(1) The term in which the answer 60 × 7
and cost of 1 table work in =
is to be calculated is always put to 21
the right hand side. = Rs. 852 = 20 days.
(2) The term to the right hand ∴ Cost of 48 tables
side is never put in the denominator. = 852 × 48 Exercise
(3) If on reducing to the unit, a = Rs. 40896 1. When a 192 metres long rod is
smaller result is expected, then the ∴ Cost of 5 dozens of chairs and 4 cut down into small pieces of
right hand term is divided by the left dozens of tables length 3·2 metres each. Then
hand term.
= 20460 + 40896 how many pieces are available ?
(4) If on reducing to the unit, a
= Rs. 61356 (A) 52
greater result is expected, then the
right hand term is multiplied by the Q. 4. A tank can be filled by 20 (B) 68
left hand term. buckets each of capacity 13·5 litres. (C) 62
If the capacity of each bucket be 9 (D) None of these
Examples litres, how many buckets will fill
the same tank ? 2. The cost of 2 tables is equal to
Q. 1. The price of one dozen the cost of 5 chairs. If the diffe-
Solution :
pens is Rs. 540. What will be the
˙.˙ To fill the tank the number of rence of the cost of one table and
price of 319 such pens ?
buckets each of capacity 13·5 litres one chair is Rs. 1200, then the
Solution : cost of one chair is—
required is 20
˙.˙ Price of 12 pens = Rs. 540
∴ To fill the tank the number of (A) Rs. 500 (B) Rs. 400
540 buckets each of capacity 1 litre
⇒ Price of 1 pen = (C) Rs. 800 (D) Rs. 600
12 required is = 20 × 13·5
540 × 319 1
∴ Price of 319 pens = ∴ To fill the tank the number of 3. If the cost of th of kg is Rs.
12 buckets each of capacity 9 litres 4
0·60, then what is the cost of 200
= Rs. 14355 Required no. of buckets gm ?
Q. 2. 12 machines take 30 hours 20 × 13·5
= (A) 42 paise (B) 48 paise
to print a certain job how long will 9
take 16 machines to print the same (C) 40 paise (D) 50 paise
= 30
job ?
Q. 5. If 4 men or 7 women do a 4. If the cost of three dozens man-
Solution : work in 60 days in how many days goes is Rs. 245, what will be the
˙.˙ 12 machines print in = 30 hours will 8 men and 7 women finish the approximate cost of 363
∴ 1 machine will print in same work ? mangoes ?
= 30 × 12 hours Solution : (A) Rs. 2200 (B) Rs. 2000
Work of 4 men (C) Rs. 2400 (D) Rs. 2600
∴ 16 machines will print in
= Work of 7 women
30 × 12 45 5. If the weight of 13 metres long
= = ∴ Work of 1 man
16 2 rod is 23·4 kg. What is the weight
7 of 6 metres long rod ?
1 = Work of women
= 22 hours 4
2 (A) 7·2 kg. (B) 10·8 kg.
∴ Work of 8 men
Q. 3. A chair costs Rs. 341 and (C) 12·4 kg. (D) 18·0 kg.
a table costs Rs. 852. What will be 7
= Work of × 8
the approximate cost of 5 dozens of 4 6. Cost of 1 chair is Rs. 214 and
chairs and 4 dozens of tables ? = 14 women cost of one table is Rs. 937, then

Quantitative Aptitude Test | 53


More PDF download : http://www.gksolve.com

what is the approximate cost of 6 (A) Rs. 430 (B) Rs. 540 Rs. 805. What is the cost at per
dozen chairs and 4 dozen tables? (C) Rs. 380 (D) Rs. 400 thousand bricks ?
(A) Rs. 60000 (A) Rs. 900
8. Five dozen toys are packed in a
(B) Rs. 58000 box and 98 boxes are kept in a (B) Rs. 800
tempo. How many tempos can (C) Rs. 935
(C) Rs. 55000
lift 29400 toys in one round ? (D) Rs. 750
(D) Rs. 62000
(A) 4 (B) 5 10. 357 mangoes cost Rs. 1517·25,
1 (C) 7 (D) 6 then what is the approximate cost
7. Cost of dozen of bananas is
4 of 49 dozen of mangoes ?
Rs. 2·35, then what is the 9. Cost of 7000 bricks is Rs. 5740.
Cost of luggage to carry the (A) Rs. 2500 (B) Rs. 2600
approximate cost of 42 dozen of
bananas ? bricks to the building place is (C) Rs. 3000 (D) Rs. 2200

Answers with Hints


192 ∴ Total cost
1. (D) No. of pieces = = 60
3·2 = 15408 + 44976
2. (C) Let the cost of the table be Rs. x and the cost of = Rs. 15408 + 44976 = Rs. 60384
the chair = Rs. y ∼
– Rs. 6000 (Approximately)
From Ist condition, 2x = 5y 7. (D) ˙·˙ 1 dozen = 12
5y 1 1
x = ⇒ dozen = × 12 = 3
2 4 4
From IInd condition, ˙.˙Cost of 3 bananas = Rs. 2·35
x – y = Rs. 1200 2·35
⇒ Cost of 1 banana = Rs.
5y 3
– y = 1200
2 ∴ Cost of 42 × 12 bananas
3y 2·35 × 42 × 12
= 1200 =
2 3
y = Rs. 800 = Rs. 394·80
3. (B) ˙·˙ 1 kg = 1000 grams = Rs. 400 (Approximately)
1 1 8. (B) ˙·˙ Five dozen = 5 × 12 = 60
⇒ kg = 1000 × = 250 grams
4 4 ⇒ No. of toys can be kept in 1 box = 60
˙.˙ The cost of 250 grams = 60 paise ∴ No. of toys can be kept in 98 boxes
60
∴ The cost of 200 grams = × 200 = 60 × 98 = 5880
250
= 48 paise ˙.˙ 5880 toys can be lifted by = 1 tempo
4. (C) Three dozen = 3 × 12 = 36 1
∴ 29400 toys can be lifted by = × 29400
˙.˙ The cost of 36 mangoes = Rs. 245 5880
245 = 5 tempos
∴ The cost of 1 mango = Rs.
36 9. (C) ˙·˙ Total cost of 7000 bricks
245 = 5740 + 805 = Rs. 6545
∴ The cost of 363 mangoes = Rs. × 363
36 6545
= Rs. 2470·42 ⇒ Total cost of 1 brick = Rs.
7000

– Rs. 2400. 6545
(Approximately) ∴ Total cost of 1000 bricks = × 1000
7000
5. (B) ˙.˙ Weight of 13 m long rod = 23·4 kg = Rs. 935
23·4
∴ Weight of 1 m long rod = kg 10. (A) 49 dozen = 49 × 12 = 588
13
˙.˙ The cost of 357 mangoes = Rs. 1517·25
23·4 × 6
∴ Weight of 6 m long rod = ⇒ The cost of 1 mango
13
= 10·8 kg 1517·25
= = Rs. 4·25
6. (A) ˙.˙ Cost of 6 dozen chairs 357
= 6 × 12 × 214 = Rs. 15408 ∴ The cost of 588 mangoes
and cost of 4 dozen tables = 4·25 × 588 = Rs. 2499
= 4 × 12 × 937 = Rs. 44976 –∼ Rs. 2500 (Approximately)

Quantitative Aptitude Test | 54


More PDF download : http://www.gksolve.com

Square Root and Cube Root


Square Root of a Number— 9720 = 2 × 2 × 2 × 3 × 3 × 3 ⎯√⎯⎯⎯
⎯√⎯1323
1323
The square root of a number is that ×5×3×3 (ii) =
⎯⎯75
√ 75
number, the product of which by
To make it a perfect cube, the
itself, is equal to the given number. ⎯⎯441

The square root of x is denoted
given number must be multiplied by
5 × 5 × 3 i.e., 75.
=
⎯⎯441
√ 25
=
⎯√⎯25
by √
⎯ x. 21
Cube root of the new number =
5
Thus, ⎯√ 9 = 3, √
⎯⎯16 = 4, ⎯⎯⎯⎯
√ 10000 = 2 × 3 × 5 × 3 = 90
= 100 etc. Q. 6. Evaluate √
⎯⎯·4 upto four
Q. 3. If √
⎯⎯⎯⎯
1369 = 37, find the places of decimal.
Square Root by Factoriza-
value of Solution : Making even number
tion—When a given number is a
perfect square we resolve it into prime of decimal places, we have
⎯⎯⎯⎯
√ 13·69 + ⎯
√⎯⎯⎯
·1369 + ⎯
√⎯⎯⎯⎯⎯
·001369
factors and take the product of prime ⎯⎯·4 = √
√ ⎯⎯·40.
factors. Choosing one out of every Solution : Given expression
6 00 ·40 (·6324) 0
pair.
⎯⎯ √
√ ⎯⎯1369 1369 00036
= +
Examples 100 10000 123 00400

⎯√⎯⎯
1369 00369
Q. 1. Given that √
⎯⎯15 = 3·8729, +
1000000 1262 03100
⎛√
⎯ 5 + ⎯√ 3 ⎞⎟ .
evaluate ⎜⎜ ⎯√⎯⎯⎯
1369 √⎯⎯⎯ 02524
⎯ 3⎟⎠
1369
⎝√
⎯5– √ = + 12644 57600
⎯⎯⎯
√ 100 √⎯⎯⎯⎯
10000
50576
Solution :
(√⎯ 5 + ⎯√ 3) +
⎯√⎯⎯⎯
1369
(√⎯ 5 – √⎯ 3) ⎯⎯⎯⎯⎯⎯
√ 1000000
∴ ⎯⎯·4 = 0·6324.

=
(√⎯ 5 + ⎯√ 3) × (√⎯ 5 + ⎯√ 3) = (
37 37
+ +
10 100 1000
37
) Cube Root—The cube root of a
number x is the number whose cube
(√⎯ 5 – √⎯ 3) (√⎯ 5 + ⎯√ 3) = (3·7 + 0·37 + 0·037) = 4·107 is x.
Q. 4. If √
⎯⎯21 = 4·582 find the We denote the cube root of x by
(√⎯ 5 + ⎯√ 3) 2
3

⎯√
=
(5 – 3) value of
3
. ⎯√ x.
7 Cube Root by Factorization—
5+3+2×√⎯ 5 × ⎯√ 3 Resolve the given number into prime
= Solution :
2 factors and take the product of prime
= (4 + √⎯⎯15) ⎯√ 37 = √⎯√⎯ 37 = √⎯√⎯ 37 × ⎯√⎯√77 numbers, choosing one out of three
of each type.
= (4 + 3·8729) = 7·8729. 3 3
⎯ 3 × ⎯√ 7 = √
√ ⎯⎯21 Thus, ⎯8 = √
√ ⎯⎯⎯⎯⎯⎯
2×2×2=2
Q. 2. By what least number =
7 7
should 9720 multiplied to get a 3 3
perfect cube. Find the cube root of =
4·582
= 0·6546 and ⎯⎯⎯
√ 343 = √
⎯⎯⎯⎯⎯⎯
7×7×7=7
the number so obtained. 7
Q. 5. Evaluate Exercise
Solution : We have
(i) √
⎯⎯⎯
486 × ⎯√6

⎯ ⎯
2 9720 54 x
\1. If = , then x is equal
169 39
2 4860 √
⎯⎯⎯⎯
1323 to—
(ii) .
2 2430 ⎯⎯75
√ (A) 108 (B) 324
3 1215
Solution : (i) √
⎯⎯⎯
486 × ⎯
√6 (C) 2916 (D) 4800
3 0405
3 0135 = ⎯√⎯⎯⎯⎯
486 × 6 112 ⎯√⎯⎯
576 ⎯√⎯256 = ?
\2. × ×
5 0045 = √⎯⎯⎯⎯⎯⎯⎯
6 × 81 × 6 ⎯⎯⎯
√ 196 12 8
3 0009 = √⎯⎯⎯⎯⎯⎯⎯⎯⎯
6 × 6 × 9 × 9) (A) 8 (B) 12
0003 = (6 × 9) = 54 (C) 16 (D) 32

Quantitative Aptitude Test | 55


More PDF download : http://www.gksolve.com

⎯⎯⎯⎯⎯⎯⎯⎯⎯⎯
√ ⎯⎯⎯⎯

248 + √
⎯⎯52 + ⎯√⎯⎯
⎯√⎯⎯
\3. 144 — 14. 64009 = ? ·289
23. =?
(A) 14 (B) 16 (A) 803 (B) 363 ·00121
(C) 16·6 (D) 18·8 (C) 253 (D) 347 17
(A) 1·7 (B)
11
⎯⎯⎯⎯⎯⎯⎯
√ 176 + √
⎯⎯⎯⎯
⎯√ 43 – √
⎯ 34 = ?
\4. 2401 = ? 170 17
15. (C) (D)
(A) 14 (B) 15 11 10
(C) 18 (D) 24 1 1
(A) (B) – 24. If ⎯√ 3 = 1·732 and √
⎯ 2 = 1·414,
\5. Given that √
⎯⎯⎯⎯
4096 = 64, the value 2√
⎯3 2√
⎯3 1
the value of is—
of ⎯
√⎯⎯⎯
4096 + √
⎯⎯⎯⎯
40·96 + √
⎯⎯⎯⎯⎯⎯
·004096 5√
⎯3 ⎯ 3 + ⎯√ 2

is— (C) 1 (D)
3 (A) 0·064 (B) 0·308
(A) 70·4 (B) 70·464 (C) 0·318 (D) 2·146
(C) 71·104 (D) 71·4 16. If ⎯
√ 2 = 1·4142, then the approx-
25. If ⎯
√ 6 = 2·55, then the value of


2
\6. ⎯⎯·04 = ?
√ imate value of is—
⎯ √
√ ⎯
9 2 3
(A) ·02 +3 is—
(A) 0·2321 (B) 0·4714 3 2
(B) ·2
(C) 0·3174 (D) 0·4174 (A) 4·48
(C) ·002
(B) 4·49
(D) None of these
17. If √
⎯ 3 = 1·732, then the approx- (C) 4·50
\7. If ⎯
√⎯⎯
256 ÷ ⎯
√ x = 2, then x is equal imate value of
1
is— (D) None of these
to— ⎯3

(A) 64 (B) 128 26. If √
⎯⎯2n = 64, then the value of n
(A) 0·617 (B) 0·313
(C) 512 (D) 1024 is—
(C) 0·577 (D) 0·173 (A) 2 (B) 4
⎯√⎯⎯
288 (C) 6 (D) 12
08. =? 18. If ⎯
√⎯24 = 4·899, then the value of
⎯⎯⎯
√ 128

√ ⎯⎯15625
√ ⎯⎯30625

8 25 ?
is— 27. =
⎯3
√ 3 3
(A) (B)
2 ⎯2
√ (A) 0·544 (B) 2·666 (A) 2 (B) 35
(C) 1·633 (D) 1·333 (C) 49 (D) 1225
⎯√ 32
3
(C) (D)
2 ⎯√⎯32 + ⎯√⎯48 = ?
09. ⎯⎯10 × ⎯√⎯15 = ?

19.
⎯ 8 + ⎯√⎯12

28.
⎯√⎯1169 = ?
3
(A) 5√⎯6 (B) 6√
⎯5 (A) √⎯2 (B) 2 (A) 1
4
(C) 5 (D) √⎯⎯30 (C) 4 (D) 8
1
(B) 1
? 4
⎯ ? = 0·02—
√ 20. = 550
10.
200 ⎯⎯⎯
√ 2·25 (C) 1·125
(A) 0·4 (B) 4 (A) 825 (B) 82·5 (D) None of these
(C) 16 (D) 1·6 (C) 3666·66 (D) 2 1872
29. = 234
11.
250
= 10— 21. If √
⎯⎯⎯⎯⎯⎯⎯⎯
(75·24 + x) = 8·71, then the ⎯?

⎯?
√ value of x is— (A) 324 (B) 64
(A) 25 (B) 250 (A) ·6241 (C) 8 (D) 256
(C) 625 (D) 2500 (B) 6·241
⎯⎯⎯
√ 324 ?
(C) 62·41 30. =
12. ⎯⎯10 × ⎯√⎯⎯
√ 250 = ? 1·5 ⎯⎯⎯
√ 256
(D) None of these
(A) 46·95 (B) 43·75 (A) 192 (B) 432

⎯⎯

(C) 50·25 (D) 50 36·1 (C) 288 (D) 122
22. =?
102·4
⎯⎯⎯⎯
√ 4375
⎯√⎯⎯⎯
13. =? 29 19 1·21 × 0·9
(A) (B) 31. =?
⎯7
√ 32 72 1·1 × 0·11
(A) 24·75 (B) 27·25 19 29 (A) 2 (B) 3
(C) (D)
(C) 25 (D) 35 32 62 (C) 9 (D) 11

Quantitative Aptitude Test | 56


More PDF download : http://www.gksolve.com

(3 – 2√ 2)
⎯⎯⎯⎯⎯⎯⎯⎯⎯⎯
√ (C) √
⎯⎯⎯
0·324 × 0·081 × 4·624 0·18 (C)
32.
1·5625 × 0·0289 × 72·9 × 64 (D) None of these
=? (D) (3 + 2 √
 2)
(A) 24 42. ⎯⎯⎯⎯⎯
√ 0·0009 ÷ ⎯
√⎯⎯
0·01 = ?
√ 5 – √ 3 is equal to—
(B) 2·40 (A) 3 49.
(C) 0·024 (B) 0·3  5 + √ 3

(D) None of these 1 (A) 4 + √ 15 (B) 4 – √15
(C)
3 1
33. If ⎯
√⎯⎯⎯
15625 = 125, then the value (C)
2
(D) 1
(D) None of these
of [√⎯⎯⎯⎯
15625 + ⎯
√⎯⎯⎯⎯
156·25
43. Which of the following numbers, 50. The least perfect square number
+√⎯⎯⎯⎯⎯
1·5625 is ] where in some of the digit have divisible by 3, 4, 5, 6, 8 is—
(A) 900 (B) 1200
been suppressed by Symbols, can
(A) 1·3875 (B) 13·875
possibly be the perfect square of (C) 2500 (D) 3600
(C) 138·75 (D) 156·25 a 3 digit odd number ?
24 + √
√ 216
⎯⎯⎯⎯
√ 1296 ? (A) 65×××1 (B) 9××1 51. =?
34.
?
=
2·25 (C) 10×××4 (D) 9××××××5 96

(A) 6 (B) 7 (A) 2√6 (B) 6
√2
(C) 8 (D) 9 44. ⎯⎯⎯⎯⎯⎯⎯⎯⎯⎯
√ (12 + √⎯⎯⎯⎯⎯⎯⎯⎯
12 + √ 12 + … ) = ?
⎯⎯⎯⎯⎯ (C) 2 (D)
2
6


⎯⎯⎯⎯
(A) 3
35. If (1 + 169
27
) = (1 + 13x ), (B) 4 52. The least number to be sub-
then x equals— (C) 6 tracted from 16800 to make it a
perfect square is—
(A) 1 (B) 3 (D) Greater than 6
(A) 249 (B) 159
(C) 5 (D) 7
45. If ⎯
√⎯⎯⎯⎯⎯⎯⎯⎯⎯
0·04 × 0·4 × a = 0·4 × 0·04 × (C) 169 (D) 219
a

⎯⎯⎯⎯⎯
√ ⎯ b, then the value of b is—
√ 53. The least number by which 216
36. ⎯√⎯⎯⎯
2⎯√⎯⎯⎯
⎯√⎯⎯ 2 2 2⎯ √2 = ? (A) 0·016
must be divided to make the
result a perfect square is—
(A) 0 (B) 1 (B) 1·60 (A) 3 (B) 4
(C) 2 (D) 231/32 (C) 0·16 (C) 6 (D) 9
(D) None of these
37. The value of ⎯
√⎯0·9 is— 54. The least number by which 176
 1 1 1  be multiplied to make the result
(A) 0·3 (B) 0·03 46.  3 + + +
 3 – 3 
a perfect square is—
(C) 0·33 (D) 0·94  √ 3 3+ √
3 √ (A) 8 (B) 9
equals—
38. If ⎯
√⎯⎯⎯
2401 = √
⎯⎯7x, then the value (C) 10 (D) 11
of x is— (A) 0 (B) 1
55. What smallest number must be
(A) 3 (B) 4 (C) 3 (D) 3 + √
3 added to 269 to make it a perfect
(C) 5 (D) 6 square ?
√ 5 + 1 and b = √
 5 – 1,
47. If a = (A) 31 (B) 16
39. If √⎯ 2 = 1·4142, the value of 5–1
√ √ 5 + 1 (C) 7 (D) 20
7 a2 + ab + b2
is— then the value of 56. The smallest number of 4 digits
3+√⎯2 a2 – ab + b2
is— which is a perfect square is—
(A) 1·5858 (B) 4·4142
3 4 (A) 1000 (B) 1016
(C) 3·4852 (D) 3·5858 (A) (B)
4 3 (C) 1024 (D) 1036
40. ⎯⎯⎯⎯⎯⎯⎯
√ ·00059049 = ? (C)
3
(D)
5 57. The largest number of 5 digits,
(A) ·243 5 3 which is a perfect square is—
(B) ·0243 (A) 99999 (B) 99764
1
48. =? (C) 99976 (D) 99856
(C) ·00243 9– √
√ 8
√(B)0·16
(D) ·000243
1
(A) (
3 – 2√
2 ) 58. The value of
0·4
is—
41. ⎯⎯⎯⎯⎯⎯⎯⎯
√ 0·01 + ⎯
√⎯⎯⎯⎯
0·0064 = ?
2
(A) 0·2 0·02
1
(A) 0·3 (B) 2
√5
(B) 0·03 3 + 2√
2 (C) 0·63 (D)
5

Quantitative Aptitude Test | 57


More PDF download : http://www.gksolve.com

them are left over. How many


59. The value of ⎯
√⎯⎯⎯
0·064 is— 3
⎯⎯91
√ are left ?
(A) 0·8 (B) 0·08 (C) 1 –
6 (A) 36
(C) 0·008 (D) 0·252 (D) None of these (B) 65
60. The value of ⎯
√⎯⎯⎯
0·121 is— 3 (C) 81
⎯√⎯4125
12
(A) 0·11 (B) 1·1 64. =? (D) 97
(C) 0·347 (D) 0·011 3 2
(A) 1 (B) 1 68. A group of students decided to
61. What is the smallest number by 5 5 collect as many paise from each
which 3600 be divided to make 2 4 member of group as is the
it a perfect cube ? (C) 2 (D) 1
5 5 number of members. If the total
(A) 9 (B) 50 collection amounts to Rs. 22·09,
65. The cube root of ·000027 is—
(C) 30 (D) 450 the number of members in the
(A) ·3 group is—
62. By what least number must (B) ·03
(A) 37
21600 be multiplied to make it a (C) ·003
perfect cube ? (D) None of these (B) 47
(A) 6 (B) 10 66. The length of diagonal of a (C) 107
(C) 30 (D) 60 square is 8 cm. The length of the (D) 43
side of the square is— 69. A gardener wants to plant 17956
3

⎯⎯⎯
√ 91 (A) 2 cm (B) 2·8 cm trees and arranges them in such a
63. 1– =?
216 (C) 1·414 cm (D) 5·64 cm way that the are as many rows as
5 67. A general wishes to draw up his there are treres in a row. The
(A) 1 – number of trees in a row is—
6 36562 soldiers in the form of a
5 solid square. After arranging (A) 144 (B) 136
(B) (C) 154 (D) 134
6 them, he found that some of

Answers with Hints

⎯√⎯ ⎯√⎯1004 = √⎯√⎯⎯⎯1004 = 102 = ·2.


x 54 x 54 54
1. (B)
169
=
39
⇒ = ×
169 39 39 6. (B) ⎯⎯·04 =

∴ x = (
54 54
× × 169 = 324
39 39 ) ⎯√⎯⎯
256
= 2 or 16 = 2 ⎯
7. (A) √ x or ⎯√ x = 8 or x = 64.
⎯x

2. (D) Given expression = ( 112 24 16
× ×
14 12 8
= 32. )
⎯√⎯⎯
⎯√ 94 = 32
288
3. (B) ⎯⎯⎯⎯⎯⎯⎯
√ 248 + √
⎯⎯⎯⎯⎯⎯
52 + √
⎯⎯⎯
144 = ⎯√⎯⎯⎯⎯⎯⎯⎯
248 + √
⎯⎯⎯⎯⎯
52 + 12
8. (C)
⎯⎯⎯
√ 128
=

= ⎯⎯⎯⎯⎯⎯
√ 248 + √
⎯⎯64 9. (A) ⎯⎯10 × ⎯√⎯15 = √
√ ⎯⎯⎯⎯⎯
10 × 15 = ⎯
√⎯⎯
150
= ⎯⎯⎯⎯⎯
√ 248 + 8 = ⎯
√⎯⎯
256
= ⎯⎯⎯⎯
√ 25 × 6 = ⎯⎯
√25 × ⎯√ 6 = 5·⎯√ 6
= 16
⎯ x = 0·02, then x = 200 × 0·02 = 4.

4. (B) √
⎯⎯⎯⎯⎯⎯⎯
176 + √
⎯⎯⎯⎯
2401 = ⎯⎯⎯⎯⎯⎯
√ 176 + 49 10. (C) Let
200 ⎯√
= ⎯⎯⎯
√ 225 = 15 So, x = 16
5. (B) √
⎯⎯⎯⎯
4096 + ⎯
√⎯⎯⎯
40·96 + ⎯
√⎯⎯⎯⎯
·004096 250
11. (A) Let = 10.
⎯x

⎯√⎯4096
100 √ ⎯⎯⎯
4096
= ⎯⎯⎯⎯
√ 4096 + +
1000000 250
Then ⎯ x = 10 = 25

⎯√⎯⎯⎯
4096 √
⎯⎯⎯⎯
4096
= ⎯⎯⎯⎯
√ 4096 + +
⎯⎯100 √
√ ⎯⎯⎯⎯⎯⎯
1000000 12. (D) ⎯⎯10 × ⎯√⎯⎯
√ 250 = √
⎯⎯⎯⎯⎯⎯
10 × 250 = ⎯⎯⎯⎯
√2500 = 50
64 64
= 64 + + = 64 + 6·4 + ·064 ⎯⎯⎯⎯

⎯√⎯4375
10 1000 4375
13. (C) = =√
⎯⎯625 = 25
⎯7
√ 7
= 70·464

Quantitative Aptitude Test | 58


More PDF download : http://www.gksolve.com

14. (C) 2·55 7·65 5·10 + 22·95


= + =
2 –6 40
— — (253 3 2 6
09
4 28·05
= = 4·675
45 240 6
225 26. (D) ⎯⎯2n = 64 = 2 6 ⇒ 2n/2 = 26

503 01509 n
01509 ⇒ = 6
2
× ⇒ n = 12
∴ ⎯√⎯⎯⎯
⎯⎯15625
√ ⎯x⎯30625

64009 = 253 25 x
27. (C) Let =
⎯4–√
√ ⎯3 = 2 –√ ⎯ 3 = 4 – 3= 1
15. (A) 25
⎯3 √
√ ⎯4 ⎯ 3 2 2√
√ ⎯ 3 2√ ⎯3 Then, =
15625 30625
⎯2= √
√ ⎯ 2 = 1·4142 = 0·4714
⎯√ 2 1 x
16. (B) = ⇒ =
9 ⎯9 3
√ 3 625 30625
30625
1 1 ⎯ √3 = √ ⎯ 3 = 1·732 = 0·577 ∴ x = = 49
17. (C) = × 625
⎯3
√ ⎯3 √
√ ⎯3 3 3

18. (C)
⎯√ 8
3
=
⎯ 8 × ⎯√ 3 = √
√ ⎯⎯24 = 4·899 = 1·633 28. (B)
⎯ 2516 = √⎯√⎯⎯⎯2516 = 54 = 114
⎯√⎯1169 = √
⎯3 √
√ ⎯3 3 3
1872
29. (B) Let = 234.
⎯⎯32 + ⎯√⎯48 = √
√ ⎯⎯⎯⎯
16 × 2 + ⎯
√⎯⎯⎯
16 × 3 ⎯x

19. (B)
⎯ 8 + ⎯√⎯12
√ ⎯⎯⎯
√ 4 × 2+ ⎯
√⎯⎯
4×3 1872
Then, ⎯ x = 234 = 8

⎯ 2 + 4⎯√ 3 = 4(√
4√ ⎯ 2 + ⎯√ 3) ∴ x = (8 × 8) = 64
= =2
⎯ 2 + 2⎯√ 3 2(√
2√ ⎯ 2 + ⎯√ 3) ⎯⎯⎯
√ 324 x
x 30. (A) =
20. (A) Let = 550. 1·5 ⎯⎯⎯
√ 256
⎯⎯⎯
√ 2·25 18 x
x ⇒ =
Then, = 550 1·5 16
1·5 18 × 16
∴ x = (550 × 1·5) = ( 550 × 15
10 ) = 825
∴ x =
1·5 (18 × 1516 × 10) = 192
=

⎯⎯⎯

21. (A) 75·24 + x = 8·71 × 8·71 121 × 9
31. (B) Given expression = =√⎯9=3
x = 75·8641 – 75·24 11 × 11

⎯⎯⎯⎯⎯⎯⎯⎯

⇒ x = ·6241 324 × 81 × 4624
32. (C) Given expression =
15625 × 289 × 729 × 64
⎯⎯361 = 19

⎯√⎯102·4 ⎯⎯1024

36·1 361
22. (C) = = (Sum of decimal places being equal in Num. and
⎯⎯⎯⎯
√ 1024 32
Denom.)
18 × 9 × 68
⎯√⎯⎯ ⎯⎯⎯
√ ·00121 √ ⎯⎯28900
·289 ·28900 3
23. (C) = = = =
·00121 121 125 × 17 × 27 × 8 125
= 0·024
⎯√⎯⎯⎯
28900 170
= = 33. (C) Given expression
⎯⎯⎯
√ 121 11

24. (C)
1
=
1 (√⎯ 3 – √⎯ 2)
×
= √ ⎯√⎯15625
⎯⎯⎯⎯
15625 +
100
+

⎯⎯15625
10000
⎯ 3 + ⎯√ 2
√ (√⎯ 3 + ⎯√ 2) (√⎯ 3 – √⎯ 2) = (125 +
10 100)
125 125
+
⎯3– √
√ ⎯ 2 = (√
= ⎯3– √⎯ 2) = (125 + 12·5 + 1·25) = 138·75
3–2
= (1·732 – 1·414) = 0·318 ⎯⎯⎯⎯
√ 1296 x
34. (D) Let =
x 2·25
25. (D)
⎯√ 23 + 3 ⎯√32 = √⎯√⎯ 23 × ⎯√⎯√33 + 3√⎯√⎯ 32 × ⎯√⎯√22 Then,
36 x 225
x = 2·25 or x = 36 × 100
2

⎯ 6 + 3√
√ ⎯ 6 = 2·55 + 3 × 2·55
⎯√⎯⎯
36 × 225 6 × 15
= ∴ x = = =9
3 2 3 2 100 10

Quantitative Aptitude Test | 59


More PDF download : http://www.gksolve.com

⎯√⎯⎯⎯⎯
0·016 a = 0·016 × √
⎯b
⎯⎯⎯⎯

45. (A) ˙·˙
35. (A) (1 + 169
27
) = (1 + 13x ) ⎯ a = 0·016 = √

⇒ ⎯⎯⎯⎯
0·016

⎯⎯196
√ = (1 + ) ⎯b
√ ⎯⎯⎯⎯

x 0·016

169 13


a a
b = √
Thus, ∴ ⎯⎯⎯⎯
0·016 and so b = 0·016
= (1 + )
14 x

13 13
1 1 1
46. (C) Given Exp. = 3 + + +
= ( – 1) =
x 14 1

13 13 13 ⎯3 3+ √
√ ⎯3 √ ⎯3–3
1 ⎯√ 3 + 1 × 3 – √
⎯ 3 + 1 × ⎯√ 3 + 3
x = (13 × ) = 1
1 =3+ ×

13 ⎯3 √
√ ⎯3 3+ √ ⎯3 3– √
⎯3 √ ⎯3– 3 √
⎯3+3
⎯ 3 + 3 –√
√ ⎯3+ √
⎯3+3
36. (D) Given expression = ⎯⎯⎯⎯⎯⎯⎯
√ ⎯⎯⎯⎯⎯⎯⎯
√ ⎯√⎯⎯⎯⎯⎯
2× ⎯⎯⎯⎯⎯⎯
√ 2× 2 (2 × 21/2)
=3+
3 6 –6
18 + 2√
⎯ 3 + 3 –√
⎯3– ⎯ 3 – 3 = 18 = 3

= ⎯⎯⎯⎯⎯⎯⎯
√ 2 ×√
⎯⎯⎯⎯⎯⎯⎯
2⎯
√⎯⎯⎯⎯⎯
(2 × 2 ) 3/4 =
6 6

⎯√ 5 + 1 × ⎯√ 5 + 1 = (⎯√ 5 + 1)
2
= ⎯⎯⎯⎯⎯⎯⎯
√ 2 ×⎯
√⎯⎯⎯⎯
2 × 27/8
47. (B) a =
⎯5–1 √
√ ⎯5+1 (5 – 1)
⎯⎯⎯⎯⎯⎯
√ 2 × 215/16 = 2 31/32
=
90 √ ⎯⎯90 = 9·4 = 0·94 √ 5 = ⎛⎜ 3 + ⎯√ 5⎞⎟
5 + 1 + 2⎯
37. (D) √
⎯⎯0·9 = √
⎯⎯⎯
0·90 = =
100 10⎯√ 10
=
4 ⎝ 2 ⎠
⎯√ 5 – 1 × ⎯√ 5 – 1 = (⎯√ 5 – 1 )
2
38. (B) ⎯⎯⎯⎯
√ 2401 = √
⎯⎯ ⇒ 7x = 2401 = 74 ⇒ x = 4
7x
b =
⎯5+1 √
√ ⎯5–1 (5 – 1)
7 7 (3 –√
⎯ 2)
39. (A) = ×
3+2 (3 + √⎯ 2) (3 – √
⎯ 2) =
5 + 1 – 2√
⎯5= 3–√ ⎯5
4 2
7(3 – √⎯ 2)
=
7 a2 + b2 =
(3 + √⎯ 5) + (3 – √⎯ 5)
2 2

4
= 3–√
⎯ 2 = (3 – 1·4142) = 1·5858 2 (9 + 5)
= = 7 and ab = 1
4
⎯⎯⎯⎯
√ 59049
40. (B) ⎯⎯⎯⎯⎯⎯⎯
√ ·00059049 = a2 + ab + b2 7+1 8 4
100000000 ∴ 2 = = =
a – ab + b2 7–1 6 3
⎯⎯⎯⎯
√ 59049 243 1
= = = 0·0243 48. (D) Exp. =
10000 10000
⎯9– √
√ ⎯8
41. (A) Given expression = ⎯⎯⎯⎯⎯⎯⎯⎯
√ 0·01 + 0·08
1 ⎯√ 9 + ⎯√ 8 = 3 + 2 √
⎯2
= ×
= ⎯⎯⎯
√ 0·09 = 0·3 ⎯9– √
√ ⎯8 √ ⎯ 9 + ⎯√ 8 9 – 8

⎯√⎯⎯⎯⎯ = (3 + 2 √
⎯ 2)
⎯√⎯0·0009
0·0009
42. (B) Given expression = =
⎯⎯⎯
√ 0·01 0·0100
⎯√ 5 – √
⎯3
49. (B) Exp. =

⎯9 (√⎯ 5 + ⎯√ 3)
⎯⎯
√ 9
= =
100 √⎯⎯⎯
100
3 =
(√⎯ 5 – √⎯ 3) = (√⎯ 5 – √⎯ 3) 2

= = 0·3
10 (√⎯ 5 + ⎯√ 3) (5 – 3)
43. (A) The square of an odd number cannot have 4 as
the unit digit. The square of a 3 digit number will 5 + 3 – 2√ ⎯⎯15 = 2(4 – √⎯⎯15)
=
have at least 5 digit and at the most 6 digits. So, 2 2
answer (A) is correct.
44. (B) Let given expression = x
= 4 –√ (
⎯⎯15 )
50. (D) L.C.M. of 3, 4, 5, 6, 8 is 120.
Then, ⎯√⎯⎯⎯
12 + x = x ⇒ 12 + x = x2 Now, 120 = 2 × 2 × 2 × 3 × 5
⇒ x2 – x – 12 = 0 ⇒ (x – 4) (x + 3) = 0 ∴ Required number = 2 × 2 × 2 × 3 × 5 × 2 × 3 × 5
So, x = 4 (neglecting x = –3) = 3600

Quantitative Aptitude Test | 60


More PDF download : http://www.gksolve.com

⎯⎯24 + ⎯√⎯⎯
√ ⎯√⎯⎯⎯
4 × 6 + ⎯⎯⎯⎯
√36 × 6 ⎯⎯⎯

⎯√⎯10000
216 640 640
51. (C) = 59. (D) ⎯⎯⎯⎯
√ 0·064 = √
⎯⎯⎯⎯⎯
0·0640 = =
⎯⎯96
√ ⎯⎯⎯⎯
√ 16 × 6 100
25·2
2√
⎯ 6 + 6⎯√ 6 =
100
= 0·252.
=
4√⎯6 1210 √ ⎯⎯⎯⎯
⎯√⎯10000 1210
60. (C) ⎯⎯⎯⎯
√ 0·121 = √
⎯⎯⎯⎯⎯
0·1210 = =
8√
⎯6=2 100
=
4√
⎯6 =
34·7
= 0·347.
100
52. (B)
–— — 61. (D) 3600 = 2 × 2 × 2 × 2 × 3 × 3 × 5 × 5.
1 16800 (129 To make it a perfect cube, the given number must be
1 divided by 2 × 3 × 3 × 5 × 5 = 450.
22 0 68 62. (B) 21600 = 6 × 6 × 6 × 10 × 10
0 48 To, make the given number a perfect cube, it must be
multiplied by 10.
249 002400
002241
1590
63. (
(B) 1 –
216)
91 1/3
= ( )125 1/3
216
5 × 5 × 5 1/3 5
∴ Required number to be subtracted = 159.
53. (C) 216 = 2 × 2 × 2 × 3 × 3 × 3
= ( )
6×6×6
=
6
512 1/3 (8 × 8 × 8)1/3
Clearly, in order to make it a perfect square, it must
be divided by 2 × 3 i.e., 6.
64. (A) ( )4
12 1/3
125
= ( )
125
=
(5 × 5 × 5)1/3
8 3
54. (D) 176 = 2 × 2 × 2 × 2 × 11 = =1
5 5
So, in order to make it a perfect square, it must be
( )
27 1/3 (3 3)1/3
multiplied by 11. 65. (B) (·000027)1/3 = =
1000000 (106)1/3
55. (D)
– –– 3
1 2 69 (16 = = ·03
100
1 66. (D) a2 + a2 = (8) 2 ⇒ 2a2 = 64 ⇒ a2 = 32
26 169
∴ a = ⎯⎯32 = 4⎯√ 2 = (4 × 1·41) = 5·64 cm

159
67. (C)
0013 –— —
∴ Required number to be added = (17)2 – 269 = 20. 1 36562 (191
56. (C) The smallest number of 4 digits = 1000 1
—— 29 265
3 1000 (31
261
09
381 00462
61 0100
00381
0061
00081
0039 ∴ Number of men left over
∴ Required number = (32)2 = 1024.
= 36562 – (191) 2
57. (D) The largest number of 5 digits = 99999
= 36562 – 36481
–— —
3 99999 (316 = 81.
9 68. (B) Number of members = ⎯
√⎯⎯⎯
2209 = 47.
61 099 69. (D)
061 –— —
1 17956 (134
626 03899 1
03756
23 079
00143 069
∴ Required number = (99999 – 143) = (99856).
264 01056
58. (C)
⎯√⎯0·16
0·4
=
⎯⎯0·16
√ 0·40 √
=
⎯ 1640 = √
⎯ 104 = √⎯⎯0·4 01056
00×
40 √ ⎯⎯40 = 6·3 = 0·63
⎯√100 ∴ Required no. of trees = 134
= √
⎯⎯⎯
0·40 = =
10 10 ∴ Number of trees in a row = 134.

Quantitative Aptitude Test | 61


More PDF download : http://www.gksolve.com

Simplification
In Simplifying an expression Q. 3. Simplify 3 49 1 5 2
= 1 ÷ of + × –
first of all vinculum or bar must be 7 10 5 7 7
1 1
removed. 4 –2 21 1 2
7 4 1 = 1÷ + –
Example. We know that : ÷ 10 7 7
1 1 1
– 6 – 8 = – 14 3 +1 2+
2 7 1 10 1 2 10 1 2
2+ = 1× + – = + –
⎯⎯ 1 21 7 7 21 7 7
But – 6 – 8 = – (–2) = 2 5–
After removing the bar, the
brackets must be removed strictly in Solution : Given expression
5
= (10 + 3 – 6
21 )=
7 1
21 3
=

the order ( ), { } and [ ]. 29 9 Q. 7. Simplify



After removing the brackets, we 7 4 1 1 ÷ [1 + 1 ÷ {1 + 1 ÷ (1 + 1 ÷ 3)}].
= ÷
must use the following operations, 7 8 1 Solution : Given expression
+ 2+
strictly in the order, given below— 2 7 5
(i) Of, (ii) Division, (iii) Multi-
plication, (iv) Addition and (v) Sub-
2+
24 [
= 1÷ 1+1÷ 1+1 { 1
2
traction. = ( 53 14
×
28 65 ) ÷
2+
1
24 ÷ 1+( )} ]
1
3
Remark—Remember the word 53
‘BODMAS’, where B, O, D, M, A
and S stand for bracket, Of, division, =
53
÷
130 130
53
= ( 53 130
130 53
× ) [
= 1÷ 1+1÷ 1+1÷ { }] 4
3
multiplication, Addition and Subtrac-
tion respectively. = 1 = 1÷ 1+1÷[ 7
4 ]
Strictly follow the above order Q. 4. Evaluate
of operations.
⎛ 0·47 × 0·47 + 0·35 × 0·35 ⎞

= 1÷ 1+ [ ]4
7
=1÷
11
7
– 2 × 0·47 × 0·35⎟⎟
Examples ⎜⎝ 7 7
0·12 ⎠ = 1× =
11 11
Q. 1. Evaluate Solution : Given expression Q. 8. Simplify
0·125 + 0·027
=
0·25 – ·15 + ·09
(0·47)2 + (0·35)2 – 2
× 0·47 × 0·35
[ { (
2– 3– 6– 5– 4–3 )}]
= Solution : Given expression
Solution : Given expression 0·12
(0·5)3 + (0·3)3 (0·47 – 0·35) 2 = 2 – [3 – {6 – (5 – 1)}]
= =
(0·5) – 0·5 × 0·3 + (0·3)2
2
0·12 = 2 – [3 – {6 – 4}]
a3 + b3 0·12 × 0·12 = 2 – [3 – 2] = (2 – 1) = 1
= 2 = = 0·12
a – ab + b2 0·12
(where a = 0·5 and b = 0·3) Exercise
Q. 5. Simplify
= (a + b) = (0·5 + 0·3) = 0·8 7+7+7÷7
3·5 ÷ ·7 of 7 + ·5 × ·3 – ·1 01. =?
Q. 2. Evaluate 5+5+5÷5
Solution : Given expression 1
·7541 × ·7541 × ·7541 = 3·5 ÷ 4·9 + ·15 – ·1 (A) 1 (B)
5
– ·2459 × ·2459 × ·2459
3·5 15 3
·7541 × ·7541 + ·7541 = + 0·15 – ·1 (C) (D)
× ·2459 + ·2459 × ·2459 4·9 11 11
Solution : Given expression 5 3 1 (272 – 32) (124 + 176)
= + – 02. =?
7 20 10 17 × 15 – 15
a3 – b3
= 2
a + ab + b2 = (100 + 21 – 14
140 )
=
107
140
(A) 0
(B) 2·25
(where a = ·7541 and b = ·2459)
Q. 6. Simplify (C) 300
(a – b) (a2 + ab + b2)
=
(a2 + ab + b2)
3
(
1 ÷ of 2 + 2
7
3
10
3
5 )1
+ ÷1 –
5
2 2
5 7
(D) None of these
69 – 14 × 3 + 2
= (a – b) 03. =?
Solution : Given expression 9 × 5 – (5)2
= 0·7541 – 0·2459
= 0·5082
3
= 1 ÷ of
7 (
23 13
+
10 5 )
1 7 2
+ ÷ –
5 5 7
(A) 1·45
(C) 26·5
(B) 2·75
(D) 265

Quantitative Aptitude Test | 62


More PDF download : http://www.gksolve.com

48 – 12 × 3 + 9 1 1 1 1
04. =? 17. 3 + 4 + ? + = 10 (C) 4
12 – 9 ÷ 3 4 6 4 30
(A) 3 (B) 21 1 1 (D) None of these
(A) 2 (B) 4
7 1 6 3
(C) (D) 1 1
3 3 1 1 –
(C) 1 (D) 2 3 1 2 2 3 1 5
3 3 26. ÷ 2 of – ×3 + =?
05. ·01 × ·3 ÷ ·4 × ·5 = ? 4 4 3 1 1 3 6
+
(A) ·015 (B) ·0375 18. The value of 2 3

06.
(C) ·00375 (D) ·1
·05 × 5 – ·005 × 5 = ?
48 ÷ 12 ×( 9 4 3 2
of ÷ of
8 3 4 3 ) is— (A)
7
18
(B)
49
54
(A) 2·25 (B) ·225 1 1 2 1
(A) 1 (B) 5 (C) (D)
(C) 2·025 (D) ·29875 3 3 3 6
07. 4 – 3·6 ÷ 4 + 0·2 × 0·5 = ? (C) 3 (D) 12
5 6 8 3 3 1 7
(A) 3·2 (B) ·2 19. 10 – [9 – {8 – (7 – 6)}] – 5 is 27. 6 ÷ 7 × ? – 9 ÷ 1 5 + 4 × 3 3 = 2 9
(C) 1·65 (D) ·15 equal to—
7
08. 0·5 × 0·5 + 0·5 ÷ 5 = ? (A) –5 (B) 1 (A)
6
(A) 0·15 (B) 0·25 (C) 3 (D) 9
6

09.
(C) 0·35 (D) 0·45
8 ÷ 4 (3 – 2) × 4 + 3 – 7 = ?
[ {
20. 3 ÷ (8 – 5) ÷ (4 – 2)
2
1 (B)
7
(C) 1
(A) – 3
(C) 4
(B) – 4
(D) 5 ( )} ]
÷ 2+
8
13
=? (D) None of these
10. (20 ÷ 5) ÷ 2 + (16 ÷ 8) × 2 + (10 13 68 2 1 1 7
(A) (B) 28. 15 × 3 + 6 = 11 + ?
÷ 5) × (3 ÷ 2) = ? 17 13 3 6 3 18
(A) 9 (B) 12 17 13
(C) (D) 5
(C) 15 (D) 18 13 68 (A) 39
9
11. 3120 ÷ 26 + 13 × 30 = ?
(A) 2400
{ ( )}
21. 1 + 1 ÷ 1 + 1 ÷ 1 +
1
3
=?
(B) 137
4
9
(B) 3900 1 4
(A) 1 (B) 1 7
(C) 536 3 7 (C) 29
1 2 9
(D) None of these (C) 1 (D) 1
8 3 (D) None of these
12. 7+7÷7×7=?
3 3 1
(A)
2
7
(B) 14 22. 2 of ÷ + = ?
4 4 4
29. (4·59 × 1·8 ÷ 3·6 + 5·4 of 19 – 15) = ?
1 3 5 (A) 3·015
(C) 7 (D) 42 (A) (B)
7 2 2 (B) 2·705
8 9
13. 5005 – 5000 ÷ 10·00 = ? (C) (D) (C) 2·695
3 4
(A) 0·5 (B) 50 (D) None of these
(C) 5000 (D) 4505 1 1 1
23. + ÷ = ?
2 2 2 2 2 2
14. 171 ÷ 19 × 9 = ? 30. 4 ÷1 ×2 =?
1 17 5 33
(A) 0 (B) 1 (A) 2 (B)
2 2 2
(C) 18 (D) 81 3 3 (A) 4 (B) 6
(C) (D) 33 33
1 2 3 2 4
15. 108 ÷ 36 of + ×3 =? 1 1
3 5 4 17·28 ÷ ? (C) 6 (D) 8
24. =2 11 11
3 1 3·6 × 0·2
(A) 8 (B) 6
4 4 (A) 120 (B) 1·20 31 3 7
31. × + ÷ 20 = ?
1 1 (C) 12 (D) 0·12 10 10 5
(C) 2 (D) 10
2 2 (A) 0 (B) 1
25. { 1 1 1 1 2
7 + ÷ of – × 2 ÷ 1
1 7
16. (135 – 23 ÷ 1213 + 75 ×13 ) is equal 2 2 2 4 5 3 8
(C) 100 (D)
107

to— ( 2
of 1 – 1
5
1
3 )} =?
200
69842 × 69842 – 30158 × 30158
31 19 1 32. =?
(A) 1 (B) (A) 3 69842 – 30158
90 30 5
(A) 100000 (B) 69842
11 1
(C) (D) 30 (B) 2 (C) 39684 (D) 30158
30 24

Quantitative Aptitude Test | 63


More PDF download : http://www.gksolve.com

1 50 ? 47. When Simplified, the product


÷ 4 + 20 40. =
33.
2
1
× 4 + 20
=?
?
12
1
2 ( )( )( )
2–
1
3
2–
3
5
2–
5
7

2
(A)
81
(B) 2
3
(A)
25
2
(B)
4
25 ( )
2–
999
1001
is equal to—

88 11 (C) 4 (D) 25 991


(A)
161 1001
(C) (D) 1 1 1 1
176 ÷ of 1001
5 5 5 (B)
92 × 184 41. =? 13
34. The value of is— 1 1 1
316 of ÷ 1003
5 5 5 (C)
2 4 (A) 1 (B) 5 3
(A) (B)
3 9 1 (D) None of these
16 32 (C) (D) 25
(C) (D) 5 885 × 885 × 885 + 115
81 243 × 115 × 115
1 1 5 48. is equal
1 1 1 1 + ÷ 885 × 885 + 115 × 115
35. + + + is 4 4 4
1·2·3 2·3·4 3·4·5 4·5·6 42. =? – 885 × 115
1 1 1
equal to— × +2 to—
4 4 4
7 11 (A) 115 (B) 770
(A) (B) 16
30 30 (A) (C) 885 (D) 1000
25
13 17
(C) (D) 32 1 1
30 30 (B) 49. How many ’s are there in 37 ?
185 8 2
1 1
36. The value of 1 + + + 36 (A) 300
4 × 3 4 × 32 (C)
185 (B) 400
1
upto four places of (D) None of these
4 × 33 (C) 500
decimals is— 1 (D) Cannot be determined
43. The value of is—
(A) 1·1202 2
3+ 3 1
(B) 1·1203 1 50. is what part of ?
2+ 48 12
(C) 1·1204 2
(D) None of these 5 19 3
(A) (B) (A)
1 19 5 7
37. The simplification of 1 +
1 4 5 1
2+ (C) (D) (B)
1 5 4 12
1–
3 4
x 4 (C)
yields the result— 44. If y = , then the value of 3
2 7 5
(A)
7
(B)
9 ⎛ 4 2y – x⎞ (D) None of these
⎜ 7 + 2y + x⎟ is—
9 13 ⎝ ⎠ 51. A boy was asked to multiply a
(C) (D)
7 7
1 (A)
3
7
(B) 1 given number by ( )
8
17
. Instead
38. The value of 1 + is— he divided the given number by
1 1
1+
1+
1
9
(C) 1
7
(D) 2
( )
8
17
and got the result 225
a 1 3a + 2b more than what he should have
29 10 45. If b = , then is equal
(A) (B) 3 3a – 2b got if he had multiplied the
19 19
(C)
29
10
(D)
10
9
to—
(A) 3 (B) – 3
number by ( )
8
17
. The given
(C) – 5 (D) – 1 number was—
39.
1
2 [ { 1
7 – 2 ÷ 1 –
4
1 1
4 2 46. If
x 3
= , the value of
2x + y (A) 8 (B) 17
2y 2 x – 2y (C) 64 (D) 136
( 1 1 1
1 – –
2 3 6 )} ] equals— 52. In an examination, a student was
1
is equal to—
(A)
2
(B) 1
(A)
7 asked to find ( )
3
14
of a certain

(C) 4
9
1
(D) 1
77
(B) 7
(C) 7·1
number, by mistake he found () 3
4
2 288 (D) None of these of it. His answer was 150 more

Quantitative Aptitude Test | 64


More PDF download : http://www.gksolve.com

than the correct answer. The make the result a whole number 5 4
given number is— is— (A) (B)
18 9
(A) 180 (B) 240 5 7
(A) (B) 11 17
(C) 280 (D) 290 12 12 (C) (D)
18 36
53. If we multiply a fraction by itself 1
(C) (D) 7
and divide the product by its 2 7
59. Gopal was asked to find of a
reciprocal, the fraction thus 1 9
26 56. In a family, the father took of fraction. But he made a mistake
obtained is 18 . The original 4
27 of dividing the given fraction by
the cake and he had 3 times as
fraction is— 7
much as others had. The total and got an answer which
8 number of family members is— 9
(A) exceeded the correct answer by
27 (A) 3 (B) 7
8
2 (C) 10 (D) 12 . The correct answer is—
(B) 2 21
3 57. Ravi earns twice as much in
1 3
(C) 1 January as in each of the other (A)
3 months. What part of his annual 7
(D) None of these earnings he earns in that 7
(B)
1 month ? 12
54. In a college th of the girls and 2 1
5 (A) (B) 2
1 13 10 (C)
th of the boys took part in a 21
8 5 1
(C) (D) 1
Social Camp. What of the total 7 5 (D)
number of students in the college 3
took part in the camp ?
13
58. In a certain office () 1
3
of the
60. The highest score in an inning
(A)
40
13
workers are women () 1
2
of the was
3
11
of the total and the next

(B)
80
2
women are married and () 1
3
of
3
highest was of the remainder.
11
(C) the married women have chil- If the scores differed by 9, then
13 the total score is—
(D) Data inadequate dren. If ()
3
4
of the men are
(A) 99
55. The smallest fraction which
should be subtracted from the married and () 2
3
of the married (B) 110
(C) 121
3 1 7 1 1 men have children. What part of
sum of 1 , 2 , 5 , 3 and 2 to
4 2 12 3 4 workers are without children ? (D) 132

Answers with Hints


7 8. (C) Given Exp. = 0·5 × 0·5 + 0·5 + 5
7+7+
7 15 0·5
1. (C) Given expression = = = 0·5 × 0·5 +
5 11 5
5+5+
5 = 0·25 + 0·1 = 0·35
240 × 300 9. (C) Given expression = 8 ÷ 4 × 1 × 4 + 3 – 7
2. (C) Given expression = = 300
240
1
69 – 42 + 2 29 = 8× ×1×4+3–7
3. (A) Given expression = = = 1·45 4
45 – 25 20
= 8+3–7=4
48 – 36 + 9 21 7
4. (C) Given expression = = = 4 3
12 – 3 9 3 10. (A) Given expression = + 2 × 2 + 2 ×
2 2
·3
5. (C) Given expression = ·01 × × ·5 = 2+4+3=9
·4
11. (D) ? = 3120 ÷ 26 + 13 × 30
·0015
= = ·00375 = 120 + 13 × 30
·4
6. (B) Given expression = ·25 – ·025 = 0·225 = 120 + 390 = 510
3·6 12. (B) ? = 7+7÷7×7
7. (A) Given expression = 4– + 0·2 × 0·5
4 1
= 7 + 7 × × 7 = 7 + 7 = 14
= 4 – 0·9 + 0·1 = 3·2 7

Quantitative Aptitude Test | 65


More PDF download : http://www.gksolve.com

5000 17·28 ÷ x
13. (D) Given expression = 5005 – = 5005 – 500 24. (C) Let = 2
10 3·6 × 0·2
= 4505 17·28
Then = 1·44
1 x
14. (D) ? = 171 ÷ 19 × 9 = 171 × × 9 = 81
19 17·28
∴ x = = 12
2 15 1·44
15. (D) Given expression = 108 ÷ 12 + ×
5 4 25. (C) Given expression
= 9+ =
3 21
2 2
= 10
1
2 =
15 1 1 2 7 15
+ ÷ – × ÷ ×
2 2 8 5 3 8 ( )7 4
5 3

8 2 13 7 1 15 2 7 15 1
16. (A) Given expression = – × + × = +4– × ÷ ×
5 3 12 5 3 2 5 3 8 15
8 13 7 15 2 7 1 15 2 7 8
= – + = +4– × ÷ = +4– × ×
5 18 15 2 5 3 8 2 5 3 1
144 – 65 + 42 15 112 23 112 121 1
= = +4– = – = =4
90 2 15 2 15 30 30
121 31 26. (C) Given expression
= =1
90 90

17. (D) Let


13 25
+
1
+ x + = 10, then 3 9 2
= ÷ of –
1
6() 10 5
× +
4 6 4

x = 10 – ( 13 25 1
+ + ) = 10 –( )
39 + 50 + 3
4 4 3 5
6() 3 6

4 6 4 12 3 2 1 6 10 5 1 2 5
= × – × × + = – +
(
= 10 –
92
12 ) 7
= =2
3
1
3
4 3 6 5 3 6 2 3 6
3–4+5 4 2
= = =
18. (D) Given expression =
48
12
× ( )
3 1
÷
2 2
27.
5 6
6 6 3
8 8 3 10
(B) Let ÷ × x – ÷ + × =
25
6 7 9 5 4 3 9
= 4× ( )
3
2
× 2 = 12
Then,
5 7 8 5 5
× ×x– × + =
25
19. (C) Given expression = 10 – [9 – {8 – 1}] – 5 6 6 9 8 2 9
35 25 5 5
= 10 – [9 – 7] – 5 ⇒ x = + –
36 9 9 2
= 10 – 2 – 5 = 3
20. (A) Given expression ⇒
35
36
x = (50 + 10 – 45
18 ) =
5
6
[ { }]
= 3÷ 3÷ 2÷
34
13 ∴ x = (5 36
×
6 35 ) =
6
7
[ { }]
= 3÷ 3÷ 2×
13
34 28. (D) Let
47 19 19
3
× +
6 3
=
205
18
+x

= 3÷ 3÷[ ] [ ]
13
17
=3÷ 3×
17
13 Then, x =
893 19 205
18
+
3

18
13 13 893 + 114 – 205
= 3× = ⇒ x =
51 17 18
802 5
21. {
(B) Given expression = 1 + 1 ÷ 1 + 1 ÷ } 4
3
∴ x =
18
= 44
9
1·8
= 1 + 1 ÷ {1 + }
3 29. (C) Given expression = 4·59 × + 0·6 – 0·2
3·6
4
= 2·295 + 0·6 – 0·2 = 2·695
7 4
= 1+1÷ =1+1× 70 7 68
4 7 30. (B) Given expression = ÷ ×
17 5 33
4 4 70 5 68 200 2
= 1+ =1 = × × = =6
7 7 17 7 33 33 33
3 4 1 1 9 93 7 1
22. (D) Given expression = × + = 2 + = 31. (B) Given expression = + ×
2 3 4 4 4 100 5 20
1 3 93 7
23. (C) Given expression = + 1 = = + =1
2 2 100 100

Quantitative Aptitude Test | 66


More PDF download : http://www.gksolve.com

32. (A) Given expression 50 x


40. (D) Let x =
=
(69842) 2 – (30158) 2
(69842 – 30158) ( ) 25
2
(69842 – 30158) (69842 + 30158) 25
= ⇒ x2 = 50 × = 625
(60842 – 30158) 2
= 100000
∴ x = ⎯⎯⎯
√ 625 = 25
1 1
× + 20 1 1 1
2 4 161 1 ÷ × 25
33. (C) Given expression = = × 5 25 5
2 + 20 8 22 41. (D) Given expression = =
161 1 1 1
= ÷ ×5
176 25 5 25
(32)2 × (3 × 3 × 2)4 = (5 × 5) = 25
34. (C) Given expression =
316 1 1 4 1 1
+ × +
3 × 3 × 24
4 8 4 4 5 4 5
= 42. (C) Given expression = =
316 1 9 37
+
24 16 16 4 16
= 4=
3 81 9 16 36
× =
=
35. (A) Given expression 20 37 185
4 × 5 × 6 + 5× 6 + 2 ×6 + 2 ×3 1 1
= 43. (A) Given expression = =
2 ×3×4×5×6 2 4
3+ 3+
=
168
=
24 × 30 30
7 5
2 ()
5

108 + 9 + 3 + 1 121 1 5
36. (B) Given expression = = = =
= 1·1203
108 108
( )
19
5
19

1 1 4 2y – x
37. (C) Given expression = 1 + =1+ 44. (B) Given Exp. = +
1 3 7 2y + x
2+ 2+
2 2 x 4
3 2 –y 2–
4 4 5
1 2 9 = + = +
= 1+ =1+ =
7
2
7 7
7
2+ y
x
() 7
2+
4
5
4 6 5 4 3
1 =+ × = + =1
38. (A) Given expression = 1 + 7 5 14 7 7
1
1+ 3a + 2b
( ) 10
9
45. (B) Given Exp. =
3a – 2b

= 1+
1+
1
9
=1+
1
19
=
a
()
3 b + 2 3 ×1 + 2
=
3
=
3
= –3

= 1+
10 29
=
10 10 a
() 1
3 b –2 3× –2
3
–1

19 19 46. (B) Dividing Num and Denom by y we get.


39. (C) Given expression Given expression
=
15
2
– [ { ( )}]
9
4
÷
5 1 3 1 1

4 2 2 3 6
– –
2x + y
x
()
2 y +1
2×3+1 7
= = = = =7
2 [4 {4 2 }] ()
15 9 5 1 x – 2y x 3–2 1
= – ÷ – ×1 – 2
y

2 [4 { 4 2}] [Q 2yx = 32 ⇒ xy = (32 × 2) = 3 ]


15 9 5 1
= – ÷ –

2 [4 4] 2 [4 3]
15 9 3 15 9 4 5 7 9 1003
= – ÷ = – × 47. (C) Given expression = × × ×…×
3 5 7 1001

= ( – 3) = = 4
15 9 1 1003
=
2 2 2 3

Quantitative Aptitude Test | 67


More PDF download : http://www.gksolve.com

⎛ a3 + b3 ⎞ ∴ x = 9
48. (D) Given expression = ⎜ a2 + b2 – ab⎟
⎝ ⎠ Hence, the total number of members
(a + b) (a2 + b2 – ab) = x + 1 = 10.
=
(a2 + b2 – ab) 57. (A) Suppose Ravi earns Rs. x in each of the 11
= (a + b) = (885 + 115) months.
= 1000 Then earning in January = Rs. 2x.
1 75 1 75 ∴ Total annual income = (11x + 2x) = Rs. 13x
49. (A) Number of s = ÷ = × 8 = 300
8 2 8 2 Part of total earning in January
1 3 3 3 2x 2
50. (D) Let x of = . Then, x = × 12 = = =
12 48 48 4 13x 13
17 8 225
51. (D) ˙·˙ x × –x× = 225 ⇒ x = 225 58. (C) Let, total number of workers be x.
8 17 136
x
∴ (
x = 225 × )
136
225
= 136 Then, number of women =
3
3 3 2x
52. (C) ˙·˙ x – x = 150 Number of men =
4 14 3
15 Number of women having children
⇒ x = 150
28 1 1 1 x
= of of x =
21 – 6 3 2 3 18
⇒ x = 150
28 Number of men having children
∴ (
x = 150 × )28
15
= 280 2 3 2x x
= of of =
3 4 3 3
1 26 Number of workers having children
53. (B) ˙·˙ x × x ÷ x = 18
27 x x 7
512 = + = x
⇒ x3 = 18 3 18
27
Number of workers having no children
⇒ x3 = () 8 3
3 (
= x– x =
7
)
18
11x
18
8 2
∴ x = =2

54.
3 3
(C) Out of the 5 girls, 1 took part in the camp.
= ( 11
18 )
of all workers

Out of the 8 boys, 1 took part in the camp. 59. (B) ˙·˙ Let the fraction = x
Out of 13 students, 2 took part in the camp. 9 7 8
˙·˙ x– x =
2 7 9 21
∴ of total number of students took part in the
13 32 8
camp. ⇒ x =
63 21
7 5 67 10 9 8 63 3
55. (A) ˙·˙ + + + + ⇒ x = x =
4 2 12 3 4 21 32 4
= ( 21 + 30 + 67 + 40 + 27
12 ) =
185
12 ∴
7
Correct answer = x = × =
9
7 3 7
9 4 12
This is nearly greater than 15. Let required fraction 60. (C) Let total score be x.
be x.
3
185 Then, highest score = x
then, – x = 15, 11
12
∴ x = ( 185
12 )
– 15 =
5
12
( )
Remainder = x –
3x
11
8
= x
11
56. (C) Let there be x members, other than father. 3 8 24
Next highest score = of x = x
11 11 121
1 3
Father’s share = , other’s share = . 3x 24x
4 4 Now, ˙·˙ – = 9
3 11 121
Each of other’s share =
4x 9x
⇒ = 9
3 1 121
˙·˙ 3× =
4x 4 ∴ x = 121

Quantitative Aptitude Test | 68


More PDF download : http://www.gksolve.com

Ratio and Proportion


Important Points/Facts
Ratio—The ratio of two quan-
If 7 litres of water is added to the
mixture, the ratio of alcohol and 1st part = Rs. (420 ×105 )
water becomes 3 : 4. Find the
tities in the same units is a fraction = Rs. 210
quantity of alcohol in the mixture.
that one quantity is of the other.

Thus, a to b is a ratio b , ()a Solution : Let the quantity of


alcohol and water be 4x and 3x litres
2nd part = Rs. (420 × 103 )
respectively. Then, = Rs. 126
written as a : b.
The first term of a ratio is called
antecedent, while the second term is
4x
3x + 7
=
3
4
3rd part = Rs. (420 ×102 )
known as consequent. ⇒ x = 3 = Rs. 84
4 ∴ Quantity of alcohol in the ∴ Number of one-rupee coins
Thus, the ratio 4 : 7 represents mixture = 12 litres. = 210
7
with antecedent 4 and consequent 7. Q. 3. In a mixture of 35 litres Number of 50 paise coins
Rule—The multiplication or di- the ratio of milk and water is 4 : 1 = (126 × 2) = 252.
vision of each term of a ratio by a now, 7 litres of water is added to
Number of 25 paise coins
same non-zero number does not affect the mixture. Find the ratio of milk
the ratio. and water in the new mixture. = (84 × 4) = 336
Thus, 3 : 5 is the same as 6 : 10 Solution : Milk in 35 litres of Q. 6. Divide Rs. 455 in the
ratio 4 : 3.
or 9 : 15 or 12 : 20 etc.
Proportion : The equality of
(
mix. = 35 × )4
5
= 28 litres.
Solution : Sum of the terms of
two ratios is called proportion. Water in this mix. = (35 – 28) the ratio = (4 + 3) = 7
Thus, 2 : 3 = 8 : 12 is written as
2 : 3 : : 8 : 12 and we say that 2, 3, 8
= 7 litres
New mixture contains milk = 28
∴ First part = Rs. 455 ×( 4
7 )
and 12 are in proportion. litres = Rs. 260
In a proportion, the first and
fourth terms are known as extremes,
Water in new mixture = (7 + 7)
litres = 14 litres. (
Second part = Rs. 455 ×
3
7 )
while second and third terms are ∴ Ratio of milk and water in = Rs. 195
known as means. new mix. = 28 : 14 = 2 : 1.
In a proportion, we always have Q. 7. If a : b = 2 : 3 and b : c =
Q. 4. Find three numbers in 5 : 7, find a : c and a : b : c.
Product of means = Product of the ratio 2 : 3 : 5 the sum of whose
Extremes. squares is 608. a 2
Solution : b =
Solution : Let the numbers be 3
Examples 2x, 3x and 5x. b 5
and
Q. 1. Three utensils contain Then, 4x2 + 9x2 + 25x2 = 608 c = 7
equal mixtures of milk and water
in the ratio 6 : 1 : 5 : 2 and 3 : 1
respectively. If all the solutions are
⇒ 38x2 = 608
608

a
( )
c = b ×c
a b

⇒ x2 = = 16
= ( × )=
mixed together, find the ratio of 38 2 5 10
milk and water in the final mixture. ∴ x = 4 3 7 21
Solution : So, the numbers are 8, 12 and Hence a : c = 10 : 21
In final mixture we have quantity 20.
Now L.C.M. of 3 and 5 is 15
of milk = ( 6 5 3
+ +
7 7 4 ) =
65
28
Q. 5. A bag contains rupee, 50
paise and 25 paise coins in the ratio ∴ a : b = 2 : 3 = 10 : 15
5 : 6 : 8. If the total amount is and b : c = 5 : 7 = 15 : 21
quantity of Rs. 420. Find the number of coins
Hence a : b : c = 10 : 15 : 21.
water = ( 1 2 1
+ +
7 7 4 ) =
19
28
of each type.
Solution : Ratio of Q. 8. A stick 1·4 m long caste a
shadow 1·3 m long at the same time
65 19 6 8
∴ Milk : Water = : values = 5 : : when a pole casts a shadow 5·2 m
28 28 2 4 long. Find the length of the pole.
= 65 : 19 = 5:3:2 Solution : Clearly more is the
Q. 2. A mixture contains Divide Rs. 420 in the ratio length of shadow, more is the length
alcohol and water in the ratio 4 : 3. 5:3:2 of the object.

Quantitative Aptitude Test | 69


More PDF download : http://www.gksolve.com

Let the length of the pole be x 07. A right cylinder and a right cir- 15. If a carton containing a dozen
metres cular cone have the same radius mirrors is dropped, which of the
Then 1·3 : 5·2 : : 1·4 : x and the same volume. The ratio following cannot be the ratio of
of the height of the cylinder to broken mirrors to unbroken
∴ 1·3 × x = 5·2 × 1·4 that of the cone is— mirrors ?
5·2 × 1·4 (A) 3 : 5 (B) 2 : 5 (A) 2 : 1 (B) 3 : 1
or x = = 5·6
1·3 (C) 3 : 1 (D) 1 : 3 (C) 3 : 2 (D) 7 : 5
Hence the length of the pole is 08. A circle and a square have same 16. Two whole numbers whose sum
5·6 m. area. Therefore, the ratio of the is 64, cannot be in the ratio—
side of the square and the radius
Exercise of the circle is—
(A) 5 : 3 (B) 7 : 1
(C) 3 : 4 (D) 9 : 7
1. The ratio of money with Ram
and Gopal is 7 : 17 and that with (A) √
⎯π:1 (B) 1 : √
⎯π 17. The weight of a 13 m long iron
Gopal and Krishan is 7 : 17. If (C) 1 : π (D π : 1 rod is 23·4 kg. The weight of
Ram has Rs. 490, Krishan has— 09. In a class, the number of boys is 6 m long of such rod will be—
(A) Rs. 2890 (B) Rs. 2330 more than the number of girls by (A) 7·2 kg (B) 12·4 kg
(C) Rs. 1190 (D) Rs. 2680 12% of the total strength. The (C) 10·8 kg (D) 18 kg
ratio of boys to girls is—
02. Rs. 5625 is divided among A, B 18. The mean proportional between
(A) 11 : 14 (B) 14 : 11 0·32 and 0·02 is—
1
and C so that A may receive as (C) 25 : 28 (D) 28 : 25
2 (A) 0·34 (B) 0·3
much as B and C together 10. A, B and C can do a work in 20, (C) 0·16 (D) 0·08
1 25 and 30 days respectively.
receive and B receives of what 19. The third proportional to 0·8 and
4 They undertook to finish the 0·2 is—
A and C together receive. The work together for Rs. 2220, then
share of A is more than that of B the share of A exceeds that of B (A) 0·4 (B) 0·8
by— by— (C) 0·05 (D) 0·032
(A) Rs. 750 (B) Rs. 775 (A) Rs. 120 (B) Rs. 180 20. The fourth proportional to 0·2,
(C) Rs. 1500 (D) Rs. 1600 (C) Rs. 300 (D) Rs. 600 0·12 and 0·3 is—
11. Three friends divide Rs. 624 (A) 0·13 (B) 0·15
03. A certain amount was divided
1 (C) 0·18 (D) 0·8
between Kavita and Reena in the among themselves in the ratio :
ratio 4 : 3. If Reena’s share was 2 21. What number should be subtrac-
Rs. 2400. The amount is— 1 1 ted from each of the numbers 54,
: . The share of the third friend
3 4 71, 75 and 99 so that the remain-
(A) Rs. 5600
is— ders may be proportional ?
(B) Rs. 3200 (A) Rs. 288 (B) Rs. 192 (A) 1 (B) 2
(C) Rs. 9600 (C) Rs. 148 (D) Rs. 144 (C) 3 (D) 6
(D) None of these 12. The monthly salary of A, B, C is 22. What number should be added to
04. The prices of a scooter and a tele- in the proportion of 2 : 3 : 5. If each one of 6, 14, 18 and 38 to
vision set are in the ratio 3 : 2. If C’s monthly salary is Rs. 1200 make them equally proportio-
a scooter costs Rs. 6000 more more than that of A, then B’s nate ?
than the television set the price annual salary is—
of the television set is— (A) Rs. 14400 (A) 1 (B) 2
(A) Rs. 6000 (B) Rs. 24000 (C) 3 (D) 4
(B) Rs. 10000 (C) Rs. 1200 23. A fraction bears the same ratio to
(D) Rs. 2000 1 3 5
(C) Rs. 12000 as does to . The fraction
13. A bag contains 25 paise, 10 paise 27 7 9
(D) Rs. 18000
and 5 paise coins in the ratio 1 : is—
05. If 8 : x = x : 18, then x is equal 2 : 3. If their total value is Rs. 7 1
to— 30, the number of 5 paise coins (A) (B)
45 35
(A) 144 (B) 72 is—
45 5
(A) 50 (B) 100 (C) (D)
(C) 26 (D) 12 7 21
(C) 150 (D) 200
06. The surface areas of two spheres 14. The ratio of two numbers is 3 : 4 24. What must be added to each term
are in the ratio 1 : 4. The ratio of and their sum is 420. The greater of the ratio 7 : 13 so that the ratio
their volumes is— of the two numbers is— becomes 2 : 3 ?
(A) 1 : 2 (B) 1 : 4 (A) 175 (B) 200 (A) 1 (B) 2
(C) 1 : 8 (D) 1 : 6 (C) 240 (D) 315 (C) 3 (D) 5

Quantitative Aptitude Test | 70


More PDF download : http://www.gksolve.com

25. In a ratio which is equal to 5 : 8. 1 1 44. The ratio of father’s age to son’s
35. If A = B and B = C, then A :
If the antecedent is 40 then con- 3 2 age is 4 : 1. The product of their
sequent is— B : C is— ages is 196. The ratio of their
(A) 25 (A) 1 : 3 : 6 (B) 2 : 3 : 6 ages after 5 years will be—
(B) 64 (C) 3 : 2 : 6 (D) 3 : 1 : 2 (A) 3 : 1 (B) 10 : 3
(C) 48 36. If A : B = 5 : 7 and B : C (C) 11 : 4 (D) 14 : 5
(D) None of these = 6 : 11, then A : B : C is— 45. The ratio between the ages of
26. Out of the ratio 7 : 15, 15 : 23, (A) 55 : 77 : 66 Kamla and Savitri is 6 : 5 and
17 : 25 and 21 : 29 the smallest (B) 30 : 42 : 77 the sum of their ages is 44 years.
one is— (C) 35 : 49 : 42 The ratio of their ages after 8
(A) 17 : 25 (B) 7 : 15 years will be—
(D) None of these
(C) 15 : 23 (D) 21 : 29 (A) 5 : 6 (B) 7 : 8
37. If A : B = 7 : 9 and B : C = 3 : 5
1 1 1 1 (C) 8 : 7 (D) 14 : 13
then. A : B : C is—
27. If : x = x : , then the value
5 1·25 (A) 7 : 9 : 5 46. A father’s age was 5 times his
of x is— (B) 21 : 35 : 45 son’s age 5 years ago and will be
(A) 1·25 (B) 1·5 3 times son’s age after 2 years
(C) 7 : 9 : 15
(C) 2·5 (D) 2·25 the ratio of their present ages
(D) 7 : 3 : 15 is—
28. If one-third of A, one-fourth of 38. 0·6 of a number equals 0·09 of (A) 5 : 2 (B) 5 : 3
B and one-fifth of C are equal, another number. The ratio of the
then A : B : C is— (C) 10 : 3 (D) 11 : 5
numbers is—
(A) 3 : 4 : 5 (B) 4 : 3 : 5 (A) 2 : 3 (B) 1 : 15 47. The average age of 3 girls is 20
1 1 1 (C) 20 : 3 (D) 3 : 20 years and their ages are in the
(C) 5 : 4 : 3 (D) : : proportion 3 : 5 : 7. The age of
3 4 5
39. If 10% of x is the same as 20% youngest girl is—
29. The ratio which ( 1
3
of Rs. 9·30) of y then x : y is equal to—
(A) 1 : 2 (B) 2 : 1
(A) 4 years
bears to (0·6 of Rs. 1·55) is— (B) 6 years 8 months
(A) 1 : 3 (B) 10 : 3 (C) 5 : 1 (D) 10 : 1 (C) 8 years 3 months
(C) 3 : 10 (D) 3 : 1 40. If A : B = 2 : 3 and B : C = 4 : 5, (D) 12 years
then C : A is equal to—
30. Two numbers are in the ratio 48. A man has some hens and cows.
(A) 15 : 8 (B) 12 : 10
3 : 5. If each number is increased If the number of heads be 48 and
(C) 8 : 5 (D) 8 : 15
by 10, the ratio becomes 5 : 7. number of feet equal 140 the
The numbers are— 41. One year ago the ratio between number of hens will be—
(A) 3, 5 (B) 7, 9 Laxman’s and Gopal’s salary
(A) 22 (B) 23
was 3 : 4. The ratio of their
(C) 13, 22 (D) 15, 25 individual salaries between last (C) 24 (D) 26
31. If x : y = 2 : 3 and 2 : x = 1 : 2, year’s and this year’s salaries are 49. 6 men, 8 women, 6 children
then the value of y is— 4 : 5 and 2 : 3 respectively. At complete a job for a sum of
present the total of their salary is Rs. 950. If their individual wages
(A) 4 (B) 6 Rs. 4,160. The salary of Laxman are in ratio 4 : 3 : 2, the total
1 3 now is—
(C) (D) money earned by the children
3 2 (A) Rs. 1040 (B) Rs. 1600 is—
32. If 2A = 3B and 4B = 5C, then (C) Rs. 2560 (D) Rs. 3120 (A) Rs. 190 (B) Rs. 195
A : C is— 42. The ratio between Sumit’s and (C) Rs. 215 (D) Rs. 230
(A) 3 : 4 (B) 8 : 15 Prakash’s age at present is 2 : 3.
Sumit is 6 years younger than 50. The ratio between two numbers
(C) 15 : 8 (D) 4 : 3 is 3 : 4 and their L.C.M. is 180.
Prakash. The ratio of Sumit’s
33. If 2A = 3B = 4C, then A : B : C age to Prakash’s age after 6 The first number is—
is— years will be— (A) 15 (B) 20
(A) 2 : 3 : 4 (B) 4 : 3 : 2 (A) 1 : 2 (B) 2 : 3 (C) 45 (D) 60
(C) 3 : 4 (D) 3 : 8
(C) 6 : 4 : 3 (D) 3 : 4 : 6 51. In a mixture of 60 litres. The
43. The ages of Vivek and Sumit are
34. If A : B = 2 : 3, B : C = 4 : 5 and ratio of milk and water is 2 : 1.
in the ratio 2 : 3. After 12 years,
C : D = 6 : 7, then A : D is equal What amount of water must be
their ages will be in the ratio
to— added to make the ratio 1 : 2 ?
11 : 15. The age of Sumit is—
(A) 2 : 7 (B) 7 : 8 (A) 32 years (B) 42 years (A) 42 litres (B) 56 litres
(C) 16 : 35 (D) 4 : 13 (C) 48 years (D) 56 years (C) 60 litres (D) 77 litres

Quantitative Aptitude Test | 71


More PDF download : http://www.gksolve.com

52. A sum of Rs. 1300 is divided 59. Vinay got thrice as many marks 65. 729 ml of a mixture contains
between A, B and C and D such in Maths as in English. The milk and water in the ratio 7 : 2.
that proportion of this marks in How much more water is to be
A’s Share B’s Share Maths and History is 4 : 3. If his added to get a new mixture
= total marks in Maths, English containing milk and water in the
B’s Share C’s Share
and History are 250. What are ratio of 7 : 3 ?
C’s Share 2
= = his marks in English ? (A) 60 ml (B) 70 ml
D’s Share 3
(A) 120 (B) 90 (C) 81 ml (D) 90 ml
Then A’s share is—
(A) Rs. 140 (B) Rs. 160 (C) 40 (D) 80 66. A and B are two alloys of gold
(C) Rs. 240 (D) Rs. 320 60. One-fourth of the boys and and copper prepared by mixing
three-eight of the girls in a metals in proportions 7 : 2 and
53. Two equal glasses are respecti- 7 : 11 respectively. If equal
1 1 school participated in the annual
vely and full of milk. They sports. What proportional part of quantities of the alloys are
3 4 melted to form a third alloy C,
the total student population of
are then filled up with water and the proportion of gold and cop-
the school participated in the
the contents mixed in a tumbler. per in C will be—
annual sports ?
The ratio of milk and water in (A) 5 : 9 (B) 5 : 7
the tumbler is— 4
(A) (C) 7 : 5 (D) 9 : 5
(A) 7 : 5 (B) 7 : 17 12
(C) 3 : 7 (D) 11 : 23 5 67. Rs. 1870 has been divided into
(B) three parts in such away that half
54. The ratio of milk and water in 85 8
of the first part, one-third of the
kg of adulterated milk is 27 : 7. 8
(C) second part and one-sixth of the
The amount of water which must 12 third part are equal. The third
be added to make the ratio 3 : 1 (D) Data inadequate part is—
is—
61. Gold is 19 times as heavy as (A) Rs. 510 (B) Rs. 680
(A) 5 kg (B) 6·5 kg
(C) 7·25 kg (D) 8 kg water and copper 9 times as (C) Rs. 850 (D) Rs. 1020
heavy as water. The ratio in 68. Rs. 385 has been divided among
55. A mixture contains milk and which these two metals be mixed A, B, C in such a way that A
water in the ratio 5 : 1. On so that the mixture is 15 times as 2
adding 5 litres of water the ratio receives th of what B and C
heavy as water is— 9
of milk and water becomes 5 : 2.
The quantity of milk in the (A) 1 : 2 (B) 2 : 3 together receive. Then A’s share
mixture is— (C) 3 : 2 (D) 19 : 135 is—
(A) 16 litres (A) Rs. 70
ma + nc
(B) 25 litres 62. If a : b = c : d, then is (B) Rs. 77
mb + nd
(C) 32·5 litres (C) Rs. 82·50
equal to—
(D) 22·75 litres (D) Rs. 85
(A) m : n (B) na : mb
56. The proportion of Zinc and 69. Some money is divided among
Copper in a brass piece is 13 : 7. (C) a : b (D) md : nc
A, B and C in such a way that 5
How much zinc will be there in 63. Rs. 1050 is divided among P, Q times A’s share 3 times, B’s
100 kg of such a piece ? 2 share and 2 times C’s share are
(A) 20 kg (B) 35 kg and R. The share of P is of the all equal. The ratio between the
5
(C) 55 kg (D) 65 kg combined share of Q and R. shares of A, B, C is—
57. A’s money is to B’s money as Thus P gets— (A) 5 : 3 : 2
4 : 5 and B’s money is to C’s (A) Rs. 200 (B) Rs. 300 (B) 2 : 2 : 5
money as 2 : 3. If A has Rs. 800, (C) 15 : 10 : 6
(C) Rs. 320 (D) Rs. 420
C has— (D) 6 : 10 : 15
(A) Rs. 1000 (B) Rs. 1200 64. Divide Rs. 600 among A, B and
(C) Rs. 1500 (D) Rs. 2000 2 70. 94 is divided into two parts in
C so that Rs. 40 more than th of
5 such a way that fifth part of the
58. 15 litres of a mixture contains
2 first and eighth part of the second
20% alcohol and the rest water. A’s share, Rs. 20 more than th
7 are in the ratio 3 : 4. The first
If 3 litres of water be mixed in it.
of B’s share and Rs. 10 more part is—
The percentage of alcohol in the
new mixture will be— 9 (A) 27 (B) 30
than th of C’s share may all be
2 17 (C) 36 (D) 48
(A) 17 (B) 16 equal. What is A’s share ?
3 71. Rs. 680 has been divided among
1 (A) Rs. 280 (B) Rs. 150 2
(C) 18 (D) 15 A, B, C such that A gets of
2 (C) Rs. 170 (D) Rs. 200 3
Quantitative Aptitude Test | 72
More PDF download : http://www.gksolve.com

1 75. Rs. 53 is divided among A, B Rs. 1100 is collected as fare, the


what B gets and B gets of what
4 and C in such a way that B gets amount collected from first class
C gets. Then B’s share is— Rs. 7 more than what B gets and passengers is—
(A) Rs. 60 (B) Rs. 80 B gets Rs. 8 more than what C (A) Rs. 275
(C) Rs. 120 (D) Rs. 160 gets. The ratio of their shares
(B) Rs. 315
is—
72. Rs. 2430 has been divided (C) Rs. 137.50
(A) 16 : 9 : 18
among A, B, C in such a way (D) Rs. 100
that if their shares be diminished (B) 25 : 18 : 10
by Rs. 5, Rs. 10 and Rs. 15 res- (C) 18 : 25 : 10 79. The students in three classes are
pectively. The remainders are in in the ratio 2 : 3 : 5 if 20 students
(D) 15 : 8 : 30 are increased in each class. The
the ratio 3 : 4 : 5. Then A’s share
is— 76. A sum of money is divided ratio changes to 4 : 5 : 7. The
(A) Rs. 800 (B) Rs. 600 among A, B, C so that to each total number of students before
rupee A gets, B gets 65 paise and the increase were—
(C) Rs. 595 (D) Rs. 605
C gets 35 paise. If C’s share is (A) 10
73. The sides of triangle are in the Rs. 28, the sum is—
1 1 1 (B) 90
ratio : : and its perimeter is (A) Rs. 120 (B) Rs. 140
3 4 5
(C) Rs. 160 (D) Rs. 180 (C) 100
94 cm. the length of smallest
side is— 77. The incomes of A and B are in (D) None of these
(A) 18·8 cm. (B) 23·5 cm. the ratio 3 : 2 and their expendi-
(C) 24 cm. (D) 31·3 cm. 80. The cost of making an article is
tures in the ratio 5 : 3. If each
divided between materials, labour
74. The speeds of three cars are in saves Rs. 1,000. A’s income is—
and overheads in the ratio of
the ratio 3 : 4 : 5. The ratio bet- (A) Rs. 3000 (B) Rs. 4000 3 : 4 : 1. If the materials cost
ween times taken by them to (C) Rs. 6000 (D) Rs. 9000 Rs. 67·50, the cost of articles is—
travel the same distance is—
78. The ratio of the first and second (A) Rs. 180
(A) 3 : 4 : 5 class fares between two stations (B) Rs. 122·50
(B) 5 : 4 : 3 is 4 : 1 and that of the number
(C) 12 : 15 : 20 of passengers travelling by first (C) Rs. 380
(D) 20 : 15 : 12 and second class is 1 : 40. If (D) Rs. 540

Answers with Hints


01. (A) Ram : Gopal = 7 : 17 = 49 :119 04. (C) Let the price of a scooter be Rs. 3x and that of a
Gopal : Krishan = 7 : 17 = 119 : 289 television set be Rs. 2x.
∴ Ram : Gopal : Krishan = 49 : 119 : 289 Then 3x – 2x = 6000 ⇒ x = 6000
⇒ Ram : Krishan = 49 : 289 ∴ Cost of a television set = 2x = Rs. 12000
Thus, 49 : 289 = 490 : x 05. (D) ˙·˙ 18 × 8 = x2
289 × 490 ∴ x = ⎯⎯⎯
√ 144 = 12
∴ x =
49
4πr2 1
= Rs. 2890 06. (C) =
4πR2 4
1 2
02. (A) ˙·˙ A = (B + C) ⇒ B + C = 2 A r 1
2 ⇒ =
R2 4
⇒ A + B + C = 3A
r 1
Thus 3A = 5625 ⇒ A = Rs. 1875 ⇒ =
R 2
1
Again B = (A + C) ⇒ A + C = 4B r 3
1
4 ⇒ =
⇒ A + B + C = 5B R3 8
∴ 5B = 5625 ⇒ B = Rs. 1125 4 3
πr
Thus A’s share is more than that of B by Rs. (1875 – v 3
Hence, =
1125) i.e. Rs. 750. V 4
πR3
03. (A) Let their shares be Rs. 4x and Rs. 3x. 3
Then 3x = 2400 1
=
⇒ x = 800 8
∴ Total amount = 7x = Rs. 5600 Thus their volumes are in the ratio 1 : 8.

Quantitative Aptitude Test | 73


More PDF download : http://www.gksolve.com

07. (D) Let the heights of the cylinder and cone be h and
H respectively.
14. (C) Greater number = (420 ×47) = 240
1 15. (C) For dividing 12 into two whole numbers, the sum
˙·˙ πr2 h = πr2 H of the terms of the ratio must be a factor of 12.
3
So, they cannot be in the ratio 3 : 2.
h 1
⇒ = 16. (C) For dividing 64 into two whole numbers, the sum
H 3
of the terms of the ratio must be a factor of 64.
∴ h : H = 1 : 3. ∴ So they cannot be in the ratio 3 : 4.
So their heights are in the ratio 1 : 3. 17. (C) Less length, less weight
08. (A) Let the side of the square be x and let the radius ˙·˙ 13 : 6 : : 23·4 : x
of the circle be y ⇒ 13x = 6 × 23·4
x2 6 × 23·4
Then, x2 = πy2 ⇒ 2 = π ∴ x = = 10·8 kg
y 13
x Mean proportional = √ ⎯⎯⎯⎯⎯⎯⎯⎯
0·32 × 0·02 = ⎯
√⎯⎯⎯⎯
y = √
⇒ ⎯π 18. (D) 0·0064
= 0·08
∴ x:y = √ ⎯π:1 19. (C) Let 0·8 : 0·2 : : 0·2 : x
09. (B) Let the number of boys and girls be x and y Then 0·8 x = 0·2 × 0·2
respectively. Then 0·2 × 0·2 0·04 4
˙·˙ (x – y) = 12% of (x + y) ∴ x = = = = 0·05
0·8 0·80 80
3 20. (C) Let 0·2 : 0·12 : : 0·3 : x
⇒ x–y = (x + y)
25 ˙·˙ 0·2x = 0·12 × 0·3
⇒ 25x – 25y = 3x + 3y
0·12 × 0·3
x 28 14 ∴ x = = 0·18
⇒ 22x = 28y ∴ y = = 0·2
22 11 54 – x 75 – x
10. (B) Ratio of shares of A, B and C 21. (C) ˙·˙ =
71 – x 99 – x
1 1 1 ⇒ (54 – x) (99 – x) = (75 – x) (71 – x)
= : : = 15 : 12 : 10
20 25 30 ⇒ x2 – 153x + 5346 = x2 – 146 x + 5325
∴ (
A’s share = Rs. 2220 × )15
37
= Rs. 900 ⇒

7x = 21
x = 3
B’s share = Rs. (2220 × ) = Rs. 720
12
6+x 18 + x
37 22. (B) ˙·˙ =
14 + x 38 + x
Thus, the share of A exceeds that of B by Rs. (900 –
720) = Rs. 180. ⇒ (6 + x) (38 + x) = (18 + x) (14 + x)
1 1 1 ⇒ x2 + 44x + 228 = x2 + 32x + 252
11. (D) Ratio = : : = 6 : 4 : 3
2 3 4 ⇒ 12x = 24 ∴ x = 2

(
∴ Share of third friend = Rs. 624 ×
3
13 ) 23. (B) ˙·˙
1
27
x:= :
3 5
7 9
= Rs. 144 5 1 3
⇒ x = ×
12. (A) Let the monthly salary of A, B, C be Rs. 2x, Rs. 9 27 7
3x and Rs. 5x respectively.
Then, 5x – 2x = 1200 ⇒ x = 400

5
9
x =
1
63
∴x=
1 9
×
63 5 ( =
1
35 )
∴ B’s monthly salary = 3x = Rs. 1200 7+x 2
24. (D) ˙·˙ =
Hence, B’s annual salary = Rs. (12 × 1200) 13 + x 3
= Rs. 14400 ⇒ 3 (7 + x) = 2 (13 + x) ∴ x = 5
1 2 3 5 5 × 8 40
13. (C) Ratio of their values = : : =5:4:3 25. (B) ˙·˙ = =
4 10 20 8 8 × 8 64
∴ Consequent = 64
∴ Value of 5 paise coins = Rs. 30 × ( 3
12 ) 26. (B) 7 : 15 =
7
= 0·466
= Rs. 7·50 15
750 15
∴ Number of 5 paise coins = = 150 15 : 23 = = 0·652
5 23

Quantitative Aptitude Test | 74


More PDF download : http://www.gksolve.com

17 38. (D) 0·6 x = 0·09 y


17 : 25 = = 0·68
25 x 0·09 9 3

21 y = 0·60 = 60 = 20 = 3 : 20
and 21 : 29 = = 0·724
29 39. (B) 10% of x = 20% of y
∴ The smallest one is 7 : 15. 10 20
27. (C) ⇒ x = y
100 100
1 1 1 x y x 10 2
28. (A) ˙·˙ A = B= C=x ⇒ = ⇒y= =
3 4 5 10 5 5 1
Then A = 3x, B = 4x and C = 5x Hence x:y = 2:1
∴ A : B : C = 3x : 4x : 5x = 3 : 4 : 5 A 2 B 4
40. (A) = and =
1 B 3 C 5
of Rs. 9·30
29. (B) Req. ratio
3
0·6 of Rs. 1·55
=
3·10 310 10
=
0·93 93
=
3

A
C ( ) ( ) ( )
=
A B
B C
× =
2 4
3 5
× =
8
15
30. (D) Let the numbers be 3x and 5x C 15
So, =
3x + 10 A 8
5
Then = ⇒ 7 (3x + 10) = 5 (5x + 10) Hence C : A = 15 : 8
5x + 10 7
41. (B) Let the salaries of Laxman and Gopal one year
⇒ 4x = 20 ∴ x = 5 before be x1, y1 respectively.
So, the numbers are 15, 25. x1 3
∴ y1 = 4
x 2 2 1 …(1)
31. (B) ˙·˙ y = 3 and x = 2 x2 + y2 = 4160 …(2)
x 2 2 1
⇒ y ×x = 3 ×2
3
y2 = y1 = ×
2 2
3
( ) 4
3 1
x = × ×
3 4
2 3( ) 4
x
5 2
2 1 3 4 4 8
⇒ y = 3∴y=6 = × × x2 = x2
2 3 5 5
32. (C) ˙·˙ 2A = 3B and 4B = 5C
⇒ 8A = 12B and 12B = 15C
8
5 ( )
∴ x2 + y2 = x2 + x2 = 1 +
8
5
=
13
5 2
x = Rs. 4160

⇒ 8A = 12B = 15C = x 5
∴ x2 = × 4160 = 5 × 320 = Rs. 1600.
x x x 13
∴ A = ,B= ,C=
8 12 15 42. (C) Let their ages be 2x and 3x years
x x ˙·˙ 3x – 2x = 6 ⇒ x = 6
So, A:C = : = 15 : 8
8 15 ∴ Sumit’s age = 12 years,
33. (C) Let 2A = 3B = 4C = x Prakash’s age = 18 years
x x x After 6 years, Sumit’s age = 18 years
Then A = , B = and C =
2 3 4 After 6 years, Prakash’s age = 24 years
x x x ∴ Required ratio of their ages = 18 : 24 = 3 : 4
∴ A:B:C = : :
2 3 4 43. (C) Let their ages be 2x and 3x years
Hence A:B:C = 6:4:3 2x + 12 11
=
34. (C) ˙·˙
A
D
= ( A B C
× ×
B C D ) ( =
2 4 6
× ×
3 5 7 ) =
16
35
3x + 12 15
⇒ 15 (2x + 12) = 11 (3x + 12)
∴ A : D = 16 : 35 ⇒ 3x = 48
35. (A) Let A = x ∴ x = 16
Then B = 3x, C = 2 B = 6x ∴ Age of Sumit = 3x = 48 years
∴ A : B : C = x : 3x : 6x = 1 : 3 : 6 44. (C) Let their ages be 4x and x years.
36. (B) L.C.M. of 7 and 6 is 42 ˙·˙ 4x × x = 196
∴ x2 = 49 ⇒ x = 7
∴ A : B = 5 : 7 = 30 : 42
Their ages are 28 years and 7 years.
and B : C = 6 : 11 = 42 : 77
Ratio of their ages after 5 years = 33 : 12 = 11 : 4
Hence A : B : C = 30 : 42 : 77 45. (C) Let their ages be 6x and 5x years
37. (C) A:B = 7:9 ˙·˙ 6x + 5x = 44 ∴ x = 4
and B : C = 3 : 5 = 9 : 15 So their present ages are 24 years and 20 years
∴ A : B : C = 7 : 9 : 15 Ratio of their ages after 8 years = 32 : 28 = 8 : 7.

Quantitative Aptitude Test | 75


More PDF download : http://www.gksolve.com

46. (C) Let son’s age 5 years ago = x years. 67·5 3


˙·˙ =
Then father’s age at that time = (5x) years. (17·5 + x) 1
After 2 years, son’s age = (x + 7) years. ⇒ 3 (17·5 + x) = 67·5 ∴ x = 5
After 2 years, father’s age = (5x + 7) years ∴ Water to be added = 5 kg.
˙·˙ 3 (x + 7) = 5x + 7 ∴ x = 7 55. (B) Let quantity of milk and water be 5x and x litres
Father’s age now = (5x + 5) = 40 years 5x 5
Then =
Son’s age now = (x + 5) = 12 years x+5 2
∴ Ratio of their present ages = 40 : 12 = 10 : 3 ⇒ 10x = 5x + 25 ∴ x = 5
47. (D) Their total age = (3 × 20) years = 60 years ∴ Quantity of milk = 5x = 25 litres
Let their ages be 3x, 5x and 7x years. Then 3x + 5x +
7x = 60 ⇒ x = 4 56. (D) ˙·˙ 20 kg of brass contains zinc = 13 kg
∴ Youngest girl is = 3x = 12 years old.
48. (D) Let the number of hens = x and number of
∴ 100 kg of brass contains zinc = ( 13
20 )
× 100 kg.

cows = y = 65 kg.
Then x + y = 48 and 2x + 4y = 140 57. (C) A : B = 4 : 5 = 8 : 10 and B : C = 2 : 3 = 10 : 15
Solving these equations, we get 2y = 44 ∴ y = 22 ∴ A : B : C = 8 : 10 : 15
So, x = (48 – 22) = 26 If A has Rs. 8, C has Rs. 15
∴ Number of hens = 26
49. (A) Ratio of wages of 6 men, 8 women and 6 children
If A has Rs. 800 C has Rs. ( 15
8 )
× 100 = Rs. 1500
= 6 × 4 : 8 × 3 : 6 × 2 = 24 : 24 : 12
= 2:2:1 58. (B) Alcohol = ( 20
100 )
× 15 litres = 3 litres,

∴ Total money earned by children = Rs. 950 ×( )1


5 New mix. contains alcohol = 3 litres,
Water = 12 litres

= Rs. 190 Water = 15 litres


50. (C) Let the numbers be 3x and 4x ∴ Percentage of alcohol in new mix.
Then, their L.C.M. = 12x
⇒ 12x = 180 ∴ x = 15 =( 3
18 )
× 100 %
Hence, the first number = 45 2
= 16 %
51. (C) (
Milk = 60 ×
2
3) litres = 40 litres
59. (C) M = 3E and =
M 4
3

Water = (60 – 40) litres = 20 litres H 3


40 1 3 3 9
˙·˙ = ∴ H = M = × 3E = E
20 + x 2 4 4 4
⇒ 20 + x = 80 ∴ x = 60 Now M + E + H = 250
Hence, water to be added = 60 litres. 9
⇒ 3E + E + E = 250
52. (B) ˙·˙ A : B = 2 : 3, B : C = 2 : 3 4
and C:D = 2:3 ⇒ 25 E = 1000 ∴ E = 40
⇒ A : B = 8 : 12, B : C = 12 : 18 60. (D) Data is inadequate.
and C : D = 18 : 27 61. (C) Let, 1 gm of gold be mixed with x gm of copper
∴ A : B : C : D = 8 : 12 : 18 : 27 to give (1 + x) gm of mixture.

So (
A’s share = Rs. 1300 ×
8
)
65
= Rs. 160
Now, 1G = 19W and 1C = 9W and mixture = 15W
Now, 1 gm gold + x gm copper = (1 + x) gm mixture
1 2 ˙·˙ 19 W + 9W × x = (1 + x) × 15W
53. (B) First glass contains milk = and water =
3 3 4W 4 2
Thus 4W = 6W x ∴ x = = =
1 3 6W 6 3
Second glass contains milk = and water = 2
4 4 So the required ratio is 1 : i.e. 3 : 2
3
∴ Now tumbler contains water = ( )2 3
3 4
+ =
17
12 62. (C) Let
a c
b = d=k
7 17
∴ Ratio of milk and water = : = 7 : 17 Then a = b k and c = dk
12 12
ma + nc mbk + ndk ⎛ mb + nd⎞
54. (A) Milk = 85 × ( )
27
34
kg =
135
2
kg = 67·5 kg ∴
mb + nd
=
mb + nd
=k⎜
⎝ mb + nd⎠

(
Water = 85 × ) ( )
7
34
kg =
35
2
kg = 17·5 kg
a
=k = b=a:b

Quantitative Aptitude Test | 76


More PDF download : http://www.gksolve.com

63. (B) ˙·˙ P : (Q + R) = 2 : 5


71. (C) Suppose C gets Re. 1. Then B gets Re. (14)
∴ (
P’s share = Rs. 1050 ×
2
) = Rs. 300

64. (B) ˙·˙


2 2 9
A + 40 = B + 20 = C + 10 = x
7
˙.˙ A gets = Re. (23 ×14) = Re. 16
5 7 17 1 1
∴ A:B:C = : : 1 = 2 : 3 : 12
5 7 6 4
∴ A = (x – 40), B = (x – 20)

and C =
2
17
(x – 10)
2
Hence (
B’s share = Rs. 680 ×
3
17 ) = Rs. 120
9 72. (D) ˙·˙ Remainder = Rs. [2430 – (5 + 10 + 15)]
5 7 17 = Rs. 2400
⇒ (x – 40) + (x – 20) + (x – 10) = 600
2 2 9
⇒ 45x – 1800 + 63x – 1260 + 34x – 340 = 10800 ∴ A’s share = Rs. [( 2400 ×
3
12 ) ]
+5
14200 = Rs. 605
⇒ 142x = 14200 ∴ x = = 100
142 1 1 1
5 73. (C) Ratio of sides : : = 20 : 15 : 12
Hence A’s share = (100 – 40) = Rs. 150 3 4 5
2
65. (C) (
Milk = 729 ×
7
) = 567 ml
(
Length of smallest side = 94 ×
12
47 ) cm.
9 = 24 cm.
Water = 729 ×( 2
9) = 162 ml 74.
1 1 1
(D) Ratio of time taken = : : = 20 : 15 : 12
3 4 5
567 7
˙·˙ = 75. (B) Suppose C gets Rs. x
162 + x 3
Then, B gets Rs. (x + 8) and A gets Rs. (x + 15)
⇒ 3 × 567 – 7 × 162 = 7x
⇒ x + x + 8 + x + 15 = 53 ∴ x = 10
⇒ 1701 – 1134 = 7x
567 So, A gets Rs. 25, B gets Rs. 18 and C gets Rs. 10
∴ 7x = 1701 – 1134 ∴ x = = 81 ml
7 ∴ A : B : C = 25 : 18 : 10
66. (C) Gold in C = ( 7 7
+
9 18 ) =
21 7
18 6
= 76. (C) A : B : C = 100 : 65 : 35 = 20 : 13 : 7
If C’s share is Rs. 7, the sum is Rs. 40
Copper in C = ( 2 11
+
9 18 ) =
15 5
18 6
= If C’s share is Rs. 28, the sum is Rs. ( 40
7
× 28 )
7 5 = Rs. 160
∴ Gold : Copper = : = 7 : 5
6 6 77. (C) Let their incomes be 3x, 2x and expenditures 5y,
1 1 1 3y respectively. Then,
67. (D) ˙·˙ A = B= C=x
2 3 6 3x – 5y = 1000 and 2x – 3y = 1000
⇒ A = 2x, B = 3x, C = 6x Solving these equations we get x = 2000, y = 1000
∴ A:B:C = 2:3:6 ∴ A’s Income = 3x = Rs. 6000
(
Third part = Rs. 1870 ×
6
11 ) = Rs. 1020 78. (D) Ratio of amounts collected from 1st and 2nd
class
68. (A) A : (B + C) = 2 : 9 = (4 × 1 : 1 × 40) = (1 : 10)
∴ (
A’s share = Rs. 385 ×
2
) = Rs. 70 ∴ Amount collected as 1st class fare

69. (D) ˙·˙ 5A = 3B = 2C = x


11
(
= Rs. 1100 ×
1
11 )= Rs. 100
x x x 79. (C) Let the number of students be 2x, 3x and 5x
⇒ A = , B = and C =
5 3 2 ˙·˙ (2x + 20) : (3x + 20) : (5x + 20) = 4 : 5 : 7
x x x 2x + 20 3x + 20 5x + 20
∴ A : B : C = : : = 6 : 10 : 15 ⇒ = =
5 3 2 4 5 7
1 1 ⇒ 5 (2x + 20) = 4 (3x + 20) ∴ x = 10
70. (B) ˙·˙ A: B = 3:4
5 8 Hence, total number of students before increase
8A 120 = 10x = 100.
⇒ =
5B 160 80. (A) If material cost Rs. 3, the cost of the article is
A 120 5 15 = Rs. (3 + 4 + 1) = Rs. 8
⇒ = × =
B 160 8 32 If material cost Rs. 67·50, the cost of the article
∴ (
First part = Rs. 94 ×
15
47) = Rs. 30 = Rs. ( 8
3 )
× 67·50 = Rs. 180

Quantitative Aptitude Test | 77


More PDF download : http://www.gksolve.com

Logarithm
Important Points/Facts Q. 4. If log x = log 5 + 2 log 3 – Short Method : Applying the
1 formula, we have
We are familiar with a simple log 25, find the value of x.
2
exponential identity ax = b, Here ‘a’ 2
Solution : x = =2
is called the base ‘x’ the exponent 2–1
and ‘b’ the result. log x = log 5 + 2 log 3 Q. 9. If log (x – 2) = log x –
1
Now, just as we can say ⎯ √ 4 = 2, – log 25 log (2) then find the value of x.
which is basically another way of 2 Solution :
saying 2 × 2 = 4 we can say = log 5 + log 32 – log We have,
loga b = x (25) 1/2 log (x – 2) = log (x) – log (2)
It is another way of saying ax = b = log 5 + log 9 – log 5
= log x/2
Thus a log or logarithm is an = log 9
x
equivalent way of expressing an ∴ x = 9 ⇒ x–2 =
2
exponential identity and the follow- Q. 5. If log10 m = b – log10 n,
ing two expressions are completely ⇒ 2x – 4 = x
find the value of m.
equivalent. ∴ x = 4
Solution :
ax = b ⇔ loga b = x
We have, log10 m = b – log10 n Short Method : Applying the
loga b generally expressed as log formula, we have
of b to the base a generally, the base ⇒ log10 m + log10 n = b
(2) 2
is taken as 10 in which case the ⇒ log10 (mn) = b x = =4
2–1
subscript for the base is not written. ⇒ 10b = mn Q. 10. Find the no. of digits in
Hence log b means log1 0 b . 10b 247 (Given that log10 2 = 0·3010)
Thus, if no base is given assume that ∴ m = n
the base is 10. Solution :
Q. 6. If log10 (m) = b + log10 (n), Applying the rule, we have the
Examples find the value of m. required answer = (Integral part of
Solution : 47 log10 2) + 1
Q. 1. If log3 a = 4, find the value We have log10 m = b + log10 n = (47 × 0·3010) + 1
of a.
⇒ log10 m – log10 n = b = [14·1470 + 1]
Solution :
= 14 + 1 = 15
log3 a = 4 ⇒ 34 = a
a = 81
⇒ () m
log n = b

m Exercise
Q. 2. Find the value of 2log2 5. b
n = 10

01. If A = log27 625 + 7 log11 13 and
Solution :
∴ m = n 10 b B = log9 125 + 13 log10 7 then
Let, 2log2 5 = x which of the following is true ?
∴ log2 (x) = log2 (5) ()
log n
m

log m
log n (A) A > B (B) A < B
⇒ x = 5 (C) A = B (D) Can’t say
Q. 7. If log10 m = b log10 n, find
∴ 2 25 = 5
log
the value of m. 02. If log 2 = 0·3010, then the
Short Method : Applying the Solution : number of digits in 264 is—
formula we can directly get the log10 m (A) 18 (B) 19
answer We have = b (C) 20 (D) 21
log10 n
2log2 5 = 5
⇒ logn m = b 03. Find the number of digits in 810.
Q. 3. Find the value of ∴ m = nb (Given that log 10 2 = 0·3010) :
log25 125 – log8 4 Q. 8. If log (x + 2) = log (x) + (A) 19 (B) 20
Solution : log (2) then find the value of x. (C) 17 (D) 10
log25 (125) – log8 (4) Solution : 04. Find the no. of digits in 8 57
= log5 2 (53) – log2 3 (22) We have log (x + 2) = log (x) + (given that log10 2 = 0·3010)
3 2 5 log (2) = log (2x) or x + 2 = 2x (A) 52 (B) 50
= – (from the formula) =
2 3 6 ∴ x = 2 (C) 51 (D) 53

Quantitative Aptitude Test | 78


More PDF download : http://www.gksolve.com

05. If log (x – 5) = log (x) – log (5) 75 24. If log 90 = 1·9542 then log 3
15. The simplified form of log
and log (y – 6) = log (y) – log (6) 16 equals to—
then which of the following is 5 32 (A) 0·9771 (B) 0·6514
– 2 log + log is—
correct ? 9 343 (C) 0·4771 (D) 0·3181
(A) x > y (B) x < y (A) log 2 (B) 2 log 2 –
(C) log 3 (D) log 5 25. If log (0·57) = 1·756, then the
(C) x = y (D) Can’t say value of log 57 + log (0·57)3 +
16. If log 2 = 0·3010 then log 5
06. If log (x + 4) = log (4) + log (x) equals to— log ⎯
√⎯⎯
0·57 is—
and log (x + 6) = log (y) + log (6) (A) 0·902 (B) 1·902
(A) 0·3010
then which of the following is – –
(B) 0·6990 (C) 1·146 (D) 2·146
correct ?
(C) 0·7525
(A) x = y (B) x < y 26. If log 2 = x, log 3 = y and log 7 =
(D) Given log 2, it is not
(C) x > y (D) Can’t say possible to calculate log 5 3
z, then the value of log (4 × ⎯
√⎯63)
loga x 17. If log10 2 = 0·3010 and log10 7 = is—
07. The value of – loga b is— 0·8451, then the value of log10
logab x 2 1
2·8 is— (A) – 2x + y + z
3 3
(A) 0 (B) 1
(A) 0·4471 (B) 1·4471 2 1
(C) a (D) ab (B) 2x + y + z
(C) 2·4471 (D) 14·471 3 3
08. The value of log2 3 × log3 2 × 18. If log10 2 = 0·301, then the value 2 1
(C) 2x + y – z
log3 4 × log4 3 is— of log 10 (50) is— 3 3
(A) 1 (B) 2 (A) 0·699 (B) 1·301 2 1
(D) 2x – y + z
(C) 3 (D) 4 3 3
(C) 1·699 (D) 2·301
27. If log 3 = 0·477 and (1000) x = 3,
09. If a x = b, b y = c, cz = a, then the
value of xyz is— 19. Find the value of log ()a2
bc + then x equals to—
(A) 0·159 (B) 10
(A) 0
(C) 2
(B) 1
(D) 4
()b2
( )
c2
log ac + log ab : (C) 0·0477 (D) 0·0159
(A) 0 (B) 1 28. If 2log4 x = 1 + log4 (x – 1), find
10. If logx y = 100 and log2 x = 10 (C) abc (D) a2 b2 c2 the value of x.
then the value of y is—
1 (A) 2 (B) 1
(A) 210 (B) 21000 20. Find the value of log 8 + log —
100
8 (C) 4 (D) 3
(C) 2 (D) 210000
(A) 0 (B) 1 29. If 5 5 – x = 2x – 5, find the value of
11. If logx 4 = 0·4 then the value of x (C) 2 (D) log (64) x.
is—
21. The equation loga x + loga (1 + (A) 5
(A) 4 (B) 16 x) = 0 can be written as— (B) 0
(C) 1 (D) 32 (A) x2 + x – 1 = 0 (C) 1
12. If log12 27 = a then log6 16 is— (B) x2 + x + 1 = 0 (D) Can’t be determined
4 (3 – a) 4 (3 + a) (C) x2 + x – e = 0
(A) (B) 30. If log8 x + log4 x + log2 x = 11,
3+a 3–a (D) x2 + x + e = 0 then the value of x is—
3+a 3–a
(C) (D) (A) 2 (B) 4
4 (3 – a) 4 (3 + a) 22. Find the value of log x + log
(C) 8 (D) 64
13. Given that log10 2 = 0·3010, then
log2 10 is equal to— (1x)— 31. If 100·3010 = 2, then find the
value of log0·125 125.
(A) 0·3010 (B) 0·6990 (A) 0 (B) 1
699 699
1000 699 1 (A) (B) –
(C) (D) (C) – 1 (D) 301 301
301 301 2
(C) – 1 (D) – 2
9 27 1
14. The value of log – log 23. Find the value of log 25 – 2 32. Find the value of log0·125 64—
8 32 2
3 log10 3 + log10 18— (A) – 2
+ log is—
4 (A) 0 (B) 1 (B) 2
(A) 0 (B) 1 1 (C) 0
(C) 2 (D)
(C) 2 (D) 3 2 (D) Can’t be determined

Quantitative Aptitude Test | 79


More PDF download : http://www.gksolve.com

33. Find the value of log 3 2 28 + 43. The value of log6 log5 15625 is— 51. (log tan 1°. log tan 2°…………
log243 3 7 – log36 1296— (A) 1 log tan 50°) is—
(A) 3 (B) 2 (B) 2 (A) 0 (B) 1
(C) 1 (D) 0 (C) 3 (C) 2 (D) –1
34. Find the value of log49 16807 – (D) None of these 52. The mantissa of log 3274 is
log9 27— 0·5150, then the value of
1
44. If log10000 x = – , then x is— log 32·74 is—
(A) 0 (B) 1 4
(A) 1·5150
3 1
(C) (D) – 1 (A) (B) 2·5150
2 100
(C) 0·5150
35. Find the value of log9 81 – log4 1
(B) –
32— 10 (D) 1·5150
1 3 1 53. If log10 (10x) = 2·7532, then
(A) (B) – (C)
2 2 20 log10 (10000x) is—
1 (D) None of these
(C) – (D) 2 (A) 4·7532
2
36. log10 10 + log 10 100 + log 10 1000
45. logx (1625) = – 21, then x is— (B) 5·7532
(C) 3 × 2·7532
+ log10 10000 + log10 100000 is 625 (D) None of these
(A)
equal to— 256
log 125
(A) 15 256 54. If = x, then x is equal
(B) log 5
(B) log 11111 625 to—
(C) log101111 526 (A) 2 (B) 3
(C)
(D) 14 log10 100 265 1
(D) None of these (C) 4 (D)
37. log10 x + log10 y = z, then x is 2
equal to—
z
46. The value of log2 (641 ) is— 55. If log5 (x 2 + x) – log5 x = 2, then
the value of x is—
(A) y (A) 6 (A) 24 (B) 25
10 (B) –6 (C) 23 (D) 120
(B) z (C) 7
56. (log5 3) × (log3 625) is equal
102 (D) None of these
(C) x to—
1 (A) 1 (B) 2
(D) None of these 47. If loga 3 = , then value of a is—
3 (C) 3 (D) 4
38. log–1/3 81 is equal to— (A) 27 57. log9 27 – log27 9 is equal to—
(A) – 27 (B) – 4 (B) 81
6 5
(C) 4 (D) 127 (C) 72 (A) (B)
5 6
39. If log10 {log10 [log10 (log10 x)]} (D) None of these
(C) 3 (D) 32
= 0, then the value of x is— 48. If log10 x = 7, then value of x
58. The value of 3– 1/2 log39 is—
(A) 1010 (B) 10 1010 is—
2 (A) 1010 (A) 3
(C) 10– 10 (D) 1010 1
(B) 107 (B)
40. The value of 25 log5 4 is— 3
(C) 710
(A) 16 2
(D) None of these (C)
(B) 5 3
(C) 25 49. If px = q, then— (D) None of these
(D) None of these (A) logp , x = q 59. If 10x = 1·73 and log10 1730 =
41. The value of log10 0·000001 is— (B) logx q = p 3·2380, then x is equal to—
(A) 6 (B) – 6 (C) logp q = x (A) 1·2380 (B) 0·2380
(C) 5 (D) – 5 (D) logq p = x (C) 2·380 (D) 2·2380
42. The value of log10 (0·00001) is—
50. Given that log10 2 = 0·3010 the 60. If log a, log b, log c are in A.P.
(A) – 5
value of log10 5 is— then—
(B) – 6
(C) – 7 (A) 0·3241 (B) 0·6911 (A) a, b, c are in G.P.
(D) None of these (C) 0·6990 (D) 0·7525 (B) a2 , b2 , c2 are in G.P.

Quantitative Aptitude Test | 80


More PDF download : http://www.gksolve.com

(C) a, b, c are in A.P. 12% per annum compounded loga n


annually is— 68. The value of is given
(D) None of these logab n
(A) 25350 (B) 23550 by—
61. The population of a town at the (C) 2550 (D) 25550
beginning of the year 1986 was (A) 1 + logab (B) 1 + logb a
2,65,000. If the rate of increase 64. The number of digits in the (C) loga b (D) logb a
be 52 per thousand of the numeral for (8·75) 16—
population. Find the population (A) 47 digit (B) 48 digit 69. Given log10 2 = 0·30103, log10 3
at the beginning of the year (C) 49 digit (D) 50 digit = 0·47712. Find the number of
1991. 65. The number of digits in the digit, in 3 12 × 28—
(A) 3,40,400 (B) 3,41,400 numeral for 264— (A) 6
(C) 3,42,400 (D) 3,43,400 (A) 18 digit (B) 19 digit (B) 7
62. What rate per cent per annum (C) 20 digit (D) 21 digit (C) 8
compound interest will Rs. 2000 5 (D) 9
amount to Rs. 3,000 in 3 years if 66. The value of ⎯
√⎯⎯
42·7—
the interest is reckoned half (A) 2·1187 (B) 2·1287 1 1
70. The value of + is—
yearly ? (C) 2·8711 (D) 2·2287 log2 π log6 π
(A) 12% (B) 13% (A) greater than 1
3
(C) 14% (D) 15% 67. The value of ⎯
√⎯⎯⎯⎯
0·0847— (B) less than 1
(A) ·4392 (B) ·4239 (C) between 5 and 6
63. The compound interest on Rs.
12,000 for 10 years at the rate of (C) ·2349 (D) ·4329 (D) None of these

Answers with Hints


01. (B) A = log27 625 + 7 log1113 4 4
06. (C) ˙·˙ x = =
= log3 3 54
+ 7 log1113 4–1 3
5 5
4 y = =
= log3 5 + 7 log11 13 5–1 4
3
∴ x > y
B = log9 125 + 13 log11 7 = log32 53
logab x
+ 13 log117 07. (B) ˙·˙ loga x =
logab a
3
= log3 5 + 13 log11 7 logab x
2 ∴ The given expression = – loga b
logab a
Let log 3 5 = x and by the above rule
logab x
7 log11 13 = 13 log11 7
1 ab
4 = – loga b = loga ab – loga b = loga b
Therefore, A = x + 13 log11 7 logaba
3
= loga a = 1
3
and B = x + 13 log11 17 log 3 log 2 log 4 log 3
2 08. (A) Given Exp. = × × × =1
log 2 log 3 log 3 log 4
Clearly, A < B hence (B) is the correct answer. 09. (B) ˙·˙ ax = b ⇒ loga b = x
02. (C) Required answer = [64 log10 2] + 1 ⇒ b = cy ⇒ logb c = y
= [64 × 0·3010] + 1 = [19·264] + 1 = 19 + 1 = 20 ⇒ cz = a ⇒ logc a = z
03. (D) 810 = (23 )10 = 230 ∴ x × y × z = loga b × logb c × logc a = 1
∴ Required answer = [30 log10 2 + 1] 10. (B) logx y = 100, log2 x = 10
= [30 × 0·3010] + 1 = (9·03) + 1 = 9 + 1 = 10 log y log x
⇒ = 100 and = 10
04. (A) 857 = (23 )57 = 2171 log x log 2
log y
∴ Required answer = (171 log10 2 + 1) ⇒ = 100 × 10 = 1000
log 2
= [171 × 0·3010] + 1 = [51·4710] + 1 ⇒ log2 y = 1000 ∴ y = 21000
= 51 + 1 = 52 log 4 2
25 1 11. (D) ˙·˙ logx 4 = =
05. (B) ˙·˙ x = =6 log x 5
4 4 2 log 2 2
⇒ =
36 1 log x 5
and y = =7
5 5 ⇒ log x = 5 log 2 = log 2 5 = log 32
∴ x < y ∴ x = 32

Quantitative Aptitude Test | 81


More PDF download : http://www.gksolve.com

12. (A) ˙·˙ log12 27 = a 1


22. (A) log x + log x = log x + log 1 – log x
log 27
⇒ = a
log 12 = log 1 = 0
⇒ a log 12 = log 33 1
23. (B) log10 25 – 2 log10 3 + log10 18
⇒ a log (3 × 4) = 3 log 3 2
⇒ a [log 3 + log 4] = 3 log 3 = log10 (25)1/2 – log10 (3)2 + log10 18
⇒ a log 4 + a log 3 = 3 log 3 = log10 5 – log10 9 + log10 18
5 × 18


a log 22 = (3 – a) log 3
2a log 2 = (3 – a) log 3
= log10 ( ) 9
= log10 10 = 1

24. (C) log 90 = 1·9542


log 2 3–a
∴ = … (1) ⇒ log (3 2 × 10) = 1·9542
log 3 2a
⇒ 2 log 3 + log 10 = 1·9542
log 16 log 24 4 log 2
Now log6 16 = = = 0·9542
log 6 log (2 × 3) log 2 + log 3 ∴ log 3 = = 0·4771
2
log 2 4 ⎛⎜ 3 – a⎞⎟ 25. (A) Given Exp.
4 ⎝ 2a ⎠ 4 (3 – a) 57 × 100
=
log 3
log 2
+1
=
3–a
+ 1
=
(3 + a)
= log ( 100 ) 1
+ 3 log (0·57) + log (0·57)
2
log 3 2a = log (0·57) + log 102 + 3 log(0·57)
log 10 1 1·0000 1000 1
13. (C) log2 10 = = = = + log (0·57)
log 2 log 2 0·3010 301 2
14. (A) Given expression,
(
= 1+3+
1
) log (0·57) + 2 [˙.˙ log 102 = 2]
= log ( 9 27 3
÷ ×
8 32 4 ) (
= log
9 3 32
× ×
8 4 27 ) –
2

= (4·5 × 1·756) + 2 = 4·5 × (– 1 + 0·756) + 2


= log 1 = 0
= 3·402 – 4·5 + 2
75 5 32
15. (A) Given Exp. = log – 2 log + log = 0·902
16 9 343
25 × 3 25 16 × 2 3
= log – log + log 26. (B) Exp. = log (4 × ⎯ √⎯63) = log [2 2 × (3 × 3 × 7)1/3]
4×4 81 81 × 3
= log 2 + log (3 × 3 × 7)1/3
2
= log (25 × 3) – log (4 × 4) – log (25) + log 81
1
+ log (16 × 2) – log (81 × 3) = 2 log 2 + log (32 × 7)
3
= log 25 + log 3 – log 16 – log 25 + log 81 1
+ log 16 + log 2 – log 81 – log 3 = 2 log 2 + [log 32 + log 7]
3
= log 2 2 1
10 = 2 log 2 + log 3 + log 7
16. (B) log 5 = log = log 10 – log 2 3 3
2 2 1
= 1 – 0·3010 = 0·6990 = 2x + y + z
3 3
28 27. (A) ˙·˙ (1000) x = 3
17. (A) log10 2·8 = log10 = log 28 – log 10
10 ⇒ x log 103 = log 3
= log (7 × 4) – log 10 = log 7 + 2 log 2 – log 10 ⇒ 3x = log 3
= 0·8451 + 2 × 0·3010 – 1
log 3 0·477
= 0·8451 + 0·6020 – 1 = 0·4471 ∴ x = = = 0·159
3 3
50 × 2
18. (C) log10 50 = log10 = log 100 – log 2 28. (A) ˙·˙ 2 log4 x = 1 + log4 (x – 1)
2
= log10 10 2 – log 2 ⇒ log4 x2 = log4 4 + log4 (x – 1)
= 2 – 0·301 = 1·699 ⇒ x2 = 4 (x – 1)
⇒ x – 4x + 4 = 0
2
⎛ a2 b2 c2⎞
19. (A) Given expression = log ⎜ 2 2 2⎟ = log 1 = 0 ⇒ (x – 2)2 = 0
⎝a b c ⎠
∴ x = 2
20. (A) log 8 + log (18) = log (8 ×18) = log 1 = 0 29. (A) ˙·˙ 55 – x = 2x – 5
21. (A) loga x + loga (1 + x) = 0 ⇒ 55 – x = 2– (5 – x)
⇒ loga x (x + 1) = loga 1 (since log 1 = 0) ⇒ (5 – x) log 5 = – (5 – x) log 2
⇒ x (x + 1) = 1 ⇒ (5 – x) log 5 + (5 – x) log 2 = 0
∴ x2 + x – 1 = 0 ⇒ (5 – x) {log 5 + log 2} = 0

Quantitative Aptitude Test | 82


More PDF download : http://www.gksolve.com

⇒ (5 – x) {log 102 + log 2} = 0 39. (B) If log10 {log10 [log 10 (log 10 x)]}
⇒ log10 {log10 (log 10 x)}
=
=
0
1
⇒ (5 – x) {log 10 – log 2 + log 2} = 0 ⇒ log10 (log 10 x) = 10
⇒ 5–x = 0 ⇒ log10x = 1010
∴ x = 5 ∴ x = 101010
1 1
30. (D) ˙·˙ log2 3 x + log22 x + log2 x = 11 40. (A) Exp. = (25) log5 4 = 52 log5 4
1 1 = 5 log5 4 2
⇒ log2 x + log2 x + log2 x = 11
3 2
= 16
⇒ (1 1
)
+ + 1 log2 x =
3 2
11 41. (B)
42. (A)
Exp. = log10 10 –6 = – 6
Exp. = log10 10 – 5 = – 5
11
⇒ log2 x = 11 43. (A) log6 log5 15625 = log6 log5 (5)6
6
11 × 6 = log6 6 (log5 5) = 1
⇒ log2 x = =6
11 1
44. (B) ˙.˙ log104 x = –
∴ x = 26 = 64 4
1
3
31. (B) Exp. = log0·125 125 = log2 – 3 53 = – log2 5 ⇒ x = (104)–4
3
1
= – log2 5 =
10
˙.˙ 10 0·3010 = 2
⇒ log10 2 = 0·3010 45. (A) ˙·˙ logx ()
16
25
= –
1
2
10 16 1
˙·˙ log10 5 = log10 = log10 10 – log10 2 ⇒ = (x)– 1/2 = 1/2
2 25 x
= 1 – 0·3010 = 0·6990 625
log105 0·6990 699 ∴ x =
∴ – log2 5 = – =– =– 256
log102 0·3010 301
32. (A) Exp. = log0·125 64 = log2 – 3 26 46. (B) log2 (641 ) = log 1 – log 2
2 2
6

6 = 0–6=–6
= log22 = – 2 [˙.˙ log2 2 = 1]
(– 3) 1
33. (C) Exp. = log32 2 8 + log243 3 7 – log36 1296 47. (A) ˙.˙ loga 3 =
3
= log2 5 28 + log35 37 – log36 36 2 ⇒ 3 = (a) 1/3
8 7 ⇒ a = 27
= log2 2 + log3 3 – 2 log36 36
5 5 48. (B) log10 x = 7
8 7 then x = 107
= + –2=1
5 5
49. (C) ˙.˙ px = q
34. (B) Exp. = log49 16807 – log9 27
⇒ logp px = logp q
5 3
= log7 2 75 – log3 2 33 = log7 7 – log3 3 ⇒ x logp p = logp q
2 2
5 3 ∴ x = logp q (˙.˙ logp p = 1)
= – =1

35. (C)
2 2
4 5
log3 2 34 – log2 2 25 = – = –
1
50. (C) log10 5 = log10 (102)
2 2 2 = log10 10 – log10 2
36. (A) = 1 – 0·3010
37. (D) log10 xy = z = 0·6990
⇒ xy = 10 z 51. (A) ˙.˙ log tan 45 = 0
10 z Hence, Whole expression = (something) × zero
⇒ x = y 52. (A)
38. (B) Let log– 1/3 81 = x 53. (B) log10 (103 . 10x) = log10 10 3 + log10 (10x)
= 3 + 2·7532
⇒ 81 = – ( ) 1 x
3
log 125
= 5·7532
54. (B) If = x
⇒ 34 = –( ) 1 x
3
= 3–x log 5
3 log 5
then x = =3
∴ x = –4 log 5

Quantitative Aptitude Test | 83


More PDF download : http://www.gksolve.com

(x x+ x) = 2
2 = 5·5332
55. (A) ˙·˙ log5 ∴ P = antilog (5·5332) = 341400


log5 (x + 1) = 2
x + 1 = 25
62. (C) ˙·˙ (
3000 = 2000 1 + )
r 6
200
∴ x = 24
log 3 log 5

3
2 ( )
= 1+
r 6
200
56. (D) (log5 3) × log3 5 4 = ×4
= 4
log 5 log 3 ⇒ 1+
r
200
=() 3 1/6
2

57. (B) log9 27 – log27 9 = log9 27 –


1
log9 27
⇒ (
log 1 +
r
200 ) 1
= (log 3 – log 2)
6
But log9 27 = log9 (9 × 3)
= log9 9 + log9 3
⇒ (
log 1 +
r
200 ) 1
= (0·4771 – ·3010)
6
= log9 9 + log9 9 1/2 = 0·02935

= 1+ =
1 3
2 2
⇒ ( 1+
r
200 ) = antilog (·02935)

3 2 r 7
∴ log9 27 – log27 9 = – ⇒ 1+ = 1·070 = 1 +
2 3 200 100
9–4 ∴ r = 14%
=

=
5
6 63. (A) ˙·˙ ( )
A = 12,000 1 +
12 10
100

Short-cut Method :
6
= 12000 ( )
28 10
25
Given Exp. = log9 27 – log27 9 ⇒ log A = log 12000 + 10 [log 28 – log 25]
⇒ log A = 4·0792 + 10 (1·4472 – 1·3979)
log 27 log 9 = 4·0792 + 0·493
= –
log 9 log 27 = 4·5722
3 log 3 2 log 3 ∴ A = antilog 4·5722 = 37342
= –
2 log 3 3 log 3 C. I. = 37342 – 12000 = 25342
3 2 5 ~
– 25350
= – =
2 3 6 64. (B)
65. (C) ˙·˙ x = 264
58. (B) 3– 1/2 log3 9 = 3 log3 9 – 1/2 ⇒ log x = log2 64
1 ⇒ log x = 64 log 2
= 9– 1/2 =
3 = 64 × ·3010 = 19·264
1730 ∴ No. of digits = 19 + 1 = 20
59. (B) ˙.˙ 10x =
1000 5
∴ log 10x = log10 1730 – log10 1000 66. (A) Let x = √⎯⎯⎯
42·5
⇒ x = 3·2380 – 3 then log x = log (42·5)1/5
= 0·2380 1
= log 42·5
60. (A) ˙.˙ log a, log b, log c are in A.P. Then, 5
⇒ log b – log a = log c – log b 1
= × (1·6304) = 0·3260
b c 5
⇒ log a = log b ∴ x = antilog (0·3260) = 2·1187
b c 67. (A) Same as Q. 66.
⇒ a = b log n
⇒ b2 = ac loga n log a
68. (A) =
∴ a, b, c are in G.P. logab n log n
61. (B) We have r = Rate of increase log (a.b)
52 log (a.b)
= × 100 =
1000 log a
= 5·2, n = 5, P0 = 265000 log a + log b
=
˙·˙ P = 265000 1 +( )
5·2 5
100
= 1+
log a
log b
= 1 + loga b
⇒ log P = log 265000 + 5 (log 105·2 – log 100) log a
= 5·4232 + 5 (2·0220 – 2) 69. (D)
= 5·4232 + 0·1100 70. (A)

Quantitative Aptitude Test | 84


More PDF download : http://www.gksolve.com

H. C. F. & L. C. M.
Factors and Multiples—If a Solution : Required number 7. L. C. M. of 22, 54, 108, 135 and
number x divides another number y = (L. C. M. of 6, 7, 8, 9 and 12) 198 is—
exactly, we say that x is a factor of y. +2 (A) 330 (B) 1980
Also in this case y is called a multiple = (504 + 2) = 506. (C) 5940 (D) 11880
of x.
777 8. L. C. M. of 87 and 145 is—
Highest Common Factor (H. Q. 6. Reduce to lowest
1147 (A) 870 (B) 1305
C. F. or G. C. D. or G. C. M.)—The terms. (C) 435 (D) 1740
H. C. F. of two or more than two
Solution : H. C. F. of 777 and 9. Which of the following is a pair
numbers is the greatest number that
1147 is 37. of co-primes ?
divides each one of them exactly.
On dividing the numerator and (A) (14, 35) (B) (18, 25)
The Highest Common Factor is
denominator by 37, we get (C) (31, 93) (D) (32, 62)
also known as Greatest Common
Divisor or Greatest Common Mea- 777 21 10. H. C. F. of 23, 32 and 15 is—
=
sure. 1147 31 (A) 23 (B) 32
H. C. F. by Factorization— (C) 1 (D) 360
Express each of the given numbers as Exercise
11. H. C. F. of 42, 63 and 140 is—
the product of prime factors. Now, 1. Which of the following fractions (A) 14 (B) 9
choose common factors and take the is the greatest of all ?
product of these factors to obtain the (C) 21 (D) 7
7 6 4 5
required H. C. F. , , , — 12. H. C. F. of 1485 and 4356 is—
8 7 5 6
(A) 189 (B) 89
Examples 6 4
(A) (B) (C) 99 (D) 83
7 5
1095
Q. 1. Find the smallest number 5 7 13. in simplest form is—
(C) (D) 1168
exactly divisible by 12, 15, 20 and 6 8
27. 13 15
2 4 5 7 (A) (B)
2. L. C. M. of , , and is— 16 16
Solution : Required number = 3 9 6 12
L. C. M. of 12, 15, 20 and 27 = 540. 17 25
1 1 (C) (D)
(A) (B) 26 26
Q. 2. Find the largest number 18 36
which can exactly divide 513, 783 561
35 140 14. when reduced to lowest terms
and 1107. (C) (D) 748
9 3 is—
Solution : Required number
H. C. F. of 513, 783 and 1107 = 27. 3 6 9 13 3
3. L. C. M. of , , is— (A) (B)
4 7 8 14 4
Q. 3. Three drums contains 36
litres, 45 litres and 72 litres of oil. (A) 18 (B) 3 11 23
(C) (D)
What biggest measure can measure 3 9 14 24
(C) (D)
all the different quantities exactly ? 56 28 15. The product of two numbers is
Solution : Biggest measure 4928. If 8 be their H. C. F. find
1 3 5 7 9
4. H. C. F. of , , , , is— how many pairs of such num-
= (H. C. F. of 36, 45, 72) litres 2 4 6 8 10 bers—
= 9 litres. 1 1 (A) 3 (B) 4
(A) (B)
Q. 4. The H. C. F. of two 2 10 (C) 2 (D) 1
numbers is 4 and their L. C. M. is 9 1
(C) (D) 16. Among how many children may
576. If one of the numbers is 64, 120 120
429 mangoes and also 715 oran-
find the other number. 1 2 3 4
5. H. C. F. of , , , is— ges be equally divided ?
Solution : The other number 2 3 4 5 (A) 143 (B) 15
H. C. F. × L. C. M. (A) 1 (B) 12 (C) 18 (D) 153
=
Given number 4 1
(C) (D) 17. In a long division sum the succe-
4 × 576 5 60 ssive remainders from the first to
= = 36.
64 6. L. C. M. of 33, 4, 42 and 3 is— the last were 312, 383 and 1. The
Q. 5. Find the least number (A) 12 dividend be 86037, find the
which when divided by 6, 7, 8, 9 (B) 48 divisor and the quotient—
and 12 leaves the same remainder (C) 432 (A) 548, 157
2 in each case. (D) None of these (B) 274, 1

Quantitative Aptitude Test | 85


More PDF download : http://www.gksolve.com
(C) 1096, 158 (C) 433, 5299 exactly the lengths 7m, 3m 85cm,
(D) None of these (D) Can’t be determined 12m 95 cm, is—
(A) 15 cm (B) 25 cm
18. Find the number lying between 25. In a long divison sum the divi-
900 and 1000 which when dend is 529565 and the succes- (C) 35 cm (D) 42 cm
divided by 38 and 57, leaves in sive remainders from the first to 33. Three different containers con-
each case a remainder 23— the last are 246, 222, 542. Find tain different qualities of mix-
(A) 935 (B) 945 the divisor and the quotient— tures of milk and water, whose
(C) 925 (D) 955 (A) 561, 943, (B) 669, 493 measurements are 403 kg, 434 kg
(C) 516, 943 (D) 561, 493 and 465 kg. What biggest mea-
19. Find the sum of three numbers sure must be there to measure all
which are prime to one another 26. The product of two numbers is the different quantities exactly ?
such that the product of the first 7168 and their H. C. F. is 16. (A) 1 kg (B) 7 kg
two is 437 and that of the last Find the sum of all possible
two is 551— numbers— (C) 31 kg (D) 41 kg
(A) 91 (B) 81 (A) 640 34. Three pieces of timber 42 m, 49
(C) 71 (D) 70 (B) 860 m and 63 m long have to be
divided into planks of the same
20. Find the two numbers whose (C) 460
length. What is the greatest
L. C. M. is 1188 and H. C. F. is (D) Data inadequate possible length of each plank ?
9— 27. The sum of two numbers is 1215 (A) 7 m (B) 14 m
(A) 27, 396 and their H. C. F. is 81. How (C) 42 m (D) 63 m
(B) 9, 27 many pairs of such numbers can
be formed ? Find them— 35. The largest number which
(C) 36, 99
exactly divides 210, 315, 147
(D) Data inadequate (A) 1 (B) 2
and 168 is—
(C) 3 (D) 4
21. What least number must be sub- (A) 3 (B) 7
tracted from 1936, so that the 28. The numbers 11284 and 7655 (C) 21 (D) 4410
remainder when divided by 9, when divided by a certain num-
10, 15 will leave in each case the ber of three digits, leave the same 36. The sum of two numbers is 216
same remainder 7 ? remainder. Find the number and and their H. C. F. is 27. The
the remainder— numbers are—
(A) 46 (B) 53
(A) 54, 162
(C) 39 (D) 44 (A) 119, 15 (B) 191, 15
(B) 108, 108
22. In a school 391 boys and 323 (C) 192, 52 (D) 191, 51
(C) 27, 189
girls have been divided into the 29. 21 mangoes trees, 42 apples trees
largest possible equal classes. So (D) None of these
and 56 orange trees have to be
that there are equal number of planted in rows such that each 37. The H. C. F. of two numbers is
boys and girls in each class. What row contains the same number of 12 and their difference is also
is the number of classes ? trees one variety only. Minimum 12. The numbers are—
(A) 23 girls classes, 19 boys number of rows in which the (A) 66, 78, (B) 70, 82
classes above trees may be planted is— (C) 94, 106 (D) 84, 96
(B) 23 boys classes, 19 girls (A) 15 (B) 17 38. The product of two-digit num-
classes
(C) 3 (D) 20 bers is 2160 and their G. C. M. is
(C) 17 boys classes, 23 girls 12. The numbers are—
classes 30. Five bells begin to toll together (A) 72, 30
(D) 23 boys classes, 17 girls and toll respectively at intervals
(B) 36, 60
classes of 6, 7, 8, 9 and 12 seconds. How
many times they will toll toge- (C) 96, 25
23. An inspector of schools wishes (D) None of these
to distribute 84 balls and 180 ther in one hour, excluding the
bats equally among a number of one at the start ? 39. The H. C. F. of two numbers is
boys. Find the greatest number (A) 3 (B) 5 16 and their L. C. M. is 160. If
receiving the gift in this way— one of the numbers is 32, then
(C) 7 (D) 9
the other numbers is—
(A) 14 (B) 15
31. The least perfect square number (A) 48 (B) 80
(C) 16 (D) 12 which is divisible by 3, 4, 5, 6 (C) 96 (D) 112
24. In finding H. C. F. of two and 8 is—
numbers, the last divisor is 49 40. The product of two numbers is
(A) 900 (B) 1200 1600 and their H. C. F. is 5. The
and the quotients 17, 3, 2. Find
the numbers— (C) 25 (D) 3600 L. C. M. of the numbers is—
(A) 343, 5929 32. The greatest possible length (A) 320 (B) 1605
(B) 434, 2959 which can be used to measure (C) 1595 (D) 8000

Quantitative Aptitude Test | 86


More PDF download : http://www.gksolve.com

41. H. C. F. of three numbers is 12. 50. The greatest number which can gallons. What is the least number
If they be in the ratio 1 : 2 : 3, divide 1354, 1866 and 2762 of full casks of equal size in
the numbers are— leaving the same remainder 10 in which this can be stored without
(A) 12, 24, 36 (B) 10, 20, 30 each case is— mixing ?
(C) 5, 10, 15 (D) 4, 8, 12 (A) 64 (B) 124 (A) 21 (B) 29
(C) 156 (D) 260 (C) 33 (D) 31
42. About the number of pairs which
have 16 as their H. C. F. and 136 51. When in each box 5 or 6 dozens 57. The circumferences of the fore
as their L. C. M. We can defi- of oranges were packed three and hind wheels of a carriage are
nitely say that— dozens were remaining. There- 3 1
6 metres and 8 metres
(A) Only one such pair exists fore, bigger boxes were taken to 14 18
(B) Only two such pair exists pack 8 or 9 dozens of oranges. respectively. At any given
However still three dozens moment a chalk mark is put on
(C) Many such pairs exists oranges remained. What was the the point of contact of each
(D) No such pair exists least number of dozens of oran- wheel with the ground. Find the
2 3 5 ges to be packed ? distance travelled by the carriage
43. L. C. M. of , and is— (A) 216 (B) 243 so that both the chalkmaks are
7 14 3
(C) 363 (D) 435 again on the ground at the same
(A) 45 (B) 35 time—
(C) 30 (D) 25 52. Three persons A, B, C run along (A) 218 m (B) 217·5 m
a circular path 12 km long. They
1 3 3 4 start their race from the same (C) 218·25 m (D) 217 m
44. Which of the fractions , , ,
2 7 5 9 point and at the same time with a 58. The sum and difference of the L.
is the smallest ? speed of 3 km/hr. 7 km/hr. and C. M. and the H. C. F. of two
4 3 13 km/hr. respectively. After numbers are 592 and 518 respec-
(A) (B)
9 5 what time will they meet again ? tively. If the sum of two numbers
3 1 (A) 12 hrs (B) 9 hrs be 296, find the numbers—
(C) (D)
7 2 (C) 24 hrs (D) 16 hrs (A) 111, 185
45. Which of the following is in 53. Four bells toll at intervals of 6, (B) 37, 259
descending order ? 8, 12 and 18 minutes respecti- (C) Data inadequate
vely. If they start tolling together
3 5 7 7 5 3 (D) None of these
(A) , , (B) , , at 12 a. m. Find after what inter-
5 7 9 9 7 5 val will they toll together and 59. The smallest number, which
5 7 3 7 3 5 how many times will they toll when divided by 20, 25, 35 and
(C) , , (D) , ,
7 9 5 9 5 7 together in 6 hours ? 40 leaves the remainder 14, 19,
46. Which of the following is in (A) 6 times 29 and 34 respectively is—
ascending order ? (B) 5 times (A) 1394 (B) 1404
5 7 9 5 9 7 (C) 4 times (C) 1406 (D) 1664
(A) , , (B) , ,
7 8 11 7 11 8 (D) Data inadequate 60. The least number, which when
7 5 9 9 7 5 54. Three pieces of timber 24 metres, divided by 35, 45 and 55 leaves
(C) , , (D) , ,
8 7 11 11 8 7 28·8 metres and 33·6 metres long the remainder 18, 28 and 38
have to be divided into planks of respectively, is—
47. The smallest number which is the same length. What is the (A) 2468 (B) 3448
divisible by 12, 15, 20 and is a greatest possible length of each
perfect square is— (C) 3265 (D) 3482
plank ?
(A) 400 (B) 900 (A) 8·4 km (B) 4·8 m 61. The smallest number, which
(C) 1600 (D) 3600 (C) 4·5 m (D) 5·4 m when diminished by 3, is divisi-
ble by 21, 28, 36 and 45 is—
48. The least number of square tiles 55. Find the least number of square
required to pave the ceiling of (A) 420 (B) 1257
tiles required for terrace 15 m
room 15 m 17 cm long and 9 m long and 9 m broad— (C) 1260 (D) 1263
2 cm broad is—
(A) 841 62. The least number, which when
(A) 656 (B) 738 divided by 16, 18 and 21 leaves
(B) 714
(C) 814 (D) 902 the remainders 3, 5 and 8 respec-
(C) 814
tively, is—
49. The largest number which divi- (D) None of these
des 77,147 and 252 to leave the (A) 893 (B) 992
56. A merchant has three kinds of
same remainder in each case is— (C) 995 (D) 1024
wine; of the first kind 403
(A) 9 (B) 15 gallons, of the second 527 63. Six bells commence tolling toge-
(C) 25 (D) 35 gallons and of the third 589 ther and toll at intervals of 2, 4,

Quantitative Aptitude Test | 87


More PDF download : http://www.gksolve.com

6, 8, 10 and 12 seconds respecti- least length of cloth that can be vely. If the first number is divi-
vely. In 30 minutes, how many measured exact number of times ded by 2, the quotient is 44. The
times do they toll together ? using any one of the above rods other number is—
(A) 4 (B) 10 is— (A) 33 (B) 66
(C) 15 (D) 16 (A) 0·96 m (B) 19·20 m (C) 132 (D) 264
(C) 9·60 m (D) 96·00 m
64. The greatest number of four 74. The least number which when
digits which is divisible by each 69. The number of prime factors in divided by 5, 6, 7 and 8 leaves a
one of the numbers 12, 18, 21 2222 × 3333 × 5555 is— remainder 3, but when divided
and 28 is— (A) 3 (B) 1107 by 9 leaves no remainder is—
(A) 9848 (B) 9864 (C) 1110 (D) 1272 (A) 1677 (B) 1683
(C) 9828 (D) 9636 (C) 2523 (D) 3363
70. The total number of prime factors
65. The least multiple of 7, which of the product (8)20, (15)24, (7)15 75. The greatest number by which if
leaves a remainder of 4, when is— 1657 and 2037 are divided the
divided by 6, 9, 15 and 18 is— remainders will be 6 and 5 res-
(A) 59 (B) 98
(A) 74 (B) 94 pectively, is—
(C) 123 (D) 138
(C) 184 (D) 364 (A) 127
71. The number of prime factors in (B) 235
66. The least number which when
the expression (6)10 × (7)17 ×
divided by 15, 27, 35 and 42, (C) 260
(11) 27 is—
leaves in each case a remainder 7
(A) 54 (B) 64 (D) 305
is—
(A) 1883 (B) 1897 (C) 71 (D) 81 76. The traffic lights at three diffe-
rent road crossings change after
(C) 1987 (D) 2007 72. What least number must be every 48 sec., 72 sec. and 108
67. The largest natural number, subtracted from 1294 so that the sec. respectively. If they all
which exactly divides the product remainder when divided by 9, change simultaneously at 8 : 20 :
of any four consecutive natural 11, 13 will leave in each case the 00 hrs. then they will again
numbers, is— same remainder 6 ? change simultaneously at—
(A) 6 (B) 12 (A) 0 (B) 1 (A) 8 : 27 : 12 hrs.
(C) 24 (D) 120 (C) 2 (D) 3 (B) 8 : 27 : 24 hrs.
68. The measuring rods are 64 cm, 73. The H. C. F. and L. C. M. of two (C) 8. 27 : 36 hrs.
80 cm and 96 cm in length. The numbers are 44 and 264 respecti- (D) 8 : 27 : 48 hrs.

Answers with Hints


7 6 4 5 ∴ L.C.M. = 2 × 3 × 3 × 3 × 11 × 2 × 5
1. (D) = 0·875, = 0·857, = 0·8 and = 0·833
8 7 5 6 = 5940
Now, 0·875 > 0·857 > 0·833 > 0·8 8. (C) H.C.F. of 87 and 145 is 29
7 87 × 145
So, is the greatest. ∴ L.C.M. = = 435
8 29
L. C. M. of 2‚ 4‚ 5‚ 7 140 9. (B) H. C. F. of 18 and 25 is 1
2. (D) L.C.M. = =
H. C. F. of 3‚ 9‚ 6‚ 12 3 So, 18 and 25 are co-primes.
L. C. M. of 3‚ 6‚ 9 18 10. (C) Clearly, 1 is the highest common factor of 23 , 32 ,
3. (A) L.C.M. = = = 18
H. C. F. of 4‚ 7‚ 8 1 and 15.
H. C. F. of 1‚ 3‚ 5‚ 7‚ 9 1 11. (D) H. C. F. of 42 and 63 is 21.
4. (D) H.C.F. = =
L. C. M. of 2‚ 4‚ 6‚ 8‚ 10 120 ˙·˙ H. C. F. of 21 and 140 is 7.
H. C. F. of 1‚ 2‚ 3‚ 4 1 ∴ H. C. F. of 42, 63 and 140 is 7.
5. (D) H.C.F. = =
L. C. M. of 2‚ 3‚ 4‚ 5 60 12. (C) H. C. F. of 1485 and 4356 is 99.
6. (C) L.C.M. = 33 × 24 = 27 × 16 = 432 13. (B) H. C. F. of 1095 and 1168 is 73.
7. (C) 2 22, 54, 108, 135, 198 Dividing Nume. and Denom. by 73
3 11, 27, 54, 135, 99 1095 15
we get =
1168 16
3 11, 9, 18, 45, 33
14. (B) H. C. F. of 561, 748 is 187.
3 11, 3, 6, 15, 11
Dividing Nume. and Denom. by 187
11 11, 1, 2, 5, 11 561 3
we get =
1, 1, 2, 5, 1 748 4

Quantitative Aptitude Test | 88


More PDF download : http://www.gksolve.com

15. (C) Let the numbers be 8x and 8y, where x and y are and (6, 22) are not prime to each other and, therefore,
prime to each other, not admissible.
then 8x × 8y = 4928 Hence the admissible pairs are
or 64xy = 4928 1, 132, 3, 44 : 4, 33, 11, 12
∴ xy = 77, ∴ x = 1 or 7 and y = 77 or 11 ∴ a = 1, b = 132; a = 3, b = 44, a = 4, b = 33,
∴ These two pairs of required numbers will be (8, 77 a = 11, b = 12
× 8) or (8 × 7, 8 × 11) that is (8, 616) or (56, 88). Hence, the required numbers are 9, 9 × 132; 9 × 3,
16. (A) The number of children required must be a 9 × 44; 9 × 4, 9 × 33; 9 × 11, 9 × 12
common factor of 429 and 715. Now the H. C. F. of or, 9, 1188, 27, 396, 36, 297, 99, 108.
429 and 715 is 143. 21. (C) The L. C. M. of 9, 10, 15 = 90
∴ The number of children required must be 143 or a On dividing 1936 by 90, the remainder = 46
factor of 143, but 143 = 13 × 11
But a part of this remainder = 7
∴ The number of children required is 143, 13 or 11.
Hence, the two numbers = 46 – 7 = 39.
17. (A) Since the last but one remainder is 383 and the
last figure to be affixed to it is 7, the last partial 22. (B) The largest possible number of persons in a class
product is 3837 – 1 = 3836. is given by the H.C.F. of 391 and 323 i.e. 17
Similarly, the other partial products will be 2740 and 391
∴ No. of classes of boys = = 23
548 17
548) 86037 (157 323
and No. of classes of girls = = 19.
548 17
23. (D) Find the H.C.F. of 84 and 180, which is 12 and
3123
this is the required answer.
2740
24. (A) ˙.˙ The last divisor = 49 and quotient = 2
3837 ∴ dividend = 49 × 2 = 98
3836 343) 5929 (17
1 98) 343 (3
The H. C. F. of these three partial products = 548
49) 98 (2
∴ The divisor = 548 or a factor of 548. But the
divisor must be greater than each of the partial ×
remainders 312, 383 and 1. Now, divisor = 98, quotient = 98 × 3 + 49 = 343
∴ The divisor is 548. Hence the quotient is 157. Again divisor = 343, quotient = 17
18. (A) The least common multiple of 38 and 57 is 114 and remainder = 98
and the multiple which is between 900 and 1000 is
∴ dividend = 343 × 17 + 98 = 5929
912.
Now, 912 + 23 i.e.; 935 lies between 900 and 1000 Hence, the required numbers are 343, 5929.
and when divided by 38 and 57 leaves in each case 25. (A) On subtracting the remainders 246, 222, 542
23 as the remainder. Therefore, 935 is number requi- from the numbers giving rise to them, the successive
red. partial products will be found to be 5049, 2244,
19. (C) From the question we see that the second number 1683.
is a common factor of the two products and since the ) 529565 (
numbers are prime to one another. It is their H. C. F. 2466
and is, therefore, 19.
2225
∴ The first number = 437 ÷ 19 = 23
542
and the third number = 551 ÷ 19 = 29
Hence, the numbers are 23, 19 and 29 Hence, the divisor must be a common factor of these
three partial product.
∴ Sum = 23 + 19 + 29
= 71. Now, 561 is their H.C.F. and no smaller factor (for
example 51) will serve the purpose, since 5049 ÷ 51
20. (A) Let the two numbers be 9a and 9b where a and b
= 99 a two-digit number which is absurd.
are two numbers prime to each other. The L.C.M. of
9a and 9b is 9ab. ∴ The divisor = 561 and the quotient = 943.
∴ 9ab = 1188 26. (A) Let the numbers be 16a and 16b, where a and b
∴ ab = 132 are two numbers prime to each other.
Now, the possible pairs of factors of 132 are 1 × 132, ∴ 16a × 16b = 7168
2 × 66, 3 × 44, 6 × 22, 11 × 12 of these pairs (2, 66) ∴ ab = 28

Quantitative Aptitude Test | 89


More PDF download : http://www.gksolve.com

Now, the pairs of numbers whose product is 28, are 32. (C) Required length
(28, 1); (14, 2), (7, 4) = (H. C. F. of 700, 385, 1295) cm
14 and 2 which are not prime to each other should be = 35 cm.
rejected.
33. (C) Biggest measure
Hence, the required numbers are
= (H. C. F. of 403, 434 and 465) kg
28 × 16, 1 × 16, 7 × 16, 4 × 16
= 31 kg
or 448, 16, 112, 64
34. (A) Greatest possible length of each plank
Hence, the required answer
= (H. C. F. of 42, 49, 63) m = 7 m
= 448 + 16 + 112 + 64 = 640.
35. (C) The largest number is the H. C. F. of 210, 315,
27. (D) Let the numbers be 81a and 81b where a and b 147 and 168, which is 21.
are two numbers prime to each other. 36. (C) Let the numbers be 27a and 27b
∴ 81a + 81b = 1215
Then, 27a + 27b = 216
1215
∴ a+b = = 15 or 27 (a + b) = 216
81
216
Now, find two numbers, whose sum is 15, the pos- or a+b = =8
sible pairs are (14, 1), (13, 2), (12, 3), (11, 4), (10, 5), 27
(9, 6), (8, 7) of these the only pairs of numbers that ∴ Values of co-primes (with sum 8) are (1, 7) and
are prime to each other are (14, 1), (13, 2), (11, 4) (3, 5)
and (8, 7). So, the numbers are (27 × 1, 27 × 7) i.e., (27, 189)
Hence, the required numbers are 37. (D) The difference of requisite numbers must be 12
(14 × 81, 1 × 81); (13 × 81, 2 × 81); (11 × 81, 4 × 81); and each one must be divisible by 12, so the numbers
(8 × 81, 7 × 81) are 84, 96.
or (1134, 81); (1053, 162); (891, 324); (648, 567) 38. (B) Let the numbers be 12a and 12b
So, there are four such pairs. Then, 12a × 12b = 2160
28. (B) The required number must be a factor of (11284 or ab = 15
– 7655) or 3692. ∴ Values of co-primes a and b are (1, 15); (3, 5)
Now, 3692 = 19 × 191 So, the two digit numbers are 12 × 3 and 12 × 5 i.e.,
191) 7655 (40 36 and 60.
764 39. (B) 40. (A) 41. (A) 42. (D) 43. (C)
15 44. (C) 45. (B) 46. (B) 47. (D)
∴ 191 is the required number and 15 is the re- 48. (C) Side of each tile
mainder. = (H. C. F. of 1517 and 902) cm
29. (B) H.C.F. of 21, 42, 56 = 7 = 41 cm
Number of rows of mango trees, apple trees and ∴ Required number of tiles
21 42 56
orange trees are = 3, = 6 and = 8 1517 × 902
7 7 7 = = 814
41 × 41
∴ Required number of rows = (3 + 6 + 8) = 17
49. (D) Required number is the H. C. F. of (147 – 77),
30. (C) L.C.M. of 6, 7, 8, 9, 12 is 504 (252 – 147) and (252 – 77) i.e., H. C. F. of 70, 105
So, the bells will toll together after 504 sec. and 175. This is 35.
In hour, they will toll together 50. (A) 51. (C)
60 × 60
= ( 504 ) times 52. (A) Time taken by A, B, C to cover 12 km is 4 hours,
12 12
= 7 times hours and hours respectively.
7 13
31. (D) 3 3, 4, 5, 6, 8 12 12
L.C.M. of 4, and = 12
2 1, 4, 5, 2, 8 7 13
2 1, 2, 5, 1, 4 So, they will meet again after 12 hours.
1, 1, 5, – 1, – 2 53. (A) L.C.M. of 6, 8, 12, 18 min. = 72 min.
L.C.M. of 3, 4, 5, 6, 8 = 1 hr. 12 min.
= 3 × 2 × 2 × 5 × 2 = 120 So, they will toll together after 1 hr. 12 min.
Required number In 6 hours, they will toll together
= (3 × 3 × 2 × 2 × 5 × 5 × 2 × 2) 6 × 60
=1+ = 5 + 1 = 6 times.
= 3600 72

Quantitative Aptitude Test | 90


More PDF download : http://www.gksolve.com

54. (B) Find the H.C.F. of 2400 cm, 2880 cm and 3360 So, required number = (L.C.M. of 16, 18, 21) – 13
cm, which is 480 cm. = (1008 – 13) = 995
Hence required answer is 4·8 metres. 63. (D) L.C.M. of 2, 4, 6, 8, 10 and 12 is 120
55. (D) Tiles are least, when size of each is largest. So, So, the bells will toll together after 120 seconds i.e. 2
H.C.F. of 1500 cm and 900 cm gives each side of a
tile, which is 300 cm. minutes. In 30 min. they will toll together in( )
30
2
+1
1500 × 900
∴ Number of tiles = ( 300 × 300) = 15
64.
times, i.e. 16 times.
(C) Required number is divisible by the L.C.M. of
56. (D) H.C.F. of 403, 527 and 589 is 31 12, 18, 21, 28 i.e. 252.
∴ Required answer = 31. Now, greatest number of four digits = 9999
57. (B) The required distance in metres On dividing 9999 by 252, the remainder is 171
87 145 ∴ Required number = (9999 – 171) = 9828.
= L.C.M. of and
14 18 65. (D) Let the least multiple of 7 be x, which when
=
L.C.M. of 87 & 145
H.C.F. of 14 & 18
= ( )
435
2
m divided by 90 leaves the remainder 4. Then, x is of
the form 90k + 4
= 217·5 m. Now, the minimum value of k for which 90k + 4 is
58. (A) Let the L.C.M. and H.C.F. be h and k respectively. divisible by 7 is 4
∴ h + k = 592 and h – k = 518 ∴ x = 90 × 4 + 4 = 364.
592 + 518 66. (B) 67. (C)
Consequently h = = 555
2 68. (C) Required length = L.C.M. of (64, 80, 96) cm
592 – 518 = 960 cm = 9·60 m.
and k = = 37
2 69. (C) The number of prime factors in the given product
i.e. L.C.M. = 555 = (222 + 333 + 555) = 1110.
and H.C.F. = 37 70. (C) Since 2, 3, 5, 17 are prime numbers and the given
expression is (23 )20 × (3 × 5)24 × (17)15 i.e. 260 × 324
Now, let the numbers be 37a and 37b, where a and b
× 5 24 × 1715, so the total number of prime factors in
are co-primes
the given expression is (60 + 24 + 24 + 15) = 123.
∴ 37a + 37b = 296 71. (B) Since 2, 3, 7, 11 are prime numbers and the given
or a+b = 8 expression is 210 × 310 × 717 × 1127, so the number of
Possible pairs of co-primes, whose sum is 8 are (1, 7) prime factors in the given expression is
and (3, 5) (10 + 10 + 17 + 27) = 64.
∴ Possible pairs of numbers are 72. (B) L. C. M. of 9, 11, 13 is 1287
(37 × 1, 37 × 7) or (37 × 259) On dividing 1294 by 1287, the remainder is 7
and (37 × 3, 37 × 5) or (111, 185) ∴ 1 must be subtracted from 1294, so that 1293
when divided by 9, 11, 13 leaves in each case the
Now, H.C.F. × L.C.M. = 555 × 37 = 20535 same remainder 6.
Also, 111 × 185 = 20535 73. (C) First number = 2 × 44 = 88
while 37 × 259 ≠ 20535 44 × 264
Second number = = 132.
Hence, the required number are 111 and 185. 88
59. (A) Here (20 – 14) = (25 – 19) = (35 – 29) 74. (B) L. C. M. of 5, 6, 7, 8 is 840
= (40 – 34) = 6 So, the number is of the form 840k + 3
∴ Required number = L.C.M. of (20, 25, 35, 40) – 6 Least value of k for which (840k + 3) is divisible by
= (1400 – 6) = 1394. 9 is k = 2
60. (B) Here, (35 – 18) = 17, (45 – 28) = 17 ∴ Required number = (840 × 2 + 3) = 1683.
and (55 – 38) = 17 75. (A) Required number
= (H. C. F. of 1657 – 6) and (2037 – 5)
∴ Required number = (L.C.M. of 35, 45, 55) – 17
= H. C. F. of 1651 and 2032 = 127.
= (3465 – 17) = 3448
76. (A) Interval of change
61. (D) Required number
= (L. C. M. of 48, 72, 108) sec. = 432 sec.
= (L.C.M. of 21, 28, 36, 45) + 3 So, the lights will simultaneously change after every
= (1260 + 3) = 1263. 432 seconds i.e. 7 min. 12 sec.
62. (C) Here (16 – 3) = 13, (18 – 5) So, the next simultaneous change will take place at
= 13 and (21 – 8) = 13 8 : 27 : 12 hrs.

Quantitative Aptitude Test | 91


More PDF download : http://www.gksolve.com

Series
Important Points/Facts (C) Convergent if x = 1 06. The series :
1. Series—The sum of the cor- (D) All are true 1 2 3
+ +
responding terms of the sequence u1, 02. Which of the following series is 1 + 2 –1 1 + 2 – 2 1 + 2 – 3
u2 , u3 , …… i.e. convergent series ? + … is—
u1 + u2 + u3 … is called a series. (A) 1 + 2 – 3 + 1 + 2 – 3 + …… (A) Convergent
A series is called a finite or (B) Divergent
1 1
infinite according as the correspond- (B) – (C) Not convergent
ing sequence is finite or infinite. 2–1 √
√ 3–1 (D) None of these
1
A series u1 + u 2 + u 3 + …… is + – …… 1 1 1
called a positive term series if un > 0, 4–1
√ 07. The series 1 – + – + … is—
2 3 4
∀n .
A series u1 – u2 + u3 …… is
(C) 1 +( ) ( )
1 1
1
+ 1+
1 2
2
+… (A) Convergent
(B) Divergent
called an alternating series if un > 0, 1 22 33
(D) 1 + 2 + 3 + 4 + … (C) Oscillatory
∀n . 2 3 4 (D) None of these
The infinite series u1 + u2 + u3 + 03. Which one of the following is
…… + un + …… is denoted by Σ un 08. Which of the following is not
false ? true ?
and the sum of the first n terms of the
2 3 4 5 1 1 1
series is denoted by S n . (A) The series + + + + (A) The series 1 – + 2 – 3 +
1 4 9 16 2 2 2
∴ S n = u1 + u2 + u3 + … + un n+1 … is convergent
2. Nature of the Series …+ 2 + … is divergent
n (B) The series 1 + 2 + 3 + … is
(i) Convergent—The infinite divergent
1 √ 2 + √ 3 +
series u1 + u2 + u3 + ……is said to be (B) The series + (C) The series 1 – 2 + 3 – 4 + 5
2 5 10
convergent. If limn → ∞ Sn – 6 + … is oscillatory
 n + … is
√ (D) Only two of the above are
= limn → ∞ (u1 + u2 + … + un ) is … + 2
n +1 true.
finite. convergent
(ii) Divergent—The infinite (C) The series whose nth term is 09. The series :
series u1 + u 2 + u 3 + … is said to be
divergent. If limn → ∞ Sn 
√ n3 + 1 – 
√n3 is convergent 1p + () ( )
1
2
p+
1·3 p
2·4
(D) The series whose nth term is
= limn → ∞ (u1 + u2 + … + un)
= + or – ∞.
1
sin n is convergent ( )
+
1·3·5 p
2·4·6
+ …… is—

(iii) Oscillatory—The infinite (A) Convergent if p ≥ 2 and


series u1 + u2 + u3 + …is said to be 04. Which of the following is true ? divergent, if p < 2
an oscillatory series. If, 1 1 1
(A) 1 + 1/3 + 1/3 + 1/3 + … is (B) Convergent if p > 2 and
2 3 4
limn → ∞ Sn = limn → ∞ (u1 + u2 + divergent, if p ≤ 2
convergent
… + un ) is not defined or oscillates (C) Convergent, if p ≤ 2 and
1 1 1
between two limits. (B) 1 + + + + … is
2 3 4 divergent, if p > 2
(iv) Geometric Series Test— convergent (D) Convergent, if p < 2 and
The geometric series a + ar + ar2 + … 1 1 1
(C) 1 + 2 + 2 + 2 + … is divergent, if p ≥ 2
is— 2 3 4
(a) Convergent when | r | < 1. convergent 10. The series
(b) Divergent when r ≥ 1 1 1 1 1 α α (α + 1)2 2
(D) k + k + k + k + … is 1+ x+ x
(c) Oscillatory when r ≤ – 1 1 2 3 4 1.β 1.2 β (β + 1)
divergent, if k > 1
Exercise α (α + 1)2 (α + 2)2
1 1 1 + x3
1.2.3 β (β + 1) (β + 2)
x x2 x3 05. The series : 1 – 3 + 9 – 27 + …
01. The series : + + + … is—
1·2 3·4 5·6 is :
x4 (A) Convergent (A) The series is convergent, if
+ + ……, x > 0 is :
7·8 (B) Divergent x<1
(A) Convergent if x < 1 (C) Oscillatory (B) The series is divergent, if
(B) Divergent if x > 1 (D) Not Convergent x>1

Quantitative Aptitude Test | 92


More PDF download : http://www.gksolve.com

(C) The series is convergent, (C) Not convergent xn


23. The series Σ is divergent,
if x = 1 and β > 2 α (D) Oscillatory x2 +1
(D) The series is divergent, (log 2)2 (log 3)2 if—
17. The series +
if x = 1 and β ≤ 2 α 22 32 (A) x < 1
(log 4)2
(E) All of the above are true. + + … is— (B) x = 1
42
11. The series (A) Convergent (C) x > 1
1 2 3 (B) Divergent (D) None of these
1 + x + 2 x2 + 3 x3 + … x (C) Oscillatory
2 3 4 ∞ n
> 0 is— (D) Not convergent 24. The series Σ
2 n is—
n=1
(A) Convergent, if x ≥ e (n + √
⎯ n)n is—
(B) Divergent, if x ≤ e 18. The series Σ (A) Oscillatory
2 nn + 1
n
(B) Divergent
(C) Divergent, if x ≥ e (A) Convergent
(C) Convergent
(D) Convergent, if x ≤ e (B) Divergent
(D) None of these
12. The series (C) Not convergent
(D) None of these 2 3 4
22 4 22 . 42 25. The series + + + … is—
x2 + x + x 6 + 1p 2p 3p
3·4 3.4.5.6 19. The series
22 . 42 . 62 8 (A) Convergent, if p ≥ 2 and
x + … is— 12 12 3 2 12 . 32 . 52
3.4.5.6.7.8 + + + … is divergent, if p < 2
22 22 . 42 22 . 42 . 62
(A) Convergent, if x 2 > 1 and (A) Convergent (B) Convergent, if p > 2 and
divergent, if x2 ≤ 1 divergent, if p ≤ 2
(B) Divergent
(B) Convergent, if x 2 ≤ 1 and (C) Oscillatory (C) Convergent, if p ≤ 2 and
divergent, if x2 > 1 (D) Non convergent divergent, if p > 2
(C) Convergent, if x 2 < 1 and (D) Convergent, if p < 2 and
20. Which of the following infinite
divergent, if x2 ≥ 1 divergent, if p ≥ 2
series is convergent ?
(D) Convergent, if x 2 ≥ 1 and ∞ 1
(A) Σ 2 26. The sum of the series
divergent, if x2 < 1 n –n
1 ∞ 1
13. The series ∞ 1 Σ is—
(B) Σ 1/2 n=1 n (n + 1) (n + 2)
3 5 7 1 n +n
1+ + + + …… is— ∞
1 1
2 3 4 1 (A) (B)
(C) Σ 2 4
1 n –√ ⎯n
(A) Convergent 1
∞ n2 (C) log 2 – (D) log 2 + 1
(B) Divergent (D) Σ 3 2
2
(C) Not convergent 1 n –n +1
27. For the geometric series—
(D) None of these x2 x3 x4
21. The series x – + – +… 1 – k + k 2 – k3 + ……
2 3 4
14. The series Which one of the following is
is convergent for—
1 1 1 (A) All real values of x false ?
1– + – + … is
⎯2 √
√ ⎯3 √ ⎯4 (B) | x | < 1 only (A) The series is convergent, if
(A) Conditionally convergent (C) | x | ≤ 1 1
k=–
3
(B) Absolutely convergent (D) – 1 < x ≤ 1
(C) Divergent (B) The series is divergent, if
3 4 8 6 k=–3
(D) Oscillatory 22. The series x2 + x + x
5 10
1 1 1 n2 – 1 2n (C) The series is oscillatory, if
15. The series 1 – + – + … 15 8
2 4 8 + x +…+ 2 x +… k=–1
17 n +1
is— is— (D) The series is divergent, if
(A) Convergent (A) Convergent, if x 2 ≥ 1 and k=–2
(B) Semi-convergent divergent, if x2 < 1 1 2 3
(C) Absolutely convergent 28. The series + + +…
(B) Convergent, if x 2 ≤ 1 and 1.2 3.4 5.6
(D) All of the above divergent, if x2 > 1 ∞ is—
1 (C) Convergent, if x 2 < 1 and (A) Divergent
16. The series Σ is—
n⎯√⎯⎯⎯
n2 – 1 divergent, if x2 ≥ 1 (B) Convergent
(A) Convergent (D) Convergent, if x 2 > 1 and (C) Oscillating
(B) Divergent divergent, if x2 ≤ 1 (D) None of these

Quantitative Aptitude Test | 93


More PDF download : http://www.gksolve.com

29. For a positive term series Σa n, (– 1)n 40. The series


35. The series Σ is—
the ratio test states that— (n + 1) p a a (a + 1) a (a + 1) (a + 2)
(A) The series converges, (A) Conditionally convergent if b + b (b + 1) + b (b + 1) (b + 2) +
a 0 <p≤1 … is
if limn → ∞ an + 1 > 1
n
(B) Absolutely convergent if (A) Convergent if b – a < 1 and
(B) The series converges, p>1 divergent, if b – a ≥ 1
a
if lim na + 1 < 1 (C) Oscillatory if p ≤ 0 (B) Convergent if b ≤ 1 + 9 and
n →∞ n divergent if b > 1 + a
(D) All (A), (B) and (C) are
(C) The series diverges, (C) Convergent if b > 1 + a and
correct
a divergent if b ≤ 1 + a
if lim na + 1 = 1
n →∞ n 36. Which of the following series is
non absolutely convergent ? (D) Convergent, if b ≥ 1 + a
(D) None of these and divergent if b < 1 + a
1 1 1
30. If u n ≥ un + 1 ≥ 0 for all n, then (A) 1 – + 2 – 3 + ……
2 2 2 41. Both D Alembert’s ratio test and
for convergence of the series Raabe’s test are particular cases
∞ 1 1 1
Σ un the condition lim un = 0 (B) 1 – 4 + 4 – 4 + …… of—
n=1 n →∞ 2 3 4
(A) Kummar’s test
is— 1
(C) Σ (– 1) n sin n (B) Gauss’s test
(A) Necessary but not sufficient
(B) Sufficient but not necessary (C) Both Kummar’s test and
(– 1)n
(D) Σ Gauss’s test
(C) Necessary and sufficient 3
(n + 1) (D) None of these
(D) None of these 2
42. The alternating series u1 – u2 +
31. The series 1 – 2x + 3x 2 – 4x 3 + 37. Which one of the following is
u3 – u 4 + …… (0 < u n < n) is
…, where 0 < x < 1, is— false ?
convergent, if—
(A) Convergent (A) Absolute Converges ⇒
Convergence (A) un < un + 1 and un → 0
(B) Divergent
(C) Oscillatory (B) A conditionally convergent as n → ∞
(D) Semi-convergent series cannot be absolute (B) un < un + 1 and un → ∞
convergent
1 3 7 as n → ∞
32. The series 1 – + 1 – + 1 – + (C) Convergent ⇒ absolute
2 4 8 (C) un > un + 1 and un → 0
… is— convergence
(D) Convergent series may also as n → ∞
(A) Convergent
be conditionally convergent (D) un > un + 1 or un → ∞
(B) Conditionally convergent
(C) Absolutely convergent 38. If x is positive, then the series : as n → ∞

()
3 2 2
()x
(D) Oscillatory 2 4 3
x+ x + 3 +… 22 x2 33 x3
33. Which one of the following is 3 4 5 43. The series x + + +
is convergent, if— 2 3
false ?
(A) A series is said to converge 44 x4
(A) x = 1 + …… is convergent, if—
if the sequece {S n } of its 4
(B) x < 1
partial sums is converges
(C) x > 1 1 1
(B) The nature of the series is (A) 0 < x < e (B) x > e
not affected by adding or (D) None of these
removing some terms 2 3 3 4
39. The series : (C) e < x < e (D) e < x < e
(C) If lim n → ∞ un = 0, then the
series Σ un is convergent x + x 1 + 1/2 + x1 + 1/2 + 1/3 + x1 + 1/2 +
1/3 + 1/4 + … is— 44. The sum of the alternating har-
(D) If from and after some fixed monic series—
terms un ≥ un ≥ ∞, then Σ vn 1
(A) Convergent, if x ≥ e 1 1 1
is divergent, if Σ u n is 1 – + – + …… is
2 3 4
divergent. 1
(B) Divergent, if x ≥ e (A) Zero
1
34. The series is divergent (B) Infinite
n (log n)p 1
if— (C) Convergent, if x > e (C) log 2
(A) p > 1 (B) p ≥ 1 1 (D) Not defined as the series is
(C) p < 1 (D) p ≤ 1 (D) Divergent, if x ≤ e not convergent

Quantitative Aptitude Test | 94


More PDF download : http://www.gksolve.com

45. The series divergence of a series Σ u n of 1 1 1 1


(C) – + – + … ad inf.
x 1 x3 1.3 x5 1.3.5 x7 positive terms is applied when 2 3 4 5
+ . + . + . + …… fails.
1 2 3 2.4 5 2.4.6 7 (D) x + x2 + x 3 + …where | x | < 1
… is— (A) Comparison ratio test
(A) Convergent, if x 2 > 1 and (B) Cauchy’s root test 50. Let Σ u n be a series of positive
divergent, if x2 ≤ 1 (C) Raabe’s test 1
(D) Logarithmic test terms and let Σ n be a divergent
(B) Convergent, if x 2 ≥ 1 and d
divergent, if x2 < 1 nn 2 series of positive terms such that
48. The series Σ is—
(C) Convergent, if x 2 < 1 and
divergent, if x2 ≥ 1
(n + 1)n 2
(A) Convergent
( u
)
lim dn u n – dn + 1 = k,
n →∞ n+1
(D) Convergent, if x 2 ≤ 1 and (B) Divergent then—
divergent, if x2 > 1 (C) Cannot determined (A) The series is convergent
46. The series Σ n m x n is convergent (D) Not convergent if k > 0
if— 49. Which of the following series is
(A) x > 1 and x = 1 when m < – 1 not convergent ? (B) The series is divergent
(B) x > 1 and x = 1 when m > – 1 1 1 1 if k ≥ 0
(A) + + + … ad inf.
(C) x < 1 and x = 1 when m < – 1 2 3 4 (C) The series is divergent
(D) x < 1 and x = 1 when m > – 1 ⎯2 √
√ ⎯3 √
⎯4 if k < 0
47. Demorgan and Bertrand’s Test 1 1 1
(B) 1 – 1 + 1 … ad inf. (D) Only (A) and (C) are true
for testing the convergence or 2 3 4

Answers with Hints


xn Hence all (A) (B) and (C) are true
01. (D) Here un =
(2n – 1) (2n) ∴ The correct answer is (D)
xn + 1 02. (B)
and un + 1 =
(2n + 1) (2n + 2) (A) The given series is
Σ un = 1 + 2 – 3 + 1 + 2 – 3 + ……
un xn (2n + 1) (2n + 2)
⇒ = × ⇒ S 3n = (1 + 2 – 3) + (1 + 2 – 3) +
un + 1 (2n – 1) (2n) xn + 1
(1 + 2 – 3) + … + (1 + 2 – 3) = 0
(2n + 1) (2n + 2) 1 ∴ lim S3n = 0
= . n →∞
(2n – 1) (2n) x
⇒ S 3n + 1 = (1 + 2 – 3) + (1 + 2 – 3) + … +
u (2n + 1) (2n + 2) 1
∴ lim u n = lim . (1 + 2 – 3) + 1 = 1
n →∞ n + 1 n →∞ (2n – 1) (2n) x
∴ lim S3n + 1 = 1
n →∞
1
= x ⇒ S 3n + 2 = (1 + 2 – 3) + (1 + 2 – 3) +
… + (1 + 2 – 3) + 1 + 2 = 3
1
Hence, if x > 1 i.e. x < 1 the series is convergent and ∴ lim S3n + 2 = 3
n →∞
1
if x < 1 i.e. x > 1 the series is divergent. The limit does not exist because the sum of infinite
terms are 0, 1 and 3. Hence the given series is
If x = 1, D’ Alembert’s test fails. In this case by oscillatory.
comparison test ∴ In (A) the series is not convergent.
(B) The given series is
1 1
un = take vn = 2 1 1 1 1
(2n – 1) (2n) n Σ un = – + –…+
⎯2–1 √
√ ⎯3–1 √ ⎯4–1 ⎯√⎯⎯⎯
n+1–1
u n2 1
∴ lim = v n = lim = Here we see that
n →∞ n n → ∞ (2n – 1) (2n) 4
(i) The series is alternating.
which is finite and non-zero auxiliary series Σv n = 1 1 1
(ii) > > > ……
1
Σ 2 is a p-series, where p = 2 > 1. ⎯2–1 √
√ ⎯3–1 √ ⎯4–1
n
1
(iii) lim un = lim =0
Therefore, Σ vn is convergent. Hence Σ un is also n →∞ n →∞ √⎯⎯⎯⎯
n+1–1
convergent.
Hence all the three conditions of Leibnitz’s test are
∴ When x = 1, the series is convergent. satisfied. Hence, the series is convergent.

Quantitative Aptitude Test | 95


(C) The given series is
= lim (1 + 1n)
( ) ( )+
1 1 1 2 n →∞
Σ un = 1+ + 1+
1 2 = 1, which is finite and non-zero.
…… + (1 + n)
1 n
By comparison test Σun and Σv n behave alike.
1
Now Σvn = Σ n is a p-series, where p = 1
= (1 + n)
1 n
Here un
Therefore, Σv n is divergent. Hence, Σ un is also
= lim (1 + n) = e ≠ 0
1 n
and lim un divergent.
n →∞ n →∞ (B) The given series is
Hence the series is divergent
1 √ ⎯2+ √
⎯ 3 + …… + √
⎯ n + ……
∴ In (C) the series is not convergent +
(D) The given series is : 2 5 10 n2 + 1
1 22 33 ⎯√ n
Σun = 1 + 2 + 2 + 4 + Here, un =
2 3 4 (n2 + 1)
nn
…+
(n + 1)n + 1 ⎯n= 1

Take vn =
nn n2 n3/2
Here un = ⎛ √ ⎞
⎜ ⎯2 n ⎟
(n + 1)n + 1
nn 1 u ⎝ n + 1⎠
Take vn =
nn + 1
=n lim = v n = limn → ∞
n →∞ n 1
u nn n n3/2
lim v n = lim n + 1 ×
n →∞ n n → ∞ (n + 1) 1 n2
= lim
nn + 1 n →∞ n2 + 1
= lim n+1
n → ∞ (n + 1) 1
= lim
= lim
1 n →∞
1+ (n1 )
2
n →∞
( )
1+n
1 n+1
= 1 which is finite and non zero.
1 1 By comparison test Σ un and Σ vn behave alike.
= lim ×
n →∞
( ) ( )
1+n
1 n 1+n
1 1
Now Σ vn = Σ 3/2 is a p–series, where p =
n
3
2
1 Therefore, Σ vn is convergent. Hence Σ un is also
= e ×1
convergent.
1
= e (C) The given series is Σ [√⎯⎯⎯⎯ ⎯⎯n ]
n +1–√3 3

which is finite and non-zero


By comparison test Σun and Σv n behave alike. Here un = ⎯√⎯⎯⎯
n3 + 1 – √
⎯⎯n3
1
Now Σ vn = Σ n is a p–series, where p = 1. ⎯⎯⎯⎯
√ n3 + 1 – √
⎯⎯n3 × ⎯√⎯⎯⎯
n3 + 1 + ⎯⎯
√n3
=
1 ⎯⎯⎯⎯
√ n3 + 1 + ⎯⎯
√n3
Therefore, Σvn is divergent. Hence Σun is also
divergent 1
=
∴ In (D) the series is not convergent. ⎯⎯⎯⎯
√ n ⎯⎯

3 + 1 + n3
The correct answer is (B). 1
3. (D) Take vn = 3/2
n
(A) The given series is
2 3 4 n+1 ⎛ 1 ⎞
+ + + …… + 2 … ⎜ n3 + 1 + n3 ⎟
1 4 9 n u ⎝√⎯⎯⎯⎯ ⎯⎯
√ ⎠
∴ lim v n = lim
n+1 n 1 n →∞ n n →∞ 1
∴ un = 2 , Take vn = 2 = n
n n n3/2
⎛n + 1⎞ = lim
1
⎜ n2 ⎟
⎯⎯

n →∞
u ⎝ ⎠ 1
∴ lim = v n = lim 1+ +⎯
√1
n →∞ n n →∞ 1 n3
n 1 1
= =
n+1 ⎯ 1 + ⎯√ 1
√ 2
= lim n
n →∞
which is finite and non-zero.
Quantitative Aptitude Test | 96
By comparison test Σun and Σv n behave alike. n
06. (B) Here un =
1 3 1 + 2– n
Now Σvn = Σ 3/2 is a p-series, where p = > 1 n
n 2 ∴ lim un = lim
Therefore, Σv n is convergent. n →∞ n →∞ 1 + 2– n
Hence Σun is also convergent. n
= lim n= ∞≠0
(D) Here given series is :
1
n →∞
1+ ()1
2
un = sin n Hence the series is divergent.
∴ The correct answer is (B).
1 1 1 3
= n– ()
3 n
+ ……
7. (A) Here we see that—
(i) The series is alternating series.
1 1
= n– 3+… 1 1 1
6n (ii) 1 > > > ……
2 3 4
1
Take vn = n 1
(iii) lim un = lim n = 0
n →∞ n →∞
u
( 1 1
lim v n = lim n – 3 + … ×
n →∞ n n →∞ 6n ) n
1 All the three conditions of Leibnitz Test are satisfied.
Hence the given series is convergent.
n →∞
(
= lim 1 – 2 + …
6n
1
) The correct answer is (A).
08. (D)
= 1, which is finite and non-zero.
1 1 1
1 (A) The series 1 – + 2 – 3 + … is a G.P.
Now, auxiliary series Σvn = Σ n is a p-series 2 2 2

where p = 1
∴ Sn =
[ (
1 1– –
2 )]
1 n

∴ Σvn is divergent. Hence Σun is also divergent.


The correct answer of this question is (D). (
1– –
1
2 )
Note : Comparison test is found useful, when in the
given series Σv n , terms do not contain power of x or =
2
3 [ (
1– –
2 )]
1 n

involving x or factorials.
04. (C) Comparing the series
∴ lim Sn = lim
n →∞
[
n →∞ 3
2
1– – ( )]
1 n
2
1 1 1 2
(A) + 1/3 + 1/3 + …… with p-series test, we get = (1 – 0)
1 2 3 3
1 2
p = <1 = (which is finite)
3 3
∴ The series is divergent ∴ The series is convergent.
(B) The series 1 + 2 + 3 + …… is an A.P.
1 1 1
(B) In the series 1 + + + + …, 1
2 3 4 S n = n (n + 1)
2
p = 1 1
lim Sn = lim n (n + 1) = ∞
∴ The series is divergent. n →∞ n →∞ 2
1 1 1 ∴ The series is divergent.
(C) In the series 1 + 2 + 2 + 2 + …, p = 2 > 1
2 3 4 (C) The series 1 – 2 + 3 – 4 + 5 – 6 + ……is an alter-
∴ The series is convergent. nating series.
1 1 1 The series can be written as
(D) In the series k + k + k + … p = k > 1
1 2 3 S n = (1 – 2) + (3 – 4) + (5 – 6) + … +
∴ The series is convergent. {(n – 1) – n}
The correct answer is (C). = (– 1) + (– 1) + (– 1) + … + (– 1)
05. (A) The given series 1
= – n
1 1 1 2
1– + – + …… is a G.P.
3 9 27
1 n →∞ n →∞
( )1
lim Sn = lim – n = – ∞
2
where a = 1, r = which lies between – 1 and 1.
3 The series can also be written as
∴ The given series is convergent. S n = 1 – (2 – 3) – (4 – 5) – (6 – 7)
The correct answer is (A). …… – {(n – 1) – n}

Quantitative Aptitude Test | 97


= 1 + 1 + 1 + 1 + …… + 1 1
The series is divergent, if p < 1 i.e. p < 2.
1 2
= (n + 1)
2 1
The test fails, if
p = 1 i.e. p = 2.
1 2
lim Sn = lim (n + 1) = ∞
n →∞ n →∞ 2

Since the limit does not exist because the sum of


u
Now, n log u n = 2
n+1
1 3
– + (
2 8n 24 n2
7
+… )
infinite terms of the series is not unique.
Therefore, the series is oscillatory.
{ u
⇒ n log u n – 1
n+1
}
(D) Since (A), (B), (C) are all correct. Hence only
(D) is incorrect statement. i.e., not true.
= {(
1– +
3
4n 12n2
7
+… –1 ) }
09. (B) Neglecting the first term 3 7
= – + +…
4n 12n2
⎛ 1.3.5……(2n – 1)⎞ p
un = ⎜ ⎟
⎝ 2.4.6……2n ⎠ { u
⇒ n log u n – 1 log n
n+1
}
⎛ 1.3.5…… (2n – 1) (2n + 1)⎞ p 3 log n 7 log n
and un + 1 = ⎜ 2.4.6 …… (2n) (2n + 2) ⎟ = – × n + × 2 + ……
⎝ ⎠ 4 12 n

un ⎛ 2n + 2 ⎞ p ( 1 + n)
1 p
⇒ lim
n →∞
[{
n log u
un
n+1
– 1 log n } ]
∴ = ⎜ ⎟ = ⎛ 3 log n 7 log n ⎞
⎝ 2n + 1 ⎠
un + 1
(1 + 2n1 ) p
= lim ⎜ – 4 × n + 12 × n2 ⎟
n →∞ ⎝ ⎠
( 1 + n)
1 p
= 0<1
un Hence by higher logarithmic test the given series is
⇒ lim u = lim =1
( 2n) 1 p
n →∞ n+1 n →∞ divergent, if p = 2.
1 +
Hence the given series is convergent when p > 2 and
∴ Ratio test fails divergent when p ≤ 2.
⎧⎪ (1 + 1n) ⎫⎪p The correct answer is (B).
log ⎨ ⎬
un 10. (E) Neglecting the first term
∴ log u =
n+1
⎪⎩(1 + 2n1 ) ⎪⎭p
un =
α (α + 1)2 (α + 2)2 … (α + n – 1)2 n
1.2.3… n β (β + 1)
x
(β + 2) … (β + n – 1)
= p log (1 + 1n) – p log (1 + 2n1 ) α (α + 1)2 (α + 2)2 …
(α + n – 1)2 (α + n)2
= p [(n – ……)
1 1 1 un + 1 = xn+1
+ 2 3 1.2.3 … n (n + 1) β (β + 1) (β + 2)
2n 3n
…… (β + n – 1) (β + n)
–( – +…… )]
1 1 1
+ 2 3
2n 8n 24n
un (n + 1) (β + n) 1
= p [ (n – ) – (
2n 8n )
1 1 1 1 ∴u =
– 2 2 n+1 (a + n)2 x
2n
u ⎡⎢ n2 + (1 + β) n + β 1 ⎤⎥ 1
+( + … )]
1 1 ∴ lim u n = lim
3n 24n
– 3 3 n →∞ n + 1 n →∞
⎢⎣ n2 + 2 an + a2 x ⎥⎦ = x
∴ From ratio test
= p[ – + …]
1 3 7
+ 2 3
2n 8n 24n 1
The series is convergent, if x > 1 i.e. x < 1.
= p( – + + …)
un 1 3 7
⇒ n log u
2 8n 24n 2 1
n+1
The series is divergent, if x < 1 i.e. x > 1.
un
∴ lim n log u 1
n →∞ n+1 The test fails if x = 1 i.e. x = 1
= lim p
n →∞
(12 – 8n3 + 24n7 + …) 2

un
un + 1 =
n2 + n(1 + β) + β
n2 + 2αn + α2
p
) = n ⎡⎢⎢⎣n n+ +n 2nα
(1 + β) + β ⎤⎥
= 2
2
From logarithmic test.
⇒ n (uu n+1
n
–1 2 +α 2 – 1⎥

1 n2 (1 + β – 2α) + n(β – α2)
The series is convergent, if p > 1 i.e. p > 2. =
2 n2 + 2nα + α2

Quantitative Aptitude Test | 98


⇒ lim n
n →∞
(uu n
n+1
–1 ) e
The ratio test fails if x = 1 i.e. x = e

⎡⎢ n2 (1 + β – 2α) + n (β – α2)⎤⎥
( )
un 1 n+11

un + 1 = 1 + n + 1
˙·˙
= lim
n →∞ ⎣
⎢ n2 + 2nα + α2 ⎥⎦ e

= 1 + β – 2α
From Raabe’s test the series is convergent if (1 + β –

u
log u n = (n + 1) log 1 +
n+1
1
n+1 (
) – log e
= (n + 1) [ ……] – 1
2a) > 1 i.e. β > 2α. 1 1 1
– + 2 3
The series is divergent, if 1 + β – 2α < 1, i.e. β < 2α. n + 1 2 (n + 1) 3 (n + 1)

= [1 – ……] – 1
The Raabe’s test fails if 1 + β – 2α = 1, i.e. β = 2α. 1 1
+ 2
(2n + 1) 3(n + 1)
– nα2 – α2
( u
n+1
)
∴ n u n –1 –1 = 2
n + 2αn + α2 = –
1
+
1
2 (n + 1) 3 (n + 1)2
– ……
∴ lim
n →∞
{[ ( ) ] } u
n u n – 1 – 1 log n
n+1 ⇒ n log u
un

⎡ – α (1 + 1n) ⎤
n+1

= lim

2
log n ⎥
⎢{1 + 2α + α } × n ⎥ 2
= n[2 (n– 1+ 1) + 3 (n1+ 1) ……] 2
n →∞
⎣ ⎦ = – (1 + n) + (1 + n) + …
1 1 1 –1 1 2
n n 2
2 3n
= a2 × 0
= – [1 – n + ……]
1 1 1
=0<1 2 n 2

∴ From De Morgan’s test, the given series is


3n [ n n
1 – + ……] + ……
1 2 3
divergent if β = 2α. + 2
If x < 1, the given series is convergent (A). 1 5
If x > 1, the given series is divergent (B). = – + + ……
2 6n
If x = 1 and β > 2α, the series is convergent (C). un
∴ lim n log u
If x = 1 and β ≤ 2α, the series is divergent (D). n →∞ n+1
Hence all A, B, C and D are correct.
Thus the correct answer is (E).
=
n →∞
lim (– 21 + 6n5 + …)
11. (C) If Σ u n is the given series and neglecting first 1
= – <1
term 2
n ∴ From logarithmic test the given series is divergent,
un = xn if x = e.
(n + 1)n
Hence the series is convergent, if x < e and divergent,
n+1 if x ≥ e.
and un + 1 = xn + 1
(n + 2)n + 1 ∴ The correct answer is (C).
un n xn(n + 2)n + 1 Note : If logarithmic test fails, then it is useful to
un + 1 = (n + 1)n × n + 1 xn + 1 apply higher logarithmic test.
12. (B) If Σ u n be the given series, then neglecting the
⎛n + 2⎞ n + 1 1 first term.
= ⎜n + 1⎟
⎝ ⎠ x 22 . 42 . 62 …… (2n)2
∴ un = x2n + 2
⎡ (n + 1) + 1⎤⎥ n + 1 1 3.4.5.6. …… (2n + 1) (2n + 2)
= ⎢⎢
⎣ (n + 1) ⎥⎦ x 22 . 42 . 62 ……(2n) 2
(2n + 2)2
(1 + n +1 1) 1x n+1
and un + 1 = x2n + 4
= 3.4.5.6…… (2n + 1)
(2n + 2) (2n + 3) (2n + 4)
= lim [(1 +
n + 1) x] = x
un 1 1 e n+1
lim un (2n + 3) (2n + 4) 1
n → ∞ un + 1 n →∞ ∴ un + 1 = . 2
(2n + 2)2 x
From the ratio test the series is convergent.
e u (2n + 3) (2n + 4) 1
If x > 1, i.e. x < e. ⇒ lim u n = lim . 2
n →∞ n + 1 n →∞ (2n + 2)2 x
e 1
The series is divergent, if x < 1 i.e. x > e. =
x2

Quantitative Aptitude Test | 99


1 1
The series is convergent, if > 1 i.e. x2 < 1. This is a geometric series with r = < 1.
x2 2
1 Thus, the series Σ | un | is convergent, therefore, the
The series is divergent, if < 1 i.e. x2 > 1.
x2 given series is absolutely convergent.
1 Hence the correct answer is (C).
The test fails, if = 1 i.e. x2 = 1
x2 16. (A) ∑ un is the given series
⎡⎢ (2n + 3) (2n + 4) ⎤⎥
∴ n ( un
)
un + 1 – 1 = n ⎢⎣ (2n + 2)2
– 1⎥
⎦ ∴ un =
n⎯
√⎯⎯⎯
1
n2 – 1
n (6n + 8) 1
= Let f (x) =
(4n2 + 8n + 4)
x⎯
√⎯⎯⎯
x2–1

n →∞
u
(
lim n u n – 1 = lim
n+1
)
n →∞
n (6n + 8)
(4n 2 + 8n + 4)
⇒ f (n) =
1
3 n⎯
√⎯⎯⎯
n2 – 1
= >1
2 = un
∴ By Raabe’s test the given series is convergent, if Also f (x) is a continuous, positive and decreasing
x2 = 1. function of x for x > 0.
Hence the series is convergent when x2 ≤ 1 and ∞ ∞
divergent when x2 > 1.
Hence the correct answer of this question is (B).
Now ∫ 1
f (x) dx = ∫ 1 x⎯
1
√⎯⎯⎯
x2 – 1
dx

(2n – 1) ∞
13. (A) Here un = = [sec– 1 x]1
n

and un + 1 =
(2n + 1) = (π2 – 0)
n+1 π
=
2
un (2n – 1) n + 1 ∞
∴ = ×
un + 1 n (2n + 1) ∴ ∫ 1
f (x) dx is convergent and by Cauchy’s integral
(2n – 1) (n + 1) ∞


=
(2n + 1) test f (x) dx and Σ un behave alike.
1
un (2n – 1) (n + 1)
⇒ lim = lim Therefore Σ un is also convergent.
n → ∞ un + 1 n →∞ (2n + 1)
= ∞>1 Hence the correct answer is (A).
∴ The series is convergent. 17. (A) The given series is Σ un then
The correct answer is (A). (log n)2
14. (A) The given series un =
n2
1 1 1
1– + – + …… also un + 1 < un
⎯2 √
√ ⎯3 √ ⎯4 ∴ By Cauchy’s condensation test
is an alternating series. By Leibnitz’s test the given
series is convergent. (log an )2
Σ an ua n = Σ an
(an)2
1 1 1
Now Σ | un | = 1 + + + + ……
⎯2 √
√ ⎯3 √ ⎯4 n2
= (log a)2 Σ
(an)
1
which is a p-series with p = < 1. Hence Σ | u n | is
2 Let Σvn = Σan ua n
divergent. Hence the given series is conditionally n2 (log a)2
convergent series. = Σ
an
Hence the correct answer is (A).
n2 (log a)2
15. (C) The given series is : vn =
an
1 1 1
1 – + – + …… (n + 1)2 (log a)2
2 4 8 vn + 1 =
By Leibnitz’s test this series is convergent. an + 1
1 1 1 vn n2 (log a)2 an + 1
Σ | un | = 1 + + + + …… ⇒ = ×
2 4 8 vn + 1 a n (n + 1)2 (log a)2

Quantitative Aptitude Test | 100


a 1 1
= 20. (A) (A) Hereun = take vn = 2
[1 + ( )] 1 2 n2 – n n
n u 1 n2
v a ∴ lim v n = lim 2 × = 1 which is finite
n →∞ n n →∞ n – n 1
∴ lim v n = lim
[1 + (1n)] 2
n →∞ n + 1 n →∞ and non-zero.
1
∴ By p-series test Σ vn = Σ 2 is convergent because
Hence from Ratio test Σ v n is convergent i.e. Σan ua n n
is convergent and if Σan ua n is convergent, then from p=2>1
Cauchy’s condensation test Σun is convergent. This by comparison test Σ un is also convergent.
Hence the correct answer is (A). 1 1
(B) Here un = 1/2 , take v n = n
18. (A) If Σ un is the given series, then n +n
u 1 n
(n + √
⎯ n)n ∴ lim v n = lim 1/2 × =1
un = n →∞ n n →∞ n + n 1
2 (n)n + 1
n
which is finite and non-zero.
⎡⎢ ⎧n + √⎯ n⎫ n 1⎤⎥1/n 1
⇒ (un)1/n = ⎢⎣ ⎨⎩ 2n ⎬⎭ . n⎥⎦ By p-series test Σvn = Σ n is divergent because p = 1.
Therefore, by comparison test ∑un is also divergent.
⎛n + √⎯ n⎞⎟ 1
= ⎜ 1 1
⎝ 2n ⎠ n1/n (C) Here un = , take vn = n
n –√⎯n
∴ lim (un)1/n un n
n →∞ 1
lim = lim × =1
⎡⎢ 1 ⎧⎛ 1 ⎞ 1 ⎫ ⎤⎥ n → ∞ vn n →∞ n – √ ⎯n 1
= lim
n →∞ ⎢⎣ 2 ⎨⎩⎜⎝ 1 + ⎯√ n⎟⎠ n1/n⎬⎭ ⎥⎦ 1
By p-series test ∑v n = ∑ n is divergent.
1
= (1 + 0). 1 Therefore, by comparison test Σun is also divergent.
2
n2
1 (D) Here un = 3
= <1 n – n2 + 1
2
n2 1
Hence from Cauchy’s root test the given series is take vn = 3 = n
convergent. n
∴ The correct answer is (A). un n2 n
lim v = lim 3
n – n 2 + 1 ×1
19. (B) If Σ un is the given series, then n →∞ n n →∞
1
12 .3 2 .5 2 …… (2n – 1) 2 By p-series test Σun = Σ n is divergent because p = 1.
un =
22 .4 2 .6 2 …… (2n) 2 Therefore by comparison test Σun is also divergent.
12 .3 2 .5 2 …… (2n – 1) 2 (2n + 1)2 (– 1)n – 1 xn
and un + 1 = 21. (D) Here un =
22 .4 2 .6 2 ……(2n) 2 (2n + 2)2 n
(2n + 2)2

un
un + 1 = (2n + 1)2 ⇒ lim
un
n → ∞ un + 1
= –( ) n+1 1
n x
(2n + 2)2
u
lim u n = lim
n →∞ n + 1 n → ∞ (2n + 1)
2 =1 ∴ | | | |
un
un + 1 =
n+1
nx
∴ Ratio test fails
˙·˙
un
=
(2n + 2)2 ∴ lim
n →∞
|uu | = | 1x |
n
n+1
un + 1 (2n + 1)2 ∴ By ratio test, we find that the series Σ | un | is
4n + 3 1
= 1+ convergent, if > 1, i.e. | x | < 1.
(2n + 1)2 |x|
⎛ 4n + 3 ⎞ 1 1
= 1+⎜
⎝ 4n ⎠ ( )
2 ⎟ 1+
1 –2
2n
If x = 1, then Σun = 1 – + ……
2 3
= log 2
⎛ 4n + 3 ⎞
⎟ (1 – n + 4n …)
1 3 So, Σun is convergent.
= 1+⎜
⎝ 4n ⎠ 2 2
If x = – 1 we have
1 1
= 1 + n – 2 + ……
4n (
Σ un = – 1 + + + ……
1 1
2 3 )
Comparing by Gauss’s test λ = 1. Hence the given 1
= – Σn
series is divergent.
The correct answer is (B). which is divergent, because p = 1.

Quantitative Aptitude Test | 101


If x > 1; un does not tend to zero as n → ∞, So Σ un is 1
not convergent, when x > 1. =
np – 1
∴ Σun is convergent, if – 1 < x ≤ 1. un n + 1 np – 1
22. (C) Leaving the first term, ∴ = ×
vn np 1
n2 – 1 2n
un = 2 x n+1 1
n +1 = n =1+n
(n + 1)2 – 1 2n + 2
un + 1 =
(n + 1)2 + 1
x u
lim v n = lim
n →∞n n →∞
(1 + 1n) = 1 which is finite and
(n2 + 2n) 2n + 2 non-zero.
= 2 x
n + 2n + 2 1
un n2 – 1 2n n2 + 2n + 2 1 By p-series test, Σ vn = Σ p – 1 is convergent.
n
∴ un + 1 = n2 + 1 x × (n2 + 2n) . x2n + 2
If p – 1 > 1, i.e. p > 2 and divergent if
(n2 – 1) (n2 + 2n + 2) 1
= p – 1 ≤ 1 i.e. p ≤ 2
(n2 + 1) (n2 + 2n) x2
Hence by comparison test Σun is convergent if p > 2
u (n2 – 1) (n2 + 2n + 2) 1
lim u n = lim and divergent if p ≤ 2.
n →∞ n + 1 n →∞ (n2 + 1) (n2 + 2n) x2
1
1 26. (B) Here Tn =
= 2 n (n + 1) (n + 2)
x
1 1 1
1 = – +
By ratio test Σun is convergent, if 2 > 1, i.e. x2 < 1 2n (n + 1) 2 (n + 2)
x
1 Put n = 1, 2, 3
and the series Σun is divergent, if 2 < 1 i.e. x2 > 1
x S = T1 + T2 + …… +
1
The ratio test fails, if 2 = 1 i.e. x2 = 1
x = (11 + 12 + 13 + ……)
1
2
But in this case
– ( + + …… ) + ( + ……)
1 1 1 1 1
n2 – 1 2 3 2 3 4
lim un = lim 2 = 1≠ 0
n →∞ n →∞ n + 1 1 1 1
= – =
Hence the series is divergent when x2 = 1 2 4 4
Hence the series is convergent if x2 < 1 and divergent 1
if x2 ≥ 1. 27. (C) Note that sum of this geometric series is , if
1+k
u 1 the value of k is numerically less than ‘1’. Clearly if
23. (C) Here lim u n = lim x
n →∞ n + 1 n →∞ k = – 1. Series cannot be oscillatory, for k = – 1
1 series is
The series is divergent, if x < 1 ⇒ x > 1. 1 + 1 + 1 + 1 + 1 + ……
n whose sum is ∞. (Hence option C is false).
24. (C) Here ˙·˙ un =
2n n
un n 2n + 1 28. (A) Here un =
∴ lim u = lim n × (2n – 1) 2n
n →∞ n+1 n →∞ 2 n+1
1
1 =
= lim 2. 2 (2n – 1)
n →∞
( )
1+n
1
and
1
vn = n
= 2 > 1.
u 1
∴ By ratio test Σun is convergent. Here lim n =
n → ∞ vn 4
25. (B) The given series is
which is non-zero and finite.
2 3 4 n+1
Σ un = p + p + p + …… + p + …… 1
1 2 3 n By p-series test Σ n is divergent because p = 1.
(n + 1)
Here un = Hence by comparison test Σun is also divergent.
np
n 29. (B)
and vn = p
n 30. (A)

Quantitative Aptitude Test | 102


31. (A) The given series is : 1 – 2x + 3x2 – 4x3 + …… 1
34. (D) Let un =
S ∞ = 1 – 2x + 3x 2 – 4x3 + …… n (log n)p
– x S∞ = – x + 2x2 – 3x3 + …… Here u2 ≥ u3 ≥ u4 ≥ ……… ≥ un ≥ ………
Therefore, the sequence < u2, u3 , u4 …… un …… >
(1 + x) S∞ = 1 – x + x 2 – x3 + ……
Satisfied the conditions of Cauchy’s condensation
1 test hence the series
(1 + x) S∞ =
1+x ∞ ∞
Σ un and Σ an ua n , (a > 1)
1 2 2
⇒ S∞ = = finite because 0 < x < 1
(1 + x)2 will converge or diverge together.
Hence the given series is convergent. n ∞ 1
Now Σ an ua n = Σ an n
1 3 7 2 2 a (log an )p
32. (D) Given series is 1 – + 1 – + 1 – + ……
2 4 8 ∞ 1
= Σ p
2 (n log a)
( ) ( ) ( )
S 2n = 1 –
1
2
+ 1–
3
4
+ 1–
7
8
=
1 ∞ 1
Σ
( ) (log a) 2 np
2n – 1 p
+ …… + 1 – n
2 1
=
1 1 1 1 (log a)p
= + + + …… + n
2 4 8 2
1
[ ( )]
1–
1 n (1
2p 3p
1 1
+ + …… + p + …
n )
2 2 1 1 1 1
= Here p + p + p + …… + p + …… is a p-series.
1 2 3 4 n
1–
2 This series is convergent, if p > 1 and divergent if
[ ( )]
= 1–
1 n
2
p ≤ 1.

∴ The series Σ a n u a n is convergent if p > 1 and
n →∞
[ ( )]
lim S2n = lim 1 –
n →∞
1 n
2
=1 2
divergent if p ≤ 1.

( ) ( )
S 2n + 1 = 1 –
1
2
+ 1–
3
4
+ ∴ The given series is convergent, if p > 1 and
divergent if p ≤ 1.

( ) 1–
7
8( )
+ …… + 1 – n
2n – 1
2
+1 35. (D) The series is Σ
(– 1)n
(n + 1)p

( = )
1 1 1
+ + +……+ n + 1
2 4 8 2
1 which is an alternating series.
Case I : p ≤ 0

[ ( )]
=
1
2
1–
1 n
2
+1
Let p = –q
The given series becomes
1 Σ (– 1) n (n + 1)q = – 2 q + 3 q – 4q + 5 q – ……
1–
2 This is an oscillatory series.
[ ( )]
= 1–
1 n
2
+1 Case II : 0 < p ≤ 1. The series is
1 1 1 1
= 2–() 1 n
2
Σun = – p + p – p + p ……
2 3 4 5
but Leibnitz’s test, this series is convergent.
lim S2n + 1 = lim 2 –
n →∞ n →∞
() 1 n
2
=2 also
1 1 1
| Σun | = p + p + p + ……
2 3 4
Since the sum of the infinite series oscillate between This is a p-series and p ≤ 1, so it is divergent.
1 and 2. Hence the given series is oscillatory.
∴ The given series is conditionally convergent.
33. (C) The necessary condition for convergence of a
series Σun is lim un = 0 but if lim un = 0, then we Case III : p > 1. The series is
n →∞ n →∞ 1 1 1 1
cannot say that the series is always convergent. For Σun = – p + p – p + p ……
2 3 4 5
example
By Leibnitz’s test this series is convergent.
1 1 1 1
1 + + + + …… + n + … is divergent but 1 1 1
2 3 4 also | Σun | = p + p + p + ……
2 3 4
1 This is a p-series and p > 1, so it is convergent.
lim un = lim n = 0.
n →∞ n →∞ ∴ The given series is absolutely convergent.
Hence, this condition is necessary but not sufficient. ∴ All (A) (B) and (C) are correct.

Quantitative Aptitude Test | 103


36. (C) In case of (A), Σ | u n | is geometric series with
1
common ratio hence convergent. Similarly (B) and
⇒ lim
n →∞
(n log uu )
n
n+1
2 n 1
(D) are also absolutely convergent. In case of (C) = lim log x
n →∞ n+1
1
Σ | un | = Σ sin n. 1
= log x
1
Comparing with auxiliary series Σ n , we see that
∴ By Logarithmic test the series is convergent if
Σ | u n | is divergent, hence Σun is non absolutely 1 1 1
log x > 1 ⇒ x > e ⇒ x < e .
convergent.
37. (C) Convergence ⇒ absolute convergence is false. 1
The series is divergent if log x < 1,
⎛n + 1⎞ n n
38. (B) Here un = ⎜ ⎟ x
⎝n + 2⎠ 1 1
⇒ x < e, ⇒ x > e .
⎧⎛ n + 1 ⎞ n n⎫ 1/n
∴ lim (un )1/n = lim ⎨⎜ ⎟ x⎬ 1 1
n →∞ n → ∞ ⎩⎝ n + 2 ⎠ ⎭ The test fails if, log x = 1, ⇒ x = e,
⎛n + 1⎞ 1
= lim ⎜n + 2⎟ x = x ⇒ x = e . But in this case
n →∞ ⎝ ⎠
∴ By Cauchy’s root test this series is convergent if un n
x < 1. ˙·˙ n log u –1 = log e – 1
n+1 n+1
The series is divergent if x > 1. n
The Cauchy’s root test fails if x = 1, but in this case = –1
n+1
⎛n + 1⎞ n –1
un = ⎜n + 2⎟ =
⎝ ⎠ (n + 1)

= (n +n 1) (n +n 2)
n n
∴ lim
n →∞
(n log uu – 1) log
n
n+1
n

= (1 + n)
1 1n
(– 1)
= lim log n
(1 + 2n)
n
n →∞ (n + 1)
(– 1) log n
= lim
( 1 + n)
1 n

∴ lim un = lim
n →∞
( )1+n
1 n

(1 + 2n)
n
n →∞ n →∞
= (– 1) × 0 = 0 < 1
By Higher Logarithmic test the series is divergent.
e 1
= = ≠0 1
e2 e Hence the series is convergent, if x < e and diver-
Hence the series is divergent.
1
Thus, the series is convergent if x < 1 and divergent gent. If x ≥ e .
if x ≥ 1.
40. (C) In the given series
un x1 + 1/2 + 1/3 + … + 1/n
39. (B) Here = un b+n
un + 1 x 1/2 + 1/3 + … + 1/n + 1/(n + 1)
1 +
un + 1 = a + n
˙·˙
1
= 1/(n + 1
x ) u b+n
un ⇒ lim u n = lim
1 n →∞ n + 1 n →∞ a + n
∴ lim u = lim 1/(n + 1)
n →∞ n+1 n →∞ x = 1
1
= 0=1 ∴ The ratio test fails.
x
∴ D' Alembert’s ratio test fails.

u
n log u n = n log 1/(n + 1)
1
∴ lim n u n – 1
n →∞
u
(
n+1
)= lim
n →∞
(b – a) n
(a + n)
n+1 x = b–a
= n log x () 1 1/(n + 1) ∴ By Raabe’s test, the series is convergent,
if b – a > 1 i.e. b > 1 + a.
n 1
= log x
n+1 The series is divergent, if b – a < 1, i.e. b < 1 + a.

Quantitative Aptitude Test | 104


The Raabe’s test fails if b – a = 1, i.e. b = 1 + a.
But in this case,
∴ lim n
n →∞ (uu n
n+1
)=
–1 lim
n →∞
n
2n + 1
1
˙·˙ ( ) u
n u n –1 =
n+1
n
a+n 2
<1
=

Hence by Raabe’s test the series is divergent. Hence


∴ lim
n →∞
{( ) } u
n u n – 1 – 1 . log n
n+1
the series is convergent, if x 2 < 1 and divergent, if
x2 ≥ 1. Hence the correct answer is (C).
⎛ –a ⎞
= lim ⎜ ⎟ . log n 46. (C) The given series is Σun = Σnm xn
n → ∞ ⎝ a + n⎠
lim (u )1/n = lim (nm xn)1/n
= lim ⎛⎜ a
– a ⎞ log n n →∞ n n →∞
n →∞ ⎜
⎟× n
+ 1 ⎟⎠ = nlim
⎝ n →∞
nm/n x
= 0<1 = x
∴ By Logarithmic test the given series is divergent ∴ By Cauchy’s root test Σun is convergent if x < 1.
if b = 1 + a.
The Cauchy’s test fails if x = 1.
Hence, the series is convergent, if b > (1 + a) and
When x = 1, then
divergent, if b ≤ (1 + a).
1 1 1
41. (A) 42. (C) 43. (A) Σun = Σnm = Σ – m = – m + – m + ……
n 1 2
1 1 1
44. (C) Since log (1 + x) = x – x + x 3 – x 4 + …… ∴ This series is P-series and it is convergent if – m >
2 3 4
put x = 1 in both sides, we get 1, i.e., m < – 1.
1 1 1 ∴ The series is convergent if x = 1 and also
log 2 = 1 – + – + …… convergent if x = 1, m < – 1.
2 3 4
45. (C) Neglecting first term, 47. (C)
1.3.5. …… (2n – 1) 2n + 1 nn 2
un = x 48. (A) The given series is Σun = Σ
2.4.6. …… 2n (n + 1)n 2
1.3.5. …… (2n – 1) (2n + 1) 2n + 3
un + 1 = x By Cauchy’s root test.
2.4.6. …… 2n (2n + 2)
lim (u )1/n = lim ⎛ nn 2 ⎞ 1/n
un 1.3.5. …… (2n – 1) 2n + 1 ∴ ⎜ ⎟
⎝ (n + 1)n 2⎠
n →∞ n n →∞
⇒ un + 1 = x
2.4.6. …… 2n
(n +n 1) n
2.4.6. …… 2n (2n + 2) 1 = nlim
× . →∞
1.3.5. …… (2n – 1) (2n + 1) x2n + 3 1
(2n + 2) 1 = nlim
(1 + 1n)
→∞ n
= .
(2n + 1) x2
un 2n + 2 1 1
⇒ nlim = nlim . = e <1
→ ∞ un + 1 → ∞ 2n + 1 x2
∴ The given series is convergent.
1
= 2 49. (B) In (A), the series is p-series.
x
where p = 3/2 > 1, so it is convergent
1
∴ The series is convergent, if 2 > 1, i.e. x2 < 1 n+1
In (B) nlim u = nlim n =1≠0
x
→∞ n →∞
1 So it is not convergent.
The series is divergent, if 2 < 1, i.e. x2 > 1
x In (C) the series is alternating and un + 1 < u n and
1 lim u = lim 1 = 0.
The ratio test fails, if 2 = 1, i.e., x 2 = 1. But in this
x n →∞ n n →∞ n + 1
case, ∴ The series is convergent (Leibnitz’s test).
un 2n + 2 In (D) The series is geometric and | x | | −< |, so it is
un + 1 = 2n + 1 .
˙·˙
also convergent.
∴ The required answer is (B).
⇒ ( u
n u n –1 =
n+1
) n
(2n + 1) 50. (D)

Quantitative Aptitude Test | 105


Permutation & Combination
Some Important Notations and Examples
Formulae Caution –
n!
r! ()
≠ r !
n

From the examination point of For example Q. 1. If nP4 = 360, find n.


view, the following few result are 8! Solution : Given n P 4 = 360
useful. Without going into details =8×7×6×5
4! n!
you should simply remember the ∴ = 360
results.
1. Factorial Notations—The
= 1680 ≠ () 8
4
!=2!=2 (n – 4)
⇒ n (n – 1) (n – 2) (n – 3) = 360
product of n consecutive positive n!
integers beginning with 1 is denoted 3. n pr = = 6 ×5×4×3
(n – r)
by n! or n and read as factorial n. ∴ n = 6
where n pr = number of permu-
Thus, according to the definition tations or arrangements of n different [Here L.H.S. is the product of 4
things taken r at a time. consecutive integers, therefore, R.H.S.
of n
i.e. 360 is to be expressed as the
n!
n = 1 × 2 × 3 × … × 4. n C r = product of 4 consecutive integers.]
r ! (n – r)
(n – 1) × n 360 = 2 × 2 × 2 × 3 × 3 × 5,
where n C r = number of selec-
greatest of these factors is 5, there-
= n × (n – 1) × (n – 2) × tions, or groups of n different things fore, try with 5.
… ×3×2×1 taken r at a time.
Integers just before and after 5
For example 5. From (3) and (4), we have
np = r ! × nC are 4 and 6. Both 4 and 6 are factors
r r of 360. Thus we get four consecutive
6 = 1 × 2×3× 4 × 5× Total number of arrangements = integers 6, 5, 4 and 3 whose product
6=6 × 5× 4× 3× 2 total no. of group or selections × r ! is 360.
× 1 = 720 6. If n C x = n C y then either x = y If 5 does not suit, then try with
2. According to the definition of n or x + y = n 2 × 5 i.e. 10 etc.
7. Number of permutations of n
Q. 2. If 10Pr = 720, find r.
1. n = n × (n – 1) × (n – 2) things out of which P are alike and
×…×3×2×1 are of one type, q are alike and are of Solution : Given 10P r = 720
the other type, r are alike and are of
= n {(n – 1) × (n – 2) 10!
another type and remaining [n – (p + ∴ = 720
× … × 3 × 2 × 1} (10 – r)!
n!
= n (n – 1) {(n – 2) × q + r)] all are different = . ∴ 10 × 9 × 8 × …… to r factors
p!q!r!
… × 3 × 2 × 1} and = 720 = 10 × 9 × 8
8. Number of selections of r
so on.
things (r ≤ n) out of n identical things ∴ r = 3
∴ n = n n–1 =n is 1. Q. 3. How many numbers of
9. Total number of selections of four digits can be formed with the
(n – 1) n – 2 zero or more things from n identical digits 1, 2, 3, 4 and 5 ? (If repe-
= n (n – 1) (n – 2) things = n + 1. tition of digits is not allowed).
n–3 10. Total number of selection of Solution :
zero or more things from n different
2. If r and n are positive integers things
and r < n, then = n C0 + n C1 + n C2 + … Here n = number of digits = 5
n × (n – 1) × (n – 2) × … × + nCn = 2n .
and r = number of places to be
(r + 1) × r × (r – 1) 11. Number of ways to distribute
filled up = 4
n! ×…×3×2×1 (or divide) n identical things among r
= 5!
r! r × (r – 1) × (r – 2) persons where any persons may get ∴ Required number = 5P 4 =
×…×3×2×1 any no. of things = n + r –1Cr – 1 1!
12. (a) n P n = n! = 5 × 4 × 3 × 2 = 120
= n (n – 1) (n – 2)
… (r + 1) (b) 0! = 1 Q. 4. In a class of 10 students,
there are 3 girls. In how many
n! n!
= n (n – 1) (n – 2) (c) n C r = n C n – r = different ways can they be arran-
(n – r) r ! (n – r) ged in a row such that no two of
… (n – r + 1) (d) n C n = n C 0 = 1 the three girls are consecutive ?

Quantitative Aptitude Test | 106


Solution : be seated in a row so that all the Q. 8. In how many ways 4 boys
No. of girls = 3, number of boys three girls do not sit together. and 3 girls can be seated in a row
= 7. Since there is no restriction on Solution : Total number of per- so that they are alternate ?
boys, therefore, first of all fix the sons = 5 + 3 = 8. When there is no Solution : Possible arrangement
positions of the 7 boys. restriction they can be seated in a row will be of the form
Now, 7 boys can be arranged in in 8! ways.
B G B G B G B
a row in 7 ! ways. But when all the three girls sit
×B×B×B×B×B×B×B× together, regarding the three girls as There are four places namely 1st,
one person, we have only 5 + 1 = 6 3rd, 5th and 7th for four boys.
If the positions of girls are fixed persons.
at places (including the two ends) ∴ Four boys can be seated in 4!
indicated by crosses, no two of three These 6 persons can be arranged ways. Again there are three places
girls will be consecutive. in a row in 6! ways. But the three namely 2nd, 4th and 6th for three
girls can be arranged among them- girls.
Now, there are 8 places for 3 selves in 3! ways.
girls. ∴ Three girls can be seated in 3!
∴ Number of ways when three ways.
∴ 3 girls can be arranged in 8 P 3 girls are together
ways ∴ Required number = 4! 3! =
= 6! × 3! 144.
∴ Required number of ways =
∴ Required number of ways in
8 P × 7! = 8! × 7! which all the three girls do not sit Short Method : Applying the
3
5! above theorem, we have the requi-
together
red answer = 4! 3! = 144.
Short Method : Applying the = 8! – 6! × 3! = 6! (8 × 7 – 6)
above theorem, we have required = 50 × 6! = 36000
8! Exercise
answer = 7 + 1P3 × 7! = × 7!.
5! Short Method : Applying the 01. How many number between 400
Q. 5. In how many ways can 10 above theorem, we have the requi- and 1000 can be made with the
examination papers be arranged so red no. of ways digits 2, 3, 4, 5, 6 and 0 ?
that the best and the worst papers = (5 + 3)! – (5 + 1)! × 3! (A) 60 (B) 70
never come together ? = 8! – 6! × 3! = 50 × 6! (C) 40 (D) 120
Solution : The number of per- = 36000 02. Find the number of numbers bet-
mutations of 10 papers when there is
ween 300 and 3000 that can be
no restriction = 10P 10 = 10 !. There are ‘m’ boys and ‘n’ girls.
formed with the digits 0, 1, 2, 3,
When the best and the worst The no. of ways in which they can be 4 and 5 no digits being repeated
papers come together regarding the seated in row so that all the boys do in any number.
two as one paper. We have only 9 not sit together are given by [(m + n)!
(A) 90 (B) 120
papers. – (n + 1)! × m!] ways.
(C) 160 (D) 180
These 9 papers can be arranged Q. 7. In how many ways 4 boys
in 9P 9 = 9! ways. and 4 girls can be seated in a 03. How many even numbers of four
row so that boys and girls are digits can be formed with the
But these two papers can be alternate ? digits 0, 1, 2, 3, 4, 5 and 6, no
arranged among themselves in 2! digit being used more than once?
Solution : When a boy sits at the
ways. first place possible arrangement will (A) 300 (B) 140
∴ Number of arrangements when be of the form (C) 120 (D) 420
the best and the worst papers do not
come together B G B G B G B G 04. How many numbers of four digits
Now there are four places greater than 2300 can be formed
= 10! – 9! × 2! = 9!
namely 1st, 3rd, 5th and 7th for four with digits 0, 1, 2, 3, 4, 5 and 6
(10 – 2) = 8 × 9! boys, therefore, four boys can be no digit being repeated in any
seated in 4! ways. Again there are number ?
Short Method : Applying the
above theorem, we have the four places namely 2nd, 4th, 6th and (A) 480 (B) 560
required number of ways = (10 – 8th for four girls. (C) 660 (D) 580
2) × (10 – 1)! = 8 × 9! = 8 × 9! ∴ Four girls can be seated in 4!
ways. 05. How many positive numbers can
The number of ways in which be formed by using any number
∴ Number of ways in this case =
‘n’ books may be arranged on a shelf of the digits 0, 1, 2, 3 and 4, no
4! × 4!.
so that two particular books shall not digit being repeated in any
be together is (n – 2) × (n – 1)!] Short Method : Applying the numbers ?
Q. 6. There are 5 boys and 3 above theorem, we have the requi- (A) 360 (B) 260
girls. In how many ways can they red answer = 2 (4! × 4!) = 1152. (C) 620 (D) 280

Quantitative Aptitude Test | 107


06. In how many ways can the 13. How many different letter arran- which a committee of 3 students
letters of the word ‘civilization’ gements can be made from the is to be formed.
be arranged ? letters of the word RECOVER ? (A) 2200 (B) 2300
12! (A) 1210 (B) 5040 (C) 2400 (D) 3200
(A)
4! (C) 1260 (D) 1200 21. 8 men entered a lounge simulta-
12!
(B) –1 14. There are 20 books of which 4 neously. If each person shook
4!
are single volume and the other hands with the other, then find
13! are books of 8, 5 and 3 volumes the total no. of hand shakes.
(C) –1
5! respectively. In how many ways (A) 16 (B) 36
(D) None of these can all these books be arranged (C) 56 (D) 28
07. In how many ways can the letters on a self so that volumes of the
of the word ‘Director’ be arran- same book are not separated ? 22. From 4 officers and 8 Jawans in
ged so that the three vowels are (A) 7! 8! 5! 3! how many ways can be 6 chosen
never together ? to include at least one officer.
(B) 7! 8! 4! 3!
(A) 1800 (B) 18000 (A) 896 (B) 986
(C) 7! 6! 5! 3!
(C) 16000 (D) 1600 (C) 886 (D) 996
(D) None of these
08. Find the number of rearrange- 23. From a group of 6 men and 4
15. A library has two books each
ments of the letters of the word women a committee of 4 persons
having three copies and three
‘Benevolent’. How many of them is to be formed :
other books each having two
end in l ? (i) In how many different ways
copies. In how many ways can
(A) 302400, 30239 all these books be arranged in a can it be done so that the com-
(B) 302399, 30239 shelf so that copies of the same mittee has at least one woman ?
(C) 302399, 30240 book are not separated ? (A) 210 (B) 225
(D) None of these (A) 120 (B) 180 (C) 195 (D) 185
09. How many words can be formed (C) 160 (D) 140 (ii) In how many different ways
with the letters of the word 16. 4 boys and 2 girls are to be sea- can it be done so that the com-
‘Pataliputra’ without changing ted in a row in such a way that mittee has at least 2 men ?
the relative order of the vowels two girls are always together. In (A) 210 (B) 225
and consonants ? how many different ways can (C) 195 (D) 185
(A) 3600 (B) 6300 they be seated ?
(C) 3900 (D) 4600 (A) 120 (B) 720 24. The number of straight lines can
be formed out of 10 points of
10. How many different words can (C) 148 (D) 240 which 7 are collinear ?
be formed with the letters of the 17. In how many different ways can (A) 26
word ‘Pencil’ when vowels the letters of word JUDGE be
occupy even places ? arranged so that the vowels (B) 21
(A) 140 always come together ? (C) 25
(B) 147 (A) 48 (B) 24 (D) None of these
(C) 144 (C) 120 (D) 60 25. Which of the following is
(D) Can’t be determined 18. How many groups of 6 persons incorrect ?
11. How many different words can can be formed from 8 men and 7 (A) n C r = n C n – r
be formed with five given letters women ?
(B) n C r = n – 1Cr + nC n – r
of which three are vowels and (A) 5000
two are consonants no two (C) n C r = n – 1Cr + n – 1Cr – 1
(B) 5005
vowels being together in any (D) r! n C r = Pr
word ? (C) 5050
(A) 12 (B) 16 (D) None of these 26. The total number of words,
which can be formed out of the
(C) 18 (D) 10 19. There are 10 oranges in a basket.
letters a, b, c, d, e, f taken 3
Find the no. of ways in which 3
12. Letters of the word DIRECTOR together, such that each word
oranges are chosen from the
are arranged in such a way that contains at least one vowel is—
basket ?
all the vowels come together. (A) 72
Find out the total no. of ways for (A) 125 (B) 140
making such arrangement. (C) 110 (D) 120 (B) 48
(A) 4320 (B) 2720 (C) 96
20. There are 25 students in a class.
(C) 2160 (D) 1120 Find the number of ways in (D) None of these

Quantitative Aptitude Test | 108


27. The number of different permu- 37. If n P r = 720 n C r than the value r that there is a man on either side
tations of the word ‘BANANA’ is— of every woman the number of
is— seating arrangements is—
(A) 6 (B) 5
(A) 270 (B) 60 (A) (7!)2 (B) (6!)2
(C) 4 (D) 7
(C) 120 (D) 360 (C) 6 × 7 (D) 7!
38. If S = {2, 3, 4, 5, 7, 9}, then the
28. A polygon has 44 diagonals the number of different three-digit
number of its sides is— 46. The total number of permutat-
numbers (with all distinct digits)
ions of 4 letters that can be made
(A) 9 (B) 10 less than 400 that can be formed
out of the letters of the word
from S is—
(C) 11 (D) 12 EXAMINATION is—
(A) 20 (B) 40
29. If 7 points out of 12 are in the (A) 2454
same straight line, then the num- (C) 80 (D) 120
(B) 2436
ber of triangles formed is— 39. The number of triangles that can
(C) 2545
(A) 19 (B) 158 be formed by choosing the
(C) 185 (D) 201 vertices from a set of 12 points, (D) None of these
seven of which lie on the same
30. If n + 2C 8 : n – 2P 4 = 57 : 16 then straight line, is— 47. The total number of selections of
the value of n is— fruit which can be made from 3
(A) 185 (B) 175 bananas, 4 apples and 2 oranges
(A) 20 (B) 19 is—
(C) 115 (D) 105
(C) 18 (D) 17 (A) 39
31. Everybody in a room shakes 40. The middle term of 2x – 3x
hands with everybody else. The
(
1 10
) (B) 315
is— (C) 512
total number of hand shakes is
66. The total number of persons 24 25 (D) None of these
(A) 10C 4 4 (B) – 10C5 5
in the room is— 3 3
4
48. The total number of arrange-
(A) 11 (B) 12 2 25 ments of the letters in the
(C) – 10C4 5 (D) 10C 5 5
(C) 13 (D) 14 3 3 expression a3 b2 c4 when written
32. If 56Pr + 6 : 54 P r + 3 = 30800 : 1 41. The number of ways in which 5 at full length is—
then the value of r is— boys and 5 girls can sit in a ring (A) 1260
are—
(A) 40 (B) 2520
(B) 41 (A) 10! (B) 9!
(C) 610
(C) 42 (C) 5! (D) 6!
(D) None of these
(D) None of these 42. How many words of 4 conso-
nants and 3 vowels can be made 49. A library has a copies of one
33. If nC r – 1 = 36, n C r = 84 and nC r+1
from 12 consonants and 4 book, b copies of each of two
= 126 then—
vowels. If all the letters are book, c copies of each of three
(A) n = 8, r = 4 different ? books and single copy of d book.
(B) n = 9, r = 3 (A) 251820 (B) 258120 The total number of ways in
(C) n = 7, r = 5 which these books can be distri-
(C) 281520 (D) 285120 buted is—
(D) None of these
43. If n P r = 120 n C r then r is equal (a + b + c + d)!
34. If 35Cn + 7 = 35C 4 n – 2, then all the to— (A)
a! b! c!
values of n are given by— (A) 4 (B) 5 (a + 2b + 3c + d)!
(A) 28 (B) 3, 6 (C) 6 (D) 7 (B)
a! (b!)2 (c!)3
(C) 3 (D) 6
44. 12 persons are to be arranged to (a + 2b + 3c + d)!
35. If 2n + 1Pn – 1 : 2n – 1P n = 3 : 5, then a round table. If two particular (C)
a! b! c!
n is equal to— persons among them are not to
(A) 4 (B) 6 be side by side, the total number (D) None of these
(C) 3 (D) 8 of arrangements is—
50. In how many different ways can
n nP (A) 9 (10!) (B) 2 (10!) the letters of the word
r
36. The value of Σ is— (C) 45 (8!) (D) 10! ‘PADDLED’ be arranged ?
r=1 r!
(A) 2n (B) 2n – 1 45. Seven women and seven men are (A) 910 (B) 2520
(C) 2n – 1 (D) 2n + 1 to sit round a circular table such (C) 5040 (D) 840

Quantitative Aptitude Test | 109


Answers with Hints
01. (A) [Here nothing has been given about repetition of Units place can be filled up by 0 in 1 way and
digits, therefore, we will assume that repetition of remaining three places can be filled up by remaining
digit is not allowed.] 6 digits in 6 P 3 ways.
Any number between 400 and 1000 must be of three
∴ Number of numbers formed in this case
digits only.
4 or 5 or 6 6!
= 1 × 6P 3 =
3!
↓ ↓
3 ways 5 P ways = 120
2
Case II. When 0 does not occur at units place.
Since the number should be greater than 400, there-
fore, hundreds place can be filled up by anyone of Anyone of remaining 2 or 4
the three digits 4, 5 and 6 in 3 ways. six except zero or 6
Remaining two places can be filled up by remaining ↓ ↓ ↓
five digits 5 P 2 ways. 5P
5 ways 2 ways 3 ways
5!
∴ Required number = 3 × 5P 2 = 3 × = 60 Units place can be filled up by anyone of the three
3!
02. (D) Any number between 300 and 3000 must be of 3 digits 2, 4 and 6 in 3 ways.
or 4 digits. Thousand place can be filled up by any one of the
Case I. When number is of 3 digits. remaining six digits except zero in 5 ways.
3 or 4 or 5 Remaining two places can be filled up by remaining
↓ ↓ five digits in 5 P 2 ways.
3 ways 5 P ways
2 ∴ Number of numbers formed in this case.
Hundreds place can be filled up by anyone of the 5!
= 5 × 3 × 5 P 2 = 15 × = 300
three digits 3, 4 and 5 in 3 ways. 3!
Remaining two places can be filled up by remaining ∴ Required number = 120 + 300 = 420.
five digits in 5 P 2 ways. 04. (B) [Since number must be of four digits and greater
∴ Number of numbers formed in this case = 3 × 5 P 2 than 2300, therefore anyone of the five digits 2, 3, 4,
5! 5 and 6 will occur at thousands, place. When anyone
= 3 × = 60 . of 4, 5, 6 occurs at thousands, place the number will
3!
be definitely greater than 2300 but when 2 occurs at
Case II. When number is of 4 digits
thousands, place there will be also restriction on
1 or 2 hundreads, place to make the number greater than
↓ ↓ 2300.]
2 ways 5 P ways
3 When 2 occurs at thousands, place.
Thousands place can be filled up by anyone of the Case I. 2 3 or 4 or
two digits 1 and 2 in 2 ways and remaining three 5 or 6
places can be filled up by remaining five digits in 5 P 3
↓ ↓ ↓
ways. 5P
1 way 4 way ways
∴ Number of numbers formed in this case 2

5! Thousand’s place can be filled up by 2 in 1 way and


= 2 × 5P 3 = 2 × = 120 hundred’s place can be filled up by any one of the
2!
∴ Required number = 60 + 120 = 180 four digits 3, 4, 5 and 6 in 4 ways.
03. (D) Each even number must have 0, 2, 4 or 6 in its Remaining two places can be filled up by remaining
units, place. five digits in 5 P 2 ways.
Here total number of digits = 7 ∴ Number of numbers formed in this case
0 or 2 or 4 or 6 5!
= 1 × 4 × 5 P 2 = 4 × = 80
[When 0 occurs at units place there is no restriction 3!
on other places and when 2 or 4 or 6 occurs at units Case II.
place there is restriction, on thousands place as 0 can- When anyone of 3, 4, 5 and 6 occurs at thousand’s
not be put at thousands’ place.] place.
Case I. When 0 occurs at units place 3 or 4 or 5
0 or 6
↓ ↓ ↓ ↓
6 P ways 1 way 6P
3 4 ways 3 ways

Quantitative Aptitude Test | 110


Thousand’s place can be filled up by anyone of the When three vowels are together, regarding them as
four digits 3, 4, 5 and 6 in 4 ways and remaining one letter, we have only 5 + 1 = 6 letters.
three places can be filled up by remaining six digits 6!
in 6P 3 ways. These six letters can be arranged in ways, since r
2!
∴ Number of numbers formed in this case occurs twice.
6! But the three vowels can be arranged among
= 4 × 6P 3 = 4 × = 480
3! themselves in 3! ways.
∴ Required number = 80 + 480 = 560. Hence number of arrangements when the three
05. (B) When number is of five digits 6!
vowels are together = × 3!
2!
Case I. 1 or 2 or
3 or 4 ∴ Required number
↓ ↓ 8! 6! 6!
= – × 3! = (8·7 – 6) = 18,000
4 P ways 2! 2! 2!
4 ways 4
08. (C) There are ten letters in the word ‘Benevolent’ of
Ten thousand’s place can be filled up by anyone of which three are e’s and two are n’s.
the four digits 1, 2, 3 and 4 in 4 ways and the remain-
ing four places can be filled up by the remaining four ∴ Total number of arrangements
digits in 4 P 4 ways. 10!
= = 302400
∴ Number of numbers formed in this case = 4 × 4 P 4 . 3! × 2!
Case II : When number is of four digits But one word is Benevolent itself.
1 or 2 or 3 or 4 10!
∴ Number of re-arrangements = – 1 = 302399
↓ ↓ 3! 2!
4 ways 4 P ways When l is put in the end, number of remaining letters
3
is 9 of which three are e’s and two are n’s
Number of numbers formed in this case = 4 × P 3 .4

Case III : When number is of three digits. 9!


∴ Number of words ending in l = = 30240
3! 2!
1 or 2 or 3 or 4
09. (A) There are eleven letters in the word ‘Pataliputra’
↓ ↓ and there are two p’s, two t’s three a’s and four other
4 ways 4 P ways
2 different letters.
∴ Number of numbers formed in this case Number of consonants = 6, number of vowels = 5
= 4 × P24 Since relative order of the vowels and consonants
remains unchanged, therefore, vowels will occupy
Case IV : When number is of two digits. only vowel’s place and consonants will occupy only
1 or 2 or 3 or 4 consonants place.
↓ ↓ Now 6 consonants can be arranged among them-
4 ways 4 P ways
1 6!
selves in ways. [˙.˙ There are two P’s and
∴ Number of numbers formed in this case = 4 × 4P 1. 2! 2!
two t’s]
Case V : When number is of one digit.
and five vowels can be arranged among themselves
Number of positive numbers formed in this case = 4
5!
∴ Required number in ways, since ‘a’ occurs thrice.
3!
= 4 × 4P 4 + 4 × 4P 3 + 4 × 4P 2 + 4 × 4P 1 + 4
6! 5!
= 96 + 96 + 48 + 16 + 4 = 260 ∴ Required numbers = ×
2! 2! 3!
06. (B) There are 12 letters in the word ‘civilization’ of = 3600
which four are i’s and other are different letters.
10. (C) There are 6 letters in the word ‘pencil’ and no
12!
∴ Total number of permutations = letter is repeated. There are two vowels e and i .
4!
Places are :
But one word is civilization itself.
12! 1st 2nd 3rd 4th 5th 6th
∴ Required number of rearrangements = –1
4! Even places are : 2nd, 4th and 6th.
07. (B) Total number of letters = 8.
Now there are three even places for two vowels.
number of vowels = 3 and r occurs twice.
Total number of arrangements when there is no ∴ 2 vowels can be arranged in 3 P 2 = 3! = 6 ways.
restriction Four consonants can be arranged in remaining four
8! places in 4! = 24 ways.
=
2! ∴ Required number = 6 × 24 = 144.

Quantitative Aptitude Test | 111


11. (A) Since there is no resriction on consonants, 19. (D) Required number of ways = 10C 3
therefore, first of all we arrange the two consonants. 10! 10! 10 × 9 × 8
Two consonants can be arranged in 2! ways. = = = = 120
3! (10 – 3)! 3! 7! 3×2
Now if the vowels are put at the places (including the 20. (B) Required number of ways
two ends) indicated by the ‘ב then no two vowels
25 × 24 × 23
will come together = 25C 3 = = 2300
1×2×3
× Consonant × Consonant × 21. (D) Applying the given rule, we have
There are three places for three vowels and hence the 8 (8 – 1)
three vowels can be arranged in these three places in required no. of hand shakes = = 28
2
3 P = 3! ways
3 22. (A) No. of Officers No. of Jawans No. of Ways
Hence, the required number of words, when no two Case I 1 5 4 C × 8C = 224
1 5
vowels are together 4 C × 8C = 420
Case II 2 4 2 4
2! 3! = 12 4 C × 8C = 224
Case III 3 3 3 3
12. (C) Taping all vowels (IEO) as a single letter (since 4 C × 8C = 28
they come together) there are six letters with two Case IV 4 2 4 2
‘R’s ∴ Required number = 224 + 420 + 224 + 28 = 896.
6! Short Method : Applying the above theorem, we
Hence no. of arrangements = × 3! = 2160
2! have, x = 4, y = 8 and n = 6.
[3 vowels can be arranged in 3! ways among The value of
themselves, hence multiplied with 3!.] xC × yC
n –x = C 4 × C 6 – 4 = C 4 × C 2
4 8 4 8
x
7!
13. (C) Possible arrangements are = 1260 Now, from the above theorem,
2! 2!
Required answer
[Division by 2 times 2! is because of the repetition of
= 4 C 1 × 8C 5 + 4 C 2 × 8C 4 + 4 C 3 × 8C 3 + 4 C 4 × 8C 2
E and R.]
14. (A) [Volumes of the same book are not to be separa- = 224 + 420 + 224 + 28 = 896.
ted i.e. all volumes of the same book are to be kept 23. (i) (C) Required no. of ways
together]. Regarding all volumes of the same book as
= 4 C 1 × 6C 3 + 4 C 2 × 6C 2 + 4 C 3 × 6C 1 + 4C 4
one book, we have only 4 + 1 + 1 + 1 = 7 books.
These seven books can be arranged in 7! ways. 6×5 ×4 4 ×3 6 ×5 4 ×3×2
= 4× + × + ×6+1
Volumes of the book having 8 volumes can be 1×2×3 1×2 1×2 1×2×3
arranged among themselves in 8! ways, volumes of = 80 + 90 + 24 + 1 = 195.
the book having 5 volumes can be arranged among
themselves in 5! ways. (ii) (D) Required no. of ways
And Volumes of the book having 3 volumes can be = 6 C 2 × 4C 2 + 6 C 3 × 4C 1 + 6 C 4
arranged among themselves in 3! ways. 6 ×5 4×3 6 ×5×4 6×5×4×3
∴ Required number = 7! 8! 5! 3! = × + ×4+
1×2 1×2 1×2×3 1 ×2×3×4
15. (A) Regarding all copies of the same book as one = 90 + 80 + 15 = 185
book, we have only 5 books.
24. (C) If there were no three points collinear. We should
These 5 books can be arranged in 5! ways. But all have 10C 2 lines but since 7 points are collinear we
copies of the same book being identical can be arran- must subtract 7 C 2 lines and add the one correspond-
ged in only one way.
ing to the line of collinearity of the seven points.
∴ Required number = 5! × 1 × 1 × 1 × 1 = 120
Thus, the required number of straight lines.
16. (D) Assume the 2 girl students to be together i.e.
= 10C 2 – 7 C 2 + 1 = 25
(one). Now there are 5 students.
25. (B)
Possible ways of arranging them are 5! = 120.
26. (C) The required number of words is :
Now they (two girls) can arrange themselves in 2!
ways. (2C 1 × 4C 2 + 2 C 2 × 4C 1 ) 3! = 96
Hence, total ways = 120 × 2 = 240. 27. (B) There are 3A’s 2N’s and one B. We have to find
the total number of arrangements of 6 letters out of
17. (A) Required number = 4! 2! = 48. which 3 are alike of one kind, 2 are alike of second
18. (B) Total no. of persons = 8 + 7 = 15 kind, thus the total number of words
15! 15! 6!
No. of groups = 15C 6 = = = = 60
6! (15 – 6)! 6! 9! 3! 2!
15 × 14 × 13 × 12 × 11 × 10 28. (C) Let there be n sides of the polygon. Then it has n
=
6 ×5×4×3×2×1 vertices. The total number of straight lines obtained
= 5005 by joining n vertices by taking 2 at a time is n C 2 .

Quantitative Aptitude Test | 112


These n C 2 lines also include n sides of polygon. n–r+1 7
Therefore, the number of diagonals formed is n C 2 – n. ⇒ = …(1)
r 3
Thus n C – n = 44
2 n–r 3
and = …(2)
n (n – 1) r+1 2
⇒ – n = 44
2 ⇒ 3n – 10r + 3 = 0 …(1)
n2 – 3n and 2n – 5r – 3 = 0 …(2)
⇒ = 44
2 ⇒ r = 3, n = 9
⇒ n2 – 3n = 88
34. (B) 35C n + 7 = 35C 4n – 2
⇒ 2
n – 3n – 88 = 0
[We know that n C x = n C y, x = y or x + y = n]
⇒ (n – 11) (n + 8) = 0
⇒ If n + 7 + 4n – 2 = 35
∴ n = 11
⇒ 5n + 5 = 35
29. (C) The number of ways of selecting 3 points out of
12 points is 12C 3 . The number of ways of selecting 3 ⇒ 5n = 30 ∴ n ⇒ 6
points out of 7 points, on the same straight line is ⇒ If n + 7 = 4n – 2
7 C . Hence, the number of triangle formed will be
3
12C – 7 C = 210 – 35 = 185. ⇒ 4n – n = 2 + 7
3 3
n + 2C : n – 2P = 57 : 16 ⇒ 3n = 9
30. (B) ˙·˙ 8 4
(n + 2)! (n – 6)! ∴ n = 3
57
⇒ = 2n + 1P
(n – 6)! (n – 2)! 8! 16 3
35. (A) We have 2n –1 n – 1 =
⇒ (n + 2) (n + 1) n (n – 1) = 143640 Pn 5
⇒ (n2 + n – 2) (n2 + n) = 143640 ⇒ 5 2n + 1P n – 1 = 3 2n – 1P n
⇒ (n + n)2 – 2 (n2 + n) + 1 = 143641
2 5 (2n + 1)! 3 (2n – 1 )!
⇒ =
⇒ (n2 + n – 1)2 = (379) 2 (n + 2)! (n – 1)!
[˙.˙ n2 + n – 1 > 0] 5 (2n + 1) (2n) (2n – 1)! 3. (2n – 1)!
⇒ =
⇒ n2 + n – 1 = 379 (n + 2) (n + 1) n (n – 1)! (n – 1)!
⇒ 2
n + n – 380 = 0 ⇒ 10 (2n + 1) = 3 (n + 2) (n + 1)
⇒ (n + 20) (n – 19) = 0 ⇒ 3 n 2 – 11n – 4 = 0 ∴ n = 4
n = 19 ⇒ (3n + 1) (n – 4) = 0
(˙.˙ n is not negative) 36. (B) ˙·˙ n P r = n C r r!
31. (B) Let there be n persons in the room. The total nP
r
number of hand shakes is same as the number of ⇒ = n Cr
r!
ways of selecting 2 out of n. n nP n
r
˙·˙ n C = 66
2
∴ Σ = Σ Cr
r = 1 r! r =1
n (n – 1) = C 1 + n C 2 + n C 3 + …… n C n
n
⇒ = 66
2!
= 2n – 1
⇒ n2 – n – 132 = 0
37. (A) We have P r = n C r r!
n
⇒ (n – 12) ( n + 11) = 0 n P = 720 n C
˙·˙ r r
∴ n = 12
⇒ n C . r! = 720 n C
r r
32. 56 54
(B) ˙·˙ P r + 6 : P r + 3 = 30800 : 1
⇒ r! = 720

56!
(50 – r)!
= 30800 ( 54!
)
51 – r!
∴ r = 6
38. (A) The hundreds place will be reserved for 3 or 2, 5
30800
⇒ 56 × 55 = digits are free to fill rest two places i.e., of tens and
(51 – r) unit. Number of required 3 digit numbers = 2 × 5 C 2
⇒ 51 – r = 10 = 20.
∴ r = 41 39. (A) Required no. of triangles = 12C3 – 7 C 3 = 185
33. (B) n C r – 1 = 36, n C r = 84, nC r + 1 = 126
( ) n th
40. (B) Middle term = +1 term
nC
r 84 2
⇒ = …(1)
( )
nC 36 10 th
r–1
= +1 term
nC
126 2
r+1
and = …(2)
nC
r 84 = 6th term

Quantitative Aptitude Test | 113


( 3x1 ) 5 46. (A) We have 11 letters viz. A; A; I; I; N; N; E; X
= 10C (2x)5 –
5
M, T, O
10C 25 For groups of 4 we may arrange these as follows :
= – 5 5
3 (i) Two alike, two others alike
41. (B) Here n = 5 + 5 = 10 (ii) Two alike, two different
(iii) All four different
∴ Total no. of circular permutations
(i) gives rise to 3 C 2 selections (ii) gives rise 3 × 7C 2
= n–1 selections and (iii) gives rise to 8 C 4 selections.
So, number of permutations
= 10 – 1
4 4
= 9 = 3C
2 × + 3 C 1 × 7C 2 × + 8 C4 × 4
2 × 2 2
42. (D) Required no. of words
= 12P 4 × 4P 3 = (34 + 632 + 70) × 4
= 12 × 11 × 10 × 9 × 4 × 3 × 2
= ( ) × 24
3 + 126 + 280
= 120 (100 – 1) × 24 4
= 288000 – 2880 = 285120 = 409 × 6
43. (B) ˙·˙ n P = 120 n C = 2454
r r

n n 47. (D) Required number of ways


⇒ = 120 × = (2 + 1) (3 + 1) (4 + 1) – 1
n–r n–r. r = 59
⇒ r = 120 = 5 × 4 × 3 × 2 × 1 48. (A) We have 9 letters 3 a’s, 2 b’s and 4 c’s. These 9
9!
∴ r = 5 letters can be arranged in = 1260 ways.
3! 2! 4!
44. (A) 12 persons can be seated around a round table in 49. (B) Total number of books = a + 2b + 3c + d. Since
11! ways. The total number of ways in which 2 there are b copies of each of two books, c copies of
particular persons sit side by side = 10! × 2!. Hence, each of three books and single copy of d book.
the required number of arrangements Therefore, the total number of arrangements is
= 11! – 10! × 2! = 9 × (10!) (a + 2b + 3c + d)!
45. (C) 7 women can sit on a round table in (7 – 1)! = 6 a! (b!)2 (c!)3
ways. Now seven places are created which can be 7
filled by 7 men in 7! ways. Hence required number 50. (D) Reqd. ways = = 840.
of ways = 6! × 7!. 3

Quantitative Aptitude Test | 114


Simple Interest
Important Points/Facts 7. When the time is given in Q. 2. What is the rate of simple
months and days, 12 months are interest at which a sum of money
1. If any person borrowed some
reckoned to a year and 30 days to the doubles itself in 5 years ?
money from the other for a fixed
month. But when the time is given in Solution : Let Rs. P doubles in 5
period, then he has to pay some extra
days or in years and days, 365 days years at the rate of simple interest r%
money to him for the use of money
are reckoned to a year. per annum.
lent. This extra money paid for using
other’s money is called interest. 8. If a sum of money is deposited P× r×5
for n months in a recurring deposit, ∴ P×2 = P+
Interest is the money paid by the 100
then 100 + r × 5
borrower to the lender for the use of
money lent. The money borrowed is Time =
n (n + 1)
months
⇒P×2 = P ( 100 )
called Principal or Sum. 2 100 + r × 5
2. The sum of the principal and n (n + 1) ⇒ 2 =
100
interest is called the amount. = year
2 × 12 ⇒ r × 5 = 200 – 100
∴ Amount = Principal + Interest 100
or Interest = Amount – Principal
Examples ∴ r = = 20%
5
Q. 1. A sum of money doubles
3. Interest is usually calculated at itself in 20 years. In how many Short Method
the rate of so many rupees for every years will it become four times ? (n – 1) × 100
Rs. 100 of the money lent for a year. Required rate =
This is called the rate per cent per Solution : Let Rs. P become 2 t
annum. times in 20 years at the rate of r% per (2 – 1) × 100
annum and will become 4 times in T =
‘‘Per annum’’ means for a year. 5
years.
The words per annum are sometimes = 20%
omitted. Thus 6 P.C. means Rs. 6 is P × r × 20
˙·˙ 2 × P = P + …(i) Q. 3. A deposits a sum of
interest on Rs. 100 in one year. 100
money in a Bank, who pay 10%
4. The interest is usually paid P× r×T
and4 × P = P + …(ii) simple interest. At the end of 5
yearly, half yearly or quarterly as 100 years, he receives Rs. 30000, then
agreed upon between money lender r × 20
and the borrower. 2×P = P 1+ ( 100 ) what sum he had deposited in the
Bank ?
5. If the interest on a certain sum Solution : Let that sum of money
100 + r × 20
for a certain period is reckoned ⇒ 2 = be Rs. P
uniformly. It is called simple interest, 100
P × 10 × 5
abbreviated as S.I. when interest is ⇒ r × 20 = 200 – 100 ˙·˙ S.I. =
100
calculated on the original principal 100
for any length of time. It is called ⇒ r = = 5% 50P
20 =
Simple interest. 100
Substituting the value of r in From the question,
This may be remembered in the equation (ii)
symbolic form— 50P
P (100 + 5 × T ) ˙·˙ 30000 = P +
⇒ 100
P× t×r 4P =
100
S.I. =
100
where, P = Principal, 4 =
100 + 5 × T ⇒ 30000 = P 1 + ( 50
100 )
100 30000 × 100
t = Number of years ⇒ P =
5 × T = 400 – 100 150
r = Rate% per annum. = 300 = Rs. 20000
6. When simple interest is to be 300 ∴ Amount deposited in Bank
T = = 60 years
calculated between the two dates, the 5 = Rs. 20000
day/date on which the money is paid Hence, Required time = 60 years
back should be included but not the Short Method
day on which it is borrowed, i.e., in Short Method 100A
Required sum =
counting, the first day is omitted. (n – 1)t 100 + r × t
Required time =
Example : To reckon the period m–1 100 × 30000
=
between 20th April to 20th Sep- (4 – 1) × 20 100 + 10 × 5
tember, the 20th April shall be =
(2 – 1) 100 × 30000
omitted and the required time will be =
= 3 × 20 150
counted from 21st April to 20th
= 60 years = Rs. 20000
September.

Quantitative Aptitude Test | 115


Q. 4. A man lent a sum of Short Method P × 5·5 × 1 P × 5 × 1
= –
money at the rate of simple interest 100 100
4%. If the interest for 8 years is D × 100
Required sum = P (5·5 – 5)
Rs. 340 less than the principal, then r×t =
100
what was the principal ? 120 × 100
= P × 0·5
Solution : Let the principal be 4×3 =
Rs. P 100
= Rs. 1000
P × 4 × 8 32P 676·5 × 100 × 0·5
S.I. = = Q. 6. At a certain rate of simple =
100 100 5·5 × 1 × 100
interest Rs. 800 amounted to
676 × 100
From the question,
32P
Rs. 920 in 3 years. If the rate of
interest be increased by 3%. What
[˙.˙ P =
5·5 × 1 ]
˙·˙ P– = 340 will be the amount after 3 years ? = 61·5
100
68P Solution : Let the original rate of Hence, Required sum = Rs. 61·5
⇒ = 340 interest is R%.
100 Short Method
800 × 3 × R
340 × 100 ˙·˙ 920 = 800 + Required sum
⇒ P = 100
68 R×r
800 × 3 × R = ×1
= 500 ⇒ 120 = R
100
Hence Principal = Rs. 500 5·5 – 5
120 × 100 = × 676·5
Short Method ∴ R = = 5% 5·5
800 × 3
0·5 × 676·5
100 × d At the rate of interest increased =
Principal = 5·5
100 – r × t by 3%. = Rs. 61·5
100 × 340 New rate = (5 + 3)% = 8%
= Q. 8. A sum of Rs. 810 is
100 – 4 × 8 800 × 8 × 3
∴ Interest = divided in two parts in such a way
100 × 340 100
= that the interest on first part at 4%
68 = Rs. 192 for 3 years is equal to that on
= Rs. 500 ∴ Amount = 800 + 192 second part at 5% for 3 years. Find
= 992 the two parts of sum.
Q. 5. A sum was put at S.I. at a
∴ Hence, Required amount Solution : Let the first part of
certain rate for 3 years. Had it been
the sum be Rs. x
put at 4% higher rate of interest, it = Rs. 992
would have fetched Rs. 120 more. ∴Second part of sum
Short Method
Find the sum. = Rs. (810 – x)
∴ Required amount
Solution : Let the required sum ∴Interest on the first part
be Rs. P and original rate is R%. P× t×r x×4×3
= A± =
100
∴ New rate of interest 100
800 × 3 × 3
= (R + 4)% = 920 + and interest on the second part
100
From the question, (810 – x) × 5 × 3
= 920 + 72 =
P × (R + 4) × 3 P × R × 3 100
˙·˙ – = Rs. 992
100 100 From the question,
Q. 7. The amount of interest for x×4×3
= 120 ˙·˙
a certain sum at 5·5% annual rate 100
P×R×3 P×4×3 of interest for one year is Rs. 676·5.
⇒ + (810 – x) × 5 × 3
100 100 How much the interest will be less =
P×R×3 on the same sum at 5% annual rate 100
– = 120 of interest for one year ?
100 ⇒ 4x = (810 – x) × 5
Solution : Let the certain sum be
P×4×3 ⇒ 4x = 4050 – 5x
⇒ = 120 Rs. P
100 ⇒ 9x = 4050
P × 5·5 × 1
120 × 100 ˙·˙ 676·5 =
⇒ P = 100 4050
4×3 ∴ x = = Rs. 450
P×5×1 9
∴ P = Rs. 1000 and second interest =
100 ∴Second part
Hence, Required sum ∴ Required sum = 810 – 450
= Rs. 1000 = 676·5 – Second interest = Rs. 360

Quantitative Aptitude Test | 116


Exercise 08. If the amount of Rs. 360 in 3 received altogether from both
years is Rs. 511·20. What will be Rs. 90 as simple interest. The
01. The simple interest on a certain
the amount of Rs. 700 in 5 rate of interest is—
sum for 3 years at 14% per
annum is Rs. 235·20. The sum years ? (A) 4% (B) 5%
is— (A) Rs. 1190 (B) Rs. 1230 (C) 10% (D) 12%
(A) Rs. 480 (B) Rs. 560 (C) Rs. 1060 (D) Rs. 1225 16. At the certain rate of simple
(C) Rs. 650 (D) Rs. 720 09. A sum of Rs. 2600 is lent out in interest, a certain sum doubles
02. A sum of money amounts to two part in such a way that the itself in 10 years. It will treble
Rs. 702 in 2 years and Rs. 783 in interest on one part at 10% for 5 itself in—
3 years. The rate per cent is— years is equal to that on another (A) 15 years (B) 20 years
(A) 12% per annum part at 9% for 6 years. The sum (C) 30 years (D) 12 years
(B) 13% per annum lent out at 10% is—
17. A sum of money at simple
(C) 14% per annum (A) Rs. 1150 (B) Rs. 1250
interest amounts to Rs. 2240 in 2
(D) 15% per annum (C) Rs. 1350 (D) Rs. 1450 years and Rs. 2600 in 5 years.
03. If Rs. 64 amount to Rs. 83·20 in 2 10. Simple interest on a certain sum The sum is—
years. What will Rs. 86 amount 9 (A) Rs. 1880
at a certain rate is of the sum.
to in 4 years at the same rate per 16 (B) Rs. 2000
cent per annum ? If the number representing rate (C) Rs. 2120
(A) Rs. 137·60 per cent and time in years be
(D) Data inadequate
(B) Rs. 124·70 equal, then the time is—
1 1 18. Rs. 800 amounts to Rs. 920 in 3
(C) Rs. 114·80 (A) 5 years (B) 6 years
2 2 years at simple interest. If the
(D) Rs. 127·40 interest rate is increased by 3%.
1 1
04. A sum of money amounts to (C) 6 years (D) 7 years It would amount to how much ?
4 2
Rs. 850 in 3 years and Rs. 925 in (A) Rs. 1056 (B) Rs. 1112
4 years. The sum is— 11. A sum of Rs. 2540 is lent out into (C) Rs. 1182 (D) Rs. 992
(A) Rs. 600 two parts. One at 12% and
1 19. The simple interest on a sum of
(B) Rs. 575 another one at 12 %. If the total money at 8% per annum for 6
2
(C) Rs. 625 years is half the sum. The sum
annual income is Rs. 311·60, the
(D) Data inadequate money lent at 12% is— is—
05. The simple interest on a sum of (A) Rs. 1180 (B) Rs. 1360 (A) Rs. 4800
1 (C) Rs. 1240 (D) Rs. 1340 (B) Rs. 6000
money is of the principal and
9 (C) Rs. 8000
1
the number of years is equal to 12. A man invested of his capital (D) Data inadequate
the rate per cent per annum. The 3
rate per cent per annum is— 1 20. If the interest on Rs. 1200 be
at 7%, at 8% and remainder at
4 more than the interest on
1
(A) 3 (B) 10%. If his annual income is Rs. 1000 by Rs. 50 in 3 years,
3 the rate per cent is—
Rs. 561, the capital is—
1 3 1 2
(C) 3 (D) (A) Rs. 5400 (B) Rs. 6000
3 10 (A) 10 % (B) 6 %
(C) Rs. 6600 (D) Rs. 7200 3 3
06. A moneylender finds that due to 1 2
13. A certain sum of money at simple (C) 8 % (D) 9 %
a fall in the rate of interest from 3 3
interest amounts to Rs. 1260 in 2
1
13% to 12 % his yearly income years and to Rs. 1350 in 5 years. 21. The simple interest on a sum of
2 The rate per cent per annum is— money will be Rs. 600 after 10
diminishes by Rs. 104. His capi-
(A) 2·5% (B) 3·75% years. If the principal is trebled
tal is—
(C) 5% (D) 7·5% after 5 years, what will be the
(A) Rs. 21400 total interest at the end of the
(B) Rs. 20800 14. A sum of money put at simple tenth year ?
(C) Rs. 22300 interest trebles itself in 15 years. (A) Rs. 600
(D) Rs. 24000 The rate per cent per annum is— (B) Rs. 900
1 2 (C) Rs. 1200
07. A sum of money will double itself (A) 13 % (B) 16 %
in 16 years at simple interest with 3 3 (D) Data inadequate
yearly rate of— 2
(C) 12 % (D) 20% 22. A man lends Rs. 10000 in four
1 3
(A) 10% (B) 6 % parts. If he gets 8% on Rs. 2000,
4 15. A lent Rs. 600 to B for 2 years 1 1
(C) 8% (D) 16% 7 % on Rs. 4000 and 8 % on
and Rs. 150 to C for 4 years and 2 2
Quantitative Aptitude Test | 117
Rs. 1400. What per cent must he 29. At simple interest, a sum doubles Rs. 1211. Find the rates of inte-
get for the remainder if the after 20 years. The rate of interest rest—
average interest is 8·13% ? per annum is— 1 1
(A) 3 %, 4 %
(A) 7% (B) 9% (A) 5% 2 2
1 1 (B) 10% 1 1
(C) 9 % (D) 10 % (B) 2 %, 3 %
4 2 (C) 12% 2 2
(D) Data inadequate 1 1
23. In how many years will a sum of (C) 4 %, 5 %
money double itself at 12% per 2 2
30. What annual payment will dis-
annum ? charge a debt of Rs. 580 due in 5 (D) None of these
(A) 6 years 9 months years, the rate being 8% per 36. If Rs. 5600 amount to Rs. 6678
annum ? 1
(B) 8 years 4 months in 3 years. What will Rs. 9400
(A) Rs. 166·40 2
(C) 7 years 6 months
(B) Rs. 65·60 1
(D) 8 years 6 months amount to in 5 years at the
4
(C) Rs. 100
same rate per cent per annum
24. A sum was put at simple interest (D) Rs. 120 simple interest ?
at a certain rate for 2 years. Had
it been put at 1% higher rate, it 31. A certain sum of money at simple (A) Rs. 9400
would have fetched Rs. 24 more. interest amounts to Rs. 1012 in (B) Rs. 12114·25
The sum is— 1
2 years and to Rs. 1067·20 in 4 (C) Rs. 12115
2
(A) Rs. 600 (B) Rs. 800 (D) None of these
years. The rate of interest per
(C) Rs. 1200 (D) Rs. 480 annum is— 37. The simple interest on a sum of
25. The simple interest at x% for x (A) 2·5% (B) 3% money will be Rs. 600 after 10
years will be Rs. x on a sum of— (C) 4% (D) 5% years. If the principal is trebled
after 5 years the total interest at
(A) Rs. x 32. Two equal amounts of money are the end of 10 years will be—
(B) Rs. 100x deposited in two banks, each at (A) Rs. 600
1
(C) Rs. ( )100
x
15% per annum for 3 years and
2
5 years. If the difference between
(B) Rs. 900
(C) Rs. 1200
(D) Rs. (
x )
100 their interests is Rs. 144, each (D) Data inadequate
2
sum is—
38. Ashok took a loan of Rs. 15000
26. A sum of money becomes (8/5) (A) Rs. 460 (B) Rs. 500 for 3 years at simple interest. If
of itself in 5 years at a certain (C) Rs. 640 (D) Rs. 720 the total interest paid is Rs. 2700.
rate of interest. The rate per cent What is the rate of interest per
33. If x is the simple interest on y and
per annum is— annum ?
y is the simple interest on z, the
(A) 5% (B) 8% (A) 18 (B) 5·4
rate % and the time being the
(C) 10% (D) 12% same in both cases, what is the (C) 9 (D) 6
27. The rate of interest on a sum of relation between x, y and z ?
39. Pratap borrowed some money
money is 4% per annum for the (A) x2 = yz (B) y2 = xz from Arun at simple interest. The
first 2 years, 6% per annum for (C) z2 = xy (D) xyz = 1 rate of interest for the first 3
the next 4 years and 8% per years was 12% for the next 5
annum for the period beyond 6 34. If the rate of interest rises from years was 16% and beyond this
years. If the simple interest 1 it was 20%. If the simple interest
6 to 8% a man’s annual income
accrued by the sum for a total 2 for 11 years was more than the
period of 9 years is Rs. 1120. increases by Rs. 4050. Find the money borrowed by Rs. 6080.
What is the sum ? capital. What was the money borrowed ?
(A) Rs. 1500 (B) Rs. 2000 (A) Rs. 270000 (A) Rs. 7550 (B) Rs. 8500
(C) Rs. 2500 (D) Rs. 4000 (B) Rs. 370000 (C) Rs. 8000 (D) Rs. 9000
28. The difference between the inte- (C) Rs. 300000 40. Yogesh borrowed Rs. 12000 at
rests received from two different (D) None of these simple interest for 5 years. If he
banks on Rs. 500 for 2 years is paid Rs. 3600 as simple interest
35. A man derives his income from
Rs. 2·50. The difference between after 5 years. What is the rate of
the investment of Rs. 4150 at a
their rates is— interest per cent per annum ?
certain rate of interest and
(A) 1% (B) 0·5% Rs. 3500 at 1 per cent higher. (A) 5 (B) 8
(C) 2·5% (D) 0·25% This whole income for 4 years is (C) 10 (D) 6

Quantitative Aptitude Test | 118


41. The rate of interest for the first 2 loan. What was the rate of inte- all after 5 years. What did he
years is 5% for the next 3 years rest ? borrow ?
is 8% and beyond this. It is 10% (A) 6 (A) Rs. 4600
per annum. If the simple interest (B) 8 (B) Rs. 4000
for 8 years is Rs. 1280. What is (C) 5 (C) Rs. 5000
the principal ?
(D) None of these (D) None of these
(A) Rs. 2500
44. Anil borrowed some money at 46. Prabhat borrowed some money
(B) Rs. 2000
simple interest. The rate of from a bank at 8% p.a. simple
(C) Rs. 3200 interest and lended it to Ashish
interest for the first 3 years was
(D) Rs. 2440 5% for the next 2 years was 8% at 12% p.a. simple interest. If his
42. If a sum of money double itself and for the next 4 years it was profit in 2 years is Rs. 320. What
in 20 years. What is the rate of 10%. Beyond this it was 12% per sum of money did he borrow ?
simple interest per cent per year ? annum. If he paid Rs. 8560, as (A) Rs. 2000 (B) Rs. 4000
(A) 4% (B) 8% interest for 12 years. How much (C) Rs. 6000 (D) Rs. 3000
money did he borrow ?
(C) 5% (D) 10% 47. Rs. 4200 amounts to Rs. 5712 in
(A) Rs. 8000 (B) Rs. 6000 4 years.If the rate of interest is
43. Mr. Patel borrowed Rs. 8000 (C) Rs. 5000 (D) Rs. 7000 increased by 3%. What will be
from Mr. Chobey at simple
interest. After 2 years he paid 45. Dayanand borrowed some money the amount ?
Rs. 800 more than what be at simple interest at 16% per (A) Rs. 6372 (B) Rs. 4000
borrowed and thus cleared the annum. He repaid Rs. 10080 in (C) Rs. 6216 (D) Rs. 3000

Answers with Hints


100 × 235·20 06. (B) Let capital = Rs. x
01. (B) Required sum = Rs. = Rs. 560
3 × 14 x × 13 × 1
02. (D) S.I. for 1 year = Rs. (783 – 702) = Rs. 81 Then, ( 100 ) ( – x×
25 1
×
2 100 ) = 104
S.I. for 2 years = Rs. (81 × 2) = Rs. 162 13x x
⇒ – = 104
∴ Sum = Rs. (702 – 162) = Rs. 540 100 8
100 × 162 ⇒ 26x – 25x = (104 × 200)
∴ Required rate = ( 540 × 2 ) % = 15%
⇒ x = 20800
03. (A) S.I. on Rs. 64 for 2 years = Rs. 19·20 ∴ Capital = Rs. 20800
100 × 19·20
∴ Rate = ( 64 × 2 ) 07. (B) Let principal = Rs. P then S.I. = Rs. P
Then, Rate =
100 × P 100
= =6 %
1
= 15% P × 16 16 4
86 × 4 × 15 08. (A) S.I. for 3 years = Rs. (511·20 – 360)
∴ S.I. on Rs. 86 for 4 years = Rs. ( 100 ) = Rs. 151·20
= Rs. 51·60 100 × 151·20
∴ Rate = = 14%
360 × 3
∴ Amount of Rs. 86 = Rs. (86 + 51·60)
∴ S.I. on Rs. 700 for 5 years
= Rs. 137·60
700 × 5 × 14
04. (C) S.I. for 1 year = Rs. (925 – 850) = Rs. 75
S.I. for 3 years = Rs. (75 × 3) = Rs. 225
= Rs.( 100 )
= Rs. 490
∴ Sum = Rs. (850 – 225) = Rs. 625
∴ Required amount = Rs. (700 + 490) = Rs. 1190
05. (C) Let principal = Rs. P
09. (C) Let the money at 10% be Rs. x. Then the money
P
Then S.I. = at 9% is Rs. (2600 – x)
9
x × 10 × 5 (2600 – x) × 9 × 6
Let Rate = R% per annum ∴ =
100 100
and time = R years
or 104x = 2600 × 54
P P×R×R
Then, = 2600 × 54
9 100 or x = = Rs. 1350
104
100
⇒ R2 = 9
9 10. (D) Let principal = P, then S.I. = P.
16
10 1
∴ R = = 3 % per annum Let rate = R% per annum and Time = R years
3 3
Quantitative Aptitude Test | 119
9 P×R×R 18. (D) Principal = Rs. 800
Then, P =
16 100 S.I. = Rs. (920 – 800) = Rs. 120
900 and Time = 3 years
⇒ R2 =
16 100 × 120
∴ Original rate = = 5%
30 1 800 × 3
∴ R= = 7 years
4 2 New rate = 8%
11. (A) Let money lent at 12% = Rs. x 800 × 8 × 3
Then,
1
money lent at 12 % = Rs. (2540 – x)
Now, S.I. = Rs. ( 100 ) = Rs. 192
2 ∴ Amount = Rs. (800 + 192) = Rs. 992
x × 12 × 1 25 1 19. (D) Let, Sum = P
˙·˙ + (2540 – x) × × = 311·60
100 2 100 1
Then, S.I. = P
3x 2540 – x 2
⇒ + = 311·60
25 8 Rate = 8%
⇒ 24x + 25(2540 – x) = 200 × 311·60 and Time = 6 years
∴ x = 63500 – 62320 P P×8×6
But = (Not possible)
= 1180 2 100
12. (C) Let the capital be Rs. x, then Thus, data is inadequate.
20. (C) Let rate = x% per annum. Then,
x
×
7
+ ×
3 100 4 100
x 8
[ ( )]
+ x–
x x
+
3 4
×
10
100
= 561
1200 × x × 3 1000 × x × 3
– = 50
7x x x 100 100
⇒ + + = 561
300 50 24 ⇒ 6x = 50
⇒ 42x + 36x + 75x = 1009800 1
⇒ x = 8
1009800 3
∴ x = = 6600
153 1
∴ Rate = 8 % per annum
13. (A) S.I. for 3 years = Rs. (1350 – 1260) = Rs. 90 3
S.I. for 2 years = Rs. ( )
90
3
× 2 = Rs. 60 21. (B) Let, sum = Rs. x. Then, S.I. = Rs. 600, Time
= 10 years.
∴ Sum = Rs. (1260 – 60) = Rs. 1200 100 × 600
∴ Rate =
100 × 60
= 2·5%
∴ Rate = ( )x × 10
1200 × 2
14. (A) Let principal = P. = ( )6000
x % per annum
Then S.I. = 2P S.I. on Rs. x for 1st five years
Also Time = 15 years
∴ Required rate =
100 × 2P 200
P × 15
=
15
( 6000
= Rs. x × x × 5 ×
1
100 ) = Rs. 300
1 S.I. on Rs. 3x for next 5 years
= 13 % per annum
15. (B) Let rate = x% per annum.
3
( 6000
= Rs. 3x × x × 5 ×
1
100 )
600 × x × 2 150 × x × 4 = Rs. 900
Then + = 90
2000 × 8 × 1

100 100
18x = 90
22. (B) ˙·˙
100 ( 15 1
+ 4000 × ×
2 100 )
∴ 2600 × x × 1
16.
x = 5%
(B) Let principal = P. Then, S. I. = P. and Time (
+ 1400 × ×
17 1
2 100 ) (
+
100 )
= 10 years 10000 × 8·13 × 1
∴ Required time =
(n – 1) × t = ( 100 )
(m – 1) ⇒ 160 + 300 + 119 + 26x = 813
(3 – 1) × 10
= ⇒ 26x = 234
(2 – 1)
= 20 years ⇒ x = 9%
17. (B) S.I. for 3 years = Rs. (2600 – 2240) = Rs. 360 23. (B) Let principal = Rs. P. Then, S.I. = Rs. P,
S.I. for 2 years = Rs. ( 360
) × 2 = Rs. 240
Rate = 12%
100 × P

3
Required sum = Rs. (2240 – 240) = Rs. 2000
Time = (P × 12 ) years

Quantitative Aptitude Test | 120


25 32. (C) Let each sum be Rs. P. Then,
= years
3 P × 15 × 5 P × 15 × 7
˙·˙ – = 144
= 8 years 4 months 100 100 × 2
24. (C) Let sum = Rs. x 3 21
⇒ P – P = 144
and original rate = y% per annum 4 40
x × (y + 1) × 2 x × y × 2 9P
Then, – = 24 ⇒ = 144
100 100 40
⇒ x = 1200 144 × 40
∴ P = = Rs. 640
9
⎛ 100 × x⎞
25. (C) Sum =
⎝ ⎠ ( )
100
⎜ x × x ⎟ = Rs. x 33. (B) ˙·˙ x =
y×r×t
100
26. (D) Let sum = Rs. x z×r×t
⇒ y =
Then amount = Rs. (8x5 ) ⇒
x y
100

y = z
S.I. = Rs. ( – x) = Rs. ( )
8x 3x

5 5 ∴ y2 = xz
⎛ 3x⎞ 34. (A) Due to the rise in the rate of interest, annual
⎜ 100 × 5 ⎟
∴ Required rate = ⎜⎜
⎝ x × 5 ⎟⎠
⎟ % = 12% (
income increases by Rs. 8 – 6 )
1
2
1
= Rs. 1 , when
2
the capital is Rs. 100.
27. (B) Let sum = Rs. x 100 × 2 × 4050
Thus, the required capital =
x×4×2 x×6×4 x×8×3 3
Then, + + = 1120
100 100 100 = Rs. 270000
⇒ 56x = 112000 1211
35. (A) ˙·˙ Income for 1 year = Rs.
112000 4
∴ x = = Rs. 2000 Since the rate of interest for Rs. 3500 is 1% higher
56
therefore, if we subtract 1% on Rs. 3500 from
28. (D) Let the rates be x% and y%. Then, 1211
500 × x × 2 500 × y × 2 Rs. the remainder will be 1 year’s interest on
~ = 2·5 4
100 100 (Rs. 4150 + Rs. 3500) at the lower rate of interest.
⇒ 10 (x ~ y) = 2·5 ˙·˙ Interest on (Rs. 4150 + Rs. 3500)
∴ Req. difference x ~ y = 0·25% 1211
= Rs. – 1% on Rs. 3500
29. (A) Let sum = P, then S.I. = P and Time = 20 years 4
7650 × x × 1 1211 3500 × 1
100 × P ⇒ = –
∴ Required rate = = 5% per annum 100 4 100
P × 20
1211 – 140 1071
30. (C) Let the annual instalment be Rs. x = =
4 4
Then,[x + (x ×1004 × 8)] + [x + (x ×1003 × 8)] ∴ x =
1071 × 10 7
765 × 4 2
= =3 %
1
2
x×2×8 x×1×8 1
+ [x + (
100 )] [
+ x+(
100 )]
+ x = 580 and (x + 1)% = 4 %
2
36. (B) We first find the rate per cent of Interest
33x 31x 29x 27x
⇒ + + + + x = 580 Interest on Rs. 5600 = Rs. 6678 – Rs. 5600
25 25 25 25 = Rs. 1078
⇒ (120 + 25) x = 580 × 25 100 × 1078
⇒ Rate % =
∴ x = 100 1
5600 × 3
3 2
31. (C) ˙·˙ S.I. for years 100 × 1078 × 2 1
2 = =5 %
= Rs. (1067·20 – 1012) 5600 × 7 2
9400 × 21 × 11
= Rs. 55·20 ⇒ Interest on Rs. 9400 = Rs.
100 × 4 × 2
5
2 (
⇒ S.I. for years = Rs. 55·20 × ×
2 5
3 2 ) = 92 = Rs.
10857
4
∴ Sum = Rs. (1012 – 92) = Rs. 920 = Rs. 2714·25
100 × 92 × 2 ∴ The required amount = Rs. 9400 + Rs. 2714·25
Hence, Rate = ( 920 × 5 ) = 4%
= Rs. 12114·25

Quantitative Aptitude Test | 121


37. (C) Let the sum be Rs. x Interest × 100
˙·˙ Principal =
S.I. = Rs. 600, r1 t 1 + r2 t 2 + r3 t 3 + r4 t 4
Time = 10 years 8560 × 100
=
600 × 100 (5 × 3 + 8 × 2 + 10 × 4 + 12 × 3)
∴ Rate = ( )
x × 10
%
=
856000
= Rs. 8000
= ( )
6000
x % per annum
107
45. (D) Let the principal be Rs. x
⎛ x × 5 × 6000⎞ ∴ Amount = Rs. 10080
S.I. for first 5 years = Rs. ⎜ ⎟
⎝ 100 × x ⎠ ∴ Simple interest = Amount – Principal
= Rs. 300 = 10080 – P
⎛ 3x × 5 × 6000⎞ Principal × Rate × Time
S.I. for last 5 years = Rs. ⎜ ⎟ ˙·˙ Simple interest =
100
⎝ 100 × x ⎠
P × 16 × 5
= Rs. 900 ⇒ 10080 – P =
100
Hence, total interest at the end of 10 years 4P
= Rs. 1200. ⇒ 10080 – P =
5
Interest × 100 ⇒ 50400 – 5P = 4P
38. (D) ˙·˙ Rate = %
Principal × Time ⇒ 50400 = 5P + 4P
2700 × 100 ⇒ 50400 = 9P
⇒ Rate = %
15000 × 3 50400
∴ P = = Rs. 5600
∴ Rate = 6% 9
39. (C) 46. (B) Let principal = Rs. 100
Interest × 100 ∴ Interest on Rs. 100 at 8% for 2 years
40. (D) Rate = %
Principal × Time 100 × 8 × 2
=
3600 × 100 100
= %
12000 × 5 = Rs. 16
= 6% ∴ Interest on Rs. 100 at 12% for 2 years
41. (B) r1 = 5%, t1 = 2 years 100 × 12 × 2
=
r2 = 8%, t2 = 3 years 100
r3 = 10%, t3 = 8 – (2 + 3) = Rs. 24
= 3 years Difference = 24 – 16
Interest × 100 = Rs. 8
˙·˙ Principal = When difference is Rs. 8 principal is Rs. 100. When
r1 t 1 + r2 t 2 + r3 t 3
1280 × 100 difference is Rs. 320
= 100
5 × 2 + 8 × 3 + 10 × 3 the principal = × 320
128000 128000 8
= = = Rs. 4000
10 + 24 + 30 64
= Rs. 2000 Principal × Rate × Time
47. (C) ˙·˙ S.I. =
100
42. (C) Let the principal = Rs. x
Principal × Rate × Time
⇒ Amount = Rs. 2x ∴ Amount – Principal =
100
⇒ Interest = 2x – x = Rs. x 4200 × Rate × 4
Interest × 100 ⇒ 5712 – 4200 =
˙·˙ Rate = 100
Principal × 20 ⇒ 1512 = 42 × 4 × Rate
x × 100 1512
= = 5% Rate = = 9%
x × 20 42 × 4
Interest × 100 On increasing 3%, new rate of interest
43. (C) Rate =
Principal × Time = 9 + 3 = 12%
800 × 100 Simple interest on new rate
= = 5%
8000 × 2 4200 × 12 × 4
=
44. (A) r1 = 5%, t1 = 3 years 100
r2 = 8%, t2 = 2 years = Rs. 2016
r3 = 10%, t3 = 4 years Amount = Principal + Interest
r4 = 12%, t4 = 12 – (3 + 2 + 4) = 4200 + 2016
= 3 years = Rs. 6216

Quantitative Aptitude Test | 122


Compound Interest
Important Points/Facts Q. 2. A certain sum on com- 3x
pound interest amount to Rs. 2809 = Rs.
1. Let Principal = Rs. P, 8
in 2 years and Rs. 2977·54 in 3
Time = n years and Rate = R% 217 x
years. Find the sum and rate per ˙·˙ x – 3 = 250
p.a. cent. 512 8
Case I. When interest is com- Solution : S.I. on Rs. 2809 for 1 ⇒ 217x – 192x = 250 × 512
pounded annually year 250 × 512
∴ x =
Amount = P 1 +( )
R n
100
= Rs. (2977·54 – 2809)
= Rs. 168·54
25
= Rs. 5120
Case II. When interest is com- ˙·˙ S.I. on Rs. 100 for 1 year Q. 4. If the compound interest
pounded half yearly
⎛ 1 R ⎞ 2n
⎜ 2 ⎟
= Rs.( 168·54
2809 )
× 100 %
on a certain sum of money for 3
years at 10% per annum be
Rs. 993. What would be the simple
Amount = P ⎜⎝ 1 + ⎟ = 6%
100⎠ interest ?
∴ Rate
Case III. When interest is com- Solution : Let principal = Rs. P,
pounded quarterly = 6% per annum then
Let the sum be Rs. 100
⎛ 1 R ⎞ 4n
⎜ 4 ⎟
Amount = P ⎜⎝ 1 + ⎟ Amount of Rs. 100 for 2 years (
˙·˙ P 1 +
10 3
100 )
– P = 993
100⎠
Case IV. When time is fraction [ (
= Rs. 100 × 1 + )]
6 2
100 ⇒(1110 ×1110 ×1110 – 1) P = 993
1
of a year, say 3 years, then
5 = Rs. ( )2809
25 ⇒ (
1331 – 1000
1000 ) P = 993
⎛ 1 R⎞ 2809
R 3 ⎜⎜ 5 ⎟⎟ ˙·˙ 100 : = x : 2809 993 × 1000
Amount = P 1 +( 100) × ⎝1 +
100⎠
25
100 × 2809 × 25
∴ P =
331
Case V. When rate of interest is ∴ x = = 3000
2809
R 1 %, R2% and R3 % for 1st year, 2nd = 2500 ∴ Simple interest
year and 3rd year respectively, then 3000 × 3 × 10
( )
Amount = P 1 + 1
R
Hence, the sum = Rs. 2500
Q. 3. The difference between
= Rs. (
100 )
100
compound interest and the simple = Rs. 900
( R
100 ) ( )
× 1+ 2 × 1+ 3
R
100 interest on a certain sum at 12 %
1
2
Q. 5. Find compound interest
on Rs. 51200 at 15% per annum for
2. Present worth of a sum of per annum for 3 years is Rs. 250. 9 months compounded quarterly.
Rs. x due n years hence is given by— Find the sum.
Solution : Time = 9 months
x Solution : Let the sum be Rs. x = 3 quarters Rate = 15% per annum
Present worth =
( 1+
R n
100 ) Then, amount
= ( )
15
4
% quarterly
[ ( )]
25 3
= Rs. x × 1 +
2 × 100 ∴ Amount
Examples
( ) [ ( )]
9 9 9 15 3
Q. 1. Find compound interest = Rs. × × ×x = Rs. 51200 × 1 +
on Rs. 50000 at 16% per annum 8 8 8 4 × 100
for 2 years, compounded annually.
Solution : Amount after 2 years
= Rs. ( ) 729
512
x ( 83 83 83
= Rs. 51200 × × ×
80 80 80 )
∴ Compound interest
[
= Rs. 50000 × 1 + ( 16 2
)] = Rs. 57178·70
100
= Rs. [ ] 729
512
x–x ∴ Compound interest

(
= Rs. 50000 × ×
29 29
) = Rs. (57178·70 – 51200)

= Rs. 67280
25 25
= Rs. ( ) 217
512
x = Rs. 5978·70
Q. 6. Find compound interest
∴ Compound interest Simple interest on Rs. 100000 at 20% per annum
= Rs. (67280 – 50000)
= Rs. 17280 [
= Rs. x ×
25 3
×
2 100 ] for 2 years 3 months compounded
annually.

Quantitative Aptitude Test | 123


1 07. The compound interest on 15. The compound interest on a sum
Solution : Here time = 2 years
4 Rs. 30000 at 7% per annum for a for 2 years is Rs. 832 and the
˙·˙ Amount certain time is Rs. 4347. The time simple interest on the same sum
is— for the same period is Rs. 800.
⎡⎢ (
100000 × 1 +
20 2
100 ) ⎤⎥ (A) 2 years (B) 2 years
1 The difference between the com-
pound and simple interest for 3
= Rs. ⎢ ⎛ 1 × 20⎞ ⎥ 2
years will be—
⎢⎣ ⎜ 4
× ⎝⎜ 1 +

⎟ ⎥⎦ (C) 3 years (D) 4 years
(A) Rs. 48
100 ⎠ 08. Rs. 800 at 5% per annum com- (B) Rs. 66·56
pound interest will amount to
(
= Rs. 100000 × × ×
6 6 21
5 5 20 ) Rs. 882 in—
(A) 1 year (B) 2 years
(C) Rs. 98·56
(D) None of these
= Rs. 151200 16. The difference in compound
(C) 3 years (D) 4 years
∴ Compound interest interest and simple interest for 2
09. Simple interest on a sum at 4%
= Rs. (151200 – 100000) years on a sum of money is
per annum for 2 years is Rs. 80. Rs. 160. If the simple interest for
= Rs. 51200 The compound interest on the 2 years be Rs. 2880, the rate
same sum for the same period
Exercise is—
per cent is—
5 1
01. The difference between the com- (A) Rs. 81·60 (A) 5 % (B) 12 %
pound interest and the simple 9 2
(B) Rs. 160
interest on a certain sum at 5% 1
(C) Rs. 1081·60 (C) 11 % (D) 9%
per annum for 2 years is Rs. 1·50. 9
The sum is— (D) None of these
17. The value of a machine depre-
(A) Rs. 600 (B) Rs. 500 10. The difference of compound ciates every year at the rate of
(C) Rs. 400 (D) Rs. 300 interest on Rs. 800 for 1 year at 10% on its value at the beginning
20% per annum when compound of that year. If the present value
02. If the compound interest on a
half yearly and quarterly is— of the machine is Rs. 729, its
certain sum for 2 years at 12·5%
(A) Nil (B) Rs. 2·50 worth 3 years ago was—
per annum is Rs. 170, the simple
interest is— (C) Rs. 4·40 (D) Rs. 6·60 (A) Rs. 947·10
(A) Rs. 150 (B) Rs. 152·50 (B) Rs. 800
11. The difference between simple
(C) Rs. 160 (D) Rs. 162·50 (C) Rs. 1000
interest and the compound interest
on Rs. 600 for 1 year at 10% per (D) Rs. 750·87
03. The simple interest on a certain
sum for 2 years at 10% per annum, reckoned half yearly is— 1
18. A tree increases annually by th
annum is Rs. 90. The correspond- (A) Nil (B) Rs. 6·60 8
ing compound interest is— (C) Rs. 4·40 (D) Rs. 1·50 of its height. By how much will
(A) Rs. 99 (B) Rs. 95·60 it increase after 2 years, if it
12. The compound interest of Rs. stands today 64 cm high ?
(C) Rs. 94·50 (D) Rs. 108
1 (A) 72 cm (B) 74 cm
04. What is the principal amount 20480 at 6 % per annum for 2
4 (C) 75 cm (D) 81 cm
which earns Rs. 132 as com- years 73 days is—
pound interest for the second 19. The least number of complete
(A) Rs. 3000 (B) Rs. 3131
year at 10% per annum ? years in which a sum of money
(C) Rs. 2929 (D) Rs. 3636 put out at 20% compound interest
(A) Rs. 1000
(B) Rs. 1200 13. The compound interest on Rs. will be more than doubled is—
(C) Rs. 1320 1 (A) 3 years (B) 4 years
2800 for 1 years at 10% per
(D) None of these 2 (C) 5 years (D) 6 years
annum is—
05. At what rate of compound 20. A man borrows Rs. 4000 from a
(A) Rs. 441·35 1
interest per annum will a sum of bank at 7 % compound interest.
Rs. 1200 become Rs. 1348·32 in (B) Rs. 436·75 2
2 years ? (C) Rs. 434 At the end of every he pays
Rs. 1500 as part repayment of
(A) 7% (B) 7·5% (D) Rs. 420 loan and interest. How much
(C) 6% (D) 6·5% does he still owe to the bank
14. If Rs. 7500 are borrowed at com-
06. A sum amounts to Rs. 1352 in 2 pound interest at the rate of 4% after three such instalments ?
years at 4% compound interest. per annum, then after 2 years the (A) Rs. 123·25
The sum is— amount to be paid is— (B) Rs. 125
(A) Rs. 1300 (B) Rs. 1250 (A) Rs. 8082 (B) Rs. 7800 (C) Rs. 400
(C) Rs. 1260 (D) Rs. 1200 (C) Rs. 8100 (D) Rs. 8112 (D) Rs. 469·18

Quantitative Aptitude Test | 124


21. A sum of Rs. 550 was taken as a 29. The compound interest on a (C) 8th year
loan. This is to be repaid in two certain sum of money for 2 years (D) Data inadequate
equal annual instalments. If the at 10% per annum is Rs. 420. The
rate of interest be 20% com- simple interest on the same sum 36. What will be the compound
pounded annually then the value at the same rate and for the same interest of Rs. 8000 at 10% p.a.
of each instalment is— time will be— 1
for 1 years. When compound
2
(A) Rs. 421 (B) Rs. 396 (A) Rs. 350 (B) Rs. 375
interest is payable half yearly ?
(C) Rs. 360 (D) Rs. 350 (C) Rs. 380 (D) Rs. 400
(A) Rs. 1261 (B) Rs. 1385
22. A loan was repaid in two annual 30. A sum of money becomes (C) Rs. 1480 (D) Rs. 1255
instalments of Rs. 121 each. If Rs. 6690 after three years and
the rate of interest be 10% per Rs. 10035 after six years on 37. If the interest of a certain sum for
annum. Compounded annually, compound interest. The sum is— the first year at 8% p.a. compound
the sum borrowed was— interest is Rs. 48. What will be
(A) Rs. 4400 (B) Rs. 4445
the interest for the second year ?
(A) Rs. 200 (C) Rs. 4460 (D) Rs. 4520
(A) Rs. 52·55 (B) Rs. 51·84
(B) Rs. 210 31. Rs. 1600 at 10% per annum (C) Rs. 58·60 (D) Rs. 53·04
(C) Rs. 217·80 compound interest compounded
(D) Rs. 280 half yearly amount to Rs. 38. If the simple interest on certain
1944·81 in— sum of money be Rs. 40 for 2
23. A sum amounts to Rs. 2916 in 2 years and the compound interest
years and to Rs. 3149·29 in 3 (A) 2 years (B) 3 years
on the same sum at the same rate
years at compound interest. The 1 1 and for the same time be Rs. 45.
(C) 1 years (D) 2 years
sum is— 2 2 What is the principal ?
(A) Rs. 1500 (B) Rs. 2000 32. The difference between simple (A) Rs. 70 (B) Rs. 90
(C) Rs. 2500 (D) Rs. 3000 interest and compound interest (C) Rs. 85 (D) Rs. 80
24. A sum of money amounts to on a sum for 2 years at 8% when
the interest is compounded 39. Neeraj saves Rs. 400 at the end
Rs. 10648 in 3 years and of each year and lends this saving
Rs. 9680 in 2 years. The rate of annually is Rs. 16. If the interest
were compounded half yearly at 5% p.a. compound interest.
interest is— How much will it worth at the
the difference in two interests
(A) 5% (B) 10% end of 3 years ?
would be nearly—
(C) 15% (D) 20% (A) Rs. 1312·50
(A) Rs. 16 (B) Rs. 16·80
25. A sum of Rs. 12000 deposited at (B) Rs. 1284
(C) Rs. 21·85 (D) Rs. 24·64
compound interest becomes (C) Rs. 1315
doubles after 5 years. After 20 33. Divide Rs. 3903 between A and (D) Rs. 1324·05
years it will become— B, so that A’s share at the end of
7 years may equal to B’s share at 40. If the compound interest of a
(A) 120000 (B) 192000 certain sum of money for 2 years
the end of 9 years, compound
(C) 124000 (D) 96000 interest being at 4 per cent. at 5% p.a. is Rs. 328. What will
be the simple interest on the same
26. A sum of money placed at com- (A) Rs. 2028, Rs. 1875
sum of money for the same time
pound interest doubles itself in 5 (B) Rs. 2018, Rs. 1885 and at the same rate ?
years. It will amount to eight (C) Rs. 2008, Rs. 1895 (A) Rs. 320 (B) Rs. 322
times itself in— (D) Rs. 2038, Rs. 1865 (C) Rs. 325 (D) Rs. 326
(A) 15 years (B) 20 years 34. A sum of money put out at
(C) 12 years (D) 10 years 41. What will be the approximate
compound interest amounts in 2 compound interest on Rs. 10105
27. A sum is invested at compound years to Rs. 578·40 and in 3 at 10% p.a. for 3 years ?
interest payable annually. The years to Rs. 614·55. Find the rate
(A) Rs. 4500 (B) Rs. 3000
interest in two successive years of interest.
(C) Rs. 3300 (D) Rs. 3600
was Rs. 500 and Rs. 540. The 1
(A) 6% (B) 6 %
sum is— 4 42. If the compound interests of a
1 3 certain sum of money for two
(A) Rs. 3750 (B) Rs. 5000 (C) 6 % (D) 6 %
2 4 successive years be Rs. 225 and
(C) Rs. 5600 (D) Rs. 6250 Rs. 238·50. What is the rate of
35. Find what is that first year in interest per annum ?
28. The difference between simple which a sum of money will
interest and compound interest at (A) 5% (B) 6%
become more than double in
the same rate for Rs. 5000 for 2 amount if put out at compound 1
years is Rs. 72. The rate of (C) 7 % (D) 10%
interest at the rate of 10% per 2
interest is— annum. 43. The cash price of a refrigerator is
(A) 10% (B) 12% (A) 6th year Rs. 7044. A customer paid
(C) 6% (D) 8% (B) 7th year Rs. 2000 in cash and promised to

Quantitative Aptitude Test | 125


pay the remaining money in 3 47. Find the least number of com- 51. A person invested a certain
yearly equal instalments at the plete years in which a sum of amount at simple interest at the
rate of 5% per annum compound money put out at 20 per cent rate of 6 per cent per annum
interest. What is the value of each compound interest will be more earning Rs. 900 as an interest at
instalment ? than doubled. the end of three years. Had the
(A) Rs. 1865 (B) Rs. 1868·28 (A) 2 years interest been compounded every
(C) Rs. 1752 (D) Rs. 1852·20 year. How much more interest
(B) 3 years would he have earned on the
44. A moneylender lends Rs. 2000 (C) 4 years same amount with the same
for 6 months at 20% per annum
(D) Data inadequate interest after three years ?
whereas the interest is com-
(A) Rs. 38·13 (B) Rs. 25·33
pounded quarterly. After the 48. In what time will Rs. 390625 (C) Rs. 55·08 (D) Rs. 35·30
given period he will get the amount to Rs. 456976 at 4 per
amount of— cent compound interest ? 52. The difference between the
(A) Rs. 2205 (B) Rs. 2200 simple and the compound interest
(A) 2 years (B) 4 years compounded every six months at
(C) Rs. 2160 (D) Rs. 2040
(C) 3 years (D) 5 years the rate of 10 per cent per annum
45. If a sum of money placed at com- at the end of two years is
pound interest becomes 3 times 49. Find the effective annual rate of 4
Rs. 124·05. What is the sum ?
of itself in 3 year. In how many per cent per annum compound
interest paid quarterly. (A) Rs. 10000
years will it be 9 times at the
same rate of interest ? (B) Rs. 6000
(A) 4·0604% (B) 4·604%
(A) 6 years (B) 9 years (C) Rs. 12000
(C) 5·0605% (D) 5·605% (D) Rs. 8000
(C) 12 years (D) 5 years
46. In what time will Rs. 6250 50. Find the effective annual rate of 53. Vijay obtains a loan of 64000
amount to Rs. 6632·55 at 4% 5 per cent per annum compound against his fixed deposits. If the
compound interest payable half interest paid half yearly. rate of interest be 2·5 paise per
yearly ? (A) 1·025% rupee per annum. Calculate the
compound interest payable after
3 (B) 6·0625%
(A) 3 years (B) years 3 years.
2 (C) 5·062% (A) Rs. 4921 (B) Rs. 5020
5
(C) 1 year (D) years (D) None of these (C) Rs. 4821 (D) Rs. 4920
2

Answers with Hints


01. (A) Let the sum be Rs. 100 then,
100 × 5 × 2

S.I.
C.I.
= (14 ×6417) = 1617
S.I. = Rs. ( 100 )
= Rs. 10
∴ S.I. =
16
× C.I.
17
C.I. = Rs. [{ (
100 × 1 +
5 2
100 ) } – 100] = Rs. (
16
× 170 )
17
41
= Rs. = Rs. 160
4
100 × 90
∴ Difference between C.I. and S.I. 03. (C) Sum = Rs. (2 × 10 )
= Rs. 450

= Rs. ( 41
4
– 10 ) [
C.I. = Rs. 450 × 1 + ( 10 2
100 )
– 450 ]
= Re. 0·25 = Rs. 94·50
⇒ 0·25 : 150 : : 100 : x 04. (B) Let x be the principal at the end of first year.

∴ x = (1·500·25× 100 ) = Rs. 600 Then


x × 10 × 1
100
= 132
⇒ x = 1320
02. (C)˙·˙ S.I. on Re. 1 = Rs. (1 × 2 × ×
2 100)
25 1
Let y be the original principal
1 y × 10 × 1
= Re. Then, y + = 1320
4 100
⇒ y = 1200
[1 ×(1 + 2 ×25100) – 1]
2
⇒ C.I. on Re. 1 = Rs. 05. (C) Let the rate be x% per annum. Then,

= Re.
17
64
(
1200 × 1 +
x 2
100 ) = 1348·32

Quantitative Aptitude Test | 126


×5×2
(1 + 100x ) (600100 ) = Rs. 60
2 1348·32
⇒ = 11. (D) S.I. = Rs.
1200

C.I. = Rs. [600 × (1 +


100)
– 600]
= 1·1236 = (1·06)2 5 2

⇒ (1+
x
100 )= 1·06
= Rs. 61·50
x ∴ Required Difference
⇒ = 0·06
100 = Rs. (61·50 – 60)
∴ x = 6% = Rs. 1·50
⎡ 20480 × 1 + 25 2⎤
06. (B) Let the sum be P. Then,

12. (C) C.I. = Rs. ⎢⎢
(4 × 100 ⎥⎥ )
(1 + 1004 ) 2
1352 = P ⎥
26 26 ⎣(
⎢ 1 + 1 × 25
5 4 × 100) – 20480 ⎥

⇒ 1352 = P × ×
= Rs. [(20480 × × × ) – 20480 ]
25 25 17 17 81
1352 × 25 × 25 16 16 80
⇒ P = = 1250
26 × 26
= 20480 [ ]
23409 – 20480
∴ Principal = Rs. 1250 20480

07. (
(A) ˙·˙ 30000 × 1 + ) 7 t
100
= 30000 + 4347
= Rs. 2929
13. (C) ˙·˙ Amount
⇒ ( )
107 t
100
=
34347
30000
= Rs. [2800 ×(1 + 100
10
) (1 + 1005 )]
=
11449
10000
=
107 2
100 ( ) = Rs. [2800 × × ]
11 21
10 20
∴ Time = 2 years = Rs. 3234
08. (B) Let time be t years ∴ Required C. I.

˙·˙ (
882 = 800 1 +
5 t
100 ) = Rs. (3234 – 2800)
= Rs. 434

[7500 (1 + 1004 ) ] 2
⇒ ( )
882
800
=
21 t
20
14. (D) Amount = Rs.

( ) ( ) = Rs. [7500 × × ]
21 2 21 t 26 26
⇒ =
20 20 25 25
⇒ t = 2 = Rs. 8112
∴ Time = 2 years 15. (C) S.I. for first year = Rs. 400
100 × 80 S.I. on Rs. 400 for 1 year = Rs. 32
09. (A) Principal = Rs. ( 4×2 )
= Rs. 1000
∴ Rate =
100 × 32
(400 × 1
= 8%)
∴ C.I. = Rs. [{ 1000 × 1 + ( 4 2
100 )– 1000 }] Hence, the difference for 3rd year is S.I. on Rs. 832

10.
= Rs. 81·60
(C) C.I. when reckoned half yearly
= Rs. 832 × ( 8
100 )
= Rs. 66·56
[
= Rs. 800 × 1 + ( 10 2
100 )
– 800 ] ∴ Total difference = Rs. (32 + 66·56)
= Rs. 98·56
= Rs. 168 16. (C) ˙·˙ S.I. for 1 year = Rs. 1440
C. I. when reckoned quarterly ⇒ S.I. on Rs. 1440 for 1 year = Rs. 160
100 × 160
= Rs. 800 [(
1+
5 4
100
–1 ) ] Hence, ∴ Rate per cent = (
1440 × 1
% )
100
= 800 [
194481 – 160000
160000 ] =
9
%
1
34481 = 11 %
= 9
200
= Rs. 172·40 (
17. (C) ˙·˙ P 1 –
100 )
10 3
= 729
∴ Required Difference 729 × 10 × 10 × 10
= Rs. (172·40 – 168) ∴ P = Rs. ( 9×9×9 )
= Rs. 4·40 = Rs. 1000

Quantitative Aptitude Test | 127


2916 × 25 × 25
18. (D) Increase% = (18 × 100)% = 12·5% ∴ P =
27 × 27
Height after 2 years = 64 × (1 +
2 × 100)
25 2 = Rs. 2500
24. (B) Let P be the principal and R% per annum be the
= (64 × × )
9 9 rate. Then,
8 8
= 81 cm (
P 1+ )
R 3
100
= 10648 …(i)

19. (B) ˙·˙ (


x 1+ )
20 n
100
> 2x and P( 1 + )
R 2
100
= 9680 …(ii)

⇒ () 6 n
5
> 2 On dividing (i) by (ii), we have

Now, ( 6 6 6 6
× × × ) > 2
⇒ ( 1+ )
R
100
=
10648
9680
5 5 5 5 R 968 1
1296 ⇒ = =
⇒ > 2 100 9680 10
615 1
∴ n = 4 years ∴ R = × 100 = 10%
10

[{ ( )}
15 3
20. (A) Balance = Rs. 4000 × 1 +
2 × 100 25. (B) ˙·˙ (
2P = P 1 + )
r 5
100
– { (
1500 × 1 +
15
)
2 × 100
2
+ 1500 ⇒ ( 1+ )r
100
5
= 2

(
× 1+
15
2 × 100 ) }]
+ 1500 ∴ ( 1+ )
r 20
100
= 24 = 16
= Rs. 123·25
21. (C) Let the value of each instalment be Rs. x Thus, (
P 1+ )
r 20
100
= 16P

Then, ⎪⎧ x
+
x ⎪⎫ = 550 = Rs. (12000 × 16)
⎨⎪ 20 2⎬
⎩( 1 + ) (
20
100
1 + )
100 ⎭
⎪ = Rs. 192000
5x 25x 26. (A) Let the principal be P and rate be r%. Then
⇒ + = 550


6 36
55x
= 550
(
2P = P 1 + )
r 5
100


36
x = 360 ⇒ ( 1+
100 )
r 5
= 2
22. (B) Principal = (P.W. of Rs. 121 due 1 year hence) Let it be 8 times in t years, then
+ (P.W. of Rs. 121 due 2 years hence)
= Rs. ⎢⎢
⎡ 121
+
121 ⎤

˙·˙ (
8P = P 1 + )
r t
100
10 2 ⎥
(
⎢⎣ 1 + 100
10
) ( 1+ )
100 ⎦
⎥ ⇒ ( 1+
r
100 ) t
= 8 ⇒ (2)3 ⇒ 1 +( )
r 15
100
= Rs. 210
∴ t = 15 years.
23. (C) Let P be the principal and R% per annum be rate
27. (D) S.I. on Rs. 500 for 1 year = 540 – 500
( )
R 3
Then, P 1+ = 3149·28 …(i)
100 = Rs. 40
100 × 40
and (
P 1+ )
R 2
100
= 2916 …(ii) ∴ Rate = ( 500 × 1) = 8%
On dividing (i) and (ii) we get 100 × 500
∴ ( 1+
R
) =
3149·28 and Sum = Rs. ( 8×1 )
100 2916 = Rs. 6250
R 233·28
⇒ = 28. (B)
100 2916
5000 × 2 × R
⇒ R =
233·28
2916
× 100 = 8% [
˙·˙ 5000 × 1 +( ) ] R 2
100
– 5000 –
100
= 72

Now, (
P 1+ )
8 2
100
= 2916 ⇒ 5000 [( ) ]1+
R 2
100
–1–
R
50
= 72

27 27 R2 2R R 72
⇒ P× × = 2916 ⇒ 1+ + –1– =
25 25 10000 100 50 5000

Quantitative Aptitude Test | 128


(5000 × 10000) = 144 (1 + 1004 )
72 7
⇒ R2 = 33. (A) We have (A’s present share)


= (B’s present share) (1 +
100)
R = 12% 4 9

29. (D) Let the principal be P. Then,

( 10 2
) = (1 +
100)
A’s present share 4 2
˙·˙ P 1 + – P = 420 ∴
100 B’s present share

( ) = ( ) =
121 – 100 26 2 676
⇒ P = 420
100 25 625
⇒ P = Rs. 2000 Dividing Rs. 3903 in the ratio of 676 : 625
2000 × 2 × 10
∴ Required S.I. = Rs. ( 100 ) ∴ A’s present share =
676
(676 + 625)
of Rs. 3903

= Rs. 400 = Rs. 2028


30. (C) Let the principal be P, then B’s present share = Rs. 3903 – Rs. 2028

(
P 1+
100)
R 3
= 6690 …(i)
= Rs. 1875
34. (B) Clearly the difference between Rs. 578·40 and
Rs. 614·55 is the interest on Rs. 578·40 for 1 year.
(
and P 1 +
100)
R 6
= 10,035 …(ii) ∴Interest on Rs. 578·40 for 1 year
Now, Dividing (ii) by (i), we get = Rs. 614·55 – Rs. 578·40
= Rs. 36·15
⇒ ( 1+
100)
R 3
=
10035 3
6690 2
= ∴ Interest on Rs. 100 for 1 year
3 100
⇒ P × = 6690 = Rs. 36·15 ×
2 578·40

∴ (
P = 6690 × )
2
3
= Rs. 4460 = Rs.
3615 100
57840
×
1

( )
5 T = 6·25
31. (A) 1600 1 + = 1944·81
100 1
= Rs. 6
⇒ ( )
21 T
20
=
1944·81
1600·00 1
4

∴ The required rate is 6 per cent.


194481 4
=
160000
= ( )
441 2 35. (C) Here, P ( 1 + )
10 t
100
> 2P
400

= ( )
21 4 ⇒ ( )11 t
10
> 2
20
∴ T = 4 (Half years) when t=8→ () 11 8
10
= 2·14358
⇒ Required time = 2 years
32. (D) For first year, S.I. = C.I. t=7→ () 11 7
10
= 1·9487
Thus, Rs. 16 is the S.I. on S.I. for 1 year, which at 8%
11 × 11 × 11 × 11 × 11 × 11 × 11 × 11
is thus Rs. 200, i.e., S.I. on the principal for 1 year is By trial, >2
10 × 10 × 10 × 10 × 10 × 10 × 10 × 10
Rs. 200
100 × 200 Hence, the first year in which sum of money will
∴ Principal = Rs. ( 8×1 ) become more than double in amount is 8th year.
36. (A) Yearly interest = 10%
= Rs. 2500
Amount for 2 years, compounded half yearly Half yearly interest = 5%
1
[ (
= Rs. 2500 × 1 +
100)]
4 4 Time = 1 year
2
= Rs. 2924·64 3
= × 2 half yearly
∴ C.I. = Rs. 424·64 2
2500 × 8 × 2 = 3 half yearly
Also, S.I. = Rs. ( 100 ) Amount = 8000 1 + ( ) 5 3
= Rs. 400 100
Hence [(C.I.) – (S.I.)] = Rs. (424·64 – 400)
∴ Required difference = Rs. 24·64
= 8000 × () 21 3
20

Quantitative Aptitude Test | 129


21 21 21 41
= 8000 × × × ⇒ 328 = Principal
20 20 20 400
= Rs. 9261 328 × 400
⇒ Principal =
Compound interest = Amount – Principal 41
= 9261 – 8000 = Rs. 3200
= Rs. 1261 3200 × 5 × 2
∴ Simple interest =
37. (B) S.I. or C.I. for first year are always equal 100
S.I. × 100 = Rs. 320
Principal =
Rate × Time
{(1 + 100r ) – 1 }
t
41. (C)Compound interest = P
48 × 100
= = Rs. 600
8×1
= 10105 { (1 +
100)
– 1}
10 3

Interest for second year


= 10105 { ( ) – 1}
= Amount of second year 11 3

– Amount of the first year 10

( ) (
= 600 1 +
8 2
100
– 600 1 +
8
)
100
= 10105 {
1000 }
1331
–1

= 600 ( ) ()
27 2
25
– 600
27
25
= 10105 (
1000)
331

= 600 ( ){ }
27
25
27
25
–1
= 3344·755
~
– Rs. 3300 (app.)
= 600 ( )( )
27
25
2
25 42. (B) Rate =
238·50 – 225
225
× 100%
= Rs. 51·84 = 6%
38. (D) ˙.˙ S.I. for 2 years = Rs. 40 43. (D) Remaining money = 7044 – 2000
S.I. for 1 year = Rs. 20 = Rs. 5044
C.I. for 2 years – S.I. for 2 years If each instalment is of Rs. x
Rate = × 100
S.I. for 1 year
When the amount is Rs. x at the end of first second
(45 – 40)
= × 100 = 25% and third year at the rate of 5% then principal will
20 be—
S.I. × 100
∴ Principal = x , x x
Rate × Time and
( )( ) ( )
5 5 2 5 3
40 × 100 1+ 1+ 1+
= 100 100 100
25 × 2
= Rs. 80 x x x
˙·˙ + + = 5044
39. (D) Amount after 3 years
(1+ ) ( ) ( )
5
100
1+
5 2
100
1+
5 3
100
( ) ( ) ( )
= 400 1 +
5 3
100
+ 400 1 +
5 2
100
+ 400 1 +
5
100 ⇒ x( ) ( ) ()
20
+x
20 2
+x
20 3
= 5044
21 21 21
( ) {( ) ( ) }
= 400 1 +
5
1+
5 2
+ 1+
5
+1
100 100 100
⇒ x ( ){ ( ) ( )}
20
21
1+
20
21
+
20 2
21
= 5044
= 400 ( ) {( ) ( ) }
105 105 2
+
105
+1
100 100 100
⇒ ( ){
x
20
} 1+
20 400
+ = 5044
= 400 ( ) {( )
21
20 } 21 2 21
20
+ +1
20
21 21 441

⇒ ( ){
x
20
} 441 + 420 + 400
= 5044
= 420 ( )
441 21
400 20
+ +1
21 441

= 420 ( )
441 + 420 + 400 ⇒ ( )( ) x
20
21
1261
441
= 5044
400 5044 × 21 × 441
∴ x =
= 420 ( )
1261
400
20 × 1261
= Rs. 1852·20
= Rs. 1324·05
44. (A) For the quarterly interest,
40. (A) ˙·˙ 328 = Principal {( ) } 1+
5 2
100
–1
Time =
6
year × 4
12
⇒ 328 = Principal ( ) 441
400
–1 = 2 quarterly

Quantitative Aptitude Test | 130


Rate = 20 ÷ 4 49. (A) Effective annual rate
= 5% 4
(˙.˙ On quarterly interest time is multiplied by 4 and ⎛ 4 ⎞
⎜ 4 ⎟⎟
rate is divided by 4) = 100 ⎜⎝ 1 + – 100
Hence, required amount 100⎠

(
= Principal 1 + )
Rate Time
100 = (101
100)
4
× 100 – 100

( )
= 2000 1 +
5 2
100
= 104·0604 – 100
= 4·0604%
= 2000 ( ) 21 2
20
50. (C) The amount of Rs. 100 in one year at compound
interest at 5% per annum payable half yearly.
= Rs. 2205 2
45. (A) Let the principal be P then Amount be 3P ⎛ 5 ⎞
⎜ 2 ⎟⎟
= Rs. 100 ⎜⎝ 1 +
˙·˙ (
3P = P 1 + )
r 3
100 100⎠

⇒ 3 = 1+( ) r 3
100
= Rs. 100 (102·5
100 )
2

On squaring on both the sides, = Rs. 100 (1·025)2


⇒ (3) 2 = {( ) } 1+
r 3 2
100
= Rs. 105·0625
Thus, the nominal rate of 5% payable half yearly has
∴ 9 = 1+( ) r 6
100
the same effect as the rate of 5·0625 per cent would
have, if payable yearly.
Hence, it will become 9 times in 6 years. Hence 5·0625 per cent is called the effective annual
⎡⎢ 4 ⎤⎥ 2t rate 5% per annum payable half yearly.
⎢ 2 ⎥ 51. (C) Certain sum for the person
46. (B) ˙·˙ 6250 ⎢⎣ 1 + ⎥ = 6632·55
100⎦ =
900 × 100
= Rs. 5000
6×3
⇒ ( 1+
100)
2 2t
=
663255
625000 ∴ Interest on Rs. 5000 by C.I.

=
132651
125000 ( )
=
51 3
50
= 5000 1 +( 6 3
100 ) – 5000

= Rs. 955·08
⇒ ( ) ( )
51 2t
50
=
51 3
50 ∴ More interest = Rs. (955·08 – 900)
⇒ 2t = 3 = Rs. 55·08
3 52. (D) Let the sum be Rs. x, then
∴ t = years
2 x × 10 × 2
47. (C) Here (
P 1+ )
20 t
100
> 2P [(
˙·˙ x 1 +
100) ] [
5 4
–x –
100 ] = 124·05

Solving the above equation, we get x = Rs. 8000.


⇒ () 6 t
5
> 2 53. (A) P = Rs. 64000
r = 2·5 paise per rupee per annum (given)
·.· () 6 3
5
= 1·728
= 0·025 rupee per rupee per annum
and () 6 4
5
= 2·0736 = 0·025 × 100 rupee per hundred rupee per
annum
6 6 6 6 = 0·025 × 100 per cent per annum
By trial × × × > 2
5 5 5 5
= 2·5 per cent per annum
∴ The required time is 4 years.
t = 3 years
48. (B) ˙.˙ (
P 1+ )
r t
= A
100
C.I. = 64000 [( 1+
2·5 3
) ]
–1
⇒ (
390625 1 + )
4 t
100
= 456976
100
= 64000 [(1·025)3 – 1]
⇒ ( 1+ )
4 t
100 ( )
=
456976
390625
=
26 4
25
= 64000 [1·0769 – 1]
= 64000 × 0·0769
( ) ( )
26 t 26 4
⇒ = = 4921·6
25 25
⇒ t = 4 = Rs. 4921
∴ The required time is 4 years. ∴ The compound interest payable is Rs. 4921.

Quantitative Aptitude Test | 131


Problems Based on Ages
Essential Points/Facts ⇒ 27x + 54 = 35x + 30 Short Method
1. If the present age of A is x ⇒ 35x – 27x = 54 – 30 6x + 6 + 4 11
year, then the age of A, n year ago ˙·˙ =
⇒ 8x = 24 5x + 6 + 4 10
was (x – n) year and the age of A Here, a:b = 6:5
after n year will be (x + n) year. ∴ x = 3
c:d = 11 : 10
2. The present age of A and B So that numbers are
are x year and y year respectively. If and p = 6 + 4 = 10
3x = 3 × 3 = 9
the age of A is 3 times the age of B. 10 (11 – 10)
x =
Then the linear equation will be as and 7x = 7 × 3 = 21 60 – 55
follows— 10
Short Method = =2
x = 3y 5
3x + 6 5 ∴ Present age of Suresh
or x – 3y = 0 Here, =
7x + 6 9
3. Three consecutive odd or even = 5x + 6
6 (5 – 9)
numbers are x, x + 2 and x + 4 or x = = 5×2+6
27 – 35
(x – 2), x. (x + 2) where x is an odd or = 16 year
even number. 6×4
= =3 Q. 4. The difference between a
8
number of two digits and the
Examples i.e., numbers are 9 and 21.
number obtained by interchanging
Q. 1. Arun is as older than its digits is 18. How much is the
Anil as he is younger than Bhushan. Q. 3. 6 years ago the ratio in difference between both digits of
If the sum of the age of Anil and the ages of Kamal and Suresh was that number ?
Bhushan is 58 year then what is the 6 : 5 and after 4 year the ratio in
their ages will be 11 : 10. Find the Solution : Let the number be
age of Arun ?
present age of Suresh. = (10a + b)
Solution : From the equation,
Solution : Let 6 year ago, the On interchanging the digits,
Arun – Anil = Bhushan – Arun ages of Kamal and Suresh were 6x Number = (10b + a)
⇒ Arun + Arun and 5x respectively. ∴ From the question,
= Anil + Bhushan ∴ After 6 year, the ratio of the (10a + b) – (10b + a) = 18
Anil + Bhushan ages of Kamal and Suresh
⇒ Arun = 9a – 9b = 18
2 = (6x + 6) : (5x + 6) 18
a–b = =2
58 Again, after 4 year, the ratio of 9
Hence, Age of Arun = the ages of Kamal and Suresh
2 i.e., difference between both
= 29 year = (6x + 6 + 4) : (5x + 6 + 4) digits = 2

Short Method Now, from the question, Short Method


6x + 6 + 4 11 x
In such questions the given sum ˙·˙ = Required difference =
total is to be halved 5x + 6 + 4 10 9
58 18
Age of Arun = 6x + 10 11 = =2
2 ⇒ = 9
5x + 10 10
= 29 year Q. 5. In a number of two digits,
⇒ 60x + 100 = 55x + 110
both digits are interchanged. If the
Q. 2. The ratio of two numbers ⇒ 60x – 55x = 110 – 100 sum of the number and the number
is 3 : 7. If b is added to each obtained by interchanging its digits
number then the ratio becomes ⇒ 5x = 10 is 44, then what is the sum of its two
5 : 9. What are these numbers ? x = 2 digits ?
Solution : Let the numbers are Solution : Let the number is
∴ The present age of Suresh
3x and 7x then, = 10a + b
= 5x + 6
From the question, ∴ Number obtained by inter-
3x + 6 = 5×2+6 chaning its both digits
5
˙·˙ =
7x + 6 9 = 16 year = 10b + a

Quantitative Aptitude Test | 132


∴ From the question, present age of the elder person 14. The sum of the ages of a mother
10a + b + 10b + a = 44 is— and a daughter is 50 years. Also,
(A) 25 years (B) 35 years 5 years ago, the mother’s age
11a + 11b = 44 was 7 times of the age of her
(C) 45 years (D) 55 years
44 daughter. The present ages of the
a+b = =4 07. Five years ago Viney’s age was
11 mother and the daughter respec-
one-third of the age of Vikas and tively are—
Hence, sum of the two now Viney’s age is 17 years. (A) 35 years, 15 years
digits = 4 What is the present age of Vikas ?
(B) 38 years, 12 years
Short Method (A) 9 years (B) 36 years
(C) 40 years, 10 years
Sum of the two digits of the (C) 41 years (D) 51 years
(D) 42 years, 8 years
x 44 08. The age of a man is 4 times that
number = = 15. Ten years ago A was half of B in
11 11 of his son. Five years ago, the
man was nine times as old as his age. If the ratio of their present
= 4 ages is 3 : 4. What will be the
son was at that time. The present
age of the man is— total of their present ages ?
Exercise (A) 8 years (B) 20 years
(A) 24 years (B) 32 years
01. The ratio of the father’s age to (C) 40 years (D) 44 years (C) 35 years (D) 45 years
the son’s age is 4 : 1. The product 16. The ratio of Vimal’s age and
of their ages is 196. The ratio of 09. The sum of the ages of a father
and son is 45 years. Five years Aruna’s age is 3 : 5 and sum of
their ages after 5 years will be— their ages is 80 years. The ratio
ago the product of their ages was
(A) 3 : 1 (B) 10 : 3 of their ages after 10 years will
4 times the father’s age at that
(C) 11 : 4 (D) 14 : 5 time. The present ages of the be—
02. 10 years ago, Chandravati’s father and son respectively are— (A) 2 : 3 (B) 1 : 2
mother was 4 times older than (A) 25 years, 10 years (C) 3 : 2 (D) 3 : 5
her daughter. After 10 years, the (B) 36 years, 9 years 17. Jayesh is as much younger to
mother will be twice older than (C) 39 years, 6 years Anil as he is older to Prashant. If
the daughter. The present age of the sum of the ages of Anil and
(D) None of these
Chandravati is— Prashant is 48 years. What is the
(A) 5 years (B) 10 years 10. Rajan’s age is 3 times that of age of Jayesh ?
Ashok. After 12 years, Rajan’s
(C) 20 years (D) 30 years (A) 20 years
age will be double the age of
Ashok. Rajan’s present age is— (B) 24 years
03. After five years the age of a
father will be thrice the age of (A) 27 years (B) 32 years (C) 30 years
his son whereas five years ago, (C) 36 years (D) 40 years (D) Cannot be determined
he was seven times as old as his 18. Three years ago the average age
son was. What is father’s present 11. After 10 years, A will be twice
as old as B was 10 years ago. If of A and B was 18 years. With C
age ? joining them, now the average
A is now 9 years older than B,
(A) 35 years (B) 40 years becomes 22 years. How old is C
the present age of B is—
(C) 45 years (D) 50 years now ?
(A) 19 years (B) 29 years
(A) 24 years (B) 27 years
04. The age of Arvind’s father is 4 (C) 39 years (D) 49 years
times of his age. If 5 years ago, (C) 28 years (D) 30 years
12. Mr. Sohanlal is 4 times as old as
father’s age was 7 times of the 19. One year ago the ratio between
his son. Four years hence the sum
age of his son at that time. What Samir and Ashok’s age was 4 : 3.
of their ages will be 43 years.
is Arvind’s father’s present age ? One year hence the ratio of their
The present age of son is—
(A) 35 years (B) 40 years age will be 5 : 4. What is the sum
(A) 5 years (B) 7 years of their present ages in years ?
(C) 70 years (D) 84 years (C) 8 years (D) 10 years (A) 12 years
05. Pushpa is twice as old as Rita 13. The sum of the ages of a son and (B) 15 years
was two years ago. If the diffe- father is 56 years. After four
rence between their ages be 2 (C) 16 years
years, the age of the father will
years, how old is Pushpa today ? be three times that of the son. (D) Cannot be determined
(A) 6 years (B) 8 years Their present ages respectively 20. The ages of A and B are in the
(C) 10 years (D) 12 years are— ratio 2 : 5. After 8 years their
(A) 12 years, 44 years ages will be in the ratio 1 : 2. The
06. The difference between the ages difference of their ages is—
(B) 16 years, 42 years
of two persons is 10 years. 15
years ago the elder one was twice (C) 16 years, 48 years (A) 20 years (B) 24 years
as old as the younger one. The (D) 18 years, 36 years (C) 26 years (D) 29 years

Quantitative Aptitude Test | 133


21. The ratio between the ages of A (C) 3 : 5 age is 5 years, then what’s the
and B at present is 2 : 3. Five age of Randheer ?
(D) None of these
years hence the ratio of their ages (A) 96 years (B) 84 years
will be 3 : 4. What is the present 28. Sachin was twice as old as Ajay
10 years back. How old is Ajay (C) 48 years (D) 60 years
age of A ?
today if Sachin will be 40 years 35. Ratio of Sujeet’s age to Sameer’s
(A) 10 years old 10 years hence ? age is 4 : 3 Sujeet will be 26
(B) 15 years (A) 20 years (B) 10 years years old after 6 years. Then the
(C) 25 years (C) 30 years (D) 15 years present age of Sameer is—
(D) Data inadequate (A) 21 years (B) 15 years
29. The ratio of Mona’s age to the
age of her mother is 5 : 15. The (C) 24 years (D) 18 years
22. The ratio of the ages of father and
son at present is 6 : 1. After 5 difference of their ages is 24 36. The ratio of the father’s and son’s
years the ratio will become 7 : 2. years. The ratio of their ages after age is 7 : 4. The product of their
The present age of the son is— 3 years will be— ages is 1008. The ratio of their
(A) 1 : 3 ages after 6 years hence will be—
(A) 5 years (B) 6 years
(B) 2 : 3 (A) 5 : 3 (B) 8 : 5
(C) 9 years (D) 10 years
(C) 3 : 7 (C) 7 : 4 (D) 5 : 8
23. Ratio of Ashok’s age to Pradeep’s (D) None of these 37. The sum of the ages of the father
age is equal to 4 : 3. Ashok will and son is 45 years. Five years
be 26 years old after 6 years. 30. Deepak is 4 times as old as his
son. Four years hence the sum of ago, the product of their ages was
How old is Pradeep now ? four times the father’s age at that
their ages will be 43 years. How
(A) 12 years old is Deepak’s son now ? time, then the present ages of the
(B) 15 years father and son respectively are
(A) 5 years (B) 7 years
…… and …… years.
1 (C) 8 years (D) 10 years
(C) 19 years (A) 39, 6 (B) 35, 10
2
31. The ages of A, B and C together (C) 36, 9 (D) 40, 10
(D) 21 years total 185 years. B is twice as old
as A and C is 17 years older than 38. The total ages of A, B and C at
24. One year ago a father was four present is 90 years. Ten years ago
times as old as his son. After 6 A. Then the respective ages of
A, B and C are— the ratio of their ages was 1 : 2 :
years his age exceeds than twice 3. Then the present age of B is
of his son’s age by 9 years. Ratio (A) 40, 86 and 59 years
……
of their present ages is— (B) 42, 84 and 59 years
(A) 30 years
(A) 13 : 4 (B) 12 : 5 (C) 40, 80 and 65 years
(B) 20 years
(C) 11 : 3 (D) 9 : 2 (D) None of these
(C) 40 years
25. The age of a father 10 years ago 32. A is 20 years older than B. He is (D) None of these
was thrice the age of his son. Ten also 6 times as old as B. Then
the respective age of A and B are 39. The respective ages of a father
years hence, the father’s age will and his son are 41 and 16 years.
be twice that of his son. The ratio …… and …… years.
In how many years will the father
of their present ages is— (A) 24, 4
be twice as old as his son ?
(A) 8 : 5 (B) 7 : 3 (B) 42, 7
(A) 19 years (B) 9 years
(C) 5 : 2 (D) 9 : 5 (C) 30, 5
(C) 10 years (D) 15 years
(D) None of these
26. Kamla got married 6 years ago. 40. A father’s age is four times as
1 33. Shyam is 3 times as old as his much as the sum of the ages o f
Today her age is 1 times her son. After 10 years the sum of his three children but 6 years
4
their ages will be 76 years. The hence his age will be only double
son’s age at the time of marriage.
respective ages of the father and the sum of their ages. Then the
Her son’s age is (1/10) times of
the son are …… and …… years. age of the father is—
her age. Her son’s age is—
(A) 42, 14 (A) 30 years (B) 40 years
(A) 2 years (B) 3 years
(B) 39, 13 (C) 60 years (D) 45 years
(C) 4 years (D) 5 years
(C) 45, 15 41. A father’s age is three times the
27. The ratio of Laxmi’s age to the (D) None of these sum of the ages of his two
age of her mother is 3 : 11. The children but 20 years hence his
34. If 6 years are subtracted from the
difference of their ages is 24 age will be equal to the sum of
present age of Randheer and the
years. The ratio of their ages their ages. Then the father’s age
remainder is divided by 18 then
after 3 years will be— is—
the present age of his grandson
(A) 1 : 3 Anup is obtained. If Anup is 2 (A) 30 years (B) 40 years
(B) 2 : 3 years younger to Mahesh whose (C) 35 years (D) 45 years

Quantitative Aptitude Test | 134


42. If C’s age is twice the average 50. 15 years hence, A will be twice (C) 6 years
age of A, B and C. A’s age is one as old as B but five years ago A (D) Data is inadequate
half the average of A, B and C. If was 4 times as old as B. Find the
B is 5 years old, the average age difference of their present ages. 57. The age of Sushil 6 years ago
of A, B and C is— was three times the age of Snehal.
(A) 15 years (B) 45 years 6 years hence, the age of Sushil
(A) 10 years (B) 15 years (C) 30 years (D) 25 years 5
(C) 12 years (D) 9 years would be times that of Snehal.
1 3
51. 20 years ago my age was of What is the present age of
43. A is 3 years younger to B. C is 3
two years older than A. Then B’s what it is now. What is my Snehal ?
relation to C is— present age ? (A) 14 years
(A) Two years older (A) 30 years (B) 25 years (B) 22 years
(B) One year younger (C) 35 years (D) 40 years (C) 18 years
(C) One year older 52. The ratio of the present ages of a (D) None of these
(D) Two years younger son and his father is 1 : 5 and that 58. The age of the father 4 years ago
44. Two years ago, a mother was of his mother and father is 4 : 5. was 5 times the age of his son. If
four times as old as her daughter. After 2 years the ratio of the age the sum of their present ages is
8 years hence, mother’s age will of the son to that of his mother 44 years. What is the present age
exceed her daughter’s age by 12 becomes 3 : 10. What is the of his son ?
years. The ratio of the present present age of the father ? (A) 6 years (B) 10 years
ages of mother and daughter is— (A) 30 years (C) 4 years (D) 8 years
(A) 3 : 1 (B) 4 : 1 (B) 28 years
59. The ratio in the ages of A and B,
(C) 3 : 2 (D) 5 : 1 (C) 37 years 1 year ago was 3 : 4. While it
(D) None of these will be 5 : 6 after 1 year. What is
45. Five years ago, the total of the
ages of father and son was 60 53. Present age of Rahul is 8 years the present age of B ?
years. The ratio of their present less than Ritu’s present age. If 3 (A) 8 years
ages is 4 : 1. Then the present years ago Ritu’s age was x, which (B) 4 years
age of the father is— of the following represents (C) 6 years
(A) 48 years (B) 51 years Rahul’s present age ? (D) None of these
(C) 56 years (D) 61 years (A) x + 3 (B) x – 5
60. After 6 years Pradhan’s age will
(C) x – 3 + 8 (D) x + 3 + 8
46. In ten years, A will be twice as 3
be times the age of his father.
old as B was 10 years ago. If A 54. The product of the ages of Harish 7
is now 9 years older than B. Find and Seema is 240. If twice the 10 years ago the ratio in the age
the present age of B. age of Seema is more than of Pradhan and his father was
(A) 39 years (B) 40 years Harish’s age by 4 years. What is 1 : 5. What is the present age of
Seema’s age in years ? Pradhan’s father ?
(C) 36 years (D) 49 years
(A) 12 years (B) 20 years (A) 40 years
47. A is twice as old as B was two (C) 10 years (D) 14 years (B) 50 years
years ago. If the difference in
55. If the ages of P and R are added (C) 56 years
their ages be 2 years, find A’s
age. to twice the age of Q the total (D) Data is inadequate
becomes 59. If the ages of Q and
(A) 14 years (B) 18 years 61. The sum of the ages of Yogesh,
R are added to thrice the age of P
(C) 8 years (D) 12 years Prakash and Sameer is 93 years.
the total becomes 68. And if the 10 years ago the ratio of their
48. A is as much younger than B as age of P is added to thrice the ages was 2 : 3 : 4. What is the
he is older than C. If the sum of age of Q and thrice the age of R, present age of Sameer ?
B’s and C’s age is 40 years. Find the total becomes 108. What is
the age of P ? (A) 32 years
the age of A.
(A) 15 years (B) 19 years (B) 24 years
(A) 20 years (B) 25 years (C) 34 years
(C) 17 years (D) 12 years
(C) 30 years (D) 27 years (D) None of these
56. The ratio in the present ages of
49. A says to B ‘‘I am twice as old as Ramesh and Jayesh is 3 : 2. Four 62. The total of the present age of P,
you were when I was as old as years ago Ramesh’s age was Q and R together is 88 years. If
you are.’’ The sum of their ages more than Jayesh’s age by 6 the difference between the ages
is 63 years. Find the difference of years. What is the present age of of P and R is 12 years. What is
their ages. Jayesh ? Q’s age at present ?
(A) 27 years (B) 12 years (A) 18 years (A) 28 years
(C) 9 years (D) 6 years (B) 12 years (B) 22 years

Quantitative Aptitude Test | 135


(C) 24 years 64. If a son is younger than his father years hence Kunal will be
(D) Data is inadequate by 25 years and the father was 45 younger than Ganesh by 12
years old, 4 years ago. What will years. What is the present age of
63. A man is five times as old as his be the total age of the father and Kunal ?
son. Four years hence the sum of son after 5 years ? (A) 17 years
their ages will be 56 years. How (A) 82 years (B) 88 years (B) 19 years
old is the son at present ? (C) 83 years (D) 78 years
(A) 12 years (B) 5 years (C) 22 years
65. The ratio between present age of
(C) 6 years (D) 8 years Kunal and Ganesh is 3 : 5. If 4 (D) None of these

Answers with Hints


01. (C) Let father’s age be 4x and son’s age x years. ⇒ x – 5 = 36
˙·˙ 4x × x = 196 ∴ x = 41 years
⇒ x2 = 49 08. (B) Let the son’s age be x years. Then father’s age
∴ x = 7 = 4x years.
Father’s age after 5 years = (4x + 5) = 33 years ∴ (4x – 5) = 9(x – 5)
Son’s age after 5 years = (x + 5) = 12 years ⇒ 5x = 40
∴ Ratio of their ages after 5 years ⇒ x = 8
= 33 : 12 = 11 : 4 ∴ Present age of the man = 32 years
02. (C) Let Chandravati’s age 10 years ago be x years. 09. (B) Let the present ages of father and son be x years
∴ Mother’s age 10 years ago = (4x) years and (45 – x) years respectively.
˙·˙ 2 (x + 20) = (4x + 20) Then, (x – 5) (45 – x – 5) = 4 (x – 5)
⇒ x = 10 ⇒ – x2 + 41x – 180 = 0
∴ Present age of Chandravati = (x + 10) ⇒ (x – 5) (x – 36) = 0
= 20 years ⇒ x = 36
03. (B) Let son’s age 5 years hence = x years ∴ The present ages of father and son are 36 years
Then, father’s age after 5 years hence and 9 years respectively.
= (3x) years 10. (C) Let Ashok’s present age be x years. Then Rajan’s
˙·˙ 7 (x – 10) = (3x – 10) present age = 3x years.
⇒ x (7 – 3) = 70 – 10 ∴ 2 (x + 12) = (3x + 12)
⇒ x = 15 ⇒ x = 12
∴ Father’s present age = (3x – 5) Hence, Rajan’s present age is 36 years.
= 40 years 11. (C) Let the present ages of B and A be x years and
04. (B) Let Arvind’s present age be x years (x + 9) years respectively.
Then, his father’s present age = 4x years Then, (x + 9 + 10) = 2 (x – 10)
˙·˙ (4x – 5) = 7(x – 5)
∴ x = 39
⇒ 3x = 30
∴ x = 10 12. (B) Let the son’s present age be x years
Hence, Arvind’s father’s age is 40 years. Then, (x + 4) + (4x + 4) = 43
05. (B) Let Rita’s age 2 years ago be x years. ⇒ 5x = 35
∴ Pushpa’s present age = (2x) years
∴ x = 7
⇒ 2x – (x + 2) = 2 ⇒ x = 4
13. (A) let the present ages of son and father be x years
∴ Pushpa’s present age = 8 years.
and (56 – x) years respectively. Then,
06. (B) Let the present age of the elder person be x years.
(56 – x + 4) = 3 (x + 4)
Then, the present age of the younger person
= (x – 10) years, ⇒ 4x = 48
˙·˙ (x – 15) = 2 (x – 10 – 15) ⇒ x = 12
⇒ x = 35 So, their present ages are 12 years, 44 years respec-
tively.
∴ The present age of the elder person is 35 years.
07. (C) Let the present age of Vikas be x years. Then, 14. (C) Let the daughter’s present age be x years.
1 Then, mother’s present age = (50 – x) years
17 – 5 = (x – 5)
3 Now, 7 (x – 5) = (50 – x – 5)

Quantitative Aptitude Test | 136


⇒ x = 10 ⇒ x = 5
So, the present ages of mother and daughter are 40 ∴ A’s present age = 2x = 10 years
years and 10 years respectively. 6x + 5 7
15. (C) Let A’s age 10 years ago = x years 22. (A) ˙·˙ =
x+5 2
Then, B’s age 10 years ago = 2x years ⇒ 2 (6x + 5) = 7 (x + 5)
x + 10 3 ⇒ (12 – 7) x = 35 – 10
˙·˙ =
2x + 10 4 ⇒ x = 5
⇒ 4 (x + 10) = 3 (2x + 10) ∴ Son’s present age = 5 years
⇒ x (6 – 4) = 40 – 30 23. (B) Let Ashok’s age = 4x years
⇒ x = 5
and Pradeep’s age = 3x years
∴ Total of their present ages = (x + 10 + 2x + 10)
˙·˙ 4x + 6 = 26
= (3x + 20)
⇒ x = 5
= 35 years
16. (A) ˙·˙ 3x + 5x = 80 ∴ Pradeep’s age = 3x = 15 years.
⇒ x = 10 24. (C) Let son’s age 1 year ago be x years.
Ratio of their ages after 10 years ⇒ Father’s age 1 year ago = (4x) years
= (3x + 10) : (5x + 10) ˙·˙ 4x + 7 = 2 (x + 7) + 9
= 40 : 60 = 2 : 3 ⇒ 2x = 23 – 7
17. (B) Let Anil’s age = x years ⇒ x = 8
Then, Prashant’s age = (48 – x) years Ratio of father’s and son’s present ages
Let the age of Jayesh be p years. Then, = (4x + 1 : x + 1)
p – (48 – x) = x – p = 33 : 9 = 11 : 3
⇒ 2p = 48 25. (B) Let son’s age 10 years ago be x years.
∴ p = 24
∴ Father’s age 10 years ago = (3x) years
18. (A) Sum of ages of A and B, 3 years ago
˙·˙ 3x + 20 = 2 (x + 20)
= (18 × 2) = 36 years
⇒ x = 20
Sum of ages of A, B and C, now
= (22 × 3) = 66 years ∴ Ratio of their present ages = (3x + 10 : x + 10)
Sum of ages of A and B, now = 70 : 30 = 7 : 3
= (36 + 6) years = 42 years 26. (B) Let Kamla’s age 6 years ago be x years. Then,
∴ C’s present age = (66 – 42) years = 24 years Kamla’s present age = (x + 6) years
19. (C) Let their ages one year ago be 4x and 3x years. 5
∴ x+6 = x
4x + 2 4
5
˙·˙ = ⇒ 4x + 24 = 5x
3x + 2 4
⇒ 4 (4x + 2) = 5 (3x + 2) ⇒ x = 24
⇒ x = 2 So, Kamla’s present age = (x + 6) years
∴ Sum of their present ages = (4x + 1 + 3x + 1) = 30 years

2x + 8
= 16 years
1
∴ Son’s present age = ( 1
10 )
× 30 = 3 years
20. (B) ˙·˙ = 27. (A) ˙·˙ 11x – 3x = 24
5x + 8 2
⇒ 2 (2x + 8) = (5x + 8) ⇒ x = 3
⇒ x = 8 ∴ Ratio of their ages after 3 years.
= (3x + 3 : 11x + 3)
∴ Difference of their ages = (5x – 2x)
= 12 : 36 = 1 : 3
= 3x = 24 years.
28. (A) Let Ajay’s age 10 years back be x years.
21. (A) Let the ages of A and B be 2x and 3x years
respectively. Then Sachin’s age 10 years back = 2x years
2x + 5 3 ∴ 2x + 20 = 40
˙·˙ =
3x + 5 4 ⇒ x = 10
⇒ 4 (2x + 5) = 3 (3x + 5) ∴ Present age of Ajay = x + 10 = 20 years

Quantitative Aptitude Test | 137


29. (C) Let the age of Mona and her mother be 5x and ∴ Father’s age = 7x = 42 years
13x years respectively. Son’s age = 4x = 24 years
Then, (13x – 5x) = 24 Father’s age after 6 years hence = 48 years
⇒ x = 3 Son’s age 6 years hence = 30 years
So, their present ages are 15 years and 39 years Required ratio = 48 : 30 = 8 : 5
Ratio of their ages after 3 years 37. (C) Let son’s present age be x years and father’s age
= 18 : 42 = (45 – x) years.
= 3:7 ˙·˙ (x – 5) (45 – x – 5) = 4 (45 – x – 5)
30. (B) Let the son’s present age be x years ⇒ (40 – x) (x – 9) = 0
Then, (x + 4) + (4x + 4) = 43 ⇒ x = 9 years
⇒ 5x = 35 ∴ The son’s age = 9 years
⇒ x = 7 years Father’s age = 45 – 9 = 36 years
31. (B) Let A’s age be x years. B’s age be 2x years. 38. (A) Let the respective ages of A, B and C ten years
C’s age = (x + 17) years ago be x, 2x and 3x years.
According to the question, ˙·˙ (x + 10) + (2x + 10) + (3x + 10) = 90
˙·˙ x + 2x + (x + 17) = 185 ⇒ 6x = 60
⇒ 4x = 185 – 17 = 168 ⇒ x = 10
∴ x = 42 ∴ B’s present age = 2x + 10
∴ A’s age = 42 years = 30 years
B’s age = 84 years 39. (B) Suppose x years hence the father will be twice as
C’s age = 42 + 17 = 59 years old as his son.
32. (A) Let the age of B be x years ˙·˙ x + 41 = 2 (x + 16)
According to the question, ⇒ x = 41 – 32 = 9 years
·.· x + 20 = 6x 40. (C) Let father’s age be x years and the sum of ages of
children be y years.
∴ x = 4 years
˙·˙ x = 4y …(i)
∴ A’s age = 4 × 6 = 24 years and B’s age = 4 years.
33. (A) Let son’s present age be x years. Also (x + 6) = 2 (y + 6 + 6 + 6) …(ii)
∴ Father’s present age = 3x years [6 is added thrice for three children]
Son’s age 10 years hence = (x + 10) Solving (i) and (ii)
Father’s age 10 years hence = (3x + 10) x = 60 years
As per the condition, and y = 15 years.
˙·˙ (x + 10) + (3x + 10) = 76 41. (A) Let the present age of father be x years and the
⇒ 4x = 56 sum of present ages of 2 sons be y years.
∴ x = 14 ˙·˙ x = 3y …(i)
∴ Son’s present age = 14 years
⇒ (x + 20) = (y + 20 + 20) …(ii)
Father’s present age = 42 years
34. (D) ˙·˙ Anup’s age = (5 – 2) years = 3 years [20 will be added twice as for 2 children]
Let Randheer’s age be x years. Then, Solving (i) and (ii), we get
x–6 x = 30 years
= 3
18 42. (A) Let the average age of A, B and C be x years.
⇒ x = 54 + 6 = 60
∴ Total age of A, B and C = 3 × x = 3x years
35. (B) Let the respectively ages of Sujeet and Sameer
be 4x and 3x years. Now, according to the question,
˙·˙

4x + 6 = 26
x = 5
˙·˙ ( )
3x – 2x +
2
x
= 5

∴ Sameer’s present age = 3 × x = 3 × 5 ∴ x = 10 years.


= 15 years. 43. (C) Let the age of A be (x – 3) years
36. (B) Let father’s and son’s age be 7x and 4x respec- ⇒ B’s age = x years
tively.
⇒ C’s age = (x – 3) + 2 = x – 1
˙·˙ 28x2 = 1008
⇒ x2 = 36 ∴ B’s age – C’s age = x – (x – 1) = 1 year
⇒ x = 6 ∴ B is one year older than C.

Quantitative Aptitude Test | 138


44. (A) Let the mother’s age 2 years ago be 4x and S+2 3
˙·˙ =
daughter’s age 2 years ago be x. M+2 10
˙·˙ (4x + 8) – (x + 8) = 12 ⇒ 10S + 20 = 3M + 6 …(iii)
⇒ 3x = 12 4
= 3 × × 5S + 6 = 12S + 6
⇒ x = 4 5
∴ Mother’s present age = 4x + 2 = 18 years ⇒ (12 – 10) S = 20 – 6
and daughter’s present age = x + 2 = 6 years ⇒ 2S = 14
∴ Required ratio = 3 : 1 ∴ S = 7 years
45. (C) Let the present age of the son be x and that of the ∴ F = 5S = 5 × 7
father be 4x years. = 35 years.
˙·˙ (x – 5) + (4x – 5) = 60 53. (B) Let the Rahul’s present age is ‘A’ years. Then
⇒ 5x = 70 Ritu’s present age is (A + 8)
∴ x = 14 years Now, according to the question,
∴ Father’s present age = 56 years A+8–3 = x
46. (A) Let B’s present age be x years then A’s present ∴ A = (x – 5) years
age be (x + 9) years.
Hence, (B) is the correct answer.
As per the given condition,
54. (A) Let the ages of Harish and Seema be x and y
˙·˙ (x + 9 + 10) = 2 (x – 10)
respectively.
⇒ x = 39
According to the question,
∴ The present age of B = 39 years
47. (C) Let B’s age 2 years ago be x years xy = 240 …(i)
∴ A’s present age = 2x years 2y – x = 4 …(ii)
˙·˙ 2x – (x + 2) = 2 Solving equations (i) and (ii), we get
⇒ x = 4 y = 12 years
∴ A’s age = 2 × 4 = 8 years 55. (D) P + R + 2Q = 59, …(i)
48. (A) 49. (C)
Q + R + 3P = 68 …(ii)
50. (C) Let A’s age = x years and B’s age = y years.
and P + 3 (Q + R) = 108 …(iii)
As per the first condition,
Solving the above three equations, we get
∴ (x + 15) = 2 (y + 15)
⇒ x – 2y = 15 …(i) ⇒ P + 3 (68 – 3P) = 108
As per the second condition, ⇒ P + 204 – 9P = 108
∴ (x – 5) = 4 (y – 5) ⇒ P = 12 years
⇒ x – 4y = – 15 …(ii) 56. (B) Let the present age of Ramesh be 3x years and the
Solving (i) and (ii) one gets, present age of Jayesh = 2x years. According to
question,
x = 45, y = 15
˙·˙ (3x – 4) = (2x – 4) + 6
∴ A’s age = 45 years
⇒ 3x – 4 = 2x – 4 + 6
B’s age = 15 years
∴ Difference of their ages ⇒ 3x – 4 = 2x + 2
= 45 – 15 = 30 years ⇒ 3x – 2x = 2 + 4
51. (A) Let my present age be x years. ∴ x = 6
x ∴ Present age of Jayesh = 2x
˙·˙ (x – 20) = = 2 × 6 = 12 years
3
⇒ (3x – 60) = x 57. (D)
⇒ 2x = 60 58. (B) Let the present age of father and son be x and y
years respectively.
∴ My present age = x
From 1st condition,
= 30 years
(x – 4) = 5 (y – 4)
S 1
52. (D) ˙·˙ = ⇒ x – 4 = 5y – 20
F 5
⇒ F = 5S, …(i) ⇒ x – 5y = – 16 …(i)
M 4 From 2nd condition,
˙·˙ = ⇒ x + y = 44 …(ii)
F 5
4 From equation (i) and (ii)
⇒ M = F …(ii) – 6y = – 60
5
Quantitative Aptitude Test | 139
⇒ y = 10 ∴ Present age of Yogesh = (2x + 10) years
∴ Present age of son = 10 years ∴ Present age of Prakash = (3x + 10) years
59. (D) Let the present age of A and B be x and y years ∴ Present age of Sameer = (4x + 10) years
respectively. According to question,
From 1st condition, (2x + 10) + (3x + 10) + (4x + 10) = 93
x–1 3 ⇒ 9x + 30 = 93
=
y–1 4 ⇒ 9x = 63
⇒ 4x – 4 = 3y – 3 ∴ x = 7
⇒ 4x – 3y = 1 …(i) ∴ Present age of Sameer = 4x + 10
From 2nd condition, = 4 × 7 + 10
x+1 5
= = 28 + 10
y+1 6 = 38 years
⇒ 6x + 6 = 5y + 5 62. (D) Data is inadequate. (Two equations only for 3
⇒ 6x – 5y = – 1 …(ii) unknown).
Multiplying equation (i) by 3 and equation (ii) by 2 63. (D) Let the present age of man be x years and the age
and subtract of his son be y years.
12x – 9y = 3 From 1st condition, x = 5y …(i)
12x – 10y = – 2 From 2nd condition,
– + + (x + 4) + (y + 4) = 56
y = 5 ⇒ x + y + 8 = 56
∴ Present age of B = 5 years ⇒ x + y = 48 …(ii)
60. (B) Let the present age of Pradhan be x years and his Putting x = 5y in equation (ii)
father’s age = y years. 5y + y = 48
From 1st condition, ⇒ 6y = 48
3 ⇒ y = 8
(x + 6) = (y + 6)
7 ∴ Age of son = 8 years
⇒ 7x + 42 = 3y + 18 64. (C) Let the present age of father be x years. Then the
⇒ 7x – 3y = – 24 …(i) present age of son = (x – 25) years.
From 2nd condition, According to question,
x – 4 = 45
(x – 10) 1
= ⇒ x = 45 + 4
(y – 10) 5
⇒ x = 49
⇒ 5x – 50 = y – 10
⇒ 5x – y = 40 …(ii) ∴ Age of son = 49 – 25
Multiplying equation (ii) by 3 and subtracting = 24 years
from (i) ∴ Age of son after five years = 24 + 5
⇒ 7x – 3y = – 24 = 29 years
⇒ 15x – 3y = 120 ∴ Age of father after five years = 49 + 5
– + – = 54 years
∴ – 8x = – 144 Total age of both = 29 + 54
144
x = = 18 = 83 years
8
Putting the value of x in equation (i) 65. (D) Let the present age of Kunal be 3x years and the
present age of Ganesh = 5x years.
7 × 18 – 3y = – 24
⇒ 126 – 3y = – 24 According to question,
⇒ 3y = 126 + 24 (5x + 4) – (3x + 4) = 12
⇒ 3y = 150 ⇒ 5x – 3x = 12
150 ⇒ 2x = 12
⇒ y =
3
∴ x = 6
∴ y = 50 years
∴ Present age of Kunal = 3 × x
61. (D) Let the respective ages of Yogesh, Prakash and
Sameer 10 years ago be 2x, 3x and 4x years. = 3 × 6 = 18 years

Quantitative Aptitude Test | 140


Percentage
Important Points/Facts Q. 2. Express each of the 8 × 100
⇒ x =
following as a decimal. 64
1. The term per cent means for
every hundred. It can be best defined (i) 36% (ii) 8% = 12·5
as—“A fraction whose denominator (iii) 0·3% (ii) Let x% of 36 = 144
is 100 is called a percentage, and Solution : x
numerator of the fraction is called the Then, × 36 = 144
rate per cent.” 36 100
(i) 36% = = 0·36
100 144 × 100
2. To express a is per cent of b ⇒ x =
8 36
a (ii) 8% = = 0·08
100 = 400
can be written as b × 100
0·3 (iii) Let x% of 24 = ·72
(iii) 0·3% = = 0·003
Note—The numerator is the 100 x
term to be expressed while the de- Q. 3. Express each of the Then, × 24 = ·72
100
nominator is the term in which the following as rate per cent.
percentage is to be expressed. The ·72 × 100
2 1 ⇒ x =
following example illustrate it clearly. (i) (ii) 24
3 12
Example : How much per cent = 3
(iii) 0·002
15 is of 60 ? Of what per cent of 60 is Q. 6. (i) What per cent is 120 of
15 ? Here 15 is to be expressed in Solution : 90 ?
percentage of 60, therefore, we will
take 15 as numerator, while 60 as (i)
2
3
= ( 2
3 )
× 100 %
kg ?
(ii) What per cent is 5 gms of 1
denominator, i.e., 2 (iii) What per cent is 150 ml of
15 = 66 %
× 100% = 25% 3 3·5 litres ?
60
3. Percentages are simply a con- (ii)
1
12
= ( 1
12 )
× 100 %
Solution :

venient way of expressing fractions


or decimals. = 8
1
%
(i) It is (
120
90
× 100 % )
b ab 3 1
b% of a = a × = = 133 %
100 100 (iii) 0·002 = ( 2
1000
× 100 % ) 3

and a% of b = b ×
a
=
100 100
ab
= 0·2%
(ii) It is (1000
5
× 100)%

ab Q. 4. Find : = 0·5%
i.e., a% of b = = b% of a
100
Percentages are used very fre-
(i) 70% of 70 (ii) 90% of 9
(iii) 3% of 6
(iii) It is (3500
150
× 100)%

quently in every day life and are mis- Solution : 2


= 4 %
understood by a large number of 7
people. (i) 70% of 70 = ( )
70
100
× 70 = 49 Short-cut Method :
(i) If A’s income is r% more than
(ii) 90% of 9 = (
100 )
Examples 90
× 9 = 8·1 B’s income then B’s income is less
than A’s income by
Q. 1. Express each of the
(iii) 3% of 6 = (
100 )
3
× 6 = 0·18
following as a fraction.
(i) 64% (ii) 6%
[ r
(100 + r)
× 100 %.]
Q. 5. Fill in the blanks :
(iii) 0·5% (ii) If A’s income is r% less than
(i) (… ? …)% of 64 = 8 B’s income, then B’s income is more
Solution :
(ii) (?)% of 36 = 144 than A’s income by
64 16
(i) 64% = =
100 25
(iii) (?)% of 24 = ·72
Solution :
[ r
(100 – r)
× 100 %.]
6 3
(ii) 6% = = (i) Let x% of 64 = 8 Q. 7. If A’s salary is 50% more
100 50 than that of B, then how much per
0·5 5 1 x cent is B’s salary less than that of
(iii) 0·5% = = = Then, × 64 = 8
100 1000 200 100 A?

Quantitative Aptitude Test | 141


Solution : B’s salary is less than (C) 17 30% of 80
17. = 24
that of A by [ r
(100 + r) ]
× 100 % (D) Data inadequate
07. ?% of 130 = 10·4 (A)
?
3
(B)
3
=( 50
150 )
× 100 % = 33 %
1
3
(A) 80
(C) 0·8
(B) 8
(D) 0·08
10
(C) 1
17
(D) 2
Q. 8. If A’s salary is 30% less 18. Ashok gave 40 per cent of the
2
than that of B, then how much per 08. The fraction equivalent to % amount he had to Jayant. Jayant
cent is B’s salary more than that of 5 in turn gave one-fourth of what
A? is— he received from Ashok to
1 1 Prakash. After paying Rs. 200 to
Solution : B’s salary more than (A) (B)
40 125 the taxi driver out of the amount
that of A by [ r
100 – r ]
× 100 %
(C)
1
(D)
1 he got from Jayant, Prakash now
has Rs. 600 left with him. How
250 500
= ( 30
70 )
× 100 % = 42 %
6
7 09. 30% of 140 = ?% of 840
much amount did Ashok have ?
(A) 1200
(A) 5 (B) 15
Exercise (B) 4000
(C) 24 (D) 60
(C) 8000
1 10. ?% of 250 + 25% of 68 = 67 (D) Data inadequate
01. 8 % expressed as a fraction is—
3 (A) 10 (B) 15
1
25 3 (C) 20 (D) 25 19. 8% of 96 = ? of
(A) (B) 25
3 25
11. 5% of [50% of Rs. 300] is— (A) 19·2
1 1
(C) (D) (A) Rs. 5 (B) Rs. 7·50 (B) 7·68
12 4
(C) Rs. 8·50 (D) Rs. 10 (C) 1·92
02. ·025 in terms of rate per cent
is— 12. Pradeep spends 40 per cent of (D) None of these
(A) 25% (B) 2·5% his monthly income on food 20. If 8% of x = 4% of y, then 20%
1 items and 50 per cent of the of x is—
(C) 0·25% (D) 37 % remaining on cloths and
2 (A) 10% of y
conveyance. He saves one-third
03. ·02 = (…… ? ……)% (B) 16%
of the remaining amount after
(A) 20 (B) 2 spending on food, clothes and (C) 80% of y
(C) ·02 (D) ·2 conveyance. If he saves Rs. (D) None of these
2 1 19,200 every year, what is his
04. What per cent of is ? monthly income ? 21. If x is 90% of y, then what per
7 35
cent of x is y ?
(A) 2·5% (B) 10% (A) Rs. 24000
(C) 25% (D) 20% (A) 90 (B) 190
(B) Rs. 12000
(C) 101·1 (D) 111·1
05. What per cent of 7·2 kg is 18 (C) Rs. 16000
gms ? (D) Rs. 20000 22. (x% of y + y% of x) is equal to—
(A) ·025% (B) ·25% (A) x% of y (B) y% of x
(C) 2·5% (D) 25% 13. What is 25% of 25% is equal
to ? (C) 2% of xy (D) xy% of 3
06. Out of a total 85 children playing
badminton or table tennis or both, (A) 6·25 (B) ·625 23. If 31% of a number is 46·5, the
total number of girl in the group (C) ·0625 (D) ·00625 number is—
is 70% of the total number of (A) 150 (B) 155
14. The number ·05 is how many per
boys in the group. The number (C) 160 (D) 165
cent of 20 ?
of boys playing only badminton
is 50% of the number of boys and (A) 25 (B) ·025 24. Rajesh solved 80 per cent of the
the total number of boys playing (C) ·25 (D) 2·5 questions in an examination
badminton is 60% of the total correctly. Out of 41 questions
15. ? × 15 = 37·5% of 220 solved by Rajesh 37 questions
number of boys. The number of
children playing only table tennis (A) 82·5 (B) 8250 are correct and of the remaining
is 40% of the total number of (C) 11 (D) 5·5 questions out of 8 questions, 5
children and a total 12 children questions have been solved by
play badminton and table tennis
both. What is the number of girls
16. (0·756 × 34) in terms of rate Rajesh correctly then find the
total number of questions asked
playing only badminton ? per cent is equivalent to— in the examination ?
(A) 16 (A) 18·9% (B) 37·8% (A) 75
(B) 14 (C) 56·7% (D) 75% (B) 65

Quantitative Aptitude Test | 142


(C) 60 35. 30 quintals is what per cent of 2 43. The price of an article is cut by
(D) Can’t be determined metric tonnes ? 10%. To restore it to the former
(A) 15% (B) 1·5% value. The new price must be
25. Which number is 60% less than increased by—
80 ? (C) 150% (D) 30%
1
(A) 48 (B) 42 36. The ratio of males and females in (A) 10% (B) 9 %
11
(C) 32 (D) 12 a city is 7 : 8 and the percentage 1
of children among males and (C) 11 % (D) 11%
26. A number exceeds 20% of itself 9
by 40. The number is— females is 25% and 20% respec-
tively. If the number of adult 44. The income of a broker remains
(A) 50 (B) 60 unchanged though the rate of
females in the city is 156800,
(C) 80 (D) 320 what is the total population ? commission is increased from 4%
27. If 90% of A = 30% of B and B = (A) 245000 (B) 367500 to 5%. The percentage of slump
x% of A, then the value of x is— in business is—
(C) 196000 (D) 171500
(A) 900 (B) 800 (A) 8% (B) 1%
37. x% of y is y% of ?
(C) 600 (D) 300 (C) 20% (D) 80%
(A) x (B) 100x
1 4 45. One-third of 1206 is what per
28. Which is greatest 33 %, and x y
(C) (D) cent of 134 ?
3 15 100 100
0·35 ? (A) 3
38. 12·5% of 192 = 50% of ? (B) 30
1
(A) 33 % (A) 48 (C) 300
3
4 (B) 96 (D) None of these
(B) (C) 24
15 46. Rakesh credits 15% of his salary
(C) 0·35 (D) None of these in his fixed deposit account and
(D) Cannot be compared 1 spend 30% of the remaining
39. If 37 % of a number is 900, amount on groceries. If the cash
29. 200 = ?% of 300 2
1 in hand is Rs. 2380, what is his
1 then 62 % of the number is— salary ?
(A) 33 (B) 85 2
3 (A) 35000 (B) 4000
(A) 1200 (B) 1350
2 (C) 45000 (D) 5000
(C) 66 (D) 150 (C) 1500 (D) 540
3
47. A man donated 5% of his income
30. In a class of 60 children 30% 40. Subtracting 6% of x from x is
to a charitable organisation and
children can speak only English, equivalent to multiplying x by deposited 20% of the remainder
20% Hindi and English both and how much ? in a bank. If he now has Rs. 1919
the rest of the children can speak (A) 0·94 (B) 9·4 left, what is his income ?
only Hindi. How many children (C) 0·094 (D) 94 (A) Rs. 2558·60
can speak Hindi ?
41. By how much is 30% of 80 (B) Rs. 2525
(A) 42 (B) 36
(C) 30 (D) 48 4 (C) Rs. 2500
greater than th of 25 ?
5 (D) Rs. 2300
31. 45 × ? = 25% of 900
(A) 2 (B) 4 48. The price of coffee is increased
(A) 16·20 (B) 4
(C) 10 (D) 15 by 15% and a house wife reduced
(C) 5 (D) 500 her consumption of coffee by
32. If 0·5% of x = 85 paise, then the 42. The ratio of the number of student 15% and hence her expenditure
value of x is— appearing for examination in the on coffee—
year 1998 in the states. A, B and
(A) Rs. 170 (B) Rs. 17 (A) Remains unchanged
C was 3 : 5 : 6. Next year if the
(C) Rs. 1·70 (D) Rs. 4·25 number of students in these states (B) Increase by 1%
33. What per cent is 3% of 5% ? increase by 20% 10% and 20% (C) Decreases by 4%
(A) 15% respectively, the ratio in states A (D) Decreases by 2·25%
(B) 1·5% and C would be 1 : 2. What was
49. After spending 40% in
(C) 0·15% the number of students who
machinery, 25% in building, 15%
(D) None of these appeared for the examination in
in raw material and 5% on
the state A in 1999 ?
34. 75% of 480 = (?) × 15 furniture. Hari Lal has a balance
(A) 7200 of Rs. 1305. The money with
(A) 32
(B) 18 (B) 6000 him was—
(C) 360 (C) 7500 (A) Rs. 6500 (B) Rs. 7225
(D) None of these (D) Data inadequate (C) Rs. 8700 (D) Rs. 1390

Quantitative Aptitude Test | 143


1 59. ⎯⎯⎯⎯⎯⎯⎯⎯⎯
√ (3·6% of 40) is equal to— 67. In an examination, there were
50. A number increased to 137 %
2 2000 candidates out of which 900
(A) 2·8
gives 33. The number is— candidates were boys and rest
(B) 1·8 were girls. If 32% of the boys
(A) 22 (B) 24 (C) 1·2 and 38% of the girls passed, then
(C) 25 (D) 27 (D) None of these the total percentage of failed
1 60. In an examination 65% of the candidates is—
51. A number decreased to 72 %
2 total examinees passed if the (A) 35·3% (B) 64·7%
gives 87. The number is— number of failures is 420 total (C) 68·5% (D) 70%
(A) 58 (B) 110 number of examinees is—
(C) 120 (D) 135 (A) 567 (B) 693 68. A student who secure 20% marks
(C) 1000 (D) 1200 in an examination fails by 30
52. 25% of a number is more than marks. Another student who
18% of 650 by 19. The number 61. 75% of a number when added to secures 32% marks gets 42 marks
is— 75 is equal to the number. The more than those required to pass.
(A) 380·8 number is— The percentage of marks required
(B) 450 (A) 150 (B) 200 to pass is—
(C) 225 (D) 300
(C) 544 (A) 20% (B) 25%
(D) None of these 62. 5% income of A is equal to 15% (C) 28% (D) 30%
income of B and 10% income of
53. 96% of the population of a village B is equal to 20% income of C. 69. In a college election, a candidate
is 23040. The total population of If income of C is Rs. 2000, then secured 62% of the votes and is
the village is— total income of A, B and C is— elected by a majority of 144
(A) 32256 (B) 24000 (A) Rs. 6000 votes. The total number of votes
(C) 24936 (D) 25640 (B) Rs. 18000 polled is—
54. After deducting a commission of (C) Rs. 20000 (A) 600 (B) 800
5% a T.V. set costs Rs. 9595. Its (D) Rs. 14000 (C) 925 (D) 1200
gross value is— 63. In mathematics exam. a student 70. There were 600 students in a
(A) Rs. 10000 scored 30% marks in the first school. Each offered either
(B) Rs. 10074·75 paper out of a total of 180. How English or Hindi or both. If 75%
(C) Rs. 10100 much should he score in second offered English and 45% Hindi,
paper out of a total of 150, if he how many offered both ?
(D) None of these is to get an overall average of
55. A man spends Rs. 3500 per atleast 50% ? (A) 48 (B) 60
1 (A) 74% (B) 76% (C) 80 (D) 120
month and saves 12 % of his
2 (C) 70% (D) 80% 71. What will be 80 per cent of a
income. His monthly income is— number whose 200 per cent is
64. In a examination it is required to
(A) Rs. 4400 (B) Rs. 4270 get 36% of maximum marks to 90 ?
(C) Rs. 4000 (D) Rs. 3937·50 pass. A student got 113 marks (A) 144
56. If 70% of the students in a school and declared failed by 85 marks. (B) 72
are boys and the number of girls The maximum marks are—
(A) 500 (B) 550 (C) 36
be 504, the number of boys is—
(C) 640 (D) 1008 (D) None of these
(A) 1176 (B) 1008
(C) 1208 (D) 3024 65. From the salary of an officer 10% 72. The price of sugar is increased by
deducted as house rent, 15% of 20%. If the expenditure is not
57. A fruit seller had some apples. the rest he spends on children’s allowed to increase. Then ratio
He sells 40% and still has 420 education and 10% of the between the reduction in
apples. Originally, he had— balance, he spends on clothes. consumption and the original
(A) 588 apples After this expenditure he is left consumption—
(B) 600 apples with Rs. 1377. His salary is— (A) 1 : 3 (B) 1 : 4
(C) 672 apples (A) Rs. 2000 (B) Rs. 2040
(C) 1 : 6 (D) 1 : 5
(C) Rs. 2100 (D) Rs. 2200
(D) 700 apples
66. Two numbers are less than a third 73. Water tax is increased by 20%
58. An ore contains 12% copper. number by 30% and 37% respec- but its consumption is decreased
How many kg of ore are required tively. How much per cent is the by 20%. Then the increase or
to get 69 kg of copper ? second number less than the decrease in the expenditure of
(A) 424 kg (B) 575 kg first ? the money is—
2 (A) 3% (B) 4% (A) No change
(C) 828 kg (D) 1736 kg (C) 7% (D) 10%
3 (B) 5% decrease

Quantitative Aptitude Test | 144


(C) 4% increase lation increase in per cent is 88. A mixture of 40 litres of milk and
given by— water contains 10% water. How
(D) 4% decrease
(A) 0·021% (B) 0·0021% much water should be added to
74. 3 litres of water is added to 15 (C) 21% (D) 2·1% this so that water may be 20% in
litres of a mixture of a 20% solu- the new mixture ?
tion of alcohol in water. The 81. The population of a town
increases 4% annually but is (A) 4 litres (B) 5 litres
strength of alcohol is now—
decreased by emigration annually (C) 6·5 litres (D) 7·5 litres
1 2 1
(A) 12 % (B) 16 % to the extent of %. What will be
2 3 89. The price of rice has increased by
2
60%. In order to restore to the
(C) 24% (D) 16% the increase per cent in three
original price, the new price must
years ?
75. On decreasing the price of T. V. be reduced by—
(A) 9·8 (B) 10
sets by 30%. Its sale is increased 1 1
by 20%. What is the effect on the (C) 10·5 (D) 10·8 (A) 33 % (B) 37 %
3 2
revenue received by the shop- 82. A man’s wages were decreased
keeper ? (C) 40% (D) 45%
by 50%. Again the reduced
(A) 10% increase wages were increased by 50%. 90. The boys and girls in a college
He has a loss of— are in ratio 3 : 2. If 20% of the
(B) 10% decrease
(A) 0% (B) 0·25% boys and 25% of the girls are
(C) 16% increase adults, the percentage of students
(C) 2·5% (D) 25%
(D) 16% decrease who are not adults is—
83. p is six times as large as q. The
76. In vocational course in a college per cent that q is less than p, is— (A) 58% (B) 67·5%
15% seats increase annually. If 1 2 (C) 78% (D) 82·5%
there were 800 students in 1992, (A) 83 (B) 16
3 3 91. 72% of the students of a certain
how many students will be there
(C) 90 (D) 60 class took Biology and 44% took
in 1994 ? Mathematics. If each student
84. If A’s salary is 30% more than
(A) 920 (B) 1040 took Biology or Mathematics
B’s then how much per cent is
(C) 1058 (D) 1178 and 40 took both. The total
B’s salary less than A’s ?
number of students in the class
77. The population of a town is 8000. (A) 30% (B) 25% was—
It increases by 10% during first 1 1
(C) 23 % (D) 33 % (A) 200 (B) 240
year and by 20% during the 13 3
second year. The population after (C) 250 (D) 320
85. A man spends 75% of his in-
2 years will be— come. This income is increased 92. In measuring the side of a square
(A) 10400 by 20% and he increased his an error of 5% in excess is made.
(B) 10560 expenditure by 10%. His savings The error per cent in the calcu-
are increased by— lated area is—
(C) 10620 (A) 10% (B) 25% (A) 10% (B) 10·25%
(D) None of these 1
(C) 37 % (D) 50% (C) 10·5% (D) 25%
78. A papaya was planted 2 years 2
93. One litre of water is evaporated
ago. It increases at the rate of 86. The price of an article has been from 6 litre of a solution contain-
20% every year. If at present, the reduced by 25%. In order to ing 5% salt. The percentage of
height of the tree is 540 cm, what restore the original price, the new salt is—
was it when the tree was price must be increased by—
planted ? 4 5
1 1 (A) 4 % (B) 5 %
(A) 33 % (B) 11 % 9 7
(A) 324 cms (B) 375 cms 3 9
(C) 5% (D) 6%
(C) 400 cms (D) 432 cms 1 2
(C) 9 % (D) 66 % 94. If the side of a square is increased
11 3
79. The value of a machine depre- by 30%, its area is increased
ciates 10% annually. If its present 87. If the numerator of a fraction is by—
value is Rs. 4000. Its value 2 increased by 20% and the
(A) 9% (B) 30%
years hence will be— denominator be diminished by
10%, the value of the fraction is (C) 60% (D) 69%
(A) Rs. 3200 (B) Rs. 3240 16 . 95. A reduction of 21% in the price
(C) Rs. 3260 (D) Rs. 3280 The original fraction is—
21 of wheat enables a person to buy
80. The current birth rate per thou- 3 4 10·5 kg more for Rs. 100. What
(A) (B) is the reduced price per kg ?
sand is 32 whereas corresponding 5 7
death rate is 11 per thousand. The 2 5 (A) Rs. 2 (B) Rs. 2·25
net growth rate in terms of popu- (C) (D) (C) Rs. 30 (D) Rs. 2·50
3 7
Quantitative Aptitude Test | 145
96. The radius of circle is increased 98. For a sphere of radius 10 cms. the 1
(A) 37 % (B) 60%
by 1%. What is the increased per numerical value of the surface 2
cent in its area ? area is how many per cent of the (C) 75% (D) None of these
(A) 1% (B) 1·1% numerical value of its volume ?
(C) 2% (D) 2·01% (A) 24% (B) 26·5% 100. The price of cooking oil has
increased by 25%. The percent-
97. The length and breadth of square (C) 30% (D) 45% age of reduction that a family
are increased by 30% and 20%
should effect in the use of cook-
respectively. The area of the 99. The length of a rectangle is ing oil so as not to increase the
rectangle so formed exceeds the increased by 60%. By what per expenditure on this account is—
area of the square by— cent would the width have to be
(A) 20% (B) 36% decreased to maintain the same (A) 15% (B) 20%
(C) 50% (D) 56% area ? (C) 25% (D) 30%

Answers with Hints


01. (C) 8
1
3
% = (253 ×1001 ) = 121 09. (A)

Let x% of 840 = 30% of 140
x
× 840 =
30
× 140
02. (B) ·025 = ( × 100)% = 2·5%
25 100 100
1000 ∴ x = ( 30
100
× 140 × )
100
840
=5
·02 = ( × 100)% = 2%
2
03. (B) 10. (C) Let x% of 250 + 25% of 68 = 67
100
x 25
04. (B) Required percentage = [ × 100]%
1/35 ⇒ × 250 + × 68 = 67
2/7 100 100
5x
= ( × × 100)%
1 7 ⇒ = 50
35 2 2
50 × 2
= 10% ∴ x = ( )5
= 20
05. (B) Required percentage = ( 18
7·2 × 1000 )
× 100 %
11. (B) 5% of (50% of Rs. 300) = Rs. ( 5
)
100
× 300
= 0·25% = 7·50
06. (B) Let the number of boys = x 12. (C) ·.· Food items = 40%
7x Clothes + Conveyance = 50% of 60% = 30%
then x+ = 85
10 ⇒ Remaining amount = 30%
⇒ x = 50 1 19200
˙·˙ of 30% =
∴ No. of girls = 85 – 50 = 35 3 12
(i) Number of boys playing only badminton ⇒ 10% = 1600
= 50% of boys ∴ 100% = Rs. 16000
25 25 625
50 13. (C) 25% of 25% = × = = ·0625
= × 50 = 25 100 100 10000
100
14. (C) Let x% of 20 = ·05
(ii) Number of children playing only table tennis
x
= 40% of total no. of children Then, × 20 = ·05
100
40
= × 85 = 34 ∴ x = ·25
100
15. (D) Let x × 15 = 37·5% of 220
(iii) Total no. of children playing both badminton
and table tennis = 12
Hence, number of girls playing only badminton
⇒ 15x = ( 37·5
100 )
× 220
37·5 × 220
= 85 – (25 + 34 + 12) ∴ x = = 5·5
100 × 15
= 85 – 71 = 14.
07. (B) Let x% of 130 = 10·4 16. (
(C) 0·756 × ) (
3
4
=
756 3
1000 4 )
× × 100 % = 56·7%
x 30% of 80
⇒ × 130 = 10·4 17. (C) Let = 24
100 x
10·4 × 100
∴ x =
130
=8 ⇒ 24x = ( 30
100 )
× 80
30 × 80
08. (C)
2
5
% = ( 2 1
×
5 100 ) =
1
250
∴ x = ( 24 × 100 ) =1

Quantitative Aptitude Test | 146


2 25. (C) Required number = 80 – 60% of 80
18. (C) ˙·˙ J = 40% of A = A

⇒ P =
1 2 1
× A= A
5
(
= 80 –
60
100 )
× 80 = 32
4 5 10 26. (A) ˙·˙ x – 20% of x = 40
1
⇒ A – 200 = 600 x
10 ⇒ x – = 40
5
1
⇒ A = 800 4x
10 ⇒ = 40
5
∴ A = Rs. 8000
5
1 ∴ x = × 40
19. (D) Let 8% of 96 = x of 4
25
x = 50.
8
⇒ × 96 = 90 30 30 x
100 25 27. (D) ˙·˙ A = B= × A
100 100 100 100
8
∴ x = × 96 × 25 = 192
20. (A) ˙·˙ 8% of x = 4% of y
100 ∴ (
x = 100 ×
100 90
30 100
×) = 300


8
100
x =
4
100
y
1
28. (C) 33 % =
3 ( 100 1
×
3 100 ) 1
= = 0·33;
3
4
⇒ x =( ) 4 100
100 8
× y=
y
2 15
= 0·26
Clearly 0·35 > 0·33 > 0·26
∴ 20% of x = ( ) 20
100
×x ∴ 0·35 is greatest
29. (C) Let 200 = x% of 300.
( )
=
1 y
5 2
× =
1
10
y

x
× 300 = 200
100
( )
=
1
10
× 100 % of y
∴ x =
200
3
= 66
2
3
= 10% of y
30. (A) Number of students who speak only English
21. (D) ˙·˙ x = 90% of y
= 30% of 60 = 18
90
⇒ x = ×y Number of students who speak Hindi and English
100
= 20% of 60 = 12
Let Z% of x = y
∴ Number of students who speak only Hindi
Z
then, ×x = y = (60 – 30) = 30
100
∴ Number of students who speak Hindi
Z 90
⇒ × y = y = 30 + 12 = 42
100 100
25
100 × 100 31. (C) Let 45 × x = × 900
∴ Z = = 111·1 100
90
25 × 9

22. (C) x% of y + y% of x = ( x
) (
100
×y + )
y
100
×x
0·5
x =
45
85
=5

2 32. (A) ˙·˙ of x =


= xy = 2% of xy 100 100
100
23. (A) Let 31% of x = 46·5 ∴ x = Rs. ( ) 85
0·5
= Rs. 170


31
100
x = 46·5 33. (D) Required per cent = [ 3/100
5/100 ]
× 100 %
46·5 × 100
∴ x =
31
= 150 (
=
3 100
100 5
× )
× 100 %
24. (B) Suppose there are 8x questions were asked apart = 60%
from the 41 questions. Then 34. (D) Let 75% of 480 = x × 15.
37 + 5x 4 75
= 80% = ⇒ × 480 = 15 x
41 + 8x 5 100
⇒ 185 + 25x = 164 + 32x 75 × 480
∴ x = = 24
⇒ 7x = 21 100 × 15
⇒ x = 3
∴ Total no. of questions = 41 + 8 × 3 = 65.
35. (C) Required per cent = ( 30
2 × 10 )
× 100 % = 150%

Quantitative Aptitude Test | 147


100 1 x
36. (B) Number of females = 156800 × = 196000 45. (C) ˙·˙ × 1206 = × 134
80 3 100
7 402 × 100
∴ Number of males = × 196000 = 171500 ∴ x = = 300
8 134
Total population = 196000 + 171500 46. (B) Let salary be Rs. x, then
= 367500
x – 15% of x – 30% of 85% of x = 2380
37. (A) Let x% of y = y% of z
x y 15x 30 × 85 × x
⇒ ×y = ×z ⇒ x– – = 2380
100 100 100 100 × 100
⇒ 200x – 30x – 51x = 2380 × 200
∴ z =( xy 100
100 y
× ) =x
⇒ 119x = 2380 × 200
38. (A) Let 12·5% of 192 = 50% of x, then 2380 × 200
12·5 50 ∴ x = = 4000
⇒ × 192 = ×x 119
100 100 47. (B) Let his income be Rs. x, then
12·5 × 192 x – 5% of x – 20% of 95% of x = 1919
∴ x = = 48
50
x 20 × 95 × x
1 ⇒ x– – = 1919
39. (C) Let 37 % of x = 900 20 100 × 100
2
75 × x x 19x
⇒ = 900 ⇒ x– – = 1919
2 × 100 20 100
900 × 2 × 100 ⇒ 100x – 5x – 19x = 191900
∴ x = = 2400
75 191900
∴ x = = 2525
So,
1
62 % of x =
2 ( 125 1
2 100
× )
× 2400 = 1500
48. (D) Decreases by 2·25%.
76

40. (A) Let x – 6% of x = xz. 49. (C) ˙·˙ x – [40% of x + 25% of x + 15% of x + 5%
94 1 of x]
⇒ x ×x = z
100 = 1305
∴ z = 0·94 ⇒ x – 85% of x = 1305
41. (
(B) It is
30
100
4
5 )
× 80 – × 25 = 24 – 20 = 4 ⇒ 15% of x = 1305
1305 × 100
42. (D) Let the number of students appearing for ∴ x = = 8700
examination in the year 1998 in the states A, B and C 15
be 3x, 5x and 6x respectively. 1
50. (B) ˙·˙ 137 % of x = 33
120 2
3x × 275 1
100 1 ⇒ × x = 33
According to the question = = 2 100
120 2
6x × 33 × 2 × 100
100 ∴ x = = 24
1 1 275
⇒ = 1
2 2 51. (C) ˙·˙ 72 % of x = 87
43. (C) Let original price = Rs. 100. 2
Then, new price = Rs. 90. 145 1
⇒ × x = 87
˙·˙ Increase on Rs. 90 = Rs. 10 2 100
87 × 2 × 100
Required increase% = ( 10
90 )
× 100 % = 11 %
1
9
∴ x =
145
= 120
52. (C) ˙·˙ (25% of x) – (18% of 650) = 19
44. (C) Let the business value changes from x to y. Then
4% of x = 5% of y
4 5

x
4 (
= 19 +
18
100 )
× 650
⇒ ×x = ×y = 136
100 100
4 ∴ x = (136 × 4) = 544
⇒ y = x
5 53. (B) ˙·˙ 96% of x = 23040

(
∴ Change in business = x – x = x ) 4
5
1
5 ∴ x =
23040 × 100
96
= 24000
Percentage slump in business 54. (C) ˙·˙ 95% of x = 9595
9595 × 100
=( 1
5
1
)
x × x × 100 % = 20% ∴ x=
95
= 10100

Quantitative Aptitude Test | 148


55. (C) ˙·˙ 87
1
2
% of x = 3500 Expenditure on education = Rs. (100
15
× 90)
175 1 = Rs. 13·50
⇒ × × x = 3500
2 100 Balance = Rs. 76·50
3500 × 2 × 100
∴ x =
175
= 4000 Expenditure on clothes = Rs. (
10
100
× 76·50 )
56. (A) Let total number of students be x = Rs. 7·65
Then 30% of x = 504 Balance now = Rs. 68·85
504 × 100 If balance is Rs. 68·85, Salary = Rs. 100
⇒ x = = 1680
30
∴ Number of boys = (1680 – 504) = 1176 If balance is Rs. 1377, Salary = Rs. (100
68·85
× 1377 )
57. (D) ˙·˙ 60% of x = 420 = Rs. 2000
420 × 100 66. (D) Let third number be x
∴ x = = 700
60 7x
58. (B) ˙·˙ 12% of x = 69 Then, first number = 70% of x =
10
69 × 100 63 x
⇒ x = = 575 kg and, second number = 63% of x =
12 100

⎯√⎯⎯
3·6 7x 63x 7x
59. (C) × 40 = √
⎯⎯⎯
1·44 = 1·2 Difference = – =
100 10 100 100
60. (D) 35% of x = 420 ⎛ 7x 10 ⎞
∴ Required percentage = ⎜ × × 100⎟ %
420 × 100 ⎝ 100 7x ⎠
∴ x = = 1200
35
= 10%
61. (D) ˙·˙ 75 + 75% of x = x
67. (B) Boys = 900,
3
⇒ x – x = 75 Girls = 1100
4
Passed = (32% of 900) + (38% of 1100)
1
⇒ x = 75 = (288 + 418) = 706
4
Failed = (2000 – 706) = 1294
∴ x = (75 × 4) = 300
62. (B) ˙·˙ 5% A = 15% B and 10% B = 20% C
A 3B B C
Failed % = ( 1294
2000 )
× 100 % = 64·7%

∴ = and = 68. (B) ˙·˙ 20% of x + 30 = 32% of x – 42


20 20 10 5
⇒ 12% of x = 72
⇒ B = 2C
72 × 100
A 3 3 So x = = 600
⇒ = × 2C = C 12
20 20 10
Pass marks = 20% of 600 + 30 = 150
3


=
10
× 2000 = 600
A = (600 × 20) = 12000,
Pass percentage = ( 150
600 )
× 100 % = 25%
69. (A) ˙·˙ (62% of x – 38% of x) = 144
B = (2 × 2000) = 4000 ⇒ 24% of x = 144
∴ A + B + C = (12000 + 4000 + 2000) = 18000 144 × 100
63. (A) ˙·˙ 30% of 180 + x% of 150 ∴ x = = 600
24
= 50% of (180 + 150) 70. (D) n (A) = 75% of 600 = 450
x n (B) = 45% of 600 = 270 and n (A ∪ B) = 600
⇒ 54 + × 150 = 165
100 ˙·˙ n (A ∩ B) = n (A) + n (B) – n (A ∪ B)
3x = (450 + 270 – 600) = 120
⇒ = 111
2 71. (C) ˙·˙ 200% of x = 90
111 × 2 90 × 100
∴ x = = 74 ⇒ x = = 45
3 200
64. (B) ˙·˙


36% of x = (113 + 85)

x =
100 × 198
= 550
∴ 80% of x = ( 80
100 )
× 45 = 36

65.
36
(A) Suppose that his salary = Rs. 100
72. (C) Reduction in consumption = ( 20
120 )
× 100 %
House Rent = Rs. 10 50
= %
So Balance = Rs. 90 3
Quantitative Aptitude Test | 149

Reduction in consumption
Original consumption
= (
50 1
×
3 100
=
1
6 ) =
(200 + 7)3
80000
= 1:6 8869743
= = 110·8718
73. (D) Let tax = Rs. 100 and consumption = 100 units 80000
Original expenditure = Rs. (100 × 100) Required increase %
= Rs. 10000 = (110·8 – 100)%
New expenditure = Rs. (120 × 80) = 10·8%.
= Rs. 9600 82. (D) Let original wages = Rs. 100
Reduced wages = Rs. 50
∴ Decrease in expenditure = ( 400
10000
× 100 % ) Increased wages = 150% of Rs. 50
= 4% = (
150
100 )
× 50 = Rs. 75
74. (B) Alcohol in 15 litres = ( 20
100 )
× 15 litres ∴ Loss = 25%
= 3 litres 83. (A) p = 6q. Thus q is less than p by 5q
Now, alcohol in 18 litres = 3 litres 6q – q
∴ q is less than p by = × 100%
∴ Strength of alcohol = ( 3
18 )
× 100 % = 16 %
2
3
6q
⎛ 5q ⎞ 1
75. (D) Let price = Rs. 100, = ⎜ 6q × 100⎟ % = 83 3 %
⎝ ⎠
Sale = 100
Then sale value = Rs. (100 × 100) = Rs. 10000 84. (C) B’s salary is less than A’s by (130
30
× 100)%
New sale value = Rs. (70 × 120) = Rs. 8400 1
= 23 %.
Decrease% = ( 1600
10000 )
× 100 % = 16%
85. (D) Let income = Rs. 100
13

76. (C) Required number = 800 × 1 + ( 15 2


100 ) Then expenditure = Rs. 75 and saving = Rs. 25
New income = Rs. 120

(
= 800 × ×
23 23
20 20 ) New expenditure = 110% of Rs. 75 = Rs.
165
2
= 1058
77. (B) Population after 2 years (
New saving = Rs. 120 –
165
2 )= Rs.
75
2

(
= 8000 1 +
10
100 )( 1+
20
100 ) Increase in saving = Rs. ( 75
2 )
– 25 = Rs.
25
2

(
= 8000 × ×
11 6
10 5 )= 10560 ∴ Increase% = ( 25 1
)
× × 100 % = 50%
2 25

78. (B) ˙·˙ 540 = x 1 + ( )


20 2
100
86. (A) Let original price = Rs. 100
Reduced price = Rs. 75
∴ (
x = 540 × ×
5 5
6 6) = 375 cm Increase on Rs. 75 = Rs. 25

79. (B) Value of machine 2 years hence Increase on Rs. 100 = ( 25


75 ) 1
× 100 % = 33 %
3

[
= Rs. 4000 × 1 – ( 10 2
100 )] 87.
x
(B) Let the original fraction be y

(
= Rs. 4000 × ×
9 9
10 10 ) Now,
120% of x
90% of y
=
16
21
= Rs. 3240 4 x 16
⇒ × =
80. (D) Net growth on 1000 = 21 3 y 21

Net growth on 100 = ( 21


1000
× 100 = 2·1%) ∴
x
( 16 3
)
y = 21 × 4 = 7
4

81. (D) Increase in 3 years over 100 88. (B) ˙·˙ Milk = 90% of 40 = 36 litres and water = 4
litres
( )
7 3
= 100 × 1 +
2 × 100 4+x
⇒ × 100 = 20
(40 + x)
(
= 100 ×
207 207 207
× ×
200 200 200 ) ⇒ 20 (40 + x) = 100 (4 + x)

Quantitative Aptitude Test | 150


⇒ 80x =400 2100
∴ x =
∴ x =5 litres. 10·5 × 79
89. (B) Let original price
⇒ Increased price
=
=
Rs. 100
Rs. 160
Reduced rate = Rs. ( 79
×
2100
100 10·5 × 79 ) per kg

⇒ Decrease on Rs. 160 =Rs. 60 = Rs. 2 per kg.


96. (D) Let radius of circle
∴ Decrease on Rs. 100 = (
60
160
× 100 % ) = 100 m
1 New radius = 101 m
= 37 % Original area = [π × (100)2 ]m2
2
90. (C) Suppose boys = 3x and girls = 2x New area = [π × (101)2 ]m2
π × {(101)2 – (100) 2 }
Not adults = (80% of 3x) + (75% of 2x) Increase% = [ π × 100 × 100 ]
× 100 %
= (
12x 3x
5
+
2 )
=
39x
10 =
201
% = 2·01%
100
⎛ 39x 1 ⎞
∴ Required percentage = ⎜ × × 100⎟ % 97. (D) Let length of square
⎝ 10 5x ⎠ = 100 m and breadth of square
= 78% = 100 m
91. (C) Let the total number of students be 100 Area = (100 × 100)m2 = 10000 m2
Then, n (A ∩ B) = n (A) + n (B) – n (A ∪ B) New length = 130 m
= (72 + 44 – 100)% = 16% And New breadth = 120 m
Now, ˙·˙ 16% of x = 40 New area of rectangle
16 = (130 × 120)m2 = 15600 m2
⇒ × x = 40
100

∴ x =
100 × 40
= 250
Increase% = ( 5600
10000 )
× 100 %
16 = 56%
92. (B) Let actual side = 100 cm 98. (C) Surface area sphere = 4π × (10) 2 = (400π) cm 2
⇒ Measured length = 105 cm 4
⇒ Error in area = (105) 2 – (100) 2 Volume sphere = π × (10)3
3
= (105 + 100) (105 – 100) 4000 π
= 1025
= ( 3 ) cm3

Error% = ( 1025
10000 )
× 100 % = 10·25% ∴ Required percentage = 400π × ( 3
4000π )
× 100 %

93. (D) Salt in 6 litre = 5% of 6 = 0·30 = 30%


Salt in new solution = ( 0·30
5 )
× 100 % = 6% 99. (A) Let length of a rectangle
= 100 m,
94. (D) Let side of square Breadth = 100 m
= 100 cm New length = 160 m,
Area = (100 × 100) cm2 = 10000 cm2 New breadth = x metres
New area = (130 × 130) cm2 = 16900 cm2 Then, 160 × x = 100 × 100
Increase in area = ( 6900
10000 )
× 100 % = 69% ⇒ x =
100 × 100 125
160
=
2
95. (A) Let original rate = Rs. x per kg
New rate = 79% of Rs. x per kg (
Decrease in breadth = 100 –
125
2 ) %

= Rs.( ) 79x
100
per kg = 37 %
1
2
100 100 100. (B) Reduction in consumption
˙·˙ – x = 10·5
79x
100
= ( m
(100 + m) )
× 100 %


10000 100
79x
– x = 10·5 = ( 25
125 )
× 100 %

⇒ 10000 – 7900 = 10·5 × 79x = 20%.

Quantitative Aptitude Test | 151


Profit and Loss
Important Points/Facts
1. Cost price (C.P.)—It is that
= Rs. P (1 + 100
20
) = x (1 – 100
10
)
= Rs. P (
100)
price at which a particular article or 120 90
= Rs. x
item is bought. 100
2. Selling price (S. P.)—It is
that price at which a particular article
or item is sold.
Since a sale to B for Rs. P (120
100)
˙.˙ Transport expenses (overhead)
= Rs. 150
therefore cost price for B is ∴ Cost price of that article on
3. Profit or gain—If the selling
price of an article is more than its Rs. P ( ) 120
100
.
reaching Delhi
cost price, then there is profit or gain.
Example—If an article is bought Now for 25% gain, selling price
= Rs. ( 90
100 )
x + 150
of the cycle But he earns gain of Rs. 240 on
for Rs. 1000, and sold for Rs. 1200,
then For B = P ( )[ 120
100
1+ ]
25
100
selling that article of Delhi.
Profit = Rs. (1200 – 1000)
= Rs. 200 i.e., cost price of the cycle for C
∴x – ( 90
100 )
x + 150 = 240

Hence, Profit = S. P. – C.P.


4. Loss—If the cost price of an
= P( )( )
120
100
125
100
x–
90
100x
= 240 + 150

article is more than its selling price, But according to the question, 10
= 390
then there is loss. cost price of the cycle for 100x
Example—If an article is bought C = Rs. 1500 x = Rs. 3900
for Rs. 1000 and sold for Rs. 800
∴ Loss = Rs. (1000 – 800) ∴P ( )( )
120
100
125
100
= 1500 Q. 4. When a man sells an
article to other man for A% gain
= Rs. 200 or loss. The other person sells the
1500 × 100 × 100
Hence Loss = C.P. – S.P. P = same article to third man for B%
120 × 125 gain or loss. If third man gets that
Examples P = Rs. 1000 article for Rs. x, then the cost price
Hence cost price of cycle paid by of that article for the first man
Q. 1. Mohan buys a chair for A = Rs. 1000 100 × 100x
Rs. 150 and sells it for Rs. 160. =
(100 ± A) (100 ± B)
Find his profit or loss per cent.
Short Method Solution : For memory
Solution :
Required amount x
Here C.P. of chair = Rs. 150
x C. P. =
and S.P. of chair = Rs. 160 = A'·B'
A′ B′
Profit = Rs. (160 – 150) 100 ± A
= Rs. 10 1500 where A′ =
= 100
Profit per cent =
Profit × 100
Cost Price
(
100 + 20
100 )(100 + 25
100 ) and B′ =
100 ± B
100
10 × 100 1500 × 100 × 100
= = Q. 5. A retailer purchases
150 120 × 125
goods from a wholeseller for Rs.
20 2 = Rs. 1000
= =6 % 1000. The retailer bears additional
3 3 expense of Rs. 200 on carriage of
Q. 2. A sold a cycle to B for Q. 3. A trader goes from Delhi goods by his servant and he sold
20% gain and B sold to C for 25% to Lucknow to purchase an article the goods to the consumer for
gain. If C paid Rs. 1500 for that whose price at Lucknow is 10% Rs. 1500. Find the per cent gain.
then how much amount was paid less than in Delhi. The transport
Solution : Here Rs. 200 will be
by A for that cycle ? expense is Rs. 150 and he earns
added as overhead expenses in the
Solution : Let A bought the cycle gain of Rs. 240, on selling that
cost price of goods
for Rs. P. article at Delhi. Then how much
that article cost at Delhi ? ∴ C.P. of goods = 1000 + 200
For 20% gain, selling price of the
cycle for A Solution : Let value of article at = Rs. 1200
Delhi = Rs. x ∴
(
= Cost price 1 +
Gain
100 ) ∴ Value (C.P.) at Lucknow
Gain = 1500 – 1200
= Rs. 300
Quantitative Aptitude Test | 152
∴ Per cent of gain Hence, difference between the cost of
Gain × 100 one table and that of one chair ?
=
C.P.
Loss% = (
1·70
10·20
× 100 % ) (A) Rs. 75
300 × 100 2 (B) Rs. 35
= = 16 %
1200 3 (C) Cannot be determined
= 25% (D) 125
Q. 6. A sells a manufactured Exercise 08. Profit after selling a commodity
table to a wholeseller at a profit of 01. By selling an article for Rs. 100, for Rs. 425 is same as loss after
10% the wholeseller sells that table one gains Rs. 10. Then the gain selling it for Rs. 355. The cost of
to a retailer at a profit of 15% and per cent is— the commodity is—
retailer sells it to a customer at a
(A) 9% (A) Rs. 385 (B) Rs. 390
profit of 25% for Rs. 1265. What is
the cost of production of the table (B) 10% (C) Rs. 395 (D) Rs. 400
for manufacturer ? 1
(C) 11 % 09. By selling an article for Rs. 100
Solution : 9 one loses Rs. 10. Then, the loss
x (D) None of these per cent is—
Cost of table =
A– B– C– 02. There would be 10% loss if a toy 1
(A) 11 %
1265 × 100 × 100 × 100 is sold at Rs. 10·80 per piece. At 9
= what price should it be sold to
110 × 115 × 125 1
earn a profit of 20% ? (B) 9 %
= Rs. 800 11
(A) Rs. 12
Q. 7. A bought a horse for (C) 10%
(B) Rs. 12·96
Rs. 9000. He sold this horse to B at (D) None of these
10% loss of cost price. Again B (C) Rs. 14·40
sold the horse to A at 10% profit. (D) None of these 10. A man buys 10 articles for Rs. 8
How much gain or loss happen to and sells them at the rate of
03. By selling an article for Rs. 1·25 per article. His gain is—
A in total transaction ? Rs. 247·50 we get a profit of
Solution : 1 (A) 20% (B) 50%
12 %. The cost of the article 1 1
Profit or loss to 2 (C) 19 % (D) 56 %
is— 2 4
A = 9000 × (
90
100
1– )
110
100 (A) Rs. 210 (B) Rs. 220 11. A man sold a radio for Rs. 1980
(C) Rs. 224 (D) Rs. 225 and gained 10%. The radio was
– 10
= 90 × 90 × bought for—
100 04. The selling price of 12 articles is
= Rs. (– 810) equal to the cost price of 15 (A) Rs. 1782
˙.˙ Result is negative (– ve) articles. The gain per cent is— (B) Rs. 1800
∴ Loss to A = Rs. 810 2 (C) Rs. 2178
(A) 6 % (B) 20%
Q. 8. Find gain or loss per cent 3 (D) None of these
when : (C) 25% (D) 80% 12. What per cent of selling price
(a) C.P. = Rs. 9·50 05. If the cost price of 15 tables be would be 34% of cost price if
and S.P. = Rs. 11·40 equal to the selling price of 20 gross profit is 26% of the selling
tables, the loss per cent is— price ?
(b) C.P. = Rs. 10·20
(A) 20% (B) 30% (A) 17·16 (B) 74·00
and S.P. = Rs. 8·50
(C) 25·16 (D) 88·40
Solution : (a) C.P. = Rs. 9·50, (C) 25% (D) 37·5%
13. Alok bought 25 kg of rice at the
S. P. = Rs. 11·40 06. An article when sold for Rs. 200 rate of Rs. 6·00 per kg and 35 kg
∴ Gain = Rs. (11·40 – 9·50) fetches 25 per cent profit. What of rice at the rate of Rs. 7·00 per
would be the percentage profit/
= Rs. 1·90 kg. He mixed the two and sold
loss if 6 such articles are sold for
Hence, the mixture at the rate of Rs. 6·75
Rs. 1056 ? per kg. What was his gain or loss
Gain% = ( 1·90
9·50 )
× 100 % (A) 10 per cent loss
(B) 10 per cent profit
in this transaction ?
(A) Rs. 16·00 gain
= 20% (C) 5 per cent loss (B) Rs. 16·00 loss
(b) C. P. = Rs. 10·20, (D) 5 per cent profit (C) Rs. 20·00 gain
S.P. = Rs. 8·50 (D) None of these
07. Two chairs and three tables cost
∴ Loss = (Rs. 10·20 – 8·50) Rs. 1025 and three chairs and two 14. An item costing Rs. 200 is being
= Rs. 1·70 tables cost Rs. 1100. What is the sold at 10% loss. If the price is

Quantitative Aptitude Test | 153


further reduced by 5%, the sell- 21. While selling a watch a shop- 29. If I purchased 11 books for
ing price will be— keeper gives a discount of 5%. If Rs. 10 and sold all the books at
(A) Rs. 179 (B) Rs. 175 he gives a discount of 7%, he the rate of 10 books for Rs. 11,
(C) Rs. 171 (D) Rs. 170 earns Rs. 15 less as profit. What the profit per cent is—
is the marked price of the watch ? (A) 10% (B) 11%
15. A buys oranges at Rs. 2 for 3 (A) Rs. 697·50 (C) 21% (D) 100%
oranges and sells them at a rupee
each. To make a profit of Rs. 10, (B) Rs. 712·50 30. By selling an article for Rs. 144,
he must sell— (C) Rs. 787·50 1
a man loses of his outlay. By
(A) 10 oranges (D) None of these 7
selling it for Rs. 168, his gain or
(B) 20 oranges 22. A sells a horse to B for Rs. 4860, loss per cent is—
(C) 30 oranges thereby losing 19 per cent, B sells
(A) 20% loss
(D) 40 oranges it to C at a price which would
have given A, 17 per cent profit. (B) 20% gain
16. The cost of 2 T.V. sets and a Find B’s gain— 1
(C) 4 % gain
radio is Rs. 7000, while 2 radios 6
(A) Rs. 2160 (B) Rs. 2610
and one T.V. set together cost (D) None of these
Rs. 4250. The cost of a T.V. set (C) Rs. 1260 (D) Rs. 2260
31. A shopkeeper bought 150 cal-
is— 23. The cost price of an article, culators at the rate of Rs. 250 per
(A) Rs. 3000 which on being sold at a gain of calculator. He spent Rs. 2500 on
(B) Rs. 3160 12% yields Rs. 6 more than when transportation and packing. If the
it is sold at a loss of 12% is—
(C) Rs. 3240 marked price of calculator is
(A) Rs. 30 (B) Rs. 25 Rs. 320 per calculator and the
(D) None of these
(C) Rs. 20 (D) Rs. 24 shopkeeper gives a discount of
17. If books bought at prices ranging 5% on the marked price then
from Rs. 200 to Rs. 350 are sold 24. When the price of pressure cooker
was increased by 15%, its sale what will be the percentage profit
at prices ranging from Rs. 300 to gained by the shopkeeper ?
Rs. 425, what is the greatest fell down by 15%. The effect on
the money receipt was— (A) 20% (B) 14%
possible profit that might be
(A) No effect (C) 15% (D) 16%
made in selling 8 books ?
(A) Rs. 400 (B) 15% decrease 32. The loss incurred on selling an
(C) 7·5% increase article for Rs. 270 is as much as
(B) Rs. 600
the profit made after selling it at
(C) Cannot be determined (D) 2·25% decrease 10% profit. The C.P. of the
(D) None of these 25. Subhash purchased a tape- article is—
18. A man purchased a watch for 9 (A) Rs. 90 (B) Rs. 110
recorder at th of its selling
Rs. 400 and sold it at a gain of 10 (C) Rs. 363 (D) Rs. 300
20% of the selling price. The price and sold it at 8% more than 33. There would be 10% loss if rice
selling price of the watch is— its selling price. His gain is— is sold at Rs. 5·40 per kg. At what
(A) Rs. 300 (B) Rs. 320 (A) 9% (B) 10% price per kg should it be sold to
(C) Rs. 440 (D) Rs. 500 (C) 18% (D) 20% earn a profit of 20% ?
26. By selling 100 bananas, fruit (A) Rs. 7·20 (B) Rs. 7·02
19. By selling 8 dozen of pencils, a
shopkeeper gains the selling price seller gains the selling price of 20 (C) Rs. 6·48 (D) Rs. 6
of 1 dozen pencils. His gain per bananas. His gain per cent is— 34. A retailer purchases a sewing
cent is— (A) 10% (B) 15% machine at discount of 15% and
(A) 12·5% (C) 20% (D) 25% sells it for Rs. 1955. In the
(B) 87·5% bargain he makes a profit of 15%.
27. A fruitseller buys lemons at 2 for How much is the discount which
2 a rupee and sells them at five for
(C) 14 % he got from the wholesale ?
7 three rupees. His gain per cent (A) Rs. 270
(D) None of these is—
(B) Rs. 290
20. By selling 36 oranges, a vender (A) 10%
(C) Rs. 300
loses the selling price of 4 (B) 15%
(D) None of these
oranges. His loss per cent is— (C) 20%
(D) None of these 35. A shopkeeper earns a profit of
1
(A) 12 % 12% after selling a book at 10%
2 28. By selling a book for Rs. 10, the discount on the printed price. The
1 publisher loses (1/11) of what it ratio of the cost price and printed
(B) 11 %
9 costs him. His cost price is— price of the book is—
(C) 10% (A) Rs. 9 (B) Rs. 10 (A) 45 : 56
(D) None of these (C) Rs. 11 (D) Rs. 12 (B) 50 : 61

Quantitative Aptitude Test | 154


(C) 99 : 125 4%. If C paid Rs. 91 for it, then 50. An article is sold at a certain price.
(D) None of these the price paid by A is— 2
By selling it at of that price,
(A) Rs. 82·81 3
36. A discount series of 10%, 20%
(B) Rs. 83 one loses 10%. The gain per cent
and 40% is equal to a single dis-
(C) Rs. 83·33 at original price is—
count of—
(D) None of these 1
(A) 50% (B) 56·80% (A) 20% (B) 33 %
3
(C) 70% (D) 70·28% 44. When the price of a toy was (C) 35% (D) 40%
37. By selling 12 oranges for one increased by 20%, the number of
toys sold was decreased by 15%. 51. A man sold two houses for
rupee, a man loses 20%. How
What was the effect on the sales Rs. 7·81 lakhs each. On one he
many for a rupee should he sell
of the shop ? gained 5% and on the other he
to get a gain of 20% ?
(A) 4% increase lost 5%. What per cent is the
(A) 5 (B) 8 effect of the sale on the whole ?
(C) 10 (D) 15 (B) 4% decrease
(A) 0·25% loss
(C) 2% increase
38. A dishonest dealer professes to (B) 0·25% gain
(D) 2% decrease
sell his goods at cost price. But (C) 25% loss
he uses a false weight and thus 45. Ram bought 4 dozen apples at (D) 25% gain
18 Rs. 12 per dozen and 2 dozen
gains 6 % . For a kg, he uses 52. Profit after selling commodity
47 apples at Rs. 16 per dozen. He
a weight of— sold all of them to earn 20%. At for Rs. 425 is same as loss after
(A) 953 gms (B) 940 gms what price per dozen did he sell selling it for Rs. 355. The cost of
the apples ? the commodity is—
(C) 960 gms (D) 947 gms
(A) Rs. 14·40 (B) Rs. 16·00 (A) Rs. 385 (B) Rs. 390
39. The C.P. of an article is 40% of (C) Rs. 395 (D) Rs. 400
(C) Rs. 16·80 (D) Rs. 19·20
the S.P. The per cent that the S.P.
is of C.P. is— 46. A owns a house worth Rs. 10000. 53. A merchant sold his goods for
(A) 40 (B) 60 He sells it to B at a profit of 10% Rs. 75 at a profit per cent equal
based on the worth of the house to C.P. The C.P. was—
(C) 240 (D) 250
B sells the house back to A at a (A) Rs. 40 (B) Rs. 50
40. Toffees are bought at the rate of loss of 10%. In this transaction A (C) Rs. 60 (D) Rs. 70
8 for a rupee. To gain 60% they gets—
must be sold at— 54. A horse and a cow were sold for
(A) No profit no loss
(A) 6 for a rupee Rs. 12000 each. The horse was
(B) Profit of Rs. 1000 sold at a loss of 20% and the
(B) 5 for a rupee (C) Profit of Rs. 1100 cow at a gain of 20%. The entire
(C) 9 for Rs. 2 (D) Profit of Rs. 2000 transaction resulted in—
(D) 24 for Rs. 5 (A) No loss or gain
47. A dealer professing to sell at cost
41. Tarun bought a T.V. with 20% price uses a 900 gms weight for (B) Loss of Rs. 1000
discount on the labelled price a kilogram. His gain per cent is— (C) Gain of Rs. 1000
had he bought it with 25% (A) 9 (B) 10 (D) Gain of Rs. 2000
discount he would have saved 1 55. By selling an article for Rs. 144,
Rs. 500. At what price did he (C) 11 (D) 11
9 a man gained such that the per-
buy the T.V. ?
(A) Rs. 5000 48. When the price of fans was centage gain equals the cost
reduced by 20%, the number of price. The C.P. of the article is—
(B) Rs. 10000
(C) Rs. 12000 fans sold increased by 40% what (A) Rs. 60 (B) Rs. 64
(D) None of these was the effect on the sales in (C) Rs. 72 (D) Rs. 80
rupees ?
42. Jimmy bought paper sheets for 56. By selling 45 oranges for Rs. 40,
(A) 12% increase a man loss 20%. How many
Rs. 7200 and spent Rs. 200 on
transport paying Rs. 600 he had (B) 12% decrease should he sell for Rs. 24 so as to
330 boxes made, which he sold (C) 30% increase gain 20% in the transaction ?
at Rs. 28 each. What is his profit (D) 40% increase (A) 16 (B) 18
percentage ? (C) 20 (D) 22
49. A dealer sold two T.V. sets for
(A) 15·5 Rs. 3700 each. On one he gained 57. A man sells a car to his friend at
(B) 40 10% and on the other he lost 10% loss. If the friend sells it for
(C) 60 10%. The dealer’s loss or gain Rs. 54000 and gains 20%, the
(D) None of these per cent is— original C.P. of the car was—
43. A sold a watch at a gain of 5% to (A) 0% (B) 0·1% (A) Rs. 25000
B and B sold it to C at a gain of (C) 1% gain (D) 1% loss (B) Rs. 37500

Quantitative Aptitude Test | 155


(C) Rs. 50000 bought it for 20% less and sold it been gained. The C.P. of the
(D) Rs. 60000 for Rs. 55 more. He would have bicycle is—
had a profit of 40%. The C.P. of (A) Rs. 350 (B) Rs. 400
58. If two mixers and one T.V. cost the article is—
Rs. 7000, while two T.V.s and (C) Rs. 500 (D) Rs. 600
(A) Rs. 200
one mixer cost Rs. 9800. The 72. A radio dealer sold radio at a loss
value of one T.V. is— (B) Rs. 225 of 2·5%. Had he sold it for
(A) Rs. 2800 (B) Rs. 2100 (C) Rs. 250 Rs. 100 more, he would have
(C) Rs. 4200 (D) Rs. 8400 (D) None of these 1
gained 7 %. In order to gain
66. A dealer sells a radio at a gain of 2
59. A man purchased sugar worth of 1
Rs. 400. He sold (3/4)th at a loss 10%. If he had bought it at 10% 12 % he should sell it for—
less and sold it for Rs. 132 less, 2
of 10% and the remainder at a
gain of 10%. On the whole, he he would have still gained 10%. (A) Rs. 850 (B) Rs. 925
gets— The C.P. of the radio is— (C) Rs. 1080 (D) Rs. 1125
(A) A loss of 5% (A) Rs. 1188 73. A man sells an article at a gain
1 (B) Rs. 1200 of 15%. If he had bought it at
(B) A gain of 5 % (C) Rs. 1320 10% less and sold it for Rs. 4
2
1 (D) None of these less, he would have gained 25%.
(C) A loss of 5 % The C.P. of the article is—
19 67. A man sold an article for Rs. 75
and lost something. Had he sold (A) Rs. 140 (B) Rs. 150
5
(D) A loss of 5 % it for Rs. 96, his gain would have (C) Rs. 160 (D) Rs. 180
19
been double the former loss. The 74. 6% more is gained by selling a
60. Bhajan Singh purchased 120 C.P. of the article is—
reams of paper at Rs. 80 per ream. radio for Rs. 475 than by selling
(A) Rs. 81 (B) Rs. 82 for Rs. 451. The C.P. of the radio
He spent Rs. 280 on transporta-
tion paid octroi at the rate of 40 (C) Rs. 83 (D) Rs. 85·5 is—
paise per ream and paid Rs. 72 to 68. Due to an increase of 30% in the (A) Rs. 400
the coolie. If he wants to have a price of eggs, 3 eggs less are (B) Rs. 434
gain of 8%. What must be the available for Rs. 7·80. The present (C) Rs. 440·50
selling price per ream ? rate of eggs per dozen is— (D) None of these
(A) Rs. 86 (B) Rs. 87·48 (A) Rs. 8·64
(C) Rs. 89 (D) Rs. 90 75. Rahim sells a chair at a gain of
(B) Rs. 8·88 1
61. The cost price of an article, which (C) Rs. 9·36 7 %. If he had bought it at
2
on being sold at a gain of 10% (D) None of these 1
yields Rs. 6 more than when it is 12 % less and sold it for Rs. 5
69. A man sells two horses for 2
sold at a loss of 10% is—
Rs. 4000 each, neither losing nor more. He would have gained
(A) Rs. 30 (B) Rs. 25 30%. The C.P. of the chair is—
gaining in the deal. If he sold
(C) Rs. 24 (D) Rs. 20 one horse at a gain of 25% the (A) Rs. 72 (B) Rs. 80
62. A shopkeeper sells (3/4)th of its other horse is sold at a loss of— (C) Rs. 88 (D) Rs. 96
articles at a gain of 20% and the 2
remaining at C.P. His real gain (A) 16 % 76. At what price must Kantilal sell
3
in the transaction is— a mixture of 80 kg sugar at Rs.
2
(A) 10% (B) 15% (B) 18 % 6·75 per kg with 120 kg at Rs. 8
9 per kg to gain 20% ?
(C) 20% (D) 25%
(C) 25%
63. A man gains 10% by selling an (A) Rs. 7·50 per kg
(D) None of these
article for a certain price. If he (B) Rs. 8·20 per kg
sells it at double the price, the 70. A grocer sells rice at a profit of (C) Rs. 8·85 per kg
profit made is— 10% and uses weights which are
20% less than the market weight. (D) Rs. 8·76 per kg
(A) 20% (B) 60%
(C) 100% (D) 120% The total gain earned by him will 77. The per cent profit when an
be— article is sold for Rs. 78 is twice
64. If an article is sold at a gain of
(A) 30% as when it is sold for Rs. 69. The
6% instead of at a loss of 6%
(B) 35% C.P. of the article is—
then the seller gets Rs. 6 more.
The C.P. of the article is— (C) 37·5% (A) Rs. 49 (B) Rs. 51
(A) Rs. 50 (B) Rs. 94 (D) None of these (C) Rs. 57 (D) Rs. 60
(C) Rs. 100 (D) Rs. 106
71. A bicycle is sold at a gain of 78. A trader by means of a false
65. A person bought an article and 16%. If it had been sold for balance defrauds to the extent of
sold it at a loss of 10%. If he had Rs. 20 more, 20% would have 8% in buying goods and also

Quantitative Aptitude Test | 156


defrauds to 8% in selling. His
gain per cent is— (C) Rs. (10000 –P)
10000
2
(B) 22%
7
(C) 21 %
(A) 15·48% ⎛√ 1 – P 2 ⎞⎟
⎯⎯⎯⎯ 8
(D) Rs. ⎜ (D) None of these
(B) 16% ⎝ 100 ⎠
(C) 16·64% 87. A man purchases an electric
83. A dealer marks his goods 20% heater whose printed price is
(D) None of these above cost price. He then allows Rs. 160. If he received two
some discount on it and makes a successive discounts of 20% and
79. Kabir buys an article with 25%
profit of 8%. The rate of discount 10% he paid—
discount on its marked price. He
is—
makes a profit of 10% by selling (A) Rs. 112
it at Rs. 660. The marked price (A) 12% (B) 10%
(B) Rs. 129·60
is— (C) 6% (D) 4%
(C) Rs. 119·60
(A) Rs. 600 (B) Rs. 700 84. The marked price of an article is (D) Rs. 115·20
(C) Rs. 800 (D) Rs. 685 Rs. 480. The shopkeeper allows
a discount of 10% and gains 8%. 88. A trader lists his articles 20%
80. A discount series of 10%, 20% If no discount is allowed, his gain above C.P. and allows a discount
and 40% is equal to a single per cent would be— of 10% on cash payment. His
discount of— gain per cent is—
(A) 18% (B) 20%
(A) 10% (B) 8%
(A) 50% (B) 56·8% (C) 18·5% (D) 20·5%
(C) 6% (D) 5%
(C) 60% (D) 70·28% 85. The ratio of the prices of three
different types of cars is 4 : 5 : 7. 89. The difference between a dis-
81. An umbrella marked at Rs. 80 is If the difference between the count of 40% on Rs. 500 and two
sold for Rs. 68. The rate of costliest and the cheapest cars is successive discounts of 36% and
discount is— Rs. 60000 the price of the car of 4% on the same amount is—
(A) 15% (B) 12% modest price is— (A) 0 (B) Rs. 2
11 (A) Rs. 80,000 (C) Rs. 1·93 (D) Rs. 7·20
(C) 17 % (D) 20% (B) Rs. 1,00,000
17 90. The marked price is 10% higher
(C) Rs. 1,40,000
82. The price of an article was than the cost price. A discount of
(D) Rs. 1,20,000
increased by P%. Later the new 10% is given on the marked
price was decreased by P%. If the 86. A tradesman marks his goods price. In this kind of sale, the
latest price was Re. 1, the origi- 30% more than the cost price. If seller—
nal price was— 1 (A) Bears no loss, no gain
he allows a discount of 6 %
4
(A) Re. 1 (B) Gains
then his gain per cent is—
(C) Losses 1%
(B) Rs. ( )1 – P2
100 (A) 23 %
3
4 (D) None of these

Answers with Hints


01. (C) S.P. = Rs. 100, gain = Rs. 10 04. (C) Let C.P. of each article = Re. 1
∴ C.P. = (S.P.) – gain = Rs. 90 Then, C.P. of 12 articles = Rs. 12
∴ Gain% =
10
90 ( )
× 100 % = 11 %
1
9
S.P. of 12 articles = C.P. of 15 articles
= Rs. 15
02. (C) 90 : 10·80 : : 120 : x

∴ x =
10·80 × 120
= 14·40
∴ Gain % = (
3
12
× 100 %)
90 = 25%
03. (B) S.P. = Rs. 247·50 05. (C) Let C.P. of each table = Re. 1
25
Gain = % C.P. of 20 tables = Rs. 20
2
S.P. of 20 tables = C.P. of 15 tables = Rs. 15
⎧ ⎫
C.P. = Rs. ⎪ × 247·50⎪
100
∴ ⎨⎪ ⎬⎪
(
⎩ 100 + 2 )
25

∴ Loss = ( 5
20 )
× 100 % = 25%

100 × 2
= Rs. ( 225
× 247·50) 06. (B) ˙·˙ C.P. of 1 article =
200
125
× 100 = Rs. 160

= 220 ⇒ C.P. of 6 articles = 6 × 160 = 960

Quantitative Aptitude Test | 157


∴ Profit = 1056 – 960 = 96 17. (D) ˙·˙ Profit is maximum when C.P. is minimum and
96 S.P. is maximum
Percentage profit = × 100 = 10%
960 Thus, C.P. = Rs. (200 × 8) = Rs. 1600
07. (A) Let the cost of each table and chair be Rs. x and S.P. = Rs. (425 × 8) = Rs. 3400
Rs. y respectively ∴ Gain = Rs. 1800
˙·˙ 2y + 3x = 1025 18. (D) Let S.P. = Rs. x
and 3y + 2x = 1100 ˙·˙ 400 + 20% of x = x
Solving the above two equations, we get x
x = Rs. 175 and y = Rs. 250 ⇒ 400 + = x
5
∴ Difference between the cost of one table and one 4x
chair = Rs. (250 – 175) = Rs. 75 ⇒ = 400
5
08. (B) Let C.P. = Rs. x
400 × 5
˙·˙ 425 – x = x – 355 ∴ x = = 500
4
⇒ 2x = 780 19. (C) ˙·˙ Gain = (S.P. of 8 dozen) – (C.P. of 8 dozen)
∴ x = 390 ⇒ (S.P. of 1 dozen) = (S.P. of 8 dozen)
09. (B) S.P. = Rs. 100, – (C.P. of 8 dozen)
Loss = Rs. 10 ∴ (C.P. of 8 dozen) = (S.P. of 7 dozen)
∴ C.P. = (S.P.) + (Loss) = Rs. 110 Let C.P. of each dozen be Re. 1
∴ Loss% = ( 10
110 )
× 100 % = 9 %
1
11
⇒ C.P. of 7 dozen = Rs. 7
⇒ S.P. of 7 dozen = Rs. 8
10. (D) C.P. of 10 articles = Rs. 8
S.P. of 10 articles = Rs. (1·25 × 10) ∴ Gain % = ( 1
7 )
× 100 % = 14 %
2
7
= Rs. 12·50
20. (C) ˙·˙ Loss = (C.P. of 36 oranges)
∴ Gain = ( 4·5
8 )
× 100 % = 56 %
1
4 – (S.P. of 36 oranges)
⇒ (S.P. of 4) = (C.P. of 36) – (S.P. of 36)
11. (B) S.P. = Rs. 1980
Gain = 10% ∴ (S.P. of 40) = (C.P. of 36)
Let C.P. of each orange
∴ C.P. = Rs. ( 100
110 )
× 1980 = Rs. 1800 = Re. 1
12. (C) Let the selling price of the article = Rs. 100 ⇒ C.P. of 40 = Rs. 40
∴ Profit = Rs. 26 ⇒ S.P. of 40 = Rs. 36
∴ Cost price of the article = 100 – 26 = Rs. 74
∴ Required % =
34 × 74
= 25·16%
∴ Loss = ( 4
40 )
× 100 % = 10%
100 21. (D) Let the marked price be Rs. x
13. (D) C.P. of 60 kg mix = Rs. (25 × 6 + 35 × 7) Then, (7% of x) – 15 = 5% of x
= Rs. 395
7x 5x
S.P. of 60 kg mix = Rs. (60 × 6·75) = Rs. 405 ⇒ – = 15
100 100
Gain = Rs. (405 – 395) = Rs. 10
14. (C) S.P. = 90% of Rs. 200 = Rs. 180 ∴ x = 750
Required S.P. = (95% of Rs. 180) = Rs. 171 22. (A) Cost of the horse paid by
15. (C) Suppose he sells x oranges 100
A = 4860 × = Rs. 6000
2 81
Then, C.P. of x oranges = Rs. x
3 Cost of the horse paid by
S.P. of x oranges = Rs. x 117
C = 6000 × = Rs. 7020
( 2
)
Profit on x oranges = Rs. x – x = Rs.
3
x
3
100
Gain of B = Rs. 7020 – Rs. 4860
x
˙·˙ = 10 ⇒ x = 30 = Rs. 2160
3
16. (D) 2x + y = 7000 …(i) 23. (B) Let the C.P. be Rs. x
x + 2y = 4250
Solving (i) and (ii) we get,
…(ii)
Then, S.P. when gain is 12% = ( 12x
100 )
+x =
112x
100
x = 3250 112x 88x
˙·˙ – = 6
∴ Cost of a T.V. set is Rs. 3250. 100 100

Quantitative Aptitude Test | 158



24x
100
= 6 ⇒ S.P. = Rs. (1110 × 110) = Rs. 121
600 ∴ Gain % = 21%
⇒ x = = Rs. 25
24 30. (D) Let C.P. = Rs. x
24. (D) Let the original cost of each cooker be Re. 1 and
let the number sold originally be 100. Then loss = Rs. (x7)
Total sale proceeds = Rs. (100 × 1) = Rs. 100 S.P. = (C.P.) – (Loss)
New rate = (115% of Re. 1) = Rs. 1·15
Number sold now = 85 ( )
= x–
x
7
= Rs.
6x
7
∴ Sale proceeds now = Rs. (1·15 × 85) 6x
= Rs. 97·75 ˙·˙ = 144
7
So, there is a decrease of 2·25% in the money receipt. 144 × 7
25. (D) Let the S.P. be Rs. x ∴ x = = Rs. 168
6
9x Hence, no loss and no gain.
Then, C.P. paid by Subhash = Rs.
10 31. (B) C.P. of 150 calculators
S.P. received by Subhash = (108% of Rs. x) = 150 × 250 = Rs. 37500
27x ∴ Total C.P. = 37500 + 2500 = Rs. 40000
= Rs.
25 Marked price of 150 calculators
∴ Gain = Rs. ( )27x 9x

25 10
= 150 × 320 = Rs. 48000
Selling price after discount
= Rs. ( )9x
50 = 48000 ×
95
100
= Rs. 45600

⎛ 9x 10 ⎞ ∴ Percentage profit =
45600 – 40000
× 100
Hence, Gain% = ⎜ × × 100⎟ % 40000
⎝ 50 9x ⎠
= 14%
= 20%
32. (D) Let C.P. be Rs. x then,
26. (D) ˙·˙ Gain = (S.P. of 100 bananas)
x
– (C.P. of 100 bananas) x – 270 = 10% of x =
10
⇒ (S.P. of 20) = (S.P. of 100) – (C.P. of 100)
∴ x = 300
⇒ S.P. of 80 = C.P. of 100
33. (A) Let C.P. per kg be Rs. x then,
Let C.P. of each banana = Re. 1
x – 10% of x = 5·40
C.P. of 80 banana = Rs. 80
⇒ x = 6
S.P. of 80 banana = Rs. 100
∴ Required S.P. = Rs. [6 + 20% of 6] = Rs. 7·20
∴ Gain % = ( 20
8 )
× 100 % = 25% 34. (C) Let the marked price be Rs. x
Discount availed by the retailer = 15% of Rs. x
27. (C) Suppose he buys 10 lemons
˙·˙ C.P. of the machine by the retailer
˙·˙ C.P. = Rs.( ) 10
2
= Rs. 5,
= (x – 15% of x) = Rs.
17x
20
⇒ ( )
S.P. = Rs.
3
5
× 10 = Rs. 6 So, 15% of
17x
20
= 1955 –
17x
20
∴ Gain % = ( )1
5
× 100 % = 20% ⇒
51x 17x
+
400 20
= 1955
28. (C) Let C.P. = Rs. x or x = 2000
x Discount received by retailer
Then ˙·˙ x– = 10
11 = (15% of 2000) = Rs. 300
10x 35. (A) Let the printed price of the book be Rs. 100.
⇒ = 10 After a discount of 10%,
11
S.P. = Rs. 90
⇒ x = 11
Profit earned = 12%
∴ C.P. = Rs. 11
29. (C) Suppose 1 purchased 110 books ∴ C.P. of the book = Rs. ( 100
112 )
× 90

˙·˙ C.P. = Rs. ( 10


11 )
× 110 = Rs. 100 = Rs.
1125
14
Quantitative Aptitude Test | 159
1125 Difference between two S.P.
Hence, (C.P.) : (Printed price) = : 100

14
= 45 : 56
= Rs. (100 – 3754)
36. (B) Let original price = Rs. 100 25
= Rs.
Price after 1st discount = Rs. 90 4
Price after 2nd discount = Rs. ( 80
100
× 90 ) If difference is Rs.
25 ,
4
S.P. = Rs. 100

Price after 3rd discount =


= Rs. 72
Rs.
60
× 72
If difference is Rs. 500, S.P. = Rs. (100 ×254 × 500)
100 = Rs. 8000
= Rs. 43·20 42. (A) C.P. of 330 boxes = Rs. (7200 + 200 + 600)
∴ Single equivalent discount = Rs. 8000
= (100 – 43·20) = 56·8% S.P. of 330 boxes = Rs. (330 × 28)
37. (B) Suppose he buys 12 oranges. Then S.P. = Re. 1 = Rs. 9240
Now,


80 : 1 : : 120 : x

x =
1 × 120 3
=
∴ Gain% = ( 1240
8000
× 100 % )
80 2 = 15·5%
3 43. (C) Let A’s C.P. = Rs. 100
∴ For Rs. , oranges sold = 12
2 B’s C.P. = Rs. 105
For Re. 1 oranges sold = 12 × ( 2
3) =8
˙·˙
C’s C.P. = 104% of Rs. 105 = Rs. 109·20
109·20 : 91 = 100 : x
So, he must sell them at 8 for a rupee. 91 × 100
∴ x = = Rs. 83·33
38. (B) Let the error be x gms. Then, 109·20
x 300 44. (C) Let original cost of each toy be Rs. 100 and
× 100 =
(1000 – x) 47 number originally sold be 100.
x 3 ∴ Original sale proceeds = Rs. (100 × 100)
⇒ =
1000 – x 47 = Rs. 10000
47x = 3000 – 3x New sale proceeds = Rs. (120 × 85)
⇒ x = 60 = Rs. 10200
200 × 100
So, he uses a weight = (100 – 60) gm
= 940 gms for 1 kg.
∴ Increase% = ( 10000 )
% = 2%

39. (D) Let S.P. = Rs. 100 45. (B) C.P. of 6 dozen apples = Rs. (12 × 4 + 16 × 2)
Then C.P. = Rs. 40 = Rs. 80
Gain = 20%
∴ Required per cent = ( 100
40 )
× 100 % = 250%
∴ S.P. = Rs. (120
× 80 )
40. (B) Suppose he buys 8 toffees 100
Then, C.P. = Re. 1 = Rs. 96
Gain = 60%
S.P. per dozen = Rs. ( )
96
= Rs. 16
∴ S.P. = Rs. ( 160
100 )
× 1 = Rs.
8
5 46. (C)
6
C.P. of B = 110% of Rs. 10000 = Rs. 11000
8 Loss of B = 10%
For Rs. , toffees sold = 8
5 S.P. of B = 90% of Rs. 11000 = Rs. 9900
For Re. 1 toffees sold = 8 ×( ) 5
8
=5 Thus, C.P. of A = Rs. 9900
So, A gets [(10% of Rs. 10000) + (10000 – 9900)]
So, he must sell them at 5 for a rupee. = Rs. 1100
41. (D) Let S.P. of T.V. (by trader) = Rs. 100
If S.P. is Rs. 80, then M.P. = Rs. 100 47. (D) Gain% = { Error
(True value) – Error
× 100 % }
If S.P. is Rs. 100, M.P. = Rs. ( 100
80 )
× 100
= ( 100
900 )
× 100 % = 11 %
1
9
= Rs. 125
Now, if discount is 25% then, S.P. 48. (A) Let original cost of each be Rs. 100 and number
= (75% of Rs. 125) originally sold be 100.
375 Original sale proceeds = Rs. (100 × 100)
= Rs.
4 = Rs. 10000

Quantitative Aptitude Test | 160


New sale proceeds = Rs. (80 × 140) Solving (i) and (ii), we get,
= Rs. 11200 y = 4200

∴ Increase% = ( 1200
10000 )
× 100 % 59. (A) S.P. = 90% of (
3
4 )
of Rs. 400

+ 110% of ( of Rs. 400)


= 12% 1
4
49. (D) Loss% = ( Common gain or loss% 2
10 ) = Rs. ( × × 400)
90 3
100 4
( ) 10 2
= = 1%
+ Rs. ( × × 400)
10 110 1
100 4
50. (C) Let C.P. = Rs. 100
= Rs. (270 + 110) = Rs. 380
S.P. at 10% loss = Rs. 90

˙·˙
2
of actual S.P. = Rs. 90
∴ Loss% = ( 20
400 )
× 100 % = 5%
3 60. (D) ˙·˙ C.P. of 120 reams = Rs. (120 × 80 + 280
So, Actual S.P. = Rs. 90 ×( ) 3
2
= Rs. 135 + 72 + 120 × 0·40)
= (9600 + 280 + 72 + 48)
∴ Gain = 35%
= Rs. 10000
51. (A) Loss% ( Common gain or loss % 2
10 ) ⇒ C.P. of 1 ream = ( 10000
120) = Rs.( ) 250
3
( )
=
10
5 2 1
= % = 0·25%
4 ∴ S.P. of 1 ream = Rs. ( 108 250
×
100 3 ) = Rs. 90
52. (B) Let C.P. = Rs. x then,
61. (A) Let C.P. = Rs. x
425 – x = x – 355
110 90
⇒ 2x = 780 Then x– x = 6
100 100
∴ x = 390 ⇒ 20x = 600
53. (B) Let C.P. = Rs. x 600
⇒ x = = 30
˙·˙ x + x% of x = 75 20
x2 ∴ C.P. = Rs. 30
⇒ x+ = 75
100 62. (B) Let total C.P. of all the articles = Rs. 100
⇒ x2 + 100x – 7500 = 0
⇒ (x + 150) (x – 50) = 0

3
C.P. of th part = Rs.
4 ( 3
4 )
× 100

= Rs. 75
∴ x = 50

54. (B) ˙·˙ Loss% = ( ) 20 2


10
= (2)2 = 4%

3
S.P. of th part = Rs.
4 ( 120
100
× 75)
= Rs. 90
⇒ Total S.P. = Rs. 24000

1
S.P. of th part = Rs. ( 1
)
× 100
⇒ Total C.P. = Rs. ( 100
96
× 24000) 4
= Rs. 25
4

= Rs. 25000 Total S.P. = Rs. (90 + 25)


∴ Loss = Rs. 1000 = Rs. 115
∴ Gain = 15%
55. (D) 56. (B) 63. (D) Let C.P. = Rs. x
57. (C) ˙·˙ S.P. = Rs. 54000
Gain earned = 20% ˙·˙ First S.P. = 110% of x = Rs. ( )11
10
x

∴ C.P. = Rs. ( 100


120 )
× 54000 = Rs. 45000 ⇒ Second S.P. =
22
10
x
Now,
And
S.P. = Rs. 45000
Loss = 10%
⇒ New gain = ( 22
10 )
x–x =
12x
10
⎛ 12x ⎞
∴ Original C.P. = Rs. ( 100
90 )
× 45000 = Rs. 50000 ∴ New gain% = ⎜
⎝ 10 × x
× 100⎟ % = 120%

58. (C) 2x + y = 7000 …(i) 64. (A) ˙·˙ 6% of C.P. + 6% of C.P. = Rs. 6
x + 2y = 9800 …(ii) ⇒ 12% of C.P. = Rs. 6

Quantitative Aptitude Test | 161


12 69. (A) C.P. of two horses = Rs. 8000
⇒ ×x = 6
100 S.P. of one horse = Rs. 4000
100 × 6 Gain = 25%
⇒ x = = 50
12
∴ C.P. = Rs. 50
∴ C.P. of this horse = Rs. (
100
125
× 4000 )
65. (C) Let C.P. = Rs. x = Rs. 3200
C.P. of another horse = Rs. (8000 – 3200)
Then S.P. = Rs. ( 90
100 )
× x = Rs. ( )9
10
x = Rs. 4800
S.P. of this horse = Rs. 4000
New C.P. = Rs. ( 80
100 )
× x = Rs. ()4x
5 ∴ Loss% = ( 800
)
× 100 % = 16 %
2
4800 3
Now, Gain = 40%
70. (C) Let us consider a packet of rice marked 1 kg
∴ New S.P. = Rs. ( 140 4x
×
100 5 )
= Rs. ( )28
25
x Then its actual weight = (80% of 1 kg) = 0·8 kg
28 9 Let C.P. of 1 kg be Rs. x
˙·˙ x– x = 55
25 10 Then C.P. of 0·8 kg = Rs. 0·8x
∴ C. P. = x = Rs. 250 Now, S.P. of 1 kg = 110% of C.P. of 1 kg
66. (B) Let C.P. = Rs. x = (110
100 )
× x = Rs. 1·1x
Then S.P. = Rs. ( 110
100 )
× x = Rs.
11x
10 ⇒ Gain = (1·1 – 0·8) x = 0·3x
⎛ 0·3x ⎞
New C.P. = Rs. ( 90
100 )
× x = Rs.
9x
10
⇒ Required gain% = ⎜ 0·8x × 100⎟ %
⎝ ⎠
New S.P. = ( 110 9x
×
100 10 ) = Rs.
99x
100
= 37·5%
71. (C) ˙·˙ (20% of C.P.) – (16% of C.P.) = Rs. 20
11x 99x ⇒ 4% of C.P. = 20
˙·˙ – = 132
10 100
⇒ x = 1200 ⇒ ( 4
100 )
× C.P. = 20
∴ C.P. = Rs. 1200 100 × 20
67. (B) Let the loss be Rs. x ⇒ C.P. = Rs. ( 4 )
Then 75 = C.P. – x ∴ C. P. of the bicycle = Rs. 500
and 96 = C.P. + 2x 72. (D) It is clear that
On subtracting we get, ˙·˙(2·5% of C.P. + 7·5% of C.P.) = 100
3x = 21 ∴ C.P. = Rs. 1000
⇒ x = 7 1
Now, gain = 12 %
2
∴ C.P. = 75 + x = Rs. 82
1
68. (C) Let the original rate be x paise per egg. So, Required S.P. = 112 % of C.P.
2
Number of eggs bought for Rs. 7·80 = (780/x)
New rate = (130% of x) paise per egg = Rs. (225
200
× 1000 )
13x = Rs. 1125
= paise per egg
10 73. (C) ˙·˙ (115% of C.P.) – (125% of 90% of C.P.) = 4
Number of eggs bought for 115 125 90
780 × 10 600 ⇒ x– × x = 4
Rs. 7·80 = = x 100 100 100
13x 23x 9x
780 600 ⇒ – = 4
20 8
x – x = 3
˙·˙
⇒ x = 160
⇒ 3x = 180 ∴ C.P. = Rs. 160
⇒ x = 60 74. (A) Difference between two selling prices
13 × 60
So, Present rate = ( 10 ) paise per egg
˙·˙
= 475 – 451 = Rs. 24
6% of C.P. = Rs. 24
24 × 100
= 78 paise per egg
= Rs. 9·36 per dozen
Hence, C.P. = Rs. ( 6 )= Rs. 400

Quantitative Aptitude Test | 162


75. (B) (100 – P) (100 + P)
˙·˙ ×x = 1
76. (D) Total C.P. of 200 kg of sugar 100 × 100
= Rs. (80 × 6·75 + 120 × 8) 100 × 100
∴ x =
(100 – P) (100 + P)
= Rs. (500 + 960)
10000
= Rs. 1460 =
(10000 – P2 )
1460 83. (B) Let C.P. = Rs. 100
C.P. of 1 kg = Rs. = Rs. 7·30
200 ⇒ Marked price = Rs. 120,
Gain required = 20% ⇒ S.P. = Rs. 108
∴ S.P. of 1 kg = (120% of Rs. 7·30)
∴ Rate of discount = ( 12
)
× 100 % = 10%
= Rs.( 120
100 )
× 7·30
84. (B) ˙·˙
120
S.P. = (90% of Rs. 480)
= Rs. 8·76 per kg
77. (D) Let the C.P. be Rs. x
= Rs. ( 90
100 )
× 480 = Rs. 432
Gain earned on it = 8%
2 (69 – x) 78 – x
Then
100
=
100 ∴ C.P. = Rs. ( 100
108 )
× 432 = Rs. 400
⇒ 138 – 2x = 78 – x If no discount is allowed, S.P. = Rs. 480


x = 60
C.P. = Rs. 60
∴ Required gain% = ( 80
400 )
× 100 % = 20%

78. (C) In such questions we adopt the rule 85. (B) Let the prices be 4x, 5x and 7x rupees.
Required gain % Then, 7x – 4x = Rs. 60000
⇒ x = 20000
= [ (100 + common gain%)2
100 ]
– 100 % ∴ Required modest price = 5x = Rs. 100000
86. (C) Let the C.P. be Rs. 100
= { (108) 2
100 }
– 100 % = 16·64% Then, Marked price = Rs. 130
79. (C) Let the marked price be Rs. x
3x
⇒ ( 3
S.P. = 93 % of Rs. 130
4 )
˙·˙ C.P. = (x – 25% of x) =
4 = Rs. ( 375
4 × 100 )
× 130

⇒ S.P. = ( 3x
4
+ 10% of )3x
4
=
33x
40 = Rs.
975
8
33x 7
But, = 660 = Rs. 121
40 8
∴ x = 800 7
∴ Required gain% = 21 %
80. (B) Let original price = Rs. 100 8
Price after first discount = Rs. 90 87. (D) Price after 1st discount = 80% of Rs. 160
Price after second discount = Rs. ( 80
100 )
× 90
= Rs. 128
Price after 2nd discount
= Rs. 72 = 90% of Rs. 128
Price after third discount = Rs. ( 60
100 )
× 72 88. (B)
= Rs. 115·20
Let C.P. = Rs. 100
= Rs. 43·20 Then marked price = Rs. 120
∴ Single equivalent discount = (100 – 43·20) S.P. = 90% of Rs. 120 = Rs. 108
= 56·8% ∴ Required gain% = ( 108 – 100
100 ) × 100%
81. (A) Rate of discount = ( 12
80 )
× 100 % = 15% = 8%
82. (C) Let original price be Rs. x 89. (D) Sale after 40% discount = 60% of Rs. 500
Price after P% increase = Rs. 300
Price after 36% discount = 64% of Rs. 500
(100 + P)x
= (100 + P)% of x = = Rs. 320
100 Price after next 4% discount = 96% of Rs. 320
New price after P% decrease = Rs. 307·20
(100 + P)x ∴ Required difference in two prices
= (100 – P)% of
100 = (307·20 – 300)
(100 – P) (100 + P) = Rs. 7·20
= × ×x
100 100 90. (C)

Quantitative Aptitude Test | 163


Time and Work
Important Points/Facts Q. 2. Neelam, Usha, Meena can 3+2
=
do a work in 10, 12 and 15 days 40
1. If a man can do piece of work
respectively. If Usha left after 2 1
1 =
in x days, he will do x of the work in days. How long would it take 8
Neelam and Meena to complete the
1 day. And conversely, if a man can Hence, A and B working together
remaining work ?
1 will finish that work in 8 days.
do x of the work in 1 day. He will do Solution : The part of work com- Q. 4. A can do a work in 12
the work in x days. pleted by Neelam, Usha and Meena days. B works 60% more fast than
in 2 days A then in how many days B alone
2. If A is x times as good a work-
1
man as B, then A will take x of the = 2× ( 1
+
1
10 12 15
+ )
1 could finish this work ?
Solution :
time that B takes to do a certain
work, i.e.,
= 2× ( 6+5+4
60 ) =
1
2
˙.˙ A’s work in 1 day =
1
12
Time taken by A to do a work 1 ∴ B’s work in 1 day
∴ Remaining work = 1 –
1
= x × Time taken by B to
1
2 =
1
12( 1+ )
60
100
=
do the same work 2 1 160
= ×
3. The multiple ratio of the ˙.˙ Work done by (Neelam and 12 100
capacity of doing work by A and B is Meena) together in 1 day 2
=
inverse of the multiple ratio of their 1 1 15
efficiency of doing work, e.g.,— = + ∴ B alone can finish this work
10 15
(namely)—If the work done by 3 men 15 1
is equal to the work done by 4 3+2 1 in days, i.e., 7 days.
= = part 2 2
women, then what will be the ratio of 30 6
Q. 5. If 5 men and 2 boys
the work done by a man and a 1
˙.˙ th part of work completed working together can do four times
woman ? 6 as much work per hour as a man
Here, work done by 3 men by both Neelam and Meena in 1 day and a boy together. Compare the
= Work done by 4 women 1 work of a man with that of a boy.
∴ part of work will be
Work of a man 4 2 Solution :
⇒ = 1 1
Work of a woman 3 completed in = × Here (5M + 2B)’s 1 day’s work
1 2
∴ Required ratio = 4 : 3 = (1M + 1B)’s 4 days work
6
In other words, we can say that ⇒ 5M + 2B = 4 (1M + 1B)
= 3 days
‘‘Efficiency is indirectly proportional [where M = Man, B = Boy]
to the number of days taken to 3
Q. 3. A can do th part of a ⇒ 5M + 2B = 4M + 4B
complete a work.’’ 4
work in 10 days and B completed ⇒ 1M = 2B
4. If x can do a work in a days
the remaining part of work in 5 M 2
and y can do it in b days, then x and y ⇒ =
days. In how many days A and B B 1
working together will do the same
ab working together will finish that ∴ Required ratio = 2 : 1
work in days. work ?
a+b Q. 6. One man, 2 women and 3
Solution : boys working together do a work
Examples 3 1 in 36 days, while 3 men, 2 women
Work of A in 1 day = × and one boy do the same work in
Q. 1. Dinesh and Ram can do a 4 10
work in 80 minutes and 120 minutes 12 days. Then in how many days
3
respectively. In how much time this = will 6 men, 6 women and 6 boys
40
work will be completed if they work will do this work ?
1 1
altogether ? Work of B in 1 day = × Solution : ˙.˙ (1 + 3) men (2 + 2)
4 5 women and (3 + 1) boys work for 1
Solution :
(80 × 120) 1 1 1 1+3
Required time = = day = + =
(80 + 120) 20 36 12 36
9600 ∴ Work of (A + B) in one day i.e., 4 men + 4 women + 4 boys
=
200 3 1 1
= + work for 1 day =
= 48 minutes 40 20 9

Quantitative Aptitude Test | 164


∴ (1 man + 1 woman + 1 boy) Hence, (A + B + C) working 9 days. The time taken by B
1 1 1 together can complete the work in 2 alone to do the same work is—
work for 1 day = × =
9 4 36 days. (A) 18 days (B) 15 days
∴ (6 men + 6 women + 6 boys) 1
Exercise (C) 12 days (D) 7 days
1 1 2
work for 1 day = ×6= 01. Twenty-four men can complete a
36 6 07. A can do a piece of work in 12
work in sixteen days. Thirty-two
∴ Required time = 6 days women can complete the same days, B can do the same work in
work in twenty-four days. 8 days and C can do the same job
Q. 7. Uday working 4 hours a 4
day completes a work in 10 days, Sixteen men and sixteen women in th time required by both A
started working and worked for 5
while Subhash working 8 hours a
twelve days. How many more and B. A and B work together for
day can complete the same work in
men are to be added to complete 3 days, then C completes the job.
15 days. How many days will it take
the remaining work in 2 days ? How many complete days did C
for both of them working together work ?
10 hours a day to complete the (A) 48
work ? (A) 8
(B) 24
(B) 6
Solution : It is clear from the (C) 36
question that (C) 3
(D) None of these (D) None of these
˙.˙ Uday completes the work in
(4 × 10) hours = 40 hours 02. A can do a piece of work in 30 08. A can do (1/3) of a work in 5
days while B can do it in 40 days. days and B can do (2/5) of the
and Subhash completes the same
In how many days can A and B work in 10 days. In how many
work in (8 × 15) = 120 hours
working together do it ? days both A and B together can
∴ Work done in 1 hour, when
3 do the work ?
both working together will be (A) 70 days (B) 42 days
4 3 4
1 1 1 (A) 7 (B) 8
= + = 1 1 4 5
40 120 30 (C) 27 days (D) 17 days
7 7 3
Hence, both working together (C) 9 (D) 10
8
will complete the work in 30 hours. 03. 25 men and 15 women can com-
˙·˙ Both working together 10 plete a piece of work in 12 days. 09. A completes a work in 15 days. B
hours a day All of them start working together completes the same work in 20
and after working for 8 days the days. A started working alone
⇒ Both working together 30 women stopped working. 25 men after 1 day B joined him. How
1 completed the remaining work in many days will they now take
hours in × 30 = 3 days
10 6 days. How many days will it together to complete the remain-
∴ Required time = 3 days take for completing the entire job ing work ?
if only 15 women are put on the (A) 8 days
Q. 8. A is two times work-
job ? (B) 7 days
efficient than B and three times
than C. If C alone can do that work (A) 60 days (C) 6 days
in 11 days then how long will it (B) 88 days (D) None of these
take for all the three A, B and C to (C) 94 days
complete the same work ? 10. A and B can do a piece of work
(D) None of these in 18 days; B and C in 24 days, A
Solution : Amount of work, C and C in 36 days. In what time
1 04. A and B can together do a piece
can do in 1 day = can they do it all working
11 of work in 15 days. B alone can
together ?
do it in 20 days. In how many
Similarly, Amount of work A (A) 12 days (B) 13 days
days can A alone do it ?
3
can do in 1 day = (A) 30 days (B) 40 days (C) 16 days (D) 26 days
11
(C) 45 days (D) 60 days 11. A can do a piece of work in 10
and amount of work, B can do in
days and B can do the same piece
1 day 05. 10 men and 15 women finish a of work in 20 days. They start the
3 3 work in 6 days. One man alone work together but after 5 days A
= =
2 × 11 22 finishes that work in 100 days. leaves off. B will do the re-
∴ Amount of work (C + A + B) In how many days will one maining piece of work in—
together can do in 1 day woman can finish the work ? (A) 5 days (B) 6 days
1 3 3 11 (A) 125 days (B) 150 days (C) 8 days (D) 10 days
= + + = (C) 90 days (D) 225 days
11 11 22 22 12. Twelve men can complete a work
1 06. A and B can do a piece of work in 8 days. Three days after they
=
2 in 6 days and A alone can do it in started the work, 3 more men

Quantitative Aptitude Test | 165


joined them. In how many days also joins him. In all the work 1
reaped in 37 days after they
will all of them together complete would be completed in— 2
the remaining work ? (A) 7 days leave the work ?
(A) 2 (B) 4 (A) 6 days
(B) 9 days
(B) 4 days
(C) 5 (D) 6 (C) 11 days (C) 5 days
13. A and B can do a piece of work (D) None of these (D) None of these
in 12 days. B and C in 15 days. C
and A in 20 days. A alone can do 19. A can do a piece of work in 80 25. A can do a piece of work in 12
the work in— days. He works at it for 10 days days. B is 60% more efficient
and then B alone finishes the than A. The number of days, it
2 work in 42 days. The two take B to do the same piece of
(A) 15 days (B) 24 days
3 together could complete the work work is—
(C) 30 days (D) 40 days in— 1 1
(A) 7 (B) 6
(A) 24 days (B) 25 days 2 4
14. A can complete a job in 9 days,
B in 10 days and C in 15 days. B (C) 30 days (D) 35 days (C) 8 (D) 6
and C start the work and are 26. 15 men would finish a piece of
forced to leave after 2 days. The 20. A can do a certain job in 25 days work in 210 days. But at the end
time taken to complete the which B alone can do in 20 days. of every 10 days. 15 additional
remaining work is— A started the work and was men are employed. In how many
joined by B after 10 days. The
(A) 6 days (B) 9 days days will it be finished ?
work lasted for—
(A) 30 days (B) 70 days
(C) 10 days (D) 13 days 1 2
(A) 12 days (B) 14 days (C) 35 days (D) 60 days
15. A, B and C contract a work for 2 9
27. Ramesh can finish a job in 20
Rs. 550. Together A and B are to 2
(C) 15 days (D) 16 days days. He worked for 10 days
7 3
do of the work. The share of alone and completed the remain-
11
21. If 5 men and 3 boys can reap 23 ing job working with Dinesh, in
C should be—
hectares in 4 days and if 3 men 2 days. How many days would
1 and 2 boys can reap 7 hectares in both Dinesh and Ramesh together
(A) Rs. 183 (B) Rs. 200
3 2 days. How many boys must take to complete the entire job ?
(C) Rs. 300 (D) Rs. 400 assist 7 men in order that they (A) 4 (B) 5
may reap 45 hectares in 6 days ? (C) 10 (D) 12
16. Sunil completes a work in 4 days
whereas Dinesh completes the (A) 2 boys (B) 6 boys 28. A can copy 75 pages in 25 hours.
work in 6 days. Ramesh works (C) 4 boys (D) 5 boys A and B together can copy 135
1 pages in 27 hours. In what time
1 times as fast as Sunil. How 22. 12 men can complete a work can B copy 42 pages ?
2 within 9 days. After 3 days they
many days it will take for the (A) 21 hours
started the work, 6 men joined
three together to complete the (B) 5 hours, 36 seconds
them to replace 2 men. How
work ? (C) 18 hours
many days will they take to
7 complete the remaining work ? (D) 24 hours
(A)
12 (A) 2 (B) 3 29. A can do a piece of work in 5
5 hours, B in 9 hours and C in 15
(B) 1 1
12 (C) 4 (D) 4 hours. If C could work with them
2 for 1 hour only the time taken by
5
(C) 1 23. A piece of work was to be com- A and B together to complete the
7 work is—
pleted in 40 days a number of
(D) None of these men employed upon it did only (A) 2 hours (B) 3 hours
17. A can do a piece of work in 20 half the work in 24 days, 16 more 1
(C) 3 hours (D) 4 hours
days which B can do in 12 days. men were then set on, and the 2
B worked at it for 9 days. A can work was completed in the speci-
30. Mohan can move his lawn in x
finish the remaining work in— fied time, how many men were
employed at first ? hours after 2 hour it begins to
(A) 3 days (B) 5 days rain. The unmoved part of the
(A) 16 men (B) 32 men lawn is—
(C) 7 days (D) 11 days
(C) 24 men (D) 48 men 2 2–x
18. Mahesh and Umesh can complete (A) x (B)
a work in 10 days and 15 days 24. 25 men reap a field in 20 days. 2
respectively. Umesh starts the When should 15 men leave the x x–2
(C) (D) x
work and after 5 days Mahesh work, if the whole field is to be 2

Quantitative Aptitude Test | 166


31. A can do a piece of work in 24 They begin together but 3 days in 3 days. Find the time for A to
days while B alone can do it in before the completion of the complete the job alone.
16 days. But with the help of C, work. A leaves off. The total 1 1
they finish the work in 8 days. C number of days to complete the (A) 6 days (B) 6 days
2 4
alone can do the work in— work is— 3 1
(A) 32 days (B) 36 days 3 1 (C) 6 days (D) 12 days
(A) 6 days (B) 8 days 4 2
(C) 40 days (D) 48 days 5 2
44. A is twice as good a workman as
32. The rates of working of A and B 1 1 B and together they finish a piece
(C) 10 days (D) 13 days
are in the ratio 3 : 4. The number 5 2 of work in 14 days. A alone can
of days taken by them to finish finish the work in—
the work are in the ratio— 39. A does half as much work as B
in three-fourth of the time. If (A) 11 days (B) 21 days
(A) 3 : 4 together they take 18 days to (C) 28 days (D) 42 days
(B) 9 : 16 complete a work, how much
time shall B take to do it ? 45. A is thrice as good a workman as
(C) 4 : 3 B and takes 10 days less to do a
(D) None of these (A) 30 days piece of work than B takes. B
33. A and B can together finish a (B) 35 days can do the work in—
work in 30 days. They worked (C) 40 days (A) 12 days (B) 15 days
for it for 20 days and then B left. (C) 20 days (D) 30 days
(D) None of these
The remaining work was done by
A alone in 20 more days. A alone 40. A and B working separately can 46. A sum of money is sufficiently to
can finish the work in— do a piece of work in 9 and 12 pay A’s wages for 21 days or B’s
days respectively. If they work wages for 28 days. The money is
(A) 48 days (B) 50 days
for a day alternately. A beginning sufficient to pay the wages of
(C) 54 days (D) 60 days both for—
in how many days the work will
34. A can do a certain job in 10 days. be completed ? (A) 12 days
B is 60% more efficient than A. 1
1 1 (B) 12 days
The number of days, it takes B to (A) 10 days (B) 10 days 4
2 4
do the same piece of work is—
2 1 (C) 14 days
1 (C) 10 days (D) 10 days
(A) 6 (B) 6 3 3 (D) None of these
4
2 41. If factory A turns out x cars an 47. If 3 men or 4 women can con-
(C) 6 (D) 8
3 hour and factory B turns out y struct a wall in 43 days. Then the
cars every 2 hours, the number of number of days that 7 men and 5
35. A, B and C together earn Rs. 150
cars which both factories turn out women take to construct it is—
per day while A and C together
earn Rs. 94 and B and C together in 8 hours is— (A) 12 (B) 18
earn Rs. 76. The daily earning of y (C) 24 (D) 30
C is— (A) 8 (x + y) (B) 8x +
2 48. 12 men or 18 women can reap a
(A) Rs. 75 (B) Rs. 56 field in 14 days. The number of
(C) 16 (x + y) (D) (2x + y)4
(C) Rs. 34 (D) Rs. 20 days that 8 men and 16 women
42. A does half as much work as B will take to reap it is—
36. A alone can finish a work in 10
in three-fourth of the time. If
days and B alone can do it in 15 (A) 5 (B) 7
together they take 18 days to
days. If they work together and (C) 8 (D) 9
complete a work. How much
finish it, then out of a total wages
time shall B take to do it ? 49. A and B can do a piece of work
of Rs. 75, A will get—
(A) 40 days in 45 and 40 days respectively.
(A) Rs. 30 (B) Rs. 37·50
They began the work together,
(C) Rs. 45 (D) Rs. 50 (B) 35 days but A leaves after some days and
37. A alone can do a piece of work (C) 30 days B finished the remaining work in
in 6 days and B alone can do it (D) None of these 23 days. After how many days
in 8 days. A and B undertook to did A leaves ?
do it for Rs. 320 with the help of 43. Two workers A and B working (A) 6 days (B) 8 days
C, they finished it in 3 days. together completed a job in 5 (C) 9 days (D) 12 days
How much is paid to C ? days. If A worked twice as
efficiently as he actually did and 50. 10 men can finish a piece of work
(A) Rs. 37·50 (B) Rs. 40
1 in 10 days, whereas it takes 12
(C) Rs. 60 (D) Rs. 80 B worked as efficiently as he women to finish it in 10 days. If
3
38. A can do a piece of work in 14 actually did the work, then they 15 men and 6 women undertake
days which B can do in 21 days. would have completed the work to complete the work, how many

Quantitative Aptitude Test | 167


days will they take to complete many days will 3 men and 3 How many days will 10 men and
it ? women finish it ? 10 boys together take to complete
(A) 2 (B) 4 2 the same work ?
(A) 6 days (B) 5 days
(C) 5 (D) 11 3 (A) 10 days (B) 8 days
51. A certain number of men com- 1 (C) 12 days (D) 9 days
(C) 8 days (D) 8 days
plete a piece of work in 60 days. 3
65. 14 men can complete a work in
If there were 8 men more, the 59. Two men undertake to do a piece 12 days. 4 days after they started
work could be finished in 10 days of work for Rs. 400. One alone the work, 2 more men joined
less. How many men were can do it in 6 days. The other in them. How many days will they
originally there ? 8 days with the help of a boy they take to complete the remaining
(A) 30 (B) 40 finish it in 3 days. The boy’s work ?
(C) 32 (D) 36 share is— (A) 9 days (B) 5 days
(A) Rs. 40 (B) Rs. 50
52. If 5 men or 9 women can finish a (C) 6 days (D) 7 days
piece of work in 19 days 3 men (C) Rs. 60 (D) Rs. 80
66. 14 persons can complete a work
and 6 women will do the same 60. 16 men and 12 women together in 16 days. 8 persons started the
work in— complete a work in 20 days. If 18 work 12 days after they started
(A) 10 days (B) 12 days women complete the same work the work 8 more persons joined
(C) 13 days (D) 15 days in 40 days. Then how many days them. How many days will they
will be taken by 12 men and 27 take to complete the remaining
53. 4 men and 6 women finish a job women together to complete the
in 8 days. While 3 men and 7 work ?
same work ?
women finish it in 10 days. 10 (A) 12 days
(A) 18 days (B) 15 days
women working together will (B) 7 days
finish it in— (C) 14 days (D) 16 days
(C) 9 days
(A) 24 days (B) 32 days 61. A report of 100 pages is to be
(D) None of these
(C) 36 days (D) 40 days typed by three typists. Typist A
can type 100 pages in 10 hours. 67. Rohan and Sunil separately can
54. 8 children and 12 men complete Typist B can type the same pages complete a work in 8 hours and 4
a certain piece of work in 9 days. in 20 hours and typist C in 25 hours respectively. How much
If each child takes twice the time hours. All the three typist started time will they take when working
taken by a man to finish the typing at 09·00 a.m. At 01·00 together ?
work, in how many days will 12 p.m. typist A stopped typing. 2 1
men finish the same work ? (A) 2 hours (B) 1 hours
The other two typists finished
(A) 8 (B) 15 3 3
the job, approximately at what
(C) 9 (D) 12 time the report was typed ? (C) 3 hours (D) 2 hours
55. 3 men and 4 boys do a piece of (A) 2·40 p.m. (B) 5·00 p.m. 68. Typist A can type a report in 16
work in 8 days, while 4 men and (C) 6·00 p.m. (D) 3·40 p.m. hours. Typist B can type the
4 boys finish it in 6 days. 2 men same report in 20 hours and the
and 4 boys will finish it in— 62. 14 workers can make 1400 toys typist C in 24 hours. All the
in 5 days. One day after they three typists started typing at
(A) 9 days (B) 10 days
started the work 14 more workers 9·00 a.m. at 1·00 p.m. the typist
(C) 12 days (D) 14 days joined them. How many days will A stopped typing. The other two
56. 8 men can dig a pit in 20 days. If they take to complete the remain- typists finished the job. Approxi-
a man works half as much again ing work ? mately at what time the report
as a boy, then 4 men and 9 boys (A) 2 days (B) 3 days was typed ?
can dig a similar pit in— 1 (A) 04 : 10 p.m.
(A) 10 days (B) 12 days (C) 4 days (D) 3 days
2 (B) 05 : 11 p.m.
(C) 15 days (D) 16 days
63. 24 boys can complete a work in (C) 05 : 45 p.m.
57. If 1 man or 2 women or 3 boys 12 days while 16 men can com- (D) 06 : 15 p.m.
can do a piece of work in 44 days plete the same work in 9 days. In
then the same piece of work will how many days will 12 boys and 69. A and B together can do a work
be done by 1 man, 1 woman and 12 men together complete the in 8 days. If A alone can do it in
1 boy in— same work ? 12 days, then in how many days
can B alone do it ?
(A) 21 days (B) 24 days (A) 6 days (B) 8 days
(A) 12 days (B) 20 days
(C) 26 days (D) 33 days (C) 7 days (D) 10 days
(C) 24 days (D) 28 days
58. 2 men and 3 women can finish a 64. 15 men can complete a work in
piece of work in 10 days, while 4 10 days while 20 boys can com- 70. 24 men can do a work in 16 days
men can do it in 10 days. In how plete the same work in 15 days. while 36 women can do the same

Quantitative Aptitude Test | 168


work in 24 days. In how many (C) 24 days then in how many days will they
days will 8 men and 6 women (D) 72 days complete the work ?
together complete the same (A) 4 days (B) 3 days
72. Ganesh, Ram and Sohan together
work ?
can do a work in 16 days. If (C) 2 days (D) 5 days
(A) 36 days (B) 28 days Ganesh and Ram together can do
(C) 32 days (D) 24 days the same work in 24 days then, 74. Ram, Dilip and Shekhar can
71. A and B together can do a work how long will take Sohan alone complete a work in 20 days. If
1 to do the same work ? Ram and Dilip together can com-
in 24 days. B alone does its part
3 (A) 42 days (B) 24 days plete the same work in 30 days,
in 12 days. How long will A then how long will Shekhar take
(C) 36 days (D) 48 days
alone take to do the remaining to complete it ?
work ? 73. A and B separately can complete (A) 60 days (B) 62 days
(A) 48 days a work in 6 days and 3 days res-
pectively. If they work together, (C) 40 days (D) 56 days
(B) 36 days

Answers with Hints


01. (B) 24 men complete the work in 16 days
04. (D) A’s 1 day’s work = (151 – 201 ) = 601
⇒ 16 men complete ( 16 12
×
24 16 ) 1
= part of work in
2 ∴ A alone can finish it in 60 days.
12 days. 05. (D) ˙·˙ One man alone finishes the work in 100 days.
32 women complete the work in 24 days ⇒ 10 men can finish the work in 10 days
16 14 7
∴ 16 women complete × = part of work in From the question,
32 24 24
⇒ 15 women can finish work in one day
(12 + 2) = 14 days
So, the remaining part of the work which is done by 1 1 1
= – = work
(sixteen men + sixteen women) and required addi- 6 10 15
tional no. of men in 2 days ⇒ 15 women finish the whole work in 15 days
∴ 1 woman finishes the whole work in
= 1– ( 1 7
+
2 24 ) = –
1 7
2 24 24
=
5
(Part)
= 15 × 15 = 225 days.
5
Now, in 2 days part of the work is done by
24 06. (A) B’s one day’s work = ( )1 1
6 9
– =
1
18
16 5 ∴ B alone can finish it in 18 days.
24 × × = 40 men
2 24 07. (D) As per question,
∴ Required additional no. of men 1
= 40 – 16 = 24. Work of A for 1 day =
12
02. (D) (A + B)’s 1 day’s work = ( 1
+
30 40
1
) =
7
120 Work of B for 1 day =
1
8
∴ Time taken by both to finish the work ∴ Work of (A + B) together for 1 day
120 1
= days = 17 days 1 1 2+3 5
7 7 = + = =
12 8 24 24
03. (D) 25 men and 15 women can complete, a piece of
work in 12 days. ⇒ Work of (A + B) together for 3 days
8 2 5 5
∴ Work done by them in 8 days = = = 3× =
12 3 24 8
Remaining work is completed by 25 men in 6 days ⇒ Remaining work after 3 days
∴ Time taken by 25 men to complete the whole work 5 3
3×6 = 1– =
= = 18 days 8 8
1
˙·˙ C can do the same work in
From the question,
Time taken by 15 women to complete the whole 4
= th time required by (A + B)
1 1 36 5
work = = = = 36 days
( 1 1

12 18 ) ( ) 3–2
36
(3 – 2) 4 24 96
= × =
5 5 25
days
1 1 1 ⇒ Work of C for 1 day
[˙.˙ 12 – 18 = 36 work is completed by 15 women in
25
one day.] = part
96
Quantitative Aptitude Test | 169

25
96
part work can be done by C in 1 day 11. (A) (A + B)’s 5 day’s work = 5 (101 + 201 ) = 34
3
Remaining work = (1 – ) =
⇒ part work can be done by C in 3 1
8 4 4
96 3 1
= × days work is done by B in = 1 day
25 8 20
36 11
= day = 1
25 25
days
∴ The complete day C did the work

1
4
work is done by B in = 20 × ( 1
4 )
i.e., 5 days.

1
= 1 day. 12. (B) 1 man’s one day’s work =
96
1
08. (C) ˙·˙ of work is done by A in 5 days.
3
∴ Whole work will be done by A in 15 days.
12 men’s 3 day’s work = 3 × ( ) 1
8
=
3
8

2
˙·˙ of work is done by B in 10 days.
3
Remaining work = 1 – ( ) 3
8
=
5
8
15
( )
Whole work will be done by B in 10 ×
5
2
15 men’s 1 day’s work =
15
96
i.e., 25 days Now, work is done by them in 1 day
96
∴(A + B)’s 1 day’s work
= ( 1
+
15 25
1
) =
8
75
5
∴ work will be done by them in =
8 (
96 5
15 8
× ) i.e.,

= 4 days
75
So, both together can finish it in days, 13. (C) ˙·˙ [(A + B) + (B + C) + (C + A)]’s 1 day’s work
8
3
i.e., 9 days.
8
= (1
+
1
12 15 20
+
1
) =
1
5
1
1 ⇒ 2 (A + B + C)’s 1 day’s work =
09. (A) Work of A for 1 day = 5
15
1
1 ⇒ (A + B + C)’s 1 day’s work =
Work of B for 1 day = 10
20
Work of (A + B) together for 1 day
1 1 4+3 7
⇒ A’s 1 day’s work = (1 1

10 15 ) =
1
30
= + = = ∴ A alone can finish it in 30 days.
15 20 60 60
Remaining work after A alone does for 1 day
1 14
14. (A) (B + C)’s 2 day’s work = 2 (1
+
10 15
1
) =
1
3

7
= 1– =
15 15 Remaining work = 1 – ( ) 1
3
=
2
3
˙·˙ part-work can be completed by (A + B) in 1 1
60 ˙·˙ work is done by A in 1 day
day 9
14
∴ part-work can be completed by (A + B) in
15
2
∴ work is done by A in 9 ×
3 ( ) 2
3
= 6 days

=
60 14
7 15
× = 8 days. 15. (B) Work to be done by C = 1 – ( ) 7
11
=
4
11
1 7 4
10. (C) (A + B)’s 1 day’s work = ∴ (A + B) : C = : =7:4
18 11 11

(B + C)’s 1 day’s work =


1
24
∴ C’s share = Rs. 550 × ( 4
11 )
1 = Rs. 200
(A + C)’s 1 day’s work =
Adding 2 (A + B + C)’s 1 day’s work
36
16. (D) Time taken by Ramesh alone = ( )
2
3
×4

= ( 1
+
1
18 24 36
+
1
) =
1
8
8
= days
3
∴ (A + B + C)’s 1 day’s work
=
1
∴ Their 1 day’s work = (1 1 3
+ +
4 6 8 )
16 19
=
Hence, all working together can finish it in 16 days. 24

Quantitative Aptitude Test | 170


24 ∴ From (i),
So, together they can finish the work in days,
19 ⇒ 14 (5 men + 3 boys) can reap 23 × 14 hectares in
5 4 days …(iii)
i.e., 1 days.
19 Now, from (ii)
1 3 ⇒ 23 (3 men + 2 boys) can reap 7 × 2 × 23 hectares
17. (B) B’s 9 day’s work = 9 × =
12 4 in 4 days …(iv)
Remaining work = 1 – ( ) 3
4
=
1
4
˙·˙14 (5 men + 3 boys) = 23 (3 men + 2 boys)
⇒ 70 men + 42 boys = 69 men + 46 boys
1 1 ∴ 1 man = 4 boys
work is done by A in = 20 ×
4 4 Now, 5 men + 3 boys = 23 boys
= 5 days. ⇒ 23 boys can reap 23 hectares in 4 days
1 1 ⇒ 1 boy can reap 1 hectare in 4 days
18. (B) Umesh’s 5 day’s work = 5 × =
15 3 ⇒ 4 boys can reap 1 hectare in 1 day
Remaining work = 1 – ( ) 1
3
=
2
3
⇒ 4 × 45 boys can reap 45 hectares in 1 day
4 × 45
⇒ boys can reap 45 hectares in 6 days
( 1
+
10 15
1
)work is done by both in 1 day 6
∴ 30 boys can reap 45 hectares in 6 days
2
∴ work is done by both in 6 ×
3 ( ) 2
3
= 4 days But 30 boys = 28 boys + 2 boys
= 7 men + 2 boys
Hence, the work was completed in 4 + 5 = 9 days. Hence, 2 boys can assist 7 men for the work.
19. (C) A’s 10 day’s work = 10 ×( ) 1
80
=
1
8
1
22. (D) 12 men can complete of the work in 3 days and
3
2
Remaining work = 1 – ( ) 1
8
=
7
8
the remaining of the work in 6 days.
3
7 2
˙·˙ work is done by B in 42 days 1 man can complete of the work in (12 × 6)
8 3
= 72 days
⇒ Whole work will be done by B in 42 × ( )
8
7 2
∴ (12 – 2 + 6) = 16 men can complete of the work
= i.e., 48 days. 3
72 1
in = 4 days
∴ (A + B)’s 1 day’s work = (1
80 48
+
1
) 16 2
23. (B) Let x men were employed at first
8 1 1
= = ˙·˙ x men do of the work in 24 days
240 30 2
Hence, A and B together can finish it in 30 days. ∴ 1 man do the whole work in 24 × 2 × x
20. (D) A’s 10 day’s work = 10 ×( ) 1
25
=
2
5 Now, from the question,
= 48x days

Remaining work = 1 – ( ) 2
5
=
3
5 (
(x + 16) men do the remaining work 1 – =
1 1
2 2 ) in

( 1
+
25 20
1
)=
9
100
work was done by (A + B) in 1 day
(40 – 24 = 16) days
∴1 man do the whole work in 16 × 2 (x + 16) days

3
∴ work was done by (A + B) in
5 (100 3
9 5
× ) ∴
48x = 32 (x + 16)
x = 32 men
20 24. (C) 25 men reap the field in 20 days
= days 20 × 25
3 ∴ 10 men can reap the field in = 50 days.
10
(
Hence, the work lasted for 10 + 6
2
3 ) When 15 men leave the work, 10 men remain and
1
2 37
= 16 days. 1 2 3
3 they can reap in 37 days = = of the field.
2 50 4
21. (A) ˙·˙ (5 men + 3 boys) can reap 23 hectares in 4
days …(i) ( )
Hence, all men must work till 1 –
3
4
1
= of the
4
(3 men + 2 boys) can reap 7 hectares in 2 days. 1
field is reaped in × 20 = 5 days.
…(ii) 4
Quantitative Aptitude Test | 171
1 31. (D) ˙·˙ C’s 1 day’s work
25. (A) A’s 1 day’s work =
12 = [(A + B + C)’s 1 day’s work]
1 1
B’s 1 day’s work = + 60% of – [(A + B)’s 1 day’s work]
12 12
=
1 160 2
×
12 100 15
= = [ (
1
8

1
+
24 16 )] (
1
=
1 5

8 48 ) =
1
48
15 1 ∴ C alone can do it in 48 days.
∴ B can do the work in = 7 days.
2 2 1 1
32. (C)Ratio of times taken = : = 4 : 3
10 1 3 4
26. (D) 10 day’s work by 15 men = =
210 21
At the end of every 10 days 15 additional men are 33. (D) (A + B)’s 20 day’s work = 20 ×( ) 1
30
=
2
3
employed i.e., for the next 10 days we have
15 + 15 = 30 men
2
Remaining work = 1 –( ) 2
3
=
1
3
∴ Next 10 day’s work by 30 men = 1
21 work is done by A in 20 days
3
Hence in 20 days only ( 1
+
21 21 21
2
= )
3
work is com- Whole work can be done by A in (3 × 20) days
pleted. = 60 days.
To complete the whole work we have to reach the 34. (C)
value of ( )
21
21
work. 35. (D) ˙·˙ B’s daily earning = Rs. (150 – 94) = Rs. 56
⇒ A’s daily earning = Rs. (150 – 76) = Rs. 74
Now, ( 1
+
2
21 21 21
+
3
+ …… )
6
21
=
21
21
=1 ∴ C’s daily earning = Rs. [(150 – (56 + 74)]
Hence total time to complete the whole work = Rs. 20
= 10 + 10 + 10 + 10 + 10 + 10 36. (C) 37. (B) 38. (C) 39. (A) 40. (B)
= 60 days 41. (D) ˙·˙ Factory A turns out x cars in one hour. Factory
1 y
27. (A) Ramesh alone finished of the work in 10 days. B turns out cars in one hour.
2 2
1 ⇒ In one hour both the factories A and B can turn
Remaining of the job was finished by Ramesh and
2
Dinesh together in 2 days. ( )
out x +
y
2
cars
Therefore, they both together can finish the complete
∴ In 8 hours both factories turn out
job in 4 days.
28. (A) In 25 hours A can copy 75 pages
In 1 hour A can copy
75
= 3 pages
( )
=8 x+
y
2
cars = 4 (2x + y) cars.

25 42. (C) Let B takes x days to do the work.


In 25 hours (A and B) can copy 135 pages
In 1 hour (A and B) can copy =
135
27
= 5 pages ( 3
) 3x
∴ A takes 2 × x = days to do it.
4 2
∴ In 1 hour B can copy (5 – 3 = 2) pages 1
˙·˙ (A + B)’s 1 day’s work =
∴ B can copy 42 pages in 21 hours. 18
29. (A) ˙·˙( 1 1 1
+ +
5 9 15 ) =
17
45
work is finished in 1 hour. ⇒
1 2
x + 3x = 18
1

17 28 ∴ x = 30
∴ Remaining work = 1 – =
45 45
1 1 14 43. (B) Efficiency is proportional to work done per day.
⇒ (A + B)’s 1 hour’s work = + = Work done per day × number of days worked
5 9 45
= Amount of work done. Considering efficiency of A
14
work is done by (A and B) in 1 hour and B initially as 1.
45
Let A alone can do the work in x days and B alone
28
45
work will be done by A and B in ( 45 28
×
14 45) can do the same work in y days.
= 2 hours 5 5
Then,
30. (D) ˙·˙ Mohan moves the whole lawn in x hours. x + y = Total work done = 1
2 1
⇒ Mohan moves in 2 hours = x part of the lawn. Since efficiency of A and B are 2 and respectively
3
2 x–2 1 1 1
∴ x × 2 × 3 + y ×3 × 3 = 1
Unmoved part = 1 – x = x part ˙·˙

Quantitative Aptitude Test | 172


6 1 60 20
⇒ ∴
x+y = 1 …(i) x = = hours
9 3
1 1 1 = 6 hours 40 min
and x+y = 5 …(ii)
∴ The time at which the report was typed 03·40 p.m.
Now, subtracting equation (ii) from equation (i), we 62. (A) ˙·˙ In 5 days 14 workers make = 1400 toys
have 1400
∴ In 1 day 14 workers make toys =
25 1 5
x = = 6 days.
4 4 = 280 toys
44. (B) (A’s 1 day’s work) : (B’s 1 day’s work) = 2 : 1 Number of remaining toys = 1400 – 280
= 1120
1
Now,˙·˙ (A + B)’s 1 day’s work = and number of total workers = 14 + 14
14
= 28
⇒ A’s 1 day’s work = (1 2
×
14 3
=) 1
21 ˙.˙ 14 workers make 1400 toys in 5 days
5 × 14 × 1120
∴ A alone can finish the work in 21 days. ∴ 28 workers make 1120 toys =
28 × 1400

[ Dividing
1
14 ]
in the ratio 2 : 1
63.
= 2 days
(B) ˙·˙ In 12 days the work is done by = 24 boys
45. (B) 46. (A) 47. (A) 48. (D) 49. (C) 50. (C) 51. (B) ∴ In 1 day the work is done by = 24 × 12
52. (D) 53. (D) 54. (D) 55. (C) 56. (D) 57. (B) 58. (C) = 288 boys
59. (B) ˙·˙ In 9 days the work is done by = 16 men
60. (D) ˙.˙ In 20 days the work is completed by ∴ 1 day work is done by = 16 × 9
= 16 men + 12 women = 144 men
∴ In 1 day the work is completed by ⇒ 144 men = 288 boys
288
= 20 × (16 men + 12 women) ⇒ 1 man =
144
= 320 men + 240 women
= 2 boys
In 40 days the work is completed by 18 women ⇒ 12 men + 12 boys = 12 × 2 + 12
∴ 1 day the work is completed by = 36 boys
= 18 × 40 ˙.˙ 24 boys complete 1 work in 12 days
= 720 women 12 × 24
˙·˙ 720 women = 320 men + 240 women ∴ 36 boys complete 1 work in = 8 days
36
⇒ (720 – 240) women = 320 men 64. (A) ˙.˙ In 10 days a work is completed by 15 men
⇒ 480 women = 320 men ∴ In 1 day a work is completed by
480 = 15 × 10 = 150 men
∴ 1 man =
320 ˙.˙ In 5 days the work is completed by
3 = 20 boys
= women
2 ∴ 1 day the work is completed
3 = 20 × 15 = 300 boys
∴ 12 men + 27 women = 12 × + 27
2 ⇒ 150 men = 300 boys
= 45 women 300
or 1 man = = 2 boys
˙.˙ 18 women complete 1 work in 40 days 150
40 × 18 10 men = 2 × 10 = 20 boys
∴ 45 women complete 1 work = 10 men + 10 boys = 20 + 10
45
= 16 days = 30 boys
61. (D) ˙·˙ No. of pages typed by the typist A in 4 hours ˙.˙ 20 boys complete the work in 15 days
100 × 4 15 × 20
= = 40 ∴ 30 boys complete the work in
10 30
= 10 days
∴ No. of remaining pages = 100 – 40 = 60
65. (D) ˙.˙ In 12 days work done by 14 men = 1
Let B and C worked for x hours
1×4 1
100 × x 100 × x ∴ In 4 days work done by 14 men = =
˙·˙ + = 60 12 3
20 25 1 2
∴ Remaining work = 1 – =
⇒ 5x + 4x = 60 3 3

Quantitative Aptitude Test | 173


and number of total men = 14 + 2 In 24 days the work is done by
= 16 = 36 women
If 1 work is done by 14 men in 12 days ∴ In 1 day the work is done by
2 12 × 14 2 = (36 × 24) women
∴ work is done by 16 men in × = 7 days
3 16 3
˙·˙ (16 × 24) men = 36 × 24 women
66. (D) ˙.˙ 14 persons complete in 16 days = 1 work
36 × 24
1 × 8 12 ⇒ 1 man =
∴ 8 persons complete in 12 days = × 16 × 24
14 16
36 × 24
3 ⇒ 8 men = ×8
= 16 × 24
7
= 18 women
3 4
∴ Remaining work = 1 – = ∴ 8 men + 6 women = (18 + 6)
7 7
= 24 women
and total number of persons = 8 + 8 = 16
˙·˙ 36 women do 1 work in 24 days
∴ 14 persons do 1 work in 16 days
24 × 36
4 16 × 14 4 ∴ 24 women do 1 work in = 36 days
∴ 16 persons do work in × = 8 days 24
7 16 7
1
67. (A) Required time taken to complete the work by 71. (A) Work done by (A + B) in 1 day =
xy 24
both together = 1 1
x+y Work done by B alone in 1 day = =
3 × 12 36
8 × 4 32
(Here x = 8 and y = 4) = = 1 1 1
8 + 4 12 ∴ Work of A for 1 day = – =
24 36 72
2
= 2 hours 1 2
3 After 12 days the remaining work = 1 – =
3 3
68. (B) Work of (A + B + C) for 1 hour
1
1 1 1 37 ˙.˙ work is done by A in 1 day
= + + = 72
16 20 24 240
2 72 2
∴ Work of (A + B + C) for 4 hours ∴ work is done by A in 1 × × = 48 days
3 1 3
37 × 4 37
= = 72. (D) ˙·˙ Work of (Ganesh, Ram and Sohan) for 1 day
240 60
37 23 1
∴ Remaining work = 1 – = =
60 60 16
∴ Work of (B + C) for 1 hour and work of Ganesh and Ram for 1 day
1 1 11 1
= + = =
20 24 120 24
11 1 1 1
˙.˙ work is done by (B + C) in 1 hour ⇒ Work of Sohan for 1 day = – =
120 16 24 48
23 ∴ Sohan alone will complete the work in
∴ work is done by (B + C)
60
120 23
(
= 1÷
1
48) days
= ×
11 60 = 48 days
46 73. (C) Required time taken by (A + B) to complete the
= hours xy
11 work =
= 4 hours 11 min. x+y
∴ The type at which the report was typed [Here x = 6 and y = 3]
= 01 : 00 + 04 : 11 6×3
= = 2 days
= 05 : 11 p.m. 6+3
69. (C) Required time taken by B to complete the work 74. (A) ˙·˙ Work of (Ram + Dilip + Shekhar) for 1 day
xy 1
= =
x–y 20
[Here x = 12 and y = 8] and work of (Ram + Dilip) for 1 day
12 × 8 96 1
= = =
(12 – 8) 4 30
= 24 days ∴ Work of Shekhar for 1 day
70. (A) In 16 days the work is done by 24 men 1 1 1
= – =
∴ In 1 day the work is done by 20 30 60
= (24 × 16) men Hence, Shekhar will complete the work in 60 days.

Quantitative Aptitude Test | 174


Probability
Important Points/Facts (i) E (an even no.) = {2 4 6}, days. The two days can be {Sunday
n (E) = 3 and Monday, Monday and Tuesday,
(1) Probability of the occu- Tuesday and Wednesday, Wednes-
rrence of at least one of several n (E) 3 1 day and Thursday, Thursday and
independent events of a random ∴ P (E) = = =
n (S) 6 2 Friday, Friday and Saturday, Satur-
experiment—If A1 , A2 , A3 , …… An day and Sunday} i.e. n (S) = 7
(ii) E (an odd no.) = {1, 3, 5}
– – –
are independent events then A1 A2 A3 n (E) = 3 Out of these 7 cases, cases
– favourable for one more Sunday are
……… An are also independent n (E) 3 1
∴ P (E) = = = {Sunday and Monday, Saturday
events then probability that at least n (S) 6 2
and Sunday}
one of the events occurs. (iii) E (a no. divisible by 2) i.e. n (E) = 2
= 1 – probability of none of = {2, 4, 6}, n (E) = 3
events occurs 2
3 1 ∴ P (E) =
∴ P (E) = = 7
= 1 – (1 – p1) (1 – p2 ) … (1 – pn ) 6 2 (ii) When the year is not a leap
where p1 p 2 p 3 ……… pn are (iv) E (a no. divisible by 3) year. It has 52 complete weeks and 1
probabilities of events A1 , A2 , A3 , = {3, 6} n (E) = 2 more day that can be {Sunday, Mon-
…… An . day, Tuesday, Wednesday, Thurs-
2 1
(2) Probability of occurrence ∴ P (E) = = day, Friday, Saturday} n (S) = 7.
6 3
of exactly one of the two indepen- Out of these 7 cases, cases
(v) E (a no. less than 4)
dent events—p1 q2 + p 2 q1 , where E favourable for one more Sunday is—
and F are two events with probability = {1, 2, 3} n (E) = 3
{Sunday}, n (E) = 1
p and q respectively. 3 1
∴ P (E) = = 1
(3) Bayer’s formula or Bayer’s 6 2 ∴ P (E) =
7
theorem on probability. (vi) E (a no. less than or equal
Q. 3. When two dice are thr-
to 4) = {1, 2, 3, 4}
Let H1 , H2 ……… H n be a set of own, what is the probability that :
mutually exclusive and exhaustive n (E) = 4
(i) Sum of numbers appeared
events and E is any other event, then 4 2 is 6 and 7 ?
∴ P (E) = =
n 6 3
(ii) Sum of numbers appeared
(i) P (E) = Σ P (E ∪ Hi) (vii) E (a no. greater than 6) =
i=1 ≤8?
{}, i.e., there is no number greater
(ii) P (E/Hi). P (Hi) (iii) Sum of numbers is an odd
than 6 in the sample space.
no. ?
= P (H/E i) ·P (E) = P (E ∩ Hi) 0
∴ P (E) = = 0 (iv) Sum of numbers is a
6
P (E ∪ Hi) multiple of 3 ?
(iii) P (Hi/E) = Probability of an impossible
P (E) (v) Numbers shown are equal ?
event = 0.
P (E/Hi) P (Hi) (viii) E (a no. less than or equal (vi) The difference of the num-
= n
to 6) = {1, 2, 3, 4, 5, 6}, bers is 2 ?
Σ P (E/Hi) P (Hi)
i=1 n (E) = 6 (vii) Sum of the numbers is at
least 5 ?
6
Examples ∴ P (E) = = 1
6 Solution :
Q. 1. A dice is thrown. What is ˙·˙ Probability of a certain event (i) For 6, reqd. probability
the probability that the number = 1 n (E) 5
shown on the dice is (i) an even no. = =
0 < P (E) < 1 n (S) 36
(ii) an odd no. (iii) a no. divisible 2 Q. 2. (i) What is the chance For 7, reqd. probability
(iv) a no. divisible by 3, (v) a no. that a leap year selected randomly
less than 4. (vi) a no. less than or will have 53 Sundays ? 6 1
equal to 4. (vii) a no. greater than = =
36 6
6. (viii) a no. less than or equal to 6. (ii) What is the chance, if the
year selected is a not a leap year (ii) Desired sums of the numbers
Solution : will have 53 Sundays ? are 2, 3, 4, 5, 6, 7 and 8.
In all the above cases Solution : n (S) = 1 + 2 + 3 + 4
S = {1, 2, 3, 4, 5, 6} (i) A leap year has 366 days so it +5+6+5
n(S) = 6 has 52 complete weeks and 2 more = 26
Quantitative Aptitude Test | 175
∴Reqd. probability 26 1 bability that balls drawn contain
(i) P (E) = = exactly two green balls ?
26 13 52 2
= =
36 18 ⎡⎢ 26C1 26 ⎤⎥ Solution :
(iii) Desired sums of the num- ⎢⎣ or 52C1 = 52 (˙.˙ n C1 = n)⎥⎦ Total no. of balls
bers are 3, 5, 7, 9 and 11 = 3+5+4
13 1
n (S) = 2 + 4 + 6 + 4 + 2 (ii) P (E) = = = 12
52 4
= 18 n (S) = 12C 3
4×2 2
∴Reqd. probability (iii) P (E) = = 12 × 11 × 10
52 13 = = 220
18 1 4 1 3×2
= = (iv) P (E) = =
36 2 52 13 2 green balls can be selected
(iv) Desired sums of the num- 5×4 5 from 4 green balls in 4 C 2 ways and
bers are 3, 6, 9 and 12, (v) P (E) = = the rest one ball can be selected from
52 13
n (S) = 2 + 5 + 4 + 1 the remaining (12 – 4) = 8 balls in
3×4 3 8 C ways.
(vi) P (E) = = 1
= 12 52 13
n (E) = 4 C 2 × 8C 1 = 6 × 8 = 48
∴Reqd. probability 4 1
(vii) P (a king) = = 48 12
12 1 52 13 ∴P(E) = =
= = 220 55
36 3 4 1
P (a queen) = =
(v) Events = {1, 1}, {2, 2}, 52 13 Short Method : Applying the
{3, 3}, {4, 4}, {5, 5}, {6, 6}, n(S) = 6 ∴ P (a king or a queen) theorem, we have the reqd. answer
6 1 1 1 2 3 × 4 × (4 – 1) × 8
∴ P (E) = = = + = =
36 6 13 13 13 12 × 11 × 10
9 3 × 4 × 3 × 8 12
(vi) Events = {3, 1}, {4, 2}, = =
(viii) P (E) = 12 × 11 × 10 55
{5, 3}, {6, 4}, {4, 6}, {3, 5}, {2, 4}, 52
{1, 3}, n (S) = 8 2 1 Q. 7. A bag contains 3 red, 5
(ix) P (E) = = yellow and 4 green balls. 3 balls are
8 2 52 26
∴ P (E) = = drawn randomly. What is the
36 9 Q. 5. A bag contains 3 red, 5
probability that the balls drawn
(vii) Events; either 2 or 3 or 4 yellow and 4 green balls. 3 balls are
contain no yellow ball ?
or 5 drawn randomly. What is the
probability that the balls drawn Solution :
n (E) = 1 + 2 + 3 + 4 = 10 contain balls of different colours ? Total no. of balls
n (S) = 36 Solution : = 3 + 5 + 4 = 12
n (E) 10 5 Total no. of balls n (S) = 12C 3
∴ P (E) = = =
n (S) 36 18 = 3 + 5 + 4 = 12 12 × 11 × 10
=
Q. 4. A card is drawn from a n (S) = 12C 3 3×2
pack of cards. What is the proba- 12 × 11 × 10 = 220
bility that it is : = 3 balls can be selected from
3×2
(i) a card of black suit ? 3 (red) + 4 (green)
= 220
(ii) a spade card ? = 7 balls
In order to have 3 different
(iii) an honours card of red coloured balls the selection of one = 7 balls in 7 C 3 ways
suit ? ball of each colour is to be made. 7×6×5
n(E) = 7 C 3 = = 35
(iv) an honours card of club ? n (E) = 3 C 1 × 5C 1 × 4C 1 3×2
= 3 × 5 × 4 = 60 35 7
(v) a card having the number ∴P (E) = =
less than 7 ? 60 3 220 44
∴ P (E) = =
(vi) a card having the number 220 11 Short Method : Applying the
a mutiple of 3 ? Short Method : Applying the theorem, we have the reqd. answer
(vii) a king or a queen ? theorem, we have the required 7×6×5
answer =
(viii) a digit card of heart ? 12 × 11 × 10
6 ×3×5×4 3 35 7
(ix) a jack of black suit ? = = . = = .
12 × 11 × 10 11 220 44
Solution :
Q. 6. A bag contains 3 red, 5
For all the above cases yellow and 4 green balls. 3 balls are Q. 8. There are 4 boys and 4
n (S) = 52C 1 = 52 drawn randomly. What is the pro- girls. They sit in a row randomly.

Quantitative Aptitude Test | 176


What is the chance that all the girls Short Method : Applying the Q. 11. A bag contains 5 black
do not sit together ? theorem, and 7 white balls. A ball is drawn
Solution : out-of it and replaced in the bag.
Reqd. answer Then a ball is drawn again. What
Total no. of arrangements 4 (4 – 1) + 3 (3 – 1) + 5 (5 – 1) is the probability that (i) both the
= n (S) = 8P 8 = 8! =
(4 + 3 + 5) (4 + 3 + 5 – 1) balls drawn were black (ii) both
Consider all the 4 girls as one, 12 + 6 + 20 19 were white (iii) the first ball was
= = white and the second black (iv) the
we have 4 boys + 1 girl = 5 persons. 12 × 11 66
Which can be arranged in 5 P 5 = 5! first ball was black and the second
Note : The probability that both white ?
ways. But the girls can also be the balls are not of the same colour is
arranged in 4 P 4 = 4! ways among Solution :
given by (1 – P) (probability of the
themselves. same colour). The events are independent and
So, in 4! × 5! ways can the Case II. If r = 3, then the for- capable of simultaneous occurrence.
persons be arranged so that girls are The rule of multiplication would be
mula for required probability is given
applied.
4 × 5 by
together = The probability that
8! ⎡x (x – 1) (x – 2) + y (y – 1)⎤
P (girls are togeher) ⎢ (y – 2) + z (z – 1) (z – 2) ⎥ (i) Both the balls were black

4×3×2 1 ⎢ (x + y + z) (x + y + z – 1) ⎥ =
5 5
× =
25
= =
8 × 7 × 6 14 ⎣ (x + y + z – 2) ⎦ 12 12 144
(ii) Both the balls were white
∴ P (All girls are not together) Q. 10. A bag contains 5 red
7 7 49
= 1–P and 8 black balls. Two draws of = × =
three balls each are made, the ball 12 12 144
(All girls are together)
being replaced after the first draw. (iii) The first was white and the
1 13 What is the chance that the balls second black
= 1– =
14 14 were red in the first draw and 7 5 35
black in the second ? = × =
12 12 144
Short Method : Applying the Solution :
theorem, we have the required (iv) The first was black and the
Total no. of balls second white
answer
= 5 + 8 = 13 5 7 35
5! 4! = × =
P (E) = 1 – 11 × 12 × 13 12 12 144
8! n (S) = 13C 3 =
1×2×3 From the above example we can
5! 4! 1
= 1– =1– = 286 see that how the quicker methods for
8! 14
Chance that the balls were red in such questions have been derived.
13 5C
= Q. 12. A bag contains 6 red
14 first draw = 13 3
C3 and 3 white balls. Four balls are
Chance that the balls were black drawn out one by one and not
Q. 9. A box contains 4 black 8C replaced. What is the probability
balls, 3 red balls and 5 green balls. in the second draw = 13 3 that they are alternatively of
2 balls are drawn from the box at C3 different colours ?
random. What is the probability [˙.˙ balls are replaced Solution :
that both the balls are of the same after first draw]
colour ? Balls can be drawn alternately in
Required probability the following order. Red, White, Red,
Solution : 5C 8C White, or White, Red, White, Red.
140
Total no. of balls = 13 3 × 13 3 =
C3 C 3 20449 If red ball is drawn first, the
= 4 + 3 + 5 = 12 In the above example the two probability of drawing the balls alter-
events are independent and can natively
12 × 11
n (S) = 12C 2 = = 66 occur simultaneously. So, we used 6 3 5 2
2 = × × × …(i)
multiplication. 9 8 7 6
n (E) = 4 C 2 + 3 C 2 + 5 C 2
Short Method : Applying If white ball is drawn first the
4×3 3×2 5×4 theorem, we have the required probability of drawing the balls alter-
= + +
2 2 2 probability nately
= 6 + 3 + 10 = 19 (5 × 4 × 3) × (8 × 7 × 6) 3 6 2 5
= = × × × …(ii)
∴ Reqd. probability, P(E) (13 × 12 × 11)2 9 8 7 6
n (E) 19 20160 140 Required probability (i) + (ii)
= =
= = 2944656 20449
n(S) 66 … (*)

Quantitative Aptitude Test | 177


6 3 5 2 3 6 2 5 “A bag contains ‘x’ white and ‘y’ Then n (S) = the number of
= × × × + × × ×
9 8 7 6 9 8 7 6 red balls. If two balls are drawn in points on which B can stand = 11.
5 5 5 succession at random, then the pro- If there be exactly 3 persons bet-
= + = bability that one of them is white and ween A and B, then corresponding to
84 84 42
the other red is given by any position occupied. B can take up
Short Method : Applying the ⎡⎢ 2xy ⎤⎥ only two position the 4th place and
theorem, we have the required
probability ⎢⎣ (x + y) (x + y – 1)⎥⎦ the 8th place as counted from A.
Thus n (E) = 2
⎡ 6 × 3 × (6 – 1) × (3 – 1)⎤⎥ Q. 14. A basket contains 3
= ⎢⎢ ×2 n (E) 2
+ 3) (6 + 3 – 1) ⎥ white and 9 black balls. There is ∴ P (E) = =
⎢⎣ (6 + (6 ⎥
3 – 2) (6 + 3 + 3) ⎦ another basket which contains 6 n (S) 11
white and 8 black balls. One ball is Short Method : Applying the
= [69 ×× 38 ×× 57 ×× 26] × 2 = 425 to be drawn from either of the two
baskets. What is the probability of
theorem we have the required pro-
2 2
Note : Wherever we find the drawing a white ball ? bability = =
(10 + 1) 11
word AND between two events, we Soultion :
use multiplication. Mark that both Since there are two baskets, each Q. 16. 10 persons are seated at
also means first and second. On the equally likely to be chosen the a round table. What is the probabi-
other hand, if the two events are probability of choosing either basket lity that two particular persons sit
joined with OR we use addition as in 1 together ?
is . Solution :
the above example. 2
Q. 13. A bag contains 4 white If the first basket is chosen, the n (S) = no. of ways of sitting 10
and 6 red balls. Two draws of one probability of drawing a white ball persons at round table = (10 – 1) !
ball each are made without repla- 1 3C 1 3 1 = 9 since 2 particular persons will be
cement. What is the probability = × 12 1 = × = always together. Then the no. of
2 C 1 2 12 8
that one is red and other white ? persons = 8 + 1 = 9.
Similarly, if the second basket be
Solution : chosen, the probability of drawing a ∴ 9 persons will be seated in
Such problems can be very easily 1 6C 1 6 3 (9 – 1) ! = 8 ! ways at round table and
solved with the help of the rules of white ball = × 14 1 = × = 2 particular persons will be seated
2 C 1 2 14 14
permutation and combination. themselves in 2 ! ways
Since, the two events are mutu-
Two balls can be drawn out of ∴ The number of ways in which
ally exclusive, we use addition,
10! 10 × 9 two persons always sit together at
10 balls in 10C2 = = = 45 therefore, the probability of drawing
2! 8! 2 round table = 8! 2! = n (E)
a white ball from either basket is
ways. n (E) 8! × 2!
1 3 7 + 12 ∴ P(E) = =
One white ball can be drawn out P (E) = + = n (S) 9!
8 14 56
of 4 white balls in 19 8! × 2 2
4 C = 4! = 4 ways
= = =
1 56 9 × 8! 9
1! 3!
One red ball can be drawn out of Short Method : Applying the Short Method : Applying the
6 red balls in 6 C 1 = 6 ways. The total theorem we have the required pro- theorem we have the required pro-
number of ways of drawing a white
and a red ball are 4 C 1 × 6C1 = 4 × 6 =
bability =
1 3
+[
2 12 14
6 19
= .
56] bability =
(10 – 2)! 2! 8! 2! 2
(10 – 1)!
=
9!
= .
9
24 Q. 15. A and B stand in a ring Q. 17. An unbiased coin is
The required probability would with 10 other persons. If the tossed 7 times, find the chance that
be arrangement of the 12 persons is at exactly 5 times head will apear.
No. of cases favourable random. What is probability that Solution :
to the event there are exactly 3 persons between
= Here, n = 7, r = 5
Total no. of ways in A and B ?
which the event can happen Solution : 1
p = Probability of happening =
24 8 7 2
= = 6 8
45 15 q = Probability of not happening
B 5 B 1
Short Method : Applying the =
theorem, we have the required 9
2
4
probability ∴ Required probability
3 10
2×6×4 8
(12) × (12)
5 7–5
= = 2 11 = 7C ×
10 × 9 15 A
5

The above theorem may be put Let A stand on some point of the 21
as given below : =
ring. 128
Quantitative Aptitude Test | 178
Exercise (A)
3
(B)
3 14. In a simultaneous throw of two
5 8 coins, the probability of getting
01. Ticket numbered 1 to 20 are 1 1 at least one head is—
mixed up and then a ticket is (C) (D) 1 2
3 5 (A) (B)
drawn at random. What is the 2 3
probability that the ticket drawn 08. A bag contains 5 blue and 4
black balls. Three balls are 3 1
bears a number which is a (C) (D)
multiple of 3 or 7 ? drawn at random. What is the 4 3
1 1 probability that 2 are blue and 1 15. In a throw of a coin, the proba-
(A) (B) is black ? bility of getting a head is—
15 2
2 7 1 1
(C) (D) (A) (A)
5 20 3 2
2 1
02. Ticket numbered 1 to 20 are (B) (B)
5 4
mixed up and then a ticket is
drawn at random. What is the 1 (C) 1
(C)
probability that the ticket drawn 6 (D) None of these
bears a number which is a (D) None of these 16. The probability that a teacher
multiple of 3 ? 09. A bag contains 8 red and 5 white will give one surprise test during
3 3 balls. 2 balls are drawn at 1
(A) (B) any class meeting in a week is .
20 10 random. What is the probability 5
2 1 that both are white ? If a student is absent twice. What
(C) (D) is the probability that he will
5 2 5 2
(A) (B) miss at least one test ?
03. What is the probability that a 16 13
4 1
number selected from the num- 3 5 (A) (B)
(C) (D) 15 15
bers 1, 2, 3, 4, 5 …… 16 is a 26 39
91 16
prime number ? (C) (D)
10. A bag contains 6 black balls and 25 125
1 5 8 white balls. One ball is drawn
(A) (B) 17. Out of 15 students studying in a
16 8 at random. What is the pro- class 7 are from Maharastra, 5
3 7 bability that the ball drawn is
(C) (D) are from Karnataka and 3 are
8 16 white ? from Goa. Four students are to
04. An urn contains 9 red, 7 white 4 3 be selected at random. What are
(A) (B)
and 4 black balls. A ball is 7 4 the chances that at least one is
drawn at random. What is the 4 1 from Karnataka ?
probability that the ball drawn is (C) (D) 12 11
5 8 (A) (B)
not red ? 13 13
11. A fair coin is tossed 100 times.
1 9 10 1
(A) (B) The probability of getting head (C) (D)
11 20 15 15
an odd number of times is—
2 11 18. In a box carrying one dozen of
(C) (D) 1 2
11 20 (A) (B) oranges, one third have become
4 3
05. In a lottery there are 20 prizes bad. If 3 oranges are taken out
1 3 from the box at random, what is
and 15 blanks. What is the pro- (C) (D)
2 4 the probability that at least one
bability of getting prize ?
12. Three unbiased coins are tossed. orange out of the three oranges
1 2
(A) (B) What is the probability of getting picked up is good ?
10 5
at most 2 heads ? 1 54
4 2 (A) (B)
(C) (D) 55 55
7 7 1 3
(A) (B) 45 3
4 8 (C) (D)
06. The odds against the occurrence 55 55
of an event are 5 : 4. The proba- 7 1
(C) (D) 19. A coin is successively tossed
bility of its occurrence is— 8 2
two times. Find the probability
4 4 13. Three unbaised coins are tossed, of getting :
(A) (B)
5 9 what is the probability of getting (1) exactly one head
1 1 exactly two heads ? (2) at least one head
(C) (D)
5 4 1 3 1 3 2 1
(A) (B) (A) , (B) ,
07. The odds in favour of an event 3 4 2 4 3 4
are 3 : 5. The probability of 2 3 1 4 1 2
(C) (D) (C) , (D) ,
occurrence of the event is— 3 8 4 5 2 3

Quantitative Aptitude Test | 179


20. In a simultaneous throw of two 26. What is the probability of getting passing in atleast one subject
dice find the probability of a king or a queen in a single 50% chance of passing in atleast
getting a total of 8. drawn from a pack of 52 cards ? two subject and 40% chance of
2 1 passing in exactly two. Which of
(A) (A) the following relation are true ?
9 26
5 1 19
(B) (B) (A) p + m + c =
36 13 20
1 2 17
(C) (C) (B) p + m + c =
6 13 20
(D) Data inadequate (D) None of these 1
(C) pmc =
10
21. What is the probability that a 27. A card is drawn from a pack of 1
leap year selected randomly will 52 cards. A card is drawn at (D) pmc =
4
have 53 Mondays ? random. What is the probability
2 that it is neither a heart nor a 33. If the integers m and n are
(A) king ? chosen at random between 1 and
7
5 4 9 100, then the probability that a
(B) (A) (B) number of the form 7m + 7n is
7 13 13
2 4 divisible by 5 equals ?
1 (C) (D)
(C) 13 13 1 1
7 (A) (B)
4 7
(D) Data inadequate 28. A card is drawn at random from
1 1
a pack of 52 cards. What is the (C) (D)
22. What is the probability that an 8 49
probability that the card drawn is
ordinary year has 53 Sundays ?
a spade or a king ? 34. There are four machines and it is
53 1
(A) (B) 4 3 known that exactly two of them
365 7 (A) (B) are faulty. They are tested. One
13 13
2 48 by one in a random order till
(C) (D) 2 1
7 53 (C) (D) both the faulty machines are
13 13
23. One card is drawn at random identified. Then the probability
29. A basket contains 3 blue. 5 black that only two tests are needed
from a pack of 52 cards. What is
and 3 red balls. If two balls are is—
the probability that the card
drawn at random what is the 1 1
drawn is a face card ?
probability that none of them is (A) (B)
4 1 3 6
(A) (B) blue ?
13 4 1 1
21 3 (C) (D)
9 1 (A) (B) 2 4
(C) (D) 55 55
52 13 28 9 35. If from each of the three boxes
(C) (D) containing 3 white and 1 black, 2
24. One card is drawn at random 55 11
from a pack of 52 cards. What is white and 2 black and 1 white
30. If 2 balls are drawn at random and 3 black balls, one ball is
the probability that the card what is the probability that one is
drawn is either a red card or a drawn at random, then the
black and one is red ? probability that 2 white and 1
king ? 2 8 black ball will be drawn is—
6 1 (A) (B)
(A) (B) 11 11 13 1
13 2 9 3 (A) (B)
(C) (D) 32 4
7 27 11 11
(C) (D) 1 3
13 52 (C) (D)
31. If 3 balls are drawn at random 32 16
25. Two cards are drawn at random what is the probability that all 36. Seven white balls and three black
from a pack of 52 cards. What is are black ? balls are randomly placed in a
the probability that the drawn 2 1 row. The probability that no two
cards are both aces ? (A) (B)
23 11 black balls are placed adjacently
1 3 8 equals.
(A) (C) (D)
221 11 33 1 7
(A) (B)
2 2 15
(B) 32. The probabilities that 9 students
13 2 1
pass in Mathematics, Physics (C) (D)
2 and Chemistry are m . p and c 15 3
(C)
26 respectively, of these subjects, 37. A box contains 3 white and 2 red
(D) None of these the students has a 75% chance of balls. If we draw one ball and

Quantitative Aptitude Test | 180


without replacing the first ball. The probability that neither of 99% probability of getting at
The probability of drawing red them is alive after 10 years is— least one head is—
ball in the second draw is— 1 1 (A) 5 (B) 7
(A) (B)
8 2 2 12 (C) 6 (D) 8
(A) (B)
25 5 7 3
(C) (D) 50. If an integer is selected at
3 21 12 4 random from 1 to 100. The pro-
(C) (D)
5 25 44. A fair coin is tossed 100 times. bability that it is relatively prime
The probability of getting tails to 100 is—
38. A rifleman is firing at a distant
target and has only 10% chance an odd number of times is— 2 61
(A) (B)
of hitting it. The number of least 1 5 100
(A)
rounds, he must fire in order to 2 17 19
(C) (D)
have more than 50% chance of 1 100 100
hitting it atleast once is— (B)
8 51. n books are to be arranged on a
(A) 5 (B) 7 3 self. These include m volumes of
(C)
(C) 9 (D) 11 8 a science book (m < n). The
(D) None of these probability that in any arrange-
39. A six-faced dice is so biased that ment, the volumes of science
is twice as likely to show an 45. The probability that a person
will hit a target in shooting books are in ascending order
even number as an odd number is—
when throw. It is thrown twice. practice is 0.3. If he shoots 10
times. Then the probability of his 1 1
The probability that the sum of (A) (B)
two numbers thrown is even is— shooting the target is— n n–m
(A) 1 (B) 1 – (0·7)10
1 1 10 m
(A) (B) (C) (0·7) (D) (0·3)10 1
12 6 (C) (D)
46. A and B are two independent m n
1 5 events. The probability that both
(C) (D)
3 9 1 52. If P (A) = 0·3, P(B) = 0·4,
A and B occurs is and the
40. The chance of throwing a total of 6 P (C) = 0·8
3 or 5 or 11 with two dice is— probability that neither of them P (AB) = 0·08, P (AC) = 0·28
1 P (BC) = P, P (ABC) = 0·09
5 1 occurs is . Then the probability
(A) (B) 3
36 9 P (A + B + C) ≥ 0·75
of the two events are respecti-
2 19 vely. then—
(C) (D) (A) 0·23 ≤ P ≤ 0·48
9 36 1 1 1 1
(A) and (B) and (B) 0·32 ≤ P ≤ 0·72
2 3 5 6
41. If the probability for A to fail in (C) 0·25 ≤ P ≤ 0·71
1 1 2 1
an examination is 0.2 and that (C) and (D) and (D) None of these
for B is 0.3, then the probability 2 6 3 4
that either A or B fails is— 47. The probability that a marksman 53. The probability that a man aged
1 x years will die in a year is p.
(A) 0·38 (B) 0·44 will hit a target is given as .
5 The probability that out of n men
(C) 0·50 (D) 0·94 M1 , M2, M3, …… Mn each aged
Then his probability of atleast
42. The probability of occurrence of one hit in 10 shots, is— n years. Mk will die and be the
5
an event A is . The probability
9
(A) 1 – ()4 10
5
(B)
1
510
first to die is—
(A) 2
1
of non-occurrence of the event B
5
is . The probability that atleast
(C) 1 – 10
5
1
(D) () 4 10
5
n
(B) 1 – (1 – p) n
11
48. A natural number is selected at 1
one of them will occur. (C) 2
random from the set n [1 – (1 – p)n
6 5 x = {x : 1 ≤ x ≤ 100}. The proba-
(A) (B) 1
11 9 bility that the number satisfies (D) n [1 – (1 – p)n]
4 the inequality x2 – 13x ≤ 30 is—
(C) (D) 0·8 54. If the probability of machine
9 5 9
(A) (B) failing during a day is 0·95 the
9 50 probability of its working for
43. The probability that a man lives
3 7 four consecutive days without
1 (C) (D)
after 10 years is and that his 20 9 failing is—
4
1 49. The number of tosses that have (A) 0·00000625
wife is alive after 10 years is .
3 to be made in order that there is (B) 0·0625

Quantitative Aptitude Test | 181


(C) 0·16548375 60. If n integers taken at random are (A) 1/5
(D) 0·000625 multiplied together then the (B) 1/8
probability that the last digit of (C) 1/6
55. Seven white balls and three the product is 2, 4, 6, 8 is—
black balls are randomly placed (D) None of these
in a row. The probability that no 2n 4n – 2n
(A) n (B) 66. A bag contains 7 white and 9 red
two black balls are placed 5 5n
n balls. The probability of drawing
adjacently equals— 4 8 – 4n
n
(C) n (D) a white ball is—
7 5 5n
(A) 1/2 (B) 1 1
15 61. Two dice and two coins are (A) (B)
16 52
2 1 tossed. The probability that both 7 7
(C) (D) (C) (D)
15 3 the coins show heads and the 52 16
sum of the numbers found on the
56. The probability of occurrence of 67. A card is drawn at random from
two dice is a prime number is—
a multiple of 2 on a dice and a pack of 100 cards numbered 1
multiple of 3 on the other dice. If 5 1
(A) (B) to 100. The probability drawing
both are thrown together is— 72 12
a number which is a square is—
7 1 13 5
(A) (B) (C) (D) (A) 1/5
26 3 144 48
(B) 2/5
71 1 62. When two dice are thrown, the
(C) (D) (C) 1/10
36 4 probability that the difference of
the number on the dice is 2 or 3 (D) None of these
3 – 1
57. If P (B) = , P (A ∩ B ∩ C) = is— 68. In suffling a pack of card 3 are
4 3
7 3 accidentally dropped then the
– – 1 (A) (B)
and P (A ∩ B ∩ C) = , then 18 11 chance that missing card should
3
5 1 be of different suit is—
P (B ∩ C) is— (C) (D)
18 2 (A) 169/425
(A) 1/12 (B) 1/6 (B) 261/425
(C) 1/15 (D) 1/9 63. From a set of 17 cards numbered
1, 2, 3……… 17 one is drawn at (C) 104/425
58. Two numbers are selected random. The probability that the (D) None of these
randomly from the set S = {1, 2, number is divisible by 3 or 7
3, 4, 5, 6} without replacement 69. A card is drawn from a well
is— shuffled pack of cards. The pro-
one by one. The probability that
(A) 2/17 (B) 1/7 bability of getting a queen of
minimum of the number is less
than 4 is— (C) 7/17 (D) 10/17 club or king of heart is—
(A) 1/15 (B) 14/15 (A) 1/52
64. The probability of getting heads
(C) 1/5 (D) 4/5 in both trials when a balanced (B) 1/26
coin is tossed twice will be— (C) 1/13
59. Four whole number taken at
random are multiplied together. (A) 1/4 (B) 1/2 (D) None of these
The chance that the last digit in (C) 1 (D) 3/4 70. The probability that a card
the product is 1, 3, 7 or 9 is— drawn from a pack of 52 cards
65. A number is chosen at random
16 1 will be a diamond or king being
(A) (B) among the first 120 natural
625 210 to—
numbers. The probability of the
8 4 number chosen being a multiple (A) 4/52 (B) 4/13
(C) (D)
125 25 of 5 or 15 is— (C) 1/52 (D) 2/13

Answers with Hints


01. (C) Clearly, n (S) = 20 and E = {3, 6, 9, 12, 15, 18, 03. (C) S = {1, 2, 3, ……, 16}
7, 14} i.e., n (E) = 8. and E = {2, 3, 5, 7, 11, 13}
n (E) 6 3
n (E) 8 2 ∴ P (E) = = =
∴ P (E) = = = n (S) 16 8
n (S) 20 5 9 9
04. (D) P (red) = =
02. (B) S = {1, 2, 3, …… 20} 9 + 7 + 4 20
and E = {3, 6, 9, 12, 15, 18} ∴ P (not-red) = 1 – ( 9
20) =
11
20
n (E) 6 3 20 20 4
∴ P (E) = = = 05. (C) P (getting a prize) = = =
n (S) 20 10 (20 + 15) 35 7
Quantitative Aptitude Test | 182
06. (B) Number of cases favourable to E = 4 E = Event of getting exactly two
Total number of cases = (5 + 4) = 9 heads
4 = [HHT, HTH, THH}
∴ P (E) =
9 n (E) 3
∴ P (E) = =
07. (B) Number of cases favourable to E = 3 n (S) 8
Total number of cases = (3 + 5) = 8 14. (C) S = {HH, HT, TT, TH}
3 and E = {HH, HT, TH}
∴ P (E) =
8 n (E) 3
∴ P (E) = =
08. (C) Let S be the sample space and E be the event of n (S) 4
drawing 3 balls out of which 2 are blue and 1 is black. 15. (A) Here S = {H, T}
Then, n (S) = Number of ways of drawing 3 balls out
and E = {H}
9×8×7
of 9 = 9 C 3 = = 84 and n (E) = Number of n (E) 1
3×2×1 ∴ P (E) = =
ways of drawing 2 balls out of 5 and 1 ball out of 4. n (S) 2
5×4 16. (B) The probability of absenting of the student in
= 5 C2 + 4 C1 = ( 2×1 )
+ 4 = 14
the class = =
2 1
6 3
n (E) 14 1
∴ P (E) = = = 1 1 1
n (S) 84 6 ∴ The probability of missing his test = × =
5 3 15
09. (D) n (S) = Number of ways of drawing 2 balls out
17. (B) Total possible ways of selecting 4 students out of
of 13 15 students
13 × 12 15 × 14 × 13 × 12
= 13C 2 = = 78 = 15C 4 = = 1365
2 1 ×2×3×4
n (E) = No. of ways of drawing 2 balls out of 5 The no. of ways of selecting 4 students in which no
5×4 student belongs to karnataka = 10C 4 .
= 5 C2 = = 10
2 ∴ Number of ways of selecting atleast one student
n (E) 10 5 from karnataka = 15C4 – 10C 4 = 1155.
∴ P (E) = = =
n (S) 78 39 ∴ Required probability
10. (A) Total no. of balls = (6 + 8) = 14 1155 77 11
= = =
No. of white balls = 8 1365 91 13
8 4 12 × 11 × 10
∴ P (drawing a white ball) = = 18. (B) n (S) = 12C 3 =
14 7 3×2
11. (C) n (S) = 2100 = 2 × 11 × 10 = 220
n (E) = No. of favourable ways No. of selection of 3 oranges out of the total 12
oranges
= 100 C 1 + 100 C 3 + …… + 100C 99
= 12C 3 = 2 × 11 × 10 = 220
= 2100 – 1 = 299
No. of selection of 3 bad oranges out of the total 4
[˙.˙ n C 1 + n C 3 + n C 5 + …… = 2n – 1] bad oranges
n (E) 299 1 = 4 C3 = 4
∴ P (E) = = =
n (S) 2100 2 ∴ n(E) = No. of desired selection of oranges
Note : The given case can be generalised as “If a = 220 – 4 = 216
unbiased coin is tossed ‘n’ times, then the chance
n (E) 216 54
that the head will present itself an odd number of ∴ P (E) = = =
1 n (S) 220 55
times is .”
2 19. (A) In tossing a coin 2 times the sample space is 4
12. (C) ˙·˙ n (S) = (2) 3 = 8 i.e. (H, H), (H, T), (T, H), (T, T)
E = Event of getting 0, or 1 or 2 heads (1) If A 1 denotes exactly one head
= {TTT, TTH, THT, HTT, HHT, then A1 = {(H, T) (T, H)}
HTH, THH} 2 1
So, P (A 1 ) = =
⇒ n (E) = 7 4 2
n (E) 7 (2) If A denotes at least one head
∴ P (E) = =
n (S) 8 then A = {(H, T) (T, H) (H, H)}
13. (D) S = {HHH, HHT, HTH, THH, TTH, 3
∴ P (A) =
THT, HTT, TTT} and 4
Quantitative Aptitude Test | 183
20. (B) In a simultaneous throw of two dice 13 1
So, P (E) = =
Sample Space = 6 × 6 = 36 52 4
Favourable cases are = (2, 6) (3, 5) 4 1
P (F) = =
(4, 4) (5, 3) (6, 2) 52 13
5 1
So, The required probability = and P (E ∩ F) =
36 52
21. (A) A leap year has 366 days = 52 weeks + 2 days. ∴ P (a spade or a king) = P (E ∪ F)
These 2 days can be (Sunday, Monday), (Monday, = P (E) + P (E) –
Tuesday), (Tuesday, Wednesday) …… or (Saturday, P (E ∩ F)
Sunday). Out of these total 7 out comes there are 2
cases favourable to the desired event i.e. (Sunday,
Monday) and (Monday, Tuesday)
= ( 1 1 1
+
4 13 52
– =
4
13 )
29. (C) Total number of ways,
2
∴ Required probability = 11
7 11C
2 =
22. (B) An ordinary year has 365 days i.e. 52 weeks and 9 2
1 day. So the probability that this day is a Sunday is
1 11 × 10
. = = 55
7 2
23. (A) Clearly n (S) = 52 and there are 16 face cards. When none is blue then either 2 should be black or 2
should be red or 1 black and 1 red.
16 4
∴ P (E) = = ∴ No. of favourable ways
52 13
= 5 C 2 + 3 C 2 + 5C 1 × 3 C 1
24. (C) Clearly n (S) = 52. There are 26 red cards
(including 2 kings) and there are 2 more kings. = 10 + 3 + 15 = 28
Let (E) be the event of getting either a red card or a 28
Hence reqd. probability =
king. 55
Then, n (E) = 28 30. (D) Total number of ways
= 11C 2 = 55
n (E) 28 7
∴ P (E) = = = and favourable ways = 5 C 1 × 3C 1
n (S) 52 13
= 5 × 3 = 15
25. (A) n (S) = Number of ways of drawing 2 cards
15 3
out of 52 ∴ Reqd. probability = =
55 11
52 × 51 31. (A) Total number of ways
= 52C 2 = = 1326
2×1
11
n (E) = Number of ways of drawing 2 cards = 11C 3 = = 165
out of 4 8 3
4×3 5
= 4 C2 = =6
2 No. of favourable ways = 5C
3 =
6 1 3 2
∴ P (E) = =
1326 221 5×4
== 10
26. (C) Clearly, n (S) = 52, there are 4 kings and 4 2
queens 10 2
∴ Reqd. probability = =
n (E) 8 2 165 23
∴ P (E) = = = 32. (C) P (M) = m, P (p) = p, P (c) = c
n (S) 52 13
27. (B) There are 13 hearts and 3 more kings Q The probability of at least one success
(13 + 3) 4 = P (M ∪ P ∪ C)
∴ P (heart or a king) = =
52 13 3
= m + p + c – mp – mc – pc + mcp = …(1)
4 9 4
∴ P (neither a heart nor a king) = 1 – = The probability of at least two successes
13 13
= mc –p + mcp
28. (A) Let E and F be the event of getting a spade and – + mcp
– + mcp
that of getting a king respectively. = mc (1 – p) + mp (1 – c) + (1 – m) cp + mcp
Then E ∩ F is the event of getting a king or spade 1
= mc + mp + cp – 2mcp = …(2)
∴ n (E) = 13 2
n (F) = 4 The probability of exactly two success
= mc –p + mcp– + mcp

and n (E ∩ F) = 1

Quantitative Aptitude Test | 184


= mc (1 – p) + mp (1 – c) + cp (1 – m) Hence, reqd. probability
2 3 1 4 2
= mc + mp + cp – 3 mcp = …(3) = + = =
5 10 10 10 5
(2) & (3) gives, 38. (B)
1 2 1 39. (D) Let Probability for odd =p
⇒ mcp = – =
2 5 10 ∴ Probability for even =2p
2 1 1 1 7 ˙·˙ p + 2p =1
∴ mc + mp + cp = + = + =
10 2 5 2 10 ⇒ 3p =1
From (1), 1
7 1 3 ⇒ p =
m+p+c– + = 3
10 10 4 1 2
3 7 1 27 ∴ Probability for odd = , Probability for even = .
⇒ m + p +c = + – = 3 3
4 10 10 20 Sum of two nos. is even means either both are odd or
1 both are even
Thus, pmc = is a true relation.
10 1 1 2 2
∴ Reqd. probability = × + ×
33. (C) Number of possible cases = 100 × 100 for 7m 3 3 3 3
+ 7n to be divisible by 5, one of the term has to end [˙.˙ die is thrown twice]
with 9 and other with 1 (7m cannot be divided by 9) 1 4 5
⇒ m can be 2, 6, 10, 14, ……… 98 (25 values) = + =
9 9 9
and n can be 4, 8, 12 ………… 100 (25 values) 2
40. (C) ˙·˙ Probability for 3 = (1, 2), (2, 1) =
Since m and n can interchange 36
∴ Required probability ⇒ Probability for 5 = (1, 4), (2, 3), (3, 2), (4, 1)
2 × 25 × 25 1 4
= = =
100 × 100 8 36
34. (B) 2
⇒ Probability for 11 (5, 6), (6, 5) =
35. (A) You can have W, W, B or W, B, W or B, W, W 36
I II III I II III I II III 2 4 2
Reqd. probability ∴ Reqd. probability = + +
36 36 36
3 2 3 3 2 1 1 2 1 8 2
= · · + · · + · · = =
4 4 4 4 4 4 4 4 4 36 9
9 3 1 13 –
= + + = 41. (B) ˙·˙ P (A) = 0·2
32 32 32 32
∴ P (A) = 1 – 0·2 = 0·8
36. (B) W.W.W.W.W.W.W –
First we place seven white balls at places marked W. and P (B) = 0·3
If we place three black balls at dot places. Then no ∴ P (B) = 1 – 0·3 = 0·7
two black balls will be placed adjacently. Required probability
Total no. of dot places = 8 – –
= P ( A ∪ B) = P ( A ∩ B )
∴No. of favourable ways for black balls = 1 – P (A ∩ B) = 1 – P (A) P (B)
= 8 C3 = 1 – (0·8) (0·7) = 1 – 0·56 = 0·44
8×7×6 5
= = 56 42. (D) ˙·˙ P (A) =
1×2×3 9
10! – 5 4
Total no. of equally likely cases = ∴ P (A) = 1 – =
7! 3! 9 9
10 × 9 × 8 – 5
= = 15 × 8 = 120 ˙·˙ P (B) =
1×2×3 11
56 7 5 6
∴ Reqd. probability = = ∴ P (B) = 1 – =
120 15 11 11
37. (B) Total balls in the box = 5 Probability that none of them will occur
Second red ball can be drawn in two ways – – – –
= P (A ∩ B) = P (A). P (B)
Case I : First ball is white and second ball is red. 4 5 20
= × =
3 2 6 3 9 11 99
Its probability = · = =
5 4 20 10 Hence,
Case II : First ball is red and second ball is red 20 79
Reqd. probability = 1 – =
2 1 2 1 99 99
Its probability = . = = = 0·798 ~ – 0·8
5 4 20 10
Quantitative Aptitude Test | 185
1 1 volumes of the science book in ascending order and
43. (A) ˙·˙ P(M) = ; P (W) =
4 3 m arrangement of the m volumes in random order
– 1 3 1
⇒ P (M) = 1 – = ∴ Reqd. Prob. = p = .
4 4 m
– 1 2
⇒ P (W) = 1 – = 52. (A) ˙·˙ P (A + B + C)
3 3
– – = P (A) + P (B) + P (C) – P (AB)
3 2 1
Reqd. probability = P (M) P (W) = . = – P (BC) – P (AC) + P (ABC)
4 3 2
⇒ 0·75 = 0·3 + 0·4 + 0·8 – 0·08 – P – 0·28 + 0·09
44. (A) The total number of cases is 2100 . The number of
favourable cases are 100 C 1 + 100 C 3 + …… + 100C 99 = 1·23 – P
⇒ P ≤ 1·23 – 0·75
= 2100 – 1 = 2 99
[˙.˙ P (A + B + C) ≥ 0·75]
299 1
∴ Reqd. probability = 100 = ⇒ P ≤ 0·48 …(1)
2 2
Again P (A + B + C) ≤ 1
45. (D) ˙.˙ Probability in each trial (shooting)
⇒ P = 1·23 – P (A + B + C)
= 0·3
∴ P ≥ 1·23 – 1 ≥ 0·23
∴ Reqd. probability = (0·3)10 Thus, 0·23 ≤ P ≤ 0·48.
46. (A) 53. (D) ˙·˙ Probability that no man out of n men aged x
1 years will die in a year = (1 – p)n .
47. (A) ˙·˙ p =
5 ∴ Probability that out of n men at least one will die
1 4 in a year = 1 – (1 – p)n .
∴ q = 1– =
5 5 When at least one man dies, any one out of the n men
The probability that none will hit in 10 shots may be the first to die
= (4/5)10 ∴ Reqd. Probability that a particular man Mk will die
∴ Reqd. probability = 1 – () 4 10
5
and be the first to die
1
= n [1 – (1 – p)n]
48. (C) ˙·˙ x2 – 13x – 30 ≤ 0
⇒ (x + 2) (x – 15) ≤ 0 54. (A) ˙·˙ Probability of machine failing during a day,
⇒ – 2 ≤ x ≤ 15 i.e. p = 0·95
But x is a natural number. ∴ q = probability of its working during a day
∴ 1 ≤ x ≤ 15 = 1 – p = 1 – 0·95 = 0·05
Required probability
15 3
∴ Reqd. Probability P = = = q4 = (0·05)4
100 20
49. (B) Corresponding to n tosses, the Probability of = 0·00000625
55. (B) n (S) = 10C 7 = 10C 3
getting no head = () 1 n
2
and, therefore, the Probabi- n (E) = 8 C 3
lity of getting at least one head n (E) 7
∴ P (E) = =
= 1– () 1 n
2
n (S) 15
56. (B) Probability of multiple of 2 = =
3 1
6 2
() 1 n 99
Now, 1– ≥
2 100 2 1
Probability of multiple of 3 = =
6 3
⇒ () 1 n
2

1
100
⇒n≥7 Since there are two dice.
∴ The required probability
[( ) 1 7
2 ]
= 7·8 × 10–3
= 2× × =
1 1 1
2 3 3
50. (A) From the set of odd numbers < 100, if we – – – –
exclude multiples of 5. We get the set of numbers 57. (A) ˙·˙ B ∩ C = (A ∩ B ∩ C) ∪ (A ∩ B ∩ C)
< 100 and relatively prime to 100. The number of …(1)
such numbers = 50 – 10 = 40 – – –
40 2 and (A ∩ Β ∩ C) ∩ (A ∩ Β ∩ C) = φ
∴ Reqd. Probability = p = = – – – –
100 5 ⇒ P (B ∩ C) = P (A ∩ B ∩ C) + P (A ∩ B ∩ C)
51. (C) Corresponding to each arrangement of (n – m) 1 1 2
other books, there is a unique arrangement of the m = + =
3 3 3
Quantitative Aptitude Test | 186
– 63. (C) Total number of cases is 17.
Also, B = (B ∩ C) ∪ (B ∩ C) …(2)
– ˙·˙ Number divisible by 3 are 3, 6, 9, 12, 15
and (B ∩ C) ∩ (B ∩ C) = φ
3 – (These are 5 in number)
⇒ P (B) = P (B ∩ C) + P (B ∩ C)
4 Number divisible by 7 are 7, 14.
2 (These are 2 in number)
⇒ = P (B ∩ C) +
3 There are two favourable number of cases
3 2 1
∴ P (B ∩ C) = – = Total no. of favourable number
4 3 12
58. (D) Total no. of cases = 6 P 2 = 6 × 5 = 30 = 5+2=7
Non-favourable cases are Required probability = 7/17.
(4, 5), (5, 4), (4, 6), (6, 4), (5, 6), (6, 5) 1
64. (A) Probability of getting head in one trial =
∴ Probability that event will not happen 2
6 1 ∴ Reqd. Probability of getting heads in both the
= =
6×5 5 1 1 1
1 4 trials = × =
∴ Reqd. probability = 1 – = 2 2 4
5 5 65. (A) ˙·˙ Total no. of favourable cases i.e., (5, 10, 15,
59. (A) If the product of the four numbers ends in one of 20, 25, 30, ……, 105, 110, 115, 120) = 24.
the digits 1, 3, 7 or 9, each number should have the
last digit as one of these 4 digits. 24 1
∴ Reqd. Prob. = =
∴ The number of favourable cases = 44 120 5
Total number of all possible cases = 104 66. (D) No. of white balls = 7
Hence, the required probability Red balls = 9
44 24 16 Total no. of balls = 7 + 9 = 16
= 4= 4 =
10 5 625 7
Probability of drawing a white ball =
60. (B) If the last digit in the product is to 2, 4, 6, 8 the 16
last digit in all the n numbers should not be 0 and 5 67. (C) Total ways = 100
and the last digit of all numbers should not be
Squares of following no’s lie between 1 and 100, 12,
selected exclusively from the set of number {1, 3, 7,
22 , 32 , 42 , 52 , 62 , 72 , 82 , 92 , 102
9}
∴ Favourable number of cases (which are 10 in numbers.)
= 8n – 4n 10 1
But generally the last digit can be any one of 0, 1, 2, So, Required probability = =
100 10
3, …… 9.
68. (A) Total ways 52C3 = 22100
Hence, the total number of ways = 10 n
Hence, the required probability There are 4 suits in a pack of cards so three suit can
8n – 4n 4n – 2n be selected in 4 C 3 ways and one card each from
= = different unit can be selected as = 13C 1 × 13C 1 × 13C 1
10n 5n
61. (D) The probability that head is shown in one coin is ways
1 So, favourable ways = 4 C 3 × 13C1 × 13C1 × 13C1
. The probability that the sum of the numbers on the
2
= 8788
dice is a prime = the proability that the following pair
of numbers on the dice is a prime = the probability 8788 169
∴ Required Probability = =
that following pair of number are getting on the dice, 22100 425
namely (1, 1), (1, 2), (2, 1), (1, 4), (4, 1), (2, 3), 69. (B) Total ways = 52
(3, 2), (1, 6), (6, 1), (2, 5), (5, 2), (3, 4), (4, 3), (6, 5),
15 There is one queen of club and one king of heart
(5, 6) = . favourable ways = 1 + 1 = 2.
36
1 1 15 5 2 1
∴ The required probability = . . = . ∴ Required probability = =
2 2 36 48 52 26
62. (A) The favourable cases are (1, 3), (2, 4), (3, 5),
70. (B) Total ways = 52
(4, 6) and (1, 4), (2, 5), (3, 6) and their reversed cases
like (3, 1), (4, 2), (5, 3) ………. There are 13 cards of diamond, 4 cards of king, but
Total number of favourable cases one card is king of diamond which is counted both in
= 2×7 diamond and king cards
∴ Required Probability ⇒ Favourable ways = 13 + 4 – 1 = 16
14 7 16 4
P (E) = = ∴ Required probability = =
36 18 52 13
Quantitative Aptitude Test | 187
Partnership
Important Points/Facts share of A in the profit of Q. 2. The partners A, B, C
When two or more than two Rs. 55,000 ? start a business. Twice the
persons run a business jointly. They Solution : Ratio of two Capital investment of A is equal to thrice
are called partners and the deal is of A and B the capital of B and the capital of B
known as partnership. It is of two is four times the capital of C. Find
= 50,000 : 60,000
kinds : (i) Simple and (ii) Compound. the share of each out of a profit of
= 5:6 Rs. 2,97,000.
1. Simple Partnership : If the ˙.˙ A gets 10% of the total profit Solution : Let C’s capital = Rs. x
capitals of the partners are invested for looking after the business. then B’s capital = Rs. 4x
for the same period the partnership is ∴ Income received by A for Now, 2 (A’s capital) = 3 (B’s
called simple. looking after the business capital) = 3 × 4x
Rule I. When investment of all = 10% Rs. 55,000 3 × 4x
the partners are for the same time, the
profit or loss is divided among them = Rs. 5500 ∴ A’s capital = ( ) 2
= 6x
in the ratio of their investments. Rest of the profit So, ratio of share of A, B and C
2. Compound Partnership : If = 55,000 – 5500 = 6x : 4x : x = 6 : 4 : 1.
the capitals of the partners are inves-
ted for different lengths of time, the
partnership is called compound.
= Rs. 49500
Now, from this amounts of
(
∴ A’s share = Rs. 297000 ×
6
11)
profit. A and B will get their shares = Rs. 1,62,000
Rule II. When investments are of profit in the ratio of their invested
for different times then equivalent
capitals are calculated for a unit of
capitals. (
B’s share = Rs. 297000 × )
4
11
time by multiplying the capital with Profit received by = Rs. 108000
the number of units of time. The 5 and C’s share
profit or loss is now divided in the A = 49,500 ×
11
ratio of these capitals.
For example : If A and B inves-
= Rs. 22,500 (
= Rs. 297000 ×
1
11)
ted their capitals for different length ∴ Total share of = Rs. 27000.
of time, then A = 22500 + 5500 Q. 3. A, B, C hire a meadow
for Rs. 2934·60. A puts in 10 oxen
Capital of A × A’s time = Rs. 28,000
for 20 days. B 30 oxen for 8 days C
of investment Again share of 16 oxen for 9 days. Find the rent
Capital of B × 6 paid by each.
B’s time of investment B = 49500 ×
11 Solution :
Profit of A Ratio of rent to be paid by A, B
= = 27000
Profit of B and C
∴ Hence share of B is Rs. 1,000
= [28,000 – 27,000] less than the = (10 × 20 : 30 × 8 : 16 × 9)
Note I : A partner who manages
share of A. = 25 : 30 : 18
the business is called a working part-
ner and the one who simply invests Short Method : ∴ Rent to be paid by
the money is called a sleeping part-
ner.
˙.˙ Since A gets 10% of the
total profit for looking after the (
A = Rs. 2934·60 × )
25
73
II. In every examination the business therefore, B will get the = Rs. 1005
questions are generally set on com- share of profit from 90% of the Rent to be paid by
total profit.
pound partnership. But such quest-
ions are first converted into simple
partnership and then solved.
∴ Share of profit B gets
6 90
(
B = Rs. 2934·60 × )
30
73
= 55000 × × = Rs. 1206
11 100
Examples = Rs. 27,000 Rent to be paid by
Q. 1. A and B started a ∴ Share of income A gets C = Rs. [2934·60 – (1005
business in partnership with + 1206)
= 55,000 – 27,000
Rs. 50,000 and Rs. 60,000 respecti- = Rs. 723·60
= Rs. 28,000
vely. A is also a working partner Q. 4. A began a business with
∴ Required difference Rs. 2100 and is joined afterwards
and gets 10% of the total profit for
looking after the business. How = Rs. (28,000 – 27,000) by B with Rs. 3600. After how
much is the share of B less than the = 1000 Ans. many months did B join, if the

Quantitative Aptitude Test | 188


profits at the end of the year are 1 1 1 months. If the annual profit be
proportion of : : . A with-
divided equally ? 3 4 5 Rs. 18,000, C’s share is—
Solution : Suppose B joines draws half his capital at the end (A) Rs. 7500 (B) Rs. 7200
after x months. Then B’s money of 4 months. Out of a total (C) Rs. 6000 (D) Rs. 5750
remained invested for (12 – x) annual profit of Rs. 847 A’s
share is— 08. A, B and C invest Rs. 2,000, Rs.
months. 3,000 and Rs. 4,000 in a
(A) Rs. 252 (B) Rs. 280
˙·˙ 2100 × 12 = 3600 × (12 – x) business. After one year A
(C) Rs. 315 (D) Rs. 412 removed his money but B and C
⇒ 3600 x = 43200 – 25200
03. A and B start a business with continued for one more year. If
18000 the net profit after 2 years be
∴ x = =5 initial investments in the ratio 12
3600 : 11 and their annual profits were Rs. 3,200 then A’s share in the
So, B, joined after 5 months. in the ratio 4 : 1. If A invested profit is—
Q. 5. Dilip and Manohar the money for 11 months. B (A) Rs. 1000 (B) Rs. 600
started a business by investing invested the money for— (C) Rs. 800 (D Rs. 400
Rs. 1,00,000 and Rs. 1,50,000 res- 2
(A) 3 months (B) 3 months 09. A and B enter into partner-
pectively. Find the share of each 3 ship investing Rs. 12,000 and
out of a profit of Rs. 24,000. (C) 4 months (D) 6 months Rs. 16,000 respectively. After 8
Solution : Ratio of shares of months C joins them with a
04. A and B started a joint firm. A’s
Dilip and Manohar capital of Rs. 15,000. The share
investment was thrice the invest-
= 100000 : 150000 = 2 : 3 ment of B and the period of his of C in a profit of Rs. 45600
∴ Dilip’s share investment was two times the after 2 years will be—
period of investment of B. If B (A) Rs. 21200
(
= Rs. 24000 ×
2
5) = Rs. 9600 got Rs. 4,000 as profit then their
total profit is—
(B) Rs. 19200
(C) Rs. 14400
Manohar’s share (A) Rs. 24,000 (D) Rs. 12000
(
= Rs. 24000 ×
3
5) = Rs. 14400 (B) Rs. 16,000
(C) Rs. 28,000
10. Dilip, Ram and Amar started a
shop by investing Rs. 27,000,
Q. 6. Sanjay and Raju started (D) Rs. 20,000 Rs. 81,000 and Rs. 72,000
a business and invested Rs. 20,000
05. A, B and C enter into partnership respectively. At the end of one
and Rs. 25,000 respectively. After 4
by making investments in the year the profit was distributed. If
months, Raju left and Naresh
ratio 3 : 5 : 7. After a year, C Ram’s share of profit be
joined by investing Rs. 15,000. At
invests another Rs. 3,37,600 Rs. 36,000, then the total profit
the end of the year there was a
while A withdrew Rs. 45,600. was—
profit of Rs. 4,600. What is the
share of Naresh ? The ratio of investments then (A) Rs. 108000
changes to 24 : 59 : 167. How (B) Rs. 16000
Solution : Ratio of shares of much did A invest initially ?
Sanjay, Raju and Naresh (C) Rs. 80000
(A) Rs. 45,600 (D) None of these
= 20,000 × 12 : 25,000 × (B) Rs. 96,000
4 : 15000 × 8 11. Manoj got Rs 6000 as his share
(C) Rs. 1,41,600 out of a total profit of Rs. 9,000
= 12 : 5 : 6 (D) None of these which he and Ramesh earned at
∴ Share of Naresh the end of one year. If Manoj
06. A and B enter into partnership.
(
= Rs. 4600 ×
6
23) = Rs. 1200 A invests Rs. 16,000 for 8
months and B remains in the
invested Rs. 20,000 for 6 months.
Whereas Ramesh invested his
business for 4 months. Out of a amount for the whole year, what
Exercise 2 was the amount invested by
total profit, B claims of the Ramesh ?
7
1. In a partnership A invests (1/6) profit. B contributed— (A) Rs. 30000
of the capital for (1/6) of the
(A) Rs. 11,900 (B) Rs. 40000
time, B invests (1/3) of the
capital for (1/3) of the time and (B) Rs. 10,500 (C) Rs. 10000
C, the rest of the capital for the (C) Rs. 13,600 (D) Rs. 5000
whole time. Out of a profit of (D) Rs. 12,800 12. Rs. 700 is divided among A, B
Rs. 4,600 B’s share is— and C so that A receives half as
(A) Rs. 800 (B) Rs. 1000 07. A, B and C enter into partner- much as B and B half as much as
ship. A invests some money at
(C) Rs. 650 (D) Rs. 960 C. Then C’s share is—
the beginning. B invests double
02. A, B, C enter into a partnership the amount after 6 months and C (A) Rs. 200 (B) Rs. 300
and their capitals are in the invests thrice the amount after 8 (C) Rs. 400 (D) Rs. 600

Quantitative Aptitude Test | 189


13. A and B entered into a partner- 19. If 6 (A’s capital) = 8 (B’s 26. Four milkmen rented a pasture.
ship investing Rs. 16000 and Rs. capital) = 10 (C’s capital) then A grazed 18 cows for 4 months,
12000 respectively. After 3 the ratio of their capitals is— B, 25 cows for 2 months. C, 28
months. A withdrew Rs. 5000 (A) 3 : 4 : 5 cows for 5 months and D, 21
while B invested Rs. 5000 more. cows for 3 months. If A’s share
(B) 12 : 15 : 20
After 3 more months. C joins the of rent is Rs. 360, the total rent
business with a capital of (C) 20 : 15 : 12 of the field is—
Rs. 21000. The share of B (D) 6 : 8 : 10 (A) Rs. 1500 (B) Rs. 1600
exceeds that of C, out of a total (C) Rs. 1625 (D) Rs. 1650
profit of Rs. 26400 after one 20. A’s capital is equal to twice B’s
year, by— capital and B’s capital is three 27. Jagmohan, Rooplal and Pandeyji
times C’s capital. The ratio of rented a video cassette for one
(A) Rs. 1200 (B) Rs. 2400 the capital is— week at a rent of Rs. 350. If they
(C) Rs. 3600 (D) Rs. 4800
(A) 2 : 1 : 3 (B) 1 : 2 : 6 use it for 6 hours. 10 hours and
14. Jayant started a business, invest- (C) 6 : 3 : 1 (D) 1 : 3 : 6 12 hours respectively. The rent
ing Rs. 6000. Six months later to be paid by Pandeyji is—
Madhu joined him, investing 21. Three partners A, B and C invest
(A) Rs. 75 (B) Rs. 125
Rs. 4000. If they made a profit Rs. 26000, Rs. 34000 and
of Rs. 5200 at the end of the Rs. 10000 respectively in a busi- (C) Rs. 135 (D) Rs. 150
year. How much must be the ness. Out of a profit of Rs. 28. A, B and C start a business. A
share of Madhu ? 3500. B’s share is— invests 3 times as much as B
(A) Rs. 2080 (B) Rs. 1300 (A) Rs. 1300 (B) Rs. 1700 invests and B invests two-third
(C) Rs. 1800 (D) Rs. 2600 (C) Rs. 500 (D) Rs. 1500 of what C invests. Then, the ratio
of capitals of A, B and C is—
15. A, B and C subscribe Rs. 47,000 22. A and B invest in a business in
for a business. A subscribes the ratio 3 : 2 if 5% of the total (A) 3 : 9 : 2 (B) 6 : 10 : 15
Rs. 7000 more than B and B profit goes to charity and A’s (C) 5 : 3 : 2 (D) 6 : 2 : 3
Rs. 5,000 more than C. Out of a share is Rs. 855, total profit is—
29. A sum of money is to be divided
total profit of Rs. 9,400, B recei- (A) Rs. 1576 (B) Rs. 1537.50 among A, B and C in the ratio 2 :
ves— (C) Rs. 1500 (D) Rs. 1425 3 : 7. If the total share of A and
(A) Rs. 4400 (B) Rs. 3000 B together is Rs. 1500 less than
(C) Rs. 2000 (D) Rs. 1737.90 23. A, B and C contract a work for C. What is A’s share in it ?
Rs. 550. Together A and B are to
16. Karim invests Rs. 30000 for one 7 (A) Rs. 1000
year in a shop. How much his do of the work. The share of (B) Rs. 1500
11
partner Raunaq should invest in C should be— (C) Rs. 2000
order that the profit after one
year must be in the ratio 2 : 3 ? 1 (D) Data inadequate
(A) Rs. 183 (B) Rs. 200
(A) Rs. 20000 3 30. A and B invest Rs. 3000 and
(B) Rs. 40000 (C) Rs. 300 (D) Rs. 400 Rs. 4000 in a business. A
(C) Rs. 45000 receives Rs. 10 per months out
24. A, B and C enter into a
of the profit as a remuneration
(D) Rs. 18000 partnership with shares in the
for running the business and the
7 4 6
17. Kanti started a business invest- ratio : : · After 4 months, A rest of profit is divided in pro-
2 3 5
ing Rs. 9000. Five months later portion to the investments. In a
increases his share by 50%. If
Sudhakar joined him by invest- year A totally receive Rs. 390.
the total profit at the end of one
ing Rs. 8000. If they make a What does B receive ?
year be Rs. 21600, then B’s share
profit of Rs. 6970 at the end of (A) Rs. 630 (B) Rs. 360
in the profit is—
year. Sudhakar’s share of profit
(A) Rs. 2100 (B) Rs. 2400 (C) Rs. 480 (D) Rs. 380
is—
(A) Rs. 3690 (B) Rs. 1883.78 (C) Rs. 3600 (D) Rs. 4000 31. Three Hikers A, B and C start on
(C) Rs. 2380 (D) Rs. 3864 a trip with Rs. 50 each and agree
25. A is a working and B is a sleep- to share the expenses equally. If
18. A, B and C are three partners in ing partner in a business. A puts at the end of the trip. A has
a business. If twice the invest- in Rs. 12,000 and B Rs. 20,000. Rs. 20 left with him. B Rs. 30
ment of A is equal to thrice the A receives 10% of the profits for and C Rs. 40 how must they
capital of B and the capital of B managing, the rest being divided settle their accounts ?
is four times the capital of C. in proportion to their capitals.
Out of a total profit of Rs. 9600 (A) C must pay Rs. 10 to A
Out of a total profit of Rs. 5,940
the share of C is— the money received by A is— (B) A must pay Rs. 10 to B
(A) Rs. 700 (B) Rs. 900 (A) Rs. 3240 (B) Rs. 4200 (C) A must pay Rs. 10 to C
(C) Rs. 740 (D) Rs. 540 (C) Rs. 3600 (D) Rs. 4500 (D) Can’t be settled

Quantitative Aptitude Test | 190


32. A, B and C enter into partnership (C) All 1, 2 and 3 together are Rs. 180000. Total profit, in what
by making investments in the required ratio should be divided between
ratio 3 : 5 : 7. After a year C (D) Even with all 1, 2 and 3 the them ?
invests another Rs. 337600 while answer cannot be arrived at (A) 8 : 7
A withdraws Rs. 45600. The ratio (B) 6 : 5
of investments then changes to 36. Pramod started a business with
Rs. 40000. After 4 months Vikas (C) 7 : 5
14 : 29 : 167. How much did A
also joined him with Rs. 60000. (D) None of these
invest initially ?
If at the end of the year, the 41. Subodh started a business, inves-
(A) Rs. 233364·71 profit is Rs. 16000, what is the ting Rs. 45000. After 4 months.
(B) Rs. 9600 share of Vikas in the profit ? Nepal joined him investing Rs.
(C) Rs. 141600 (A) Rs. 8000 30000. If the total profit earned
(D) 233374·72 (B) Rs. 4000 by them at the end of the year
33. A, B and C contract a work for (C) Rs. 12000 was Rs. 13,000 what would be
Rs. 1100. Together A and B are the difference between the shares
(D) Rs. 10000
7 of Subodh and Nepal ?
to do of the work. The share 37. Alok started a business with (A) Rs. 7000
11 Rs. 75000. After 3 months
of C should be— (B) Rs. 3000
Chandan also joined him with
(A) Rs. 400 (B) Rs. 300 Rs. 60000. If at the end of the (C) Rs. 9000
1 year, the profit is Rs. 16000. (D) None of these
(C) Rs. 200 (D) Rs. 183
3 What is the share of Chandan in 42. Four persons decided to start a
the profit ? restaurant by sharing equal
34. A, B and C can do a work in 20,
25 and 30 days respectively. (A) Rs. 6000 (B) Rs. 10000 amount of the total investment.
They undertook to finish the (C) Rs. 8000 (D) Rs. 4500 After some calculations they
work together for Rs. 2220, then 38. Gopal started a business with found that if they get two more
the share of A exceeds that of B Rs. 3000. After 6 months Dinesh equal investors, each of the four
by— also joined him with Rs. 2000. If will have to pay Rs. 12000 less.
(A) Rs. 120 (B) Rs. 180 at the end of the year the profit is What was the total investment ?
Rs. 2600. What is the share of (A) Rs. 24000
(C) Rs. 300 (D) Rs. 600
Dinesh in the profit ? (B) Rs. 72000
35. What is the difference in the (C) Rs. 144000
(A) Rs. 1733 (B) Rs. 1950
share of profit between P and Q
(C) Rs. 866 (D) Rs. 650 (D) Rs. 288000
in a joint business at the end of
the year ? 39. A started a business by investing 43. Sanjay and Komal started a
(1) P invested Rs. 80000 and Rs. 8000. 3 months later B joined business with Rs. 15000 each.
withdrew Rs. 20000 after him with Rs. 5000. If at the end After 8 months Komal withdrew
6 months. of a year total profit earned was Rs. 10000. If at the end of a year
(2) Q joined four months after Rs. 2350. How much is the share there was a profit of Rs. 32000.
the business was started. of A more than the share of B ? What was the share of Sanjay in
(A) Rs. 600 (B) Rs. 800 the profit ?
(3) Q’s amount was 80 per cent
of P’s amount during the (C) Rs. 400 (D) Rs. 850 (A) Rs. 18000
last 6 months. 40. Rakesh started a business by (B) Rs. 18500
(A) Only 1 and 2 investing Rs. 70000. 8 months (C) Rs. 16500
(B) Only 2 and 3 later Vinod also joined him with (D) Rs. 16000

Answers with Hints


x y 1 1 1
01. (A) Suppose A invests Rs. for months. B invests 02. (B) Ratio of capitals in the beginning = : :
6 6 3 4 5
x y
[ ( )]
Rs. for months and C invests Rs. x –
3 3
x x
+
6 3
= 20 : 15 : 12.
Ratio of investments for the whole year
for y months.
= (20 × 4 + 10 × 8) : (15 × 12) :
Ratio of their investments =( )( )
x y
×
6 6
:
x y
×
3 3
: (12 × 12)

( ) x
2
×y
= 40 : 45 : 36

=
1 1 1
: : = 1 : 4 : 18
∴ (
A’s share = Rs. 847 ×
40
121 ) = Rs. 280
36 9 2
03. (A) Suppose B invested the money for x months.
(
∴ B’s share = Rs. 4600 ×
4
)
23
= Rs. 800 Then the ratio of investments = (12 × 11 : 11x)

Quantitative Aptitude Test | 191


= 12 : x 12. (C) Let C’s share = Rs. x
12 4 x
∴ x = 1 ⇒ x = 3 months. Then B’s share = Rs.
2
04. (C) Suppose B invested Rs. x for y months. x
and A’s share = Rs.
Then A’s investment is Rs. 3x for 2y months. 4
Ratio of investments of A and B = 6xy : xy = 6 : 1. x x
∴ A:B:C = : :x=1:2:4
4 2
Now B’s share = Rs. 4,000

Hence,
A’s share = Rs. 24,000
Total profit = Rs. 28,000
Hence C’s share = Rs. 700 ×( )
4
7
= Rs. 400

13. (C) A : B : C = Rs. (16000 × 3 + 11000 × 9)


05. (C) Let initial investments be 3x, 5x and 7x rupees.
: (12000 × 3 + 17000 × 9) : (21000 × 6)
˙·˙ (3x – 45600) : 5x : (7x + 337600) = 24 : 59 : 167
= 147 : 189 : 126
3x – 45600 24
⇒ = = 7:9:6
5x 59
∴ x = 47200 ∴ (B’s share) – (C’s share) = Rs. [( 26400 ×
9
22)
∴ Initial investment of A = Rs. (47200 × 3)
= Rs. 141600 (– 26400 ×
6
22)]
5 2 = Rs. (10800 – 7200) = Rs. 3600
06. (D) Ratio of profits of A and B = : = 5 : 2
7 7
14. (B) Ratio of their shares = 6000 × 12 : 4000 × 6 = 3 : 1
16000 × 8 5
˙·˙


x×4
=
2
20x = 256000
∴ (
Madhu’s share = Rs. 5200 × )1
4
= Rs. 1300
∴ x = 12800
15. (B) Suppose C invests Rs. x
So, B contributed Rs. 12800.
07. (C) Suppose A invested Rs. x. Then B’s investment = Rs. (x + 5000)
Then A : B : C = 12x : 6 × (2x) : 4 × (3x) and A’s investment = Rs. (x + 12000)
= 1:1:1 ˙·˙ x + x + 5000 + x + 12000
= 47000
∴ (
C’s share = Rs. 18000 × )
1
3 ⇒ x = 10000
= Rs. 6000 Thus A : B : C = 22000 : 15000 : 10000
08. (D) A : B : C = 2000 × 12 : 3000 × 24 : 4000 × 24 = 22 : 15 : 10
= 1:3:4 ∴ (
B’s share = Rs. 9400 × )
15
47
(
A’s share = Rs. 3200 ×
1
8) = Rs. 400 = Rs. 3000
09. (D) Ratio of shares = 12000 × 24 : 16000 × 24 : 30000 2
16. (C) ˙·˙ =
15000 × 16 = 6 : 8 : 5 x 3
⇒ 2x = 90000
∴ (
C’s share = Rs. 45600 ×
5
)
19 ∴ x = 45000
= Rs. 12000 17. (C) ˙·˙ Ratio of shares = 9000 × 12 : 8000 × 7
10. (C) Ratio of shares = 27000 : 81000 : 72000 = 108 : 56 = 27 : 14
= 3:9:8
If Ram’s share is Rs. 9 then total profit ∴ (
Sudhakar’s share = Rs. 6970 × )
14
41
= Rs. 20 = Rs. 2380
If Ram’s share is Rs. 36000 then total profit
18. (D) Let C’s capital = Rs. x. Then B’s capital = Rs. 4x
= Rs. ( 20
9
× 36000) ˙·˙ 2 (A’s capital) = 3 (B’s capital) = 12x
= Rs. 80000 ⇒ A’s capital = 6x
11. (D) Let the amount invested by Ramesh = Rs. x ⇒ A : B : C = 6x : 4x : x
Then 20000 × 6 : 12x = 6000 : 3000 = 6:4:1


120000
12x
=
2
1
∴ (
C’s share = Rs. 5940 ×
1
)
11
∴ x = 5000 = Rs. 540

Quantitative Aptitude Test | 192


19. (C) Let 6 (A’s capital) = 8 (B’s capital) Let the total rent = Rs. x

Then A’s capital =


= 10 (C’s capital) = x
x
B’s capital =
x Then A’s share = Rs. (x × 325
72
) = Rs. (72x
325)
6 8 72x
x ˙·˙ = 360
and C’s capital = 325
10 325 × 360
∴ Ratio of capitals of A, B and C = : :
x x x ∴ x = (
72 )
= Rs. 1625
6 8 10 27. (D) Ratio of rent = 6 : 10 : 12 = 3 : 5 : 6
= 20 : 15 : 12.
20. (C) Let C’s capital = Rs. x ∴ Pandeyji has to pay = Rs. 350 × ( 6
14 )
Then B’s capital = Rs. 3x and A’s capital = Rs. 6x. = Rs. 150
∴ Ratio of capitals of A, B and C = 6x : 3x : x
= 6:3:1 28. (D) Suppose C invests Rs. x then, B invests Rs. (2x3 )
21. (B) Ratio of shares of A, B, C = 26000 : 34000 : and A invests Rs. (2x).
10000 = 13 : 17 : 5 2x
∴ Ratio of investments of A, B, C = 2x : :x
∴ (
B’s share = Rs. 3500 ×
17
35 ) 3
= 6:2:3
= Rs. 1700 29. (B) ˙·˙ 7x – (2x + 3x) = Rs. 1500
22. (C) Let the total profit be Rs. 100.
⇒ x = Rs. 750
After paying to charity,
∴ Share of A = 2x = Rs. 1500
A’s share = Rs. 95 ×( 3
5 ) = Rs. 57 30. (B) Total Profit – Remuneration = Balance profit
If A’s share is Rs. 57, then total profit = Rs. 100 This balance profit is divided in proportion to their
investments
If A’s share is Rs. 855, then total profit
Balance Profit of A Investment of A
(
= Rs. 855 ×
100
51 ) ˙·˙
Balance Profit of B
390 – 10 × 12
=
Investment of B
3000 3
= Rs. 1500 ⇒ = =
Balance of Profit of B 4000 4
23. (B) (
C’s share = Rs. 550 ×
4
11 )= Rs. 200 (Since remuneration of A is
Rs. 10 per month)
7 4 6
24. (D) Given Ratio = : : = 105 : 40 : 36 270
2 3 5 ⇒ Balance profit of B = 4 × = Rs. 360
3
Let them initially invest Rs. 105, Rs. 40 and Rs. 36
respectively. Since B does not get any remuneration, hence B
receives Rs. 360 at the end of the year.
Ratio of investments
= [105 × 4 + (150% of 105) × 8] : 31. (A)
(40 × 12) : (36 × 12) 32. (A) Let the initial investments of A, B, C be Rs. 3x,
= 1680 : 480 : 432 = 35 : 10 : 9 Rs. 5x and Rs. 7x respectively. Then
(3x – 45600) : 5x : (7x + 337600) = 14 : 59 : 167
(
∴ B’s share = Rs. 21600 ×
10
54 ) ⇒
3x – 45600
=
14
= Rs. 4000 5x 29
25. (B) For management A receives ⇒ x = 77788·24
= Rs. 960
∴ A invested initially Rs. (77788·24 × 3)
Balance = Rs. (9600 – 960)
= Rs. 233364·71
= Rs. 86400
33. (A)
Ratio of their investments = 12000 : 20000 = 3 : 5
34. (B) Ratio of shares = Ratio of 1 day’s work
∴ (
A’s share = Rs. 8640 ×
3
8 ) =
1 1 1
:
20 25 30
:
= Rs. 3240 = 15 : 12 : 10
So A receives = Rs. (3240 + 960)
= Rs. 4200 ∴ (
A’s share = Rs. 2220 ×
15
37 ) = Rs. 900
26. (C) Ratio of rent = (18 × 4 : 25 × 2 : 28 ×
5 : 21 × 3) (
B’s share = Rs. 2220 ×
12
37 ) = Rs. 720
= 72 : 50 : 140 : 63 ∴ A’s share exceeds B’s share = Rs. 180

Quantitative Aptitude Test | 193


35. (D) The profit earned at the end of the year is not 40. (D) Ratio between the investment of Rakesh and
given, hence all the three statements together are also Vinod
not sufficient to answer the question. = 70000 × 12 : 180000 × 4
36. (A) Ratio between the investment of Pramod and = 840000 : 720000
Vikas = 7:6
= 40000 × 12 : 60000 × 8 Hence profit sharing ratio = 7 : 6.
= 480000 : 480000 41. (D) Ratio between the investment of Subodh and
= 1:1 Nepal
Total profit = Rs. 16000 = 45000 × 12 : 30000 × 8
1 = 540000 : 240000
Share of Vikas in profit = × 16000
2 = 9:4
= Rs. 8,000. Total profit = Rs. 13000
37. (A) Ratio between the investment of Alok and 9
Chandan ∴ Subodh’s share = × 13000
(9 + 4)
= 75000 × 12 : 60000 × 9 = Rs. 9000
= 900000 : 540000 ∴ Nepal’s share = 13000 – 9000
= 90 : 54 = 5 : 3 = Rs. 4000
Total profit = Rs. 16000 ∴ Required difference = 9000 – 4000
3 = Rs. 5000
∴ Chandan’s share = × 16000
(5 + 3) 42. (C) Let investment of each person be Rs. x
= Rs. 6000 Total investment of 4 people = 4 × x = Rs. 4x
38. (D) Ratio between the investment of Gopal and Investment of each person when two more people
Dinesh join them = Rs. (x – 12000)
= 3000 × 12 : 2000 × 6 ∴ Total investment of 6 people = 6 (x – 12000)
= 36000 : 12000 ˙·˙ 4x = 6 (x – 12000)
= 3:1 ⇒ 4x = 6x – 72000
1 ⇒ 2x = 72000
Dinesh’s share = × 2600
(3 + 1) ∴ x = 36000
= Rs. 650 Hence total investment = 4 × 36000
39. (D) Ratio between the investment of A and B = Rs. 144000
= 8000 × 12 : 5000 × 9 43. (A) Sanjay’s investment for 1 month
= 96000 : 45000 = 15000 × 12
= 32 : 15 = Rs. 180000
Total profit = Rs. 2350 Komal’s investment for 1 month
32 = 15000 × 8 + (15000 – 10000) × 4
˙.˙ A’s share = × 2350 = 120000 + 20000
(32 + 15)
= Rs. 1600 = Rs. 140000
∴ Ratio between the investment of Sanjay and Komal
∴ B’s share = 2350 – 1600 = 180000 : 140000
= Rs. 750 = 9:7
∴ Required difference 9
∴ Sanjay’s profit = × 32000
= 1600 – 750 (9 + 7)
= Rs. 850 = Rs. 18000

Quantitative Aptitude Test | 194


Volume and Surface Area
Formulae 5. Cone—Let r be the radius of the
l
1. Cuboid—Let, length = l, base, h the height and l the slant
height of a cone, then— S
breadth = b and height = h units. h
(i) Volume of Cuboid (i) Slant height l = ⎯
√⎯⎯⎯⎯
h2 + r2
= (l × b × h) cubic units. (ii) Volume of the cone
R
(ii) Whole surface of cuboid
= 2 (lb + bh + lh) sq. units.
= ( 1 2
3 )
πr h cu. units.
Frustum of a Cone
(iii) Curved surface area of the
(iii) Diagonal of cuboid cone
= √⎯⎯⎯⎯⎯⎯⎯⎯
l2 + b2 + h2 units. = (πrl) sq. units Examples
2. Cube—Let each edge (or side) (
= πr ⎯ √⎯⎯⎯⎯ )
r2 + h2 sq. units. Q. 1. The surface area of a cube
of a cube be a units, then— (iv) Total surface area of the is 384 cm2. Find its volume.
(i) Volume of the cube cone = (πrl + πr2 ) sq. units. Solution : 6a2 = 384
= a3 cubic units. ⇒ a2 = 64
(ii) Whole surface of the cube l h ⇒ a= 8
= (6a2) sq. units. ∴ Volume = (8 × 8 × 8) cm3
b
(iii) Diagonal of the cube Cuboid = 512 cm3
= (a√⎯ 3) units. Q. 2. The diagonal of a cube is
4 √⎯ 3 m. Find its volume and sur-
3. Cylinder—Let the radius of the face area.
base of a cylinder be r units and
Solution : Let the edge of the
its height (or length) be h units,
cube be a metres
then—
(i) Volume of the cylinder Then ⎯√ 3a = 4 ⎯√ 3 or a = 4
= (πr2 h) cu. units. ∴ Edge = 4 m
So, Volume = (4 × 4 × 4)m3
(ii) Curved surface area of the
cylinder = 64 m3
= (2 πrh) sq. units. Sphere Surface area = 6a2
= (6 × 4 × 4)m 2 = 96 m2
(iii) Total surface area of the
cylinder Q. 3. Find the length of the
longest pole that can be placed in a
= (2 πrh + 2πr2 ) sq. units. room 5 m long, 4 m broad and 3 m
4. Sphere—Let the radius of a high.
sphere be r units, then— Solution : Length of longest pole
(i) Volume of the sphere l = Length of diagonal
= ( )
4 3
3
πr cu. units. =⎯
√⎯⎯⎯⎯⎯⎯⎯
52 + 4 2 + 3 2 m

(ii) Surface area of the sphere =⎯√⎯50 m = 5 ⎯√ 2 m


r
= (4 πr2 ) sq. units. = (5 × 1·41) m = 7·05 m.
Q. 4. Find the volume, surface
(iii) Volume of a hemisphere Cylinder area and the length of diagonal of a
= ( )
2 3
3
πr cu. units. cuboid 12 m long, 9 m broad and
8 m high.
(iv) Curved surface area of the Solution : Volume
hemisphere = (12 × 9 × 8) m3 = 864 m3
h
= (2πr2 ) sq. units. Surface area
=2 × (12 × 9 + 9 × 8 + 12 × 8)m2
(v) Whole surface area of the
r = 552 m2
hemisphere
= (2πr2 + πr2 ) Diagonal = ⎯√⎯⎯⎯⎯⎯⎯⎯⎯⎯⎯⎯
(12) 2 + (9)2 + (8)2
= (3πr2 ) sq. units. Cone = √⎯⎯⎯
289 m = 17 m.

Quantitative Aptitude Test | 195


Q. 5. Three cubes of sides 6 6. The length of longest rod that (C) 1038 cm2
cm, 8 cm and 1 cm are melted to can fit in a cubical vessel of side (D) None of these
form a new cube. Find the length 10 cm, is—
of the edge of the new cube. 15. The surface area of a cube of
(A) 10 cm
side 27 cm is—
Solution : Volume of new cube (B) 10 ⎯√ 2 cm (A) 2916 cm2 (B) 729 cm2
= [(6)3 + (8)3 + (1)3 ] cm3
(C) 10 √⎯ 3 cm (C) 4374 cm2 (D) 19683 cm2
= 729 cm3
(D) None of these 16. A tank 3 m long, 2 m wide and
∴ Edge of new cube
7. The length of longest pole that 1·5 m deep is dug in a field 22 m
= (9 × 9 × 9)1/3 = 9 cm
can be placed on the floor of a long and 14 m wide. If the earth
Q. 6. The capacity of a cylin-
room is 10 m and the length of dug out is evenly spread out over
drical tank is 6160 m3. If the radius
the longest pole that can be the field, the rise in level of the
of its base is 14 m, find the depth of field will be—
the tank. placed in the room is 10 √⎯ 2 m. (A) 0·299 cm
Solution : The height of the room is—
(B) 0·29 cm
(A) 6 m (B) 7·5 m
22 (C) 2·98 cm
˙·˙ × 14 × 14 × h = 6160 (C) 8 m (D) 10 m
7 (D) 4·15 cm
8. The length of diagonal of a cube
(
∴ h = 6160 × ×
7 1
22 14 × 14 ) (
is 14 × √ )
⎯ 3 cm. The volume of
17. Two cubes have volumes in the
ratio 1 : 27. The ratio of their
= 10 m the cube is— surface areas is—
(A) 2744 √⎯ 3 cm3 (A) 1 : 3 (B) 1 : 8
Exercise (B) 2744 cm3 (C) 1 : 9 (D) 1 : 18
1. A wooden box of dimensions 8 m (C) 588 cm3 18. If each edge of a cube is in-
× 7 m × 6 m is to carry rect- (D) 3528 cm3 creased by 50%, the percentage
angular boxes of dimensions 8 increase in surface area is—
9. The maximum length of a pencil
cm × 7 cm × 6 cm. The maxi- (A) 50% (B) 75%
that can be kept in a rectangular
mum number of boxes that can box of dimensions 8 cm × 6 cm (C) 100% (D) 125%
be carried in 1 wooden box is— × 2 cm is— 19. The difference in volumes of
(A) 1200000 (B) 1000000 two cubes is 152 m 3 and the
(A) 2 ⎯√⎯13 cm
(C) 9800000 (D) 7500000 difference in their one face areas
(B) 2 √⎯⎯14 cm is 20 m2 . If the sum of their
2. Given that 1 cubic cm of marble
(C) 2 √⎯⎯26 cm edges is 10 m, the product of
weighs 25 gms, the weighs of a their edges is—
marble black 28 cm in width and (D) 10 √⎯ 2 cm (A) 21 (B) 24
5 cm thick is 112 kg. The length
10. The length of longest rod that (C) 36 (D) 48
of the block is—
can be placed in a room 20 m
(A) 36 cm (B) 37·5 cm 20. If each side of a cube is doubled,
long, 16 m broad and 12 m high,
(C) 32 cm (D) 26·5 cm then its volume—
is—
(A) Is doubled
3. The sum of length, breadth and (A) 20 m (B) 16·4 m
(B) Becomes 4 times
depth of a cuboid is 19 cm and (C) 48 m (D) 28·2 m
(C) Becomes 6 times
its diagonal is 5⎯
√ 5 cm. Its sur- 11. The length of the diagonal of a (D) Becomes 8 times
face area is— cuboid 30 cm long, 24 cm broad
(A) 361 cm2 (B) 125 cm2 and 18 cm high, is— 21. Three metal cubes of sides 5 cm,
4 cm and 3 cm are melted and
(C) 236 cm2 (D) 486 cm2 (A) 30 cm (B) 15 √
⎯ 2 cm recast into a new cube. The
4. If the length breadth and the (C) 60 cm (D) 30 √⎯ 2 cm length of the edge of this cube,
height of cuboid are in the ratio is—
12. The volume of a cube is 512
6 : 5 : 4 and if the total surface (A) 6 cm
cm3. Its surface area is—
area is 33300 cm2 , then the (B) 8 cm
length, breadth and height in cm. (A) 64 cm2 (B) 256 cm2
(C) 384 cm (D) 512 cm2
2 (C) 10 cm
are respectively— (D) None of these
(A) 90, 85, 60 (B) 85, 75, 60 13. The surface area of a cube is 726
m2 . Its volume is— 22. If the volumes of two cubes are
(C) 90, 75, 70 (D) 90, 75, 60
(A) 1300 m3 (B) 1331 m3 in the ratio 8 : 1, the ratio of their
5. If the length of diagonal of a edges, is—
(C) 1452 m3 (D) 1542 m3
cube is 8 ⎯
√ 3 cm, then its surface 14. The surface area of a cuboid 22
(A) 8 : 1
area is— cm by 12 cm by 7·5 cm is— (B) 2√
⎯2:1
(A) 512 cm2 (B) 384 cm2 (A) 1980 cm2 (C) 2 : 1
(C) 192 cm2 (D) 768 cm2 (B) 2076 cm2 (D) None of these

Quantitative Aptitude Test | 196


23. Half cubic metre of gold sheet is (C) 8316 sq. cm 39. The number of solid spheres,
extended by hammering so as to (D) 4158 sq. cm each of diameter 6 cm that could
cover an area of 1 hectare. The be moulded to form a solid metal
thickness of the sheet is— 32. The curved surface area of a cylinder of height 45 cm and
(A) 0·5 cm (B) 0·05 cm sphere is 1386 sq. cm. Its volume, diameter 4 cm, is—
(C) 0·005 cm (D) 0·0005 cm is—
(A) 3 (B) 4
(A) 2772 cm3 (B) 4158 cm3
24. In a shower, 5 cm of rain falls. (C) 5 (D) 6
The volume of water that falls on (C) 4851 cm3 (D) 5544 cm3
2 hectares of ground, is— 33. A right cylindrical vessel is full 40. The length of the wire of 0·2 mm
(A) 100 m3 (B) 1000 m3 with water. How many right radius that can be drawn after
(C) 10 m3 (D) 10000 m3 cones having the same diameter melting a solid copper sphere of
and height as those of the right diameter 18 cm, is—
25. A river 2 m deep and 45 m wide
is running at the rate of 3 km/hr. cylinder will be needed to store (A) 24·3 m (B) 243 m
The amount of water that runs that water— (C) 2430 m (D) 24300 m
into the sea per minute is— (A) 2 (B) 3 41. The radius of a wire is decreased
(A) 4500 m3 (B) 27000 m3 (C) 4 (D) 5 to one-third. If volume remains
(C) 3000 m3 (D) 2100 m3 34. A right cylinder and a right the same, length will increase—
26. Bricks are worth Rs. 750 per circular cone have the same (A) 1 time (B) 3 times
1000 and their length, breadth radius and the same volume. The (C) 6 times (D) 9 times
and height 25 cm, 12·5 cm and ratio of the height of the cylinder
7·5 cm respectively. The cost of to that of the cone is— 42. The height of cylinder is 14 cm
bricks required to build a wall (A) 3 : 5 (B) 2 : 5 and its curved surface area is 264
200 m long, 1·8 m high and 37·5 (C) 3 : 1 (D) 1 : 3 sq. cm. The volume of the cylin-
cm thick is— der is—
35. A cylindrical vessel of radius 4
(A) Rs. 42,600 (A) 308 cm3 (B) 396 cm3
cm contains water. A solid sphere
(B) Rs. 43,200 of radius 3 cm is lowered into (C) 1448 cm3 (D) 1232 cm3
(C) Rs. 40,750 the water until it is completely 43. The length of cylinder is 80 cm
(D) Rs. 41,860 immersed. The water level in the and the diameter of its base is 7
27. A metal sheet 27 cm long, 8 cm vessel will rise by— cm. The whole surface of the
broad and 1 cm thick is melted (A) 4·5 cm (B) 2·25 cm cylinder is—
into a cube. The difference bet- 4 2 (A) 1837 cm2 (B) 1760 cm2
ween the surface areas of two (C) cm (D) cm
9 9 (C) 3080 cm2 (D) 1942 cm2
solids is—
36. The radius of a circular cylinder 44. The area of the card board
(A) 284 cm2 (B) 286 cm2 is the same as that of a sphere. needed to make a box of size 25
(C) 296 cm2 (D) 300 cm2 Their volumes are equal. The cm × 15 cm × 8 cm will be—
28. The area of the base of a rectan- height of the cylinder is—
gular tank is 6500 sq. cm and the (A) 390 cm2 (B) 1000 cm2
4
volume of water contained in it (A) times its radius (C) 1390 cm2 (D) 2780 cm2
3
is 2·6 cubic metres. The depth of 2
(B) times its radius 45. The dimensions of an open box
water is— 3 are 52 cm, 40 cm and 29 cm. Its
(A) 3·5 m (B) 4 m (C) Equal to its radius thickness is 2 cm. If 1 cm3 of
(C) 5 m (D) 8 m
(D) Equal to its diameter metal used in the box weight 0·5
29. The volume of a wall, 5 times as gm, the weight of the box is—
high as it is broad and 8 times as 37. If 1 cubic cm of cast iron weighs
21 gm, then the weight of a cast (A) 8·56 kg (B) 7·76 kg
long as it is high, is 12·8 m3. The
breadth of the wall is— iron pipe of length 1 m with a (C) 7·576 kg (D) 6·832 kg
(A) 30 cm (B) 40 cm bore of 3 cm and in which the
thickness of the metal is 1 cm, 46. If the volumes of two cones are
(C) 22·5 cm (D) 25 cm in the ratio 1 : 4 and their dia-
is—
30. The number of small cubes with meters are in the ratio 4 : 5, then
(A) 21 kg (B) 24·2 kg
edges of 10 cm that can be the ratio of their heights is—
(C) 26·4 kg (D) 18·6 kg
accommodated in a cubical box (A) 1 : 5 (B) 5 : 4
1 metre edge is— 38. A hollow garden roller 63 cm
(C) 5 : 16 (D) 25 : 64
(A) 10 (B) 100 wide with a girth of 440 cm is
(C) 1000 (D) 10000 made of iron 4 cm thick. The 47. If a right circular cone of vertical
31. The volume of a sphere is 38808 volume of iron used is— height 24 cm has a volume of
cu. cm. The curved surface area (A) 56372 cubic m 1232 cm3 , then the area of its
of the sphere is— (B) 107712 cubic cm curved surface is—
(A) 5544 sq. cm (C) 54982 cubic cm (A) 1254 cm2 (B) 704 cm2
(B) 1386 sq. cm (D) 57636 cubic cm (C) 550 cm2 (D) 154 cm2

Quantitative Aptitude Test | 197


48. A radius of the base of a right 56. A spherical lead ball of radius 10 65. If the height of a cone is dou-
circular cone is 6 cm and its cm is melted and small lead balls bled, then its volume is increased
slant height is 28 cm. The curved of radius 5 mm are made. The by—
surface area of the cone is— total number of possible small (A) 100% (B) 200%
(A) 268 sq. cm lead balls— (C) 300% (D) 400%
(B) 528 sq. cm (A) 800 (B) 125 66. The percentage increase in the
(C) 462 sq. cm (C) 400 (D) 8000 surface area of a cube when each
(D) 658 sq. cm 57. Two circular cylinders of equal side is doubled, is—
volume have their heights in the (A) 25% (B) 50%
49. A cylindrical piece of metal of
ratio 1 : 2. Ratio of their radii (C) 150% (D) 300%
radius 2 cm and height 6 cm is
shaped into a cone of same is— 67. The cost of painting the four
radius. The height of the cone (A) 1 : √
⎯2 (B) √
⎯2:1 walls of a room is Rs. 350. The
is— (C) 1 : 2 (D) 1 : 4 cost of painting a room three
(A) 18 cm (B) 14 cm times in length, breadth and
58. The radii of two spheres are in height respectively will be—
(C) 12 cm (D) 8 cm the ratio 1 : 2. The ratio of their (A) Rs. 1050 (B) Rs. 1400
50. A cylindrical vessel 60 cm in surface areas is—
(C) Rs. 3150 (D) Rs. 4200
diameter is partially filled with (A) 1 : 2 (B) 1 : 4
water. A sphere 60 cm in dia- 68. The length of canvas 1·1 m wide
meter is gently dropped into the (C) 1 : √
⎯2 (D) 3 : 8 required to build a conical tent of
vessel. To what further height 59. The curved surface areas of two height 14 m and the floor area
will water rise in the cylinder ? spheres are in the ratio 1 : 4. The 346·5 m2 , is—
(A) 15 cm (B) 30 cm ratio of their volumes is— (A) 665 m (B) 525 m
(C) 490 m (D) 860 m
(C) 40 cm (D) 25 cm (A) 1 : 4 (B) 1 : 2 ⎯
√2
69. The area of the base of a right
51. The radius of two cylinders are (C) 1 : 8 (D) 1 : 64
circular cone is 154 cm2 and its
in the ratio of 2 : 3 and their 60. Volume of a hemisphere is 19404 height is 14 cm. The curved
heights are in the ratio 5 : 3. The cubic cm. The total surface area surface of the cone is—
ratio of their volumes is— is—
(A) 27 : 20 (B) 20 : 27 (A) 2772 sq. cm
(
(A) 154 × √ )
⎯ 5 cm2
(C) 4 : 9 (D) 9 : 4 (B) 11 cm2
(B) 4158 sq. cm
52. The ratio of total surface area to (C) 5544 sq. cm (
(C) 154 + √ )
⎯ 7 cm2
lateral surface area of a cylinder (D) 1386 sq. cm (D) 5324 cm2
whose radius is 80 cm and height 88 70. A cone of height 7 cm and base
20 cm, is— 61. The volume of a sphere is × radius 3 cm is curved from a
21
(A) 2 : 1 (B) 3 : 1 3 3
(14) cm . The curved surface of rectangular block of wood 10 cm
(C) 4 : 1 (D) 5 : 1 this sphere is— × 5 cm × 2 cm. The percentage
(A) 2424 cm2 (B) 2446 cm2 of wood wasted is—
53. If the volume and surface area of (A) 34% (B) 46%
a sphere are numerically the same (C) 2464 cm2 (D) 2484 cm2
(C) 54% (D) 66%
then its radius is— 62. A cylinder and a cone have the 71. The material of a cone is
(A) 1 unit (B) 2 units same height and same radius of converted into the shape of a
(C) 3 units (D) 4 units the base. The ratio between the cylinder of equal radius. If the
volumes of the cylinder and the height of the cylinder is 5 cm,
54. A spherical ball of lead, 3 cm in cone is— the height of the cone is—
diameter is melted and recast (A) 1 : 3 (B) 3 : 1
into three spherical balls. The (A) 10 cm (B) 15 cm
(C) 1 : 2 (D) 2 : 1 (C) 18 cm (D) 24 cm
diameter of two of these are 1·5
cm and 2 cm respectively. The 63. If the radius of a sphere is dou- 72. A solid consists of a circular
diameter of the third ball is— bled, then its volume is increased cylinder with an exact fitting
by— right circular cone placed on the
(A) 2·66 cm (B) 2·5 cm
(A) 100% (B) 200% top. The height of the cone is h.
(C) 3 cm (D) 3·5 cm
(C) 700% (D) 800% If the total volume of the solid is
55. How many bullets can be made three times the volume of the
out of a cube of lead whose edge 64. If the radius of a sphere is cone, then the height of the
measures 22 cm, each bullet doubled, then its surface area is cylinder is—
being 2 cm in diameter ? increased by— (A) 2h (B) 4h
(A) 5324 (B) 2662 (A) 100% (B) 200% 2h 3h
(C) 1347 (D) 2541 (C) 300% (D) 50% (C) (D)
3 2

Quantitative Aptitude Test | 198


Answers with Hints
1. (B) Number of boxes 10. (D) Length of the rod = ⎯√⎯⎯⎯⎯⎯⎯⎯⎯⎯⎯⎯⎯⎯
(20) 2 + (16)2 + (12)2 m
Volume of wooden box in cm3
=
Volume of 1 small box = √⎯⎯⎯
800 m
800 × 700 × 600 = 20 √
⎯ 2 m = (20 × 1·41) m
= = 1000000
8×7×6 = 28·2 m
112 × 1000
2. (C) Volume =
25 ( cu. cm. ) 11. (D) Diagonal = ⎯√⎯⎯⎯⎯⎯⎯⎯⎯⎯⎯⎯⎯⎯
(30) 2 + (24)2 + (18)2

= 4480 cm3 = √⎯⎯⎯⎯


1800 cm = 30 ⎯ √ 2 cm.
∴ x × 28 × 5 = 4480 12. (C) ˙·˙ a3 = 512 = 8 × 8 × 8
⇒ a = 8 cm
⇒ x =
4480
28 × 5 ( )
cm = 32 cm.
∴ Surface area = 6a2
3. (C) ˙·˙ l + b + h = 19 = [6 × (8)2] cm2
= 384 cm2
and l2 + b2 + h2 = (5 ⎯√ 5) 2
= 125
13. (B) 2
6a = 726
˙·˙ (l + b + h)2
= (19) 2
⇒ a2 = 121
⇒ (l + b + h ) + 2 (lb + bh + lh)
2 2 2
⇒ a = 11 cm
= 361
∴ Volume of the cube = (11 × 11 × 11) cm3
⇒ 2 (lb + bh + lh) = (361 – 125)
= 1331 cm3
= 236
14. (C) Surface area
∴ Surface area = 236 cm2 .
= 2 (22 × 12 + 12 × 7·5 + 22 × 7·5) cm2
4. (D) Let length = 6x, breadth = 5x and height = 4x in
= 1038 cm2
cm
15. (C) Surface area = [6 × (27)2] cm2
˙·˙ 2 (6x × 5x + 5x × 4x + 6x × 4x) = 33300
= 4374 cm2
⇒ 148x2 = 33300
16. (C) Earth dug out = (3 × 2 × 1·5) m3
33300 = 9 m3
⇒ x2 = = 225
148 Area on which earth has been spread
⇒ x = 15 = (22 × 14 – 3 × 2) m2
∴ Length = 90 cm, Breadth = 75 cm, Height = 60 cm = 302 m2
5. (B) ˙·˙ ⎯

√ 3a = 8 ⎯√ 3 ⇒ a = 8
Surface area = 6a2
∴ Rise in level =
Volume
Area ( )
=
9
302
m

9 × 100
= (6 × 8 × 8) cm2 =( 302 ) cm = 2·98 cm
= 384 cm2 17. (C) Let the volumes be x3 and 27x3
6. (C) Longest rod = ⎯⎯⎯⎯⎯⎯⎯⎯⎯⎯⎯⎯⎯⎯
√ (10) 2 + (10)2 + (10)2 cm ∴ Their edges are x and 3x
= √
⎯⎯⎯
300 cm = 10 ⎯
√ 3 cm. Ratio of their surface area
7. (D) ˙·˙ l2 + b2 2
= (10) = 100 = 6x2 : 54x2 = 1 : 9
18. (D) Let original length of cube = x
and l2 + b2 + h2 = (10√⎯ 2) 2
= 200 Then, its surface area = 6x2
⇒ h2 = (200 – 100) = 100
∴ h = 10 m
New edge = ( )150
100
x

3
8. (B) ˙·˙ ⎯ 3a = 14 × ⎯√ 3 ⇒ a = 14
√ = x
2
∴ Volume of the cube = (14 × 14 × 14) cm3
= 2744 cm3
New surface area = 6 × ( ) 3 2
2
x

9. (C) Length of pencil = [(8 2 ) + (6) 2 + (2)2] cm


= ⎯⎯⎯
√ 104 cm
( )
= 6 × x2
9
4

= 2 ⎯⎯26 cm
√ = ( )27
2
x2

Quantitative Aptitude Test | 199


Increase in surface area = (272 – 6) x 2 26. (B) Number of bricks required

=
Volume of wall in cm3
15 2 Volume of 1 brick
= x
200 × 100 × 1·8 × 100 × 37·5
2
= ( 25 × 12·5 × 7·5 )
∴ Increase per cent = (
15 2 1
2
x × 2 × 100 %
6x ) = 57,600
= 125% ∴ Required cost
750 × 57‚600
19. (B) Let the edges of the two cubes be x and y metres
Then, x3 – y3 = 152
= Rs. ( 1000 )
and (x2 – y2) = 20 = Rs. 43,200
Also, (x + y) = 10 27. (B) Volume of cube formed = 216 cm3
⎛ x2 – y2 ⎞ ∴ Edge of the cube = (6 × 6 × 6)1/3
So, (x – y) = ⎜ x+y ⎟ = 6 cm
⎝ ⎠
Surface area of original metal sheet
20
= =2 = 2 (27 × 8 + 8 × 1 + 27 × 1) cm2
10
x3 – y3 = 502 cm2
Now, x–y = ( )
152
2 Surface area of the cube formed
⇒ 2 2
x + y + xy = 76 = [6 × (6)2] cm2 = 216 cm2
⇒ (x + y)2 – xy = 76 ∴ Required difference in areas of two solids
⇒ xy = (x + y)2 – 76 = (502 – 216) cm2
= (10) 2 – 76 = 24 = 286 cm2
20. (D) Let the edge of original cube = x cm 28. (B) Let depth = x cm.
Edge of new cube = (2x) cm Then, x × 6500 = 2·6 × 100 × 100 × 100
Ratio of their volumes = x3 : (2x)3
= x3 : 8x3 ∴ x = (2·6 × 1006500
× 100 × 100
) cm
= 1:8 = 400 cm = 4 m
Thus the volume be comes 8 times. 29. (B) Let breadth = x metres. Then, height = 5x metres
21. (A) Volume of new cube = [(5)3 + (4)3 + (3)3] cm3 and length = 40x metres
= 216 cm3 ∴ x × 5x × 40x = 12·8
Edge of this cube = (6 × 6 × 6)1/3 = 6 cm 12·8 128
22. (C) Let their volumes be 8x3 and x3 respectively ⇒ x3 = =
200 2000
Then, their edges are 2x and x respectively 64
=
∴ Ratio of their edges = 2 : 1 1000
Volume 4
23. (C) Thickness = ∴ x =
Area 10
= ( 1
×
2 10000
1
m ) Thus, breadth = ( )
4
10
m
1 × 100
= ( 2 × 10000
cm ) = (
4 × 100
10 )
cm = 40 cm
= 0·005 cm 100 × 100 × 100
30. (C) Number of Cubes =
24. (B) (
Volume = 2 × 10000 ×
5
100
m3) 10 × 10 × 10
= 1000
= 1000 m3 4 22 3
31. (A) ˙·˙ × × r = 38808
3 × 1000
25. (A) Speed per min. = (
60 )
m = 50 m
3 7

Volume of water running per min.


⇒ r3 = (38808 ×227 ×34)
= (45 × 2 × 50) m2 = (21) 3
= 4500 m3 ∴ r = 21 cm

Quantitative Aptitude Test | 200


So, Curved surface area of sphere 38. (B) ˙·˙ Circumference of the girth = 440 cm
= 4πr2 ∴ 2πr = 440

( 22
)
= 4 × × 21 × 21 cm2
7
⇒ r = ( 440 7
2 22
×)
= 5544 cm2 = 70 cm
32. (C) 2
4πr = 1386 Thus, Outer radius = 70 cm

⇒ (
r2 =
1386 7
4
×)22
Inner radius = (70 – 4) cm
= 66 cm
441 Volume of iron = π [(70)2 – (66) 2 ] × 63cm3
=
4

∴ r =
21
= ( 22
7 )
× 136 × 4 × 63 cm3
2 = 107712 cm3
∴ Volume = ( 4 22 21 21 21
× × × ×
3 7 2 2 2 ) cm3 39. (C) Let the number of spheres be x
4
= 4851 cm3 Then, x × π × (3)3 = π × (2) 2 × 45
3
33. (B) Let x cones be needed ⇒ 36x = 180
1 180
Then, πr2 h × x = πr2 h ⇒ x = 3. ∴ x = =5
3 36
34. (D) Let the height of cylinder = h 40. (D) Radius of sphere = 9 cm
and height cone = H

Then,
1
πr2 h = πr2 H
Volume of sphere = [ 4
3 ]
× π × (9)3 cm3
3 = (972 π) cm3
h 1 Radius of wire = 0·2 mm
⇒ = =1:3
H 3

35.
4
(B) ˙·˙ π × (4) 2 × h = π × (3)3
= ( 2
10 × 10) cm
3 1
= cm
9 50
∴ h = cm
4 Let the length of wire be
= 2·25 cm = x cm
4 3
36. (A) ˙·˙
3
πr = πr2 h Then, 972 π = π × ( ) 1 2
50
×x
4 ⇒ x = (972 × 50 × 50) cm
⇒ h = r
3 972 × 50 × 50

4
Height = times its radius.
∴ Length of wire = ( 100 ) m
3 = 24,300 m
37. (C) External radius = 2·5 cm, 41. (D) Let original radius = r and original length = h
length = 100 cm 1
New radius = r
∴ External volume = [π × (2·5)2 × 100] cm3 3
Internal radius = 1·5 cm Let new length = H
∴ Internal volume = [π × (1·5)2 × 100] cm3
Volume of metal
Then, πr2 h = π ( )
1 2
3
r ×H

= [π × (2·5)2 × 100 – π × (1·5) 2 × 100] cm3 πr2 H


=
= π × 100 × [(2·5)2 – (1·5)2 ] cm3 9
∴ H = 9h
= ( 22
7 )
× 100 × 4 × 1 cm3
Thus, the length becomes 9 times.
∴Weight of metal 42. (B) ˙·˙ 2πrh = 264

= (
22
7
× 100 × 4 × 1 ×
21
1000 ) kg ⇒2 ×
22
7
× r × 14 = 264

= 26·4 kg. ∴ r = 3

Quantitative Aptitude Test | 201


So, Volume = πr2 h 1
49. (A) ∴ π × (2)2 × h = π × (2) 2 × 6
3
(
=
22
7 )
× 3 × 3 × 14 cm3
⇒ h = 18 cm
= 396 cm3 50. (C) Let h and H be the heights of water level before
43. (A) ˙·˙ r = 3·5 cm and after the dropping of the sphere.
and h = 80 cm Then, [π × (30) 2 × H] – [π × (30)2 × h]
∴Whole surface 4
= π × (30)3
= 2πrh + 2πr2 = 2πr (h + r) 3
4
[ 22
= 2 × × 3·5 × (80 + 3·5) cm2
7 ] ⇒ π × 900 × (H – h) = π × 27000
3
= 22 × 83·5 ∴ (H – h) = 40 cm
= 1837 cm2 51. (B) Let their radii be 2r and 3r and heights 5h and 3h
44. (C) Area needed respectively.
= 2 (25 × 15 + 15 × 8 + 25 × 8) cm2 π (2r) 2 × 5 h
∴ Ratio of their volumes =
= 2 [375 + 120 + 200] π (3r) 2 × 3 h
= 1390 cm2 20
= = 20 : 27
45. (D) 27
46. (D) Since the diameters are in the ratio 4 : 5. It Total surface area 2π rh + 2πr2
follows that their radii are in the ratio 4 : 5. 52. (D) =
Lateral surface area 2πrh
Let them be 4r and 5r. Let the heights be h and H.
1
3
π × (4r)2 × h
=
2πr (h + r)
2π rh
= ( )h+r
h
˙·˙ Ratio of volumes =
1
3
π × (5r)2 × H = (2020+ 80) = 51
16 h = 5:1
=
25 H 4 3
53. (C) πr = 4πr2
16 h 1 3
⇒ = ⇒ r = 3 units.
25 H 4


h
H
= ( 1 25
×
4 16) 54. (B) ˙·˙
4 3
3
4
πr = π ×
3 [( ) {( ) }]
3 3
2

3 3
4
+ 13

=
25
64
= 25 : 64. ⇒ r3 =
125
64
=() 5 3
4
1 22 2 5
47. (C) ˙·˙ × × r × 24 = 1232 ⇒ r =
3 7 4

⇒ (
r2 = 1232 × ×
7 3
22 24 ) ∴ Diameter = ( )
5
4
× 2 cm

= 2·5 cm.
= 49
Volume of cube
∴ r = 7 cm 55. (D) Number of bullets =
Volume of 1 bullet
Now, r = 7 22 × 22 × 22 ⎞
= ⎛⎜ ⎟
and h = 24 ⎜⎝ × 22 × 1 × 1 × 1⎟⎠
4
3 7
So, l = ⎯⎯⎯⎯⎯⎯⎯
√ 72 + (24)2
= 2541.
= √
⎯⎯⎯
625 = 25 cm Volume of big ball
56. (D) Number of balls =
∴ Curved surface area = πrl Volume of 1 small ball
4
= (22
7 )
× 7 × 25 cm2
=
3
× π × 10 × 10 × 10
4
= 550 cm2 × π × 0·5 × 0·5 × 0·5
3
48. (B) Curved surface area = πrl = 8000
=
22
7 (
× 6 × 28 cm2 ) 57. (B) Let their heights be h and 2h and radii be x and y
respectively.
= 528 cm2 Then, πr2 h = πy2 (2h)

Quantitative Aptitude Test | 202


x2 2 66. (D) Original area = 6a2
⇒ =
y2 1 New area = 6 (2a)2 = 24a2
x ⎯2 ⎯2:1
√ ⎛ 18a2 ⎞
y = 1 =√
∴ Required increase % = ⎜ 6a2 × 100⎟ %
⎝ ⎠
58. (B) Let their radii be x and 2x.
4πx2 = 300%
1
Ratio of their surface areas = = 67. (C) Area of 4 walls of the room
4π (2x)2 4
= 1:4 = [2 (l + b) × h] m2
59. (C) Area of 4 walls of new room
2 22 3 = [2 (3l + 3b) × 3h]m2
60. (B) × × r = 19404 c.c.
3 7
= 9 [2 (l + b) × h] m2
⇒ (
r3 = 19404 × ×
7 3
22 2 ) = 9261 c.c. ∴ Cost of painting the 4 walls of the new room
= (21) 3 c.c. = Rs. (9 × 350)
∴ r = 21 cm = Rs. 3150
Total Surface area = 3πr2 68. (B)
( 22
= 3 × × 21 × 21 cm2
7 ) 69. (A) ˙·˙
22 2
7
× r = 154
= 4158 cm2
61.
4 22
(C) ˙·˙ × × r3 =
3 7
88
21
× (14)3
⇒ (
r2 = 154 ×
7
22 )
= 49
⇒ r = 14
∴ r = 7 cm
( 22
∴ Curved surface = 4 × × 14 × 14 cm2
7 ) and h = 14
= 2464 cm2 So, l = ⎯⎯⎯⎯⎯⎯⎯⎯
√ (7) 2 + (14)2
62. (B) Ratio of their volumes
= √ ⎯⎯⎯
245 = 7 ⎯
√ 5 cm
πr2 h 3
= = ∴ Area of curved surface
1 2 1
πr h
3 = πrl

63.
= 3:1
(C) Original volume = πr3
4 = (
22
7
× 7 × 7⎯√ 5 cm2 )
3
4 32 = 154 ⎯ 5 cm2

New volume = π (2r)3 = πr3
3 3 70. (A) Total volume of cuboid
= (10 × 5 × 2) cm3
Required increase % = ( 28 3
3
3
)
πr × 3 × 100%
4πr = 100 cm3
= 700%
64. (C) Original area = 4πr2 , New area = 4π (2r)2 Volume curved = (
1 22
× × 3 × 3 × 7 cm3
3 7 )
= 16 πr2 = 66 cm3
⎛ 12 πr2 ⎞ % of Wood wasted = (100 – 66)%
Required increase % = ⎜ 4πr2 × 100⎟ %
⎝ ⎠ = 34%
= 300% 1 2
71. (B) ˙·˙ πr × h = πr2 × 5
1 3
65. (A) Original volume = πr2 h ;
3 ∴ h = 15 cm
1 2
New volume = πr (2h) 72. (C) Let the height of the cylinder be H and its radius
3
2 = r
= πr2 h 1 1
3 Then, πr2 H + πr2 h = 3 × πr2 h
⎛ 1 πr2 h ⎞ 3 3
⎜3 ⎟ 2
Required increase % = ⎜⎜ × 100⎟⎟ % ⇒ πr2 H = πr2 h
⎜⎝ 13 πr2 h 3
⎟⎠
2
∴ H = h.
= 100% 3

Quantitative Aptitude Test | 203


Races and Games of Skill
Races—A contest of speed in Time taken by A to cover 1 km can A give B in a kim race. So that
running, riding, driving sailing or = (240 – 90) seconds the race may and in a dead heat.
rowing is called a race. The ground Solution : Time taken by A to
= 150 seconds
of path on which contests are made is run 1 km = 4 mts. 50 sec. = 290 sec.
called a race course. The point from Time taken by B to cover 1 km Time taken by B to run 1 km = 5
which a race begins is known as a = (240 – 30) seconds mts. = 300 sec.
starting point. The point set to bound
a race is called a winning post or a = 210 seconds. ∴ A can give B, a start of (300 –
goal. The person who first reaches Q. 2. In a kilometre race, if A 290) or 10 second
the winning post is called a winner. If gives B, a start of 40 metres, then A Now, in 300 seconds, B runs
all the persons contesting a race wins by 19 seconds, but if A gives 1000 metres.
reach the goal exactly at the same B, a start of 30 seconds then B wins ∴ In 10 seconds, B runs
time, then the race is said to be a
dead heat race.
by 40 metres. Find the time taken
by each to run a kilometre. ( 1000
300 )
× 10 m = 33 m
1
3
Suppose A and B are two Solution : Suppose that the time 1
contestants in a race. If before the taken by A and B to run 1 km is x So, A can give B a start of 33
3
start of the race, A is at the starting and y seconds respectively. metres.
point and B is ahead of A by 15
When A gives B a start of 40 Q. 4. In a 100 metres race, A
metres. Then A is said to give B, a
metres then A has run 1000 metres, runs with 6 kms. per hour. If A
start of 15 metres. To cover a race of
while B has to run only 960 metres. gives B a start of 4 metres and still
200 metres in this case A will have to
cover a distance of 200 metres and B Time taken by A to run 1000 m beats him by 12 seconds. What is
the speed of B ?
will have to cover (200 – 15) or 185 = x sec.
metres only. Solution : Time taken by A to
Time taken by B to run 960 cover 100 metres
In a 100 metres race A can give
B 15 metres or A can give B, a start metres = ( ) y
1000
× 960 sec.
=( 60 × 60
6000 )
× 100 seconds
of 15 metres or A beats B by 15
metres means that while A runs 100
metres, B runs (100 – 15) or 85
= ( ) 24
25
y sec. = 60 seconds
∴ B covers (100 – 4) or 96 m in
metres. 24 (60 + 12) sec. or 72 sec.
Clearly, ˙·˙ y – x = 19
Games—A game of 100 means 25 Hence, speed of B
that the person among the contestants ⇒ 24y – 25x = 475 …(i) 60 × 60 × 60
who scores 100 points first is the
winner. If A scores 100 points, while Again, A gives B, a start of 30
=( 72 × 1000 ) km/hr.

B scores only 80 points, then we say seconds, then B runs for y seconds, = 4·8 km/hr.
that A can give B 20 points. while A runs for (y – 30) seconds.
Q. 5. A can run a km in 3 min.
Now, ˙·˙ In x seconds, A covers 10 sec. and B in 3 min. 20 sec. By
Examples 1000 metres what distance can A beat B ?
∴ In (y – 30) seconds A will Solution :
Q. 1. A and B run a km and A
wins by 1 minute. A and C run a
km and ‘A’ wins by 375 metres. B
cover = [ 1000
]
x × (y – 30) metres
˙·˙ A beats B by 10 seconds.
Distance covered by B in 200
and C run a km and B wins by 30 1000 × (y – 30) seconds = 1000 metres.
So, 1000 – = 40
seconds. Find the time taken by x Distance covered by B in 10
each to run a km. 1000
⇒ 25y – 24x = 750 …(ii) seconds = × 10
Solution : Since A beats B by 60 200
Solving (i) and (ii) we get,
seconds and B beats C by 30 seconds. = 50 metres
So, A beats C by 90 seconds. But, it x = 125 and y = 150
∴ A beats B by 50 metres.
being given that A beats C by 375 ∴ Time taken by A to run 1 km Q. 6. A, B and C are the three
metres. So it means that C covers 375 = 125 seconds contestants in a km race. If A can
metres in 90 seconds.
Time taken by B to run 1 km = give B a start of 40 metres and A
∴ Time taken by C to cover 1 150 seconds can give C a start of 64 metres how
km = ( 90
375 )
× 1000 seconds Q. 3. A can run a kilometre in many metres start can B give C ?
4 minutes 50 seconds and B in 5 Solution : While A covers 1000
= 240 seconds minutes. How many metres start metres, B covers (1000 – 40) or 960

Quantitative Aptitude Test | 204


metres and C covers (1000 – 64) or 02. A can run 20 metres while B gives B a start of 8 metres and
936 metres. runs 25 metres. In a km race B still beats him by 8 seconds.
Now, when B covers 960 metres. beats A by— Speed of B is—
C covers 936 metres. (A) 250 metres (A) 5·15 km/hr
∴ When B covers 1000 metres, (B) 225 metres (B) 4·14 km/hr
C covers ( 936
960 )
× 1000 metres (C) 200 metres
(D) 125 metres
(C) 4·25 km/hr
(D) 4·4 km/hr
= 975 metres 03. In a 100 metres race A can give 11. A can beat B by 31 metres and C
So, B can give C a start of (1000 B 10 metres and C 28 metres. In by 18 metres in a race of 200
– 975) or 25 metres. the same race, B can give C— metres. In a race of 350 metres C
2 (A) 18 metres (B) 20 metres will beat B by—
Q. 7. A runs 1 times as fast as (A) 22·75 metres
3 (C) 27 metres (D) 9 metres
B. If A gives B a start of 80 metres (B) 25 metres
04. In a 100 metres race, A can beat
how far must the winning post be 4
B by 25 metres and B can beat C (C) 7 metres
so that A and B might reach it at 7
by 4 metres. In the same race, A
the same time ? (D) 19·5 metres
can beat C by—
Solution : The races of A and B (A) 29 metres (B) 21 metres 12. In a 100 metres race, A beats B
are as 5 : 3 (C) 28 metres (D) 26 metres by 10 metres and C by 13 metres.
i.e., in a race of 5 metres, A In a race of 180 metres, B will
05. In a 300 metres race A beats B
gains 2 metres over B. beat C by—
by 15 metres or 5 seconds. A’s
2 metres are gained by A in a time over the course is— (A) 5·4 metres
race of 5 metres. (B) 4·5 metres
(A) 100 seconds
80 metres will be gained by A in (B) 95 seconds (C) 5 metres
a race of (D) 6 metres
(C) 105 seconds
=( )5
2
× 80 metres = 200 metres (D) 90 seconds 13. In a 500 metres race, the ratio of
speeds of two contestants A and
06. A can run a kilometre in 4 min.
∴ Winning post is 200 metres B is 3 : 4. A has a start of 140
54 sec. and B in 5 min. How
away from the starting point. metres. Then, A wins by—
many metres start can A give B
Q. 8. In a km race A beats B in a km race so that the race may (A) 60 metres (B) 40 metres
by 35 metres or 7 seconds. Find A’s end in a dead heat ? (C) 20 metres (D) 10 metres
time over the course. (A) 20 metres (B) 16 metres 14. A can run 20 metres while B
Solution : Here B runs 35 metres (C) 18 metres (D) 14·5 metres runs 25 metres. In a km race B
in 7 seconds. beats A by—
07. A can run 100 metres in 27
∴ B’s time over the course seconds and B in 30 seconds. A (A) 25 m (B) 225 m
(C) 200 m (D) 125 m
= ( 7
35 )
× 1000 sec.
will beat B by—
(A) 9 metres 15. A can run a kilometre in 4
= 200 seconds. (B) 10 metres minutes 50 seconds and B in 5
1 minutes. How many metres start
So, A’s time over the course (C) 11 metres can A give B in a km race, so
8
= (200 – 7) sec. that the race may end in a dead
(D) 12 metres
= 193 seconds heat ?
08. At a game of billiards, A can (A) 30 m
= 3 min. 13 sec. give B 15 points in 60 and A can 2
give C 20 in 60. How many (B) 16 m
Exercise points can B give C in a game of 3
90 ? 1
3 (C) 33 m
01. A runs 1 times as fast as B. If A (A) 30 points (B) 20 points 3
4
(C) 10 points (D) 12 points (D) None of these
gives B a start of 60 metres.
How far must the winning post 09. In a game of 100 points, A can 16. A can run a km in 3 min. 10 sec.
be in order that A and B reach it give B 20 points and C 28 points. and B in 3 min. 20 sec. By what
at the same time ? Then B can give C— distance can A beat B ?
(A) 105 metres (A) 8 points (B) 10 points (A) 40 m (B) 50 m
(B) 80 metres (C) 14 points (D) 40 points (C) 45 m (D) 60 m
(C) 140 metres 10. A and B take part in a 100 metres 17. A takes 4 mins 50 secs. while B
(D) 45 metres race. A runs at 5 km per hour. A takes 5 mins. to complete the

Quantitative Aptitude Test | 205


1 24. P can run a kilometre in 4 31. A can give B 40 metres start and
race. A beats B by 33 metres.
3 minutes 50 seconds and Q in 5 A can give C 50 metres start in a
Find the length of the course— minutes. By what distance can P 200 metres race. While B can
(A) 1·1 km (B) 100 m beat Q ? give C two seconds over the
(C) 10 km (D) 1000 m 2 course. How long does each take
(A) 30 m (B) 16 m
3 to run 200 metres ?
18. In a 500 metres race, B gives A a 1 2 (A) 24 sec., 30 sec., 32 sec.
start of 160 metres. The ratio of (C) 33 m (D) 26 m (B) 20 sec., 31 sec., 32 sec.
3 3
the speeds of A and B is 2 : 3. (C) 20 sec., 30 sec., 32 sec.
Who wins and by how much ? 25. P runs 1 km in 4 minutes and Q (D) 24 sec., 30 sec., 31 sec.
in 4 minutes 10 secs. How many
2 32. X, Y and Z are the three
(A) 6 m (B) 8 m metres start can P give Q in 1
3 contestants in one km race. If X
kilometre race, so that the race
1 1 may end in a dead heat ? can give Y a start of 50 metres
(C) 8 m (D) 6 m
3 3 (A) 40 m and X can also give Z a start of
(B) 50 m 69 metres, how many metres
19. A can run one km in half a
start Y can give Z ?
minute less time than B. In a (C) 30 m
(A) 10 m (B) 40 m
kilometre race, B gets a start of (D) None of these (C) 20 m (D) 25 m
100 metres and loses by 100 1
metres. Find the time A and B 26. A runs 1 as fast as B. If A 33. In one km race A beats B by 5
3 seconds or 40 metres. How long
take to run a kilometre— gives B a start of 30 minutes. does B take to run the kilometre ?
1 How far must b e the winning
(A) 5 min, 5 min (A) 125 sec
2 post, so that the race ends in a (B) 120 sec
1 dead heat ? (C) 130 sec
(B) 2 min, 2 min
2 (A) 150 m (D) None of these
1 (B) 100 m 34. Rashid can run 880 metres race
(C) 3 min, 3 min
2 (C) 120 m in 2 minutes 24 seconds and
(D) None of these (D) None of these Hamid in 2 minutes 40 seconds.
27. A, B and C are the three contes- How many metres start can
20. A can run a km in 3 minutes 10
tants in one km race. If A can Rashid give Hamid in a 880
secs. and B in 3 mins. 20 secs.
give B a start of 40 metres and A metres race to make a dead
By what distance can A beat B ?
can give C a start of 64 metres. heat ?
(A) 50 metres (B) 40 metres (A) 88 ms (B) 77 ms
How many metres start can B
(C) 45 metres (D) 55 metres give C ? (C) 80 ms (D) 98 ms
21. In a 400 metres race, A gives B a (A) 20 m 35. A can run 440 metres in 51
start of 5 seconds and beats him (B) 25 m seconds and B in 55 seconds. By
by 15 metres. In another race of (C) 35 m how many seconds will B win if
1 (D) None of these he has 40 metres start ?
400 metres. A beats B by 7 (A) 10 sec
7
28. A can give B a start of 20 metres (B) 1 sec
seconds. Find their speeds—
and C a start of 39 metres in a (C) 4 sec
(A) 8 m/sec, 6 m/sec walking race of 400 metres. How (D) Can’t be determined
(B) 9 m/sec, 6 m/sec much can B give C a start ?
(C) 8 m/sec, 7 m/sec 36. A can run 200 metres in 35
(A) 20 m (B) 15 m seconds and B in 38 seconds. By
(D) None of these (C) 18 m (D) 25 m what distance can A beat B ?
22. A can run 330 metres in 41 29. In a flat race, A beats B by 15 11
seconds and B in 44 seconds. By (A) 15 m
metres and C by 29 metres. 19
how many seconds will B win if When B and C run over the 15
he has 30 metres start ? course together, B wins by 15 (B) 15 m
19
(A) 2 sec. (B) 1 sec. metres. Find the length of the 5
(C) 3 sec. (D) 15 sec. course— (C) 15 m
19
23. In a 100 metres race. A runs at a (A) 220 m (B) 325 m
speed of 2 metres per seconds. If (C) 225 m (D) 250 m (D) None of these
A gives B a start of 4 metres and 30. A, B and C are three participants 1
still beats him by 10 seconds, 37. A can run 100 m in 15 and B in
in one km race. If A can give B a 3
find the speed of B— start of 40 metres and B can give 16 seconds. If B receives 4
(A) 1·6 m/sec. C a start of 25 metres, how many metres start, who wins and by
(B) 4 m/sec. metres A can give C a start ? what distance ?
(C) 1 m/sec. (A) 64 m (B) 32 m 1
(A) A wins by m
(D) 2·6 m/sec. (C) 60 m (D) 44 m 6

Quantitative Aptitude Test | 206


1 1 46. A can give B 10 metres and C 20
(B) B wins by m 42. A can beat B by 25 m in a km
6 4 metres in a 100 metres race. B
(C) A wins by 6 m race and B can beat C by 20 can give C 1 second over the
1 course of 100 metres. How long
(D) B wins by 8 m metres in a km race. By how does each take to run 100 metres ?
2
38. A can run 440 m in 1 min 30 much can A beat C in one km (A) 7·2 sec., 8 sec., 9 sec.
seconds and B in 1 min 39 race ? (B) 6·2 sec., 8 sec., 10 sec.
seconds. If B receives 40 metres (A) 130 m (C) 7·2 sec., 9 sec., 10 sec.
start, who wins by what distance ? (B) 126 m (D) Data inadequate
(A) A wins by 4 metres (C) 136 m
(B) B wins by 4 metres 47. A can give B 40 metres and C 80
(D) Data inadequate metres in a 400 metres race. B
(C) A wins by 8 metres
43. In a race of 600 m, A can beat B can give C 4 seconds over the
(D) Dead heat course of 400 metres. How long
by 60 m and in a race of 500 m,
1 B can beat C by 50 m. By how does A take to run 400 metres ?
39. Two boys, A and B run at 4 and
2 many metres will A beat C in a (A) 28 sec. (B) 28·2 sec.
6 km an hour respectively. A race of 400 m ? (C) 28·8 sec. (D) 29 sec.
having 190 metres start, who (A) 70 m
wins and by how much the (B) 76 m 48. A can give B 20 points, A can
course being 1 km ? give C 32 points and B can give
1
(A) B wins by 60 m (C) 77 m C 15 points. How many points
3 make the game ?
(B) A wins by 60 m (D) None of these
(C) A wins by 80 m (A) 1000 (B) 100
44. In a race of 600 m. A can beat B (C) 500 (D) 250
(D) B wins by 80 m by 50 m and in a race of 500 m,
40. In one km race A beats B by 100 B can beat C by 60 m. By how 49. A can give B 20 points in 100
metres and C by 200 metres, by many metres will A beat C in a and B can give C 20 points in
how much can B beat C in a race race of 400 m ? 100. How many in 100 can A
of 1350 metres ? 1 give C ?
(A) 76 m (B) 76 m (A) 26 (B) 36
(A) 150 m (B) 160 m 3
(C) 140 m (D) 135 m 1 (C) 46 (D) 30
(C) 77 m (D) 77 m
41. In a 100 metres race A can beat 3 50. Two persons A and B run a 5 km
B by 10 metres and B can beat C 45. A can give B 40 metres and C 82 race on a round course of 400 m.
by 10 metres. By how much can metres in a 880 metres race If their speed be in the ratio 5 : 4
A beat C in the same race ? while B can give C 9 seconds how often does the winner pass
(A) 10 m over the course. Find the time C the other ?
(B) 12 m takes to run 880 metres— (A) 3 times (B) 1 time
(C) 19 m (A) 1 min. (B) 180 min. 1
(C) 3 min. (D) 60 sec. (C) 2 times (D) 2 times
(D) Can’t be determined 2

Answers with Hints


1. (C) Ratio of races of A and B = 7 : 4 100
= = (100 : 80)
i.e., 3 metres are gained by A in a race of 7 metres 80
So, B can give C 20 metres.
∴ 60 metres are gained by A in a race of ( )7
3
× 60 4. (C) A : B = 100 : 75 and B : C = 100 : 96
A B 100 100 100
= 140 metres ∴ A:C = × = × = = 100 : 72
B C 75 96 72
2. (C) In a 25 metres race B beats A by 5 metres.
So, A beats C by (100 – 72) = 28 metres
In one km race B beats A by ( 5
25
× 1000) 5. (B) 15 metres are covered by B in 5 seconds.

= 200 metres 300 metres are covered by B in (5


15
× 300 )
3. (B) A : B : C = 100 : 90 : 72 = 100 seconds.
∴ Time taken by A = (100 – 5) = 95 seconds

∴ B:C =
90
=
(
90 ×
100
90 ) 6. (A) Distance covered by B in 6 seconds
72
(
72 ×
100
90 ) = ( 1000
300 )
× 6 metres = 20 metres

Quantitative Aptitude Test | 207


Thus, A beats B by 20 metres. 15. (C) Applying the given rule, we have, here A is
So, for a dead heat race A must give B a start of 20 290 300
winner and B is loser =
metres. (1000 – x) 1000
7. (B) Distance covered by B in 3 seconds ⇒ 2900 = 3000 – 3x

= ( 100
30 )
× 3 metres = 10 metres ∴ x =
100
3
1
= 33 metres.
3
∴A beats B by 10 metres. 1
Hence, A can give B a start of 33 metres in a race
8. (C) ˙·˙ A : B : C = 60 : 45 : 40 3
of 1 km.
45 9 9 × 10 90
∴ B:C = = = = 16. (B) A beats B by 10 seconds.
40 8 8 × 10 80
So, if B scores 90, then C scores 80. ∴ Distance covered by B in 200 sec.
∴ B can give C 10 points in a game of 90. = 1000 metres.
9. (B) ˙·˙ A : B : C = 100 : 80 : 72 1000
⇒ Distance covered by B in 10 sec. = × 10
200
80 10 100
∴ B:C = = = = 50 metres.
72 9 90
Thus, if B scores 100, then C scores 90. ∴ A beats B by 50 metres in a race of 1 km.
∴ B can give C 10 points in a game of 100 points. Loser’s time Beat time
17. (D) ˙·˙ =
Winner’s distance Beat distance
10. (
(B) A’s speed = 5 × )5
18
25
m./sec. = m/sec.
18 Since winner’s distance = Length of course
5 × 60 10
˙·˙ Time taken by A to cover 100 metres ⇒ =
L 1
(
= 100 × )
18
25
sec. = 72 sec.

33
3
L = 1000 metres.
⇒ B covers 92 metres in (72 + 8) or 80 sec.
18. (A) Let after time t seconds, B reaches 500 metres,
∴ B’s speed = ( 92 18
80 5
× ) km/hr. then A reaches XA metres.
XA – 160 V ×t
= 4·14 km/hr. ˙·˙ = A
500 VB × t
11. (B) ˙·˙ A : B : C = 200 : 169 : 182
(Since B gives A a start of 160 m)


C
=
182
=
( )
182 ×
350
182
=
350 ⇒
XA – 160
=
2
500 3
B 169
( )
169 ×
350
182
325
∴ XA = 493 metres.
1
So, while C covers 350 metres, B covers 325 metres. 3

12.
∴ C beats B by 25 metres in a race of 350 metres.
(D) A : B : C = 100 : 90 : 87
(
∴ B beats A by 500 – 493
1
3 ) 2
= 6 metres
3
B 90 90 × 2 180 Winner’s time Loser’s time
∴ = = = 19. (B) ˙·˙ =
C 87 87 × 2 174 Loser’s distance Winner’s distance
So, while B covers 180 metres, C covers Since A is the winner and B is the loser
= 174 metres.
∴ B beats C by 6 metres. ⇒
( )
t–
1
2
=
t
13. (C) To reach the winning post A will have to cover a 1000 – (100 + 100) 1000
distance of (500 – 140) = 360 metres. 5
∴ t = minutes
While, A covers 3 metres, B covers 4 metres. 2
1
∴ A covers 360 metres, B covers ( 4
3 )
× 360 = 2 min.
2
= 480 metres. 1
Hence time of A and B are 2 min. and 2 minutes.
So, A reaches the winning post while B remains 20 2
metres behind. 20. (A) We have A is the winner and B is the loser.
∴ A wins by 20 metres. 190 200
˙·˙ = [where x = required distance]
14. (C) In a 25 metres race, B beats A by 5 metres. 1000 – x 1000
⇒ 1000 – x = 950
In a km race B beats A by ( 5
25 )
× 1000
∴ x = 50 metres
= 200 metres. Hence, A will beat 3 by 50 metres in a race of 1 km.

Quantitative Aptitude Test | 208


21. (C) 27. (B) While A covers 1000 metres, B covers (1000 –
22. (B) B runs 330 metres in 44 seconds. 40) or 960 metres and C covers (1000 – 64) or 936
44 metres.
∴ B runs (330 – 30) metres in × 300 secs. Now, when B covers 960 metres, C covers 936
330
metres.
i.e., 40 secs.
∴ When B covers 1000 metres
But A runs 330 metres in 41 seconds.
So, B wins by (41 – 40) seconds, i.e., 1 second. (
C covers
936
960 )
× 1000 = 975 metres
23. (A) Here A is the winner and B is the loser.
So, B can give C a start of (1000 – 975) m
˙·˙ Loser’s time – winner’s time = beat time + start = 25 metres.
time ⇒ B’s time – A’s time = 10 + 0.
28. (A) ˙·˙ (L – x12)x23 = L (x13 – x12)
B’s distance A’s distance
⇒ – = 10 ⇒ (400 – 20)x23 = 400 (39 – 20)
B’s speed A’s speed
(100 – 4) 100 ∴ x23 = 20 metres
⇒ – = 10
B’s speed 2 29. (C) Using (L – x12)x23 = L(x13 – x12)
96 where x12 = A beats B by 15 metres
⇒ = 10 + 50 = 60
B’s speed x23 = B beats C by 15 metres
∴ B’s speed = 1·6 metres/sec. x13 = A beats C by 29 metres
Hence, the speed of B is 1·6 metres/sec. L = Length of course = ?
24. (C) Here P is the winner and Q is the loser. (L – 15)15 = L(29 – 15)
˙·˙
Loser’s time beat time + start time ∴ L = 225 metres
˙·˙ winner’s distance = beat distance + start distance
30. (A) Here A is the winner (Ist).
300 10 + 0 Since B can give C a start therefore B becomes IInd
⇒ =
1000 beat distance + 0 and C becomes IIIrd in the race.
100 ˙·˙ (L – x12)x23 = L(x13 – x12)
⇒ Beat distance = metres.
3 ⇒ (1000 – 40) × 25 = 1000 × (x13 – 40)
1 ⇒ 960 × 25 = 1000 × (x13 – 40)
∴ P beats Q by 33 metres in 1 kilometre race.
3 96 × 25
⇒ (x13 – 40) = = 24
25. (A) P runs 1 kilometre in 4 minutes (= 240 seconds) 100
Q runs a kilometre in 4 minutes 10 sec. (= 250 ⇒ x13 = 64 metres
seconds) Hence, A can give C a start of 64 metres.
∴ P can beat Q by 10 seconds in 1 km. race 31. (A)
But if P gives Q a start of 10 seconds or x metres so 32. (C) Here X becomes Ist, Y becomes IInd and Z
that the race may end in a dead heat, i.e., beat time = becomes IIIrd in the race.
0 and beat distance = 0 (L – x12)x23 = L(x13 – x12)
˙·˙
Loser’s time beat time + start time where,
˙·˙ winner’s distance = beat distance + start distance
Ist (X) gives IInd (Y) a start of x12 = 50 metres
250 0 + 10
⇒ = Ist (X) gives IIIrd (Z) a start of x13 = 69 metres
1000 0+x
IInd (Y) gives IIIrd (Z) a start of x23 = ?
∴ x = 40
Length of race (L) = 1000 metres
Hence if P gives Q a start of 40 metres in a race of ⇒ (1000 – 50)x23 = 1000(69 – 50)
one kilometre the race will end in a dead heat.
∴ x23 = 20 metres
26. (C) Assuming L = distance of the winning post such
that the race ends in a dead heat, i.e., both the Hence Y gives Z a start of 20 metres.
participants A and B reach the winning post at the 33. (A) Time taken by A to complete the course
same time. 5
= (1000 – 40) = 120 seconds
˙·˙ Time taken by A = time taken by B 40
L L – 30 ∴ Time taken by B to run the km = (120 + 5)
⇒ v ∴ L = 120
=
4 = 125 seconds
v
3 34. (A) We have the distance by which Rashid can beat

[ d
]
Since t = v ; where d = distance and v = speed Hamid (
= 880 1 – )144
160
= 88 m

∴ Length of race (distance) of winning post is 120 ∴ Rashid can give Hamid 88 metres start in the race
metres. to make a dead heat.

Quantitative Aptitude Test | 209



35. (B) A can beat B by 440 (1 – 5155) m ∴
x – 40 = 36
x = 40 + 36 = 76 m
= 32 metres 50
But from the question, B has a 40 metres start i.e., B 44. (D) A can beat B by = × 400
600
will beat A by 40 – 32 = 8 metres. 100
55 = m in 400 m race
∴ Required time = × 8 = 1 second. 3
440 60
B can beat C by = × 400
36. (B) A can beat B by 200 1 –( )
35
38
m=
300
19
m 500
= 48 m in 400 m race
15
= 15 m. Let A can beat C in a race of 400 m by x m
19
(x – 48) 100
37. (A) We first calculate the distance by which A will ˙·˙ × 400 =
beat B i.e., (400 – 48) 3
352 88
100 1 –( 46
48 ) =
100 25
24
=
6
1
= 4 metres.
6
⇒ (x – 48) =
4×3 3
=

But from the question, B receives 4 metres start still ∴ x = 48 +


88
(= 48 + 29 )1
m
( 1
A wins by 4 – 4 = m
6 ) 1
6
= 77 m.
1
3 3

38. (D) A can beat B by 440 1 –( )90


99
=
440
11
m
3

= 40 m.
45. (C) Required time = 9 ( )
880 – 40
82 – 40
= 180 sec.

39. (A) Time taken by A to cover 1 km = (2


9 )
× 60 =
40
3 46. (A) Time taken by A
= 3 min.

1
min. and time taken by B to cover 1 km = × 60
= 10 min. B will beat A (if we do not take into
6 =[ (100 – 10) (100 – 20)
(20 – 10) ] 1
100
account the fact that A having 190 metres start) by 72
= = 7·2 sec.
3 × 10
(
1000 1 –
40 ) = 250 metres. Now, we consider
10

the fact that the A is having 190 metres start, Time taken by B = ( )
100 – 20
20 – 10
80
1 = = 8 sec.
10
therefore, B wins the race by (250 – 190) = 60 metres.
40. (A) Required distance Time taken by C = ( )
100 – 10
20 – 10
90
1 = = 9 sec.
10
(200 – 100)
= × 1350 = 150 metres 47. (C) Time taken by A
(1000 – 100)

41. (C) ˙·˙ ( x – 10


100 – 10 ) × 100 = 10
=[ (400 – 40) (400 – 80)
(80 – 40) ] ·
4
400
⇒ (x – 10) = 9 = 28·8 sec.
∴ x = 19 m 48. (B) Let the number of points make the game = x
42. (C) A can beat B by (25 × 4) = 100 metres in one km
race B can beat C by (20 × 2) = 40 metres in one km
˙·˙ x( 32 – 20
)
x – 20
= 15
race. Let the required distance = x metres. ⇒ 12x = 15x – 300
˙·˙( x – 100
1000 – 100 ) × 1000 = 40 ⇒

– 3x = – 300
x = 100 points
⇒ x – 100 = 36
49. (B) Let A can give C x points in 100
∴ Req. distance = x = 100 + 36 = 136 metres

43. (B) A can beat B by


60
600
× 400 ˙·˙ ( x – 20
100 – 20) × 100 = 20

= 40 m in 400 m race ⇒ (x – 20) 5 = 80


50 ⇒ x – 20 = 16
B can beat C by × 400
500 ∴ x = 36
= 40 m in 400 m race 5000
Let A will beat C in a race of 400 m by x m 50. (C) Required answer =
400 × 5
˙·˙ ( x – 40
400 – 40 ) × 400 = 40 5 1
= = 2 = 2 times
2 2

Quantitative Aptitude Test | 210


Odd Man Out and Series
1. Turn odd man out : As the 03. 835, 734, 642, 751, 853, 981, 17. 25, 36, 49, 81, 121, 169, 225 :
phrase speaks itself in this type of 532 : (A) 36 (B) 49
problems, a set of numbers is given (A) 751 (B) 853 (C) 121 (D) 169
in such a way that each one, except (C) 981 (D) 532
one satisfies a particular definite pro- 18. 8, 13, 21, 32, 47, 63, 83 :
perty. The one which does not satisfy 04. 385, 462, 572, 396, 427, 671, (A) 47 (B) 63
that characteristic is to be taken out. 264 : (C) 32 (D) 83
Some important properties of (A) 385 (B) 427
19. 56, 72, 90, 110, 132, 150 :
numbers are given below : (C) 671 (D) 264
(A) 72 (B) 110
(I) Prime numbers : A counting 05. 1, 5, 14, 30, 50, 55, 91 :
number greater than 1, which is (C) 132 (D) 150
(A) 5 (B) 50
divisible by itself and 1 only, is 20. 6, 13, 18, 25, 30, 37, 40 :
called a prime number, e.g., 2, 3, 5, (C) 55 (D) 91
(A) 25 (B) 30
7, 11, 13, 17, 19, 23, 29, 31, 37, 41, 06. 8, 27, 64, 100, 125, 216, 343 : (C) 37 (D) 40
43, 47, 53, 59, 61, 67, 71, 73, 79, 83, (A) 27 (B) 100
89, 97 etc. 21. 5, 16, 6, 16, 7, 16, 9 :
(C) 125 (D) 343
(II) Odd numbers : A number (A) 9
not divisible by 2, is an odd number 07. 1, 4, 9, 16, 20, 36, 49 : (B) 7
e.g., 1, 3, 5, 7, 9, 11, 13, 15 etc. (A) 1 (B) 9 (C) 6
(III) Even numbers : A number (C) 20 (D) 49 (D) None of these
divisible by 2, is an even number
08. 10, 25, 45, 54, 60, 75, 80 : 22. 1, 8, 27, 64, 124, 216, 343 :
e.g., 2, 4, 6, 8, 10 etc.
(A) 10 (B) 45 (A) 8 (B) 27
(IV) Perfect squares : A count-
(C) 54 (D) 75 (C) 64 (D) 124
ing number whose square root is a
counting number, is called a perfect 09. 16, 25, 36, 72, 144, 196, 225 : 23. 46080, 3840, 384, 48, 24, 2, 1 :
square. e.g., 1, 4, 9, 16, 25, 36, 49, 64 (A) 1 (B) 2
(A) 36 (B) 72
etc. (C) 24 (D) 384
(C) 196 (D) 225
(V) Perfect cubes : A counting 24. 582, 605, 588, 611, 634, 617,
number whose cube-root is a count- 10. 41, 43, 47, 53, 61, 71, 73, 81 : 600 :
ing number, is called a perfect cube, (A) 61 (B) 71
e.g., 1, 8, 27, 64, 125 etc. (A) 634 (B) 611
(C) 73 (D) 81 (C) 605 (D) 600
(VI) Multiples of a number : A
number which is divisible by a given 11. 6, 9, 15, 21, 24, 28, 30 : 25. 36, 54, 18, 27, 9, 18·5, 4·5 :
number is called the multiple of a (A) 28 (B) 21 (A) 4·5 (B) 18·5
number e.g., 3, 6, 9, 12 etc. are all (C) 54 (D) 18
(C) 24 (D) 30
multiples of 3.
12. 1, 4, 9, 16, 23, 25, 36 : 26. 22, 33, 66, 99, 121, 279, 594 :
(VII) Numbers in A.P. : Some
given numbers are said to be in A.P. (A) 9 (B) 23 (A) 33 (B) 121
If the difference between two (C) 25 (D) 36 (C) 279 (D) 594
consecutive numbers is same e.g., 13, 27. 4, 5, 7, 10, 14, 18, 25, 32 :
11, 9, 7, 5, 3, 1, – 1 – 3 etc. 13. 3, 5, 9, 11, 14, 17, 21 :
(A) 7 (B) 14
(VIII) Numbers in G.P. : Some (A) 21 (B) 17
(C) 18 (D) 32
given numbers are in G.P. If the ratio (C) 14 (D) 9
between two consecutive numbers 28. 2, 5, 10, 50, 500, 5000 :
remains the same e.g., 48, 12, 3 etc. 14. 10, 14, 16, 18, 21, 24, 26 : (A) 0 (B) 5
(A) 26 (B) 24 (C) 10 (D) 5000
Exercise (C) 21 (D) 18
29. 253, 136, 352, 460, 324, 631,
01. 2, 5, 10, 17, 26, 37, 50, 64 : 15. 3, 5, 7, 12, 13, 17, 19 : 244 :
(A) 50 (B) 26 (A) 19 (B) 17 (A) 136 (B) 324
(C) 37 (D) 64 (C) 352 (D) 631
(C) 13 (D) 12
02. 331, 482, 551, 263, 383, 242,
16. 1, 2, 6, 15, 31, 56, 91 : 30. 19, 28, 39, 52, 67, 84, 102 :
111 :
(A) 263 (B) 383 (A) 31 (B) 91 (A) 52 (B) 102
(C) 242 (D) 111 (C) 56 (D) 15 (C) 84 (D) 67

Quantitative Aptitude Test | 211


31. 11, 13, 17, 19, 23, 29, 31, 37, 41, 46. Find the wrong number in the 61. 196, 169, 144, 121, 100, 80, 64 :
(…) : series 3, 8, 15, 24, 34, 48, 63 : (A) 169 (B) 144
(A) 43 (B) 47 (A) 15 (B) 24 (C) 121 (D) 100
(C) 53 (D) 51 (C) 34 (D) 48 (E) 80
32. 1, 8, 27, 64, 125, 216, (…) : 47. Complete the series 2, 5, 9, 19, 62. 1, 3, 10, 21, 64, 129, 356, 777 :
(A) 354 (B) 343 37 … : (A) 10 (B) 21
(C) 392 (D) 245 (A) 76 (C) 64 (D) 129
(B) 74 (E) 356
33. 1, 4, 9, 16, 25, 36, 49, (…) :
(C) 75
(A) 54 (B) 56 63. 3, 7, 15, 39, 63, 127, 255, 511 :
(D) None of these
(C) 64 (D) 81 (A) 7 (B) 15
48. 9, 12, 11, 14, 13, (…), 15 : (C) 39 (D) 63
34. 5, 10, 13, 26, 29, 58, 61, (…) :
(A) 12 (B) 16 (E) 127
(A) 122 (B) 64
(C) 10 (D) 17
(C) 125 (D) 128 64. 445, 221, 109, 46, 25, 11, 4 :
49. 71, 76, 69, 74, 67, 72, (…) : (A) 221 (B) 109
35. 4 – 8, 16 – 32, 64 — (…) :
(A) 77 (B) 65 (C) 46 (D) 25
(A) 128 (B) – 128
(C) 80 (D) 76 (E) 11
(C) 192 (D) – 192
50. 1, 2, 4, 8, 16, 32, 64, (…), 256 : 65. 19, 26, 33, 46, 59, 74, 91 :
36. 8, 27, 125, 343, 1331 :
(A) 148 (B) 128 (A) 26 (B) 33
(A) 1331
(C) 154 (D) 164 (C) 46 (D) 59
(B) 343
(C) 125 51. 10, 5, 13, 10, 16, 20, 19, (…) : (E) 74
(D) None of these (A) 22 (B) 40 66. 2880, 480, 92, 24, 8, 4, 2 :
(C) 38 (D) 23 (A) 2880 (B) 480
37. 10, 14, 28, 32, 64, 68, 132 :
(A) 32 (B) 68 52. 8, 7, 11, 12, 14, 17, 17, 22, (…) : (C) 92 (D) 24
(C) 132 (D) 28 (A) 27 (B) 20 (E) 4
(C) 22 (D) 24 67. 3, 7, 15, 27, 63, 127, 255 :
38. 4, 9, 19, 39, 79, 160, 319 :
(A) 319 (B) 160 53. 2, 4, 12, 48, 240, (…) : (A) 7 (B) 15
(C) 79 (D) 39 (A) 960 (B) 1440 (C) 27 (D) 63
(C) 1080 (D) 1920 (E) 127
39. 3, 10, 21, 36, 55, 70, 105 :
(A) 105 (B) 70 54. 7, 26, 63, 124, 215, 342, (…) : 68. 7, 8, 18, 57, 228, 1165, 6996 :
(C) 36 (D) 55 (A) 481 (B) 511 (A) 8 (B) 18
(C) 391 (D) 421 (C) 57 (D) 228
40. 125, 123, 120, 115, 108, 100,
84 : 55. 165, 195, 255, 285, 345, (…) : (E) 127
(A) 123 (B) 115 (A) 375 (B) 420 69. 64, 71, 80, 91, 104, 119, 135,
(C) 100 (D) 84 (C) 435 (D) 390 155 :
41. 16, 36, 64, 81, 100, 144, 190 : 56. 8, 24, 12, 36, 18, 54, (…) : (A) 71 (B) 80
(A) 81 (B) 100 (A) 27 (B) 108 (C) 104 (D) 119
(C) 190 (D) 36 (C) 68 (D) 72 (E) 135

42. 125, 127, 130, 135, 142, 153, 57. 2, 6, 12, 20, 30, 42, 56, (…) : 70. 40960, 10240, 2560, 640, 200,
165 : 40, 10 :
(A) 60 (B) 64
(A) 130 (B) 142 (A) 640 (B) 40
(C) 72 (D) 70
(C) 153 (D) 165 (C) 200 (D) 2560
58. 15, 31, 63, 127, 255, (…) : (E) 10240
43. 4, 6, 8, 9, 10, 11, 12 : (A) 513 (B) 511
(A) 10 (B) 11 71. 1, 1, 2, 6, 24, 96, 720 :
(C) 517 (D) 523 (A) 720 (B) 96
(C) 12 (D) 9
59. 3, 7, 6, 5, 9, 3, 12, 1, 15, (…) : (C) 24 (D) 6
44. 105, 85, 60, 30, 0, – 45, – 90 : (E) 2
(A) 18 (B) 13
(A) 0 (B) 85
(C) – 1 (D) 3 72. 190, 166, 145, 128, 112, 100,
(C) – 45 (D) 60 91 :
45. 52, 51, 48, 43, 34, 27, 16 : 60. 16, 33, 65, 131, 261, (…) : (A) 100 (B) 166
(A) 27 (B) 34 (A) 523 (B) 521 (C) 145 (D) 128
(C) 43 (D) 48 (C) 613 (D) 721 (E) 112

Quantitative Aptitude Test | 212


73. Find out the wrong number in 74. Find out the wrong number in 75. Find out the wrong number in
the series 125, 106, 88, 76, 65, the series 5, 15, 30, 135, 405, the series 2, 9, 28, 65, 126, 216,
58, 53. 1215, 3645. 344 :
(A) 125 (B) 106 (A) 3645 (B) 1215 (A) 2 (B) 28
(C) 88 (D) 76 (C) 405 (D) 30 (C) 65 (D) 126
(E) 65 (E) 15 (E) 216

Answers with Hints


01. (D) The pattern is x 2 + 1, where x = 1, 2, 3, 4, 5, 6, 7, 25. (B) The terms are alternately multiplied by 1·5 and
8 etc. But 64 is out of pattern. divided by 3. However 18·5 does not satisfy it.
02. (B) In each number except 383, the product of first 26. (C) Each number except 279 is a multiple of 11.
and third digit is the middle one. 27. (C) 2nd = (1st + 1) : 3rd = (2nd + 2); 4th = (3rd + 3);
03. (A) In each number except 751, the difference of 5th = (4th + 4). But 18 = 6th ≠ 5th + 5 = 14 + 5 = 19.
third and first digit is the middle one. 28. (D) Pattern is 1st × 2nd = 3rd; 2nd × 3rd = 4th,
04. (B) In each number except 427, the middle digit is 3rd × 4th = 5th. But 4th × 5th = 50 × 500 = 25000
sum of the other two. ≠ 5000 = 6th.
05. (B) The pattern is 12 , 12 + 22, 12 + 22 + 32, 12 + 22 + 29. (B) Sum of the digits in each number, except 324 is
32 + 42 , 12 + 22 + 32 + 42 + 52 , 12 + 22 + 32 + 42 + 52 10.
+ 62 . But 50 is not of this pattern. 30. (B) The pattern is x 2 + 3, where x = 4, 5, 6, 7, 8, 9
06. (B) The pattern is 23 , 33 , 43 , 53 , 63 , 73 . But 100 is not etc. But 102 is out of pattern.
a perfect cube. 31. (A) Numbers are all primes. The next prime is 43.
07. (C) The pattern is 12 , 2 2 , 3 2 , 4 2 , 5 2 , 6 2 , 7 2 . But, 32. (B) Numbers are 13 , 23 , 33 , 43 , 53 , 63 . So, the missing
instead of 5 2 , it is 20, which is to be turned out. number is 73 = 343.
08. (C) Each of the numbers except 54, is a multiple of 5. 33. (C) Numbers are 12 , 22 , 32 , 42 , 52 , 62 , 72 . So, the next
09. (B) Each of the numbers except 72, is a perfect number is 82 = 64.
square. 34. (A) Numbers are alternately multiplied by 2 and
10. (D) Each of the numbers except 81, is a prime increased by 3. So, the missing number = 61 × 2 =
number. 122.
11. (A) Each of the numbers except 28, is a multiple of 3. 35. (B) Each number is the preceding number multiplied
by – 2. So, the required number is – 128.
12. (B) Each of the given numbers except 23, is a perfect
36. (D) The numbers are cubes of primes i.e., 23 , 3 3 , 5 3 ,
square.
73 , 113 . Clearly, none is wrong.
13. (C) Each of the numbers except 14, is an odd number.
37. (C) Alternately, we add 4 and double the next. So,
14. (C) Each of the numbers except 21, is an even 132 is wrong. It must be (68 × 2) i.e., 136.
number.
38. (B) Double the number and add 1 to it, to get the
15. (D) Each of the numbers except 12, is a prime next number. So, 160 is wrong.
number. 39. (B) The pattern is 1 × 3, 2 × 5, 3 × 7, 4 × 9, 5 × 11, 6
16. (B) Add 1 2 , 22 , 32 , 42 , 52 , 62 . So, 91 is wrong. × 13, 7 × 15 etc.
17. (A) The numbers are squares of odd natural numbers, 40. (C) Prime numbers 2, 3, 5, 7, 11, 13 have
starting from 5 up to 15. So, 36 is wrong. successively been subtracted. So, 100 is wrong. It
18. (A) Go on adding 5, 8, 11, 14, 17, 20. So, the number must be (108 – 11) i.e., 97.
47 is wrong and must be replaced by 46. 41. (C) Each number is the square of a composite
19. (D) The numbers are 7 × 8, 8 × 9, 9 × 10, 10 × 11, number except 190.
11 × 12, 12 × 13. So, 150 is wrong. 42. (D) Prime numbers 2, 3, 5, 7, 11, 13 are to be added
20. (D) The difference between two successive terms successively. So, 165 is wrong.
from the beginning are 7, 5, 7, 5, 7, 5. So, 40 is 43. (B) Each number is a composite number except 11.
wrong. 44. (A) Subtract 20, 25, 30, 35, 40, 45 from successive
21. (A) Terms at odd places are 5, 6, 7, 8 etc. and each numbers. So 0 is wrong.
term at even place is 16. So, 9 is wrong. 45. (B) Subtract 1, 3, 5, 7, 9, 11 from successive
22. (D) The numbers are 13 , 23 , 33 , 43 etc. So, 124 is numbers. So, 34 is wrong.
wrong. It must have been 53 i.e., 125. 46. (C) The difference between consecutive terms are
23. (C) The terms are successively divided by 12, 10, 8, respectively. 5, 7, 9, 11 and 13. So, 34 is a wrong
6 … etc. So, 24 is wrong. number.
24. (A) Alternately 23 is added and 17 is subtracted from 47. (C) Second number is one more than twice the first
the terms. So, 634 is wrong. third number is one less than twice the second, fourth

Quantitative Aptitude Test | 213


number is one more than twice the third, fifth 62. (E) A × 2 + 1, B × 3 + 1, C × 2 + 1, D × 3 + 1 and so
number is one less than the fourth. Therefore, the on ∴ 356 is wrong.
sixth number is one more than twice the fifth. So, the 63. (C) Go on multiplying 2 and adding 1 to get the next
missing number is 75. number. So, 39 is wrong.
48. (B) Alternately, we add 3 and subtract 1. 64. (C) Go on subtracting 3 and dividing the result by 2
49. (B) Alternately, we add 5 and subtract 7. to obtain the next number. Clearly, 46 is wrong.
50. (B) Each previous number is multiplied by 2. 65. (B) Go on adding 7, 9, 11, 13, 15, 17 respectively to
51. (B) There are two series (10, 13, 16, 19) and (5, 10, obtain the next number. So, 33 is wrong.
20, 40) one increasing by 3 and another multiplied by 66. (C) Go on dividing by 6, 5, 4, 3, 2, 1 respectively to
2. obtain the next number. Clearly, 92 is wrong.
52. (B) There are two series (8, 11, 14, 17, 20) and (7, 67. (C) Go on multiplying the number by 2 and adding 1
12, 17, 22). Increasing by 3 and 5 respectively. to it to get the next number. So, 27 is wrong.
53. (B) Go on multiplying the given numbers by 2, 3, 4, 68. (D) Let the given numbers be A, B, C, D, E, F, G.
5, 6. So, the correct next number is 1440. Then A × 1 + 1, B × 2 + 2, C × 3 + 3, D × 4 + 4, E
54. (B) Numbers are (23 – 1), (33 – 1), (43 – 1) (53 – 1), × 5 + 5, F × 6 + 6 are the required numbers. Clearly,
(63 – 1) (73 – 1) etc. So, the next number is (8 3 – 1) = 228 is wrong.
(512 – 1) = 511. 69. (E) Go on adding 7, 9, 11, 13, 15, 17, 19 respectively
55. (C) Each number is 15 multiplied by a prime number to obtain the next number. So, 135 is wrong.
i.e., 15 × 11, 15 × 13, 15 × 17, 15 × 19, 15 × 23. So, 70. (C) Go on dividing by 4 to get the next number. So,
the next number is 15 × 29 = 435. 200 is wrong.
56. (A) Numbers are alternately multiplied by 3 and 71. (B) Go on multiplying with 1, 2, 3, 4, 5, 6 to get the
divided by 2. So, next number = 54 ÷ 2 = 27. next number. So, 96 is wrong.
57. (C) The pattern is 1 × 2, 2 × 3, 3 × 4, 4 × 5, 5 × 6, 72. (D) Go on subtracting 24, 21, 18, 15, 12, 9 from the
6 × 7, 7 × 8. So, the next number is 8 × 9 = 72. numbers to get the next number. Clearly, 128 is
58. (B) Each number is double the preceding one plus 1. wrong.
So, the next number is (255 × 2) + 1 = 511. 73. (C) Go on subtracting prime numbers 19, 17, 13, 11,
59. (C) There are two series, beginning respectively with 7, 5 from the numbers to get the next number. So, 88
3 and 7. In one 3 is added and in another 2 is is wrong.
subtracted. The next number is 1 – 2 = – 1. 74. (D) Multiply each term by 3 to obtain the next term.
Hence, 30 is a wrong number.
60. (A) Each number is twice the preceding one with 1
added or subtracted alternately. So, the next number 75. (E) 2 = (13 + 1); 9 = (23 + 1); 28 = (33 + 1);
is (2 × 261 + 1) = 523. 65 = (43 + 1); 126 = (53 + 1); 216 ≠ (63 + 1) and
61. (E) Numbers must be (14)2, (13) 2 , (11) 2 , (10) 2 , (9) 2 , 344 = (73 + 1)
(8) 2 . So, 80 is wrong. ∴ 216 is a wrong number.

Quantitative Aptitude Test | 214


Calendar
Under this heading we mainly Sol. In order to prove the Odd days are as under :
deal with finding the day of the week required result, we have to show that Year 1990 1991 1992 1993 1994 1995
on a particular given date. The the total number of odd days between Odd 1 1 2 1 1 1
process of finding it lies in obtaining last day of February and last day of days
the number of odd days. October is zero. Year 1996 1997 1998 1999 2000 (leap)
Odd 2 1 1 1 2
The number of days more than Number of days between these day
the complete number of weeks in a dates are : ∴ Total number of odd days =
given period are called odd days. March, April, May, June, July, 14 days i.e., 0 odd day. Hence, the
LEAP AND ORDINARY Aug, Sept, Oct result follows.
YEAR. Every year which is divisible 31 + 30 + 31 + 30 + 31 + 31 +
by 4 such as 1992 is called a leap 30 + 31 = 245 days = 35 weeks = 0 Exercise
year. Every 4th century is a leap year odd day. Hence the result follows.
but no other century is a leap year 01. Smt. Indira Gandhi died on 31st
viz, 400, 800, 1200, 1600 are all leap Q. 2. Prove that the last day of October, 1984. The day of the
years, but none of 700, 900, 1100 etc. a century cannot be either Tues- week was :
is a leap year. day, Thursday or Saturday. (A) Monday
An ordinary year has 365 days Sol. Ist century, i.e., 100 years (B) Tuesday
i.e., (52 weeks + 1 day). contain 76 ordinary years and 24 leap (C) Wednesday
years and therefore, (76 + 48) or 124 (D) Friday
A leap year has 366 days i.e., (52 odd days or 5 odd days.
weeks + 2 days). 02. Today is Friday. After 62 days it
∴ The last day of Ist century is will be :
An ordinary year has 1 odd day ‘Friday’.
and a leap year has 2 odd days. (A) Friday (B) Thursday
Two centuries i.e., 200 years (C) Saturday (D) Monday
A century i.e., 100 years has 76 contains 152 ordinary years and 48
ordinary years and 24 leap years. leap years and therefore (152 + 96) or 03. Monday falls on 4th April, 1988.
248 or 3 odd days. What was the day of 3rd
∴ 100 years = 76 ordinary year November, 1987 ?
+ 24 leap years. ∴ The last day of 2nd century is
‘Wednesday’. (A) Monday (B) Sunday
= (76 × 52) weeks + 76 days (C) Tuesday (D) Wednesday
Three centuries i.e., 300 years
+ (24 × 52) weeks + 48 days
contain 228 ordinary years and 72 04. On July, 2, 1985, it was Wednes-
= (5217 weeks + 5 days) leap years and therefore (228 + 144) day the day of the week on July
= 5 odd days or 372 or 1 odd day. 2, 1984 was :
i.e., ∴ The last day of third century (A) Wednesday
is ‘Monday’. (B) Tuesday
100 years contain 5 odd days.
Four centuries i.e., 400 years (C) Monday
200 years contain 10 and
therefore 3 odd days. contain 303 ordinary years and 97 (D) Thursday
leap years and therefore, (303 + 194)
300 years contain 15 and or 497 or 0 odd day. 05. On January 12, 1980, it was
therefore 1 odd day. Saturday. The day of the week
∴ The last day of 4th century is
400 years contain (20 + 1) and on January 12, 1979 was :
‘Sunday’.
therefore 0 odd day. (A) Saturday (B) Friday
Since the order is continually
Similarly, the years 800, 1200, kept in successive cycles, we see that (C) Sunday (D) Thursday
1600 etc. contain no odd day. the last day of a century cannot be 06. January 1, 1992 was a Wednes-
We count days according to Tuesday, Thursday or Saturday. day, what day of the week will it
number of odd days. Sunday for 0 Q. 3. Prove that the calendar be on January 1, 1993 ?
odd day. Monday for 1 odd day and for 1990 will serve for 2001 also. (A) Monday (B) Tuesday
so on.
Sol. In order that the calendar for (C) Sunday (D) Friday
Examples 1934 and 1945 be the same, Ist
07. P. V. Narsimha Rao was elected
January of both the years must be on
party leader on 29th May, 1991.
Q. 1. Prove that any date in the same day of the week. For this,
What was the day of the week ?
March is the same day of the week the total number of odd days between
as the corresponding date in 31st Dec. 1933 and 31st Dec. 1944 (A) Tuesday
November of that year. must be zero. (B) Friday

Quantitative Aptitude Test | 215


(C) Wednesday leap year. The day of the week 13. The number of odd days in a
(D) Sunday on this day after 3 years will be : leap year is :
08. The first republic day of India (A) Wednesday
(B) Thursday (A) 1 (B) 2
was celebrated on 26th January,
1950. It was : (C) Friday (C) 3 (D) 4
(A) Monday (B) Tuesday (D) Saturday
(C) Thursday (D) Friday 14. January 16, 1997 was a
11. The year next to 1991 having the
Thursday. What day of the week
09. How many days are there from same calendar as that of 1990 is :
will it be on January 4, 2000 ?
2nd January 1993 to 15th March (A) 1998 (B) 2001
1993 ? (C) 2002 (D) 2003 (A) Tuesday
(A) 72 (B) 73 12. The year next to 1990 having the (B) Thursday
(C) 74 (D) 71 same calendar as that of 1988 is :
(C) Wednesday
10. Today is Ist August. The day of (A) 1990 (B) 1992
the week is Monday. This is a (C) 1993 (D) 1995 (D) Friday

Answers with Hints


01. (C) 1600 years contains 0 odd day; 300 years contain 08. (C) 1600 years have 0 odd day and 300 years have 1
1 odd day. Also, 83 years contains 20 leap years and odd day. 49 years contain 12 leap years and 37
63 ordinary years and therefore, (40 + 0) odd days ordinary years and therefore (24 + 37) odd days i.e.,
i.e., 5 odd days. 5 odd days i.e., 1949 years contain (0 + 1 + 5) = 6
∴ 1983 years contain (0 + 1 + 5) i.e., 6 odd days odd days. 26 days of January contain 5 odd days.
Number of days from Jan. 1984 to 31st Oct 1984 Total odd days = (6 + 5) = 11 or 4 odd days.
So, the day was Thursday
= (31 + 29 + 31 + 30 + 31 + 30 + 31 + 31 + 30 + 31)
09. (B) Jan., Feb., March
= 305 days
30 + 28 + 15 = 73 days
= 4 odd days
10. (B) This being a leap year none of the next 3 years is
∴ Total number of odd days = 6 + 4 = 3 odd days a leap year. So, the day of the week will be 3 days
So, 31st Oct 1984 was Wednesday beyond Monday i.e., it will be Thursday.
02. (B) Each day of the week is repeated after 7 days 11. (C) We go on counting the odd days from 1991
∴ After 63 days, it would be Friday onwards till the sum is divisible by 7. The number of
such days are 14 upto the year 2001.
So, After 62 days, it would be Thursday
So, the calendar for 1991 will be repeated in the year
03. (C) Counting the number of days after 3rd 2002.
November, 1987 we have
12. (C) Starting with 1988, we go on counting the
Nov., Dec., Jan., Feb., March, April number of odd days till the sum is divisible by 7
days = 27 + 31 + 31 + 29 + 31 + 4 = 153 days Year 1988 1989 1990 1991 1992
containing 6 odd days i.e., (7 – 6) = 1 day beyond the Odd days 2 1 1 1 2
day on 4th April, 1988 So, the day was Tuesday.
= 7; i.e., odd days.
04. (C) The year 1984 being a leap year, it has 2 odd
days. So, the day on 2nd July, 1985 is two days ∴ Calendar for 1993 is the same as that of 1988.
beyond the day on 2nd July, 1984. 13. (B) A leap year has (52 weeks + 2 days).
But, 2nd July 1985 was Wednesday So, the number of odd days in a leap year is 2
∴ 2nd July, 1984 was Monday 14. (A) First we look for the leap years during this period
1997, 1998, 1999 are not leap years
05. (B) The year 1979 being an ordinary year, it has 1
odd day. So, the day on 12th January 1980, is one 1998 and 1999 together have net 2 odd days
day beyond the day on 12th January, 1979 No. of days remaining in 1997 = 365 – 16 = 349 days
But January 12, 1980 being Saturday = 49 weeks 6 odd days
∴ January 12, 1979 was Friday. January 4, 2000 gives 4 odd days
06. (D) 1992 being a leap year, it has 2 odd days. So, the ∴ Total no. of odd days = 2 + 6 + 4 = 12 days = 7
first day of the year 1993 will be two days beyond days (1 week) + 5 odd days
Wednesday i.e., it will be Friday.
Hence, January 4, 2000 will be 5 days beyond
07. (C) Try yourself. It was Wednesday. Thursday i.e., it will be on Tuesday.

Quantitative Aptitude Test | 216


Chain Rule
The method of finding the fourth ⎧⎪more work‚ more men (Direct) Solution : Since the length is to
proportional when the other three are ⎪more day less men (Indirect) be found out, we compare each item
given is called simple proportion or ˙·˙ ⎨⎪ more hours per day‚ with the length as shown below :
rule of three. Repeated use of the rule ⎪⎩less man (Indirect)
More men, more length built
of three is called compound pro-
2 3⎫ (Direct proportion)
portion. ⇒ Work : ⎪
5 5⎪⎬ : : 104 : x Less days, less length built (Di-
Direct proportion—Two quan- Days 26 : 30 ⎪ rect proportion)
tities are said to directly proportional Hours 9 : 8 ⎭

if on the increase or decrease of the
one, the other increases or decreases ∴ x=
3 × 30 × 8 × 104 × 5
= 160
˙·˙ Men 20 : 25
Days 6 : 3 } : : 112 : x
the same extent. 5 × 2 × 26 × 9
⇒ 20 × 6 × x
Ex. (i) Cost of articles is directly No. of additional men
proportional to number of articles i.e. = 160 – 104 = 56 = 25 × 3 × 112
more articles, more cost and less Q. 2. If 8 men, working 9 hours 25 × 3 × 112
articles, less cost. a day can build a wall 18 metres ∴ x =
20 × 6
(ii) The work done is directly long 2 metres broad and 12 metres
high in 10 days. How many men = 70 metres.
proportional to the number of men
employed to do the work i.e., more will be required to build a wall 32 Q. 4. If 36 men can do a cer-
metres long, 3 metres broad and 9 tain piece of work in 25 days. In
men, more work and less men, less
metres high, by working 6 hours a how many days will 15 men do it ?
work.
day in 8 days ?
Indirect proportion—Two Solution : Clearly, less is the
Solution : Since the number of number of men employed, more will
quantities are said to be indirectly men is to be found out, we compare be the number of days taken to finish
proportional if on the increase of the each item with the number of men, as the work. So, inverse ratio of men is
one, the other decrease to the same shown below— equal to ratio of times taken.
extent and vice-versa.
More length, more men required
Ex. (i) Less number of days Let the required number of days
(Direct proportion)
required to finish a work, more be x
persons are to be employed. More breadth, more men requi-
red (Direct proportion) Then, 15 : 36 : : 25 : x
(ii) The time taken to cover a More height, more men required 15 25
distance is inversely proportional to (Direct proportion) ⇒ = x
the speed of the car, i.e. more speed, 36
less is the time taken. Less daily working hrs. more 36 × 25
men required (Indirect proportion) ∴ x = = 60
15
Less day to finish work, more
Examples men required (Indirect proportion)
∴ Required number of days
= 60
Q. 1. A contract is to be
completed in 56 days and 104 men
Length 18 : 32 ⎫⎪ Q. 5. If 15 dolls cost Rs. 35,
˙·˙ Breadth 2 : 3
were set to work, each working 8 Height 12 : 9 ⎬::8:x what do 39 dolls cost ?
2 Daily hrs. 6 : 9 ⎪ Solution : Clearly more dolls
hours a day. After 30 days of the
5 Days 8 : 10 ⎭ more cost. (Direct proportion)
work is completed. How many
⇒ 18 × 2 × 12 × 6 × 8 × x So, ratio of dolls is the same as
additional men may be employed,
so that the work may be completed = 32 × 3 × 9 × 9 × 10 × 8 ratio of costs.
in time, each man now working 9 32 × 3 × 9 × 9 × 10 × 8 Now, let the costs of 39 dolls be
hours a day ? ∴ x= Rs. x.
18 × 2 × 12 × 6 × 8
Solution : Remaining work = 30 men. Then, 15 : 39 : : 35 : x
∴ Required number of men
= (1 – 25) = 35 = 30 ⇒
15
39
35
= x
Remaining period Q. 3. If 20 men can build a wall 35 × 39
112 metres long in 6 days, what ∴ x =
= (56 – 30) 15
length of a similar wall can be built
= 26 days by 25 men in 3 days ? = Rs. 91

Quantitative Aptitude Test | 217


Exercise working 7 hours a day. In how 14. If 18 binders bind 900 books in
many days will 16 pumbs raise 10 days, how many binders will
1. If x men working x hours per day 1736 tonnes, working 9 hour a be required to bind 660 books in
can do x units of a work in x day ? 12 days ?
days, then y men working y (A) 9 days (B) 8 days (A) 55 (B) 14
hours per day would be able to (C) 7 days (D) 6 days (C) 13 (D) 11
complete in y days—
8. If 4 examiners can examine a 15. If the rent for grazing 40 cows
x2 certain number of answer books
(A) 3 units of work for 20 days is Rs. 370, how
y in 8 days by working 5 hours a many cows can graze for 30 days
x3 day. For how many hours a day on Rs. 111 ?
(B) 2 units of work would 2 examiners have to work
y (A) 6 (B) 8
in order to examine twice the
y2 (C) 5 (D) 12
(C) 3 units of work number of answer books in 20
x days ? 16. In a hospital there is a consump-
y3 (A) 6 hours (B) 8 hours tion of 1350 litres of milk for 70
(D) 2 units of work patients for 30 days. How many
x 1
(C) 9 hours (D) 7 hours patients will consume 1710 litres
2. If 17 labourers can dig a ditch 26 2 of milk in 28 days ?
metres long in 18 days working 9. If 20 men working 7 hours a day (A) 59 (B) 85
8 hours a day. How many can do a piece of work in 10 (C) 95 (D) 105
labourers should be engaged to days. In how many days will 15
dig a similar ditch 39 metres men working for 8 hours a day 17. If 40 persons consume 60 kg of
long in 6 days, each labourer to the same piece of work ? rice in 15 days, then in how
working 9 hours a day ? 5 many days will 30 persons con-
(A) 51 (B) 68 (A) 15 days sume 12 kg of rice ?
21
(C) 85 (D) 34 3
2 (A) 3 days (B) 4 days
(B) 11 days 4
3. 20 men complete one-third of a 3
piece of work in 20 days. How 1
9 (C) 6 days (D) 9 days
many more men should be (C) 6 days 4
16
employed to finish the rest of the 1 18. On a scale of a map 0·8 cm
work in 25 more days ? (D) 4 days represents 8·8 km. If the distance
5
(A) 10 (B) 12 between two points on the map
10. If 300 men can do a piece of is 80·5 cm., the distance between
(C) 15 (D) 20
work in 16 days. How many men these two points is approxima-
4. A garrison had provisions for a would do (1/5) of the work in 15 tely—
certain number of days. After 10 days ?
days (1/5)th of the men desert (A) 9 km (B) 70 km
(A) 56 (B) 64
and it is found that the provi- (C) 90 km (D) 885 km
(C) 60 (D) 72
sions will now last just as long 19. If 22·5 metres of a uniform iron
as before. How long was that ? 11. If 20 men can build a wall 112
rod weighs 85·5 kg. What will be
(A) 35 days (B) 15 days metres long in 6 days, what
the weight of 6 metres of the
length of a similar wall can be
(C) 25 days (D) 50 days same rod ?
built by 25 men in 3 days ?
5. A garrison of 500 men had (A) 140 metres (A) 22·8 kg
provisions for 24 days. However (B) 44·8 metres (B) 25·6 kg
a reinforcement of 300 men arri- (C) 28 kg
(C) 105 metres
ved. The food will now last for—
(D) 70 metres (D) None of these
1
(A) 18 days (B) 17 days 20. 16 men can reap a field in 30
2 12. If 3 persons weave 168 shawls in
14 days, how many shawls will 8 days. In how many days will 20
(C) 16 days (D) 15 days
persons weave in 5 days ? men reap the field ?
6. 120 men had provisions for 200 (A) 25 days (B) 24 days
(A) 90 (B) 105
days. After 5 days, 30 men died
due to an epidemic. The remain- (C) 126 (D) 160 2 1
(C) 10 days (D) 37 days
ing food will last for— 3 2
13. If six men working 8 hours a day
1 earn Rs. 840 per week then 9 21. If 21 cows eat that much as 15
(A) 150 days (B) 146 days
4 men working 6 hours a day will buffaloes, how many cows will
(C) 245 days (D) 260 days earn per week— eat that much as 35 buffaloes ?
7. If 18 pumps can raise 2170 (A) Rs. 840 (B) Rs. 945 (A) 49
tonnes of water in 10 days, (C) Rs. 1620 (D) Rs. 1680 (B) 56

Quantitative Aptitude Test | 218


(C) 45 28. If 3 men or 6 boys can do a piece 33. A contract is to be completed in
(D) None of these of work in 10 days, working 7 56 days and 104 men were set to
hours a day, how many days will work each working 8 hours a
22. Ten pipes through which water it take to complete a work twice 2
flows at the same rate can fill a day. After 30 days of the work
as large with 6 men and 2 boys 5
tank in 24 minutes. If two pipes working together for 8 hours a is completed. How many addi-
go out of order, how long will day ? tional men may be employed, so
the remaining pipes take to fill
1 1 that the work may be completed
the tank ? (A) 7 days (B) 8 days
2 2 in time each man now working 9
(A) 40 minutes hours a day ?
(C) 9 days (D) 6 days
(B) 45 minutes (A) 60 (B) 56
1 29. If 5 engines consume 6 metric
(C) 19 minutes tonnes of coal when each is (C) 70 (D) 42
5
running 9 hours a day. How
(D) 30 minutes 34. A contractor undertakes to do a
much coal will be needed for 8
piece of work in 40 days. He
23. If (4/5)th of a cistern is filled in engines each running 10 hours a
engages 100 men at the beginn-
1 minute, how much more time day, it being given that 3 engines
ing and 100 more after 35 days
will be required to fill the rest of o f the former type consume as
and completes the work in
it ? much as 4 engines of latter type ?
stipulated time. If he had not
(A) 20 seconds (A) 8 metric tonnes engaged the additional men, how
(B) 15 seconds 8 many days behind, schedule
(B) 8 metric tonnes
(C) 12 seconds 9 would it be finished ?
(D) 22 seconds 1 (A) 5 (B) 6
(C) 3 metric tonnes
24. If Raghu can walk a distance of 8 (C) 3 (D) 9
5 kms in 20 minutes, how long (D) 6·48 metric tonnes
he can go in 50 minutes ? 35. A contractor undertook to do a
1 certain piece of work in 9 days.
(A) 10·5 km (B) 12 km 30. If 9 men working 7 hours a
2 He employed certain number of
(C) 12·5 km (D) 13·5 km day can finish a work in 20 days, labourers but 6 of them being
25. A rope makes 140 rounds of the then how many days will be absent from the very first day.
circumference of a cylinder, taken by 12 men, working 6 The rest could finish the work in
whose radius of the base is 14 hours a day to finish the work, it 15 days. The number of men
cms. How many times can it go being given that 3 men of latter originally employed were—
round a cylinder with radius 20 type work as much as 2 men of (A) 12 (B) 15
cms ? the former type in the same
time ? (C) 18 (D) 24
(A) 98
(B) 17 1 36. If 12 boys can earn Rs. 240 in 5
(A) 12 (B) 13
2 days. How many boys can earn
(C) 200
1 Rs. 420 in 21 days ?
(D) None of these (C) 9 (D) 11
2 (A) 15 doys
26. A contractor employed 30 men
to do a piece of work in 38 days. 31. If a certain number of workmen (B) 5 boys
After 25 days, he employed 5 can do a piece of work in 25
(C) 17 boys
men more and the work was days, in what time will another
set of an equal number of men (D) None of these
finished one day earlier. How
many days he would have been do a piece of work twice as great 37. If 27 kg of corn would feed 42
behind if he had not employed supposing that 2 of the first set horses for 21 days, in how many
additional men ? can do as much work in an hour days would 36 kg of it feed 21
as 3 of the second set can do in
1 horses ?
(A) 1 day (B) 1 days an hour ?
4 (A) 28 days (B) 42 days
(A) 60 days (B) 75 days
3 1 (C) 90 days (D) 105 days 1
(C) 1 days (D) 1 days (C) 56 days (D) 31 days
4 2 2
32. 15 men take 21 days of 8 hours
27. 2 men and 7 boys can do a piece each to do a piece of work. How 38. If 5 men working 6 hours a day
of work in 14 days. 3 men and 8 many days of 6 hours each can reap a field in 20 days. In
boys can do the same in 11 days. would 21 women take if 3 how many days will 15 men reap
8 men and 6 boys can do 3 times women do as much work as 2 the field, working 8 hours a day?
the amount of this work in— men ? (A) 5 days (B) 6 days
(A) 21 days (B) 18 days (A) 20 (B) 25 1
(C) 18 (D) 30 (C) 7 days (D) 9 days
(C) 24 days (D) 36 days 2

Quantitative Aptitude Test | 219


Answers with Hints
1. (D) More men, more work (Direct) 6. (D) The remaining food is sufficient for 120 men for
More working hrs., more work (Direct) 195 days.
More days, more work (Direct) But, now remaining men = 90
Men x : y⎫
⎪ Less men, more days (Indirect)
⇒ Working hrs. x : y⎬ ⎪::x:z ˙·˙ 90 : 120 : : 195 : x
Day x : y⎭ 120 × 195

∴ Z =
y×y×y×x
∴ x = ( 90 )
x×x×x = 260 days
y3 7. (C) Less pumps, more days (Indirect)
= 2 units of work.
x Less water, less days (Direct)
2. (B) More length more labourers (Direct) More working hrs., less days (Indirect)
More daily hours, less labourers (Indirect) Pumps 0016 : 0018⎪⎫
⇒ Water 2170 : 1736⎬
Less days, more labourers (Indirect) ⎪ : : 10 : x
Working hours 0009 : 0007⎭
Length 26 : 39⎪⎫
⇒ Daily hrs. 09 : 08⎬
Days 06 : 18⎭
⎪ : : 17 : x ∴ x = (1816× 1736 × 7 × 10
× 2170 × 9 )
39 × 8 × 18 × 17 = 7 days.
∴ x =
26 × 9 × 6 8. (B) Less examiners, more hours per day (Indirect)
= 68 labourers. More days, less hours per day (Indirect)
1 2 More answer books, more hours per day (Direct)
3. (B) Work done = , work to be done =
3 3 Examiners 02 : 04⎫ ⎪
Now, more work, more men (Direct) ⇒ Days 20 : 08⎬
⎪::5:x
More days, less men (Indirect) Ans. Books 01 : 02⎭
1 2 ⎫ 4 ×8×2×5
Work : ⎪ ∴ x =
⇒ 3 3 ⎬ ⎪ : : 20 : x 2 × 20 × 1
Days 25 : 20⎭ = 8 hours per day

∴ x = (23 × 20 × 20 ×253 ) 9. (B) Less men, more days (Indirect)


More working hours, less days (Indirect)
= 32 men.
So, 12 more men should be employed.
Men 15 : 20
}
⇒ Working hrs. 08 : 07 : : 10 : x

20 × 7 × 10
4. (D) Let initially there be x men having provisions for
y days.
∴ x = (15 × 8 ) 2
= 11 days.
3
After 10 days, x men had provisions for (y – 10) days 10. (B) Less days, more men (Indirect)
Less work, less men (Direct)
These provisions were for x – ( x5) i.e., 4x5 men for y ⇒
Days 15 : 16⎫

1⎬ : : 300 : x
days. Work 01 : ⎪
5⎭
4x
∴ x (y – 10) = ·y


5
xy – 50x = 0
∴ x = (16 ×15 × 300 × 151× 1)
= 64 men
⇒ x (y – 50) = 0
11. (D) More men, more length built (Direct)
⇒ y – 50 = 0
Less days, less length built (Direct)
∴ y = 50 days.
5. (D) More men less number of days (Indirect)
Men 20 : 25
Days 06 : 03 } : : 112 : x
800 : 500 : : 24 : x 25 × 3 × 112
x = (
20 × 6 )
∴ = 70 metres
500 × 24
∴ x = (800 ) 12. (D) More persons, more shawls (Direct)
= 15 days Less days, less shawls (Direct)
Quantitative Aptitude Test | 220
6 × 85·5
Persons 03 : 08
Day }
14 : 05 : : 168 : x ∴ x =
22·5
8 × 5 × 168 = 22·8 kg.
∴ x = = 160 shawls.
3 × 14 20. (B) More men, less days (Indirect)
13. (B) More men, more earning (Direct) ˙·˙ 20 : 16 : : 30 : x
Less hours, less earning (Direct)
Men
}
6:9
Hours/Day 8 : 6 : : 840 : x
∴ x = (1620× 30)
= 24
9 × 6 × 840
∴ x = 21. (A) ˙·˙ 15 buffaloes = 21 cows
6×8

14.
= Rs. 945.
(D) Less books, less number of binders (Direct)
∴ 35 buffaloes =
21
15 (
× 35 cows)
More days, less number of binders (Indirect) = 49 cows.
22. (D) ˙·˙ Less pipes, more time (Indirect)
Books 900 : 0660
Day 012 : 0 10 } : : 18 : x
⇒ 8 : 10 : : 24 : x
660 × 10 × 18
∴ (
x = )
900 × 12 ∴ x = (10 8× 24)
= 11. = 30 minutes
15. (B) More days, less cows (Indirect) 23. (B) Remaining part = (1/5)
Less rent, less cows (Direct)
Less part to be filled, less time taken (Direct)
Days 030 : 020
}
Rent 370 : 111 : : 40 : x ⇒
4 1
: = 1:x
5 5
20 × 111 × 40
∴ (
x = )
30 × 370 ∴ x = ( 1
×1×
5
) 1
= min.
5 4 4
= 8 cows.
16. (C) More litre, more patients (Direct) = 15 seconds.
Less days, more patients (Indirect) 24. (C) ˙·˙ More time, more distance covered (Direct)

Litres 1350 : 1710
}
Days 0028 : 0030 : : 70 : x
20 : 50 : : 5 : x

⇒ 1350 × 28 × x = 1710 × 30 × 70 ∴ x = (5020× 5)


1710 × 30 × 70
∴ x = = 12·5 km.
1350 × 28
25. (A) ˙·˙ More radius, less rounds (Indirect)
= 95 patients
17. (B) Less men, more days (Indirect) ⇒ 20 : 14 : 140 : x
Less kg. less days (Direct)
∴ x = (14 20× 140)
˙·˙ Men 30 : 40
}
∴ Kgs 60 : 12 : : 15 : x = 98 times.
⇒ 30 × 60 × x = 40 × 12 × 15 26. (A) After 25 days, 35 men complete the work in 12
40 × 12 × 15
∴ x = ( )
30 × 60
days.
˙·˙ 35 men can finish the remaining work in 12 days.
= 4 days 12 × 35
18. (D) More distance on the map, more actual distance. ∴ 30 men can finish it in = = 14 days
30
(Direct)
i.e., 1 day behind.
˙·˙ 0·8 : 80·5 : : 8·8 : x. 27. (A) ˙·˙ (2 × 14) men + (7 × 14) boys
⇒ 0·8 × x = 80·5 × 8·8 = (3 × 11) men + (8 × 11) boys
80·5 × 8·8 ⇒
∴ x = = 885·5 km 5 men = 10 boys
0·8 ⇒ 1 man = 2 boys
= 885 km. ∴ 2 men + 7 boys = 11 boys
19. (A) Less length, less weight (Direct) and 8 men + 6 boys = 22 boys
˙·˙ 22·5 : 6 : : 85·5 : x Now, more boys, less days (Indirect)
⇒ 22·5 × x = 6 × 85·5 More work, more days (Direct)

Quantitative Aptitude Test | 221


Boys 22 : 11
Work 01 : 03 } : : 14 : x 33. (B) Remaining work = (1 – 25) = 35
11 × 3 × 14 Remaining period = (56 – 30)
∴ x = = 21 days
22 × 1 = 26 days
28. (A) (6 men + 2 boys) = 14 boys Now, the problem becomes : 104 men working 8 hrs.
Now, more work, more number of days (Direct) a day can finish (2/5) work in 30 days, how many
men working 9 hrs. a day can finish (3/5) work in 26
More boys, less number of days (Indirect) days ?
More hours per day, less number of days (Indirect) More work, more men (Direct)
Work 01 : 02⎫
⎪ Less days, more men (Indirect)
Boys 14 : 06⎬
⎪ : : 10 : x More hours, less men (Indirect)
Hrs./Day 08 : 07⎭
2 3⎫
Work : ⎪
∴ x =
2 × 6 × 7 × 10 1
= 7 days. ˙·˙ 5 5⎪ ⎬ : : 104 : x
1 × 14 × 8 2 Days 26 : 30⎪

29. (A) More engine, more coal (Direct) Hrs./Day 19 : 18⎭
More hrs. a day, more coal (Direct)
More rate of consumption, more coal (Direct) ∴ x = (35 × 30 × 8 × 104 × 25 ×261 ×19)
Engine 5:8 ⎪ ⎫ = 160
Hrs/Days 9 : 10⎪ ⎬
Rate 1 1 ⎪::6:x So, additional no. of men to be employed
: ⎪
3 4 ⎭ = (160 – 104) = 56.
34. (A) [(100 × 35) + (200 × 5)] working for 1 day can
∴ ( 1
x = 8 × 10 × × 6 × × × 3
4
1 1
5 9 ) finish the work
Thus, 4500 men can finish it in 1 day
= 8 metric tonnes.
30. (A) More men, less days (Indirect)
Less hours a day, more days (Indirect)
So, 100 men can finish it in = ( ) 4500
100
More speed, less days (Indirect) = 45 days
Men 12 : 9 ⎫⎪
1 35.
i.e., 5 days behind schedule.
(B) Let there be x men at the beginning

Hrs./Day 06 : 7 2 : : 20 : x
1 1 ⎪ Now, less men would take more days
Speed :
2 3 ⎭ ˙·˙ 15 : 9 : : x : (x – 6)
⇒ 15 × (x – 6) = 9x
∴ ( 15 1
x = 9 × × × 20 ×
2 3
2
)
12 × 6 × 1
∴ x = 15 men.
36. (B) More money, more boys (Direct)
1
= 12 days. More days, less boys (Indirect)
2
31. (B) Speed of doing work of first and second set of
1 1
men is : .
Money 240 : 420
Days }
221 : 225 : : 12 : x
420 × 5 × 12
2 3
Now, more work, more time (Direct)
∴ (
x =
240 × 21)
Less speed, more time (Indirect) = 5 boys
Work 1 : 2⎫ ⎪ 37. (C) More corn, more days (Direct)
˙·˙ Speed 1 : 1⎬ ⎪ : : 25 : x Less horses, more days (Indirect)
3 2⎭

∴ ( 1
x = 2 × × 25 × )
3
= 75 days.
Corn 27 : 36
}
Horse 21 : 42 : : 21 : x
2 1×1 36 × 42 × 21
32. (D) 3 Women = 2 men ∴ x = = 56 days.
27 × 21
So 21 women = 14 men 38. (A) More men, less days (Indirect)
Now, less men, more days (Indirect)
More working hrs., less days (Indirect)
Less hours, more days (Indirect)
Men 14 : 15
}
Working hrs. 16 : 18 : : 21 : x
Men 15 : 5
Working hrs. 28 : 6 } : : 20 : x
15 × 8 × 21
∴ x = ( 14 × 6 ) = 30 days. ∴ x =
5 × 6 × 20
15 × 8
= 5 days.

Quantitative Aptitude Test | 222


Boats and Streams
Important Points Solving (iii) and (iv) Q. 4. A man can row 6 km/hr
1 1 in still water. It takes him twice as
(i) Direction along the stream is we get u = and v =
5 11 long to row up as to row down the
called downstream.
river. Find the rate of stream.
(ii) Direction against the stream 1 1

is called upstream. x = 5 Solution : Let men’s rate up-
stream = x km/hr
(iii) Let, speed of boat in still 1 1
and
water be a km/hr and the speed of y = 11 Then, men’s rate downstream
stream be b km/hr. Then = 2 x km/hr
i.e. x = 5 and y = 11
Speed downstream ∴ Man’s rate in still water
∴ Rate in still water
= (a + b) km/hr. 1
1 = (x + 2x) km/hr
Speed upstream = (5 + 11) km/hr. 2
2
= (a – b) km/hr. 3x
= 8 km/hr. ˙·˙ = 6
(iv) If a man rows in still water 2
Rate of current
at a km/hr and the rate of current or ⇒ x = 4 km/hr
stream is b km/hr. then— 1
= (11 – 5) km/hr. Thus, man’s rate upstream
2
Man’s rate with the current
= 3 km/hr. = 4 km/hr
= (a + b) km/hr.
Q. 2. A man can row 6 km/hr Man’s rate downstream
Man’s rate against the current
in still water. When the river is = 8 km/hr
= (a – b) km/hr. running at 1·2 km/hr, it takes him ∴ Rate of stream
Rate in still water 1 hour to row to a place and back.
1
1 How far is the place ? = (8 – 4) km/hr
= [(rate with the current) + 2
2 Solution : Man’s rate down
(rate against the current)] stream = (6 + 1·2) km/hr = 7·2 km/hr. = 2 km/hr
Rate of current Man’s rate upstream = (6 – 1·2) Q. 5. A man rows downstream
= 4·8 km/hr. 30 km and upstream 18 km, taking
1 5 hours each time. What is the
= [(rate with the current) – Let required distance be x km.
2 velocity of current ?
(rate against the current)] Then
Solution : Man’s rate down-
x x
+ = 1 stream
Examples 7·2 4·8
Q. 1. A man can row 30 km
⇒ 4·8x + 7·2 x = 7·2 × 4·8 =( ) 30
5
km/hr
upstream and 44 km downstream 7·2 × 4·8 = 6 km/hr
∴ x =
in 10 hours. Also, he can row 40 km 12 Man’s rate upstream
upstream and 55 km downstream = 2·88 km.
in 13 hours. Find the rate of current
and the speed of the man in still
Q. 3. In a stream running at 2 =( ) 18
5
km/hr
km/hr a motor boat goes 10 km ∴ Velocity of current
water. upstream and back again to the
Solution : Let, rate upstream = x starting point in 55 minutes. Find
the speed of motor boat in still
= (1
2
6– )
18
5
km/hr
km/hr. and rate downstream = y
water. = 1·2 km/hr
km/hr.
Solution : Let the speed of motor Q. 6. A man can row upstream
30 44 boat in still water be x km/hr at 11 km/hr and downstream at 16
Then x + y = 10 …(i)
km/hr. Find man’s rate in still
Then Speed down stream
40 55 water and the rate of current.
= (x + 2) km/hr
x + y = 13 …(ii) Solution : Rate in still water
and Speed upstream
⇒ 30u + 44v = 10 …(iii) 1
= (x – 2) km/hr = (11 + 16) km/hr
2
40u + 55v = 13 …(iv) 10 10 55
˙·˙ + = = 13·5 km/hr
1 x + 2 x – 2 60 Rate of current
where u = x
⇒ 11x2 – 240 x – 44 = 6 1
= (16 – 11) km/hr
1 ⇒ (x – 22) (11x + 2) = 0 2
and v = y
∴ x = 22 km/hr = 2·5 km/hr

Quantitative Aptitude Test | 223


Exercise 8. A man can row with the stream 15. A man rows to a place 48 km
at 11 km/hr and against the distant and back in 14 hours. He
01. A boat travels upstream from B stream at 8 km/hr. The speed of finds that he can row 4 km with
to A and downstream from A to the stream is— the stream in the same time as 3
B in 3 hours. If the speed of the (A) 3 km/hr (B) 9·5 km/hr km against the stream. The rate
boat in still water is 9 km/hr and of the stream is—
(C) 1·5 km/hr (D) 6 km/hr
the speed of the current is 3 (A) 0·5 km/hr (B) 1 km/hr
km/hr the distance between A 9. A man can swim 3 km/hr in still
(C) 3·5 km/hr (D) 1·8 km/hr
and B is— water. If the velocity of the
(A) 4 km (B) 6 km stream be 2 km/hr, the time taken 16. A boat moves upstream at the
(C) 8 km (D) 12 km by him to swim to a place 10 km rate of 1 km in 10 minutes and
upstream and back is— downstream at the rate of 1 km
2. A boat goes 40 km. upstream in 1 1 in 6 minutes. The speed of the
8 hours and 36 km. downstream (A) 8 hrs (B) 9 hrs current is—
3 5
in 6 hours. The speed of the boat (A) 1 km/hr (B) 1·5 km/hr
in standing water is— (C) 10 hrs (D) 12 hrs
(C) 2 km/hr (D) 2·5 km/hr
(A) 6·5 km/hr (B) 6 km/hr 10. A man can row three quarters of
(C) 5·5 km/hr (D) 5 km/hr a kilometre against the stream in 17. A man rows upstream 16 km and
1 1 downstream 28 km taking 5
3. If a man’s rate with the current is 11 minutes and return in 7 hours each time. The velocity of
4 2
12 km/hr. and the rate of the minutes. The speed of the man in the current is—
current is 1·5 km/hr then man’s still water is— (A) 2·4 km/hr (B) 1·2 km/hr
rate against the current is—
(A) 2 km/hr (B) 3 km/hr (C) 3·6 km/hr (D) 1·8 km/hr
(A) 9 km/hr
(C) 4 km/hr (D) 5 km/hr 18. Speed of a boat in standing water
(B) 6·75 km/hr
1 is 6 km/hr and the speed of the
(C) 5·25 km/hr 11. A man can row 9 km/hr in still
3 stream is 1·5 km/hr. A man rows
(D) 7·5 km/hr to a place at a distance of 22·5
water and he finds that it takes
4. If a man rows at 5 km/hr in still him thrice as much time to row km and comes back to the
water and 3·5 km/hr against the up than as to row down the same starting point. The total time
current his rate along the current distance in river. The speed of taken by him, is—
is— the current is— (A) 6 hrs 30 min
(A) 8·5 km/hr (B) 6·5 km/hr 1 1 (B) 8 hrs 24 min
(A) 3 km/hr (B) 3 km/hr
(C) 6 km/hr (D) 4·25 km/hr 3 9 (C) 8 hrs
1 2 (D) 4 hrs 12 min
5. The speed of a boat downstream (C) 1 km/hr (D) 4 km/hr
4 3
is 15 km/hr and the speed of the 19. A man can row at 5 km/hr in still
stream is 1·5 km/hr. The speed 12. The current of a stream runs at 1 water and the velocity of current
of the boat upstream is— km/hr. A motor boat goes 35 km is 1 km/hr. It takes him 1 hour to
(A) 13·5 km/hr upstream and back again to the row to a place and back. How far
starting point in 12 hours. The is the place ?
(B) 16·5 km/hr speed of motor boat in still water (A) 2·5 km (B) 2·4 km
(C) 12 km/hr is—
(C) 3 km (D) 3·6 km
(D) 8·25 km/hr (A) 6 km/hr (B) 7 km/hr
(C) 8·5 km/hr (D) 8 km/hr 20. The speed of a boat in still water
6. The speed of a boat in still water is 2 km/hr. If its speed upstream
is 2 km/hr. If its speed upstream 13. A boat covers 24 km upstream
be 1 km/hr, then speed of the
be 1 km/hr, then speed of the and 36 km downstream in 6 hours
stream is—
stream is— while it covers 36 km upstream
1 (A) 2 km/hr
(A) 1·5 km/hr and 24 km downstream in 6
2 (B) 3 km/hr
(B) 3 km/hr hours. The velocity of the current (C) 1 km/hr
(C) 1 km/hr is— (D) None of these
(D) None of these (A) 1·5 km/hr (B) 1 km/hr
21. A boat goes 14 km upstream in
(C) 2 km/hr (D) 2·5 km/hr
7. A man can row downstream at 56 minutes. The speed of stream
14 km/hr and upstream at 9 km/ 14. The current of a stream runs at is 2 km/hr. The speed of boat in
hr. Man’s rate in still water is— the rate of 4 km/hr. A boat goes 6 still water is—
km and back to the starting point
(A) 5 km/hr (A) 6 km/hr (B) 15 km/hr
in 2 hours. The speed of the boat
(B) 23 km/hr in still water is— (C) 14 km/hr (D) 17 km/hr
(C) 11·5 km/hr (A) 6 km/hr (B) 7·5 km/hr 22. The speed of a boat in still water
(D) None of these (C) 8 km/hr (D) 6·8 km/hr is 10 km/hr. If its speed down

Quantitative Aptitude Test | 224


stream be 13 km/hr, then speed (C) 1·6 km/hr 35. A man can row 4·5 km/hr in still
of the stream is— (D) 1·5 km/hr water and he finds that it takes
(A) 1·5 km/hr him twice as long to row up as to
29. A person rows a kilometre down row down the river. Find the rate
(B) 3 km/hr the stream in 10 minutes and of stream—
(C) 11·5 km/hr upstream in 30 minutes. Find the
(A) 2 km/hr (B) 1·5 km/hr
(D) 5·75 km/hr velocity of the stream—
(C) 2·5 km/hr (D) 1·75 km/hr
23. A boat goes 12 km upstream in (A) 1 km/hr (B) 2 km/hr
48 minutes. The speed of stream (C) 3 km/hr (D) 4 km/hr 36. The speed of a boat in still water
is 2 km/hr. The speed of boat in is 15 km/hr and the rate of
30. A man can row three quarters of
still water is— current is 3 km/hr. The distance
a km against the stream in 11
(A) 13 km/hr travelled downstream in 12
minutes 15 seconds and return in minutes is—
(B) 2·25 km/hr 7 minutes 30 seconds. Find the
speed of the man in still water (A) 3·6 km (B) 2·4 km
(C) 17 km/hr
(D) 15 km/hr and also the speed of the (C) 1·2 km (D) 1·8 km
stream—
24. The speed of a boat in still water 37. Speed of a boat in standing water
(A) 5 km/hr; 2 km/hr is 7 km/hr and the speed of the
is 12 km per hour. Going down-
(B) 5 km/hr; 1 km/hr stream is 1·5 km/hr. A distance
stream it moves at the rate of 19
km per hour. The speed of the (C) 6 km/hr; 2 km/hr of 7·7 km, going upstream is
boat against the stream is— (D) 4 km/hr; 1 km/hr covered in—
(A) 5 km/hr 31. A boat’s man goes 48 km down- (A) 1 hr. 15 min.
(B) 3 km/hr stream in 8 hours and returns (B) 1 hr. 12 min.
(C) 8 km/hr back in 12 hours. Find the speed (C) 1 hr. 24 min.
(D) Data inadequate of the boat in still water and the (D) 2 hr. 6 min.
rate of the stream—
25. If a man rows at the rate of 5 (A) 5 km/hr; 1 km/hr 38. A man can row upstream 32 km
km/hr in still water and his rate in 4 hours. If the speed of current
(B) 10 km/hr; 2 km/hr
against the current is 3·5 km/hr, is 2 km/hr, find how much he
then the man’s rate along the (C) 6 km/hr; 1·5 km/hr can go downstream in 6 hours ?
current is— (D) None of these
(A) 70 km (B) 72 km
(A) 8·5 km/hr 32. The speed of a boat downstream (C) 64 km (D) 81 km
(B) 6·5 km/hr is 15 km/hr and the speed of the
(C) 6 km/hr stream is 1·5 km/hr. The speed 39. A man can row upstream 36 m
(D) 4·25 km/hr of the boat upstream is— in 6 hours. If the speed of a man
(A) 13·5 km/hr in still water is 8 km/hr, find how
26. A man can row 44 km down- much he can go downstream in
stream in 4 hours. If the man’s (B) 16·5 km/hr
10 hours—
rowing rate in still water is 8 (C) 12 km/hr
(A) 150 km (B) 80 km
km/hr, then find in what time (D) 8·25 km/hr
will he cover 25 km upstream ? (C) 90 km (D) 100 km
33. A man can row 60 km down-
(A) 5 hours (B) 6 hours stream in 6 hours. If the speed of 40. The speed of a boat in still water
(C) 4·5 hours (D) 4 hours the current is 3 km/hr, then find is 4 km/hr and the speed of
in what time will he be able to current is 2 km/hr. If the time
27. A man rows upstream 16 km and
cover 16 km upstream ? taken to reach a certain distance
downstream 27 km taking 5
(A) 4·5 hours (B) 4 hours upstream is 9 hours, find the
hours each time. What is the time it will take to go to same
velocity of current ? (C) 5 hours (D) 5·5 hours distance downstream—
(A) 2 km/hr 34. A man can row three quarters of (A) 2 hrs (B) 2·5 hrs
(B) 2·1 km/hr 1
a km against the stream in 11 (C) 3·5 hrs (D) 3 hrs
(C) 1·1 km/hr 2
(D) None of these 1 41. A man rows 8 km/hr in still
minutes and return in 7 minutes. water. If the river is running at 2
28. A boat moves downstream at the 3
Find the speed of the man in still km/hr, it takes 32 minutes to row
1 to a place and back. How far is
rate of one km in 7 minutes and water. What is the speed of the
2 the place ?
stream ?
upstream at the rate of 5 km an
(A) 5·025 km/hr, 1·115 km/hr (A) 1·5 km (B) 2·5 km
hour. What is the velocity of
current ? (B) 6 km/hr, 2 km/hr (C) 2 km (D) 3 km
(A) 1·3 km/hr (C) 4 km/hr, 1 km/hr 42. A motor boat can travel at 10
(B) 1·2 km/hr (D) None of these km/hr in still water. It travelled

Quantitative Aptitude Test | 225


91 km downstream in a river and (A) 2·4 km/hr (C) 6 km
then returned taking altogether (B) 2 km/hr (D) Data inadequate
20 hours. Find the rate of flow of (C) 3 km/hr
river— (D) Data inadequate 45. A man rows to a place 48 km
(A) 6 km/hr (B) 2 km/hr distance and back in 14 hours.
44. A boat takes 3 hours to travel
(C) 3 km/hr (D) 4 km/hr from place M to N downstream He finds that he can row 4 km
43. Ramesh can row a certain and back from N to M upstream. with the stream in the same time
distance downstream is 6 hours If the speed of the boat in still as 3 km against the stream. Find
and return the same distance in 9 water is 4 km, what is the dis- the rate of the stream—
hours. If the speed of Ramesh in tance between the two places ?
(A) 1 km/hr (B) 2 km/hr
still water is 12 km/hr, find the (A) 8 km
speed of the stream. (B) 12 km (C) 1·5 km/hr (D) 2·5 km/hr

Answers with Hints


1. (D) Speed downstream 8. (C) Speed of stream
= (9 + 3) km/hr = 12 km/hr 1
= (11 – 8) km/hr = 1·5 km/hr
Speed upstream = (9 – 3) km/hr = 6 km/hr 2
Let the distance 9. (D) Speed upstream = (3 – 2) km/hr = 1 km/hr
AB = x km Speed downstream = (3 + 2) km/hr = 5 km/hr

Then,
x x
+
6 12
= 3 ⇒ 2x + x = 36 Total time taken = ( 10 10
1
+
5) hr = 12 hrs.

⇒ x = 12
∴ Distance AB = 12 km
10. (D) ˙·˙ Speed upstream = ( 3 4
)
× × 60 km/hr
4 45
2. (C) Speed upstream = 4 km/hr

= ( )
40
8
km/hr = 5 km/hr Speed upstream = ( 3 2
)
× × 60 km/hr
4 15
Speed downstream = 6 km/hr
1
= ( )
36
6
km/hr = 6 km/hr ∴ Speed in still water = (4 + 6) km/hr
2
= 5 km/hr
Speed of boat in still water
11. (D) Let speed upstream
1
= (5 + 6) km/hr = 5·5 km/hr = x km/hr
2
3. (A) Let the rate against the current be x km/hr. Then, speed downstream
12 – x = 3x km/hr
Then, = 1·5 ⇒ 12 – x = 3
2 ∴ Speed in still water
⇒ x = 9 km/hr 1
= (x + 3x) km/hr = 2x km/hr
4. (B) Let the rate along the current be x km/hr 2
Speed of the current
x + 3·5
Then, = 5 1
2 = (3x – x) km/hr = x km/hr
2
⇒ x = (10 – 3·5) = 6·5 km/hr
28 14 2
5. (C) Let the speed of boat in still water be x km/hr ˙·˙ 2x = ⇒x= = 4 km/hr.
3 3 3
Then, x + 1·5 = 15 ⇒ x = 13·5 12. (A) Let the speed in still water be x km/hr
∴ Speed upstream = (13·5 – 1·5) km/hr 35 35
= 12 km/hr ˙·˙ + = 12
x–1 x+1
6. (C) Let the speed of stream be x km/hr ⇒ 35 (2x) = 12 (x2 – 1)
Then, speed upstream = (2 – x) km/hr ⇒ 12x2 – 70x – 12 = 0
2 – x = 1 ⇒ x = 1 km/hr ⇒ 12x2 – 72x + 2x – 12 = 0
7. (C) Man’s rate in still water ⇒ 12x (x – 6) + 2 (x – 6) = 0
1 ⇒ (x – 6) (12x + 2) = 0
= (14 + 9) km/hr = 11·5 km/hr
2 ∴ x = 6 km/hr

Quantitative Aptitude Test | 226


13. (C) Let the speed upstream be x km/hr and the speed 1
∴ Rate of stream = (8 – 6) km/hr 0
downstream be y km/hr respectively. 2
24 36 = 1 km/hr
Then, x + y = 6 16. (C) Speed upstream = 6 km/hr
⇒ 24u + 36 v = 6, …(1) Speed downstream = 10 km/hr

[ where
1 1
u = x‚ v = y ] 1
∴ Speed of the current = (10 – 6) km/hr
2
36 24 13 = 2 km/hr
and x + y = 2

⇒ 36u + 24v =
13
…(2)
17. (B) Speed downstream = ( )28
5
km/hr
2 = 5·6 km/hr
Adding these equations, we get

60 (u + v) =
25
Speed upstream = ( )16
5
km/hr
2 = 3·2 km/hr
5 1
⇒ u+v = …(3) Velocity of current = (5·6 – 3·2) km/hr
24 2
Subtracting, we get = 1·2 km/hr
1 18. (C) Speed upstream = (6 – 1·5) km/hr
12 (u – v) =
2 = 4·5 km/hr
1
⇒ u–v = …(4) Speed downstream = (6 + 1·5) km/hr
24
= 7·5 km/hr
Solving Eq. (3) and Eq. (4)

we get,
1
u = and v =
8
1
12
Total time taken = ( 22·5 22·5
4·5
+ )
7·5
hrs

= (5 + 3) hrs.
∴ x = 8 km/hr and y = 12 km/hr
= 8 hrs.
1
∴ Velocity of current = (12 – 8) km/hr 19. (B) Speed downstream = (5 + 1) km/hr
2
= 6 km/hr
= 2 km/hr
Speed upstream = (5 – 1) km/hr
14. (C) Let the speed in still water be x km/hr
= 4 km/hr
6 6
Then, + = 2 Let the required distance be x km
x+4 x–4
⇒ 6 [x – 4 + x + 4] = 2 (x2 – 16) x x
Then, + = 1
6 4
⇒ x2 – 16 = 6x
⇒ 2x + 3x = 12
⇒ x2 – 6x – 16 = 0
∴ x = 2·4 km
⇒ (x – 8) (x + 2) = 0
20. (C) ˙·˙ 2–y = 1
∴ x = 8 km/hr
∴ y = 2 – 1 = 1 km/hr
15. (B) Suppose he moves 4 km downstream in x hrs.
14 × 60
21. (D) ˙·˙ Rate upstream = = 15 km/hr
Then, Speed downstream = () 4
x km/hr
⇒ (x – 2) = 15
56

Speed upstream = (x ) km/hr


3
∴ x = 17 km/hr
22. (B) ˙·˙ 10 + y = 13
48 48
˙·˙ + = 14
4 3 ∴ y = 13 – 10 = 3 km/hr
x x 12 × 60
23. (C) ˙·˙ = (x – 2)
⇒ 12x + 16x = 14 48
1 ∴ x = 15 + 2 = 17 km/hr
⇒ x =
2 1
24. (A) ˙·˙ (x + 19) = 12
∴ Speed downstream = 8 km/hr 2
Speed upstream = 6 km/hr ∴ x = 24 – 19 = 5 km/hr

Quantitative Aptitude Test | 227


1 35. (B)
25. (B) ˙·˙ (x + 3·5) = 5
2 12
36. (A) Required distance = (15 + 3)
∴ x = 10 – 3·5 = 6·5 km/hr 60
26. (A) ˙·˙ Man’s rate in still water 18
= = 3·6 km
5
1
= [man’s rate with current plus his rate against 37. (C) ˙·˙ (7 – 1·5) T = 7·7
2
current] 7·7
∴ T = = 1 hr. 24 minutes
5·5
⇒ 8 = [
1 44 25
2 4
+ t] 38. (B) ˙·˙ Upstream rate =
32
4
= 8 km/hr
25
⇒ 16 = 11 + t ⇒ Speed of man in still water
= 8 + 2 = 10 km/hr
∴ t = 5 hours ⇒ Downstream rate = (10 + 2) = 12 km/hr
16 Now, the required distance = (10 + 2) 6 = 72 km
27. (C) ˙·˙ Man’s rate upstream = km/hr
5 36
27 39. (D) ˙·˙ Speed of current = 8 – = 2 km/hr
Man’s rate downstream = km/hr 6
5 ⇒ Rate downstream = (8 + 2) = 10 km/hr
∴ Required distance = (8 + 2)·10 = 100 km
∴ Velocity of the current = (
1 27 16
2 5 5
–) km/hr
40. (D) Upstream distance = (4 – 2)·9 = 18 km
= 1·1 km/hr 18
∴ Required time = = 3 hrs.
(4 + 2)
28. (D) ˙·˙ Rate downstream = ( 2
15 )
× 60 km/hr
˙·˙
D
+
D
=
32
(8 – 2) (8 + 2) 60
= 8 km/hr
2·D·8 32
Rate upstream = 5 km/hr 41. (C) ⇒ =
82 – 22 60
1
∴ Velocity of the current = (8 – 5) km/hr ∴ D = 2 km
2
42. (C) Let the rate of flow of river = x km/hr
= 1·5 km/hr
29. (B) 30. (B) 91 91
˙·˙ + = 20
(10 + x) (10 – x)
48
31. (A) Rate downstream = = 6 km/hr ⎡ 10 – x + 10 + x ⎤⎥
⇒ 91 ⎢⎢
8
⎣ (10 + x) (10 – x)⎥⎦
= 20
48
Rate upstream = = 4 km/hr
12 ⇒ 91 × 20 = 20 (102 – x2)
1 ⇒ x2 = 102 – 91 = 9
∴ Speed of boat in still water = (6 + 4) = 5 km/hr
2 ∴ x = 3 km/hr.
1
Rate of stream = (6 – 4) = 1 km/hr.
1
2 43. (A) ˙·˙ Z ( )
9+6
9–6
= 12
32. (C) ˙·˙ (15 – y) = 1·5 ∴ Z = 2·4 km/hr
2
44. (D) Let the distance between M and N and the speed
∴ y = 15 – 3 = 12 km/hr
of current be d km and x km/hr respectively.
33. (B) ˙·˙ 3 = [
1 60 16
2 6
– t ] According to the question =
d
+
d
4+ x 4 – x
=3
∴ t = 4 hours
In the above equation we have only one equation but
3 60 two variables. Hence cannot be determined.
34. (A) The boat travels with stream at ×
4 1 (Data inadequate)
7
3 45. (A) Suppose that the man takes x hours to cover 4 km
= 6·14 km/hr downstream and x hours to cover 3 km upstream.
The boat travels against the stream at 48x 48x
3 60 Then, + = 14
× = 3·91 km/hr 4 3
4 1
11 1
2 ⇒ x =
2
1 3
∴ Speed of man in still water = (6·14 + 3·91) ∴ Rate upstream = = 6 km/hr
2 (1/2)
= 5·025 km/hr 4
1 and rate downstream = = 8 km/hr
and Speed of stream = (6·14 – 3·91) (1/2)
2 1
= 1·115 km/hr ∴ Rate of the stream = (8 – 6) = 1 km/hr.
2

Quantitative Aptitude Test | 228


Trains
Important Points Speed of train 9 seconds to pass him. If the goods
train is 150 m long, find its speed.
(i) Time taken by a train x
metres long in passing a signal post
= (25 ×185 ) m/sec Solution : Relative speed of
goods train
or a pole or a standing man is the
= (
18 )
125
same as the time taken by the train to
cover x metres with its own speed.
m/sec.
= ( )
150
9
m/sec
(x + y)
(ii) Time taken by a train x
metres long in passing a stationary
˙·˙ 125 = 18
18
= ( )
150 18
9
×
5
object of length y metres (such as a = 60 km/hr.
bridge or a tunnel or a platform or a 18 (x + y)
⇒ = 18 ∴Speed of goods train
train at rest) is the same as the time 125 = (60 – 50)
taken by the train to cover (x +y) ⇒ x + y = 125 …(i) = 10 km/hr.
metres with its own speed. Speed of train relative to man Q. 4. Two trains are moving in
(iii) Suppose two trains or two
= (25 + 5) km/hr the same direction at 50 km/hr and
bodies are moving in the same 30 km/hr. The faster train crosses a
direction at u km/hr and v km/hr
respectively such that u > v, then their ( )
= 30 ×
5
18
m/sec man in the slower train in 18
seconds. Find the length of the
relative speed = (u – v) km/hr
If their lengths be x km and y km
= ( )25
3
m/sec. faster train.
Solution : Relative speed of faster
respectively, then Time taken by the 3 train = (50 – 30) km/hr
faster train to cross the slower train ∴ x× = 12
(moving in the same direction)
25
25 × 12 ( )
= 20 ×
5
18
m/sec

( ) x+y ⇒ x =( )
= u – v hrs.
= 100 m
3
= ( )
50
9
m/sec.
(iv) Suppose two trains or two Distance covered in 18 sec at
bodies are moving in opposite Putting x = 100 in (i) we get
directions at u km/hr and v km/hr.,
then their relative speed
y = 25 this speed = 18 ×( ) 50
9
∴ Length of train = 100 metres = 100 m
= (u + v) km/hr
Length of platform = 25 metres ∴ Length of faster train = 100 m
If their length be x km and y km,
Q. 2. A train 100 m long takes Q. 5. Two trains 128 m and 132
then,
9 seconds to cross a man walking at m long are running towards each
Time taken to cross each other other on parallel lines at 42 km/hr
5 km/hr in the direction opposite to
⎛x + y⎞ that of the train. Find the speed of and 30 km/hr respectively. In what
=⎜ ⎟ hrs
⎝ u + v⎠ the train. time will they be clear of each
other from the moment they meet ?
(v) If two trains start at the same Solution. Let the speed of the
time from two points A and B to- train be x km/hr Solution : Relative speed of both
wards each other and after crossing, trains = (42 + 30) km/hr
Relative speed = (x + 5) km/hr
they take a and b hours in reaching B
and A respectively. Then, =
5 (x + 5)
m/sec
(
= 72 × )
5
18
= 20 m/sec.
18 Distance covered in passing each
A’s speed : B’s speed : : √⎯ b : ⎯√ a. Distance covered in passing the other = (128 + 132) = 260 m
man = 100 m ∴ Required time
Examples
Q. 1. A train running at 25 ˙·˙
100
5 (x + 5)
= 9 =( )260
20
= 13 sec.
km/hr takes 18 seconds to pass a 18 Q. 6. A train 110 m long is
platform. Next, it takes 12 seconds running at 60 km/hr. In what time
⇒ 1800 = 45 (x + 5)
to pass a man walking at 5 km/hr will it pass a man, running in the
in the opposite direction. Find the ⇒ x = 35 direction opposite to that of the
length of the train and that of the ∴ Speed of the train = 35 km/hr. train at 6 km/hr ?
platform. Solution : Speed of the train
Q. 3. A man sitting in a train
Solution : Let the length of train which is travelling at 50 km/hr relative to man = (60 + 6) km/hr
= x metres and length of platform
= y metres.
observes that a goods train, tra-
velling in opposite direction, takes (
= 66 ×
5
18 ) ( )
=
55
3
m/sec

Quantitative Aptitude Test | 229


Distance covered by it in passing ˙·˙ Distance covered in passing (C) 90 metres
the man = 110 m the platform = (160 + 140) = 300 m (D) 540 metres
∴ Time taken in passing the man ∴ Required time taken
(
= 110 ×
3
55) = 6 sec.
(
= 300 ×
9
100 )
5. A person sees a train passing
over 1 km long bridge. The
Q. 7. A train 125 m long is length of the train is half that of
running at 50 km/hr. In what time = 27 sec. bridge. If the train clears the
will it pass a man, running at 5 Q. 10. Find the time taken by a bridge in 2 minutes the speed of
km/hr in the same direction in train 120 m long, running at 54 the train is—
which the train is going ? km/hr. in crossing an electric pole. (A) 50 km/hr (B) 45 km/hr
Solution : Speed of train relative Solution : (C) 60 km/hr (D) 30 km/hr
to man = (50 – 5) km/hr Speed = 54 km/hr
6. A train 50 metres long passes a
( )
= 45 ×
5
18
m/sec
(
= 54 ×
5
)
18
m/sec platform 100 m long in 10
seconds. The speed of the train
= ( )
25
2
m/sec = 15 m/sec is—
Distance moved in passing the (A) 10 km/hr (B) 15 km/hr
Distance covered in passing the
pole = 120 m (C) 54 km/hr (D) 100 km/hr
man = 125 m
∴ Time taken by it in passing Required time taken
7. A train 300 metres long passes a
(
the man = 125 ×
2
)
25
= 10 sec. =
120
15
standing man is 15 seconds. The
speed of the train is—
Q. 8. A train passes a standing = 8 sec. (A) 40 km/hr (B) 50 km/hr
man in 2 seconds and a platform
1 Exercise (C) 60 km/hr (D) 72 km/hr
50 m long in 4 seconds. Find the
2 8. The length of the train that takes
1. A train 270 metres long is mov-
length of the train and its speed. 8 seconds to pass a pole where it
ing at a speed of 25 kmph. It will
Solution : Let the speed be x runs at a speed of 36 km/hr is—
cross a man coming from the
km/hr or ( )
5x
18
m/sec opposite direction at a speed of 2
km per hour in—
(A) 288 metres
(B) 45 metres
Let the length of the train be y (C) 48 metres
(A) 36 seconds
metres. (D) 80 metres
y (B) 32 seconds
˙·˙ = 2 9. A train running at the speed of
5x (C) 28 seconds
45 kmph took 12 seconds in
18 (D) 24 seconds
passing a certain point. Then the
⇒ 10x = 18y 2. A train 100 metres long travels length of the train must be—
⇒ 5x = 9y …(1) at 70 km per hour. A man is (A) 90 metres
y + 50 9 running at 10 km per hour in the
Also, = (B) 120 metres
5x 2 same direction in which the train (C) 150 metres
18 is going. The train will pass the (D) 540 metres
⇒ 36 (y + 50) = 45x man in—
⇒ 5x – 4y = 200 …(2) 2 10. A train 120 metres long is run-
(A) 6 seconds (B) 6 seconds ning at a rate of 54 km/hr. Time
⇒ 9y – 4y = 200 3
taken by the train to cross a
∴ y = 40 metres (C) 7 seconds (D) 8 seconds
tunnel 130 metres long is—
⇒ 5x = 9 × 40 3. A train 700 m long is running at 1
9 × 40 the speed of 72 km per hour. If it (A) 8 seconds
∴ x = 3
5 crosses a tunnel in 1 minute, 2
= 72 km/hr. then the length of the tunnel is— (B) 16 seconds
3
Q. 9. A train 160 m long is (A) 500 m (B) 550 m (C) 10 seconds
running at 40 km/hr. In how much
(C) 600 m (D) 700 m (D) 15 seconds
time will it pass a platform 140 m
long ? 4. A train 100 metres long, running 11. A train 280 metres long is mov-
Solution : Speed of the train at 36 kmph takes 25 seconds to ing at a speed of 60 km/hr. The

( )
= 40 ×
5
18
pass a bridge. The length of the
bridge is—
time taken by the train to cross a
platform 220 metres long is—

= ( )
100
9
m/sec
(A) 150 metres
(B) 144 metres
(A) 20 seconds
(B) 25 seconds

Quantitative Aptitude Test | 230


(C) 30 seconds train starts from A at 7 a. m. and 54 kmph, in 12 seconds. If it also
travels towards B at 20 km per 1
(D) 35 seconds passes a railway platform in 1
hour speed. Another train starts 2
12. A train 250 metres long, running from B at 8 a. m. and travels minutes, the length of the plat-
with a speed of 50 km/hr will towards A at a speed of 25 km form is—
pass an electric pole in— per hour. At what time will they (A) 560 metres
(A) 30 seconds meet ? (B) 620 metres
(B) 18 seconds (A) 9 a. m. (C) 700 metres
(C) 72 seconds (B) 10 a. m. (D) 750 metres
(D) 60 seconds (C) 11 a. m. 24. A train speeds past a pole in 15
13. A train 75 metres long is running (D) None of these seconds and speeds past a plat-
with a speed of 20 km/hr. It will 19. A train 100 metres long moving form 100 metres long in 25
pass a standing man in— at a speed of 50 kmph crosses a seconds. Its length in metres is—
(A) 12 seconds train 120 metres long coming (A) 200
(B) 13·5 seconds from opposite direction in 6 (B) 150
(C) 14 seconds seconds. The speed of second (C) 50
(D) 15·5 seconds train is— (D) Data inadequate
(A) 132 kmph (B) 82 kmph
14. A speed of 16 metres per second 25. A 150 metre long train crosses a
(C) 60 kmph (D) 50 kmph
is the same as— man walking at the speed of 6
(A) 40·3 km/hr 20. Two trains running in the same kmph in the opposite direction in
(B) 57·6 km/hr direction at 40 kmph and 22 6 seconds. The speed of the train
kmph completely pass one in km/hr is—
(C) 51·16 km/hr
another in 1 minute. If the length (A) 66 (B) 84
(D) None of these of the first train is 125 metres, (C) 96 (D) 106
15. A train moves with the speed of the length of second train is—
180 km/hr, then its speed in (A) 125 metres 26. A train is running at the rate of
metres per second is— 40 kmph. A man also is going in
(B) 150 metres
the same direction parallel to the
(A) 5 (B) 30 (C) 200 metres train at the speed of 25 kmph. If
(C) 40 (D) 50 (D) 175 metres the train crosses the man in 48
16. A train running at a certain speed 21. Two trains are running in oppo- sec, the length of the train is—
crosses a stationary engine in 20 site directions with a speed of 62 (A) 50 metres
seconds. To find out the speed of kmph and 40 kmph respectively. (B) 100 metres
the train, which of the following If the length of one train is 250
information is necessary ? (C) 150 metres
metres and they cross each other
(A) Only the length of the train (D) 200 metres
in 18 seconds, the length of the
(B) Only the length of the engine other train is— 27. A train of length 150 metres
(C) Either the length of the train (A) 145 metres takes 10 seconds to pass over
or the length of the engine (B) 230 metres another train 100 metres long
(D) Both the length of the train coming from the opposite
(C) 260 metres
and the length of the engine direction. If the speed of the first
(D) Cannot be determined train be 30 kmph, the speed of
17. A train overtakes two persons 22. A train 100 metres in length the second train is—
who are walking in the same passes a milestone in 10 seconds (A) 54 kmph (B) 60 kmph
direction in which the train is and another train of the same (C) 72 kmph (D) 36 kmph
going, at the rate of 2 kmph and length travelling in opposite
4 kmph and passes them comple- 28. A train takes 5 seconds to pass
direction in 8 seconds. The speed
tely in 9 and 10 seconds respec- an electric pole. If the length of
of the second train is—
tively. The length of the train the train is 120 metres, the time
(A) 36 kmph (B) 48 kmph taken by it to cross a railway
is—
(C) 54 kmph (D) 60 kmph platfom 180 metres long, is—
(A) 72 metres (B) 54 metres
23. A train travelling at 36 kmph 1
(C) 50 metres (D) 45 metres (A) 12 seconds
completely crosses another train 2
18. Two stations A and B are 110 having half its length and trave- 1
(B) 7 seconds
kms apart on a straight line. One lling in the opposite direction at 2

Quantitative Aptitude Test | 231


1 (C) 64·8 km/hr 41. A train 160 metres long passes a
(C) 6 seconds
2 (D) 37·5 km/hr standing man in 18 seconds.
1 What is the speed of the train ?
(D) 3 seconds 34. Two trains 132 metres and 108
3 (A) 28 km/hr
metres long are running in
29. A train moving at the rate of 36 opposite directions, one at the (B) 36 km/hr
km per hour crosses a standing rate of 32 kmph and another one (C) 32 km/hr
man in 10 seconds. It will cross a at the rate of 40 kmph. From the (D) None of these
platform 55 metres long in— moment they meet they will
1 42. A train 120 metres long, crosses
(A) 5 seconds cross each other in—
2 a pole in 10 seconds. The speed
(A) 10 seconds of the train is—
(B) 6 seconds
(B) 11 seconds (A) 40 km/hr
1
(C) 7 seconds (C) 12 seconds
2 (B) 43·2 km/hr
1 (D) 13 seconds (C) 45 km/hr
(D) 15 seconds
2 35. A train 300 m long crossed a (D) None of these
30. A train crosses a platform 100 platform 900 m long in 1 minute 43. A train crosses a platform in 30
metres long in 60 seconds at a 12 seconds. The speed of the seconds at a speed of 45 km/hr.
speed of 45 km per hour. The train in km/hr was— How much time will it take to
time taken by the train to cross (A) 45 (B) 50 cross an electric pole if the length
an electric pole is— (C) 54 (D) 60 of the platform is 100 metres ?
(A) 8 seconds (A) 8 seconds
36. How long will a train 130 m
(B) 1 minute (B) 30 seconds
long travelling at 40 km an hour,
(C) 52 seconds (C) 52 minutes
take to pass a kilometre stone ?
(D) Data inadequate (D) None of these
(A) 12 sec (B) 11·7 sec
31. Two trains of equal length are (C) 11·2 sec (D) 11 sec 44. A train 110 metres long travels
running on parallel lines in the
37. A train travelling at 30 km an at 60 km/hr. How long does it
same direction at the rate of 46
1 take to cross a telegraph post ?
kmph and 36 kmph. The faster hour took 13 sec in passing a
2 (A) 6 sec (B) 5·6 sec
train passes the slower train in
certain point. Find the length of (C) 6·6 sec (D) 6·8 sec
36 seconds. The length of each the train—
train is— 45. Find the length of a bridge, which
(A) 113 metres a train 130 metres long travelling
(A) 50 metres (B) 72 metres
(B) 112 metres at 45 km an hour, can cross in 30
(C) 80 metres (D) 82 metres
(C) 112·5 metres seconds ?
32. Two trains are running on
(D) None of these (A) 240 m (B) 235 m
parallel lines in the same
(C) 250 m (D) 245 m
direction at a speed of 50 km and 38. A train 110 metres in length runs
30 km per hour respectively. The through a station at the rate of 46. A column of men, extending 250
faster train crosses a man in 36 km per hour. How long will it metres in length takes one hour
slower train in 18 seconds. The take to pass a given point ? to march through a street at the
length of the faster train is— (A) 11 sec (B) 12 sec rate of 50 paces a minute, each
(A) 170 metres pace being 75 cm. Find the length
(C) 13 sec (D) 15 sec
(B) 100 metres of the street ?
39. A train 135 metres long is runn- (A) 2 km (B) 1 km
(C) 98 metres
ing with a speed of 54 km per (C) 1·5 km (D) 2·5 km
(D) 85 metres hour. In what time will it pass a
33. Two trains A and B start from telegraph post ? 47. It is noticed that exactly half a
station X and Y towards Y and minute elapses between the time
(A) 9 sec (B) 12 sec
X respectively. After passing when the engine of a train 50 m
(C) 8 sec (D) 6 sec
each other they take 4 hours 48 long enters a tunnel 500 m long
minutes and 3 hour 20 minutes 40. A train 550 metres long is run- and the time when the last
to reach Y and X respectively. If ning with a speed of 55 km per carriage of the train leaves the
train A is moving at 45 km/hr, hour. In what time will it pass a tunnel. Find at how many km per
then the speed of train B is— signal post ? hour the train is travelling—
(A) 60 km/hr (A) 30 sec (B) 24 sec (A) 66 km/hr
(B) 54 km/hr (C) 42 sec (D) 36 sec (B) 55 km/hr

Quantitative Aptitude Test | 232


(C) 64 km/hr 53. A train 50 m long passes a plat- 2
(C) 6 seconds
(D) None of these form 100 m long in 10 seconds. 3
The speed of the train in m/sec (D) Data inadequate
48. A train 540 m long is running is—
with a speed of 72 km/hr. In 58. Two trains 70 m and 80 m long
(A) 150 (B) 50 respectively, run at the rates of
what time will it pass a tunnel
(C) 10 (D) 15 68 km and 40 km an hour re-
160 m long ?
(A) 40 sec (B) 30 sec 54. A train 700 m long is running at spectively on parallel rails in
the speed of 72 km/hr. If it cross opposite directions. How long do
(C) 35 sec (D) 42 sec
a tunnel in 1 minute, then the they take to pass each other ?
49. A train 200 m long is running length of the tunnel— (A) 5 seconds
with a speed of 72 km/hr. In (A) 700 m (B) 600 m (B) 10 seconds
what time will it pass a platform (C) 550 m (D) 500 m (C) 12 seconds
160 m long ?
55. A train 110 m long travels at 60 (D) 6 seconds
(A) 18 sec (B) 21 sec
km/hr. How long does it take to
(C) 15 sec (D) 20 sec 59. Two trains 132 metres and 108
cross a platform 240 metres
metres in length are running
50. A train 240 m long passes a long ?
towards each other on parallel
bridge 120 m long in 24 sec. (A) 21 sec (B) 20 sec lines. One at the rate of 32 km/hr
Find the speed with which the (C) 18 sec (D) 24 sec and another at 40 km/hr. In what
train in moving— 56. A train with 90 km/hr crosses a time will they be clear of each
(A) 45 km/hr (B) 54 km/hr bridge in 36 seconds. Another other from the moment they
(C) 36 km/hr (D) 42 km/hr train 100 m shorter crosses the meet ?
51. A train 150 m long passes a same bridge at 45 km/hr. Find (A) 12 sec.
telegraph post in 12 seconds. the time taken by the second
(B) 9 sec.
Find in what time, it will pass a train to cross the bridge—
(A) 64 sec (B) 60 sec (C) 15 sec.
bridge 250 m long ?
(C) 72 sec (D) 1 hr (D) Data inadequate
(A) 32 sec (B) 36 sec
(C) 25 sec (D) 24 sec 57. A train 110 metres in length 1
60. A train 100 metres long takes 7
travels at 60 km/hr. In what time 5
52. A train 280 m long is moving at seconds to cross a man walking
will it pass a man who is walk-
a speed of 60 km/hr. The time at the rate of 5 km/hr in a direc-
ing against the train at 6 km an
taken by the train to cross a tion opposite to that of the train.
hour ?
platform 220 m long is— Find the speed of the train—
1
(A) 20 sec (B) 25 sec (A) 7 seconds (A) 54 km/hr (B) 45 km/hr
3
(C) 30 sec (D) 35 sec (B) 6 seconds (C) 42 km/hr (D) 36 km/hr

Answers with Hints


1. (A) Relative speed = (25 + 2) = 27 km/hr ∴ Time taken by the train to pass the man

( )
= 27 ×
5
18
m/sec. (
= 100 ×
3
50 )
= 6 sec.

= ( )
15
2
m/sec. 3. (A) Speed of the train = 72 × ( 5
18 )
= 20 m/sec

Time taken by the train to pass the men Let the length of tunnel = x metres

(
= 270 ×
2
15 ) Then,
(700 + x)
60
= 20

= 36 sec. ⇒ 700 + x = 1200


2. (A) Relative speed of the train ∴ x = 500 metres
= (70 – 10)
= 60 km/hr
4. (A) (
Speed = 36 ×
5
18 )
= 10 m/sec

Let the length of the bridge


( )
= 60 ×
5
18 = xm
Distance covered = (100 + x) m
= ( )
50
3
m/sec. Time taken = 25 sec

Quantitative Aptitude Test | 233


(100 + x) 12. (B) ˙·˙ Speed of the train
˙·˙ = 10 m/sec
25
⇒ 100 + x = 250 (
= 50 ×
5
) ( )
18
=
125
9
m/sec

∴ x = 150 metre ∴ Time taken by the train to pass the pole

5. (B) Distance covered in


2
60
hours (
= 250 ×
9
125) = 18 sec

( )
= 1+
1
2
3
= km
2
13. (B) ˙·˙ Speed of the train

Distance covered in 1 hour (


= 20 ×
5
) ( )
18
=
50
9
m/sec

∴ Time taken by the train to pass the man


= ( 3 60
2 2
×) = 45 km

So, speed of the train (


= 75 ×
9
)
50
= 13·5 sec
= 45 km/hr.
6. (C) ˙·˙ Distance covered by train in 10 sec ( )
14. (B) 16 m/sec = 16 ×
18
5
= 57·6 km/hr

= (50 + 100) = 150 m


( )
15. (D) 180 km/hr = 180 ×
5
18
= 50 m/sec
∴ Speed = ( )
150
10
m/sec
16. (D) Since the sum of the length of the train and the
length of the engine is needed, so both the lengths
( )
= 15 ×
18
5
= 54 km/hr must be known.
17. (C) Let the length of the train be x km and its speed
7. (D) ˙·˙ Speed =
Distance
Time ( )
=
300
15 be y km/hr
= 20 m/sec Then, speed of train relative to first man
= (y – 2) km/hr
(
= 20 × )18
5
= 72 km/hr Speed of train relative to second man
= (y – 4) km/hr
8. (
(D) Speed of the train = 36 × ) 5
18
˙·˙
x
=
9
= 10 m/sec. y–2 60 × 60
Distance = (Time × Speed) = (8 × 10) x 10
and =
= 80 metres y–4 60 × 60
∴ Length of the train = 80 metres ∴ 9y – 18 = 3600x …(1)
and 10y – 40 = 3600x …(2)
9. (
(C) ˙·˙ Speed = 45 × ) ( )
5
18
=
25
2
m/sec
So, 9y – 18 = 10y – 40
Distance = (Time × Speed) ⇒ y = 22
x 9
(
= 12 × )
25
2
metres = 150 metres ˙·˙ 22 – 2
=
3600
∴ Length of the train 20 × 9 1
∴ x= = km
3600 20
= 150 metres
10. (B) ˙·˙ Speed of the train =( 1
20 )
× 1000 = 50 m.

(
= 54 ×
5
)
18
= 15 m/sec 18. (B) Suppose they meet x hrs after 7 a. m.
∴ Distance covered by A in x hrs
∴ Time taken by the train to cross the tunnel
= (20 × x) km
= Time taken by it to cover (120 + 130) m
Distance covered by B in (x – 1) hrs
=( )250
15
2
sec = 16 sec
3 = 25 (x – 1) km
11. (C) ˙·˙ Speed of the train ˙·˙ 20x + 25 (x – 1) = 110
⇒ 45x = 135
(
= 60 ×
5
)
18 ()
m/sec =
50
3
m/sec ∴ x = 3 hours
∴ Time taken by the train to cross the platform So, they meet at 10 a. m.
= Time taken by it to cover (280 + 220) m 19. (B) Let the speed of the second train be x km/hr

(
= 500 × )
3
50
sec = 30 sec
Relative speed of both trains
= (50 + x) km/hr

Quantitative Aptitude Test | 234


= [(50 + x) × 185 ] = (15 ×185)
= (
18 )
250 + 5x = 54 km/hr.
m/sec
23. (C) Let the length of slower train be x metres and the

˙·˙
(100 + 120)
= 6 ()
length of faster train be
x
metres
(
250 + 5x
18 ) 2
Their relative speed = (36 + 54) km/hr
⇒ 220 × 18 = 6 (250 + 5x)
⇒ 30x = 3960 – 1500 (
= 90 × )
5
18
2460 = 25 m/sec
∴ x = = 82
30 3x
˙·˙ = 12
∴ Speed of the second train 2 × 25
= 82 km/hr ⇒ 3x = 600
20. (D) ˙·˙ Relative speed of the train ⇒ x = 200
= (40 – 22) km/hr ∴ Length of slower train = 200 metres

(
= 18 × )
5
18
m/sec
Let the length of platform be y metres
200 + y
Then, = 90 sec.
= 5 m/sec.
Let the length of 2nd train be x metres. ( 36 × )
5
18
(125 + x) ⇒ 200 + y = 900
Then, = 60 sec.
5 ⇒ y = 700 metres
⇒ 125 + x = 300 Length of platform = 700 metres.
⇒ x = 175 24. (B) Let the length of the train be x metres and its
∴ Length of second train speed be y metres/sec.
x x
= 175 metres.
y = 15 ⇒ y = 15
Then, …(1)
21. (C) Let the length of another train
(x + 100) x
= x metres Now, = ⇒ x = 150 m.
25 15
Their relative speed = (62 + 40) km/hr 25. (B) Let the speed of the train be x km/hr
( )
= 102 ×
5
18
Relative speed of the train
= (x + 6) km/hr
= ( )
85
3
m/sec
[
= (x + 6) × ]5
18
m/sec
250 + x 150 (x + 6) × 5
˙·˙ = 18 =
( ) 85
3
˙·˙

6
5x + 30 = 450
18

3 (250 + x) ∴ x = 84 km/hr
⇒ = 18
85 26. (D) ˙·˙ Relative speed of the train
⇒ 250 + x = 510 = (40 – 25) km/hr
⇒ x = 260
∴ Length of another train ( )
= 15 ×
5
18
= 260 metres. = ( )25
6
m/sec
22. (C) Speed of first train =( ) 100
10
= 10 m/sec.
∴ Length of the train = 48 ×( ) 25
Let the speed of second train be x m/sec 6
= 200 metres.
˙·˙ Relative speed = (10 + x) m/sec


200
10 + x
= 8
27. (B) Relative speed of both trains = ( 10 )
150 + 100

= 25 m/sec
⇒ 200 = 80 + 8x
⇒ x = 15 (
= 25 × )18
5
∴ Speed of second train = 15 m/sec = 90 km/hr

Quantitative Aptitude Test | 235


∴ Speed of second train = (90 – 30) 34. (C) Relative speed of both trains
= 60 km/hr. = (32 + 40) = 72 km/hr
28. (A) Speed of train = ( )
120
5
= 24 m/sec (
= 72 × )5
18
= 20 m/sec

Time taken to cross the platform Distance covered in crossing each other
= ( 120 + 180
) 1
= 12 sec.
= (132 + 108) = 240 m
24 2
∴ Required time = ( ) 240
= 12 sec.
29. (
(D) Speed of train = 36 × )
5
18
= 10 m/sec.
20
35. (D) Distance covered in 72 sec.
Let the length of the train be x metres = (300 + 900) m

Then,
x
10
= 10 ⇒ x = 100 metres ∴ Speed = ( ) 1200
72
∴ Time taken to cross the platform
=( ) 50
m/sec.
= ( 100 + 55
) 1
= 15 sec
3

30.
10 2
(C) Let the length of train = x metres
=( ) 50 18
3 5
×

Speed of train = 45 × ( 5
) ( )
18
=
25
2
m/sec.
36. (B)
= 60 km/hr.

Distance covered in crossing the platform 5 25


37. (C) ˙·˙ Speed = 30 km/hr = 30 × = m/sec.
= (x + 100) m 18 3
2 ∴ Length of the train
∴ (x + 100) × = 60 25 27 225 1
25 = × = = 112 metres.
⇒ 2x + 200 = 1500 ⇒ x = 650 3 2 2 2
Now, time taken to cross the pole 38. (A) Speed of the train
36 × 5
(
= 650 ×
2
25) = 52 sec. = 36 km/hr =
18
= 10 m/sec
110
31. (A) Let the length of each train = x metres ∴ Required time = = 11 sec
10
Relative speed = (46 – 36) = 10 km/hr. 39. (A) 40. (D)
(
= 10 × )( )
5
18
=
25
9
m/sec 41. (C) Speed of the train
160 18
Distance covered in crossing = × = 32 km/hr.
18 5
= (x + x) = 2x metres 42. (B)
9 25 × 36 43. (B) Distance covered by the train in crossing the
⇒ 2x × = 36 ∴ x = = 50 metres.
25 2×9 platform
45 × 30
32. (B) Relative speed of both trains
= (50 – 30) = 20 km/hr
= ( 3600 ) 3
= km = 375 metres
8
∴ Length of train = (375 – 100) = 275 metres
= 20 ×( ) 5
18 ∴ Time taken to cross the pole

= ( )
50
9
m/sec. (
= 275 ÷
375
30 )
Let the length of the faster train be x (
= 275 ×
30
375 ) = 30 sec
9 44. (C)
Then, x× = 18
50 45. (D) Speed of the train
18 × 50 = 45 km/hr
⇒ x = = 100 metres 5 25
9 = 45 × = m/sec
18 2

⎯√⎯⎯⎯⎯⎯⎯⎯⎯⎯
A’s speed Time taken by B to reach X Let the length of the bridge be x metres
33. (B) ˙·˙ =
B’s speed Time taken by A to reach Y (x + 130)
˙·˙ = 30
⎯⎯⎯

45 10 5 5 25
⇒ = × =
B’s speed 3 24 6 2
45 × 6 ⇒ x + 130 = 15 × 25
∴ B’s speed = (
5 )
km/hr = 54 km/hr. ∴ x = 375 – 130 = 245 metres.

Quantitative Aptitude Test | 236


46. (A) Speed of the column of men 55. (A)
50 × 75 56. (A) Speed of the first train
= = 90 km/hr
100 × 60
5 90 × 5
= m/sec =
8 18
Let the length of the street be x metres = 25 m/sec
Let the length of the bridge be x metre and that of the
60 × 60 × 5
˙·˙ (x + 250) = 8
train be y metre.
∴ (x + y) = 25 × 36 = 900 …(1)
= 2250 metres
Again Speed of the second train
⇒ x = (2250 – 250) m
= 45 km/hr
∴ x = 2000 m = 2 km. 45 × 5
47. (A) Speed of the train =
18
(50 + 500) 25
= = m/sec.
30 2
55 ∴ Time taken by the second train to cross the bridge
= m/sec
3 x + (y – 100)
=
55 18 25
= ×
3 5 2
= 66 km/hr (x + y – 100) × 2
=
48. (C) ˙·˙ Speed of the train = 72 km/hr 25
Now, putting the value of (x + y) from eqn. (1)
(
= 72 × )
5
18
= 20 m/sec. we have
Sum of the length of the train and tunnel (900 – 100) × 2 800 × 2
=
25 25
= (540 + 160) = 700 metres
= 64 sec
∴ Required time taken to pass the tunnel
57. (B) ˙·˙ Relative speed of the train
= Time taken to cover 700 metres at 20 m/sec
= 60 + 6 = 66 km/hr
= ( )
700
20
= 35 sec =
66 × 5
18
49. (A) 55
= m/sec
50. (B) Required Speed = ( )
240 + 120
24
∴ Required time =
3
110 × 3
= 6 sec
= 15 m/sec 55
18 58. (A)
= 15 × km/hr 59. (A) Relative speed of the train
5
= 54 km/hr = (32 + 40)
= 72 km/hr.
51. (A) ˙·˙ Speed of the train =( ) 150
12
m/sec
(
= 72 × )
5
18
Required time taken to cross the bridge
= 20 m/sec.
[
= (150 + 250) × ] 12
150
= 32 sec Sum of lengths of the trains
= (132 + 108) = 240 m
52. (C) 53. (D) Time taken by the trains in passing each other
54. (D) Let the length of tunnel be x metres 240
=
(
Speed of the train = 72 × ) 5
18
20
= 12 sec
= 20 m/sec 60. (B) Let the speed of the train be x km/hr
Time taken by the train to cover (700 + x) m in 1 ∴ Relative speed of the train = (x + 5) km/hr
minute 5
= (x + 5) × m/sec
18
˙·˙( 20 )
700 + x
sec
Now,
(x + 5)× 5 36
× = 100
= 1 minute 18 5
700 + x ⇒ 2x + 10 = 100
⇒ = 60 ∴ x = 45
20
∴ x = 500 m ∴ Required speed of the train = 45 km/hr

Quantitative Aptitude Test | 237


Pipes and Cisterns
Intel—A pipe connected with a minutes respectively. If both the Solution : Work done by the
tank (or a cistern or a reservoir) is pipes are opened simultaneously, leak in 1 hour
called an inlet, if it fills it. after how much time B should be
Outlet—A pipe connected with closed so that the tank is full in 18
min. ?
(
=
1 1

15 20 ) =
1
60
a tank is called an outlet, if it empties
∴ Leak will empty the full tank
it. Solution : Let B be closed after
in 60 hours.
Formulae x minutes. Then part filled by (A + B)
in x min. + part filled by A in (18 – x) Q. 6. Two pipes A and B can
(i) If a pipe can fill a tank in x fill a tank in 36 hours and 45 hours
min. = 1
hours, then the part filled in 1 hour respectively. If both the pipes are
=x
1 ∴x ( 1
+
24 32 )1
+ (18 – x) ×
1
24
opened simultaneously, how much
time will be taken to fill the tank ?
(ii) If a pipe can empty a tank in = 1 Solution : Part filled by A alone
y hours, then the part of the full tank 7x 18 – x 1
or + = 1 in 1 hour =
emptied in 1 hour 96 24 36
1 or 7x + 4 (18 – x) = 96 Part filled by B alone in 1 hour
=y
1
∴ 3x = 24 =
(iii) If a pipe can fill a tank in x 45
or x= 8
hours and another pipe can empty the ∴ Part filled by (A + B) in 1 hour
full tank in y hours, then the net part So, B should be closed after 8
filled in 1 hour when both the pipes min. (
=
1
+
36 45)
1
=
9
=
180 20
1

( )
are opened = x – y
1 1 Q. 3. Two pipes A and B can
fill a cistern in 1 hour and 75 Exercise
minutes respectively. There is also
Examples an outlet C. If all the three pipes 1. A tank can be filled by one tap in
are opened together, the tank is full 20 min. and by another in 25
Q. 1. Two pipes A and B can in 50 min. How much time will be min. Both the taps are kept open
fill a tank in 36 min. and 45 min. taken by C to empty the full tank ? for 5 min. and then the second is
respectively. A waste pipe C can Solution : Work done by C in turned off. In how many minutes
empty the tank in 30 min. First A
and B are opened. After 7 min. C is
also opened. In how much time the
(1) min. = (1
+
1 1
60 75 50
– ) =
3
300
more is the tank completely
filled ?
1 1
tank is full ? = (A) 17 min. (B) 12 min.
100 2
Solution : Part filled in 7 min.
∴ C can empty the full tank in (C) 11 min. (D) 6 min.
=7× ( 1
+
36 45
1
) =
7
20
100 min.
Q. 4. Pipe A can fill a tank in
2. A cistern has two taps which fill
it in 12 min. and 15 min. respec-
Remaining part 20 hours while pipe B alone can fill tively. There is also a waste pipe
( )
= 1–
7
20
=
13
20
it in 30 hours and pipe C can empty
the full tank in 40 hours. If all the
in the cistern. When all the pipes
are opened the empty cistern is
pipes are opened together, how full in 20 min. How long will the
Part filled by (A + B + C) in 1
much time will be needed to make
min. = ( 1
+
1
36 45 30
– )
1
=
1
60
the tank full ?
waste pipe take to empty a full
cistern ?
Solution : Net part filled in 1 (A) 8 min. (B) 10 min.
1
Now, part is filled by (A + B
60
+ C) in 1 min.
hour = (
1
+
1 1
20 30 40
– ) =
7
120
(C) 12 min. (D) 16 min.
3. Two taps can separately fill a
120
13 ∴ The tank will be full in cistern in 10 minutes and 15
So, part will be filled by them 7
20 minutes respectively and when
1
60 × 13
in( 20 ) = 39 min. i.e., 17
7
hours. the waste pipe is open, they can
together fill it in 18 minutes. The
Q. 5. A pipe can fill a tank in
∴ Total time taken to fill the waste pipe can empty the full
15 hours. Due to a leak in the
tank = (39 + 7) min. = 46 min. cistern in—
bottom it is filled in 20 hours. If the
Q. 2. Two pipes A and B can tank is full, how much time will the (A) 7 minutes
fill a tank in 24 minutes and 32 leak take to empty it ? (B) 9 minutes

Quantitative Aptitude Test | 238


(C) 13 minutes 9. A tap can fill a cistern in 8 hours 10 hours, C is closed. The time
(D) 23 minutes and another can empty it in 16 taken to fill the tank is—
hours. If both the taps are opened 1
4. A tap can fill a tank in 16 mi- (A) 12 hours (B) 13 hours
simultaneously, the time (in 2
nutes and another can empty it in hours) to fill the tank is— (C) 16 hours (D) 18 hours
8 minutes. If the tank is already
half full and both the taps are (A) 8 (B) 10 15. There are two taps to fill a tank
opened together, the tank will (C) 16 (D) 24 while a third to empty it. When
be— the third tap is closed, they can
10. Pipes A and B can fill a tank in
(A) Filled in 12 min. fill the tank in 10 minutes and 12
10 hours and 15 hours respec-
(B) Emptied in 12 min. minutes respectively. If all the
tively. Both together can fill it
three taps be opened, the tank is
(C) Filled in 8 min. in—
filled in 15 minutes. If the first
(D) Emptied in 8 min. 1
(A) 12 hours two taps are closed, in what time
2
5. A tank is filled by a pipe A in 32 can the third tap empty the tank
minutes and pipe B in 36 (B) 6 hours when it is full ?
minutes. When full, it can be (C) 5 hours (A) 7 min.
emptied by a pipe C in 20 (D) None of these (B) 9 min. and 32 sec.
minutes. If all the three pipes are
11. A cistern has a leak which would (C) 8 min. and 34 sec.
opened simultaneously, half of
empty it in 8 hours. A tap is (D) 6 min.
the tank will be filled in—
turned on which admits 6 litres a
(A) 16 minutes 16. A pipe can fill a cistern in 12
minute into the cistern and it is
(B) 24 minutes minutes and another pipe can fill
now emptied in 12 hours. How it in 15 minutes, but a third pipe
(C) 48 minutes many litres does the cistern can empty it in 6 minutes. The
(D) None of these hold ? first two pipes are kept open for
6. A cistern can be filled by two (A) 7580 litres 5 minutes in the beginning and
pipes A and B in 4 hours and 6 (B) 7960 litres then the third pipe is also opened.
hours respectively. When full, In what time is the cistern emp-
(C) 8290 litres
the tank can be emptied by a tied ?
third pipe C in 8 hours. If all the (D) 8640 litres (A) 30 min. (B) 33 min.
taps be turned on at the same
12. Three pipes A, B and C can fill a 1
time, the cistern will be full in— (C) 37 min. (D) 45 min.
cistern in 6 hours. After working 2
(A) 3 hours 18 min. at it together for 2 hours, C is 17. A leak in the bottom of a tank
(B) 3 hours 26 min. closed and A and B can fill it in can empty the full tank in 6
(C) 3 hours 42 min. 7 hours. The time taken by C hours. An intel pipe fills water at
(D) 3 hours 48 min. alone to fill the cistern is— the rate of 4 litres per minute.
7. One tap can fill a cistern in 2 (A) 10 hours (B) 12 hours When the tank is full, the intel is
hours and another can empty the (C) 14 hours (D) 16 hours opened and due to the leak the
cistern in 3 hours. How long will tank is empty in 8 hours. The
13. If two pipes function simultane- capacity of the tank is—
they take to fill the cistern if
ously, the reservoir will be filled
both the taps are opened ? (A) 5260 litres
in 12 hours. One pipe fills the
(A) 5 hours (B) 6 hours (B) 5760 litres
reservoir 10 hours faster than the
(C) 7 hours (D) 8 hours other. How many hours does the (C) 5846 litres
8. A pipe can fill a tank in x hours faster pipe take to fill the reser- (D) 6970 litres
and another can empty it in y voir ?
18. Two pipes X and Y can fill a
hours. They can together fill it in (A) 25 hours (B) 28 hours cistern in 24 min. and 32 min.
(y > x)— (C) 30 hours (D) 35 hours respectively. If both the pipes are
(A) (x – y) hours opened together, then after how
14. Two pipes A and B can fill a
(B) (y – x) hours much time Y should be closed so
tank in 15 hours and 20 hours
xy that the tank is full in 18 minu-
(C) hours respectively while a third pipe C
(x – y) tes ?
can empty the full tank in 25
xy hours. All the three pipes are (A) 6 min. (B) 8 min.
(D) hours
(y – x) opened in the beginning. After (C) 10 min. (D) 12 min.

Quantitative Aptitude Test | 239


19. A cistern is normally filled in 8 24. A cistern is normally filled in 8 29. A cistern can be filled by two
hours but takes two hours longer hrs. but takes 2 hrs. longer to fill pipes in 30 and 40 minutes
to fill because of a leak in its because of a leak in its bottom. If respectively. Both the pipes were
bottom. If the cistern is full, the the cistern is full, the leak will opened at once, but after some
leak will empty it in— empty it in— time the first was shut up and the
(A) 16 hrs. (B) 20 hrs. (A) 16 hrs. (B) 40 hrs. cistern was filled in 10 minutes
(C) 25 hrs. (D) 20 hrs. more. How long after the pipes
(C) 25 hrs. (D) 40 hrs.
had been opened was the first
25. A cistern can be filled by two
2 pipe shut up ?
20. A water tank is th full pipe A pipes. A and B in 12 minutes and
5 90
can fill the tank in 10 minutes 14 minutes respectively and can (A) minutes
11
and the pipe B can empty it in 6 be emptied by a third pipe C in 8
90
minutes. If both the pipes are minutes. If all the taps be turned (B) minutes
7
open, how long will it take to on at the same moment, what part
90
empty or fill the tank comple- of cistern will remain unfilled at (C) minutes
13
tely ? the end of 7 minutes ?
45
(A) 6 minutes to empty 5 19 (D) minutes
(A) (B) 2
24 24
(B) 6 minutes to fill
7 17 30. Three taps A, B and C can fill a
(C) 9 minutes to empty (C) (D) cistern in 10, 15 and 20 minutes
24 24
(D) 9 minutes to fill respectively. They are all turned
26. A cistern has 3 pipes A, B and C,
A and B can fill it in 2 and 3 on at once, but after 3 minutes C
21. A pipe can empty a tank in 12
hours respectively. C is a waste is turned off. How many minutes
minutes and another pipe can
pipe. If all the 3 pipes be opened longer will A and B take to fill
empty it in 16 minutes. If both
7 the cistern ?
the pipes are opened simultane- at once, of the cistern will be
24 (A) 2 min.
ously, find the time in which a filled up in 30 minutes. In what (B) 2 min. 6 sec.
full tank is emptied— time can C empty the full cis- (C) 1 min. 6 sec.
(A) 6 minutes tern ?
(D) 3 min. 8 sec.
1 (A) 3 hours (B) 4 hours
(B) 6 minutes 31. Three taps A, B and C can fill a
7 (C) 5 hours (D) 6 hours
cistern in 10 min., 12 min. and
2 27. Two pipes A and B can fill a
(C) 6 minutes 15 min. respectively. They are
7 cistern in 20 minutes and 25 all turned on at once, but after
(D) None of these minutes respectively. Both are 1
opened together, but at the end 1 min. B and C are turned off.
22. There is a leak in the bottom of 2
of 5 minutes, B is turned off. How many minutes longer will
cistern. When the cistern is
thoroughly repaired. It would be How much longer will the cistern A take then to fill the cistern ?
1 take to fill ? 1 1
filled in 3 hours. It now takes (A) 6 min. (B) 7 min.
2 (A) 16 minutes 4 4
half an hour longer. If the cistern (B) 18 minutes 3 3
(C) 6 min. (D) 8 min.
is full, how long would the leak (C) 11 minutes 4 4
take to empty the cistern ? (D) None of these 32. A cistern has a leak which would
(A) 28 hours (B) 27 hours 28. Two pipes, P and Q can fill a empty it in 15 hours. A tap is
(C) 32 hours (D) 24 hours cistern in 12 and 15 minutes turned on which admits 2 litres
respectively. Both are opened per hour into the cistern and it is
23. Top A can fill a water tank in 25 together, but at the end of 3 now emptied in 10 hours. How
minutes, tap B can fill the same minutes the first is turned off. many litres does the cistern
tank in 40 minutes and tap C can How much longer will the cistern hold ?
empty the tank in 30 minutes. If take to fill ? (A) 50 litres (B) 60 litres
all the three taps are opened 1
(A) 8 minutes (C) 45 litres (D) 360 litres
together, in how many minutes 4
will the tank be completely filled 33. A cistern can be filled by one of
1 two pipes in 30 minutes and by
up or emptied ? (B) 11 minutes
4 the other in 36 minutes. Both
2 5 3 pipes are opened together for a
(A) 3 (B) 15 (C) 7 minutes
13 13 4 certain time but being particu-
2 11 3 5
(C) 8 (D) 31 (D) 8 minutes larly clogged only of the full
13 19 4 6

Quantitative Aptitude Test | 240


quantity of water flows through the start of A, C also opened. (C) B fills in an hour and the
9 Find the time in which the cistern leak empties in an hour
the former and only through
10 is just full— (D) Data inadequate
the latter. The obstructions,
however, being suddenly remo- (A) 2 hrs. 38. A cistern which could be filled
1 (B) 4 hrs. in 9 hours takes one hour more
ved the cistern is filled in 15
2 to be filled owing to a leak in its
(C) 2 hrs. 52 min.
minutes from that moment. How bottom. If the cistern is full, in
long was it before the full flow (D) 4 hrs. 52 min. what time will the leak empty
of water began ? it ?
36. A, B, C are pipes attached to a
(A) 1 minute cistern. A and B can fill it in 20 (A) 80 hours (B) 85 hours
(B) 2 minute and 30 minutes respectively. (C) 90 hours (D) 95 hours
1 While C can empty it in 15
(C) 2 minute 39. A tap can fill a cistern in 8 hours
2 minutes. If A, B, C be kept open
and another can empty it in 16
successively for 1 minute each,
1 hours. If both the taps are
(D) 1 minute how soon will the cistern be
2 opened simultaneously, the time
filled ?
(in hours) to fill the tank is—
34. Two pipes A and B can fill a (A) 167 min. (B) 160 min. (A) 8 (B) 10
tank in 15 hours and 20 hours
respectively while a third pipe C (C) 166 min. (D) 164 min. (C) 16 (D) 24
can empty the full tank in 25 37. Pipe A fills the cistern in half an 40. A tap can fill a tank in 25
hours. All the three pipes are hour and pipe B in 40 minutes, minutes and another can empty it
opened in the beginning. After but owing to a crack in the in 50 minutes. Find whether the
10 hours C is closed. Find in bottom of the cistern it is found tank will be filled up or emptied
how much time will the tank be that pipe A now takes, 40 and in how many minutes ?
full ? minutes to fill the cistern. How (A) Tank is emptied in 20
(A) 12 hrs. (B) 8 hrs. long will B take now to fill it and minutes
(C) 10 hrs. (D) 14 hrs. how long will the crack take to (B) Tank is filled up in 25
empty it ?
35. Three pipes A, B and C can fill a minutes
cistern in 10 hours, 12 hours and (A) The leak empties in 1 hour (C) Tank is filled up in 20
15 hours respectively. First A and B fills in 2 hours minutes
was opened. After 1 hour, B was (B) B fills in an hour and the (D) Tank is emptied in 25
opened and after 2 hours from leak empties in 2 hours minutes

Answers with Hints


1. (C) Work done by both the taps in 5 min 3. (B) Work done by waste pipe in 1 min.

=5(201 + 251 ) = (5 ×1009 ) = 209 =


1
( )
+
10 15
1

1
18

= ( – )=
1 1 1
Remaining part = (1 – ) =
9 11
20 20 6 18 9
1 ∴ Waste pipe can empty the cistern in 9 min.
Now, part is filled in 1 min.
20 4. (D) Part emptied in 1 min.

So,
11
20
part will be filled in 11 min. = (1 1

8 16 )=
1
16
Hence, the tank will be full in 11 min. more. ∴ Time taken to empty the full tank = 16 min.
Hence, time taken to empty the half tank = 8 min.
2. (B) Work done by waste pipe in 1 min.
5. (D) Net filling in 1 min.
= (121 + 151 ) – 201 = ( 1
+
1 1
32 36 20
– ) =
13
1440
= ( – )=
3 1 1 ∴ Time taken to fill the tank
20 20 10
∴ Waste pipe can empty the cistern in 10 min. = ( )
1440
13
min.

Quantitative Aptitude Test | 241


Time taken to fill half of the tank 1
(A + B + C)’s 1 hour’s work =
= ( )1440
13 × 2
min.
6
C’s 1 hour’s work = (16 – 212 ) = 141
= ( )720
13
min. = 55
5
13
min.
Hence, C alone can fill the cistern in 14 hours.
6. (B) Net filling in 1 hour 13. (C) Suppose that one pipe takes x hours to fill the
reservoir. Then another pipe takes (x – 10) hours.
= ( 1 1 1
+ –
4 6 8 ) =
7
24

1 1 1
∴ Time taken to fill the cistern x + x – 10 = 12

= ( ) 24
7
hrs. = 3 hrs. 26 min.
or
12 (x – 10 + x) = x (x – 10)
x2 – 34x + 120 = 0
7. (B) Net filling in 1 hour or (x – 30) (x – 4) = 0
= ( ) 1 1

2 3
=
1
6

or
x = 30
x = 4
∴ Time taken to fill the cistern So, the faster pipe takes 30 hours to fill the reservoir.
= 6 hours 14. (A) Part filled in 10 hours
8. (D) Net filling in 1 hour

( ) ( )
1 1
= x – y = xy
y–x = 10 × ( 1
+
1 1
15 20 25
– ) =
23
30
Remaining part
∴ Time taken to fill the tank
= ( ) xy
hrs.
(
= 1–
23
30 ) =
7
30
y–x

9. (C) Part filled by intel in 1 hour =


1 Now, ( 1
+
15 20 )1
part is filled by A and B in 1 hr.
8
Part emptied by outlet in 1 hour =
1
16
7
30
part will be filled by them in( 60 7
×
7 30 )
= 2 hrs.
Net filling in 1 hour = ( 1 1

8 16 ) =
1
16
∴ Total time taken to fill the tank
= (10 + 2) hrs. = 12 hrs.
∴ Time taken to fill the tank = 16 hours.
15. (C) Part emptied by the third pipe in 1 min.
1
10. (B) Part filled by A in 1 hour =
10
1
( 1
+
10 12 )
1

1
15
=
7
60
Part filled by B in 1 hour = So, the full tank will be emptied by third pipe in
15
Part filled by (A + B) in 1 hour = ( 1
+
10 15 )
1 ( ) 60
7
min. = 8 min. 34 sec.

16. (D) Part filled in 5 min.


5 1
= =
30 6
∴ Both pipes together can fill the tank in 6 hours.
5× ( 1
+
12 15
1
) = 5× =
9 3
60 4
Part emptied in 1 min. (when all the pipes are
11. (D) Part filled in 1 hour = ( 1 1

8 12 ) =
1
24 opened)
∴ Time taken to fill the cistern
= 24 hours
1
= –
6 ( 1
+
12 15
1
) ( =
1 3

6 20 ) =
1
60
1
Water moved in 24 hours = (6 × 24 × 60) litres Now, part is emptied in 1 min.
60
= 8640 litres
Hence, the capacity of the cistern is 8640 litres.
3
4 (
part will be emptied in 60 × ) 3
4
= 45 min.
1 1 17. (B) Part filled by intel in 1 hour
12. (C) Part filled in 2 hours = 2 × =
6 3

Remaining part = 1 – ( ) 1
=
2 =( ) 1 1
6 8
– =
1
24
3 3 So, the intel can fill the tank in 24 hours
2 ∴ Capacity of the tank
(A + B)’s 7 hour’s work =
3 = Water that flows in 24 hours
= (4 × 24 × 60) litres
∴ (A + B)’s 1 hour’s work = ( )2 1
×
3 7
=
2
21 = 5760 litres

Quantitative Aptitude Test | 242


18. (B) Let Y be closed after x min. Now, applying the given rule we have,
2 ×3×x
Then, x ( 1
+
24 32
1
) + (18 – x)
1
24 3 ×x+2× x–2×3
=
12
7
= 1 or 42x = 60x – 72
7x 18 – x ∴ x = 4 hours.
⇒ + = 1
96 24
or 7x + 72 – 4x = 96. 27. (A) (
25 1 – )
t
20
= 5
∴ 3x = 24 ∴ t = 16 minutes
or x = 8 min.
19. (D) Work done by leak in 1 hour 28. (A) (
12 1 – )
t
25
= 3

(
=
1 1
)

8 10
=
1
40 ∴ t =
45
4
1
= 11 minutes
4
∴ The leak will empty the cistern in 40 hours. 1 1
20. (A) Time taken to fill or empty the whole tank ∴ Required answer = 11 – 3 = 8 minutes
4 4
6 × 10 29. (B) Let the first pipe be shut up after x minutes
= = – 15 minutes
6 – 10
Now, applying the above rule, we have
– ve sign shows that the tank will be emptied.
2
∴ th full of the tank will be emptied in
5
15 × 2
5
(
30 1 –
x + 10
40 ) = x

[Here t = (x + 10) minutes]


= 6 minutes
90
21. (D) Required answer or x = minutes
7
12 × 16 48 6
= = = 6 minutes. 10 × 15 × 20 60
12 + 16 7 7 30. (B) x = = minutes
10 × 15 + 10 × 20 + 15 × 20 13
22. (A) Here x = 3·5 hours and
Now, applying the given rule, we have
y = 3·5 + 0·5 = 4 hours 60
Now apply the given rule. ×y
13
= 20
23. (D) Required answer 60
y– +3
25 × 40 × 30 13
=
40 × 30 + 25 × 30 – 25 × 40 21
or y = = 2 min. 6 seconds.
600 11 10
= = 31 minutes
19 19 10 × 12 × 15
31. (A) x = = 4 min.
24. (B) Here x = 8 hrs. and y = 8 + 2 = 10 hrs. 10 × 12 + 12 × 15 + 10 × 15
Now, applying the given rule, we have the 1
B and C are turned off after 1 minutes
2
8 × 10
required answer = = 40 hrs. ∴ B and C together can fill a cistern in
10 – 8
12 × 15 20
25. (B) Time taken to fill the whole tank
12 × 14 × 8 168
( 12 + 15 3
= ) min.
= = minutes Now, applying the given rule, we have
14 × 8 + 12 × 8 – 12 × 14 5
5 4+y 20
∴ In 7 minutes ×7 =
168 3 3
y–4+
5 2
= part of the tank will be filled
24 25 1
∴ y = = 6 minutes
∴ Required answer 4 4
5 19 32. (B) Here w = 2 litres per hour
=1– = part.
24 24 15 × 10
∴ Required answer = × 2 = 60 litres.
7 1 15 – 10
26. (B) ˙·˙ of the cistern will be filled up in hr.
24 2 1
33. (A) Net filling in last 15 minutes
∴ The whole of the cistern will be filled up in 2

( 1 24 12
2 7
× )
=
7
hrs. = (
31 1
2 30 36
+ ) 1
=
341
360
Let the pipe C be empty the whole cistern in x hours Now, suppose they remained clogged for x minutes.

Quantitative Aptitude Test | 243


Net filling in these x minutes ∴ Total time taken to fill the cistern
= 4 hours 52 min.
= ( x 5 x 9
× +
30 6 36 10
× ) =
19x
360 36. (A) Work done in 3 minutes = ( 1
+
1
–)1
Remaining part 20 30 15
1
(
= 1–
19x
360 ) (=
360 – 19x
360 ) =
60
360 – 19x 341 55
= or x = 1. Clearly, part of cistern is filled in 3 × 55 or 165
360 360 60
min.
Hence, the pipes remained clogged for 1 minute.
34. (A) Tank filled in 10 hours Remaining part = 1 –( )
55
60
=
5
=
60 12
1

= 10 ( 1
+
1 1
15 20 25
– ) =
23
30 1
Now, part is filled by A in 1 min.
20
Remaining part = 1 – ( 23
30
=) 7
30 and ( 1 1

12 20 ) 1
i.e. part is filled by B in 1 min.
30
Work done by (A + B) in 1 hour
∴ Required time = (3 × 55 + 1 + 1) min.
= ( 1
+
15 20
1
)=
7
60 = 167 min.
7 37. (B) Let the leak empties it in x hours
Now, part is filled by (A + B) in hour
60 From the given rule, we have
7
∴ part will be filled by (A + B) in
30 (60 7
×
7 30 ) hrs. x × 30
x – 30
= 40
= 2 hours ∴ x = 120 minutes = 2 hours
Total time in which the tank is full Now, from the question, applying the rule, we have
= 10 + 2 = 12 hours. time taken by B to fill the tank when crack in the
35. (D) [(A’s 1 hour work) + (A + B)’s 1 hour work] bottom develops
1
= +
10 ( 1
+
10 12
1
) =
17
60 =
120 × 40
120 – 40
= 60 minutes

Remaining part = 1 –( 17
60 ) =
43
60 38.
= 1 hour
(C) Let the leak empty the full cistern in x hours
Now, (A + B + C)’s 1 hour work = (
1
+
1
10 12 15
+
1
) Now, applying the given rule
9×x
1 = 9+1=0
= x–9
4
1 or x = 90 hours.
part is filled by 3 pipes in 1 hour. 39. (C)
4
25 × 50
43
60 ( )
part will be filled by them in 4 ×
43
60
hrs. 40. (B) T =
50 – 25
= + 25 minutes

= 2 hours 52 min. + ve sign shows that tank is filled up in 25 minutes.

Quantitative Aptitude Test | 244


Alligation or Mixture
Alligation is the rule that enable us to find the Cost of 1 gm of 2nd metal = Rs. (9·20 – 6·70)
proportion in which the two or more ingredients at the = Rs. 2·50
given price must be mixed to produce a mixture at a
given price. Now, mean price of lump
Cost price of unit quantity of the mixture is called the
Mean price.
= Rs. ( )
87
18
per gm = Rs.
6( )
29

Rule of Alligation—If two ingredients are mixed in C. P. of 1 gm C. P. of 1 gm of


a ratio, then—
of 1st metal 2nd metal
Quantity of Cheaper (C. P. of dearer) – (Mean Price)
= (Rs. 6·70) (Rs. 2·50)
Quantity of Dearer (Mean Price) – (C. P. of Cheaper)
We represent it as under—
Mean price
C. P. of a unit quantity C. P. of unit quantity
of cheaper (c) of dearer (d) (Rs.
29
6 )
Mean Price
(m)
(146) (5630)
∴ By alligation rule
(d – m) (m – c)
Quantity of 1st metal
(Cheaper quantity) : (Dearer quantity) Quantity of 2nd metal
= (d – m) : (m – c) 14 56
= :
Examples 6 30
Q. 1. A container contains 80 kg. of milk. From = 5:4
this container, 8 kg of milk was taken out and replaced In 9 gm of mix. 2nd metal
by water. This process was further repeated two times. = 4 gm
How much milk is now contained by the container ? In 18 gm of mix. 2nd metal
Remarks—Amount of liquid left after n operations
when the container originally contains x units of liquid,
from which y units is taken out each time
(
=
4
9 )
× 18 gm = 8 gm.

Q. 3. Two vessels A and B contain milk and water


⎡ ⎤
n
is ⎢⎢ x 1 – x
( ) ⎥⎥⎦ units
y mixed in the ratio 5 : 2 and 8 : 5 respectively. Find the
⎣ ratio in which these mixtures are to be mixed to get a
new mixture containing milk and water in the ratio
Solution : Amount of milk left 9 : 4.
⎡⎢ 3 ⎤
80 ( ) ⎥⎥⎦ kg = 58·34 kg.
8
⎢⎣ 1– 80
Solution : Let the C. P. of milk be Re. 1 per litre
Milk in 1 litre mix in A
Q. 2. A lamp of two metals weighing 18 gm is 5
= litre
worth Rs. 87, but if their weights be interchanged, it 7
would be worth Rs. 78·60. If the price of one metal be Milk in 1 litre mix in B
Rs. 6·70 per gm. find the weight of the other metal in 8
the mixture. = litre
13
Solution : If one lump is mixed with another lump Milk in 1 litre mix. of this mix.
with the quantities of metals interchanged then the
mixture of the two lumps would contain 18 gm of first 9
= litre
metal and 18 gm of second metal and the price of the 13
mixture would be Rs. (87 + 78·60) or Rs. 165·60. C. P. of 1 litre mix. in A
∴ Cost of (18 gm of 1st metal + 18 gm of 2nd metal) 5
= Rs.
= Rs. 165·60 7
So, cost of (1 gm of 1st metal + 1 gm of 2nd metal) C. P. of 1 litre mix. in B
165·60 8
= Rs. = Rs. 9·20 = Rs.
18 13
(Cost of 1 gm. of 1st metal) + (Cost of 1 gm of 2nd 9
Mean price = Rs.
metal) = Rs. 9·20 13

Quantitative Aptitude Test | 245


C. P. of 1 litre C. P. of 1 litre Thus out of 7 hours in all, he took 4 hours to travel
mixture in A mixture in B on foot.
Distance covered on foot in 4 hours
( )
Rs.
5
7 (Rs.
8
13) = (4 × 8) km = 32 km
Q. 6. A man possessing Rs. 8400 lent a part of it at
2
Mean Price 8% simple interest and the remaining at 6 % simple
3
1
(
Rs.
9
13 ) interest. His total income after 1 years was Rs. 882.
2
Find the sum lent at different rates.
1
1 2 Solution : Total interest on Rs. 8400 for 1 years is
2
13 91 Rs. 882
∴ (Mix. in A) : (Mix. in B) 100 × 882 × 2
∴ Rate of interest =
1 2 8400 × 3
= :
13 91 = 7%
= 7:2 Rate % of first Rate % of second
Q. 4. A sum of Rs. 41 was divided among 50 boys
and girls. Each boy gets 90 paise and a girl 65 paise.
Find the number of boys and girls.
sum (8%) ( )
sum 6 %
2
3
Solution : Average money received by each
41 Average Rate
= Rs. = 82 Paise
50 (7%)
Sum received Sum received
by each boy each girl 1
1
(90 paise) (65 paise) 3
Now,
Average By alligation rule
Money given at 8% S. I.
(82 paise) 2
Money given at 6 % S. I.
3
17 8 1
= :1
By alligation rule 3
Ratio of boys and girls = 1:3
= 17 : 8.
Q. 5. A man travelled a distance of 80 km in 7
∴ Money lent at 8% = Rs.(8400 ×14)
hours partly on foot at the rate of 8 km per hour and = Rs. 2100
Money lent at 6 % = Rs. (8400 × )
partly on bicycle at 16 km per hour. Find the distance 2 3
travelled on foot. 3 4
= Rs. 6300.
Solution : Average distance travelled in 1 hr.
Q. 7. The average weekly salary per head of the
80 entire staff of a factory consisting of supervisors and
= km
7 the labourers is Rs. 60. The average salary per head of
Dist. covered in Dist. covered in the supervisors is Rs. 400 and that of the labourers is
Rs. 56. Given that the number of supervisors is 12.
1 hr. on foot 1 hr. on bicycle
Find the number of labourers in the factory.
(8 km) (16 km) Solution :
Average salary Average salary
Average in 1 hr. of labourers of supervisors
(80
7
km ) (Rs. 56) (Rs. 400)

Average salary of
32 24
entire staff
7 7
By alligation rule (Rs. 60)
Time taken on foot 32
= =4:3
Time taken by bicycle 24 340 4

Quantitative Aptitude Test | 246


By alligation rule Step II : Mix wheats of 1st and 2nd kind to obtain a
Number of labourers mixture worth of Rs. 1·41 per kg.
Number of supervisors C. P. of 1 kg wheat C. P. of 1 kg wheat
340 85 of 1st kind of 2nd kind
= =
4 1 (120 paise) (144 paise)
Thus, if the number of supervisors is 1, number of Mean price
labourers = 85 (141 paise)
∴ If the number of supervisors is 12, number of
labourers = 85 × 12 = 1020 3 21
Q. 8. A butler stole wine from a butt of sherry By alligation rule :
which contained 40% of spirit and he replaced,what (Quantity of 1st kind of wheat) 3 1
he had stolen by wine containing only 16% spirit. The = =
Quantity of 2nd kind of wheat 21 7
butt was then of 24% strength only. How much of the i.e.; they must be mixed in the ratio 1 : 7
butt did he steel ?
Quantity of 2nd kind of wheat
Solution : Thus,
Quantity of 3rd kind of wheat
Wine containing Wine containing Quantity of 2nd kind of wheat
=
(40% spirit) (16% spirit) Quantity of 1st kind of wheat
Quantity of 1st kind of wheat
×
Quantity of 3rd kind of wheat
Wine containing
24% spirit = ( ) 7 11
1 7
×

8 16 = ( ) 11
1
∴ By alligation rule ∴ Quantities of wheat of (1st kind : 2nd kind : 3rd
Wine with 40% spirit
Wine with 16% spirit
kind) (
= 1:7: )
11
7

= (11 : 77 : 7)
8 1 Q. 10. In what proportion must water be mixed
= =
16 2 2
with spirit to gain 16 % by selling it at cost price ?
i.e., they must be mixed in the ratio (1 : 2) 3
Solution : Let C. P. of spirit be Re. 1 per litre.
1
Thus of the butt of sherry was left and hence the Then, S. P. of 1 litre of mixture
3
2 2
butler drew out of the butt. = Re. 1, Gain = 16 %
3 3
C. P. of 1 litre of mixture
Q. 9. In what ratio must a person mix three kinds
100 × 3 × 1
of wheat costing his Rs. 1·20, Rs. 1·44 and Rs. 1·74 per
kg. So, that the mixture may be worth Rs. 1·41 per
= Rs.( 350 ) = Rs.
6
7
kg ? C. P. of 1 kg C. P. of 1 kg
Solution : Step I—Mix wheats of first and third kind water pure spirit
to get a mixture worth Rs. 1·41 per kg. (Re. 0) (Re. 1)
C. P. of 1 kg wheat C. P. of 1 kg wheat
of 1st kind of 3rd kind Mean price
(120 paise)
Mean price
(174 paise)
( )
Re.
6
7
(141 paise)
1 6
33 21 7 7
1
By alligation rule :
Quantity of water 7 1
Quantity of 1st kind of wheat 33 11 = =
= = Quantity of spirit 6 6
Quantity of 3rd kind of wheat 21 7 7
i.e.; they must be mixed in the ratio 11 : 7. or Ratio of water and spirit = 1 : 6.

Quantitative Aptitude Test | 247


Exercise 8. In what proportion must wheat at (C) Rs. 3500
Rs. 1·60 per kg be mixed with (D) None of these
1. A jar full of whisky contains wheat at Rs. 1·45 per kg. So that
40% of alcohol. A part of this 15. The ratio of milk and water in 66
the mixture be worth Rs. 1·54
whisky is replaced by another kg of adulterated milk is 5 : 1.
per kg ?
containing 19% alcohol and now Water is added to it to make the
the percentage of alcohol was (A) 2 : 3 (B) 3 : 2 ratio 5 : 3. The quantity of water
found to be 26. The quantity of (C) 3 : 4 (D) 4 : 3 added is—
whisky replaced is— 9. In a mixture of 60 litres, the ratio (A) 22 kg
2 1 of milk and water is 2 : 1. If the (B) 24·750 kg
(A) (B)
5 3 ratio of the milk and water is to (C) 16·500 kg
2 3 be 1 : 2, then the amount of water
(C) (D) to be further added is— (D) 20 kg
3 5
(A) 20 litres (B) 30 litres 16. Two vessels A and B contains
2. Kantilal mixes 80 kg of sugar (C) 40 litres (D) 60 litres milk and water mixed in the ratio
worth of Rs. 6·75 per kg with 5 : 3 and 2 : 3. When these
120 kg worth of Rs. 8 per kg. At 10. A can contains a mixture of two
mixtures are mixed to form a
what rate shall he sell the mixture liquids A and B in proportion
new mixture containing half
to gain 20% ? 7 : 5 when 9 litres of mixture are
milk and half water, they must
drawn off and the can is filled
(A) Rs. 7·50 (B) Rs. 9 be taken in the ratio—
with B, the proportion of A and
(C) Rs. 8·20 (D) Rs. 8·85 B becomes 7 : 9. How many (A) 2 : 5 (B) 3 : 5
3. A mixture of 20 kg of spirit and litres of liquid A was contained (C) 4 : 5 (D) 7 : 3
water contains 10% water. How by the can initially ? 17. In what ratio must a grocer mix
much water must be added to (A) 25 (B) 10 sugar at 72 paise per kg with
this mixture to raise the percent- (C) 20 (D) 21 sugar at 48 paise per kg. So that
age of water to 25% ? by selling the mixture at 63 paise
11. A sum of Rs. 41 was divided
(A) 4 kg (B) 5 kg 1
among 50 boys and girls. Each per kg he may gain of his out-
(C) 8 kg (D) 30 kg boy gets 90 paise and a girl 65 6
4. A merchant has 50 kg. of sugar paise. The number of boys is— lay ?
part of which he sells at 8% (A) 16 (B) 34 (A) 1 : 3 (B) 3 : 1
profit and the rest at 18% profit. (C) 14 (D) 36 (C) 2 : 3 (D) 3 : 2
He gains 14% on the whole. The
12. A dishonest milkman professes 18. Sugar at Rs. 15 per kg is mixed
quantity sold at 18% profit is—
to sell his milk at C. P. But he with sugar at Rs. 20 per kg in the
(A) 20 kg (B) 30 kg mixes it with water and thereby ratio 2 : 3. Find the price per kg
(C) 15 kg (D) 35 kg gains 25%. The percentage of of the mixture—
5. Rs. 1000 is lent out in two parts, water in the mixture is— (A) Rs. 18 (B) Rs. 16
one at 6% simple interest and the (A) 25% (C) Rs. 17 (D) Rs. 19
other at 8% simple interest. The (B) 20%
yearly income is Rs. 75. The (C) 4% 19. In what proportion should water
sum lent at 8% is— (D) None of these and wine at Rs. 22·50 a litre be
(A) Rs. 250 (B) Rs. 500 mixed to reduce the price to Rs.
13. 729 ml. of a mixture contains 18 a litre ?
(C) Rs. 750 (D) Rs. 600
milk and water in the ratio 7 : 2.
6. A grocer buys two kind of rice at (A) 1 : 4 (B) 4 : 1
How much more water is to be
Rs. 1·80 and Rs. 1·20 per kg added to get a new mixture (C) 2 : 3 (D) 3 : 2
respectively. In what proportion containing milk and water in 20. Currants at Rs. 50 per kg are
should these be mixed, so that by ratio 7 : 3 ? mixed with currants at Rs. 90 per
selling the mixture at Rs. 1·75 (A) 600 ml kg to make a mixture of 17 kg
per kg, 25% may be gained ? (B) 710 ml worth Rs. 70 per kg. How many
(A) 2 : 1 (B) 3 : 2 (C) 520 ml kilograms of each are taken ?
(C) 3 : 4 (D) 1 : 2 (D) None of these (A) 8 kg, 9 kg
7. 15 litres of a mixture contains 14. Some amount out of Rs. 7000 1
20% alcohol and the rest water. (B) 8 kg of each
was lent at 6% p.a. and the 2
If 3 litres of water be mixed in it, remaining at 4% p. a. If the total
the percentage of alcohol in the (C) 7 kg, 10 kg
simple interest from both the
new mixture will be— (D) None of these
fractions in 5 years was Rs.
2 1600, the sum lent at 6% p. a. 21. A person bought 60 quintals of
(A) 17 (B) 16
3 was— rice of two different sorts for Rs.
1 (A) Rs. 2000 4642·50. The better sort costs
(C) 18 (D) 15
2 (B) Rs. 5000 Rs. 80 per quintal and the worse

Quantitative Aptitude Test | 248


Rs. 75·50 per quintal. How many (C) 520 ml 32. Three glasses of capacity 2 litres,
quintals were there of each sort ? (D) None of these 5 litres and 9 litres contain
(A) 25 quintals, 35 quintals mixture of milk and water with
27. In a mixture of 60 litres, the ratio milk concentrations 90%, 80%
(B) 20 quintals, 40 quintals of milk and water is 2 : 1. If the
(C) 32 quintals, 28 quintals and 70% respectively. The con-
ratio of the milk and water is to tents of three glasses are emptied
(D) None of these be 1 : 2, then the amount of into a large vessel. Find the milk
22. A man has whisky worth Rs. 22 water to be further added is— concentration and ratio of milk
a litre and another lot worth Rs. (A) 20 litres (B) 30 litres to water in the resultant
18 a litre. Equal quantities of (C) 40 litres (D) 60 litres mixture—
these are mixed with water to (A) 121 : 39 (B) 131 : 49
obtain a mixture of 50 litres 28. A man buys milk at Rs. 5 a litre
and mixes it with water. By (C) 39 : 121 (D) 49 : 131
worth Rs. 16 a litre. Find how
much water the mixture con- selling the mixture at Rs. 4 a litre 33. How much water should be added
tains ? 1 1
he gains 12 per cent on his to 60 litres of milk at 1 litres for
(A) 5 litre (B) 10 litre 2 2
outlay. How much water did Rs. 10, so as to have a mixture
(C) 15 litre (D) 20 litre
each litre of the mixture con- 1
23. A petrol pump owner mixed tain ? worth Rs. 5 per litre ?
leaded and unleaded petrol in 3
32
such a way that the mixture con- (A) litre (A) 16 litres (B) 15 litres
45
tains 10% unleaded petrol. What (C) 18 litres (D) 20 litres
quantity of leaded petrol should 13
(B) litre 34. How much chicory at Rs. 24 a
be added to 1 litre mixtures, so 45
kg should be added to 15 kg of
that the percentage of unleaded 32
(C) litre tea at Rs. 60 a kg as to make the
petrol becomes 5% ? 13 mixture worth Rs. 39 a kg ?
(A) 1000 ml (B) 900 ml (D) None of these (A) 21 kg (B) 20 kg
(C) 1900 ml (D) 1800 ml 29. A milk seller pays Rs. 500 per (C) 27 kg (D) 18 kg
24. In a mixture of wheat and barley kilolitre for his milk. He adds
the wheat is 60%. To 400 water to it and sells the mixture 35. How many bananas at 5 for Re.
quintals of the mixture a quantity at 56 P a litre, thereby making 1·20 should be mixed with 300
of barley is added and then the altogether 40% profit. Find the bananas at 6 for Rs. 2·10, so that
1 proportion of water to milk which they should all be worth Rs. 3·60
wheat is 53 % of resulting a dozen ?
3 his customers receive ?
mixture. How many quintals of (A) 1 : 4 (B) 2 : 3 (A) 350 (B) 280
barley are added ? (C) 1 : 5 (D) 4 : 1 (C) 320 (D) 250
400
(A) quintals 30. Four vessels of equal sizes 36. A solution of sugar syrup has
7 15% sugar. Another solution has
contains mixture of spirit and
(B) 50 quintals 5% sugar. How many litre of the
water. The concentration of spirit
2 second solution must be added to
(C) 46 quintals in 4 vessels are 60%, 70%, 75%
3 and 80% respectively. If all the 20 litres of the first solution to
2 four mixtures are mixed, find in make a solution of 10% sugar ?
(D) 53 quintals
3 the resultant mixture the ratio of (A) 10 (B) 5
25. 15 litres of a mixture contains spirit to water— (C) 15 (D) 20
20% alcohol and the rest water. (A) 57 : 13 37. From a cask of wine, containing
If 3 litres of water be mixed in it, (B) 23 : 57 64 litres, 8 litres are drawn out
the percentage of alcohol in the
(C) 57 : 23 and the cask is filled up with
new mixture will be—
(D) None of these water. If the same process is
2 repeated a second, then a third
(A) 17 (B) 16
3 31. Two casks of 48 and 42 litres are time, what will be the number of
1 filled with mixture of wine and litres of wine left in the cask ?
(C) 18 (D) 15
2 water, the proportions in the two 1 3
casks being respectively 13 : 7 (A) 42 kg (B) 42 kg
26. 729 ml of a mixture contains 8 8
and 18 : 17. If the contents of the
milk and water in the ratio 7 : 2. 7 7
two casks be mixed and 20 litres (C) 48 kg (D) 42 kg
How much more water is to be
of water added to the whole what 8 8
added to get a new mixture con-
will be the proportion of wine to
taining milk and water in ratio 38. From a vessel filled with alcohol.
water in the result ?
7:3? 1
(A) 13 : 12 (B) 12 : 13 of its contents is removed and
(A) 600 ml 5
(B) 710 ml (C) 21 : 31 (D) 31 : 21 the vessel is then filled up with

Quantitative Aptitude Test | 249


water. If this be done 5 times in much wine is now left in the 43. In what ratio must a person mix
succession, what proportion of vessel ? three kinds of wheat costing him
the alcohol originally contained (A) 54 litres Rs. 1·20, Rs. 1·44 and Rs. 1·74
in the vessel will have been (B) 25 litres per kg, so that the mixture may
removed from it ? (C) 64 litres be worth Rs. 1·41 per kg ?
1024 (D) None of these (A) 11 : 77 : 7
(A)
3125 41. There are two vessels of equal (B) 7 : 11 : 77
2101 capacity, one full of milk and the
(B) (C) 11 : 7 : 77
3125 second one-third full of water.
The second vessel is then filled (D) None of these
1024
(C) up out of the first, the contents of
2101 44. Fresh fruit contains 72% water
the second are then poured back and dry fruits contains 20%
(D) None of these into the first till it is full and then water. How much dry fruit from
39. From a cask of wine containing again the contents of the first are 100 kg of fresh fruit can be
25 litres, 5 litres are withdrawn poured back into the second, till obtained ?
and the cask is filled with water. it is full. What is the proportion
of milk in the second vessel ? (A) 32 kg (B) 33 kg
The process is repeated a second
and then a third time. Find the 20 20 (C) 30 kg (D) 35 kg
(A) (B)
ratio of wine to water in the re- 37 27 45. In two alloys Copper and Zinc
sulting mixture— 37 27 are related in the ratios of 4 : 1
(C) (D)
(A) 64 : 61 (B) 61 : 64 20 20 and 1 : 3. 10 kg of Ist alloy 16 kg
(C) 51 : 54 (D) 46 : 61 42. Three lumps of gold, weighing of 2nd alloy and some of pure
respectively 6, 5, 4 g and of 15, copper are melted together. An
40. A vessel contains 125 litres of
1 alloy was obtained in which the
wine, 25 litres of wine was taken 14, 12 carats fineness are mixed
2 ratio of copper to zinc was 3 : 2.
out of the vessel and replaced by
together, what is the fineness of Find the weight of the new
water. Then 25 litres of mixture
the resulting compound ? alloy—
was withdrawn and again
replaced by water. The operation (A) 14 carats (B) 16 carats (A) 34 kg (B) 35 kg
was repeated for third time. How (C) 12 carats (D) 18 carats (C) 36 kg (D) 30 kg

Answers with Hints


1. (C) Using the method of alligation 4. (B)
Required ratio = 7 : 14 First part profit 2nd part profit
= 1:2 (8%) (18%)
2
∴ Required quantity =
3
2. (B) Total C. P. of 200 kg of mixture
Mean profit
= Rs. (80 × 6·75 + 120 × 8)
(14%)
= Rs. 1500
Average rate = Rs. 7·50 per kg
Required rate = 120% of Rs. 7·50 4 6
= Rs. 9 per kg.
Ratio of 1st and 2nd part
3. (A) In first mixture =4:6 = 2:3
Water = ( 10
100 )
× 20 kg Quantity sold at 18% = (50 ×35) kg
and Spirit = 18 kg = 30 kg
In second mixture 5. (C) Total interest = Rs. 75
100 × 75
75 kg spirit is contained in a mixture of 100 kg
∴ 18 kg spirit is contained in a mixture of
Average rate = (
1000 × 1
% )
1
(100
75 )
× 18 = 24 kg
= 7 %
∴ (Sum at 6%) : (Sum at 8%)
2

So, water to be added 1 3


= : =1:3
= (24 – 20) kg = 4 kg. 2 2

Quantitative Aptitude Test | 250


Hence, sum at 8% = Rs. (1000 ×34) 11. (B) Average money received by each
= Rs. (41/50)
= Rs. 750. = 82 paise
6. (D) S. P. of 1 kg mixture = Rs. 1·75, Gain = 25% Ratio of boys and girls
1·75 × 100
∴ Mean price = Rs. ( 125 ) = 17 : 8
∴ Number of boys
= Rs. 1·40
∴ (Dearer rice) : (Cheaper rice) (
= 50 ×
17
25)
= 20 : 40 = 34.
= 1 : 2. 12. (B) Let C. P. of 1 litre of milk be Re. 1
7. (B) Initially, the mixture contains 3 litre of alcohol Then S. P. of 1 litre of mixture = Rs. 1
and 12 litre of water afterwards, the mixture con- Gain = 25%
tains 3 litre of alcohol and 15 litre of water.

∴ Percentage of alcohol = ( 3
18 )
× 100 %
∴ C. P. of 1 litre of mixture = Rs. (100
125 )
×1

4
2 = Rs.
= 16 % 5
3
4 1
8. (B) ∴ Ratio of milk and water = :
5 5
C. P. of 1 kg of C. P. of 1 kg of = 4:1
dearer wheat cheaper wheat Hence, percentage of water in the mixture
100 × 1
(160 paise) (145 paise) =( 5 ) %

Mean price = 20%.


(154 paise) 13. (
(D) Milk = 729 × ) 7
9
= 567 ml

Water = (729 – 567) = 162 ml


9 6
567 7
∴ (Dearer wheat) : (Cheaper wheat) Now, =
162 + x 3
= 9:6 ⇒ x = 81.
= 3:2
9. (D) Ratio of milk and water in mixture of 60 litre
14. (A) Average annual rate = ( 1600 1000
7000
×
5) %


= 2:1
Quantity of milk = 40 litre
=( )32
7
%
Quantity of water = 20 litre ∴ (Amount at 6%) : (Amount at 4%)
If ratio of milk and water is to be 1 : 2, then in 40 4 10
litres of milk, water should be 80 litre = : = 2:5
7 7
∴ Quantity of water to be added
= 60 litre. Hence, (
Sum lent at 6% = Rs. 700 × )
2
7
10. (D) Let the can initially contain 7x litres and 5x litre = Rs. 2000.
of mixtures A and B respectively. Thus out of 12x 15. (A) In first mixture
litre of total mixture, 9 litre were taken out. 66 × 5
Quantity of A in mix. left ( )
Milk =
6
= 55 kg

(
= 7x –
9
12x )(
× 7x
28x – 21
4 )litre and
In second mixture
water = 11 kg

Quantity of B in mix. left

(
= 5x –
9
)
× 5x
If milk is 55 kg then water =( 3
5
× 55 )
12x = 33 kg
=( 4 )
20x – 15
litres ∴ Water to be added = 22 kg.
5 2
16. (C) Milk in A = of whole, Milk in B = of whole,
∴ ( 28x – 21 20x – 15
4
:
4 )
+ 9 : : (7 : 9) 8
1
5
Milk in mixture of A and B =
or x = 3. 2

Quantitative Aptitude Test | 251


˙.˙ By alligation rule Hence, the proportion of water to mixture
1 1 20 – 16
(Mix. in A) : (Mix. in B) = : =
10 8 16 – 0
= 4:5 = 1 : 4.

17. ( )
(A) 1 +
1
6
1
= 1 of the cost price of a kg of the
6
Quantity of water =
50
1+4
×1
mixture = 63 p = 10 litre.
∴ Cost price of a kg of the mixture 23. (A) Here we have to find the quantity of leaded
63 petrol.
= = 54 p
1 Hence, we have to make certain changes in the given
1
6 data. % of leaded petrol in the mixture
Now, applying the given formula, we have the = 100 – 10 = 90%
54 – 48 After addition of leaded petrol (that has to be
required answer = = 1 : 3.
72 – 54 calculated) percentage of leaded petrol becomes
20 – 2 2 (100 – 5) = 95%
18. (A) =
Z – 15 3
Now, applying the given theorem, we have the
∴ Z = Rs. 18 per kg.

19. (A) Required proportion =


20·50 – 18 required answer = ( )
95 – 90
100 – 95
1000 ml
18 – 0
= 1000 ml.
[Water worths Rs. 0 a litre]
24. (B) Here barley is added
4·50
= = 1 : 4. 1 2
18 Hence y = 100 – 53 = 46 , x = 100 – 60 = 40%
3 3
90 – 70
20. (B) Required ratio = Now, applying the given rule, we have the required
70 – 50
= 1:1 ⎡ 46 2 – 40 ⎤
⎢ 3 ⎥
= ⎢⎢ ⎥ × 400
17 1
Hence, = 8 kg of each are taken. answer ⎥
1+1 2 ⎢ 100 – 46 2⎥
21. (A) Per quintal cost of two different sorts of rice ⎣ 3⎦
4642·50 = 50 quintals.
=
60 25. (B) In the mixture, water is added
= Rs. 77·375 per quintal Hence, % of water in the mixture
70·50 – 77·375 = 100 – 20
Proportion =
77·375 – 80 = 80%
1·875 Now applying the given rule, we have the percentage
=
2·625 of water in the new mixture
= 5:7 ⎛ y – 80 ⎞
The quantity of better sort = 15 ⎜ 100 – y⎟
⎝ ⎠
60
= ×5 = 3
12
500
= 25 quintals ∴ y = %
6
and the quantity of worse sort
∴ Required answer i.e. % of alcohol in the new
60
= ×7 mixture
12
500 100
= 35 quintals. 100 – =
6 6
22. (B) Two lots of whisky having equal quantities are
mixed 50 2
= = 16 %.
3 3
Let the price of mixture of whisky be Rs. x per litre
18 – x 26. (D) Percentage of water in first mixture
∴ = 1 2
x –22 = × 100
∴ x = Rs. 20 a litre 2+7
Now this mixture is mixed with water and worth 200
= %
Rs. 16 a litre 9

Quantitative Aptitude Test | 252


Percentage of water in the second mixture

=
3
× 100
and Quantity of water = 20 + (4844 ++ 4231 × 31)
7+3 186
= + 20
= 30% 5
Now, applying the given rule, 286
=
⎡⎢ 30 – 200⎤⎥ 5
⎢ 9 ⎥ 264 286
Required answer = ⎢⎣ ⎥ × 729 ∴ Required ratio = :
100 – 30⎦ 5 5
= 81 ml. = 12 : 13
⎡ 200 – 100⎤ 90 80 70
⎢ 3 3 ⎥⎥
32. (A) Ratio are
10
= 9 : 1,
20
= 4 : 1, = 7 : 3
30
27. (D) 60 ⎢⎢ ⎥ = 60 litres.
⎢ 100 – 200⎥ 10 × 2 20
33. (B) Here x = = Rs. a litre
⎣ 3 ⎦ 3 3
4 Now, applying the given rule, we have the required
28. (B) Required ratio =
⎛ 20 – 16⎞
( )
(5 – 4) +
25 5
200 answer
⎜3 3⎟
= ⎜⎜ ⎟ × 60 = 15 litres
16 ⎟
=
32
= 32 : 13 ⎜⎝ 3 ⎟⎠
13
The quantity of water that the each litre of the mixture 34. (A) By alligation method :
13 13 Tea Chicori
contains = × 1 = litre. 60 24
32 + 13 45
500
29. (A) Here x = = 50 P, y = 56 P, P = 40% 39
1000
56 15
Ratio of milk to water = 21
40
(50 – 56) + × 50 ∴ Ratio of tea and chicori = 5 : 7
100
15
4 ∴ Added chicori = × 7 = 21 kg
= =4:1 5
1
35. (D)
∴ Required answer (i.e. ratio of water to milk)
Bananas at 6 Bananas at 5
= 1 : 4.
210 = 35 120 = 24
30. (C) Ratio of spirit to water in the different vessels 6 5
60 75 360 = 30
⇒ = 3 : 2, = 3 : 1,
40 25 12
70 80 5
= 7 : 3, = 4:1 6
30 20 300
Now, applying the given rule, we have the required ∴ Required answer = × 5 = 250
6
ratio
15 × 20 + 5 × m
= [ 3 7 3 4
+ + +
5 10 4 5 ][ :
2 3 1 1
+
5 10 4 5 ]
+ +
36. (D)


20 + m
= 10

m = 20 litres
12 + 14 + 15 + 16 6 + 6 + 5 + 4
= :

= 57 : 23.
20 20 37. (D) Required answer = 1 – ( ) 8 3
64
× 64

31. (B) Ratio of wine to water, when 20 litre of water are


not added
= ()
7 3
8
× 64 = 42 kg
7
8
13 × 48 18 × 42 7 × 48 17 × 42 38. (B) The alcohol now contained in the vessel
= [ 20
+
35 ][ :
20
+ ] 35 = 1–( ) ()
1 5
=
4 5 1024
=
= 264 : 186 = 44 : 31 5 5 3125
Now, 20 litres of water are added 1024 2101
∴ Required answer = 1 – =
3124 3125
48 + 42
Quantity of wine = × 44 39. (A) Quantity of a wine left in the cask
44 + 31

=
264
5
litres = 1–( ) ()1 3
5
=
4 3 64
5
=
125

Quantitative Aptitude Test | 253


Quantity of water left in the cask By alligation rule—
64 61 quantity of 1st kind of wheat 33 11
= 1– = = =
125 125 quantity of 3rd kind of wheat 21 7
64 i.e., they must be mixed in the ratio 11 : 7.
125 64 Step II—Mix wheat of 1st and 2nd kind to obtain a
∴ Required ratio = = = 64 : 61
61 61 mixture worth of Rs 1·41 per kg.
125 C.P. of 1 kg
C.P. of 1 kg
40. (C) Amount of wine left = 125 1 – ( 125)
25 3 wheat of 1st
kind 120 P Mean price
wheat of 2nd
kind 144 P
64 141 P
= 125 ×
125 21
3
= 64 litres. ∴ By alligation rule—
41. (B) Let M be the vessel containing milk and W the quantity of 1st kind of wheat 3 1
vessel containing water. = =
quantity of 2nd kind of wheat 21 7
First vessel Second vessel i.e., they must be mixed in the ratio 1 : 7.
1
1st operation 1 M W (quantity of 2nd kind of wheat)
3 Thus,
(quantity of 3rd kind of wheat)
1 1 2
2nd operation M W+ M quantity of 2nd kind of wheat
3 3 3 =
quantity of 1st kind of wheat
1
3rd operation M +
3
2 1
3 3( W+ M
2
3 ) ( 1 1
3 3 ) W+ M
2
3 ×
quantity of 1st kind of wheat
quantity of 3rd kind of wheat
4th operation [ 1 1
3 3
M+ (
2 1
3 3 )]W+ M
2
3 = ( 7 11
×
1 7 ) ( )=
11
1

[(1 1
3 3
2
W+ M +
3 ) { 2 1
3 3
M+ (
2 1
3 3 )}]
W+ M
2
3
∴ Quantities of wheat of (1st kind : 2nd kind : 3rd
kind)
Simplifying the quantity on the right hand side, we 7
get the proportions of water and milk in the second = 1:7: = 11 : 77 : 7
11
vessel.
44. (D) We are concerned with solid part of the fruit
[ 1
9
2
W+ M+
9
2 1
3 3{ 2
M+ W+ M
9
4
9 }] (pure portion). Assume x kg of dry fruit is obtained.
∴ Solid part in fresh fruit = Solid part in dry fruit
1 2 2 4 8
= W+ M+ M+ W+ M or 0·28 × 100 = 0·8 × x
9 9 9 27 27
or x = 35 kg
2 2 8 20
∴ Proportion of milk = M + M + M = M ∴ 35 kg of dry fruit can be obtained from 100 kg
9 9 27 27
fresh fruit.
20
∴ of the second vessel is milk. 45. (B) Here two alloys are mixed to form a third alloy,
27
hence quantity of only one of the ingredients in each
42. (A) Fineness of the compound of the alloy will be considered.
1 Here, pure copper is also added, hence, quantity of
6 × 15 + 5 × 14 + 4 × 12
2 copper in all the three alloy will be considered.
= carats
6+5×4 Let the amount of pure copper = x kg.
210
= or 14 carats ∴ Pure copper + copper in 1st alloy + copper in 2nd
15 alloy = copper in 3rd alloy.
43. (A) Step I—Mix wheats of first and third kind to get
4 1 3
a mixture worth Rs. 1·41 per kg. or x + × 10 + × 16 = (10 + 16 + x)
5 4 5
C.P. of 1 kg C.P. of 1 kg
wheat of 3rd 3
wheat of 1st or 12 + x = (26 + x)
Mean price type 174 P 5
kind 120 P
141 P or x = 9 kg
33 21 ∴ Weight of new alloy = 10 + 16 + 9 = 35 kg

Quantitative Aptitude Test | 254


Stock and Shares
Stock in order to meet the who subscribe in share are called Q. 3. Find the cash realized by
expenses of a certain plan, the shareholders. Sometimes the com- 1
pany asks its shareholders to pay selling Rs. 2400, 5 % stock at 5
Government of India sometimes 2
some money immediately and balance
raises a loan from the public at a
certain fixed rate of interest. Bonds after some period. The total money
raised immediately is called the paid
(
premium brokerage % .
1
4 )
or Promisory notes each of fixed
up capital. Parts of the profits divided Solution : By selling Rs. 100
value are issued for sale to the public.
amongst the shareholders are called stock, cash realized
If a man purchases a bond of
Rs. 100 at which 5% interest has
been fixed by the Government, then
dividends. The original value of a
share is called its nominal value. The (
= Rs. 105 –
1
4) = Rs.( )
419
4
price of a share in the market is By selling Rs. 2400 stock, cash
the holder of such a bond is said to called the market value. realized
have a Rs. 100 stock at 5%. Here Rs.
419 × 2400
100 is called the face value of the
stock. Usually a period is fixed for
Different kinds of shares—
There are two kinds of shares— = Rs.( 4 × 100 )
the repayment of the loan i.e., the
(i) Preference shares—On these = Rs. 2514.
stock matures at a fixed date only. shares a fixed rate to dividend is paid Q. 4. Find the cash required to
Now if a person holding a stock is in to their holders. Subject to profits of 1
need of the money before the date of purchase Rs. 1600, 8 % stock at
the company. 2
maturity of stock. He can sell the
bond or bonds to some other person
whereby the claim of interest is
(ii) Ordinary or Equity Shares
—After paying the dividends of the (
105 brokerage % .
1
2 )
transferred to that person. Stocks are preference shareholders, the equity Solution : Cash required for
sold and bought in the open market shareholders are paid the dividends purchasing Rs. 100 stock
through brokers at stocks exchanges. which depends upon the profit of the
The broker’s charge is usually called
‘brokerage’.
company ( )
= Rs. 105 +
1
2
Examples = Rs. (
2 )
Remarks—1. When stock is 211
purchased, brokerage is added to cost
price. Q. 1. What is the annual
income derived from Rs. 1800, 5% Cash required for purchasing
2. When stock is sold, brokerage stock at 104 ? Rs. 1600 stock
is subtracted from selling price. 211 × 1600
The selling price of a Rs. 100
Solution : Income from Rs. 100
stock = Rs. 5.
= Rs. ( 2 × 100 )
stock is said to be at par above par = Rs. 1688.
(or at a premium) and below par (or Income from Rs. 1800 stock
at a discount) according as the selling Q. 5. Find the cost of—
price of this stock is Rs. 100 exactly,
more than Rs. 100 and less than
= Rs. (1005 × 1800) 3
(i) Rs. 9100, 8 % stock at 92.
4
Rs. 100 respectively. = Rs. 90. 1
(ii) Rs. 8500, 9 % stock at 6
Remark—‘By a Rs. 700, 6% 1 2
stock at 97; we mean a stock whose Q. 2. How much 4 % stock at premium.
2
face value is Rs. 700, the market 95 can be purchased by investing (iii) Rs. 7200, 10% stock at 7
price of a Rs. 100 stock is Rs. 97 and
the annual interest on this stock is 5%
of the face value.
( 1
Rs. 1905, brokerage % ?
4 ) discount.
(iv) Rs. 6400, 8% stock at par
Shares—To start a big concern Solution : By investing
( 1
brokerage % .
8 )
or a business a large amount of
money is needed. This is usually
(
Rs. 95 +
1
4 ) stock purchased =
Solution :
beyond the capacity of one or two Rs. 100 (i) Cost of Rs. 100 stock
individuals. However, some persons By investing Rs. 1905, stock = Rs. 92.
together associate to form a company. purchased
The company issues a prospectus and Cost of Rs. 9100 stock
invites the public to subscribe. The
required capital divided into equal = Rs. (100 ×3814 × 1905) = Rs. ( 92
100
× 9100 )
small parts called shares, each of a
particular fixed value. The persons = Rs. 2000 = Rs. 8372

Quantitative Aptitude Test | 255


(ii) Cost of Rs. 100 stock Cost of 44 shares incomes. The investment in 5%
stock is—
= Rs. (100 + 6)
= Rs. 106.
= Rs. ( 121
4
× 44) (A) Rs. 1500
(B) Rs. 1280
= Rs. 1331
Cost of Rs. 8500 stock (C) Rs. 1434·84
∴Investment made (D) Rs. 1640
= Rs. (106
100
× 8500) = Rs. 1331 05. A 4% stock yields 5%. The
Now, face value of 1 share market value of the stock is—
= Rs. 9010.
= Rs. 25. (A) Rs. 125 (B) Rs. 80
(iii) Cost of Rs. 100 stock
∴Face value of 44 shares (C) Rs. 99 (D) Rs. 109
= Rs. (100 – 7) = Rs. 93
= Rs. (44 × 25) 06. By investing in a 6% stock at 96
Cost of Rs. 7200 stock an income of Rs. 100 is obtained
= Rs. 1100 by making an investment of—
= Rs. (100
93
× 7200) Now, dividend on Rs. 100 (A) Rs. 1600
11 (B) Rs. 1504
= Rs. 6696. = Rs.
2 (C) Rs. 1666·66
(iv) C.P. of Rs. 100 stock ∴Dividend on Rs. 1100 (D) Rs. 5760
= Rs. (100 + 18) = Rs. ( 11
2 × 100
× 1100 ) 07. The cash realized by selling a

801 = Rs. 60·50 1 1


= Rs. 5 % stock at 106 , brokerage
8 2 4
Also income on investment of 1
C.P. of Rs. 6400 stock Rs. 1331 = Rs. 60·50 being % is—
4
∴ Income on investment of
= Rs. (8018 ××100
6400
) Rs. 100 = Rs. (
60·50
× 100 ) (A) Rs. 105
1
2
1331
= Rs. 6408. 1
(B) Rs. 106
= 4·55% 2
Q. 6. Find the cost of 96 shares
(C) Rs. 106
3 Exercise
of Rs. 10 each at discount, bro- (D) None of these
4
1 1 1
kerage being per share. 01. A man invests in a 4 % stock at 08. The income derived from a 5 %
4 2 2
Solution : Cost of 1 share 96. The interest obtained by him stock at 95 is—
is— (A) Rs. 5·50
= Rs. [(10 – 34) + 14] (A) 4% (B) 4·5%
1
(B) Rs. 5
(C) Rs. 5·28
19 (C) 4·69% (D) %
= Rs. 2 (D) None of these
2
1 09. The cost price of a Rs. 100 stock
Cost of 96 shares 02. By investing Rs. 1100 in a 5 %
2 at 4 discount, when brokerage
stock one earns Rs. 77. The stock
= Rs. ( 19
2 )
× 96 is then quoted at—
(14)% is—
(A) Rs. 93 (B) Rs. 107
= Rs. 912.
4 3 (A) Rs. 96
Q. 7. Find the income derived (C) Rs. 78 (D) Rs. 97
7 4
from 44 shares of Rs. 25 each at 5 (B) Rs. (96 + 14)
( 1
premium brokerage per share ,
4 ) 03. To produce an annual income of
Rs. 500 in a 4% stock at 90, the
(C) Rs. (96 – )
1
the rate of dividend being 5%. Also amount of stock needed is— 4
find the rate of interest on the (A) Rs. 11250 (D) Rs. 100
investment. (B) Rs. 12500
10. A man invested Rs. 4455 in
Solution : Cost of 1 share (C) Rs. 18000 Rs. 10 shares quoted at Rs. 8·25.
(
= Rs. 25 + 5 +
3
4 ) (D) Rs. 20000
04. Rs. 2780 are invested party in
If the rate of dividend be 6% his
annual income is—
171 4% stock at 75 and 5% stock at (A) Rs. 267·30
= Rs.
4 80 to have equal amount of (B) Rs. 327·80

Quantitative Aptitude Test | 256


(C) Rs. 324 17. Which is the better stock, 5% at (C) Rs. 1433·25
(D) Rs. 103·70 1
143 or 3 % at 93 ? (D) None of these
2
11. A man bought 20 shares of 1
(A) 5% at 143 25. Find the cost of Rs. 15000, 5 %,
Rs. 50 at 5 discount the rate of 2
3 1
dividend being 4 %. The rate of (B) 3 % at 93 stock at 99—(Brokerage 1%)
4 2
(A) Rs. 15000
interest obtained is— (C) Both are equally good
(B) Rs. 12500
3 1 (D) None of these
(A) 4 % (B) 3 % (C) Rs. 13000
4 4 18. I want to purchase a 6% stock (D) None of these
(C) 5·28% (D) 4·95% which must yield 5% on my
capital. At what price must I buy 1
12. A man buys Rs. 20 shares paying 26. How much 4 % stock at 95
the stock ? 2
9% dividend. The man wants to can be purchased by investing
have an interest of 12% on his (A) Rs. 111 (B) Rs. 101
money. The market value of each
share must be—
(C) Rs. 83·33 (D) Rs. 120 ( 1
Rs. 1905 ? Brokerage %
4 )
19. A invested some money in 4% (A) Rs. 2000 (B) Rs. 2500
(A) Rs. 12 (B) Rs. 15 stock at 96. Now, B wants to
invest in an equally goods 5% (C) Rs. 2200 (D) Rs. 2350
stock. B must purchase a stock, 27. What income will be derived
(C) Rs. 18 (D) Rs. 21 worth of— from Rs. 3275 of 11% stock ?
3 (A) Rs. 120 (B) Rs. 124 (A) Rs. 360·50
13. By investing in 3 % stock at 96,
4 (C) Rs. 76·80 (D) Rs. 80 (B) Rs. 350·25
one earn Rs. 100. The invest-
ment made is— 20. How much stock at 105 can be (C) Rs. 360·25
purchased for Rs. 1433·25 ? (D) None of these
(A) Rs. 36000
(A) Rs. 1365 28. What income will be derived by
(B) Rs. 3600
(B) Rs. 1635 1
(C) Rs. 2560 investing Rs. 3000 in 9 per cent
(C) Rs. 1355 2
(D) Rs. 4800 stock at par ?
(D) None of these
14. A man invested Rs. 388 in a (A) Rs. 285
stock at 97 to obtain an income 21. Find the cost of Rs. 12600 (B) Rs. 825
of Rs. 22. The dividend from the 1
Railway stock at 150 (C) Rs. 385
stock is— 2
(A) 12%
1
(B) 3% ( 1
dividend 5 % —
2 ) (D) None of these
29. What annual income will be
(C) 5 % (D) 22·68% (A) Rs. 18963 derived by investing Rs. 1547
2
(B) Rs. 19863 in 13 per cent Railway stock at
15. Which is better investment, 4% 119 ?
stock at par with an income tax (C) Rs. 18933
(A) Rs. 189 (B) Rs. 179
at the rate of 5 paise per rupee (D) None of these
1 (C) Rs. 169 (D) Rs. 159
or 4 % stock at 110 free from 22. How much stock can be purcha-
2 30. Find what sum of money 1 must
income tax ? sed for Rs. 7350 at 105 ?
invest in a 10 per cent stock at
(A) 4% at par with income tax (A) Rs. 7500 (B) Rs. 7000 102 to obtain an income of
(C) Rs. 7200 (D) Rs. 6800 Rs. 400 per year—
1
(B) 9 % at 110 (A) Rs. 4800 (B) Rs. 8040
2 23. How much stock can be pur-
(C) 5% chased for Rs. 794·50 at 112·5 ? (C) Rs. 4080 (D) Rs. 8400
(Brokerage 1%)
(D) 25% 31. What income will be derived by
(A) Rs. 650 (B) Rs. 485 investing Rs. 1900 in 8 per cent
16. A man invest some money party stock at 5 discount ?
(C) Rs. 706 (D) Rs. 700
in 3% stock at 96 and party in
4% stock at 120. To get equal (A) Rs. 160 (B) Rs. 150
24. How much must 1 pay for
dividends from both, he must Rs. 1365 stock at 104 ? (C) Rs. 100 (D) Rs. 180
invest the money in the ratio ? (Brokerage 1%)
32. What rate of interest is obtained
(A) 16 : 15 (B) 3 : 4 (A) Rs. 1433·50 1
from investing in 8 per cent
(C) 4 : 5 (D) 3 : 5 (B) Rs. 1344·25 2

Quantitative Aptitude Test | 257


stock when the quoted price is 39. Find the annual income derived (C) Both (A) and (B) are equal
6·5 per cent below par ? 1
by investing Rs. 770 in 4 % (D) Can’t say
1 1 2
(A) 8 % (B) 9 %
11
1
(C) 11 %
11
1
(D) 10 %
(
stock at 96— Brokerage %
1
4 ) 45. Which is the better investment ?
1
(i) 8 % stock at 80 or
9 9 (A) Rs. 56 (B) Rs. 46 4
33. What rate % is obtained by (C) Rs. 39 (D) Rs. 36 (ii) 9% stock at 10 discount ?
investing in 7% stock at 5 (A) Ist investment is more
40. Find the cost of 96 shares of
( 1
discount ? Brokerage %
4 ) 3
Rs. 10 each at discount broke-
4
profitable
(B) IInd investment is more
(A) 7·35% (B) 7·55% 1 profitable
(C) 7·05% (D) 8% rage being per share—
4 (C) Both (A) and (B) are equal
34. What rate of interest is obtained (A) Rs. 912 (D) Can’t say
1
from investing in 9 per cent at (B) Rs. 812
2 46. Which is the better investment ?
par ? (C) Rs. 712
1
1 (D) None of these (i) 14 % stock at 5 below par or
(A) 9 % 4
2
41. Find the income derived from 44 3
1 (ii) 15 % stock at 5 premium ?
(B) 8 % shares of Rs. 25 each at 5 pre- 4
2
(C) 18 %
2 ( 1
mium brokerage per share ,
4 ) (A) Ist investment is more
profitable
3 the rate of dividend being 5%.
(D) None of these Also find the rate of interest in (B) IInd investment is more
the investment— profitable
35. What rate of interest is obtained
(A) Rs. 60·5, 4·55% (C) Both (A) and (B) are equal
1
from investing in 9 per cent (D) Can’t say
2 (B) Rs. 60, 5%
stock. When the quoted price is
(C) Rs. 80·5, 5·55% 47. How much money is obtained
14 per cent above par ? from the sale of Rs. 30000 stock
1 (D) None of these
(A) 8 %
3 42. Find the purchase cost of 66 (
at 93 ? Brokerage 1 %
1
2 )
2 shares of Rs. 35 each at 10
(B) 8 % (A) Rs. 24750
3 premium, brokerage being 1%
1 per share— (B) Rs. 37450
(C) 9 %
3 (A) Rs. 3630 (B) Rs. 3360 (C) Rs. 27450
(D) Data inadequate (C) Rs. 3063 (D) Rs. 3036 (D) None of these
36. What rate of interest is obtained 48. How much money is obtained
43. Which is the better investment ?
3 from the sale of Rs. 1700 stock
from investing in 12 per cent (i) 9 per cent stock at 91 or
4 1
stock when the price is at a (ii) 12 per cent stock at 121 ? at 106 ?
4
premium of 2 per cent ? (A) Ist investment is more (A) Rs. 1806·25
1 profitable
(A) 25% (B) 8 % (B) Rs. 1608·25
3
(B) IInd investment is more
1 2 (C) Rs. 1808·75
(C) 12 % (D) 11 % profitable
2 3 (D) None of these
(C) Both (A) and (B) are equal
37. What is the annual income (D) Can’t say 49. How much stock must be sold to
derived from Rs. 1800, 5% stock realize Rs. 7350 from a stock at
at 100 ? 44. Which is the better investment ? 105 ?
(A) Rs. 90 (B) Rs. 100 1 (A) Rs. 7500 (B) Rs. 6920
(i) 10 % stock at 90 or
(C) Rs. 110 (D) Rs. 95 2 (C) Rs. 7000 (D) Rs. 6400
38. What is the annual income by (ii) 11% stock at par ? 50. How much stock must be sold to
investing Rs. 3000 in 6% stock (A) Ist investment is more realize Rs. 8190 from a stock at
at 120 ? profitable 118 ? (Brokerage 1%)
(A) Rs. 150 (B) Rs. 100 (B) IInd investment is more (A) Rs. 7100 (B) Rs. 7050
(C) Rs. 200 (D) Rs. 250 profitable (C) Rs. 6850 (D) Rs. 7000

Quantitative Aptitude Test | 258


Answers with Hints
9 11. (C) Face value = Rs. (50 × 20) = Rs. 1000
1. (C) On Rs. 96, he gets Rs. .
2 1000 × 19

(92 ××100
Dividend = Rs. ( = Rs. )
95

96 )
4 × 100 2
On Rs. 100, he gets = Rs.
Investment = Rs. (45 × 20) = Rs. 900
= 4·69%. 95 × 100
2. (C) To earn Rs. 77, investment Rate = Rs. (
2 × 900 ) = 5·28%
= Rs. 1100
Dividend on Rs. 20 = Rs. ( × 20) = Rs.
9 9
11 12. (B)
To earn Rs. , investment 100 5
2
Rs. 12 is an income on Rs. 100.
= Rs. ( 1100 11
77
×
2 ) = Rs. 78
4
7 9
∴ Rs. is an income on Rs.
5 (
100 9
×
12 5 )
3. (B) For an income of Rs. 4, stock needed
= Rs. 15
= Rs. 100
For an income of Rs. 500, stock needed 15
13. (C) For earning Rs. , investment
4
= Rs.( 100
4 )
× 500 = Rs. 12500 = Rs. 96
4. (B) Let the investment in 4% stock be Rs. x. For earning Rs. 100, investment
96 × 4
Then, investment in 5% stock
= Rs. (2780 – x)
= Rs. (
15
× 100 )
Income from 4% stock = Rs. 2560
14. (C) When investment is Rs. 388, income
= Rs. ( 4
75
×x) = Rs. 22
Income from 5% stock When investment is Rs. 97, income

= Rs. [( 5
80
× (2780 – x) )] = Rs.
22
388 (
× 97 )
4x 2780 – x = Rs. 5·50
∴ = or = 1500
75 16 1
So, investment in 5% stock ∴ Dividend on Rs. 100 stock = 5 %
2
= Rs. (2780 – 1500) = Rs. 1280 15. (B) Let investment in each case be Rs. (100 × 110)
5. (B) For an income of Rs. 5, investment = Rs. 100. Gross income from 4% stock
For an income of Rs. 4, investment
= Rs. ( 100
5 )
× 4 = Rs. 80
= Rs. ( 4
100 )
× 100 × 110 = Rs. 440

Net income from the stock


6. (A) For an income of Rs. 6, investment = Rs. 96.
For an income of Rs. 100, investment = Rs. (440 – 22) = Rs. 418

= Rs. ( 96
6 )
× 100 = Rs. 1600
1
Net income from 4 % stock
2
9 × 100 × 110
7. (C) Cash realized = Rs. 106 –( 1 1
4 4 ) = Rs. 106 = Rs. ( 2 × 110 )
= Rs. 450

1 1
8. (A) Income on Rs. 100 stock = Rs. 5 = Rs. 5.50 ∴ Better stock is 4 % at 110.
2 2
9. (
(B) C.P. = Rs. 96 +
1
4 )
. 16. (A) For an income of Re. 1 in 3% stock, investment

10. (C) Number of shares =


4455
8·25
= 540
= Rs.
96
3 ( )
= Rs. 32

For an income of Re. 1 in 4% stock investment


Face value = Rs. (540 × 10) = Rs. 5400

Income = Rs. ( 6
100
× 5400) = Rs.
120
4 ( )= Rs. 30

= Rs. 324 ∴ Ratio of investments = 32 : 30 = 16 : 15

Quantitative Aptitude Test | 259


17. (B) Let investment in each case be Rs. (143 × 93) 1900
31. (A) Required income = × 8 = Rs. 160
Income from 5% stock 100 – 5 + 0
[Here value of brokerage is 0]
= Rs. (5
143 )
× 143 × 93 = Rs. 465
32. (B) Required answer =
17
× 100
1 2(100 – 6·5)
Income from 3 % stock 17 1
2 = × 100 = 9 %
187 11
= Rs. ( 7
2 × 93 )
× 143 × 93 ) = Rs. 500·50
33. (A) Required answer =
7
× 100
100 – 5 + 0·25
1
∴ 3 % stock at 93 is better. = 7·349 = 7·35% (Approx.)
2
1
18. (D) For an income of Rs. 5, investment = Rs. 100 9
2
× 100
) 34. (A) Actual rate per cent =
For an income of Rs. 6, investment = Rs.
5(
100
×6 100 + 0 + 0
1
= 9 %
= Rs. 120 2
19. (A) For an income of Rs. 4, investment = Rs. 96 1
9
2
For an income of Rs. 5, investment = Rs.
96
4 (×5 ) 35. (A) Actual rate per cent =
100 + 14 + 0
× 100

25 1
= Rs. 120 = =8 %
3 3
1 105
20. (A) 1433 = amount of stock × 51
4 100 36. (C) Required answer = × 100
4 × (100 + 2) + 0
1
100 × 1433 25 1
4 = = 12 %
Amount of stock = = Rs. 1365 2 2
105
1800 × 5
301 37. (A) 100 = –0
21. (A) Required answer = 12600 × = Rs. 18963 x
200
1800 × 5
22. (B) or x = = 90
100
112·5 + 1 ∴ Required answer = Rs. 90
23. (D) Rs. 794·50 = × Amount of stock
100 3000 × 6
38. (A) 120 = –0
794·50 × 100 x
∴ Amount of stock = = Rs. 700
113·5 3000 × 6
∴ x = = Rs. 150
1365 × (104 + 1) 120
24. (C) Required answer = Rs.
100 9 9
770 × 770 ×
1365 × 105 2 1 2
= = Rs. 1433·25 39. (D) 96 = – or x = = Rs. 36
100 x 4 1
96 +
25. (A) 4
26. (A) We put market value i.e. 95 in place of (100 –
discount), in this case.
40. (
(A) Required answer = 96 10 – +
3 1
4 4 ) = Rs. 912

Required answer =
1905
95 +
1
× 100 = Rs. 2000 41. (A) (
Cost of shares = 44 25 + 5 +
3
4)
4 =
Rs. 1331
11 ∴ Investment made = Rs. 1331
27. (C) Income = 3275 × = Rs. 360·25
100 Now, face value of 1 share =
Rs. 25
19
28. (A) Required answer = 3000 × = Rs. 285 ∴ Face value of 44 shares =Rs. (44 × 25)
200
=
Rs. 1100
1547
29. (C) Required answer = × 13 = Rs. 169 11
119 Now, dividend on Rs. 100 = Rs.
2
x
30. (C) 400 = × 10
102
102 × 400
∴ Dividend on Rs. 1100 = Rs. (
11
2 × 100
× 1100 )
or x = = 102 × 40 = Rs. 4080 = Rs. 60·50
10

Quantitative Aptitude Test | 260


Also, income on investment of Rs. 1331 46. (C)
= Rs. 60·50 57 100 – 5 = 95 = 1496·25
∴ Income on investment of Rs. 100 (i)
4
= Rs. (
60·50
1331
× 100 ) (ii)
63
4
100 + 5 = 105 = 1496·25

= 4·55%
Here (i) = (ii)
42. (D) Required answer = 66[35 + 10 + 1] = 46 × 66
∴ Both investments are equal.
= Rs. 3036.
43. (B) 3
93 –
(i) 9 91 = 1089 2
47. (C) Required answer = 30000 × = 183 × 150
100
(ii) 2 121 = 1092 = Rs. 27450
Here (ii) > (i), hence 2nd investment is more 425
profitable. 48. (A) Required answer = 1700 ×
4 × 100
44. (A)
(i) 21/2 90 = 1050 = Rs. 1806·25
105 + 0
49. (C) 7350 = Amount of stock ×
(ii) 11 100 = 990 100
7350 × 100
(i) > (ii), Ist is better investment. ∴ Amount of stock = = Rs. 7000
105
45. (A) 118 – 1
33 80 = 742·5 50. (D) 8190 = × Amount of stock
(i) 100
4
(ii) 9 100 – 10 = 90 = 720 8190 × 100
∴ Amount of stock = = Rs. 7000
Here (i) > (ii), hence (i) is the better investment. 117

Quantitative Aptitude Test | 261


True Discount
Suppose a sum say Rs. 136 is Examples annum is Rs. 26·25. Find the
due 3 years hence and the borrower amount of the bill.
wants to clear off the debt right now. Q. 1. Find the present worth of Solution : S.I. on Rs. 100 for 10
The question arises as to what money 1 1 months at 6% per annum
Rs. 9950 due 3 years hence at 7 %
should be paid now. Clearly, the 4 2
money which amounts to Rs. 136
after 3 years at a standard or agreed
per annum simple interest. Also
find the discount.
(
= Rs. 100 × ×
10 6
12 100 )
rate of interest must be paid now. Let Solution : P.W. = Rs. 5
the rate of interest in this case be 100 × Amount ∴ Amount
12% per annum simple interest. Then =
100 + (R × T) = Rs. (100 + 5) = Rs. 105
clearly, with this rate, Rs. 100 after 3
⎡ 100 × 9950 ⎤⎥
year will amount to Rs. 136. So = Rs. ⎢⎢ So, T.D.

clearly, the payment of Rs. 100 now
will clear off a debt of Rs. 136 due 3 ( 15
⎢⎣ 100 + 2 × 13 )
4 ⎦
⎥ = Rs. (105 – 100) = Rs. 5
100 × 9950 × 8
years hence at 12% per annum. The
sum due is called the amount and the = Rs. ( 995 ) If T.D. is Rs. 5, sum due
= Rs. 105
money paid now is called the present = Rs. 8000
value or present worth of the sum due If T.D. is Rs. 26·25, sum due
Also, T.D.
and the difference between the
amount and the present worth (Rs. 36 = [Amount – (P.W.)]
= Rs. (9950 – 8000)
= Rs. ( 105
5 )
× 26·25
in this case) is called the True
Discount or Equitable Discount or = Rs. 1950 = Rs. 551·25
Mathematical Discount. Q. 2. The true discount on a Q. 5. The true discount on
Thus, the present value or present certain sum of money due 3 years Rs. 1860 due after a certain time at
worth (p.w.) of a sum due at the end hence is Rs. 100 and the simple 5% is Rs. 60. Find the time after
of a given time is the money which interest on the same sum for the which it is due.
amounts to the sum due in that given same time and at the same rate is Solution : P.W. = (sum due) –
time and at a given rate. Rs. 120. Find the sum and the rate (T.D.) = Rs. (1860 – 60) = Rs. 1800.
per cent. Since T.D. in interest on P.W., so
The sum due is called the amount.
Solution : Rs. 60 is the simple interest on Rs.
The difference between the sum
due at the end of a given time and its S.I. × T.D. 1800 at 5% per annum.
Sum due =
present worth is called True Discount (S.I.) – (T.D.) ∴ Time
120 × 100
(T.D.).
Thus, T.D. = (interest on P.W.)
= Rs. ( 20 ) =( )100 × 60
1800 × 5
years
and Amount = (P.W. + T.D.) = Rs. 600
= ( × 12) months
2
Remark—Interest is reckoned 100 × 120 3
on present worth and discount is Rate =
600 × 3 = 8 months.
reckoned on amount.
2
Formulae—If rate = R% p.a. and = 6 %
Time = T years. Then— 3 Exercise
100 × (Amount) Q. 3. The true discount on 01. If Rs. 10 be allowed as true dis-
(i) P.W. = Rs. 2575 due 4 months hence is
[100 + (R × T)] count on a bill of Rs. 110 due at
Rs. 75. Find the rate per cent of the end of a certain time. Then
(P.W.) × R.T. interest.
(ii) T.D. = the discount allowed on the same
100
Solution : P.W. sum due at the end of double the
Amount × R × T time is—
(iii) T.D. = = Rs. (2575 – 75)
100 + (R × T) (A) Rs. 20 (B) Rs. 21, 81
(iv) S.I. on T.D. = (S.I.) – (T.D.) = Rs. 2500
(C) Rs. 22 (D) Rs. 18, 33
(S.I.) × (T.D.) ∴ S.I. on Rs. 2500 for 4 months
(v) Sum = [ ]
(S.I.) – (T.D.) is Rs. 75. 02. The simple interest and the true
discount on a certain sum for a
(vi) When the sum is put at Hence, rate given time and at a given rate are
compound interest, then 100 × 75 × 3 Rs. 25 and Rs. 20 respectively.
= = 9%
Amount 2500 × 1 The sum is—
P.W. = (A) Rs. 500 (B) Rs. 200
(
1+
R T
100 ) Q. 4. The true discount on a
bill due 10 months hence at 6% per (C) Rs. 250 (D) Rs. 100

Quantitative Aptitude Test | 262


03. Goods were bought for Rs. 600 10% per annum in this new mode 16. A man buys a watch for Rs. 195
and sold the same day for of payment— in cash and sells it for Rs. 220 at
Rs. 650·25 at a credit of 9 months (A) There is no gain or loss to a credit of 1 year. If the rate of
and still there was a gain of 2%. any one interest is 10%, the man—
The rate per cent is— (B) A gains Rs. 7·34 (A) Gains Rs. 15
1 1 (C) A loses Rs. 7·34 (B) Gains Rs. 3
(A) 6 % (B) 8 %
3 3 (D) A loses Rs. 1·66 (C) Gains Rs. 5
43 (D) Loses Rs. 5
(C) 8% (D) 7 % 10. Rs. 20 is the true discount on
61
Rs. 260 due after a certain time. 17. Find the present worth (PW) and
04. The interest on Rs. 750 for 2 What will be the true discount on the true discount reckoning 6%
years is equal to the true dis- the same sum due after half of per annum simple interest of
count on Rs. 810 for the same the former time, the rate of inter- Rs. 176 due in 20 months time—
time and at the same rate. The est being the same ? (A) Rs. 160, Rs. 16
rate per cent is— (A) Rs. 10 (B) Rs. 10·40 (B) Rs. 130, Rs. 46
1 1 (C) Rs. 15·20 (D) Rs. 13
(A) 4 % (B) 5 % (C) Rs. 150, Rs. 26
3 6
11. A owes B, Rs. 1120 payable (D) None of these
(C) 4% (D) 5% 2 years hence and B owes A, 1
05. A trader owes a merchant Rs. 1081·50 payable 6 months 18. A owes B Rs. 456·75 payable 4
2
Rs. 901 due 1 year’s hence. hence. If they decide to settle months hence and B owes A
However, the trader wants to their accounts forthwith by pay- Rs. 455·51 payable 3 months
settle the account after 3 months. ment of ready money and the hence. If they agree to settle
How much cash should he pay, rate of interest be 6% per annum, their account by a ready money
if rate of interest is 8% per then who should pay and how payment. What sum should be
annum— much ? paid over and to whom reckon-
(A) Rs. 870 (A) A Rs. 50 (B) B Rs. 50 ing the rate of true discount at 4
(B) Rs. 850 (C) A Rs. 70 (D) B Rs. 70 per cent per annum ?
(C) Rs. 828·92 12. A man purchased a cow for (A) Re. 1, A (B) Rs. 2, B
(D) Rs. 846·94 Rs. 300 and sold it the same day (C) Rs. 2, A (D) Re. 1, B
06. The present worth of Rs. 1404 for Rs. 360, allowing the buyer 19. Find the present worth of
due in two equal half yearly at the credit of 2 years. If the rate Rs. 264 due in 2 years reakoning
instalments— 1
of interest be 7 % per annum, simple interest at 5 per cent per
2 annum—
(A) Rs. 1325 (B) Rs. 1300
then the man has a gain of—
(C) Rs. 1350 (D) Rs. 1500 (A) Rs. 240 (B) Rs. 360
1 3
07. I want to sell my scooter. There (A) 4 % (B) 5 % (C) Rs. 540 (D) Rs. 260
2 7
are two offers. One at cash pay- (C) 6% (D) 5% 20. What is the present worth of
ment of Rs. 8100 and another at Rs. 272·61 due in 2 years 73
a credit of Rs. 8250 to be paid 13. The true discount on a bill due 1
after 6 months. If money being 10 months hence at 6% per days at 7 per cent ?
2
1 annum is Rs. 26·25. The amount
worth 6 % per annum simple in- (A) Rs. 334
4 of the bill is—
(B) Rs. 254
terest, which is the better offer ? (A) Rs. 1575
(C) Rs. 234
(A) Rs. 8100 in cash (B) Rs. 500
(D) None of these
(B) Rs. 8250 due 6 months (C) Rs. 650·25
hence 21. Find the present value of
(D) Rs. 551·25
(C) Both are equally good Rs. 1051·25 due a year hence at
14. The true discount on Rs. 2575 1
(D) Can’t be said 5 %—
due 4 months hence is Rs. 75. 8
08. If the true discount on a sum due The rate per cent is— (A) Rs. 1200 (B) Rs. 1000
2 years hence at 5% per annum (A) 6% (B) 8% (C) Rs. 1500 (D) Rs. 1050
be Rs. 75, then the sum due is— (C) 9% (D) 5%
(A) Rs. 750 (B) Rs. 825 22. What sum will discharge a debt
15. The true discount on Rs. 1860 of Rs. 5300 due a year and a half
(C) Rs. 875 (D) Rs. 800
due after a certain time at 5% is hence at 4% per annum ?
09. A has to pay Rs. 220 to B after 1 Rs. 60. The time after which it is (A) Rs. 5000 (B) Rs. 4500
year. B asks A to pay Rs. 110 in due is— (C) Rs. 4200 (D) Rs. 5250
cash and defer the payment of
Rs. 110 for 2 years. A agrees to (A) 6 months (B) 8 months 23. The true discount on a bill due 8
it. Counting the rate of interest at (C) 9 months (D) 10 months months hence at 12% per annum

Quantitative Aptitude Test | 263


is Rs. 240. Find the amount of 30. If the difference between the (C) 16%, Rs. 1570
the bill and its present worth— interest and discount on a certain (D) 18%, Rs. 1560
(A) Rs. 3000, Rs. 3240 sum of money for 6 months at
6% be Rs. 2·25. Find the sum— 1
(B) Rs. 2000, Rs. 2240 38. If the interest on Rs. 50 at 4 %
2
(C) Rs. 2100, Rs. 2340 (A) Rs. 7525 (B) Rs. 2255
be equal to the discount on
(D) None of these (C) Rs. 2575 (D) Rs. 2755 Rs. 59 for the same time and at
24. The true discount on a bill due 9 31. The true discount on Rs. 1860 the same rate when is the latter
months hence at 6% per annum due 3 years hence is Rs. 60. Find sum due ?
is Rs. 180. Find the amount of the rate per cent— (A) 2 years (B) 4 years
the bill and its present worth— (A) 10% (B) 12% (C) 6 years (D) 3 years
(A) Rs. 3000, Rs. 3180 (C) 5% (D) 15%
39. If the discount on Rs. 3050 be
(B) Rs. 4000, Rs. 4180 32. The true discount on Rs. 340 due equal to the simple interest on
(C) Rs. 4500, Rs. 4680 5 years hence is Rs. 40. Find the Rs. 3000 for the same time. Find
rate per cent— the time, the rate of interest being
(D) None of these
2 5% per annum—
25. Find the difference between (A) 3% (B) 2 %
3 (A) 4 months
simple interest and true discount 1 2 (B) 6 months
on Rs. 960 due 4 year hence at (C) 3 % (D) 3 %
3 3 (C) 3 months
5% per annum simple interest—
33. The true discount on Rs. 2080 (D) None of these
(A) Rs. 32
due 2 years hence is Rs. 80. Find 40. Find the present worth of a bill
(B) Rs. 52 the rate per cent— of Rs. 2420 due 2 years hence at
(C) Rs. 42 (A) 4% 10% compound interest. Also
(D) None of these (B) 8% find the true discount—
26. The difference between the (C) 2% (A) Rs. 2000, Rs. 420
simple interest and the true (D) None of these (B) Rs. 2200, Rs. 520
discount on a certain sum for 6 34. If the true discount on Rs. 161 (C) Rs. 2100, Rs. 460
months at 4% is Rs. 15. Find the 1 (D) None of these
sum— due 2 years hence be Rs. 21. At
2 41. Find the true discount on
(A) Rs. 32850 what rate per cent is the interest Rs. 39·69 due in 2 years
(B) Rs. 28250 calculated ? reckoning compound interest at
(C) Rs. 38250 (A) 6% (B) 4% 5%—
(D) Rs. 38350 (C) 8% (D) 12% (A) Rs. 3·69 (B) Rs. 5
27. The difference between the 35. If the discount on Rs. 2273·70 (C) Rs. 5·69 (D) Rs. 4·69
simple interest and the true due at the end of a year and a 42. If Rs. 10 be allowed as true
discount on a certain sum of helf be Rs. 128·70, what is the discount on a bill of Rs. 110 due
money for 6 months at 6% is Rs. rate of interest ? at the end of a certain time, then
27. Find the sum— (A) 6% (B) 4% the discount allowed on the same
(A) Rs. 30900 1 sum due at the end of double the
(B) Rs. 39000 (C) 3% (D) 4 % time is—
2
(C) Rs. 20900 (A) Rs. 20 (B) Rs. 21·81
(D) Rs. 30600 36. What must be the rate of interest (C) Rs. 22 (D) Rs. 18·33
in order that the discount on
28. The true discount on a certain Rs. 774·76 payable at the end of 43. Rs. 20 is the true discount on
1 3 years may be Rs. 83·01 ? Rs. 260 due after a certain time.
sum of money due after 2
2 What will be the true discount on
(A) 3%
years at 6% per annum is less the same sum due after half of
(B) 2% the former time, the rate of
than the simple interest on the
same sum for the same time by (C) 4% interest being the same ?
Rs. 81. Find the sum— (D) None of these (A) Rs. 10 (B) Rs. 10·40
(A) Rs. 4140 (B) Rs. 4240 37. The present worth of a bill due 7 (C) Rs. 15·20 (D) Rs. 13
(C) Rs. 4150 (D) Rs. 4250 months hence is Rs. 1200 and if 44. The true discount on a bill due
29. The difference between the 1 10 months hence at 6% per
the bill were due at the end of 2
simple interest and discount on a 2 annum is Rs. 26·25. Then the
certain sum of money due 1 year years, its present worth would be amount of the bill is—
9 months hence at 4% is Rs. Rs. 1016. Find the rate per cent (A) Rs. 551·25
7·35. What is the sum ? and the sum of the bill— (B) Rs. 550
(A) Rs. 1605 (B) Rs. 1805 (A) 10%, Rs. 1270 (C) Rs. 551·50
(C) Rs. 1525 (D) Rs. 1625 (B) 8%, Rs. 1720 (D) Rs. 550·25

Quantitative Aptitude Test | 264


Answers with Hints
1. (D) S.I. on Rs. (110 – 10) for a given time = Rs. 10 = Rs. [110 + 91·66]
S.I. on Rs. 100 for double the time = 20 = Rs. [201·66]
Sum = Rs. (100 + 20) = Rs. 120 ∴ A loses = Rs. [200 – 201·66] = Rs. 1·66.

T.D. on Rs. 110 = Rs. ( 20


120 )
× 110 = Rs. 18·33
10. (B) S.I. on Rs. 240 for a given time = Rs. 20
S.I. on Rs. 240 for half the time = Rs. 10
(S.I.) × (T.D.) 25 × 20
2. (D) Sum =
(S.I.) – (T.D.) (
= Rs. )
25 – 20
∴ Rs. 10 is T.D. on Rs. 250

= Rs. 100
So, T.D. on Rs. 260 = Rs. ( 10
250 )
× 260 = Rs. 10·40

3. (B) S.P. = (102% of Rs. 600) = Rs. (


102
100 )
× 600 11. (B) P.W. of Rs. 1120 due 2 years hence at 6%
100 × 1120
= Rs. 612
= Rs. [ ]
100 + (6 × 2)
= Rs. 1000

∴ P.W. of Rs. 650·25 due 9 months hence is Rs. 612. P.W. of Rs. 1081·50 due 6 months hence at 6%
⇒ Rs. 38·25 is S.I. on Rs. 612 for 9 months ⎡ 100 × 1081·50 ⎤⎥ = Rs. 100 × 1081·50
∴ Rate = ⎛⎜
100 × 38·25⎞ 1
= Rs. ⎢⎢
( ) 1 ⎥
⎢⎣ 100 + 6 × 2 ⎥⎦
[ 103 ]
3 ⎟ % = 8 3%
⎜⎝ 612 × ⎟⎠
4 = Rs. 1050
4. (C) Since T.D. is S.I. on P.W., we have So, A owes B, Rs. 1000 cash and B owes A Rs. 1050
Rs. (810 – 750) or Rs. 60 as S.I. on Rs. 750 for 2 cash.
years. ∴ B must pay Rs. 50 to A.
100 × 60
∴ Rate = ( 750 × 2 ) = 4%
1
12. (D) P.W. of Rs. 360 due 2 years hence at 7 % per
7
⎧ 100 × 360 ⎫⎪
5. (B) P.W. of Rs. 901 due 9 months hence at 8% annum = Rs. ⎪⎨⎪ ⎬
= Rs. ⎪
⎧ 100 × 901 ⎫⎪ = Rs. 100 × 901 × 1
⎨⎪ 3 ⎬ ( )
50
(
⎩100 + 7 × 2 ⎪⎭ )
( )
⎩100 + 8 × 4 ⎪⎭
106
= Rs.
100 × 360 × 7
{ 800 }
= Rs. 850
= Rs. 315
6. (A) P.W. of Rs.702 due 6 months hence ∴ S.P. = Rs. 315
⎧ 100 × 702 ⎪⎫ = Rs. 675
= Rs. ⎪ 15 × 100
⎨⎪ 1⎬
⎩100 + 8 × 2⎪⎭
Hence, gain % = ( 300 )
= 5%

∴ Total P.W. = Rs. (675 + 650) = Rs. 1325 13. (D) Amount = (T.D.) × {100 R+ ×(RT× T)}
7. (A) P.W. of Rs. 8250 due 6 months hence
26·25 × 105
⎧ 100 × 8250 ⎫⎪ = Rs. 8000 = Rs. ( ) = Rs. 551·25
= Rs. ⎪ 5
⎨⎪ 25 1 ⎬
(
⎩100 + 4 × 2 ⎪⎭ ) 14. (C) P.W. = Rs. (2575 – 75) = Rs. 2500
100 × 75 × 3
∴ Rs. 8100 in cash is a better offer. ∴ Rate = (2500 × 1 )
% = 9%
100 × T.D. 100 × 75
8. (B) P.W. =
R×T
= Rs. (5×2 ) 15. (B) P.W. = (Sum due) – (T.D.)
= Rs. (1860 – 60) = Rs. 1800
= Rs. 750
Thus, Rs. 60 is S.I. on Rs. 1800 at 5% per annum.
∴ Sum due = Rs. (750 + 75) = Rs. 825
100 × 60
9. (D) A has to pay the P.W. of Rs. 220 due 1 year
hence, which is
∴ Time = (1800 × 5 )years

100 × 220 2
= Rs. [
100 + (10 × 1) ]
= Rs. 200 =
3
years = 8 months

16. (C) P.W. of Rs. 220 due 1 year hence


A, actually pays = Rs. [110 + P.W. of Rs. 110 due 2
100 × 200
years hence]
100 × 110
= Rs. ( 100 + 10 )
= Rs. 200
[
= Rs. 110 +
100 + (10 × 2) ] hence, the man gains Rs. 5

Quantitative Aptitude Test | 265


100 × 176
17. (A) P.W. = = Rs. 160 A× (4 ×12) 2

( 100 + 6 ×
20
) 26. (C) ˙·˙ 15 =
12
T.D. = Amount – Present worth (
100 100 + 4 ×
1
2 )
= Rs. 176 – Rs. 160 = Rs. 16 A×4
⇒ 15 =
1 3 100 × 102
18. (A) Time = 4 months = year,
2 8 ⇒ A = 15 × 25 × 102
Rate = 4 per cent ∴ A = Rs. 38,250.

∴ Amount of Rs. 100 = Rs.


203
2
27. (A) Req. amount =
(
27 × 100 + 6 ×
1
2 ) × 100
∴ (
P.W. = Rs. 456·75 ÷ )
203
2
× 100 (6 ×12) 2

= Rs. 450 2700 × 103


=
1 9
Again, time = 3 months = year, rate = 4 per cent
4 = Rs. 30900.
100
P.W. = Rs. 455·51 ×
= Rs. 451
101
28. (A) Req. amount =
(
81 × 100 + 6 ×
5
2 ) × 100
Hence the required sum to be paid to A
= Rs. 451 – Rs. 450 = Re. 1
(6 ×52) 2

100 × 264 8100 × 115


19. (A) P. W. = [ 100 + (5 × 2) ] =
225
100 × 264 = Rs. 4140
= Rs. [ 110 ] = Rs. 240. Here, S.I. – T.D. = Rs. 81
100 × 272·61 29. (A)
20. (C) Required answer =
( 100 +
15
2
×2
73
365) 30. (C)
100 × T.D. 100 × T.D.
27261 27261 × 2 31. (A) R = =
= = P.W. × T (A – T.D.) × T
( 100 +
33
2) 233
=
100 × 60
1800 × 3
= 10% [since P.W. = A – T.D.]
= Rs. 234. ∴ The rate per cent is 10% per annum.
100 × 1051·25
21. (B) P. W. = [ 100 + 41/8 × 1 ] 32.
33.
(B)
(C)
105125 × 8
= [ 841 ] = Rs. 1000.
34. (A) Required rate per cent =
100 × 21
1
= 6%.
100 × 5300 (161 – 21) × 2
22. (A) Required sum = 2
( 3
100 + × 4
2 ) 35. (B)
100 × 83·1 8310
100 × 5300 36. (C) Rate % = =
= = Rs. 5000 (774·76 – 83·01) × 3 691·75 × 3
106
= 4%
100 × T.D. 100 × 240
23. (A) P.W. = = = Rs. 3000 37. (A) Sum due = P.W. + T.D. = P.W. + S.I. on P.W.
R×T 8
12 × Now, sum due
12
˙·˙ P.W. is Rs. 3000 = (Rs. 1200 + S.I. on Rs. 1200 for 7 months)
∴ A = Amount of bill Also, sum due

24.
= P.W. + T.D. = 3000 + 240 = 3240
(B) Solve as Q. 23.
( 5
= Rs. 1016 + S.I. on Rs. 1016 for years
2 )
25. (A) S.I. – T.D. =
A × (R.T.)2
100(100 + R.T.)
{ (
∴ Rs. 1200 + S.I. on Rs. 1200 ×
7
12 )
for 1 year }
=
960 × (4 × 5)2
100 × (100 + 4 × 5)
{ (
= Rs. 1016 + S.I. on Rs. 1016 ×
5
2 )
for 1 year }
960 × 20 × 20 ⇒ {Rs. 1200 + S.I. on Rs. 700 for 1 year}
= = Rs. 32
100 × 120 = {Rs. 1016 + S.I. on Rs. 2540 for 1 year}

Quantitative Aptitude Test | 266


⇒ S.I. on Rs. (2540 – 700) for 1 year 40. (A) Here sum is put on compound interest,
= Rs. (1200 – 1016) A 2420
⇒ S.I. on Rs. 1840 for 1 year = Rs. 184 ˙·˙ P.W. = =

∴ Rate =
100 × 184
= 10%
( 1+
r
100 ) ( n
1+
10
100) 2

1840 × 1
Also, sum due 2420 × 100
=
= Rs. 1200 + (S.I. on Rs. 1200 for 7 months 121
at 10%) = Rs. 2000
[ (
= Rs. 1200 + 1200 × ×
7 10
12 100)] ⇒ T.D. = P.W. – P
= Rs. 1270 ∴ True discount = 2420 – 2000 = Rs. 420.

Short Method : 39·69


41. (A) ˙·˙ P.W. =

1200
5
100 + R ×
2
( 1+
5
100 ) 2

˙·˙ =
1016 7 3969 × 100 × 100
100 + R × =
12 100 × 105 × 105

(
⇒ 1200 100 + R ×
7
12 ) = Rs. 36
∴ True discount = Rs. 39·69 – 36 = Rs. 3·69
= 1016 (100 + R × )
5
42. (D) S.I. on Rs. (110 – 10) for a given time = Rs. 10
2
S.I. on Rs. 100 for double the time = Rs. 20
⇒ 3680 R = 36800
∴ R = 10% Sum = Rs. (100 + 20) = Rs. 120
∴ Sum due = 1200 + (S.I. on Rs. 1200 for 7
months at 10%).
T.D. on Rs. 110 = Rs. (
20
120
× 110 = Rs. 18·33 )
= Rs. 1270 43. (B) S.I. on Rs. 240 for a given time = Rs. 20
9 59 – 50 S.I. on Rs. 240 for half of the time = Rs. 10
38. (B) ˙·˙ = × 100
2 50 × T ∴ Rs. 10 is T.D. on Rs. 250
∴ T = 4 years

39. (A) ˙·˙ 5 =


3050 – 3000
× 100
Req. T.D. on Rs. 260 = Rs. (250
10
× 260)
3000 × T
= Rs. 10·40
1
∴ T = years = 4 months 44. (A)
3

Quantitative Aptitude Test | 267


Banker’s Discount
Suppose a merchant A purchases called the Banker’s Gain (B.G.). Solution :
goods worth of say Rs. 5000 from Thus, Banker’s Gain
another merchant B at a credit of a B.G. = (B.D.) – (T.D.) T.D. = ⎯⎯⎯⎯⎯⎯⎯⎯⎯⎯⎯
√ (P.W.) × (B.G.)
certain period say 4 months. Then B Remark—When date of the bill (T.D.)2
or B.G. =
draws up a draft i.e., prepares a is not given, grace days are not to be (P.W.)
110 × 110
special type of a bill called Hundi or
Bill of exchange. On the receipt of
added.
Formulae—
= Rs. ( 1100 )
the goods A gives an agreement (i) B.D. = S.I. on bill for = Rs. 11
dually signed on the bill stating that unexpired time. ∴ B.D. = B.G. + T.D.
he has accepted the bill and money = Rs. (11 + 110)
(ii) Banker’s Gain = (B.D.) –
can be withdrawn from his bank = Rs. 121
(T.D.)
account after 4 months of the date Q. 3. The banker’s discount
of the bill. On this bill there is an (iii) B.G. = S.I. on T.D. and the true discount on a sum of
order from A to his bank asking to (iv) T.D. = √ (P.W.) × (B.G.); money due 8 months hence are Rs.
⎯⎯⎯⎯⎯⎯⎯⎯⎯⎯⎯
pay Rs. 5000 to B after 4 months. 52 and Rs. 50, respectively. Find
(T.D.)2 the sum and the rate per cent.
Moreover 3 more days (known as B.G. =
(P.W.) Solution :
grace days) are added to the date
(called nominally due date) of expiry Amount × Rate × Time (B.D.) × (T.D.)
(v) B.D. = Sum =
of 4 months and on the date so 100 (B.D.) – (T.D.)
obtained (called the legally due date) Amount × Rate × Time 52 × 50
the bill can be presented to the bank
by B to collect Rs. 5000 from A’s
T.D. =
100 + (Rate × Time) = Rs. ( 2 )
(B.D.) × (T.D.)
account. Suppose the bill is drawn on (vi) Amount = ; = Rs. 1300
(B.D.) – (T.D.)
5th January at 4 months, then the Since B.D. is S.I. on sum due. So
B.G. × 100 S.I. on Rs. 1300 for 8 months is Rs.
nominally due date is 5th May and T.D. =
Rate × Time 52. Consequently.
the legally due date is 8th May. The
100 × 52⎞
Rate = ⎛⎜
amount given on the draft or bill Examples
is called the face value which is ⎟ % = 6%
Rs. 5000 in this case. Q. 1. The true discount on a ⎜⎝ 1300 × 2 ⎟⎠
bill of Rs. 1860 due after 8 months 3
Now, suppose that B needs the Q. 4. The banker’s discount on
is Rs. 60. Find the rate, the
money of this bill earlier than 8th
banker’s discount and the banker’s Rs. 1800 at 5% is equal to the true
May say on 3rd March. In such a discount on Rs. 1830 for the same
gain.
case, B can sell the bill to a banker or time and at the same rate. Find the
a broker who pays him the money Solution : time.
against the bill but somewhat less Amount = Rs. 1860, T.D. Solution : S.I. on Rs. 1800
than the face value. Now the natural = Rs. 60 = T.D. on Rs. 1830
questions is, as how much cash the ∴ P.W. = Rs. (1860 – 60) ∴ P.W. of Rs. 1830 is Rs. 1800
banker should pay to B on 3rd = Rs. 1800 i.e., Rs. 30 is S.I. on Rs. 1800 at
March. Actually, if the banker S.I. on Rs. 1800 for 8 months 5%
deducts the true discount on the face = Rs. 60 100 × 30
value for the period from 3rd March
to 8th May, he gains nothing. So in ∴ Rate = ⎢⎢
⎡ 100 × 60 ⎤
⎥ % = 5%
∴ Time = ( 1800 × 5) years
2⎥ 1
order to make some profit, the banker ⎢⎣ 1800 × 3 ⎥⎦ = years = 4 months
deducts from the face value. The 3
simple interest on the face value for (T.D.)2 Q. 5. If the true discount on a
B.G. = certain sum due 6 months hence at
the unexpired time i.e., from 3rd (P.W.)
March to 8th May. This deduction is 60 × 60 6% is Rs. 36, what is the banker’s
= Rs. = Rs. 2 discount on the same sum for the
known as Banker’s Discount (B.D.) 1800 same time and at the same rate ?
or commercial discount. B.D. = (T.D.) + (B.G.) Solution :
Thus B.D. is the S.I. on face = Rs. (60 + 2) B.G. = S.I. on T.D.
value for the period from the date on
36 × 6 × 1
which the bill was discounted and the
legally due date. The money paid by
= Rs. 62.
Q. 2. The present worth of a
= Rs. ( 100 × 2 )
the banker to the bill holder is called bill due sometime hence is Rs. 1100 = Rs. 1·08
the discountable value. and the true discount on the bill is ∴ (B.D.) – (T.D.) = Rs. 1·08
Also, the difference between the Rs. 110. Find the banker’s discount or B.D. = (T.D.) + Rs. 1·08
banker’s discount and the true and the extra gain the banker = Rs. (36 + 1·08)
discount for the unexpired time is would make in the transaction. = Rs. 37·08

Quantitative Aptitude Test | 268


Exercise 09. A bill is discounted at 5% per (C) Rs. 60
annum. If banker’s discount be (D) None of these
01. The banker’s gain of a certain allowed at what rate per cent
sum of money is Rs. 36 and the must the proceeds be invested, 17. Find the banker’s discount on a
true discount on the same sum so that nothing may be lost ? bill of Rs. 12750 due 2 months
for the same time and at the same hence and 3% per annum—
19
rate is Rs. 30. The sum is— (A) 5% (B) 4 % (A) Rs. 63·75 (B) Rs. 61·75
21
(A) Rs. 1080 (B) Rs. 180 (C) Rs. 64·75 (D) Rs. 63·25
5
(C) Rs. 500 (D) Rs. 300 (C) 5 % (D) 10%
19 18. The true discount on a bill of
02. The banker’s gain of a certain Rs. 3720 due after 4 months is
10. The present worth of a sum due
sum due 2 years hence at 5% per Rs. 120. Find the banker’s dis-
sometimes hence is Rs. 576 and
annum is Rs. 80. The present count—
the banker’s gain is Re. 1. The
worth is— true discount is— (A) Rs. 122
(A) Rs. 800 (B) Rs. 1600 (A) Rs. 16 (B) Rs. 18 (B) Rs. 134
(C) Rs. 1200 (D) Rs. 880 (C) Rs. 24 (D) Rs. 32 (C) Rs. 124
11. The banker’s discount on a bill (D) None of these
03. The present worth of a certain
due 6 months hence at 6% is 19. The true discount on a bill of
sum due sometime hence is
Rs. 37·08. The true discount is— Rs. 1860 due after 8 months is
Rs. 1600 and the true discount is
Rs. 160. The banker’s gain is— (A) Rs. 6·18 (B) Rs. 12·36 Rs. 60. Find the banker’s gain—
(C) Rs. 48 (D) Rs. 36 (A) Rs. 1·5 (B) Rs. 2·5
(A) Rs. 10 (B) Rs. 16
12. The banker’s gain on a bill due 1 (C) Rs. 4 (D) Rs. 2
(C) Rs. 20 (D) Rs. 24
year hence at 5% is Re. 1. The 20. Find the banker’s gain on a bill
04. The banker’s gain on a certain true discount is— of Rs. 6900 due 3 years hence at
1
sum due 2 years hence is
2 ( )
3
23
(A) Rs. 15
(C) Rs. 25
(B) Rs. 20
(D) Rs. 5
5% per annum simple interest—
(A) Rs. 135 (B) Rs. 125
of the banker’s discount. The 13. The banker’s discount on a sum
rate per cent is— (C) Rs. 185 (D) Rs. 145
1
(A) 5% (B) 6% of money for 1 years is Rs. 60 21. Find the difference between the
2
and the true discount on the banker’s discount and the true
14 2
(C) 2 % (D) 6 % same sum for 2 years is Rs. 75. discount on Rs. 8100 for 3
23 3
The rate per cent is— months at 5%—
05. The banker’s discount on a (A) 5% (B) 6% (A) Rs. 0·125
certain sum due 2 years hence is (B) Rs. 1·25
2 1
11 (C) 6 % (D) 3 %
of the true discount. The rate 3 3 (C) Rs. 12·5
10
14. The banker’s discount on (D) None of these
per cent is—
Rs. 1600 at 6% is the same as 22. The banker’s discount on a bill
(A) 11% (B) 10%
the true discount on Rs. 1624 for due 6 months hence at 6% is
1 the same time and at the same
(C) 5% (D) 5 % Rs. 37·08. Find the true
3 rate. Then the time is— discount—
06. The present worth of a certain (A) 3 months (B) 4 months (A) Rs. 38
bill due sometime hence is (C) 6 months (D) 8 months (B) Rs. 32
Rs. 800 and the true discount 15. The banker’s discount on a bill (C) Rs. 36
is Rs. 36. Then the banker’s due 1 year 8 months hence is (D) None of these
discount is— Rs. 50 and the true discount on
(A) Rs. 37 (B) Rs. 34·38 the same sum at the same rate 23. Find the banker’s discount on a
per cent is Rs. 45. The rate per bill due 3 years hence at 5%
(C) Rs. 37·62 (D) Rs. 38·98
cent is— being given that the banker’s
07. The true discount on a bill of 2 gain is Rs. 90—
Rs. 540 is Rs. 90. The banker’s (A) 6% (B) 6 % (A) Rs. 550 (B) Rs. 650
3
discount is— 1 44 (C) Rs. 690 (D) Rs. 600
(A) Rs. 60 (B) Rs. 150 (C) 6 % (D) 8 %
2 59 24. The banker’s gain on a bill due 1
(C) Rs. 108 (D) Rs. 110 1
16. The true discount on a bill of year 4 months hence at 7 % per
08. The banker’s gain on a sum due Rs. 1860 due after 8 months is 2
3 years hence at 5% is Rs. 90. Rs. 60. Find the banker’s annum simple interest is Rs. 16.
The banker’s discount is— discount— Find the sum—
(A) Rs. 690 (B) Rs. 720 (A) Rs. 62 (A) Rs. 1760 (B) Rs. 1560
(C) Rs. 810 (D) Rs. 150 (B) Rs. 52 (C) Rs. 1660 (D) Rs. 1860

Quantitative Aptitude Test | 269


25. The banker’s gain on a bill due 1 (C) Rs. 1500, 8% (C) December 19, 1990
year hence at 5% is Re. 1. The (D) None of these (D) None of these
true discount is—
32. The interest on a certain sum of 38. A bill for Rs. 5656 is drawn on
(A) Rs. 15 (B) Rs. 20 money is Rs. 67·20 and the July 14 at 5 months. It is
(C) Rs. 25 (D) Rs. 5 discount on the same sum of discounted on Oct. 5th at 5%—
money for the same time and at
26. The banker’s gain on a bill due 2 (I) Banker’s discount
the same rate is Rs. 60. What is
years hence at 5% is Rs. 8, find (A) Rs. 56·56
sum ?
the present worth of the bill—
(A) Rs. 560 (B) Rs. 480 (B) Rs. 56
(A) Rs. 800 (B) Rs. 650
(C) Rs. 590 (D) Rs. 860 (C) Rs. 56·50
(C) Rs. 750 (D) Rs. 850
33. The banker’s discount on (D) None of these
27. The present worth of a bill due Rs. 1600 at 6% is the same as (II) True discount—
sometime hence is Rs. 1100 and the true discount on Rs. 1624 for
the true discount on the bill is (A) Rs. 50
the same time and at the same
Rs. 110. Find the banker’s dis- rate. Find the time— (B) Rs. 54·56
count and the extra gain the (C) Rs. 56
(A) 3 months (B) 4 months
banker would make in the
transaction— (C) 6 months (D) 8 months (D) None of these
(A) Rs. 11, Rs. 121 34. The banker’s gain on a certain (III) Banker’s gain and—
1 2 (A) Rs. 6·56
(B) Rs. 21, Rs. 131 sum due 2 years hence is of
2 23
(C) Rs. 12, Rs. 122 (B) Rs. 1·44
the banker’s discount on it for
(D) None of these the same time and at the same (C) Rs. 0·56
rate. Find the rate per cent— (D) None of these
28. The banker’s discount on
Rs. 1650 due a certain time (A) 5% (B) 4% (IV) Money received by the
hence is Rs. 165. Find the true (C) 8% (D) 6% holder of the bill—
discount and the banker’s gain— 35. What rate per cent does a man (A) Rs. 5599·56
(A) Rs. 150, Rs. 15 get for his money when in dis- (B) Rs. 5599·44
(B) Rs. 160, Rs. 5 counting a bill due 10 months
hence he deducts 4% of the (C) Rs. 5599
(C) Rs. 145, Rs. 20
amount of the bill ? (D) None of these
(D) None of these (A) 5% (B) 6% 39. A banker paid 5767·50 for a bill
29. The present worth of a certain (C) 8% (D) 4% of Rs. 5840, drawn on April 4 at
bill due sometime hence is 6 months. On what day was the
36. A bill drawn on March 8, at 7
Rs. 1600 and true discount on bill discounted the rate of
months date and was discounted
the bill is Rs. 160. Find the interest being 7% ?
on May 18, at 5%. If the banker’s
banker’s discount and the extra
gain is Rs. 3. Find (A) 3rd Aug. (B) 4th Aug.
gain the banker would make in
the transaction— (I) The true discount— (C) 3rd Sep. (D) 3rd July
(A) Rs. 176, Rs. 18 (A) Rs. 160 (B) Rs. 152 40. The banker’s discount on a sum
(C) Rs. 153 (D) Rs. 150 1
(B) Rs. 186, Rs. 16 of money for 1 years is Rs. 60
(C) Rs. 176, Rs. 16 (II) The banker’s discount and 2
(A) Rs. 153 (B) Rs. 151 and the true discount on the
(D) None of these same sum for 2 years is Rs. 75.
(C) Rs. 155 (D) Rs. 163
30. The present worth of a sum due The rate per cent is—
(III) The sum of the bill—
some times hence is Rs. 576 and (A) 5% (B) 6%
the banker’s gain is Re. 1. The (A) Rs. 7650
2 1
true discount is— (B) Rs. 7550 (C) 6 % (D) 3 %
3 3
(A) Rs. 16 (B) Rs. 18 (C) Rs. 7850
(D) None of these 41. A bill is discounted at 5% per
(C) Rs. 24 (D) Rs. 32 annum. If banker’s discount be
37. The holder of a bill for
31. The Banker’s discount and the allowed, at what rate per cent
Rs. 17850 nominally due on 21st
true discount on a sum of money must the proceeds be invested,
May, 1991 received Rs. 357 less
due 8 months hence are Rs. 52 so that nothing may be lost ?
than the amount of the bill by
and Rs. 50, respectively. Find having it discounted at 5%. 19
(A) 5% (B) 4 %
the sum and the rate per cent— When was it discounted ? 21
(A) Rs. 1300, 6% (A) December 29, 1990 5
(C) 5 % (D) 10%
(B) Rs. 1200, 5% (B) December 30, 1989 19

Quantitative Aptitude Test | 270


Answers with Hints
B.D. × T.D. 36 × 30 B.D. × 100 ⎧ 37·08 × 100 ⎫⎪
1. (B) Sum = = Rs. ( ) 11. (D) T.D. =
100 + (R × T)
= Rs. ⎪⎨ 1 ⎬
B.D. – T.D.
= Rs. 180
6
( )
⎩⎪100 + 6 × 2 ⎪⎭
B.G. × 100 80 × 100 = Rs. 36
2. (A) T.D. = = Rs. = Rs. 800
Rate × Time 5×2 B.G. × 100 1 × 100
160 × 160
12. (B) T.D. =
R×T
= Rs. ( 5×1 )
= Rs. 20
3. (B) B.G. =
(T.D.)2
= Rs. ( )
= Rs. 16
P.W. 1600
13. (D) B.D. for () 3
2
years = Rs. 60
4. (B) Let B.D. be Re. 1. Then B.G. = Re.
3
( ) 60 × 2
23
B.D. for 2 years = Rs. ( 3
×2 )
∴ ( )
T.D. = Re. 1 –
3
23
= Re.
20
23( ) = Rs. 80
⎡(1 ×20)⎤ Now, B.D. = Rs. 80; T.D. = Rs. 75 and Time = 2
⎢ 23 ⎥ = Rs. 20 years
Sum = Rs.
⎢(1 – 20)⎥ 3 ∴ ( 80 × 75
)
⎣ 23 ⎦ Sum = Rs.
5
= Rs. 1200
20 1
∴ S.I. on Rs. for 2 years is Re. 1 ∴ Rs. 80 is S.I. on Rs. 1200 for 2 years.
3 2
100 × 80
∴ Rate = ⎛⎜
100 × 1 ⎞
⎟ % = 6%
⎜⎝ 20 × 5 ⎟⎠
So, rate = ( 1200 × 2 )
%=3 %
1
3
3 2 14. (A) S.I. a Rs. 1600 = T.D. on Rs. 1624
5. (C) Let T.D. be Re. 1. Then, B.D. ∴ Rs. 1600 is P.W. of Rs. 1624 i.e., Rs. 24 is the S.I.
on Rs. 1600 at 6%.
= Rs. ( )
11
10
= Rs. 1·10
∴ Time = ( 100 × 24
) 1
year = year
1·10 × 1 1600 × 6 4
∴ Sum = Rs. ( )
1·10 – 1
Rs.
1·10
0·10
= Rs. 11 = 3 months
B.D. × T.D. 50 × 45
So, S.I. on Rs. 11 for 2 years is Rs. 1·10
100 × 1·10
15. (B) Sum =
B.D. – T.D.
= Rs. ( 5 )
= Rs. 450
∴ Rate = ( 11 × 2 ) % = 5%
Now, Rs. 50 is S.I. on Rs. 450 for () 5
years
36 × 36 3
6. (C) B.G. =
(T.D.)2
P.W.
= Rs. ( 800 ) = Rs. 1·62
∴ Rate = ⎛⎜
100 × 50⎞ 2
5 ⎟ % = 6 3%
∴ B.D. = (T.D.) + (B.G.) = Rs. (36 + 1·62) ⎜⎝ 450 × ⎟⎠
3
= Rs. (37·62)
16. (A) Amount = Rs. 1860, True discount
7. (C) P.W. = Rs. (540 – 90) = Rs. 450
= Rs. 60
S.I. on Rs. 450 = Rs. 90
∴ Present worth = Rs. 1860 – Rs. 60 = Rs. 1800
B.D. = S.I. on Rs. 540
S.I. on Rs. 1800 for 8 months
= Rs.( 90
450 )
× 540 = Rs. 108 = Rs. 60
⎡ 100 × 60⎤⎥ % = 5%
B.G. × 100 90 × 100 Rate = ⎢⎢
8. (A) T.D. =
R×T
= Rs. ( 5×3 ) ∴
2⎥
⎢⎣ 1800 × 3 ⎥⎦
= Rs. 600
2
∴ B.D. = Rs. (600 + 90) = Rs. 690 1860 × 5 ×
3
9. (C) Let the sum be Rs. 100. Then B.D. = Rs. 5 ∴ Banker’s discount = = Rs. 62
100
Proceeds = Rs. (100 – 5) = Rs. 95
17. (A)
∴ Rs. 5 must be the interest on Rs. 95 for 1 year
18. (C)
100 × 5
So, rate = ( )
95 × 1
5
=5 %
19 19. (D) P.W. = Rs. 1860 – Rs. 60 = Rs. 1800
(T.D.)2 60 × 60
10. (C) T.D. = ⎯⎯⎯⎯⎯⎯⎯⎯⎯⎯⎯⎯⎯⎯
√ {(P.W.)} × {(B.G.)} ∴ B.G. = = = Rs. 2
P.W. 1800
= Rs. √
⎯⎯⎯⎯⎯⎯
(576 × 1) = Rs. 24 20. (A)

Quantitative Aptitude Test | 271


B.D. × T.D. 52 × 50
21. (B) Difference between banker’s discount and the
true discount = Banker’s gain. 31. (A) Sum =
B.D. – T.D.
= ( 2 ) = Rs. 1300

1 1 Since B.D. is S.I. on sum due, so S.I. on Rs. 1300 for


8100 × 5 × × 5 ×
4 4 8 months is Rs. 52 consequently.
∴ Required answer =
(
100 100 + 5 ×
1
4 ) Rate = ⎛⎜
100 × 52 ⎞
⎟ % = 6%
⎜⎝ 1300 × 2 ⎟⎠
= Rs. 1·25 3
32. (A) Interest on Sum – True discount
22. (C) 37·08 = T.D. [ 1+
3
100 ] = Interest on true Discount.
Proof Sum = P.W. + T.D.
3708
∴ T.D. = = Rs. 36 ∴ Interest on Sum = Interest on P.W.
103
+ Interest on T.D.
90 × 100
23. (C) T.D. = = Rs. 600 = T.D. + Interest on T.D.
3×5
Interest on Sum – T.D. = Interest on T.D. or
∴ Banker’s discount = True discount + Banker’s Banker’s gain = Int. on T.D.
gain
In the given question, we have
= Rs. 600 + Rs. 90
Rs. 67·20 – Rs. 60 = Interest on Rs. 60
= Rs. 690 1
∴ Rs. 7 = Interest on Rs. 60
16 × 100 5
24. (A) T.D. = = Rs. 160
4 15 60
× ∴ Re. 1 = Interest on Rs.
3 2 1
7
B.D. = Rs. 160 + Rs. 16 = Rs. 176 5
176 × 100 1 60 1
∴ Sum = = Rs. 1760 ∴ Rs. 67 = Interest on Rs. × 67
4 15 5 1 5
× 7
3 2 5
60 1
B.G. × 100
25. (B) T.D. =
R×T
= (15××1001 ) = Rs. 20 ∴ The required sum =Rs.
7
1
5
× 67 = Rs. 560
5

100 × 100 33. (A) S.I. on Rs. 1600 = T.D. on Rs. 1624
26. (A) P.W. = 8 × = Rs. 800
10 × 10 ∴ Rs. 1600 is P.W. of Rs. 1624 i.e., Rs. 24 is the S.I.
27. (A) T.D. = ⎯⎯⎯⎯⎯⎯⎯⎯
√ P.W. × B.G. on Rs. 1600 at 6%
100 × 24
or B.G. =
(T.D.)2
P.W.
= (1101100× 110) = Rs. 11 ∴ Time = ( 1600 × 6) 1
= year
4
= 3 months
∴ B.D. = B.G. + T.D. = Rs. (11 + 110) = Rs. 121
100 × 2
28. (A) Sum =
B.D. × T.D. B.D. × T.D.
B.D. – T.D.
=
B.G.
34. (D) Rate per cent =
5 [ 3
23 – 3] = 6%

35. (A) Let the amount of bill be Rs. 100


T.D. Sum 1650 10
∴ = = = Money deducted = Rs. 4
B.G. B.D. 165 1
Money received by holder of the bill
i.e., if B.G. is Re. 1, T.D. = Rs. 10 or B.D. = Rs. 11 = Rs. (100 – 4) = Rs. 96
∴ if B.D. is Rs. 11, T.D. = Rs. 10 S.I. on Rs. 96 for 10 months = Rs. 4
100 × 4 × 6
If B.D. is Rs. 165, T.D. = Rs.
10
11[ ]
× 165 = Rs. 150 Rate = [ 96 × 5 ] = 5%

Also, BG = Rs. (165 – 150) = Rs. 15 36. (A) Date on which the bill was drawn
= March 8th and 7 months
29. (C) 160 = ⎯⎯⎯⎯⎯⎯⎯⎯
√ 1600 × B.G. Nominally due date = Oct. 8th
160 × 160 Legally due date Oct. 11th.
∴ B.G. = = Rs. 16
1600 Date on which the bill was discounted = May 18th
∴ Banker’s discount = 160 + 16 = Rs. 176 Time for which the bill has yet to run
[Q B.D. = T.D. + B.G.] May, June, July, Aug, Sep, Oct.
2
30. (C) T.D. = ⎯⎯⎯⎯⎯⎯⎯⎯⎯⎯
√ (P.W. × B.G.) = ⎯
√⎯⎯⎯⎯⎯
(576 × 1) = Rs. 24 13 + 30 + 31 + 31 + 30 + 11 = 146 days years
5

Quantitative Aptitude Test | 272


Now (I) (D) Banker’s gain = S.I. on T.D. ⎪⎧ 5656 × 5 × 1 ⎪⎫
⎪ 5 ⎪
2 (II) (C) T.D. = Rs. ⎨ ⎬⎪ = Rs. 56
i.e. Rs. 3 is S.I. on T.D. for years at 5% ⎪
5
100 × 3
( )
⎪⎩100 + 5 × 51 ⎪⎭
∴ T.D. = Rs. = Rs. 150
2 (III) (C) B.G. = B.D. – T.D. = 56 paise.

5 (IV) (B) Money received by the holder of the bill
(II) (A) B.D. = T.D. + S.I. on T.D. = Rs. (5656 – 56·56) =Rs. 5599·44.
2 39. (A) B.D. = Rs. (5840 – 5767·20)
= Rs. 150 + S.I. on Rs. 150 for
5 = Rs. 72·80
years at 5%
Rs. 72·80 is S.I. on Rs. 5840 at 7%.
2 5
= Rs. 150 + Rs. 150 × × 100 × 72·80 13
5 100 So, Unexpired time = = years
7 × 5840 73
= Rs. 153
= 65 days.
B.D. × T.D. 153 × 150
(III) (A) Sum = = Now, date of draw of bill = April, 4 at 6 months.
B.D. – T.D. 153 – 150
Nominally due date = October 4.
= Rs. 7650
Legally due date = October 7
37. (A) Clearly S.I. on Rs. 17850 at 5% is Rs. 357.
So, we must go back 65 days from October 7.
100 × 357
∴ Time = ( )
17850 × 5
2
= = 146 days
5 Oct., Sept., Aug. = 7 + 30 + 28
i.e., The bill was discounted on 3rd August.
So, the bill is 146 days prior to 24th May, the legally
due date
May, April, March, Feb., Jan., Dec.,
40. (D) B.D. for () 3
2
years = Rs. 60

60 × 2
24 + 30 + 31 + 28 + 31 + 2 = 146 days
So, the bill was discounted on 29 Dec., 1990.
B.D. for 2 years = Rs. ( 3
×2 )
38. (I) (A) Face value of the bill = Rs. 5656 = Rs. 80
Date on which the bill was drawn = July 14th at 5 Now, B.D. = Rs. 80, T.D. = Rs. 75
months. and Time = 2 years.
80 × 75
Nominally due date = December 14th.
Legally due date = December 17th.
∴ Sum = Rs. ( 5 )
Date on which the bill was discounted = October 5th = Rs. 1200
period for which the bill has yet to run. ∴ Rs. 80 is S.I. on Rs. 1200 for 2 years.
100 × 80
Oct., Nov., Dec.

26 + 30 + 17 = 73 days or year
1
So, rate =( )
1200 × 2
%=3 %
1
3
5 41. (C) Let the sum be Rs. 100. Then, B.D. = Rs. 5.
1 Proceeds = Rs. (100 – 5) = Rs. 95.
∴ B.D. = S.I. on Rs. 5656 for year at 5%
5 ∴ Rs. 5 must be the interest on Rs. 95 for 1 year.
100 × 5
= Rs.( 5656 × 1 × 5
100 × 5) = Rs. 56·56 So, rate = ( )
95 × 1
=5 %
5
19

Quantitative Aptitude Test | 273


Data Analysis
Bar Diagrams Answers with Hints
(1) In multiple bar diagrams two or more adjacent
vertical bars are drawn to represent two or more
Production in 1986-87 9 3
phenomenon for the same place or period. 1. (B) = =
Production in 1987-88 12 4
Examples 3
Q. 1. Examine the following graph carefully and ∴ Production in 1986-87 is times production in
4
answer the questions given below it : 1987-88
Production of cotton bales of 100 kg each in lacs in
states A, B, C, D & E during 1985-86, 1986-87 and 1987- = 0·75 times production in 1987-88.
88. 2. (B) Clearly, there is a steady increase in production
1985-86 1986-87 1987-88 in A and C during the given period.
21 3. (B) Total number of bales produced by E during the
given period
18 18
16 = (8 + 14 + 7) i.e., 29 lacs.
15
14
12 12
14
Its weight = (29 × 100000
1000
× 100
) tonnes
9 9 = 29000 tonnes.
8
7
6 4. (D) Average production of states A, B, C, D and E
5
are 13·66, 16, 9·66, 12·33, 9·66.
So states A, B and E showed below average pro-
duction in 1985-86, but above average production in
(A) (B) (C) (D) (E) 1986-87.
1. The production of state D in 1986-87 is how many 5. (C) State B has 48 lacs of bales, while state A has
times its production in 1987-88 ? only 41 lacs of bales during the given period.
(A) 1·33 (B) 0·75
(C) 0·56 (D) 1·77 Q. 2. Study the following graph carefully and
answer the questions given below it—
2. In which states is there a steady increase in the
Export of pearls in crores of Rupees
production of cotton during the given period ?
(A) A and B (B) A and C 11.4
10.8
(C) B only (D) D and E
9.9
3. How many tonnes of cotton was produced by state E 9.5
7.8
during the given period ? 6.5
(A) 2900 (B) 29000 5.2
(C) 290000 (D) 2900000
4. How many states showing below average production
in 1985-86 showed above average production in
1986-87 ?
1981 1982 1983 1984 1985 1986 1987
(A) 4 (B) 2
(C) 3 (D) 1 1. In which year there was maximum percentage
5. Which of the following statements is false ? increase in export of pearls to that in the previous
(A) States A and E showed the same production in year ?
1986-87 (A) 1982 (B) 1987
(B) There was no improvement in the production of (C) 1985 (D) 1984
cotton in state B during 1987-88
2. In which of the following pairs of years was the
(C) State A has produced maximum cotton during
average export of pearls around 9 crores ?
the given period
(D) Products of states C and D together is equal to (A) 1982 and 1983 (B) 1983 and 1984
that of state B during 1986-87 (C) 1984 and 1985 (D) 1985 and 1986

Quantitative Aptitude Test | 274


3. In how many years was the export above the average Q. 3. Given below is a bar diagram showing the
for the given period ? percentage of Hindus, Sikhs and Muslims in a state
during the years 1989 to 1992.
(A) 2 (B) 3
Hindus Sikhs Muslims
(C) 4 (D) 5 75
Scale : 1 cm = 10%
4. In which year was the export equal to the average 70
65
export of the preceding and the following year ? 60
(A) 1982 (B) 1983 55
50
(C) 1985 (D) 1986

Percentage
45
5. What was the percentage increase in export from 40
35
1986 to 1987 ? 30
2 25
(A) 16 % (B) 20% 20
3
15
1 10
(C) 19% (D) 33 %
3 5
0
1989 1990 1991 1992
Answers with Hints Years
Study the above diagram and mark a tick against
1. (D) Percentage increase in export of pearls in— the correct answer in each one of the following ques-
1·3 tions—
(i) 1982 over 1981 = × 100 = 25% 1. The ratio between Hindus and Sikhs in 1989 was—
5·2
(A) 3 : 2 (B) 2 : 3
1·3
(ii) 1983 over 1982 = × 100 = 20% (C) Cannot be calculated (D) 4 : 5
6·5
2. If the total population of the state in 1990 is 1 million,
2·1
(iii) 1984 over 1983 = × 100 = 26·9% then the Hindus population was—
7·8 (A) 35000000 (B) 3500000
0·9 (C) 350000 (D) 35000
(iv) 1985 over 1984 = × 100 = 9·09%
9·9 3. What was the percentage of Sikhs over Hindus in
1·9 1991 ?
(v) 1987 over 1986 = × 100 = 20%
9·5 (A) 35% (B) 40%
So, the maximum percentage increase in the export (C) 140% (D) 240%
was in the year 1984. 4. What percentage was the decrease in Hindus
2. (B) Average export in 1983 and 1984 is population from 1989 to 1992 ?
(A) 15% (B) 45%
= (7·8 +2 9·9) = 8·85 crores (C) 50% (D) 25%
5. If the population of the state in 1989 be 6 lakhs, then
= 9 crores (Approx.)
what is the total population of Hindus and Muslims
3. (C) Average in this year ?
5·2 + 6·5 + 7·8 + 9·9 + 10·8 + 9·5 + 11·4 (A) 270000 (B) 3300000
=
7 (C) 330000 (D) 33000
61·1 6. During which year was the Hindu percentage maxi-
= = 8·73
7 mum ?
So, the export above the average was in the year (A) 1989 (B) 1990
1984, 1985, 1986 and 1987. (C) 1991 (D) 1992
4. (A) Average of 1981 and 1983 7. What percentage was the increase in Muslim popula-
tion from 1990 to 1992 ?
5·2 + 7·8
= = 6·5 (A) 10% (B) 100%
2
(C) 200% (D) 20%
= Export in 1982.
8. If the total population in 1992 is 2 millions, then the
5. (B) Percentage increase from 1986 to 1987 Sikhs population is—
= (11·49·5– 9·5 ) × 100 = 9·51·9 × 100 = 20%. (A) 1300000
(C) 13000
(B) 130000
(D) 13000000

Quantitative Aptitude Test | 275


Answers with Hints Study the above multiple bar chart and mark a
tick against the correct answer in each of the following
1. (B) In 1989, the percentages of Hindus and Sikhs questions—
were 30 and 45 respectively. So, the ratio of Hindus
and Sikhs was 30 : 45 or 2 : 3. 1. The percentage of students in science faculty in
2. (C) In 1990, Hindu population 1990-91 was—
= 35% of total population (A) 26·9% (B) 27·8%
35 (C) 29·6% (D) 30·2%
= × (10000,00) = 350,000
100 2. The percentage of students in law faculty in 1992-93
3. (D) In 1993, Hindus = 25%, Sikhs = 60% was—
∴ Percentage of Sikhs over Hindus (A) 18·5% (B) 15·6%
=
60
25
× 100 = 240% ( ) (C) 16·7% (D) 14·8%

4. (C) Hindus in 1989 = 30% 3. How many times was the total strength of the
strength of commerce students in 1991-92 ?
Hindus in 1992 = 15%
Over 30, decrease = 15% (A) 3 times (B) 4 times
(C) 5 times (D) 6 times
Over 100, decrease =
15
30
× 100 = 50% ( ) 4. During which year the strength of arts faculty was
5. (C) ln 1989, Sikh population = (45% of 600000) minimum ?
(A) 1990-91 (B) 1991-92
=
45
100 (
× 600000 = 270000 ) (C) 1992-93 (D) None of these
∴ (Hindus + Muslims) = 600000 – 270000 5. How much percent was the increase in science
= 330000 students in 1992-93 over 1990-91 ?
6. (B) A quick observation of the chart shows that (A) 50% (B) 150%
Hindus in 1989, 90, 91, and 92 were 30%, 35%, 2
25%, 15% respectively. (C) 66 % (D) 75%
3
So, the maximum Hindu percentage was in 1990. 6. A regular decrease in students strength was in the
7. (B) Muslim population in 1990 = 10% faculty of—
Muslim population in 1992 = 20%
(A) Arts (B) Science
Increase on 10 = 10
(C) Commerce (D) Law
Increase on 100 =
10
10 (
× 100 % = 100% ) Answers with Hints
8. (A) In 1992, Sikh population = (65% of 2000000)
=
65
100
× 2000000 ( ) 1. (C) Total number of students in 1990-91
= (600 + 400 + 200 + 150) = 1350
= 1300000 Number of science students in 1990-91 was 400
Q. 4. Shown below is the multiple bar diagram
depicting the changes in the student’s strength of a Percentage of science students in 1990-91
college in four faculties from 1990-91 to 1992-93.
(Scale 1 cm = 100) = ( 400
1350 )
× 100 % = 29·6%
Arts Science Commerce Law 2. (B) Total number of students in 1992-93
600 = (500 + 600 + 250 + 250) = 1600
550 Number of law students in 1992-93 is 250
500
Students Strength

450 Percentage of law students in 1992-93


400
350
300
= (250
1600 )
× 100 % = 15·6%

250 3. (D) Total strength in 1991-92


200
= (550 + 500 + 250 + 200) = 1500
150
100 Total strength
50

Strength of commerce students
0
1990-91 1991-92 1992-93 1500
= = 6
Years 250

Quantitative Aptitude Test | 276


4. (C) A slight look indicates that the strength in arts Answers with Hints
faculty in 1990-91, 1991-92 and 1992-93 was 550,
600 and 500 respectively. So, it was minimum in 1. (D) The difference between the results of girls and
1992-93. boys :
5. (A) Number of science students in 1990-91 was 400 in V standard is 20; in VI standard is 10; in VII
Number of science students in 1992-93 was 600 standard is 20; in VIII standard is 30; in IX standard
is 10 and in X standard is 10.
Percentage increase = (200
400
× 100)% = 50% So, it is maximum in VIII standard.
6. (A) As the diagram shows the decrease every year is 2. (A) Average result of girls
in arts faculty. So, answer (A) is correct. 1
= (60 + 70 + 60 + 60 + 80 + 60)
Q. 5. Study the following graph and answer the 6
qeustions given below—
390
90 == 65%
6
80 80 80 So, in VII standard the result of boys is less than the
70 70 70 average result of the girls. Therefore, (A) is correct.
60 60 60
3. (E) In VI standard, the result of boys and girls are in
60
the ratio 8 : 7. While in IX standard, the results of
boys and girls are in the ratio 7 : 8.
Length

40 4. (A) Average result of boys


1
= (80 + 80 + 40 + 90 + 70 + 70)
6
430
= = 71·7%
6
Clearly in IX standard the result of girls is more than
the average result of the boys.
V VI VII VIII IX X
Standard 5. (E) Maximum number of girls passed is in IX
standard.
1. In which standard is the difference between the
results of girls and boys maximum ? So, the failure of girls is lowest in IX standard.
(A) V (B) VII Q. 6. Study the following graph carefully and
answer the questions—
(C) X (D) VIII
Demand, Production of colour T.Vs. of Five
2. In which standard is the result of boys less than the Companies of October 1988
average result of the girls ?
Demand Production
(A) VII (B) IX 3300
(C) VI (D) VIII
3000 2700
(E) V 2500
2200
3. In which pair of standards are the results of girls and
1800
boys in inverse proportion ?
(A) V and X (B) V and VI 1500

(C) VI and VIII (D) V and IX 1200

(E) VI and IX 1000


600
4. In which standard is the result of the girls more than
the average result of the boys for the school?
(A) IX (B) VIII
A B C D E
(C) VI (D) X
Companies
(E) None of these
1. What is the ratio of companies having more demand
5. In which standard is the failure of girls lowest ? than production to those having more production than
(A) X (B) VII demand ?
(C) VIII (D) V (A) 2 : 3 (B) 4 : 1
(E) None of these (C) 2 : 2 (D) 3 : 2

Quantitative Aptitude Test | 277


2. What is the difference between average demand and Q. 7. Study the graph carefully and answer the
average production of the five companies taken toge- questions below it—
ther ? 7016
(A) 1400 (B) 400 Wheat Imports
(in thousand tonnes) 5832
(C) 280 (D) 138
(E) None of these 4203

3. The production of company D is approximately how 3465


many times that of the production of the company 2500
A? 2413
2000
(A) 1·8 (B) 1·5
1811
(C) 2·5 (D) 1·11
(E) None of these
4. The demand for company ‘B’ is approximately what
per cent of the demand for company ‘C’ ? 1970 1971 1973 1974 1975 1976 1981 1982
Years
(A) 4 (B) 24
(C) 20 (D) 60 1. In which year did the imports register highest
increase over its preceding year ?
5. If company ‘A’ desire to meet the demand by (A) 1973 (B) 1974
purchasing surplus T. V. sets from a single company,
(C) 1975 (D) 1982
which one of the following companies can meet the
need adequately ? 2. The imports in 1976 was approximately how many
(A) B (B) C times that of the year 1971 ?
(C) D (D) None of these (A) 0·31 (B) 1·68
(C) 2·41 (D) 3·22
Answers with Hints 3. What is the ratio of the years which have above
average imports to those which have below average
1. (D) The companies having more demand than imports ?
production are A, C and E i.e., their number is 3. (A) 5 : 3 (B) 2 : 6
The companies having more production than demand (C) 8 : 3 (D) 3 : 8
are B and D i.e., their number is 2. (E) None of these
So, the required ratio is 3 : 2.
4. The increase in imports in 1982 was what per cent of
2. (C) Average demand the imports in 1981 ?
1 (A) 25 (B) 5
= (3000 + 600 + 2500 + 1200 + 3300) = 2120
5 (C) 125 (D) 80
Average production 5. The imports in 1974 is approximately what per
1 cent of the average imports for the given years ?
= (1500 + 1800 + 1000 + 2700 + 2200) = 1840
5 (A) 125 (B) 115
∴ Difference between average demand and average (C) 190 (D) 85
production = (2120 – 1840) = 280 (E) 65
3. (A) Let K (1500) = 2700

or K =
2700
= 1·8 Answers with Hints
1500
4. (B) Let x% of (demand for C) = (demand for B) 1. (C) Increase in imports in
1973 over 1971 is (2413 – 1811)
x
i.e. , × 2500 = 600 = 602 thousand tonnes
100
1974 over 1973 is (4203 – 2413)
600 × 100
or x = ( 2500) = 24% = 1790 thousand tonnes
1975 over 1974 is (7016 – 4203)
5. (C) Since company D produces highest number of = 2813 thousand tonnes
T. V. sets and company A desires to meet the
demand by purchasing surplus T. V. sets from a 1982 over 1981 is (2500 – 2000)
single company. Clearly, D can meet the demand of = 500 thousand tonnes
A. ∴ Highest increase over preceding year is in 1975.

Quantitative Aptitude Test | 278


2. (D) Let K (1811) = 5832. 4. What was the percentage drop in export quantity
5832 from 1983 to 1984 ?
Then, K = = 3·22 thousand tonnes
1811 (A) 75 (B) Nil
3. (E) Average of the imports (C) 25 (D) 50
1 (E) None of these
= (3465 + 1811 + 2413 + 4203 + 7016 + 5832
8 5. If in 1986 the tins were exported at the same rate per
+ 2000 + 2500) = 3655 tin as that in 1985. What would be the value in cross
The years in which the imports are above average are of rupee of export in 1986 ?
1974, 1975 and 1976 i.e., there are 3 such years. (A) 400 (B) 352
The years in which the imports are below average are (C) 375 (D) 330
1970, 1971, 1973, 1981 and 1982 i.e., there are 5 (E) None of these
such years
∴ Required ratio is 3 : 5. Answers with Hints
4. (A) Increase in imports in 1982 over 1981

= ( 2500 – 2000
2000 )
× 100 % = 25%.
1. (A) In 1983, the value of 100 lakh tins
= Rs. 150 crores
5. (B) Average import = 3655 thousand tonnes
Import in 1974 = 4203 thousand tonnes
∴ Value of 1 tin = Rs. (150 crore
100 lakh )
= Rs. (
1·00 )
Let x% of 3655 = 4203 150
= Rs. 150
Then, x = (42033655× 100) = 115% Similarly in 1984 the value of 1 tin
Q. 8. Study the following graph carefully and 150
= Rs. = Rs. 200
answer the following questions— 0·75
In 1985, the value of 1 tin

= Rs. ( )
330
1·50
= Rs. 220

In 1986, the value of 1 tin

= Rs. (1·60
400
) = Rs. 250
In 1987, the value per tin
500
= Rs. = Rs. 250
2·00
So, the value per tin is minimum in 1983
2. (D) Difference between the tins exported in 1985 and
1986 is = [(160 lakhs) – (150 lakhs)]
1. In which year the value per tins was minimum ? = 10 lakhs = 1000000.
(A) 1983 (B) 1984 3. (E) Percentage increase in export value from 1983 to
1987
(C) 1985 (D) 1986
(E) 1987 = { (500 crore – 150 crores)
150 crores
× 100 %}
2. What was the difference between the tins exported in
1985 and 1986 ? = { (500 – 150)
150
× 100 %}
(A) 10 (B) 1000
(C) 100000 (D) 1000000 = ( 350
150 )
× 100 % = 233·3%
(E) None of these 4. (C) Percentage drop in export quantity from 1983 to
3. What was the approximate per cent increase in 1984
export value from 1983 to 1987 ?
(A) 350 (B) 330
= { (100 lakh tonnes) – (75 lakh tonnes)
100 lakh tonnes
× 100}
(C) 43
(E) None of these
(D) 2·4
= ( 25
100 )
× 100 % = 25%

Quantitative Aptitude Test | 279


5. (B) In 1985, the cost of 150 lakh tins 5. What percentage is B’s expenditure on food over A’s
= Rs. 330 crores expenditure on food ? (Taking equal total expendi-
∴ In 1985, the cost of 1 tin ture)
330 crores (A) 10% (B) 70%
= Rs. 1
150 lakh (C) 133 % (D) 75%
3
=( ) 330
1·50
= Rs. 220
Answers with Hints
In 1986, the export value
= Rs. (160 lakh × 220) 1. (C) In family A, money spent on education
= Rs. (1·60 × 220) crores 20
= 20% =
= Rs. 352 crores 100
Q. 9. Following bar diagram shows the monthly 1
= (of total expenditure)
expenditure of two families on food, clothing, 5
education, fuel, house rent and miscellaneous (in 2. (B) In family B, the money spent on clothes
percentage). = (20% of total expenditure)

100
= Rs. ( 20
100 )
× 10000 = Rs. 2000

90 Food
3. (B) Money spent on food, clothes and house rent in
family A
80
Clothings = (30 + 15 + 15) = 60% of total expenditure
70
Education = Rs. ( 60
)
× 30000 = Rs. 18000
Expenditure

60 100
50 Fuel 4. (A) Family A spends on education and miscellaneous
40
= (20 + 10) = 30%
House Rent
Family B spends on education and miscellaneous
30
= (15 + 5) = 20%
Miscellaneous
20 So, family A spends more on these heads.
10 5. (C) B’s expenditure on food = 40%
0
A’s expenditure on food = 30%
(A) (B)
Study the above diagram and mark a tick against
B’s percentage over A’s = ( 40
30
× 100 )
the correct answer in each question. 1
= 133 %
1. What fraction of the total expenditure is spent on 3
Education in Family A ? Q. 10. The sum-divided bar-diagram given below
13 2 depicts the result of B.Sc. students of a college for
(A) (B)
20 3 three years.
9
(C) (D) None of these
13 200
2. If the total annual expenditue of family B is 180 First division
Rs. 10,000, then money spent on clothes during the
160 Second division
year is—
(A) Rs. 200 (B) Rs. 2,000 140
Third division
(C) Rs. 600 (D) Rs. 6,000 120
Failed
3. If the total annual expenditure of family A is 100
Rs. 30,000, then money spent on food, clothes and 80
house rent is—
(A) Rs. 18,500 (B) Rs. 18,000 60

(C) Rs. 21,000 (D) Rs. 15,000 40

4. If both the families have the same expenditure, which 20


one spends more on education and miscellaneous 0
together ? 1982 1983 1984
(A) Family A (B) Family B Study the above bar-diagram and mark a tick
(C) None (D) Both equal against the correct answer in each question.

Quantitative Aptitude Test | 280


1. How many per cent passed in Ist division in 1982 ? Total number of students passed during 3 years
(A) 20% (B) 34% = (140 + 150 + 165) = 455
2 13
(C) 14 %
7
(D) 11 %
17
2. What was the pass percentage in 1982 ?
Aggregate pass percentage =
455
565( × 100 % )
(A) 65% (B) 70% = 80·4%.
(C) 74·6% (D) 88·8% Q. 11. The following graph shows the temperature
of a patient observed in a hospital at a certain interval
3. In which year the college had the best result for of time on a certain day. Starting at 5 A.M.
B. Sc. ?
(A) 1982 (B) 1983 43
(C) 1984 (D) None of these
42
4. What is the number of third divisioners in 1984 ?
41
(A) 165 (B) 75
(C) 70 (D) 65 40
5. What is the percentage of students in 1984 over 39
1982 ?
38
11
(A) 30% (B) 17 %
17 37
5 a.m. 10 p.m. 3 p.m. 8 p.m. 9 p.m.
11
(C) 117 % (D) 85% Time
17
6. What is the aggregate pass percentage during three Scale → 1 division = 1 hour
years ?
2 Scale : {Along OX → 10 small divisions = 15
(A) 51 % (B) 82·7% minutes; Along OY = 10 small divisions = 1°C}
3
(C) 80·4% (D) 77·6% Study the above graph carefully and tick against
the correct answer in each of the following questions—
Answers with Hints 1. What was the temperature of the patient at 2 p. m. ?
1. (D) Percentage of 1st divisioners (A) 40·8° C (B) 41·1° C
=( 20
170 )
× 100 = 11 %
13
17 (C) 41·5° C (D) 41·9° C
2. (D) Total students passed = 140 2. The time, when the temperature was recorded 40° C
Total students appeared = 170 was—
(A) 11 A. M. (B) 10·30 A. M.
Pass percentage = ( 140
170 )
× 100 %
(C) 11·45 A. M. (D) 11·15 A. M.
= 88·8% 3. At what time during the day, the temperature was
3. (A) Pass percentage in 1982 maximum ?
= ( 140
170 )
× 100 % = 88·8% (A) 12 P. M.
(C) 1 P. M.
(B) 12·30 P. M.
(D) 1·30 P. M.
Pass percentage in 1983
4. What was the maximum temperature during the day?
= ( 150
195 )
× 100 % = 76·9% (A) 40·7° C (B) 41·5° C
Pass percentage in 1984 (C) 40·8° C (D) 41° C

= ( 165
200 )
× 100 % = 82·5% 5. The normal temperature is 37·5° C. At what time was
the temperature normal ?
So, the college recorded best result in 1982. (A) 5 A. M. (B) 5 P. M.
4. (C) Third divisioners in 1984 = (165 – 95) = 70. (C) 9 P. M. (D) At no time
5. (C) Students in 1984 = 200
Students in 1982 = 170 Answers with Hints
Required percentage = ( 200
170 )
× 100 %
1. (A) Since we have taken origin at 5 A. M. So 2 P. M.
is 9 hours beyond this point from this point draw a
line parallel to OY to meet the graph at a point. From
11
= 117 %. this point draw a line parallel to OX to meet at a
17 point on OY. This point indicates 40·8°C.
6. (C) Total number of students appeared during 3 years 2. (B) Reach a point on OY indicating 40°C. From this
= (170 + 195 + 200) = 565 point draw a line parallel to OX to meet the graph at

Quantitative Aptitude Test | 281


a point. From this point draw a line parallel to OY to 5. Two members of a family aged 20 years and 25 years
meet a point on OX. This point represents 10·30 are to be insured for Rs. 10000 each. The total annual
A. M. So, this temperature was recorded at 10·30 premium to be paid by them is—
A. M.
(A) Rs. 836·75 (B) Rs. 845·50
3. (C) From the highest point (along OY) on the graph, (C) Rs. 870·60 (D) Rs. 885
draw a line parallel to OY to meet OX at a point
which is 8 divisions before a point indicating 1 P. M. 6. Two persons aged 21 years and 23 years respectively
So, the highest temperature was recorded at 1 P. M. are insured for rupees one lakh each. The difference
4. (B) From the heighest point (along OY) on the graph, between their premiums is—
draw a line parallel to OX to meet OY at a point (A) Rs. 100 (B) Rs. 25
indicating 41·5° C. So, the maximum temperature (C) Rs. 50 (D) Rs. 20
during the day was 41·5°C.
5. (C) We are to find the time when the temperature Answers with Hints
was 37·5° C. Along OY take the point indicating 1. (B) From the point indicating 26 years on OX draw a
37·5° C. From this point, draw a line parallel to OX vertical line parallel to OY to meet some point in the
to meet the graph at a point. From this point, draw a curve. From this point draw a line parallel to OX to
line parallel to OY to meet OX at a point. This point meet OY at a point and this point clearly indicates
indicates 9 P. M. Rs. 45·75
So, the temperature was normal at 9 P. M. 2. (C) Along OY, reach the point indicating Rs. 44·60.
From this point draw a line parallel to OX to meet
Q. 12. The following graph shows the annual the graph at a point. From this point, draw a line
premium of an insurance company. Charged for an parallel to OY to meet OX at a point indicating 24
insurance of Rs. 1000 for different ages. years.
3. (C) As indicated by graph, premium at the age of 22
48
years for an insurance of Rs. 1000 is Rs. 43·75. So
47 for an insurance of Rs. 10000, the premium is Rs.
Premium in rupees

46
(43·75 × 10) = Rs. 437·50.
4. (B) Premium for Rs. 1000 for a man aged 23 years
45
= Rs. 44
44 Premium for Rs. 1000 for a man aged 30 years
43
= Rs. 46·50
0 20 21 22 23 24 25 26 27 28 29 30 Increase % in Premium
Age in years

Scale : {Along OX → 10 small division = 1 year;


=
2·50
44 (
× 100 % = 5·68% )
Along OY → 1 small division = 5 paise} 5. (D) Premium for Rs. 10000 at 20 years
1 big division = 10 small divisions (not shown in the = Rs. (43·25 × 10) = Rs. 432·50
fig.) Premium for Rs. 10000 at 25 years
= Rs. (45·25 × 10) = Rs. 452·50
Study the graph and mark a tick against the correct
answer in each of the following questions— Total annual premium for both
= Rs. (432·50 + 452·50) = Rs. 885
1. The premium for a man aged 26 years for an
6. (C) Premium for Rs. one lakh at 21 years
insurance of Rs. 1000 is—
= Rs. (100 × 43·50) = Rs. 4350
(A) Rs. 46 (B) Rs. 45·75 Premium for Rs. one lakh at 23 years
(C) Rs. 44 (D) Rs. 45 = Rs. (100 × 44) = Rs. 4400
2. What is the age of a person whose premium is Difference in premiums
Rs. 44·60 for an insurance of Rs. 1000 ? = Rs. 50
Q. 13. Study the following graphs carefully and
(A) 22 years (B) 23 years
answer the questions that follow :
(C) 24 years (D) 25 years Distribution of proteins in human body
3. The premium for a man aged 22 years for an
insurance of Rs. 1000 is— Proteins
16%
(A) Rs. 435 (B) Rs. 440 Muscles 1/3
Other dry
(C) Rs. 437·50 (D) Rs. 43·75 Hormones elements 14%
Enzymes and
4. How much per cent of the premium is increased if a other proteins Skin 1/10
man aged 30 years is insured for Rs. 1000 instead of Bones Water 70%
a man aged 23 years ? 1/6
(A) 4·75% (B) 5·68%
(C) 6·24% (D) 6% Distribution of elements in human body

Quantitative Aptitude Test | 282


1. What is the ratio of the distribution of proteins in the
muscles to that of the distribution of proteins in the Cost of printing
35%
bones ?
(A) 1 : 2 (B) 2 : 1 Cost of paper
Binders charges
(C) 18 : 1 (D) 1 : 18 18% 16%
Mi
sc.
2. What per cent of the total weight of the human body 7%
is equivalent to the weight of the skin in the human Adv. charges
Royalty
18%
body ? 15%

(A) ·016
Study the graph carefully and answer the ques-
(B) 1·6 tions given below it.
(C) ·16 1. What should be the central angle of the sector for the
(D) Insufficient information cost of the paper ?
(A) 22·5° (B) 16°
3. To show the distribution of proteins and other dry (C) 54·8° (D) 57·6°
elements in the human body, the arc of the circle 2. If the cost of printing is Rs. 17500, the royalty is—
should subtend at the centre an angle of— (A) Rs. 8750 (B) Rs. 7500
(A) 126° (B) 54° (C) Rs. 3150 (D) Rs. 6300
(C) 108° (D) 252° 3. If the miscellaneous charges are Rs. 6000, the adver-
tisement charges are—
4. What will be the quantity of water in the body of a
(A) Rs. 90000 (B) Rs. 1333·33
person weighing 50 kg ?
(C) Rs. 27000 (D) Rs. 12000
(A) 35 kg (B) 120 kg 4. If 5500 copies are published, miscellaneous expendi-
(C) 71·42 kg (D) 20 kg tures amount to Rs. 1848 and publisher’s profit is
25%, then marked price of each copy is—
5. In the human body what is made of neither bones nor (A) Rs. 8·40 (B) Rs. 12·50
skin ? (C) Rs. 10·50 (D) Rs. 10
2 3 5. Royalty on the book is less than the advertisement
(A) (B)
5 5 charges by—
1 3 (A) 3% (B) 20%
(C) (D)
40 80 2
(C) 16 % (D) None of these
3
Answers with Hints Answers with Hints
1. (B) Required Ratio 1. (D) Requisite angle
1 1
3 6
: = 6 : 3 or 2 : 1. = ( 16
100 )
× 360 = 57·6°.

2. (B) Weight of skin 2. (B) If cost of printing is Rs. 35, royalty is Rs. 15
If cost of printing is Rs. 17500, royalty is
1
=
10
parts of 16% of proteins = Rs. (15
35 )
× 17500 = Rs. 7500
1 3. (C) If misc. charges are Rs. 4, advertisement charges
= × 16% = 1·6%.
10 = Rs. 18
3. (C) Proteins and other dry elements If misc. charges are Rs. 6000 advertisement charges
= 30%
∴ Angle subtended by the required arc
= ( 18
4 )
× 6000 = Rs. 27000
4. (C) If misc. charges are Rs. 4 total charges
= (30% of 360°) = 108°. = Rs. 100
4. (A) Quantity of water in body of person weighing 50 If misc. charges are Rs. 1848, total charges
kg.
= Rs. (100
4 )
× 1848 = Rs. 46200
= 70% of 50 kg = ( 70
100 )
× 50 kg Cost price of each copy
= 35 kg.
5. (A) Part of the body made of neither bones nor skin
= Rs. (46200
5500 )
= Rs. 8·40
Marked price of each copy
=1– ( 1 1 1
+
3 10 6
+ ) (= 1–
6
10 ) 2
= .
5
= 125% of Rs. 8·40 = Rs. 10·50
5. (C) On Rs. 18 it is less by Rs. 3
Q. 14. Circle graph given above shows the expen-
diture incurred in bringing out a book, by a publisher.
On Rs. 100 it is less by
3
18 (× 100 = 16 %
2
3)
Quantitative Aptitude Test | 283
Tabulation
Tabulation : In studying problems on statistics, the 3rd year 73520, 61218, 16736, 11000 and 3918
data collected by the investigator are arranged in a 4th year 75104, 73117, 17523, 12038 and 4102
systematic form, called the tabular form. In order to avoid
5th year 80216, 90376, 19420, 15946 and 10523
same heads again, we make tables consisting of horizon-
tal lines (called rows) and vertical lines (called columns) Putting the data in the form of a table, write the total
with distinctive heads known as captions. Units of under each head and answer the following questions—
measurements are given along with the captions. (i) During which year the outlay on Education was
maximum ?
Exercise (ii) How many times, the outlay on Education was
Q. 1. Following table gives the population of a increased over preceding year ?
town from 1988 to 1992 (iii) What is the percentage increase during 1983-84 over
Increase 1982-83 in health outlay ?
(+) or (vi) What is total outlay on Social Welfare during the
Year Men Women Children Total Decrease plan period ?
(–) over
preceding (v) What is the ratio between outlays on Transport &
year (Communication) and Housing during 1984-85.
1988 65104 60387 — 146947 — Q. 3. The table given below shows the population
literates and illiterate in thousands and percentage of
1989 70391 62516 — — + (11630) literacy in three states in a year—
1990 — 63143 20314 153922 —
Percentage
1991 69395 — 21560 — (– 5337) States Population Literates Illiterates
of literacy
1992 71274 — 23789 160998 — Chennai 49342 6421 — —
Complete the table and mark a tick against the Mumbai — 4068 16790 —
correct answer in each question—
Bengal 60314 — — 16·1
(i) The number of children in 1988 is—
(A) 31236 (B) 125491 After reading the table, mark a tick against the correct
(C) 14546 (D) 21456 answer in each question given below and hence complete
(ii) The total population in 1989 is— the table—
(A) 144537 (B) 158577 (i) Percentage of Literacy in Chennai is—
(C) 146947 (D) 149637 (A) 14·9% (B) 13·01%
(C) 12·61% (D) 15·64%
(iii) Number of children in 1989 is—
(A) 25670 (B) 14040 (ii) Percentage of literacy in Mumbai is—
(C) 13970 (D) 15702 (A) 19·5% (B) 16·7%
(C) 18·3% (D) 14·6%
(iv) Number of men in 1990 is—
(A) 40645 (B) 60454 (iii) Literates in Bengal are—
(C) 70465 (D) 58835 (A) 50599 (B) 9715
(C) 7865 (D) 9475
(v) Number of women in 1991 is—
(A) 57630 (B) 56740 Q. 4. Study the following table carefully and
answer the questions given below—
(C) 52297 (D) 62957
Number of Boys of Standard XI Participating in
(vi) Increase or decrease of population in 1992 over 1991 Different Games
is—
(A) – (12413) (B) + (12413) ↓Games XI A XI B XI C XI D XI E Total
(C) + 155661 (D) + 7086
Class →
Q. 2. The following data give yearwise outlay in Chess 8 8 8 4 4 32
lakhs of rupees in a certain 5 year plan (1980 – 1985) Bedminton 8 12 8 12 12 52
of a state, under the heads :
Table Tennis 12 16 12 8 12 60
Transport & Communication, Education, Health,
Hockey 8 4 8 4 8 32
Housing and Social Welfare respectively.
Football 8 8 12 12 12 52
1st year 56219, 75493, 13537, 9596 and 1985
Total no. of boys 44 48 48 40 48 228
2nd year 71416, 80691, 15902, 10135 and 2073

Quantitative Aptitude Test | 284


(a) Every student (boy or girl) of each class of standard Financial Statement of a Company over the Year
XI participates in a game. Rs. (in lakh)
(b) In each class the number of girls participating in each Profit
game is 25% of the number of boys participating in Year Gross before Interest Depre- Net
each game. Turnover interest Rs. ciation Profit
Rs. and Rs. Rs.
(c) Each student (boy or girl) participates in one and
depre-
only one game. ciation
(i) All the boy of class XI D passed at the annual 1980-81 1380·00 380·92 300·25 69·90 10·69
examination but a few girls failed. If all the boys and 1981-82 1401·00 404·98 315·40 71·12 18·46
girls who passed and entered XII D and if in class 1982-83 1540·00 520·03 390·85 80·02 49·16
XII D, the ratio of boys to girls is 5 : 1. What would 1983-84 2112·00 599·01 444·44 88·88 65·69
be the number of girls who failed in class XI D ? 1984-85 2520·00 811·00 505·42 91·91 212·78
(A) 8 (B) 5 1985-86 2758·99 920·00 600·20 99·00 220·80
(C) 2 (D) 1
(i) During which year did the ‘Net Profit’ exceed Rs. 1
(E) None of these crore for the first time ?
(ii) Girls playing which of the following games need to (A) 1985-86 (B) 1984-85
be combined to yield a ratio of boys to girls of 4 : 1, (C) 1983-84 (D) 1982-83
if all boys playing Chess and Badminton are com- (ii) During which year was the ‘Gross Turnover’ closest
bined ? to thrice the ‘Profit before Interest and Depreciation’ ?
(A) Table Tennis and Hockey (A) 1985-86 (B) 1984-85
(B) Badminton and Table Tennis (C) 1983-84 (D) 1982-83
(C) Chess and Hockey (iii) During which year did the ‘Net profit’ form the
(D) Hockey and Football highest proportion of the ‘Profit’ before Interest and
Depreciation ?
(iii) What should be he total number of students in the (A) 1984-85 (B) 1983-84
school if all the boys of class XI A together with all (C) 1982-83 (D) 1981-82
the girls of class XI B and class XI C were to be
equal to 25% of the total number of students ? (iv) Which of the following registered the lowest increase
in terms of rupees from the year 1984-85 to the year
(A) 272 (B) 560
1985-86 ?
(C) 656 (D) 340 (A) Gross turnover
(iv) Boys of which of the following classes need to be (B) Profit before Interest and Depreciation
combined to equal to four times the number of girls (C) Depreciaiton
in class XI B and class XI C ? (D) Interest
(A) XI D and XI E (v) The ‘Gross Turnover’ for 1982-83 is about what per-
cent of the ‘Gross Turnover’ for 1984-85 ?
(B) XI A and XI B
(A) 61 (B) 163
(C) XI A and XI C (C) 0·611 (D) 39
(D) None of these Q. 6. Study the following table carefully and
(v) If boys of class XI E participating in Chess together answer the questions given below—
with girls of class XI B and class XI C participating Loan Disbursed by 5 Banks
in Table Tennis and Hockey respectively are selected
for a course at the college of sports. What per cent of Years
Banks
the students will get this advantage approximately ? 1982 1983 1984 1985 1986
(A) 4·38 (B) 3·51 A 18 23 45 30 70
(C) 10·52 (D) 13·5 B 27 33 18 41 37
C 29 29 22 17 11
(vi) If for social work, every boy of class XI D and class D 31 16 28 32 43
XI C is paired with a girl of the same class, what E 13 19 27 34 42
percentage of the boys of these two classes cannot Total 118 120 140 154 203
participate in social work ?
(A) 88 (B) 66 (i) In which year was the disbursement of loans of all
the banks put together least compared to the average
(C) 60 (D) 75 disbursement of loans over the years ?
Q. 5. Study the following table carefully and (A) 1982 (B) 1983
answer the questions given below— (C) 1984 (D) 1985

Quantitative Aptitude Test | 285


(ii) What was the percentage increase of disbursement of Production in Million Tonnes
loans of all banks together from 1984 to 1985 ? Year Total
Wheat Rice Maize Other
(A) 110% (B) 14% cereals
10 1988-89 580 170 150 350 1350
(C) 90 % (D) 10% 1989-90 600 220 234 400 1474
11
1990-91 560 240 228 420 1538
(iii) In which year was the total disbursement of loans of 1991-92 680 300 380 460 1660
banks A and B exactly equal to the total disbursement 1992-93 860 260 340 500 1910
of banks D and E ? Total 3280 1190 1332 2130 7932

(A) 1983 (B) 1986 Read the above table and mark a tick against the
correct answer in each of the following questions—
(C) 1984 (D) None of these
(i) During the period from 1988-89 to 1992-93. What
(iv) In which of the following banks did the disbursement per cent of the total production is the wheat ?
of loans continuously increase over the years ? (A) 42·6% (B) 43·1%
(C) 41·3% (D) 40·8%
(A) A (B) B
(ii) During the year 1992-93 the percentage increase in
(C) C (D) E production of wheat over the previous year was—
(v) If the minimum target in the preceding years was (A) 26·4% (B) 20·9%
20% of the total disbursement of loans, how many (C) 23·6% (D) 18·7%
banks reached the target in 1983 ? (iii) In the year 1991-92 the increase in production was
maximum for—
(A) 1 (B) 3
(A) Wheat (B) Rice
(C) 2 (D) 4 (C) Maize (D) Other cereals
(vi) In which bank was loan disbursement more than 25% (iv) During the year 1990-91, the percentage of decrease
of the disbursement of all banks together in 1986 ? in production of maize was—
(A) 2·63% (B) 2·56%
(A) A (B) B (C) 2·71% (D) 2·47%
(C) C (D) D (v) The increase in the production of other cereals was
Q. 7. The following table shows the production of minimum during the year—
foodgrains in million tonnes in a state for the period (A) 1989-90 (B) 1990-91
from 1988-89 to 1992-93— (C) 1991-92 (D) 1992-93

Answers with Hints


01. (i) (D) Number of children in 1988 (vi) (B) Total population in 1991 was 148585 and
that in 1992 was 160998.
= (146947) – (65104 + 60387) = 21456
So, Increase = (160998 – 148585) = 12413
∴ Answer (D) is correct.
∴ Answer (B) is correct.
(ii) (B) Total population in 1988 is 146947 and
increase in 1989 is 11630. Also, number of women in 1992
Therefore, total population in 1989 is = (160998) – (71274 + 23789)
= 65935
= (146947 + 11630) = 158577.
Filling all these entries the complete table is given
(iii) (A) Number of children in 1989 below—
= (158577) – (70391 + 62516) = 25670 Increase
(iv) (C) Number of men in 1990 (+) or
Year Men Women Children Total Decrease
= (153922) – (63143 + 20314) = 70465 (–) over
(v) (A) Total population in 1990 was 153922 and preceding
year
decrease in next year was 5337. So, the total
population in 1991 1988 65104 60387 21456 14697 —
= (153922 – 5337) = 148585 1989 70391 62516 25670 158577 + (11630)

Number of women in 1991 1990 70465 63143 20314 153922 – (4655)


1991 69395 57630 21560 148585 (– 5337)
= (148585) – (69395 + 21560)
1992 71274 65935 23789 160998 + (12413)
= 57630

Quantitative Aptitude Test | 286


02. The table may be constructed as shown below— Popula- Percentage
States Literates Illiterates
Outlay (in lakh of rupees) of a State in a tion of Literacy
5 year plan (1980 to 85) Chennai 49342 6421 42921 13·01%
Mumbai 20858 4068 16790 19·5%
Trans-
Bengal 60314 9715 50599 16·1%
port & Educa- Hous- Heal- Social
Year Com- tion ing th wel- Total 04. (i) (C) Total number of boys in XI D = 40
munica- fare
tion Number of girls in XI D = 25% of 40 = 10
1980-81 Since all boys of XI D passed, so the number of
56219 75493 13537 9596 1985 156830
boys in XII D = 40
1981-82
71416 80691 15902 10135 2073 180217 Ratio of boys and girls in XII D is 5 : 1.
1982-83
73520 61218 16736 11000 3918 166392 Number of girls in XII D
1983-84

1884-85
75104 73117

80216 90376 19420


17523 12038 4102 181884

15946 10523 21648


= ( 1
5 )
× 40 = 8

So, the number of girls failed in XI D


Total 356475 380895 83118 58715 22601 901804
= (10 – 8) = 2
As given in the table. (ii) (D) Total number of boys playing Chess
(i) During 1984-85, the outlay on education was and Badminton = (32 + 52) = 84
maximum. Number of girls playing Hockey & Football
(ii) Clearly, the outlay on education was increased = 25% (32 + 52) = 25% of 84
in 1981-82, over 1980-81, in 1983-84 over
1982-83 and in 1984-85 over 1983-84.
Thus it was increased three times during the
= (1
4 )
× 84 = 21.

plan period. Since 84 : 21 is 4 : 1, so the girls playing


Hockey and Football are combined to yield a
(iii) % increase in 1983-84 over 1982-83 in health
ratio of boys to girls as 4 : 1.
= ( 12038 – 11000
11000 )
× 100% = 9·43% (iii) (A) Number of boys in XI A
= 44
(iv) Total outlay on social welfare during the plan
period is Rs. 22601 lakhs. Number of girls in XI B = 25% of 48 = 12
(v) Ratio between outlay on (transport and com- Number of girls in XI C = 25% of 48 = 12
munication) and housing during 1984-85 is ∴ (44 + 12 + 12) = 68
= 80216 : 19420 = 4·13 : 1 = (413 : 100) Let x be the total number of students in the
03. (i) (B) Percentage of literacy in Chennai school.
= (6421
49342 )
× 100 % = 13·01% Then, 25% of x = 68
68 × 100
(ii) (A) Population of Mumbai ⇒ x =
25
= (4068 + 16790)
= 272
= 20858 thousands
∴ Total number of students in the school
∴ Percentage of literacy in Mumbai
= 272.
= (
4068
20858
× 100 % ) (iv) (D) 4 times the number of girls in XI B & XI C
= 19·5% = 4(12 + 12) = 96
(iii) (B) Number of literates in Bengal But, none of these pairs of classes from (A) to
(C) has this as the number of boys.
= (16·1
100
× 60314 ) (v) (B) Number of boys of XI E playing Chess = 4
= 9715 thousands Number of girls of XI B playing Table Tennis
Also, number of illiterates in Bengal = 25% of 16 = 4.
= (60314 – 9715) Number of girls of XI C playing Hockey
= 50599 thousands = 25% of 8 = 2.
Fillings these entries, the complete table is given ∴ Number of students selected for a course at
further— the college of sports = (4 + 4 + 2) = 10

Quantitative Aptitude Test | 287


Total number of students in the school 220·80 × 100
For 1985-86, we have x =
= (228 + 25% of 228) 920
= 285 = 24%
Let x% of 285 = 10 So, in 1984-85, the ‘net profit’ forms the
highest proportion of “the profit before
∴ x = (10285× 100) interest and depreciation.”
(iv) (C) Increase from the year 1984-85 to 1985-86
= 3·51% in gross turn over is (2758·99 – 2520)
(vi) (D) Since the number of girls = 25% of the = 238·99 lakhs
number of boys, so only 25% of the boys can Profit before interest and depreciation
participate in social work.
is (920 – 810·11) = 109·89 lakhs
∴ Required % of the boys, who cannot
participate in social work = (100 – 25)% Interest is (600·20 – 505·42) = 94·78 lakhs
= 75%. Depreciation is (99 – 91·91) = 7·09 lakhs
05. (i) (B) Clearly the net profit exceeded Rs. 1 crore Net profit is (220·80 – 212·78)
in the year 1984-85. = 8·02 lakhs
(ii) (A) The ratio of ‘Gross turn over’ to the profit Clearly, the increase is lowest in depreciation.
before interest and depreciation.
(v) (A) Let x% of gross turn over for (1984-85)
1380
In 1980-81 is = 3·62 = Gross turn over for (1982-83)
380·92
1401 x
In 1981-82 is = 3·46 ⇒ × 2520 = 1540
404·98 100
1540 1540 × 100
In 1982-83 is = 2·96 ∴ x= = 61·11% –~ 61%.
520·03 2520
2112 06. (i) (A) Average disbursement of loans over the
In 1983-84 is = 3·52 years
599·01
2520 1
In 1984-85 is = 3·11 = (118 + 120 + 140 + 154 + 203) = 147
810·11 5

2758·99 Clearly, it is least in the year 1982.


In 1985-86 is = 2·999
920 (ii) (D) Percentage increase of loans from 1984 to
(iii) (A) Let net profit = x% of profit before interest
and depreciation.
1985 = ( 154 – 140
140 ) × 100% = 10%

(iii) (D) In none of the given years is the sum of


10·69 × 100
For 1980-81, we have x = loans of A and B is equals to sum of loans of D
380·92 and E.
= 2·80% (iv) (D) In bank E the disbursement of loans conti-
18·46 × 100 nuously increase over the years.
For 1981-82, we have x =
404·98 (v) (C) 20% of total loans disbursed in 1982
= 4·56% = (20% of 118) = 23·6 crore
49·16 × 100 Clearly banks B and C reached the target in
For 1982-83, we have x = 1983.
520·03
= 9·45% (vi) (A) In 1986, 25% of total disbursement
= (25% of 203) crore = 50·75 crore
65·69 × 100
For 1983-84, we have x = ∴ In bank A, the loan disbursed is more than
599·01
25% of the total disbursement of all banks in
= 10·97% 1986.
212·78 × 100 07. (i) (C) Total production during the given period
For 1984-85, we have x =
810·11 = 7932 million tonnes
= 26·269 Wheat production during the period
= 26·27% = 3280 million tonne

Quantitative Aptitude Test | 288


∴ Required percentage of wheat production tonnes respectively. So, increase in maize
over total production production is maximum.

= (3280
7932 )
× 100 % = 41·3%
(iv) (B) During the year 1990-91, the decrease in
production of maize
= (234 – 228)
(ii) (A) Increase in 1992-93 in wheat production
over 1991-92 = 6 million tonnes
= (860 – 680) ∴ Required decrease %
= 180 million tonnes
Required increase %
= (2346 × 100)% = 2·56%
(v) (B) Increase in production of other cereals in
= (180
680
× 100)% = 26·4% 1989-90, 1990-91, 1991-92 and 1992-93 over
previous year is 50, 20, 40, 40 million tonnes
(iii) (C) During 1991-92 as read from the table the respectively. So, the increase in the production
increase in the production of wheat, rice, maize of other cereals was minimum during the year
and other cereals is 120, 60, 152 and 40 millions 1990-91.

Quantitative Aptitude Test | 289


Miscellaneous Exercise – I
Directions : (Q. 1–5) Study the 05. In which of the pair of years the his wife and the three grand
following graph carefully and answer sum of production is exactly 25% children together ?
the questions given below— of the production of all the years (A) Rs. 32,500
Quantity and value of production together ? (B) Rs. 2,57,500
of sugar of a company over (1) 1998 and 2004 (C) Rs. 2,82,500
the years (2) 1998 and 2003 (D) Cannot be determined
80 125 (3) 1998 and 2001
09. In one-day cricket match the
(A) Only (1) captain of one of the teams
Quantity in thousand metric tones

70 110
(B) Only (2) scored 30 runs more than the
60 95 (C) Only (3) average runs scored by the
Value in lakh Rs.

(D) Both (1) and (3) remaining six batsmen of that


50 80
team who batted in the match. If
06. Seven men, five women and the total runs scored by all the
40 65
eight children were given an batsmen of that team were 310,
30 50 assignment of distributing 2000 how many runs did the captain
books to students in a school score ?
20 35 over a period of three days. All (A) 60
of them distributed books on the
10 20 (B) 70
first day. On the second day two
0 women and three children (C) 50
5
remained absent and on the third (D) Cannot be determined
1998
1999
2000
2001
2002
2003
2004

day three men and five children 10. The radius of a circle is more
Years remained absent. If the ratio of than the height of a right angled
01. What was the difference between the number of books distributed triangle by 20%. The base of the
the average production of sugar in a day by a man, a woman and right angled triangle is equal to
during the average production of a child was 5 : 4 : 2 respectively, the area of the circle, then what
sugar during the years 1998, a total of approximately how is the approximate area of the
1999, 2000 and 2001 and that of many books were distributed on circle ?
2001, 2002, 2003 and 2004 in the second day ? (A) 72 sq. cm
thousand metric tons ? (A) 1000 (B) 800 (B) 144 sq. cm
(A) 15 (C) 650 (D) 900 (C) 216 sq. cm
(B) 20
07. The present ages of A, B and C (D) 128 sq. cm
(C) 25
are in the ratio of 8 : 14 : 22
(D) None of these 11. Two third of one fourth of a
respectively. The present ages of number is equal to 40% of
02. In which of the following years B, C and D are in the ratio of another number which one of the
was the percentage increase in 21 : 33 : 44 respectively. Which following statements is true
value of sugar per metric ton of the following represents the about the numbers ?
from the previous year the ratio of the present ages of A, B,
maximum ? C and D respectively ? (A) Second number is 2·4 times
(A) 1999 (B) 2000 of the first number
(A) 12 : 21 : 33 : 44
(C) 2001 (D) 2003 (B) First number is 2·4 times of
(B) 12 : 22 : 31 : 44 the second number
03. In which of the following years (C) 12 : 21 : 36 : 44 (C) First number is more than
was the value of sugar per metric (D) Cannot be determined the second number by 40%
ton the highest among the given
years ? 08. Mohan distributed his total (D) Second number is less than
(A) 1999 assets to his wife three sons, two the first number by 60%
(B) 2004 daughters and five grand 12. A shopkeeper purchased rice of
children in such a way that each 3 varieties a, b, c which cost
(C) 2003
grand child got one-eight of each Rs. 34.50, Rs. 28.60 and
(D) None of these
son or one-tenth of each daugh- Rs. 32.40 per kg. respectively. In
04. In which of the following years ter. His wife got 40 per cent of which of the following bargain
was the value of sugar per metric the total share of his sons and he will earn the maximum ?
ton the lowest among the given daughters together. If each (A) He purchased (a) and (c)
years ? daughter received asset of worth each 20 kg and sold them at
(A) 1998 (B) 1999 Rs. 1·25 lakh, what was the total Rs. 38.00 and Rs. 36.00 per kg.
(C) 2001 (D) 2003 worth of the assets received by respectively

Quantitative Aptitude Test | 290


(B) He purchased (a) and (b) (C) Either (1) or (2) 18. What was the percentage drop in
30 kg. and 40 kg respectively and (D) Either (3) or (4) the number of candidates
sold them Rs. 37.00 and appeared from the year 2002 to
Directions : (Q. 16–20) Study
Rs. 33.00 per kg respectively 2003 ?
the following graph carefully and
(C) He purchased (b) and (c) 2
answer the questions given below— (A) 66 (B) 30
20 kg. and 40 kg. respectively 3
and sold them at Rs. 40.00 and Number of candidates appeared 1
Rs. 38.00 per kg. respectively (in thousand) and percentage of (C) 70 (D) 33
3
(D) He purchased (c) and (a) candidates qualified in a competitive
25 kg. and 30 kg respectively examination over the years. 19. How many candidates qualified
and sold them at Rs. 42.00 and No. of candidates appeared in the year 2002 ?
Rs. 38.00 per kg. respectively (in thousands) (A) 6750
13. When an odd number of two 40 (B) 13500
digits is divided by an even (C) 9900
number of two digits then 35
(D) Cannot be determined
quotient is 0.625. If the odd
30 20. The number of candidates
number is less than the even
number by 5, then what is the 25 qualified in 1999 was what
ratio between odd number and percentage of the number of
even number ? 20 candidates appeared in 1998 ?
(A) 5 : 8 15
(A) 68·5 (B) 70
(B) 8 : 5 (C) 32·5 (D) 67·5
(C) 6 : 9 10
Directions : (Q. 21–25) In each
(D) Cannot be determined 5 of the following questions a number
14. In the given figure ABC is an series is given. After the series a
equilateral triangle which is 0
number is given followed by (a) (b)
1997
1998
1999
2000
2001
2002
2003
2004

inscribed in a circle of radius r. (c) (d) and (e). You have to complete
Which one of the following is Years the series starting with given number,
area of the triangle ? following the sequence of original
A Percentage of candidates qualified series and answer questions that
45
follow the series—
40 21. 8 4 6 15 52·5 36·25
D
35
12 (a) (b) (c) (d) (e)
What will come in place of (c) ?
30
(A) 18.25 (B) 19
25
B C (C) 22.5 (D) 20.75
E
20
22. 3 13 37 87 191 401
15 1 (a) (b) (c) (d) (e)
(A) (r – DE) 1/2 (r + DE)2
10 What will come in place of (d) ?
(B) (r – DE) 2 (r + DE)2 (A) 169 (B) 161
(C) (r – DE) 1/2 (r + DE)3/2 5
(C) 171 (D) 159
(D) (r + DE) 1/2 (r – DE)3/2 0
23. 5 12 4 10 3 8
1997
1998
1999
2000
2001
2002
2003
2004

15. The area of a square of one side 6 (a) (b) (c) (d) (e)
8 cm is equal to the area of a Years What will come in place of (d) ?
rectangle. Which of the following 16. What was the ratio between the (A) 3 (B) 5
statements about the rectangle number of candidates appeared
is/are correct ? (C) 4 (D) 7
in 1997 and the number of candi-
(1) The length of the rectangle dates qualified in 2004 ? 24. 4 13 40 135 552 2765
is 16 times of the breadth (A) 14 : 5 2 (a) (b) (c) (d) (e)
(2) The length of the rectangle (B) 5 : 14 What will come in place of (c) ?
is 32 times of the breadth
(C) 3 : 7 (A) 123 (B) 133
(3) The breadth of the rectangle
1 (D) Data inadequate (C) 127 (D) 131
is of the length
6 17. In which of the following years 25. 3 19 103 439 1381 2887
(4) The breadth of the rectangle was the number of candidates
5 (a) (b) (c) (d) (e)
1 qualified the lowest among the
is of the length. What will come in place of (b) ?
9 given years ?
(A) Only (1) and (2) (A) 1997 (B) 2002 (A) 139 (B) 163
(B) Only (3) and (4) (C) 2001 (D) 1998 (C) 161 (D) 157

Quantitative Aptitude Test | 291


Directions : (Q. 26–30) In each 35. A boat can row a distance of will be Sudha’s age after 3 years
of the following questions a series is 16 km down the stream in 2 from now ?
given. In each one term is wrong. hours while up the stream the (A) 48 years (B) 42 years
Find out the wrong term— same distance in 4 hours. What (C) 43 years (D) 38 years
26. 7, 4, 6, 9, 20, 52·5, 16·5 is the speed of the boat in still
water ? 42. The population of a state is
(A) 6 (B) 4 counted after every three years.
(C) 20 (D) 9 (A) 4 km/hr
It is found that population each
(B) 6 km/hr time is increased by 20% as
27. 4, 6, 12, 20, 30, 75, 315, 1260
(C) 8 km/hr compared to the previous count.
(A) 315 (B) 75
(C) 12 (D) 6 (D) Data insufficient If the population in the year
1998 was 42 lakh. What will be
28. 3, 4, 13, 38, 87, 166, 289 36. The ratio of the earnings of A
the population in the year 2007 ?
(A) 38 (B) 13 and B is 4 : 7. If A’s earning is
in creased 50% and B’s earning (A) 60.48 lakh
(C) 87 (D) 166
is decreased by 25%. Then the (B) 72.576 lakh
29. 4, 5, 9, 29, 111, 556, 3335 ratio in their earnings becomes (C) 62.576 lakh
(A) 5 (B) 9 8 : 7. What is the earning of A ? (D) 70.48 lakh
(C) 29 (D) 111 (A) Rs. 26,000
43. The ratio is the salaries of A, B
30. 2, 6, 16, 38, 84, 176, 368 (B) Rs. 28,000 and C is 2 : 3 : 5. If their salaries
(A) 6 (B) 16 (C) Rs. 21,000 are increased by 15%, 10% and
(C) 38 (D) 176 (D) Data insufficient 20% respectively, then what will
31. Average weight of three boys P, be the ratio in their new salaries ?
37. Sudhanshu invested Rs. 15000
1 for a year at 10% p.a. If the inte- (A) 3 : 3 : 10
T and R is 54 kg. while the
3 rest is compounded half yearly. (B) 23 : 33 : 60
average weight of 3 boys T, F What amount will Sudhanshu get (C) 10 : 11 : 20
and G is 53 kg. What is the at the end of the year ? (D) Can not be found
average weight of P, T, R, F and (A) Rs. 16537.50
G? 44. If 3x + 2x = 47 and 11x = 7y,
(B) Rs. 16500.00
(A) 53·8 kg what is the value of x – y ?
(C) Rs. 16525.50
(B) 52·4 kg (A) 4
(D) Rs. 18150.00
(C) 53·2 kg (B) 6
38. 405 sweets were distributed (C) 7
(D) Data insufficient
equally among children in such a
32. Two girls and four boys any (D) None of these
way that each child gets 20% of
made to sit in a line in such a the total number of children. Directions : (Q. 45–49) What
way that the girls should not sit How many sweets did each child will come in place of the question
together. By how many ways get ? mark (?) in the following questions ?
this is possible ? (A) 15 (B) 45 45. 3 × ? + 30 = 0
(A) 720 (B) 480 (C) 9 (D) 18 (A) – 15
(C) 360 (D) 240 39. If 2x + 3y + z = 55, x + z – y = 4 (B) 15
33. Sunetra started a business of and y – x + z = 12, what is value (C) 10
software with Rs. 50,000. After of y ? (D) None of these
6 months Nikhil joined her with (A) 7
Rs. 80,000. At the end of 3 years 46. 40·83 × 1·02 × 1·2 = ?
(B) 8
they earned a profit of (A) 49·97592 (B) 41·64660
Rs. 24,500. What is the share of (C) 12
(C) 58·7952 (D) 42·479532
Sunetra in the profit ? (D) None of these
1 3 1 22
(A) Rs. 14,000 40. 10% of wheat sold by a Grocer 47. 3 ÷6 ×1 × =?
is of lower quality. How much 3 7 2 7
(B) Rs. 9,423
wheat of good quality should be (A) 4·4
(C) Rs. 10,250
mixed with 150 kg of wheat of 22
(D) None of these (B)
lower quantity so that the wheat 7
34. What approximate value should of lower quality quantity be 5%? 5
come in place of question mark (A) 150 kg (B) 135 kg (C)
22
in the following equation ? (C) 50 kg (D) 85 kg (D) None of these
9876 ÷ 24·96 + 215·005 – ? = 41. Present ages of Radha and Sudha
309·99 are in the ratio of 7 : 9 respecti- 48. ⎯⎯⎯⎯⎯
√ 1·5625 = ?
(A) 395 (B) 295 vely. Five years ago ratio of their (A) 125 (B) 12·5
(C) 300 (D) 315 ages that time was 3 : 4. What (C) 1·05 (D) 1·25

Quantitative Aptitude Test | 292


49. 3978 + 112 × 2 = ? ÷ 2 (C) Either C = E or F and D = F every year. If his salary today is
(A) 8180 (B) 2101 or E Rs. 10000, what will be the
(D) C = E and D = F salary after another 4 years ?
(C) 4090 (D) 8404
57. Mr. ‘X’ invested certain amounts (A) Rs. 62500
50. 4 boys and three girls are to be
in two different Scheme ‘A’ and (B) Rs. 26500
seated in a row in such a way
‘B’. Scheme ‘A’ offers simple (C) Rs. 50625
that no two boys sit adjacent to
interest @ 12 per cent p.a. and (D) Rs. 33750
each other. In how many
Scheme ‘B’ offers compound
different ways can it be done ? 63. By how much is three fifth of
interest @ 10 per cent p.a.
(A) 5040 (B) 30 Interest accrued on the amount 350 greater than four-seventh of
(C) 144 (D) 72 invested in Scheme ‘A’ in 2 210 ?
51. What will be the compound years was Rs. 3600 and the total (A) 120
interest on a sum of Rs. 25000 amount invested was Rs. 35000. (B) 210
after three years at the rate of 12 What was interest accrued on the (C) 95
per cent p.a. ? amount invested in Scheme ‘B’ ? (D) None of these
(A) Rs. 10123·20 (A) Rs. 4800
64. The inequality x (x + 3) < 10
(B) Rs. 9000·30 (B) Rs. 4200
proved for what value of x ?
(C) Rs. 10483·20 (C) Rs. 4000
(A) x > 2, x < – 5
(D) Rs. 9720·00 (D) Cannot be determined
(B) – 5 < x < 2
52. What approximate value should 58. In how many different ways can
the letters of the word (C) – 2 < x < 5
come in place of the question
mark (?) in the following CORPORATION be arranged in (D) x < – 2, x > 5
equation ? 29·98 × 37·05 ÷ ? + such a way that the vowels 65. What will come in place of both
7·45 = 100·5 always come together ? the question marks (?) in the
(A) 10 (B) 13 (A) 840 following equation ?
(C) 14·5 (D) 12 (B) 86400 17 × 32 ÷ ? + 12
(C) 8400 = 10
53. A committee of 3 members is to 62 ÷ 9 × 4 – ?
be formed out of 3 men and 4 (D) None of these (A) 16 (B) 4
women. In how many different 59. If the number obtained by (C) 8 (D) 12
ways can it be done so that at reversing the digits of a two digit
least one number is a woman ? number is more than the original 66. In a sample, if a person is picked
(A) 34 (B) 12 number by 18 and the sum of the up randomly, the probability that
digits is 8. What is the original 3
(C) 30 (D) 36 the person is a smoker is and
number ? 5
54. In how many ways can a group that of the person being male is
(A) 53
of 5 men and 2 women be made 1/2. What is the probability that
out of total of 7 men and 3 (B) 26
the person is both male as well
women ? (C) 35 as a smoker ?
(A) 63 (B) 45 (D) Cannot be determined 10
(C) 126 (D) 90 (A)
60. The digit of a two digit number 11
55. 70% of a number is equal to four are in the ratio of 2 : 3 and the 1
number obtained by inter (B)
fifth of another number. If the 5
difference between the two num- changing the digits is bigger than
3
bers is 100. What is the bigger the original number by 27. What (C)
was the original number ? 5
number ?
(A) 64 (B) 46 (D) None of these
(A) 700
(B) 750 (C) 96 (D) 69 67. What will come in place of
(C) 800 question mark (?) in the
61. A select group of 4 is to be
following equation ?
(D) Cannot be determined formed from 8 men and 6
women in such a way that the 167·5 ÷ 83·5 ÷ 27·5 = ?
56. The average of four numbers A,
group must have atleast one (A) 84 (B) 164
B, C and D is 40. The average of
woman. In how many different (C) 2 15 (D) 227
four numbers A, B, E and F is
ways can it be done ?
also 40 (A, B are common).
(A) 364 (B) 1001 68. Three men four women and six
Which of the following must be children can complete a work in
true ? (C) 728 (D) 931 7 days. A woman does double
(A) (A + B) ≠ (C + D) 62. The salary of an employee the work a man does and a child
(B) (C + D) = (E + F) increase consistently by 50% does half the work a man does.

Quantitative Aptitude Test | 293


How many women alone can Give answer (D) if p is either (C) 14 : 13
complete this work in 7 days ? equal to or smaller than q. (D) None of these
(A) 8
73. (1) 6p2 + 5p + 1 = 0 80. Total expenditure of company
(B) 7 ‘A’ in all the year together was
(2) 20q2 + 9q = – 1
(C) 12 Rs. 82·5 lakh. What was the total
(D) Cannot be determined 74. (1) 3p2 + 2p – 1 = 0 income of the company in all the
(2) 2q2 + 7p + 6 = 0 years together ?
69. Four parts out of the five parts
lettered (A) (B) (C) (D) and (E) (A) Rs. 1·23 crore
75. (1) 3p2 + 15p = – 18
in the following equation are (B) Rs. 98·75 lakh
(2) q2 + 7q + 12 = 0
exactly equal. Which one of the (C) Rs. 99·85 lakh
given part is not equal to the ⎯√ 4 (D) Cannot be determined
other four parts ? The letter of 76. (1) p =
that part is the your answer ? ⎯9
√ 81. If the expenditures companies A
(2) 9q2 – 12q + 4 = 0 and B in 2000 were equal and
(A) 136 × 12 ÷ 9 × 3
the total income of the two
(B) = 17 × 64 ÷ 6 × 3 77. (1) p2
+ 13q + 42 = 0 companies was Rs. 5·7 lakh,
(C) = 36 × 17 + 9 × 9 (2) q2 = 36 what was the total expenditure of
(D) = 56 × 8 ÷ 14 × 17 the two companies in 2000 ?
Directions : (Q. 78–81) Study
(E) = 76 × 6 ÷ 19 × 17 (A) Rs. 4 lakh
the following graph carefully and
answer the questions given below it— (B) Rs. 2 lakh
70. In a class of 45 students boys
and girls are in the ratio of 5 : 4 (C) Rs. 4·2 lakh
Percentage profit earned by two
respectively. Average marks companies A and B over the given (D) Cannot be determined
obtained by boys in Mathematics years. 82. If the incomes of company B in
out of 100 were 76 and that of 1997 and 1998 were in the ratio
girls were 78. What are average % Profit/loss =
of expenditures of that company
marks of boys and girls together Income – Expenditure in these two years ?
in Mathematics (rounded off to × 100
Expenditure (A) 20 : 29 (B) 9 : 10
two decimal points) ?
70 (C) 29 : 45 (D) 10 : 29
(A) 76·89 (B) 77
65 83. Mr. Nilesh Agarwal opened a
(C) 77·29 (D) 76·98 60
workshop investing Rs. 40,000.
55
71. On five chairs arranged in a row, 50
He invested additional amount of
five persons A, B, C, D and E 45 Rs. 10,000 every year. After two
are to be seated in such a way 40 years his brother Suresh joined
that B and D always sit together 35 him with an amount of
(side by side). In how many 30 Rs. 85,000. Thereafter Suresh
different ways can it be done ? 25 did not invest any additional
(A) 120 (B) 48 20 amount. On completion of four
15 years from the opening of work
(C) 60 (D) 24 10 shop they earned an amount of
5
72. Present ages of Seema and Rs. 1,95,000. What will be
0
Naresh are in the respective ratio Nilesh’s share in the earning ?
1995

1996

1997

1998

1999

2000

of 5 : 7. Five years hence the (A) Rs. 85,000


ratio of their ages becomes 3 : 4 78. Expenditures of company B in (B) Rs. 1,10,000
respectively. What is Naresh’s 1996 and 1997 are Rs. 12 lakh (C) Rs. 1,35,000
present age in years ? and Rs. 14·5 lakh respectively. (D) Rs. 95,000
(A) 25 What was the total income of
(B) 40 company B in 1996 and 1997 84. The average (Arithmetic Mean)
together (in lakh Rs.) ? and the Median of a set of
(C) 30 numbers is the same which of
(A) 39·75 (B) 37·95
(D) Cannot be determined the following must be true ?
(C) 38·75 (D) 38·50
Directions : (Q. 73–77) for the (A) All the numbers are odd in
two given equations I and II— 79. Ratio of expenditures of com- the set
panies A and B in 1999 was 3 : 4 (B) All the numbers are even in
Give answer (A) if p is greater
respectively. What was the the set
than q. respective ratio of their income (C) All the numbers are conse-
Give answer (B) if p is smaller in 1999 ? cutive integers in the set
than q. (A) 21 : 26 (D) The data set has even num-
Give answer (C) if p is equal to q. (B) 13 : 14 bers of observations

Quantitative Aptitude Test | 294


Directions : (Q. 85–89) Study 89. What are the average marks was the ratio of its respective
the following table carefully to ans- obtained out of 80 by all the incomes in these two years ?
wer these questions— students together in subjects (A) 4 : 5
Percentage of marks obtained by ‘Q’ ? (B) 3 : 4
six students in six different subjects— (A) 68 (B) 77·5 (C) 2 : 3
Subjects (C) 88·57 (D) 62
Student (D) Cannot be determined
P Q R S T U Directions : (Q. 90–95) Study 92. For Company Y, which year is
(70) (80) (120) (125) (75) (150) the following graph to answer the the percent of increase in per cent
A 68 84 77 72 64 82 given questions. profit over that of previous year
B 49 79 62 85 56 76 Per cent profit earned by two the highest ?
C 56 81 68 60 58 68 companies over the given years (A) 2002-03
D 75 85 82 88 72 78
% Profit = (B) 1999-00
E 70 66 65 76 77 83
F 72 70 79 68 68 71 Income – Expenditure (C) 2001-02
× 100 (D) Cannot be determined
Expenditure
Note : Figure into bracket below 93. In 1997-98, the expenditure of
70
each subject indicates the maximum Company X was Rs. 40 crores.
marks allotted. 60 What was its income in that
85. What is the average percentage year ?
of marks obtained by all students 50 (A) Rs. 50 crores
Per cent profit

in subject ‘T’ ? (B) Rs. 48 crores


40
(A) 65·6 (C) Rs. 46 crores
(B) 66·5 30
(D) Cannot be determined
(C) 66·8
20 94. What was the difference in ex-
(D) None of these
penditure of the two companies
86. Marks obtained ‘A’ in subjects 10
in 1999-2000 ?
P, Q and R together are
0 (A) 10
approximately what per cent of
1997-98

1998-99

1999-00

2000-01

2001-02

2002-03

the marks obtained by F in (B) 100


subjects S, T and U together ? (C) 1000
(A) 80 (B) 75 Years (D) Cannot be determined
(C) 85 (D) 105 90. If the income of Company X in 95. In 2002-03 the income of
87. What is the overall percentage of 1998-99 was equal to the expen- Company Y was Rs. 128 crores.
marks obtained by ‘B’ in all the diture of Company Y in 2001- What was its expenditure in that
subjects together (Rounded off 02. What was the ratio of their year ?
to two digits after decimal ? respective profits ?
(A) Rs. 76·8 crores
(A) 70·02 (B) 72·51 (A) 13 : 15
(B) Rs. 64 crores
(C) 67·83 (D) 71·50 (B) 15 : 26
(C) Rs. 48 crores
(C) 13 : 26
88. What are the total marks
(D) Cannot be determined (D) None of these
obtained by D in all the subjects
together ? 91. For Company X its income in Directions : (Q. 96–100) Study
(A) 449·8 (B) 499·9 2001-02 was equal to its the following table to answer these
(C) 480 (D) 490 expenditure in 2002-03. What questions—
Percentage of Marks obtained by seven students in six subjects
(Maximum marks for each subject are shown in bracket)
History Geography Maths Science English Hindi
Subject/Student (75) (60) (80) (50) (40) (40)
P 86 82 95 78 66 84
Q 92 78 85 89 72 76
R 76 94 89 75 62 69
T 67 74 74 84 85 82
M 74 86 64 80 78 72
L 88 89 76 88 70 64
N 90 96 86 92 65 66

Quantitative Aptitude Test | 295


96. Approximately what is the over 98. What is average of marks in (C) 180·2
all percentage of marks obtained Geography obtained by all stu- (D) 184
by ‘T’ in all the subjects ? dents ?
(A) 79 (B) 82 (Answer upto two places of 100. What is the average of percen-
(C) 86 (D) 76 decimals) tage of marks obtained by all
(A) 51·34 (B) 85·57
97. What is the percentage of marks students in Mathematics ?
(C) 52·36 (D) 76·27
obtained by M in all subjects ? 99. What is total of marks obtained (Answer upto two places of
(Answer upto two places of by ‘L’ in History, Geography decimal)
decimal) and Maths ? (A) 81·92 (B) 81·29
(A) 82·74 (B) 84·76 (A) 221·8
(C) 76·84 (D) 74·87 (B) 253 (C) 65·03 (D) 76·23

Answers with Hints


01. (D) Average production of sugar in years 1998, In 2002 the value of sugar per metric ton
1999, 2000 and 2001 102·5
= = Rs. 1·575 lakh
15 + 50 + 30 + 35 65
=
4 In 2003 the value of sugar per metric ton
= 32·5 thousands metric tons 235
= = Rs. 1·566 lakh
and average production of sugar in years 2001, 2002, 2 × 75
2003 and 2004 And in 2004 the value of sugar per metric ton
35 + 65 + 75 + 70 110
= = = Rs. 1·571 lakh
4 70
= 61·25 thousands metric tons ∴ It is the highest in the year 1998.
∴ Required difference 04. (D) It is the lowest in the year 2003.
= 61·25 – 32·5 05. (A) 25% of the total production
= 28·75 thousands metric tons 25
= × 340
02. (A) In 1999 the per cent increase in value from the 100
previous year = 85 thousand metric tons
50 – 15 And the production of the years 1998 and 2004
= × 100% = 233·33% = 15 + 70
15
In 2001 per cent increase in value from the previous = 85 thousand metric tons
year 06. (C) Let the books distributed by a man, a woman and
35 – 30 a child be 5x, 4x and 2x respectively
= × 100% = 16·66%
30 ∴ No. of books distributed in Ist day
In 2003 the per cent increase in value from the = 7 × 5x + 5 × 4x + 8 × 2x
previous year = 71x
75 – 65 No. of books distributed in IInd day
= × 100% = 15·38%
65 = 7 × 5x + 3 × 4x + 5 × 2x
∴ In the remaining there was decrease = 57x
03. (D) In 1998 the value of sugar per metric ton And no. of books distributed in IIIrd day
55 = 4 × 5x + 5 × 4x + 3 × 2x
= = Rs. 1·833 lakh
2 × 15 = 46x
In 1999 the value of sugar per metric ton Q 71x + 57x + 46x = 2000
80 2000
= = Rs. 1·600 lakh x =
50 174
In 2000 the value of sugar per metric ton 2000
57x = × 57 = 650 (App.)
50 174
= = Rs. 1·666 lakh
30 07. (A) Q A:B:C = 8 : 14 : 22
In 2001 the value of sugar per metric ton = 12 : 21 : 33
115 B:C:D = 21 : 33 : 44
= = Rs. 1·642 lakh
2 × 35 ∴ A:B:C:D = 12 : 21 : 33 : 44

Quantitative Aptitude Test | 296


08. (B) Share of each daughter = Rs. 1·25 lakh Profit on 25 kg of C and 30 kg of A
1 = 25 (42 – 32·4) + 30 (30 – 34·5)
∴ Share of grand child = × 1·25
10 = 240 + 105 = Rs. 345
= 0·125 lakh Profit on 40 kg of B and 20 kg of A
And share of each son = 0·125 × 8 = 40 (37 – 28·6) + 20 (40 – 34·5)
= Rs. 1 lakh = 336 + 110 = Rs. 446
∴ Money received by three sons and two daughters
Hence, he will earn maximum in bargain (C)
= 3 × 1 + 2 × 1·25
= Rs. 5·5 lakh odd number 5
13. (A) = 0·625 =
40 even number 8
∴ Money received by his wife = × 5·5
100 14. (C) Area of the Δ
= Rs. 2·2 lakh 1
= × AE × BC
∴ Money received by his wife and three grand 2
children
= 2·2 + 3 × 0·125
1
( 1
= × 2 × AE × BE Because BE = BC
2 2 )
= Rs. 257500 = AE × BE
09. (B) Let the average of runs made by other 6 batsman = (AD + DE) × √
⎯⎯⎯⎯⎯⎯⎯
BD2 – DE2
be x
∴ Runs made by the captain = x + 30 = (r + DE) × ⎯ √⎯⎯⎯⎯⎯⎯
(r2 – DE2)
Q x + 30 + 6x = 310 = (r + DE) × (r – DE) 1/2 (r + DE)1/2
⇒ 7x = 280 = (r + DE) 3/2, (r – DE) 1/2
∴ x = 40 15. (C) Area of the square = 8×8
∴ No. of runs scored by the captain = 64 cm2
= 40 + 30 = 70 ∴ Area of the rectangle = 64 cm2
10. (A) Let the radius of the circle and the height of the ∴ L×B = 64 cm2
right angled Δ be r and h respectively ∴ Length of the rectangle may be 16 times or 32
100 + 20 times of the breadth.
∴ r = h
100
5000 × 100
1 16. (B) Required ratio = = 5 : 14
and area of Δ = × h × 36 = 18 h 40 × 35000
2
∴ Area of the circle = 18 h 17. (A) In 1997, the number of qualified candidates
Q πr2 = 18 h 32·5
= × 5000 = 1625
22 2 18 × 100 × r 100
⇒ r = In 1998, the number of qualified candidates
7 120
18 × 100 × 7 37·5
r = = 4·77 = × 10000 = 3750
120 × 22 100
22 2 22 In 1999, the number of qualified candidates
∴ Area of the circle = r = × 4·77 × 4·77
7 7 30
= × 22500 = 6750
= 72 (App.) sq. cm. 100
11. (B) Let the first and second numbers be x and y In 2000, the number of qualified candidates
respectively 30
= × 15500 = 46500
1 2 40 100
Q x × × = y×
4 3 100 In 2001, the number of qualified candidates
40 4 × 3 32·5
∴ x = y× × = 2·4 y = × 27500
100 2 100
12. (C) Profit on each 20 kg of A and C = 8937·5 = 8938
= 20 (38 – 34·5) + 20 (36 – 32·4) In 2002, the number of qualified candidates
22·5
= 70 + 72 = Rs. 142 = × 30000 = 6750
100
Profit on 20 kg of B and 40 kg of C
In 2003, the number of qualified candidates
= 20 (40 – 28·6) + 40 (38 – 32·4) 35
= 228 + 224 = Rs. 452 = × 20000 = 7000
100
Quantitative Aptitude Test | 297
And in 2004, the number of qualified candidates 26. (A)
40 5
= × 35000 = 14000 7 4 6 9 20 52.5 160.5
100
Hence it is the lowest in 1997.
× 0.5 + 0.5 ×1+1 × 1.5 + 1.5 ×2+2 × 2 + 2.5 ×3+3
18. (D) Required percentage
30000 – 20000 1 90
= × 100% = 33 % 27. (B) 4 6 12 30 75 315 1260
30000 3
19. (A) × 1.5 ×2 × 2.5 ×3 × 3.5 ×4
20. (D) Required percentage 168
6750 × 100 28. (D) 3 4 13 38 87 166 289
= % = 67·50%
10000
+ (1)2 + (3)2 + (5)2 + (7)2 + (9)2 + (11)2
21. (C) 8 4 6 15 52.5 236.25
28
×
1
×
3
×
5
×
7
×
9 29. (C) 4 5 9 29 111 556 3335
2 2 2 2 2

(a) (b) (c) ×1+1 ×2_1 ×3+1 ×4_1 ×5+1 ×6_1


12 6 9 22.5
178
1 3 5 30. (D) 2 6 16 38 84 176 368
× × ×
2 2 2
Hence 22·5 will come in place of (c) ×2+2 ×2+4 ×2+6 ×2+8 × 2 + 10 × 2 + 12

22. (D) 3 13 37 87 191 401 31. (D)


×2+7 × 2 + 11 × 2 + 13 × 2 + 17 × 2 + 19 32. (B) No. of arrangements for boys = 4 = 24

∴ (a) (b) (c) (d) Now, after this we have (4 + 1) places in which 2
1 9 29 71 159 girls can be arranged
∴ No. of arrangements by girls = 5P 2
×2+7 × 2 + 11 × 2 + 13 × 2 + 17
5
Hence 159 will come in place of (d) =
23. (C) –1 –1 3
= 20
5 12 4 10 3 8
∴ Total number of arrangements = 24 × 20
×2+2 ×2+2 ×2+2 = 480
_ 1a (b) (c) _ 1 (d) 33. (D) Money invested by Sunetra for 1 month
= 50000 × 36 = Rs. 1800000
6 14 5 12 4 10 And money invested by Nikhil for 1 month
×2+2 ×2+2 ×2+2
= 80000 × 30 = Rs. 2400000
∴ 4 will come in place of (d) ∴ Ratio in their investments
24. (A) 4 = 1800000 : 2400000
13 40 135 552 2765
= 3:4
×1+1×9 ×2+2×7 ×3+3×5 ×4+4×3 ×5+5×1 ∴ Share of Sunetra in the profit
Similarly, 3 × 24500
= = Rs. 10500
2 (a) = 11 (b) = 36 (c) = 123 (d) = 504 (e) = 2525 3+4

×1+1×9 ×2+2×7 ×3+3×5 ×4+4×3 ×5+5×1 34. (C) 9876 = 9875 (approximate)
Hence 123 will come in place of (c) 24·96 = 25 (approximate)
215·005 = 215 (approximate)
25. (B) 3 19 103 439 1381 2887
309·99 = 310 (approximate)
× 6 + (1)3 × 6 + (2)3 × 4 + (3)3 × 3 + (4)3 × 2 + (5)3 ∴ ? = 9875 ÷ 25 + 215 – 310
Similarly, = 300 (approximate)
5 (a) = 31 (b) = 163 (c) = 679 (d) = 2101 (e) = 4327 35. (B) Let the speed of the boat in still water and the
speed of the stream be x km/hr. and y km/hr res-
Hence 163 will come in place of (b) pectively

Quantitative Aptitude Test | 298


16 Sudha’s age after 3 years = 9x + 3
∴ x+y = =8 …(1)
2 = 9×5+3
16 = 48 years.
and, x–y = =4 …(2)
4
(1 + 100 )
20 3
From (1) and (2) 42. (B) Population in 2007 = 42
x = 6, y = 2
= 42 ( )
6 3

∴ x = 6 km/hr. 5
36. (D) = 72·576 lakhs
43. (B) Let the salaries of A, B and C be Rs. 2x, Rs. 3x
37. (A) Rate of interest for 6 months
and Rs. 5x respectively.
= 5% and time
∴ The salaries of A, B and C after the increase
= 2 half years
2x × 115 3x × 110
(1 + 1005 ) 2 = Rs. , Rs. and
∴ Amount = 15000 100 100
5x × 120
21 21 Rs. respectively
= 15000 × × 100
20 20
∴ Ratio in the new salaries
= Rs. 16537·50
230x 330x 600x
38. (C) Let the number of total children be x = : :
100 100 100
x
∴ Sweets obtained by each child = = 23 : 33 : 60
5
x 44. (D) 3x + 2x = 47
∴ x × = 405 47
5 ∴ x =
⇒ x2 = 2025 5
∴ x = 45 Q 11x = 7y
Hence the number of sweets obtained by each child 47 1 517
∴ y = 11 × × =
45 5 7 35
= =9 47 517 329 – 517
5 ∴ x–y = – =
5 35 35
39. (D) 2x + 3y + z = 55 … (1)
– 188
x–y+z = 4 … (2) =
35
and –x+y+z = 12 … (3)
45. (D) ? × 3 + 30 = 0
From equations (2) and (3) z = 8 30
∴ 2x + 3y + 8 = 55 ? = – = – 10
3
⇒ 2x + 3y = 47 46. (A) ? = 40·83 × 1·02 × 1·2
and x–y+8 = 4 = 49·97592
⇒ x–y = –4
47. (D) 48. (D) 49. (D)
⇒ 2x – 2y = –8
∴ y = 11 50. (C) Reqd. different ways = 3 × 4

40. (A) Quantity of good quality wheat in 150 kg. of = 6 × 24


150 (100 – 10) = 144
wheat = = 135 kg.
[(1 + 100) – 1]
100 12 3
51. (A) C. I. = 25000
And quantity of lower quality in 150 kg of
wheat = 15 kg = 25000 [(1·12)3 – 1]
If x kg of good quality wheat is mixed, then = 25000 [1·404928 – 1]
(150 + x) × 5 = 25000 × 0·404928
15 =
100 = Rs. 10123·20
x = 150 kg. 52. (D)
53. (A) Reqd. different ways = 7C – 3 C3
41. (A) Let Radha’s age = 7x year 3
And Sudha’s age = 9x year 7 3
In view of question = –
7x – 5 3 4 0 3
3
=
9x – 5 4 = 34

Quantitative Aptitude Test | 299


Short Method : Reqd. no. of ways The new number obtained on transposing the digits
= 3 C 2 × 4C 1 + 3 C 1 × 4C 2 + 3 C 0 × 4C 3 = 10 (3x) + 2x
4×3 = 32x
= 3×4+3× +1×4
2×1 By question 32x – 23x = 27
= 12 + 18 + 4 = 34
9x = 27
54. (A) Reqd. number of groups = 7 C 5 × 3C 2 ⇒ x = 3
= 21 × 3 = 63 Original no. = 23x
55. (C) Let the bigger number = x = 23 × 3
∴ Smaller number = (x – 100) = 69
70 4 61. (D)
By question, x× = (x – 100) ×
100 5
62. (C) Required salary after another 4 years
⇒ 70x = 80 (x – 100)
70x = 80x – 8000 = 10000 1 + (50 4
100 )
⇒ 10x = 8000 3 3 3 3
∴ x = 800 = 10000 × × × ×
2 2 2 2
∴ Bigger number = 800 = Rs. 50625
56. (B) Q A+B+C+D = A+B+E+F
∴ C+D = E+F
63. (D) (35 of 350) – (47 of 210)
57. (B) Let the money invested by Mnx in scheme (A) be = 210 – 120 = 90
Rs. P 64. (B) x (x + 3) < 10
∴ Money invested in (B) = Rs. (35000 – P) ⇒ x2 + 3x – 10 < 0
P × 12 × 2 ⇒ (x – 2) (x + 5) < 0
Q = 3600
100 –5<x < 2
3600 × 100
∴ P = 32
12 × 2 17 × x + 12
= Rs. 15000 65. (C) = 10
36 ÷ 9 × 4 – x
⇒ 35000 – P = Rs. 20000
544
∴ Interest accrued in scheme (B) ⇒ x + 12 = 10 (16 –x)
= 20000 [(1+
10 2
100 ) ]
–1

544
x + 12 = 160 – 10x
121 – 100 544
= 20000 × ⇒ 10x + x = 160 – 12
100
= Rs. 4200 ⇒ 10x2 + 544 – 148x = 0
7 × 5 ⇒ 5x2 – 74x + 272 = 0
58. (D) Reqd. no. of ways = ⇒ 5x2 – 40x – 34x + 272 = 0
2 × 3
⇒ 5x (x – 8) – 34 (x – 8) = 0
= 50400
⇒ (x – 8) (5x – 34) = 0
59. (C) Let the number of two digits be 10x + y 34
∴ x = 8 and
∴ (10y + x) – (10x + y) = 18 5
⇒ 9y – 9x = 18 66. (D)
⇒ y–x = 2 67. (A) ? = 167·5 ÷ 83·5 ÷ 27·5
= (2) 30 ÷ (2)10·5 ÷ (2)7·5
and y+x = 8 = 230 – 10·5 – 7·5
∴ x = 3 and y = 5 = 212
∴ Reqd. no. is 35. = (8) 4
68. (B) Work of (3 men + 4 women + 6 children)
60. (D) Let the unit digit = 3x
and the tens digit = 2x = work of (3
2
women + 4 women + women
6
4 )
Then original number = 10 (2x) + 3x = work of 7 women
= 23x ∴ 7 women will complete the work in 7 days.

Quantitative Aptitude Test | 300


12 1
69. (D) 136 × 12 ÷ 9 × 3 = 136 × × 3 = 544 Either q = ±
9 ⎯2

64
⎯√⎯– 256 imaginary
17 × 64 ÷ 6 × 3 = 17 × × 3 = 544
6 or, q =
17
36 × 17 ÷ 9 × 8 = 36 × × 8 = 544 Hence q > p
9
8 75. (A) 1. 3p2 + 15p = – 18
56 × 8 ÷ 14 × 17 = 56 × × 17 = 544
14 ⇒ p2 + 5p + 6 = 0
6
76 × 6 ÷ 19 × 17 = 76 × × 17 = 408 ⇒ (p + 2) (p + 3) = 0
19
∴ p = – 2 or – 3
5
70. (A) No. of boys = × 45 2. q2 + 7q + 12 = 0
9
= 25 ⇒ (q + 3) (q + 4) = 0
4 ∴ q = – 3 or – 4
No. of girls = × 45
9
Hence p > q
= 20
25 × 76 + 20 × 78 ⎯√ 4
Reqd. average marks = 76. (D) 1. p =
(25 + 20)
⎯9

3460
= 2
45 p = ±
= 76·89 3
2. 9q2 – 12q + 4 = 0
71. (B) Total different ways of sitting = 4P
4 × 2P 2
⇒ (3q – 2) 2 = 0
= 4 × 2
= 48 2
q =
3
72. (D) Let the present ages of Seema and Naresh be 5x
years and 7x years respectively. Hence, p ≤ q
5x + 5 3 77. (C) 1. p2 + 13q + 42 = 0
Q =
7x + 5 4
⇒ p2 ± 78 + 42 = 0
⇒ 21x + 15 = 20x + 20
⇒ p2 = – 120 or 36
⇒ x = 5
∴ Present age of Naresh = 35 years ∴ p = ±6
73. (B) 1. 6p2 + 5p + 1 = 0 2. q2 = 36
⇒ (2p + 1) (3p + 1) = 0 ∴ q = ±6
1 1 Hence, p = q
p = – or –
2 3
2. 20q2 + 9q = –1 78. (B) Let the income of B in 1996 and 1997 be Rs. x
and Rs. y respectively
⇒ 20q2 + 9q + 1 = 0
⇒ (5q + 1) (4q + 1) = 0 x – 12
∴ × 100 = 35
1 1 12
q = – or –
5 4 12
⇒ x – 12 = 35 ×
Hence p < q 100
74. (B) 1. 3p2 + 2p – 1 = 0 420
⇒ x = 12 +
(p + 1) (3p – 1) = 0 100
1 = Rs. 16·2 lakhs.
∴ p = – 1 or
3 y – 14·5
2. 2q2 + 7p + 6 = 0 and × 100 = 50
14·5
⇒ 2q2 + 7 ( – 1 or
1
3) +6 = 0 ⇒ y – 14·5 =
50 × 14·5
100
Either 2q2 – 1 = 0
y = 7·25 + 14·5
25
or 2q2 + = 0 = Rs. 21·75 lakhs
3
Quantitative Aptitude Test | 301
∴ Total income in 1996 and 1997 Capital of Suresh =85000 × 2
= 16·2 + 21·75 =Rs. 170000
= Rs. 37·95 lakhs. Ratio of Capitals =220000 : 170000
79. (D) Let the expenditures of companies A and B in =22 : 17
1999 be Rs. 3x lakhs and 4x lakhs respectively 22
Share of Neelesh = × 195000
(22 + 17)
Profit of A = 30%
= Rs. 110000
Income – 3x
∴ 30 = × 100 84. (C)
3x
∴ Income of A in 1999 = Rs. 3·9x lakhs. 85. (D) Reqd. average percentage of marks
64 + 56 + 58 + 72 + 77+ 68
and Profit of B = 40% =
6
Income – 4x = 65·8
∴ 40 = × 100
4x
86. (C) Marks obtained by A in P, Q and R together
⇒ Income of B in 1999 = Rs. 5·6x lakhs 68 × 70 84 × 80 77 × 120
= + +
3·9x 100 100 100
∴ Reqd. ratio = = 39 : 56
5·6x = 47·6 + 67·2 + 92·4 = 207·2
80. (D) Marks obtained by F in S, T and U together
68 × 125 68 × 75 71 × 150
81. (A) Let the expenditures of companies A and B each = + +
100 100 100
in 2000 be Rs. x lakhs.
= 85 + 51 + 106·5 = 242·5
If the income of company A in 2000 be Rs. y lakhs.
207·2 × 100
Then, income of company B in 2000 = Rs. (5·7 – y) ∴ Reqd. percentage =
242·5
lakhs.
= 85·44%
y–x
∴ 40 = x × 100 = 85% (Approximate)

⇒ y = 1·4x 87. (A) Per cent of marks obtained by B in all the subject
together
5·7 – y – x
and 45 = × 100 (49 × 70 + 79 × 80 + 62 × 120
x
⇒ 5·7 = 2·85 x +(85 × 125 + 56 × 75 + 76 × 150)
=
70 + 80 + 120 + 125 + 75 + 150
5·7 × 2
∴ 2x = = Rs. 4 lakhs. (3430 + 6320 + 7440 + 10625 + 4200 + 11400)
2·85 =
620
82. (C) Let the income of company B in 1997 and 1998
be Rs. 2x and Rs. 3x respectively. 43415
= = 70·02
620
If the expenditures of company B in 1997 and 1998
be Rs. y and Rs. z respectively. 88. (B) Total marks obtained by D in all the subjects
together
2x – y 3y
∴ y × 100
50 = ⇒x = 70 × 75 80 × 85 82 × 120 88 × 125
4 = + + +
100 100 100 100
3x – z 1·45 z 72 × 75 78 × 150
z × 100
and 45 = ⇒x = + +
3 100 100
3y 1·45 z = 52·5 + 68 + 98·4 + 110 + 54 + 117
∴ =
4 3
= 499·9
y 1·45 4
⇒ z = 3 ×3 89. (D) Average marks obtained out of 80 by all these x
students together in subjects
5·8
= = 29 : 45 80
9 Q = (84 + 79 + 81 + 85 + 66 + 70)
100 × 6
83. (B) Capital of Neelesh 80 × 465
= = 62
= 40000 × 1 + 50000 × 1 600
+ 60000 × 1 + 70000 × 1 90. (D) Let the income and expenditure of the company
= Rs. 220000 X in 1998-99 be Rs. P crore and T crore respectively.

Quantitative Aptitude Test | 302


P–T 93. (B) Let the income of the company X in 1997-98 be
∴ 30 = × 100 Rs. k crores
T
⇒ P = 1·3 T k – 40
20 = × 100
∴ Profit in 1998-99 of company X 40
20 × 40
= P–T ⇒ = k – 40
100
= Rs. 0·3 T crores
∴ k = Rs. 48 crores
If the income of the company Y in 2001-02 be Rs. 1
crore. 94. (D) Since the income of the companies are not known
1– P so that answer cannot be determined.
Then 50 = × 100
P 95. (D) Let the expenditure of the company Y in 2002-
2 03 be Rs. x crores
⇒ P = I
3 128 – x
1 ∴ 60 = × 100
x
∴ Profit = I
3 ⇒ 60x = 12800 – 100x
1 3 P
= × P = = 0·65 T ⇒ 160x = 12800
3 2 2
∴ Reqd. ratio = 0·3 T : 0·65 T 12800
∴ x = = Rs. 80 crores
160
= 6 : 13
91. (C) Let the income of the company X in 2001-02 be 96. (D) Marks obtained by T in all subjects
Rs. W crore 67 × 75 74 × 60 74 × 80 50 × 84
= + + +
∴ Expenditure of the company X in 2002-03 100 100 100 100
= Rs. W crores 40 × 85 40 × 82
+ +
If the income of the company X in 2002-03 be Rs. 1 100 100
crore = 50·25 + 44·40 + 59·20 + 42·00 + 34·00 + 32·80
1–W = 262·65
then 50 = × 100
W 262·65 × 100
3 ∴ Reqd. percentage =
∴ 1 = W 345
2
= 76 (Approximate)
3
∴ Reqd. ratio = W : W
2 97. (D) Marks obtained by M in all subjects
= 2:3 74 × 75 86 × 60 80 × 64 50 × 80
= + + +
100 100 100 100
92. (C) For Company Y—
40 × 78 40 × 72
In 1998-99 increase in per cent profit over that of + +
100 100
previous year
= 55·50 + 51·60 + 51·26 + 40·00 + 31·20 + 28·80
20 – 15
= × 100 = 33·3 = 258·30
15
In 1999-2001 increase in per cent profit over that of 258·30 × 100
∴ Reqd. percentage = = 74·87
previous year 345
30 – 20 98. (A) Marks obtained in Geography by all the students
= × 100 = 50·00
20 = 49·2 + 46·8 + 56·4 + 44·4 + 51·6 + 53·4 + 57·6
In 2000-01 increase in per cent profit over that of = 359·4
previous year = 0 395·4
In 2001-02 increase in per cent profit over that of ∴ Reqd. average marks = = 51·34
7
previous year
99. (C) Total marks obtained by L in History, Geography
50 – 30
= × 100 = 66·66 and Mathematics
30
= 66 + 53·4 + 60·8 = 180·2
In 2002-03 increase in per cent profit over that of
previous year 100. (B) Reqd. average
60 – 50 95 + 85 + 89 + 74 + 64 + 76 + 86
= × 100 = 20·00 =
50 7
∴ The highest per cent of increase in per cent profit 569
= = 81·29
is in 2001-02 7
Quantitative Aptitude Test | 303
Miscellaneous Exercise – II
Directions : (Q. 1–5) Each of the following questions depends on the (3) Diameter of the base is
table given below— equal to the height of the tank.
Distribution of malnutrition in children in percentage in various (A) Only 1 and 2
years (B) Only 2 and 3
(C) Only 1 and 3
Total surveying Degree of malnutrition
Year (D) Any two of three
number (N) General Mild Moderate Serious
12. How many children are there in
1998 18,000 3·0 14·0 65·0 18·0
the class ?
1999 2,410 3·8 21·9 53·8 20·5 (1) 20% children speak only
2000 1,721 3·4 22·1 52·6 21·8 Hindi
2001 6,775 10·6 41·1 39·8 8·5 (2) 44 children can speak
2002 4,713 14·3 42·4 34·9 8·4 languages other than Hindi
2003 4,008 14·8 47·9 32·6 4·7 (3) There are 30 boys in the
class
2004 9,180 16·5 53·4 28 2·1
(A) All 1, 2 and 3
Total 46,807
(B) Any two of the three
01. In which of the following years 06. 989·001 + 1·00982 × 76·792 = ? (C) 2 and either 1 or 3
there was most improvement in (A) 1000 (B) 1100 (D) 1 and 2 only
degree of malnutrition in com- (C) 1065 (D) 110 13. What is the speed of the train ?
parison of previous year ?
(A) 2000 (B) 2001 07. ⎯⎯⎯⎯⎯⎯⎯
√ 624·9995 + (4·9989)2 (1) Length of the platform is
150% of the length of the train
(C) 2002 (D) 2003 1
=?÷ (2) The train crosses the
02. How many per cent approxi- 4·9900865
platform in 25 seconds
mately was mild malnutrition in (A) 6 (B) 50
(3) The train crosses the signal
all the years together ? (C) 10 (D) 125
pole in 10 seconds
(A) 35 (B) 32 08. 6,23,898 × 99 = ? × 60,000 (A) All 1, 2 and 3
(C) 42 (D) 46 (A) 1000 (B) 1030 (B) 1 and either 2 or 3
03. How many were normal in the (C) 1050 (D) 1065
(C) Only 2 and 3
number of the surveying in all 4 3 6 5
the years ? 09. × ÷ ÷ = ? (D) Question cannot be ans-
5 7 7 9 wered even with the information
(A) 4439 (B) 7723 9 20
(A) (B) in all three statements
(C) 4681 (D) 4192 17 49
14. What is the selling price of the
04. How many cases were moderate 18 1
(C) (D) T.V. set if no discount is
and serious malnutrition together 25 2 offered ?
in the surveying in 1998 ? 2
10. (399·98) = ? (1) Profit earned was 20%
(A) 15,000 (A) 160000 (B) 15999 (2) Had 10% discount been
(B) 11,700 (C) 1600 (D) 1599 offered on selling price the profit
(C) 14,220 Directions : (Q. 11–15) In each would have been Rs. 1200
(D) None of these of the following questions a question (3) Cost price is Rs. 15000
05. What was the approximate is followed by information given in (A) Any two of the three
percentage decrease in moderate three statements. You have to study
(B) Only 1 and 2
cases from 1998 to 2004 ? the question along with statements
and decide the information given in (C) Only 1 and 3
(A) 57 (B) 38
which of the statement(s) is nece- (D) Only 2 and 3
2 ssary and sufficient to answer the
(C) 43 (D) 11 15. What is the average weight of
3 question. girls in the class ?
Directions : (Q. 6–10) What 11. What is the volume of the (1) Average weight of all the 60
approximate value should come in cylindrical tank ? students is 42 kg
place of the question mark (?) in the (1) Area of the base is x square (2) Average weight of boys is
following question (?) metres 43 kg
(You are not expected to calcu- (2) Height of the tank is y (3) Total weight of all the girls
late the exact value) metres together is 1144 kg

Quantitative Aptitude Test | 304


(A) Any two of the three 18. A Candidate’s score (out of 100) 21. Population of two states repre-
(B) All 1, 2 and 3 in different subjects— sented by circle is equal to the
A area of the circle (in lakhs). What
(C) 1 and 2 only is the population of the smaller
B
(D) 2 and 3 only state ?
C
Directions : (Q. 16–21) Study D OP = 21
the information given in each of E
O
these questions to answer the ques- F P
tions. G
16. Cumulative frequency of number
of books purchased— The difference in scores is
maximum between which of the (A) 1386 lakhs
January 150 June 580
two subjects ? (B) 693 lakhs
February 260 July 780 (A) A and F (B) C and F (C) 132 lakhs
March 290 August 1010 (C) D and F (D) A and B (D) Cannot be determined
April 360 September 1120 19. Caloric value of food per 100 22. A 180 metres long train crosses a
May 450 gms. platform of equal length in 18
In which month were highest Food Value seconds. What is the speed of the
number of books purchased ? X 100 train ?
(A) September Y 29 (A) 22 metres/seconds
(B) July Z 110 (B) 10 metres/seconds
A 70 (C) 15 metres/seconds
(C) August
B 40 (D) None of these
(D) Cannot be determined Who consumed the most
17. Percentage of sex (male female) calories ? 23. What would be the compound
and education level (postgraduate (A) Geeta had 200 gms of X, 50 interest drawn on an amount of
or graduate) data of a college gms of A and 200 gms of B Rs. 18,400 @ 12 per cent p.a. at
the end of 3 years ?
(B) Jyoti had 50 gms each of X,
Post Y, Z and B and 250 gms of A (A) Rs. 4680·96
Graduate (C) Meera had 300 gms of Z (B) Rs. 7450·6752
35%
(D) Surekha had 100 gms of (C) Rs. 6235·2143
Graduate
65% each of food item listed above (D) Rs. 8042·16
20. Study the graph and answer the 24. When an odd number of two
question given below— digits is divided by an even
Post y
number of two digits, then
Graduate +
quotient is 0·625. If the odd
25%
number is less than the even
Graduate number by 5, then what is the
75% ratio between odd number and
– +
x even number ?
(A) 5 : 8
(B) 8 : 5
Female – (C) 6 : 9
Male 40%
60% Which of the following is true (D) Cannot be determined
about the value of x and y in the
graph ? 25. A Shopkeeper purchased rice of
3 varieties a, b, c which cost
(A) When x is negative, y is also
Total N = 2300 Rs. 34·50, Rs. 28·60 and
negative
Total how many postgraduates Rs. 32·40 per kg respectively. In
(B) When x is positive, y is which of the following bargain
are there in the college ?
negative he will earn the maximum ?
(A) 805 (C) For each value of y, there (A) He purchased (A) and (C)
(B) 1380 are four corresponding values each 20 kg and sold them at
(C) 690 of x Rs. 38·00 and Rs. 36·00 per kg
(D) None of these (D) None of these respectively

Quantitative Aptitude Test | 305


(B) He purchased (A) and (B) Directions : (Q. 30–34) In each (C) Only 1 and 3
30 kg and 40 kg respectively and of the following questions there is a (D) Either 1 or 2 and 3
sold them Rs. 37·00 and question which is followed by three
Rs. 33·00 per kg respectively statements. Read the question and all 34. To complete the construction
(C) He purchased (B) and (C) the three statements and then decide work how many labours are re-
20 kg and 40 kg respectively and which of the statement(s) is/are quired ?
sold them at Rs. 40·00 and sufficient to answer the question—
(1) 8 labours can complete 20%
Rs. 38·00 per kg respectively 30. What is the area of the hall ? of work in 8 days
(D) He purchased (C) and (A) (1) The cost of the material for (2) 20 labours can complete the
25 kg and 30 kg respectively and the floor is Rs. 250 per square work in 16 days
sold them at Rs. 42·00 and metre
Rs. 38·00 per kg respectively (3) 8 labours can complete the
(2) The cost of labour for the work in 5 days
26. What would be the cost of floor of the hall is Rs. 3500
(A) Only 1 and 3
building a 7 metres wide garden (3) Total cost of the floor of the
around a circular field with dia- hall is Rs. 14500 (B) Only 2 and 3
meter equal to 280 metres. If the (A) Only 1 and 2 (C) Only 1
cost per sq. metre for building
(B) Only 2 and 3 (D) Any one of the three
the garden is Rs. 21 ?
(C) 1, 2 and 3
(A) Rs. 1,56,242 35. Cost of 25 pencils and 20
(D) None of these
(B) Rs. 2,48,521 sharpeners is Rs. 100. If the cost
31. How many per cent was the dis- of a sharpener is more than the
(C) Rs. 1,11,624
count ? cost of a pencil by 50 paise.
(D) None of these What is the total cost of 3 pen-
(1) After allowing the discount,
27. The simple interest accrued on the profit on the article sold for cils and 4 sharpeners ?
an amount of Rs. 14,800 at the Rs. 252 was Rs. 52 (A) Rs. 15
end of three years is Rs. 6,216. (2) If there was no discount then (B) Rs. 25
What would be the compound profit was Rs. 80
interest accrued on the same (3) If there was no discount then (C) Rs. 20
amount at the same rate in the profit was 40% (D) None of these
same period ? (A) Only 1 and 2
(A) Rs. 6,986·1142 Directions : (Q. 36–40) Two
(B) 2 and either 1 or 3 equations 1 and 2 are given in each
(B) Rs. 7,042·2014 (C) Only 1 and 3 question. On the basis of these equa-
(C) Rs. 7,126·8512 (D) 1 and either 2 or 3 tions you have to decide the relation
(D) Rs. 8,321·4166 32. What is the speed of the train ? between p and q and give answer.
28. Vipul decided to donates 5% of (1) The train crosses a railway (A) If no relation can be decided
his salary. On the day of dona- signal pole in 13 seconds between p and q
tion he changed his mind and (2) The train passes a platform
donated Rs. 1687·50. Which was 250 m long in 27 seconds (B) If p > q (C) If q > p
75% of what he had decided (3) The train crosses another (D) If p = q (E) If p ≥ q.
earlier. How much is Vipul’s train going in the same direction
salary ? in 32 seconds 36. (1) p2 + 13p + 40 = 0
(A) Rs. 37,500 (A) Only 1 and 2 (2) q2 + 7q + 12 = 0
(B) Rs. 45,000 (B) Only 1 and 3 37. (1) p = (– 10)2
(C) Rs. 33,750 (C) Only 2 and 3
(2) q2 + q – 9900 = 0
(D) Cannot be determined (D) Any two out of three
38. (1) p2 – 5p + 6 = 0
29. 9 children can complete a piece 33. What is the population of the
state ‘A’ ? (2) q2 – 4q + 3 = 0
of work in 360 days, 18 men can
complete the same piece of work (1) After the increase of 15% 39. (1) p = 10
in 72 days and 12 women can the population of ‘A’ increases
complete the piece of work 162 1·61 lakh (2) q = √

100
days. In how many days can 4 (2) The ratio in the population 40. (1) p = ± 100
man, 12 women and 10 children of the States ‘A’ and ‘B’ is 7 : 8
together complete the piece of (3) The population of the State (2) q = (10000)1/2
work ? ‘B’ is 1·6 lakh Directions : (Q. 41–45) Study
(A) 124 (B) 81 (A) Only 1 the following graph carefully to ans-
(C) 68 (D) 96 (B) Only 2 and 3 wer these questions—

Quantitative Aptitude Test | 306


Number of items produced (in 2 ther number which one of the
(C) Rs. 433
thousands) and cost (in rupees) per 3 following statements is true about
hundred items in six companies. (D) None of these the numbers ?
70 7000 (A) Second number is 2·4 times
43. What is the total cost of items
Number of items produced (in thousand)

Cost (in rupees) per hundred items


produced by companies A and B of the first number
60 6000
together ? (B) First number is 2·4 times of
50 5000 (A) Rs. 17·50 lakh the second number
4000
(B) Rs. 33·25 lakh (C) First number is more than
40
(C) Rs. 15·75 lakh the second number by 40%
30 3000 (D) Rs. 32·75 lakh (D) Second number is less than
44. What was the total number of the first number by 60%
20 2000
items produced by all the com-
47. 8 men alone can complete a
10 1000 panies together ?
piece of work in 12 days. 4
(A) 28500
0 0 women alone can complete the
(B) 258000 same piece of work in 48 days
A B C D E F
(C) 25800 and 10 children alone can
41. What will be the total cost of
items produced by company C ? (D) None of these complete the piece of work in 24
(A) Rs. 32 lakh 45. If the number of items produced days. In how many days can 10
(B) Rs. 24 lakh by company ‘D’ increases by men, 4 women and 10 children
(C) Rs. 27 lakh 30%. What will be the total cost together complete the piece of
(D) Rs. 36 lakh of items produced ? work ?
42. What is the average cost per (A) Rs. 21·45 lakhs (A) 5 (B) 15
hundred items for all the given (B) Rs. 22·45 lakhs
companies ? (C) 28 (D) 6
(C) Rs. 24·25 lakhs
2
(A) Rs. 4183 (D) Rs. 22·25 lakhs Directions : (Q. 48–52) Study
3
1 46. Two-third of one-fourth of a the following table to answer these
(B) Rs. 4283 number is equal to 40% of ano- questions—
3
Percentage of Marks obtained by seven students in six subjects
(Maximum marks for each subject are shown in bracket)

Subject History Geography Maths Science English Hindi

Student (75) (60) (80) (50) (40) (40)


P 86 82 95 78 66 84
Q 92 78 85 89 72 76
R 76 94 89 75 62 69
T 67 74 74 84 85 82
M 74 86 64 80 78 72
L 88 89 76 88 70 64
N 90 96 86 92 65 66

48. Approximately what is the over- 50. What is average of marks in (A) 81·92 (B) 81·29
all percentage of marks obtained Geography obtained by all (C) 65·03 (D) 76·23
students ?
by T in all the subjects ?
(A) 51·34 (B) 85·57 53. The area of a circle is seven
(A) 79 (B) 82 times its circumference. What is
(C) 52·36 (D) 76·27 the circumference of the circle ?
(C) 86 (D) 76
51. What total of marks obtained by (A) 616
49. What is the percentage of marks ‘L’ in History, Geography and
Maths ? (B) 132
obtained by M in all subjects ?
(A) 221·8 (B) 253 (C) 88
(Answer upto two places of (D) Cannot be determined
(C) 180·2 (D) 184
decimal)
52. What is the average of percen- Directions : (Q. 54–58) Study
(A) 82·74 (B) 84·76 the following table carefully to ans-
tage of marks obtained by all
(C) 76·84 (D) 74·87 students in Mathematics ? wer these questions.

Quantitative Aptitude Test | 307


Number of Articles (in thousands) Manufactured (M) and 62. What is the respective ratio of
Defective (D) by 5 units of a company over the years the number of people preferring
to travel by bus to the number of
Units
people prefering to travel by train
Year I II III IV V in the year 2002 ?
M D M D M D M D M D (A) 15 : 11 (B) 9 : 7
1996 53 21 45 12 76 38 56 21 46 18 (C) 7 : 9 (D) 11 : 15
1997 29 18 32 10 45 24 63 24 36 14
63. In the year 2003. If all the
1998 50 18 48 18 55 16 68 30 34 15
airlines reduced their rates by
1999 65 20 68 15 57 20 54 19 48 12
50% and 50% of the people who
2000 70 31 72 13 82 22 48 27 58 10 preferred to travel by train, now
2001 44 15 56 22 38 32 40 15 60 11 preferred airlines, approximately
Directions : (Q. 60–64) Study how many millions of people
54. What is the ratio between total
the following graph carefully and prefer to travel by air ?
number of article manufactured
by unit III to that by unit V for answer the question given below. (A) 290 (B) 308
all the years together ? Preferences of People in Using (C) 330 (D) 325
(A) 353 : 282 (B) 282 : 353 Different Modes of Transport Over 64. The number of people preferring
the Year to travel by train in 2004, was
(C) 457 : 215 (D) 215 : 457 Bus Trains Airlines
how many millions fewer than
55. What is the average number of the number of people preferring
400
defective items from units II for to travel by train in 1999 ?
the given years ? 350
(A) 70 (B) 4
(A) 21,500 (B) 4,000 (C) 75 (D) 50
300
(C) 12,500 (D) 15,000
250 65. Two cars A and B are running in
56. During which year the largest the same direction. Car ‘A’ had
percentage of articles were 200 already covered a distance of
defective out of the articles 150 60 km. When car ‘B’ started
manufactured by unit IV ? running. The cars meet each
(A) 1996 (B) 1997 100 other in 3 hours after car ‘B’
started running. What was the
(C) 1998 (D) 2000 50
speed of car ‘A’ ?
57. What was the percentage (roun- 0 (A) 40 kmph
1999 2000 2001 2002 2003 2004
ded off to nearest integer) of (B) 60 kmph
Years
defective articles over the num- 60. In 2001, the people prefering to (C) 45 kmph
ber of articles manufactured by travel by bus represented (D) Cannot be determined
all units together in the year approximately what per cent of
2001 ? the people preferring to travel by 66. The radius of a circle is more
buses, trains and airlines together than the height of a right angled
(A) 42 (B) 40
in that year ? triangle by 20%. The base of the
(C) 37 (D) 33 (A) 65% (B) 25% right angled triangle is 36 cm. If
58. During which year was the per- (C) 55% (D) 45% the area of the right angled
centage increase/decrease in 61. From 1999 to 2004, the total triangle is equal to the area of
manufacture from the previous number of people who preferred the circle, then what is the
year the highest for unit ? to travel by trains, was appro- approximate area of the circle ?
ximately how many millions ? (A) 72 sq cm (B) 144 sq cm
(A) 1998 (B) 2001 (A) 1300 (B) 1800
(C) 216 sq cm (D) 128 sq cm
(C) 1999 (D) 1997 (C) 1600 (D) 1700
Directions : (Q. 67–71) Study the table carefully to answer the questions
59. A boat takes 8 hours to cover a that follow—
distance while travelling up Sale (in crores) of number of units by six
stream whereas while travelling Different Companies over the years
down stream it takes 6 hours. If
the speed of the current is 4 km/ Year →
1997 1998 1999 2000 2001 2002
ph. What is the speed of the boat Company ↓
in still water ? L 107·0 80·0 28·48 38·0 26·5 36·6
(A) 12 kmph M 175·1 76·0 31·38 43·0 27·5 32·8
(B) 28 kmph N 156·6 66·49 43·7 45·2 19·0 40·12
O 112·4 78·24 56·6 35·1 25·1 25·0
(C) 16 kmph P 95·1 111·8 53·2 48·9 22·5 37·0
(D) Cannot be determined Q 192·0 72·18 31·04 42·2 17·0 30·0

Quantitative Aptitude Test | 308


67. Which year is the percentage and decide that the information in (C) Only 1 and 3
increase/decrease in number of which of the statement(s) is/are nece- (D) Any two of the three
units sold from the previous year ssary to answer the question ?
the lowest for company L ? 77. What is the speed of the train ?
73. What is the principal amount ?
(A) 2000 (B) 2002 (1) The train crosses a signal
(1) Difference between simple pole in 14 secs.
(C) 2001 (D) 1998 interest and compound interest
(2) The train crosses another
68. What is the difference between for 2 years is Rs. 37·50
train in 6 secs.
number of units sold by company (2) Simple interest for 3 year is
O in the year 1997 and the year (3) The train crosses a 200
Rs. 2·250
2000 ? metres long platform in 24 secs.
(3) Simple interest and com-
(A) 7730000000 (A) Only 1 and 2
pound interest for the first year
(B) 773000000 are equal (B) Only 1 and 3
(C) 7030000000 (A) Only 1 (C) Only 2 and 3
(D) 77300000 (B) Only 1 and 2 (D) Any two of the three
69. What is the approximate average (C) Only 1 and 3 78. Subhash starts a business by
number of units sold in the year (D) Any two of the three investing Rs. 25,000, 6 months
1999 ? later Aditya joins him by
74. What is the area of the rectangu- investing Rs. 15,000. After ano-
(A) 407300000
lar garden ? ther 6 months Aditya invests an
(B) 4040000
(1) Perimeter of the garden is additional amount of Rs. 15,000.
(C) 42740000 220 metres At the end of 3 years they earn a
(D) 4073000000 (2) Length and the breadth of profit of Rs. 2,47,000. What is
70. Number of units sold by the garden are in the respective Aditya’s share in the profit ?
company M in the year 2002 is ratio of 7 : 4 (A) Rs. 1,30,000
what per cent of the total number (3) Length is less than twice the (B) Rs. 1,23,000
of units sold by all the companies breadth by 30 metres.
together in that year ? (rounded (C) Rs. 1,05,000
(A) Only 1 and 2
off to two digits after decimal) (D) None of these
(B) Only 1 and 3
(A) 14·16 Directions : (Q. 79–86) What
(C) 1 and either 2 or 3 only
(B) 21·18 will come in place of the question
(D) Only 2 and 3 mark (?) in the following questions ?
(C) 16·28
75. What was the population of State 79. 1984 + 523 – ? = 1899
(D) None of these
‘A’ ? (A) 718 (B) 608
71. Which company has sold the (1) Males and females were in (C) 708 (D) 618
maximum number of units over the ratio of 4 : 5 respectively
the years ? 80. 1400 × ? = 1050
(2) 70% of the population was
(A) Q literate 1 3
(A) (B)
(B) M 4 4
(3) 8 lakh males were literate in
(C) N the State 3 2
(C) (D)
(D) None of these 5 3
(A) Only 1 and 2
72. If the digits of a two digit number (B) Only 2 and 3 81. 40% of ? = 240
are interchanged, the number so (C) Any two of three (A) 60 (B) 6000
obtained is greater than the (D) Question cannot be ans- (C) 960 (D) 600
original number by 27. If the wered even with the information
sum of the two digits of the 82. (1515) 2 = ? ÷ 1515
in all three statements
number is 11. What is the origi- (A) 3030 (B) 235225
nal number ? 76. In how many days can the work (C) 4485 (D) 5115
be completed by 10 women ?
(A) 47
(1) 5 men can complete the 83. 35 + 15 × 1·5 = ?
(B) 38 work in 8 days (A) 75 (B) 5·25
(C) 74 (2) 6 men and 4 women toge- (C) 57·5 (D) 51·5
(D) Cannot be determined ther can complete the work in 5
3
days
Directions : (Q. 73–77) Each of 84. ⎯⎯⎯⎯
√ 19683 = ? × 3
these questions have a questions (3) One man and one woman
(A) 90
followed by information given in together can do thrice the work
done by a woman in one day (B) 7
three statements (1) (2) and (3). You
have to study the question along with (A) Only 1 (C) 3
the information in three statements (B) Only 2 and 3 (D) None of these

Quantitative Aptitude Test | 309


85. 3 + 33 + 333 + 3·33 = ? 91. (c)0 ×b=? batsman of that team who batted
(A) 362·3 (B) 372·33 (A) 1 in the match. If the total runs
(C) 702·33 (D) 702 scored by all the batsman of that
(B) 0 team were 310, how many runs
86. 60 = ?% of 400 (C) cb did the captain score ?
(A) 6 (B) 2 (D) None of these (A) 60
(C) 20 (D) 15
92. [(a)b] ? = 1 (B) 70
87. The area of a square of one side
(A) 1 (C) 50
8 cm is equal to the area of a
rectangle. Which of the following (B) c
(D) Cannot be determined
statement about the rectangle (C) a
99. The average of four number A,
is/are correct ? (D) None of these B, C and D is 40. The average of
(1) The length of the rectangle
Directions : (Q. 93–97) What four number A, B, E and F is
is 16 times of the breadth
should come in place of the question also 40. (A, B are common)
(2) The length of the rectangle Which of the following must be
mark (?) in the following number
is 32 times of the breadth true ?
series ?
(3) The breadth of the rectangle
93. 13 30 66 140 ? 592 (A) (A + B) ≠ (C + D)
1
is of the length (B) (C + D) = (E + F)
6 (A) 210 (B) 290
(4) The breadth of the rectangle (C) 428 (D) 430 (C) Either C = E or F and D = F
1 or E
is of the length 94. 24 ? 109 134 150 159
9 (D) C = E and D = F
(A) 71
(A) Only (1) and (2) 100. Mohan distributed his total
(B) Only (3) and (4) (B) 65
assests to his wife, three sons,
(C) Either (1) or (2) (C) 86 two daughters and five grand
(D) Either (3) or (4) (D) None of these children in such a way that each
Directions : (Q. 88–92) What 95. 3 5 15 ? 1125 84375 grand child got one-eight of each
will come in place of the question son or one-tenth of each
(A) 75 (B) 20
mark (?) in the following questions ? daughter. His wife got 40 per
(a, b, c are integers) (C) 45 (D) 80 cent of the total share of his sons
96. 3 20 78 332 1680 and daughters together. If each
88. (a) c × (b)a × ? = 0
(A) 8410 (B) 9836 daughter received assest of
(A) 1 (B) – 1 worth Rs. 1·25 lakh, what was
(C) 10098 (D) 1150
(C) 0 (D) (c)b the total worth of the assests
97. 17 9 10 ? 35 90 received by his wife and the
89. [(a)2] ? = a6
(A) 21 (B) 27·5 three grand children together ?
(A) 6 (B) 5
(C) 19 (D) 16·5 (A) Rs. 32500
(C) 3 (D) 2a
98. In one-day cricket match the (B) Rs. 257500
90. ? × (| a | × | b |) = – ab captain of one of teams scored
(A) – 1 (B) 1 (C) Rs. 282500
30 runs more than the average
(C) 0 (D) c runs scored by the remaining six (D) Cannot be determined

Answers with Hints


01. (B) There was most improvement in degree of mal- = 540 + 91·58 + 58·51 + 718·15 + 673·95
nutrition in the year 2001 in comparison of previous + 593·18 + 1514·70
year.
= 4190·07
02. (A) Required percentage
= 4192
14·0 + 21·9 + 22·1 + 41·1 + 42·4 + 47·9 + 53·4
=
7 (65 + 18) × 1800
04. (D) Required no. =
242·8 100
= % = 35%
7 = 14940
03. (D) Required number 05. (B) Required percentage = (65·0 – 28)%
3 3·8 3·4 10·6 = 37%
= × 180 + × 2410 + × 1721 +
100 100 100 100
= 38%
14·3 14·8 16·5
× 6775 + × 4713 + × 4008 + × 9180 06. (C) 7. (B) 8. (B) 9. (C) 10. (A)
100 100 100

Quantitative Aptitude Test | 310


11. (D) From 1 area of the base = x sq. m Hence to answer the question all the three statements
From 1 and 2 vol. of the tank = xy m3 are necessary.
16. (C) Months C.F. F.
From 1, diameter of the base =

⎯ 4x
π
January
February
150
260
150
110

⎯√ 4x March 290 30
and From 3 height =
π April 360 70
May 450 90
∴ From 1 and 3 vol. of the tank = x ×

√ 4x 3
π
m June
July
580
780
130
200

(y2) × y.
2
From 2 and 3 vol. of the tank = π August 1010 230
September 1120 110
∴ Any two of the three are sufficient to answer the ∴ The number of books purchased was the highest in
question. August.
12. (D) From 1, 20% children speak Hindi only 17. (D) No. of male in the college
∴ 80% children can speak other languages 60
From 2, 44 children can speak language other than = × 2300 = 1380
100
Hindi. And no. of female in the college
If x is the total number children in the class then 40 × 2300
x × 80% = 44 = = 920
100
∴ For answer 1 and 2 are sufficient. ∴ No. of post graduate in the college
13. (D) From 1, If length of the train be x metres 35 25 × 920
= × 1380 +
3x 100 100
Then length of the platform = metres = 483 + 230 = 713
2
3x 18. (B) The difference of marks obtained between
x+
2 A and F = 2·9
From 2, speed of the train =
25 The difference of marks obtained between
x C and F = 3·4
From 3, speed of the train =
10 The difference of marks obtained between
3x D and F = 0·4
x+
2 x The difference of marks obtained between
=
25 10 A and B = 0·8
as the value of x cannot be found. Hence question The difference of marks obtained between F and G
cannot be answered even with the information in all
∴ The maximum difference is between C and F
three statements.
19. (D) The value of food consumed by Geeta
14. (C) From 1 and 3 S. P. = Rs. 15000 1 + ( 20
100 ) = 200 + 35 + 80 = 315 calorie
Hence 1 and 3 are sufficient to answer the question. The value of food consumed by Jyoti
= 50 + 14·5 + 55 + 20 + 175
15. (B) From 1, total weight of 60 students
= 60 × 42 = 2520 kg = 314·5 calorie
From 3, total weight of all the girls The value of food consumed by Meera
= 1144 kg = 330 calorie
∴ From 1 and 3, total weight of all the boys The value of food consumed by Surekha
= 2520 – 1144 = 80 + 140 = 220 calorie
= 1376 kg And the value of food consumed by Sehnaz
∴ From 2, average weight of boys = 43 kg = 100 + 29 + 110 + 70 + 40
1376 = 349 calorie
∴ From 1, 2 and 3 no. of boys =
43
20. (D)
= 32
∴ No. of girls = 60 – 32 21. (D) Population of the smaller State cannot be deter-
= 28 mined as radius of the small circle is not known.
1144 180 + 180
∴ Average weight of girls = kg 22. (D) Speed of the train = = 20 m/sec.
28 18

Quantitative Aptitude Test | 311


⎡⎢
( ) – 1⎤⎥⎥⎦12 3 28. (B) Let the salary of Vipul be Rs. x
23. (B) C. I. = 18400 ⎢⎣ 1 + 100 5x 75
Q × = Rs. 1687·50
100 100
28 × 28 × 28 – 25 × 25 × 25
= 18400 ( ) 1687·50 × 100 × 100
25 × 25 × 25 ∴ x =
5 × 75
18400 × 6327
= = Rs. 45000
15625
= Rs. 7450·6752 29. (B) Q A piece work is done in 360 days by 9 children
Odd number ∴ A piece work is done = 9 × 360 children
24. (A) = 0·65
Even number Similarly in one day the work will be done by 18 × 72
5 men and one day the work will be done by 162 × 12
= women.
8
∴ 18 × 72 men = 9 × 360 children
25. (C) Profit on each 20 kg of A and C
9 × 360 × 4
= 20 (38 – 34·5) + 20 (36 – 32·4) ∴ 4 men =
18 × 72
= 70 + 72
= Rs. 142 = 10 children
Profit on 30 kg of A and 40 kg of B And 162 × 12 women = 9 × 360 children
= 30 × (37 – 34·5) + 40 (33 – 28·6) 9 × 360 × 12
∴ 12 women =
= Rs. 251 162 × 12
Profit on 20 kg of B and 40 kg of C = 12 children
= 20 (40 – 28·6) + 40 (38 – 32·4) ∴ 4 men + 12 women + 10 children
= 228 + 224 = (10 + 20 + 10)
= Rs. 452 = 40 children
Profit on 25 kg of C and 30 kg of A Q 9 children do a piece of work in 360 days
= 25 (42 – 32·4) + 30 (38 – 34·5)
∴ 40 children do a piece of work in
= 240 + 105
= Rs. 345 360 × 9
= = 81 days
40
Profit on 40 kg of B and 20 kg of A
= 40 (37 – 28·6) + 20 (40 – 34·5) 30. (C) From the statements 2 and 3 cost of material for
= 336 + 110 the hall
= Rs. 446 = 14500 – 3500 = Rs. 11000
Hence, he will earn maximum in bargain (C). And from statement 1 Area of the hall
11000
26. (D) Area of the garden = = 44 m2
250
22
= [(140 + 7)2 – (140) 2 ] 31. (D) From 1 C. P. of the article = 252 – 52
7
22 = Rs. 200
= × 7 × 287 = 6314 m2 From 2 M.P. of the article = 200 + 80
7
= Rs. 280
7m Q Discount on Rs. 280 = Rs. 28
280m
28 × 100
∴ Rate of discount =
280
= 10%
200 (100 + 40)
and from 3 M.P. of the article =
∴ Required cost = 6314 × 21 200
= Rs. 132594 = Rs. 280
6216 × 100 28 × 100
27. (C) Rate = = 14% ∴ Rate of discount = = 10%
14800 × 3 280

∴ C. I. = 14800 [( 1+
14 3
100 ) ]
–1 32. (A) Let the speed of the train be x m/sec.
∴ Length of the train from 1 = x × 13
(57 × 57 × 57 – 50 × 50 × 50)
= 14800 = 13x m
50 × 50 × 50
14800 × 60193 and from 2, 13x + 250 = 27x
=
125000 250
∴ x = m/sec.
= Rs. 7126·8512 14

Quantitative Aptitude Test | 312


33. (D) From 1 population of the State A 45. (A) No. of items produced by D
100 30000 × 130
= 1·61 × lakh = = 39000
15 100
If the population of A be x lakh, then ∴ Total cost of items produced by D
x 7 = 39000 × 55
From 2 and 3, =
1·6 8 = Rs. 21·45 lakhs
Hence either 1 or 2 and 3 are necessary. 46. (B) Let the first and second number be x and y
34. (D) From 1, 20% of work is done in 8 days by 8 respectively
labours. 1 2 40
Q x× × = y×
∴ 100% of work is done in 10 days by 8 labours 4 3 100
8 × 100 8 40 4 × 3
= × ∴ x = y× ×
20 10 100 2
From 2 in 16 days the work is done by 20 labours = 2·4y
∴ In 10 days the work will be done by labours 47. (D) Work is completed in 1 day by
20 × 16 = 12 × 8 = 96 men
=
10 Work is completed in 1 day by
From 3 in 5 days the work is done by 8 labours. = 48 × 4 = 192 women
∴ In 10 days the work is done by labours And work is completed in 1 day by
8×5
= = 24 × 10 = 240 children
10
Q 96 men = 240 children
Hence any one of the three statements is necessary.
240 × 10
35. (D) Let the price of 1 pencil = Rs. x ∴ 10 men = = 25 children
96
Then, the cost of 1 sharpner = Rs. (x + 0·50) Q 192 women = 240 children
By question, 25x + 20 (x + 0·50) = 100
240
⇒ 25x + 20x + 10 = 100 ∴ 4 women = ×4
192
⇒ 45x = 90 = 5 children
⇒ x = 2 ∴ (10 men + 4 women + 0 children)
Therefore, the required price 3x + 4 (x + 0·50) = (25 + 5 + 10)
= 3 × 2 + 4 (2 + 0·50) = 40 children
= Rs. 16 Q 10 children complete one work in 24 days
36. (C) 37. (B) 38. (D) ∴ 40 children complete one work in
39. (D) From 1 p = 10 24 × 10
= = 6 days.
40
From 2 q = ± 10
p ≥ q 48. (D) Marks obtained by T in all subjects
67 × 75 74 × 60 74 × 80 50 × 84
40. (D) From 1 p = ± 100 = + + +
100 100 100 100
From 2 q = ± 100 40 × 85 40 × 82
∴ p = q + +
100 100
41. (C) Reqd. total cost = 45000 × 60 = 50·25 + 44·40 + 59·20 + 42·00
= Rs. 27 lakhs + 34·00 + 32·80
42. (D) Required average cost = 262·65
4500 + 3500 + 6000 + 5500 + 5000 + 4000 262·65 × 100
=
6 ∴ Reqd. percentage =
345
= Rs. 4750 = 76 (Approximate)
43. (B) Total cost of the items produced by companies A
49. (D) Marks obtained by M in all subjects.
and B together
= 35000 × 45 + 50000 × 35 74 × 75 86 × 60 80 × 64 50 × 80
= + + +
= 1575000 + 1750000 100 100 100 100
= Rs. 33·25 lakhs 40 × 78 40 × 72
+ +
44. (D) Reqd. no. = 35000 + 50000 + 45000 + 30000 100 100
+ 40000 + 60000 = 55·50 + 51·60 + 51·26 + 40·00
= 260000 + 31·20 + 28·80

Quantitative Aptitude Test | 313


= 258·30 65 – 50
Reqd. % in 1999 = × 100 = 30%
258·30 × 100 50
Reqd. percentage = = 74·87 70 – 65
345 Reqd. % in 2000 = × 100 = 7·6%
65
50. (A) Marks obtained in Geography by all the students
= 49·2 + 46·8 + 56·4 + 44·4 + 51·6 70 – 44
and Reqd. % in 2001 = × 100 = 37%
+ 53·4 + 57·6 70
= 359·4 ∴ It is the highest % in 1998.
359·4 59. (B) Speed of the boat in still water
Reqd. average marks = = 51·34
7 v (t1 + t2)
=
51. (C) Total marks obtained by L in History, Geography t2 – t1
and Mathematics [Here v = 4, t1 = 6 and t2 = 8]
= 66 + 53·4 + 60·8 = 180·2 4 (6 + 8)
52. (B) Reqd. average = = 28 km/hr.
8–6
95 + 85 + 89 + 74 + 64 + 76 + 86
= 375 × 100
7 60. (D) Reqd. percentage =
(375 + 300 + 175)
567
= = 81·29 = 45%
7
61. (B) Reqd. number = 350 + 300 + 300 + 275
53. (C) Q πr2 = 7 × 2πr
+ 300 + 275
∴ r = 14 = 1800
∴ Circumference of the circle
375
22 62. (A) Reqd. ratio = = 15 : 11
= 2 × × 14 = 88 275
7
1
76 + 45 + 55 + 57 + 82 + 38 63. (D) Reqd. number = 175 + × 300
54. (A) Reqd. ratio = 2
46 + 36 + 34 + 48 + 58 + 60
= 175 + 150 = 325
353
= 64. (C) Reqd. number = 350 – 275
282
= 75 millions
12 + 10 + 18 + 15 + 13 + 22
55. (D) Reqd. average = 65. (D)
6
90 66. (A) Let the radius of the circle and the height of the
= thousands = 15,000
6 right angled Δ be r and h respectively.
56. (D) % of defective article in 1996 (100 + 20)
r = h
21 100
= × 100 = 37·5% 1
56 and area of Δ = × h × 36
2
% of defective article in 1997
= 18 h
24
= × 100 = 38·1% ∴ Area of the circle = 18 h
63
πr2 = 18 h
% of defective articles in 1998
22 2 18 × 100 × r
30 × 100 ⇒ r =
= = 44·1% 7 120
68
18 × 100 × 7
% of defective article in 1999 r =
120 × 22
19 × 100
= = 35·2% = 4·77
54
22 2
% of defective article in 2000 ∴ Area of the circle = r
7
27 × 100
= = 56·3% 22
48 = × 4·77 × 4·77
7
∴ The largest % of articles were defective in 2000. = 72 sq. cm.
95 × 100 67. (D) % Decrease from the year in 1998
57. (B) Reqd. percentage = = 40%
238 107 – 80
= × 100 = 25·23%
53 – 29 107
58. (A) Reqd. % in 1997 = × 100 = 45·3%
53 % Decrease from the year in 1999
50 – 29 80 – 28·48
Read. % in 1998 = × 100 = 72·4% = × 100 = 64·40%
29 80
Quantitative Aptitude Test | 314
% Increase from the year in 2000 P× r×3
From statement (2) 2250 =
38 – 28·48 100
= × 100 = 33·42%
28·48 ∴ Pr = 75000 …(2)
% Decrease from the year in 2001 From statement (3)
38 – 26·5
× 100 = 30·26% P× r×1
=
38
and % increase from the year in 2002
100
= P [( 1+
r
100
–1) ] …(3)

So, the principal can be found out by either two of


36·6 – 26·5
= × 100 = 38·11% the aforesaid three statements.
26·5
∴ Reqd. year is 1998. 74. (C) From statement 1 2 (L + b) = 220
68. (B) Reqd. difference = (112·4 – 35·1) crores ⇒ L+b = 110
= 77·3 crores From statement 2
= 773000000 Let the length of garden L = 7x
69. (A) Reqd. average and breadth b = 4x
28·48 + 31·38 + 43·7 + 56·6 + 53·2 + 31·04 or From statement 3 L = 2b – 30
=
6
75. (D) The question cannot be answered despite the
244·4
= crores knowledge of the three statements.
6
= 407300000 76. (D)
32·8 × 100 77. (B) From statement 1
70. (C) Reqd. percentage = = 16·28%
201·52 Length of train = Speed × 14 …(1)
71. (B) No. of units sold by company L over the years From statement 3
= 107 + 80 + 28·48 + 38 + 26·5 + 36·6 Length of train = Speed × 24 – 200 …(2)
= 316·58 Therefore, we can determine the speed of the train
No. of units sold by company M over the years from statement 1 and 3.
= 175·1 + 76 + 31·38 + 43 + 27·5 + 32·8 78. (D) Q Investment of Subhash for 3 years
= 385·78
= Rs. 25,000
No. of units sold by company N over the years
= 156·6 + 66·49 + 43·7 + 45·2 + 19 + 40·12 ∴ Investment of Subhash for 1 month
= 371·11 = 25,000 × 36
No. of units sold by company O over the years = Rs. 9,00,000
= 112·4 + 78·24 + 56·6 + 35·1 + 25·1 + 25 And investment of Aditya for 1 month
= 332·44 = 15,000 × 30 + 15,000 × 24
No. of units sold by company P over the years = Rs. 8,10,000
= 95·1 + 111·8 + 53·2 + 48·9 + 22·5 + 37 ∴ Ratio in their investments
= 368·5 = 9,00,000 : 8,10,000
No. of units sold by company Q over the years = 10 : 9
= 192 + 72·18 + 31·04 + 42·2 + 17 + 30 ∴ Aditya’s share in the profit
= 382·42 9
∴ Max. no. of units is sold by company M. = × 2,47,000
19
72. (A) Let the original no. = 10x + y = Rs. 1,17,000.
and x+y = 1 …(1) 79. (B) 80. (B) 81. (D) 82. (D) 83. (C)
and (10y + x) – (10x + 4) = 27 84. (D) 85. (B) 86. (D)
y–x = 3 …(2) 87. (C) Area of the square = 8 × 8 = 64 cm2
From equation (1) and (2) ∴ Area of the rectangle = 64 cm2
x = 4 and y = 7 ∴ L×B = 64 cm2
∴ Required no. = 47 ∴ Length of the rectangle may be 16 times or 32
times of the breadth.
Pr2
73. (D) From statement (1) 37·50 = 88. (C) Q (a) c × (b)a × ? = 0
100 × 100
∴ Pr2 = 375000 …(1) ∴ ? = 0

Quantitative Aptitude Test | 315


89. (C) [(a2 )] ? = a6 96. (C)
3
∴ a2 ×? = a6 ( + 7) × 2
6 20
∴ ? = =3
2 ( + 6) × 3
78
90. (A) Q ? × (| a | × | b |) = – ab ( + 5) × 4
⇒ ? × a × b = – ab 332
( + 4) × 5
∴ ? = –1 1680
91. (D) ? = (c)0 × b ( + 3) × 6
10098 ?
= 1 × b =b 97. (D)
17
92. (D) Q [(a)b] ? = 1 ( + 1) × 1
2
⇒ (a) b × ? = a0 9
( + 1) × 1
∴ ? = 0 10
( + 1) × 1.5
93. (B) 16.5 ?
13 ( + 1) × 2
×2+4
35
30
( + 1) × 2.5
×2+6
90
66
98. (B) Let the average of runs made by other 6 batsmen
×2+8 be x
140 ∴ Runs made by the Captain = x + 30
× 2 + 10 Q x + 30 + 6x = 310
290 ?
⇒ 7x = 280
× 2 + 12
∴ x = 40
592
∴ No. of runs scored by the Captain = 40 + 30
94. (D) = 70
24 99. (B) Q A+B+C+D = A+B+E+F
+ (7)2
73 ? ∴ C+D = E+F
+ (6)2
100. (B) Share of each daughter = Rs. 1·25 lakh
1
109 ∴ Share of grand child = × 1·25
10
+ (5)2
= 0·125 lakh
134
And share of each son = 0·125 × 8
+ (4)2 = Rs. 1 lakh
150 ∴ Money received by three sons and two daughters
+ (3)2 = 3 × 1 + 2 × 1·25
159 = Rs. 5·5 lakh
40
95. (A) ∴ Money received by his wife = × 5·5
100
3 × 5 = 15
= Rs. 2·2 lakh
5 × 15 = 75 ∴ Money received by his wife and three grand
15 × 75 = 1125 children = 2·2 + 3 × 0·125
75 × 1125 = 84375 = Rs. 257500

Quantitative Aptitude Test | 316


Miscellaneous Exercise – III
Directions : (Q. 1–5) In each of (A) Only (2) and (3) are Directions : (Q. 6–10) In each
the following questions, a is followed sufficient of these questions two equations 1
by information given in three (B) Only (1) and (3) are and 2 are given. You have to solve
statements. You have to decide the sufficient both the equations and give answer—
information given in which of the (C) Only (1) and (2) are (A) If a < b
statements is necessary and sufficient sufficient
to answer the question. (B) If a > b
(D) (1), (2) and (3) together are
01. The company earned how much not sufficient to answer the ques- (C) If relationship between a
profit in the year 2002 ? tion. and b cannot be established
(1) In 2001 the company earned
04. In a business of A, B and C (D) If a ≥ b
40% more profit which earned in
the year 2003 together, what profit B earned in (E) If a ≤ b
(2) The company earned the 2 years—
06. (1) 4a2 – 20a + 21 = 0
profit of 20 crores together in the (1) A and B started a business
year 2001 and 2002 with the capital in the ratio of 3 : (2) 2b2 – 5b + 3 = 0
(3) The company earned in 5 respectively
07. (1) 6a2 – 25a + 25 = 0
2003, 80% profit which was (2) C joined them after 6 months
with a capital of Rs. 4 lakh (2) 15b2 – 16b + 4 = 0
earned in the year 2002
(A) Any two of (1), (2) and (3) (3) At the end of 2 years the 08. (1) a2 = 4
are sufficient share of A in the profit was
(2) b2 = 9
(B) Either (1) and (2) or (2) and Rs. 60,000
(3) are sufficient (A) Only (1) and (3) are suffi- 09. (1) 2a2 + 3a + 1 = 0
(C) (1) and (2) or (3) are cient (2) 12b2 + 7b + 1 = 0
sufficient (B) Only (2) and (3) are suffi-
cient 10. (1) a2 + 5a + 6 = 0
(D) (1), (2) and (3) all the three
are necessary to answer (C) Either (1) or (2) and (3) are (2) b2 + 3b + 2 = 0
sufficient
02. What is monthly salary of an Directions : (Q. 11–16) In each
assistant of company in which (D) All the three are necessary of the following number series, a
managers, supervisors and assis- to answer the question wrong number is given, find out the
tants are appointed ? 05. How many students secured at wrong number—
(1) Each supervisor gets more least per cent marks in 11. 2 3 6 18 109 1944 209952
than an assistant by Rs. 12000 Mathematics in a class of 240 (A) 3 (B) 6
per month students ?
(C) 18 (D) 109
(2) Total salary of a supervisor (1) 20 per cent of the students
and an assistant is Rs. 32000 per in the class secured 80 per cent 12. 1 3 6 11 20 39 70
month and above marks in Mathematics (A) 3 (B) 39
(3) Total salary of a manager (2) 80 students have secured (C) 11 (D) 20
and a supervisor is Rs. 57000 per more than 50 per cent but less
month 13. 2 13 27 113 561 3369 23581
than 60 per cent marks in Mathe-
(A) Either (1) or (2) and (3) matics (A) 13 (B) 27
(B) Only (1) and (2) (3) The number of students who (C) 113 (D) 561
(C) Any two of (1), (2) and (3) secured marks between 60 and 14. 50 51 47 56 42 65 29
(D) (1), (2) and (3) all the three 79 per cent was equal to the (A) 51 (B) 47
are necessary number of students who secured (C) 56 (D) 42
03. What is the measurement of less than 50 per cent marks in
Mathematics 15. 3 9 23 99 479 2881 20159
perimeter of a semicircle in cm ?
(1) Area of the semicircle is (A) All (1) and (3) are necessary (A) 9 (B) 23
equal to the area of the to answer the question (C) 99 (D) 479
parallelogram (B) Only (1) and (3) are 16. 2 4 5 8 13 21 34
(2) Length of the parallelogram sufficient
(A) 4 (B) 5
is 1·5 times of the radius of the (C) Only (2) and (3) are
sufficient (C) 8 (D) 13
semicircle
(3) The difference of the length (D) The question cannot be Directions : (Q. 17–21) Study
and breadth of a parallelogram is answered even with all (1), (2) the following table carefully to ans-
8 cm and (3) wer these questions—

Quantitative Aptitude Test | 317


Number of students appeared and passed over the years in various state
Year 1995 1996 1997 1998 1999
State Present Passed Present Passed Present Passed Present Passed Present Passed
A 15250 1800 17800 1750 14500 1350 15400 1640 16300 1725
B 12500 1100 15400 1480 13800 1420 14000 1525 14550 1500
C 14000 1550 16200 1640 15500 1480 17500 1880 15800 1620
D 17600 1640 16500 1560 14800 1575 15600 1480 17200 1750
E 16400 1480 14700 1820 17000 1600 16400 1500 18000 1950
F 14500 1250 16800 1780 17200 1780 15200 1450 16800 1725
G 15000 1400 15200 1650 16400 1840 17200 1820 17000 1850
Total 107250 10220 112600 11680 109200 11045 111300 11295 115650 12170

17. What is the percentage of passed B, C and D are in the ratio of 21 31. 1 6 36 240 1960 ?
candidates to the appeared stu- : 33 : 44 respectively. Which of (A) 19660 (B) 3680
dents in 1998 and 1999 together the following represents the ratio
of the present ages of A, B, C (C) 36800 (D) 19600
of all the states ? (Answer upto
two places of decimal) and D respectively ? 32. 12 14 17 13 8 14 21 13 4 ?
(A) 10·84 (B) 10·32 (A) 12 : 21 : 33 : 44 (A) 14 (B) 13
(C) 10·62 (D) 10·34 (B) 12 : 22 : 31 : 44 (C) 15 (D) 2
18. In which state the percentage of (C) 12 : 21 : 36 : 44
33. 25 7 12 19 31 50 ?
passed students to the appeared (D) None of these
students in 1995 is the least ? Directions : (Q. 24–28) What (A) 53
(A) A (B) F approximate value should come in (B) 81
place of the question mark (?) in the (C) 69
(C) B (D) D
following questions ? (You are not
19. What is the percentage of passed (D) None of these
expected to calculate the exact value)
students to the appeared students 34. 15 12 17 10 ? 8 25 6
24. 12 × 958 ÷ 17 = ?
in 1997 of the states C and D (A) 3 (B) 17
together ? (A) 532 (B) 676
(C) 21 (D) 19
(A) 10 (B) 12·5 (C) 765 (D) 483
35. 4 6 12 30 90 315 ?
(C) 15 (D) 20 25. 15·002 × ? × 25·0210 = 7113·918
(A) 945 (B) 102
20. What is the average approximate (A) 19 (B) 26
(C) 1260 (D) 1417·5
number of passed students in (C) 11 (D) 31
1998 all states ? 26. 81·38 × 81·63 = ? Directions : (Q. 36–40) Each of
(A) 1630 (B) 1516 the questions is based on the table
(A) 680 (B) 218 given below—
(C) 1615 (D) 1545 (C) 726 (D) 512
36. Which of the following graphs
21. What is the average number of well illustrates expenditure for
students appeared in B for all 27. ⎯⎯⎯⎯
√ 1000 = ?
water supply and cleaning in the
years ? (A) 10 (B) 24 ratio of expenditure in public
(A) 15850 (C) 45 (D) 32 sector under various plannings ?
(B) 14550 28. 59·99% of 255·012 + 22·98% of Planning
182·005 = ? 5
(C) 15050
4
(D) None of these (A) 162 (B) 146 (A)
3
22. A right circular cylindrical tank (C) 195 (D) 225 2
has the storage capacity of 1
Directions : (Q. 29–35) What
38808 ml. If the radius of the will come in place of the question 0
III IV V VI
base of the cylinder is three mark (?) in the following numbers ?
fourth of the height. What is the Planning
29. 1 ? 27 64 125
diameter of the base ? 5
(A) 8 (B) 4 4
(A) 28 cm (B) 56 cm (B)
(C) 6 (D) 9 3
(C) 21 cm (D) 42 cm 2
23. The present ages of A, B and C 30. 25 16 ? 4 1 1
are in the ratio of 8 : 14 : 22 (A) 3 (B) 6 0
respectively. The present ages of (C) 12 (D) 9 III IV V VI

Quantitative Aptitude Test | 318


Planning 43. 127·001 × 7·998 + 6·05 × 4·001 49. (1) 4x2 – 4x – 3 = 0
5 =?
4 (2) 4y2 + 12y + 5 = 0
(C) (A) 1440 (B) 1400
3
(C) 1000 (D) 1040 50. (1) 4x2 = 49
2
1 (2) 9y2 – 66y + 121 = 0
Directions : (Q. 44–47) Study
0 the following graph to answer the 51. (1) x2 + 9x + 14 = 0
III IV V VI
given questions—
(2) y2 + y – 2 = 0
Planning Production of a company (in Lakh
5 52. (1) 9x 2 – 18x + 5 = 0
4 Units) over the given years
(D) (2) 2y 2 – 9y + 10 = 0
3 35
2
53. A committee of 6 members is to
1 30
be selected from a group of 8
0 men and 6 women in such a way
III IV V VI 25
that at least 3 men are there in
37. For which planning the expendi- 20 the committee. In how many
ture of social services in ratio of different ways can it be done ?
expenditure of public service 15 (A) 2506 (B) 2534
sector is maximum ? (C) 1120 (D) 1050
10
(A) (I) (B) (VI)
Directions : (Q. 54–58) In each
(C) (V) (D) (II) 5
of the following questions a question
38. In which sector there is a conti- 0 is followed by information given in
three statements. You have to study
1996
1997
1998
1999
2000
2001

nuous decrease in expenditure in 2002


ratio of expenditure in public the question alongwith the statement
sector in successive planning ? Years and decide the information given in
44. The production in 2002 is what which of the statement(s) is nece-
(A) Not in any sector
per cent of production in 1996 ? ssary to answer the question.
(B) Health
(A) 650 (B) 550 54. In how many days can 16 men
(C) Education
(C) 325 (D) 320 and 8 women together complete
(D) Social Services the piece of work ?
45. What is the approximate average
39. In VI planning, how many per production (in lakhs) for the (1) 8 men complete the piece of
cent expenditure of public sector given years ? work in 10 days
planning is for housing and city (A) 18 (B) 19 (2) 16 women complete the
services ? piece of work in 10 days
(C) 20 (D) 18·5
(A) 0·35 (B) 25 (3) 5 women take 32 days to
46. Which of the following is the
(C) 25·5 (D) 2·5 complete the piece of work
highest difference in production
40. What was the total difference of between two adjacent years ? (A) Only (1) and (2)
expenditure in Education and (A) 5 lakhs (B) 10 lakhs (B) Only (2) and (3)
Health for all planning ? (C) 9 lakhs (D) 7·5 lakhs (C) Only (1) and (3)
(A) Rs. 220400000 (D) Only (1) and either (2) or
47. Which year had the highest per
(B) Rs. 224000000 cent increase in production over (3)
(C) Rs. 22040000000 the previous year ? 55. What is the speed of the train ?
(D) Rs. 220400000000 (A) 2000 (B) 1999 (1) Train crosses a pole in 10
(C) 2002 (D) 1997 seconds
Directions : (Q. 41–43) What
approximate value will come in place Directions : (Q. 48–52) In each (2) Length of the train is 240
of the question mark (?) in the of the following questions a pair of metres
following equations ? equations is given. You have to find (3) Train crosses a platform of
out the value of x and y and give equal length in 20 seconds
41. 125% of 4875 + 88·005 × 14·995 answer— (A) Only (1) and (2)
=?
(A) If x < y (B) If x ≤ y (B) Only (2) and (3)
(A) 7395 (B) 7485
(C) All (1), (2) and (3)
(C) 7514 (D) 7415 (C) If x = y (D) If x > y
(D) Any two of the three
42. 1010 ÷ 36 + 187 × 20·05 = ? (E) If x ≥ y
56. What is the area of the square ?
(A) 3650 (B) 3770 48. (1) 2x2 – 7x + 6 = 0 (1) Measure of diagonal of the
(C) 3825 (D) 3800 (2) 4y2 = 9 square is given

Quantitative Aptitude Test | 319


(2) Measure of one side of 9 ÷ 2 × 27 ÷ 9 67. In which of the following years
62. =?
square is given 18 ÷ 7·5 × 5 ÷ 4 was the value of sugar per metric
(3) Perimeter of the square is (A) 4·5 (B) 5·7 ton the lowest among the given
given years ?
(C) 2·5 (D) 6·8
(A) Only (2) (A) 1998 (B) 1999
63. ? % of 280 + 18% of 550 = (C) 2001 (D) 2003
(B) Only (3) 143·8
(C) Only (1) and (3) 68. In which of the pair of years the
(A) 11 (B) 18
(D) Only (2) and (3) sum of production is exactly
(C) 21 (D) 16 25% of the production of all the
57. What is the two digit number ? Directions : (Q. 64–68) Study years together ?
(1) The number obtained by the following graph carefully and (1) 1998 and 2004
interchanging the digits of the answer the question given below— (2) 1998 and 2003
number is greater than the
original number by 18 Quantity and value of production (3) 1998 and 2001
of sugar of a company over the (A) Only (1)
(2) Sum of the two digits of the years (B) Only (2)
number in 14
(3) Difference between the two 80 125 (C) Only (3)
digits of the number is 2 (D) Only (1) and (3)
Quantity in thousand metric tones

70 110
(A) Any two of the three (E) (1), (2) and (3)
(B) Only (1) and (2) 60 95 Directions : (Q. 69–73) Study

Value in lakh Rs.


(C) (2) and either (1) or (3) 50 80
the following table carefully to
(D) All the three answer these questions—
40 65
58. What is the rate of interest Sales of a product (in million tons)
p.c.p.a. ? 30 50 for six states over the years
(1) Simple interest earned per 35 States
20 Years
annum is Rs. 5,300 A B C D E F
(2) The difference between the 10 20
1998 25 45 38 52 47 55
compound and simple interest on
0 5 1999 32 39 40 55 46 67
an amount is Rs. 1,060 at the end
1998
1999
2000
2001
2002
2003
2004

of 2 years 2000 41 50 43 57 39 64
(3) An amount doubles itself in Years 2001 37 48 43 58 32 72
5 years with simple interest 2002 28 53 46 62 37 58
64. What was the difference between
(A) All the three the average production of sugar 2003 43 55 49 63 42 62
(B) Only (3) during the years 1998, 1999,
(C) Either (2) or (3) 2000 and 2001 and that of 2001, 69. If the cost of product per
(D) Only (3) or (1) and (2) 2002, 2003 and 2004 in thousand thousand tons in 1998 was
metric tons ? Rs. 1·8 lakh. What was the cost
Directions : (Q. 59–63) What (A) 15 of average sales for the given
should come in place of the question states in that year ?
(B) 20
mark (?) in the following ques-
(C) 25 (A) Rs. 786000 lakhs
tions ?
(D) None of these (B) Rs. 786 lakhs
4 3 1
59. 1 +1 +1 =? 65. In which of the following years (C) Rs. 7860 lakhs
7 5 3
47 58 was the percentage increase in (D) Rs. 78600 lakhs
(A) 5 (B) 4 value of sugar per metric ton
105 105 70. Total sales in year 2000 were
from the previous year the
53 43 what per cent of the total sales in
(C) 4 (D) 5 maximum ?
105 105 year 2003 ? (Rounded off to two
(A) 1999 (B) 2000 digits after decimal)
60. 8·88 × 88·8 × 88 = ? (C) 2001 (D) 2003 (A) 93·63 (B) 92·65
(A) 68301·142 66. In which of the following years (C) 106·80 (D) 93·23
(B) 79391·642 was the value of sugar per metric
(C) 65365·824 ton the highest among the given 71. What was the percentage
years ? increase in total sales in 2003
(D) None of these from 1998 ?
(A) 1999
61. ⎯⎯⎯⎯⎯⎯⎯⎯
√ ⎯⎯⎯⎯
√ 2500 + ⎯
√⎯⎯
961 = (?)2 (B) 2004 (Rounded off to nearest integer)
(A) 81 (B) 3 (C) 2003 (A) 19 (B) 20
(C) 6561 (D) 9 (D) None of these (C) 16 (D) 17

Quantitative Aptitude Test | 320


72. Approximately what was the 75. If the income of company A had 4
(A) x > 3, x <
ratio between average sales of increased by 10% in year 2000 3
states B and C respectively ? from year 1999 and profit earned 4
(A) 31 : 25 (B) 26 : 31 in 1999 was 20%. What was its (B) x > – 3, x < –
3
(C) 29 : 24 (D) 29 : 26 expenditure in 1999 ? (In crore
4
Rs. rounded off to two decimal (C) <x<3
73. For which of the following years places) 3
the percentage increase/decrease (A) 36·36 4
in sales from the previous years (D) ≤x≤3
(B) 32·32 3
was highest for state ‘E’ ?
(C) 30·30 81. Abhishek invested an amount of
(A) 1999 (B) 2000 Rs. 29,000 in two parts under
(D) Cannot be determined
(C) 2001 (D) 2002 two different schemes A and B
76. If the expenditure of company G and earned a total interest of
Directions : (Q. 74–78) Study
in 2000 was 20% more than its Rs. 3,840. Schemes A and B
the following graph carefully and
expenditure in the year 1999 and offered 15% and 12% interest
answer the question given below it.
the company had earned a profit respectively. What was the
Account of Income and of 10% in 1999. What was the amount invested in scheme ‘A’ ?
Expenditure (in crore Rs.) of company’s income in 1999 in (A) Rs. 17,000
seven companies in the year crore Rs. ? (B) Rs. 12,000
2000 (A) 37·5 (C) Rs. 14,000
% Profit/loss (B) 41·25 (D) Data inadequate
Income – Expenditure (C) 34·09 82. What approximate value should
= × 100
Expenditure (D) Cannot be determined come in place of the question
Expenditure Income mark (?) in the following
77. Which of the following compa-
60 nies had the highest percentage equation ? 695·95 ÷ 29·07 × ? +
55
of profit/loss in year 2000 ? 40·25 = 399·99—
50
(A) B (B) C (A) 14 (B) 17
45
40 (C) F (D) A (C) 12 (D) 15
35 83. If the numerator of a fraction is
30 78. What is the approximate percen- increased by 40% and the
25 tage of profit earned by all the denominator is doubled the new
20 companies together in the year 7
15 2000 ? fraction obtained is . What
16
10 (A) 11 (B) 11·5 was the original fraction ?
5
0
(C) 10·5 (D) 12 5
(A)
A B C D E F G 79. In how many different ways can 8
Company
the letters of the word ADJUST 3
(B)
74. Approximately what is the per- be arranged so that the vowels 8
centage of profit/loss that com- never come together ? 7
panies C and D had together ? (A) 720 (B) 120 (C)
8
(A) 10% profit (C) 240 (D) 480 (D) Cannot be determined
(B) 12% profit 80. For which of the following values Directions : (Q. 84–88) Study
(C) 10% loss of x the inequality 3 (x2 – 4x + 4) the table carefully to answer the
(D) There was no loss or profit < x gets satisfied ? following questions—

Number of cars (in thousands) manufactured and sold by six companies over the year

Company A B C D E F
Year Manufac- Sold Manufac- Sold Manufac- Sold Manufac- Sold Manufac- Sold Manufac- Sold
tured tured tured tured tured tured
2000 2·58 1·96 1·98 1·62 1·97 1·53 2·46 2·11 2·35 2·16 1·88 1·50
2001 2·34 1·98 2·15 2·00 2·20 2·03 2·46 2·14 2·45 2·20 1·95 1·62
2002 2·85 2·05 2·35 1·99 2·18 1·87 2·55 2·23 2·60 2·13 2·25 1·93
2003 2·87 2·11 2·62 2·01 2·25 1·95 2·62 2·30 2·79 2·31 2·39 2·08
2004 2·91 2·22 2·71 2·12 2·68 2·32 2·71 2·19 2·88 2·19 2·58 2·10
2005 2·94 2·25 2·84 2·15 2·86 2·36 2·76 2·28 2·90 2·32 2·67 2·30

Quantitative Aptitude Test | 321


84. What is the total number of cars forms approximately what per Departmentwise distribution of
sold by company C in all the cent of the total number of employees
year together ? students studying Commerce in Total number of employees = 1200
(A) 120600 (B) 14205 all the years together ? Accounts
(A) 13 (B) 20 Operations
(C) 12060 (D) 142050 14%
(C) 29 (D) 33 28%
85. What is the approximate per cent 18% Administration
increase in the number of cars 90. What is the total number of stu-
dents studying Arts in all the 8%
sold by company F in the year 17%
15%
2004 from the previous year ? years together ? Computers Personal
(A) 3700
(A) 13 (B) 19 Marketing
(B) 2750 94. What was the total expenditure
(C) 0·96 (D) 8
(C) 3500 on accounts department ?
86. What is respective ratio of total (D) None of these (A) Rs. 16·8 lakh
number of cars manufactured by
91. Number of students studying (B) Rs. 1680 lakh
companies A, B and C together
in the year 2001 to those manu- Science in the year 2001 forms (C) Rs. 18·6 million
factured by companies D, E and what per cent of total number of (D) Rs. 16·8 million
F together in the year 2003 ? students studying all the disci- 95. What was per employee expen-
plines together in that year ?
(A) 164 : 217 (B) 223 : 260 diture on medical ?
(rounded off to two digits after
(C) 260 : 223 (D) 217 : 164 (A) Rs. 12000
decimal)
87. What is the percentage of number (B) Rs. 13000
(A) 46·24 (B) 23·51
of cars sold by company D in the (C) Rs. 12500
(C) 37·14 (D) 40·15
year 2002 to those manufactured (D) Rs. 13500
by it in that year ? (rounded off 92. What is the respective ratio of
96. What was the total expenditure
to two digits after decimal) total number of students study-
on salary of employees in
ing Arts, Commerce and Science
(A) 87·45 (B) 77·28 marketing department ?
in all the years together ?
(C) 92·54 (D) 79·65 (A) Rs. 6·12 lakh
(A) 77 : 75 : 76
88. In which year were the maximum (B) Rs. 61·2 million
(B) 76 : 75 : 77
number of cars manufactured by (C) Rs. 6·12 million
(C) 76 : 77 : 75
all companies together ? (D) Rs. 176 lakh
(D) 75 : 77 : 76
(A) 2001 97. What was the amount spent on
(B) 2002 93. What is the respective ratio of electricity ?
(C) 2003 total number of students study- (A) Rs. 132 million
(D) None of these ing Commerce in the years 2000
(B) Rs. 13·2 lakh
and 2002 together to those
Directions : (Q. 89–93) Study (C) Rs. 126 million
studying Arts in the years 2003
the following graph carefully to (D) None of these
and 2005 together ?
answer the questions that follow—
(A) 3 : 4 (B) 7 : 9 98. What was the expenditure on
Number of students studying telephone for employees in
different disciplines in a (C) 4 : 3 (D) 9 : 7
computer department ?
college over the year Directions : (Q. 94–98) Study (A) Rs. 11·52 lakh
Commerce Science Arts the following graph carefully to ans- (B) Rs. 11·52 million
800
wer these questions— (C) Rs. 10·72 lakh
Details about the distribution of (D) Rs. 10·72 million
700 employees and expenditure of an Directions : (Q. 99–105) Study
organization.
600 the following graph and table care-
(Distributed proportionately across
No. of Students

fully and answer the questions given


500 the departments) below it.
400 Annual expenditure on different Distribution of candidates appeared
300
items in a Competitive Examination
Total expenditure = Rs. 12 crores from seven states
200 Allowance
Maintenance 7%
100 Telephone 19% G 15%
7% F A
12% 22% 18%
0 12% B
2000 2001 2002 2003 2004 13% Medical E C
Electricity 11% 23% 6%
Years 5% D
30%
89. Number of students studying Transport Total candidates appeared = 3
Commerce in the year 2004 Salary lakh

Quantitative Aptitude Test | 322


Statewise percentage and ratio of 100. What is the total number of 103. What is the total number of
male and female qualified female candidates qualified from candidates qualified from States
candidates States ‘A’ and ‘B’ together ? ‘E’ and ‘D’ together ?
% qualified Ratio of (A) 26526 (B) 25426 (A) 54450 (B) 45540
State over appeared qualified (C) 54540 (D) 54410
(C) 26426 (D) 24526
from State candidates
104. How many male candidates have
Male : Female 101. Which of the following pair of appeared from State ‘D’ ?
A 49 4:5 States have equal number of (A) 41400
B 61 6:4 qualified male candidates ?
(B) 27600
C 54 7:8 (A) B and F (B) C and G
D 45 3:2 (C) 32400
E 65 7:6 (C) C and E (D) A and E (D) Cannot be determined
F 57 11 : 8
102. What is the percentage of candi- 105. How many female candidates
G 48 9 : 11 have qualified from State ‘F’ ?
dates qualified from States ‘A’
99. What is the number of male and ‘C’ together, rounded off to (A) 32490
candidates qualified from State two decimal points ? (B) 18810
‘G’ ?
(A) 4568 (B) 5454 (A) 49·93 (B) 51·26 (C) 13810
(C) 5544 (D) 4536 (C) 50·43 (D) 50·93 (D) None of these

Answers with Hints


01. (D) Let the profit earned by the company in the year 05. (D) From (1) No. of candidates who get marks 80%
2002 be Rs. x crores and above
∴ From 2 Profit earned in 2001 = 20% of 240
= 48
⇒ Rs. (20 – x) crores
From 3 if the no. of candidates who secured marks
And from 1 Profit earned in 2003. between 60% and 79% be x.
140 Then no. of candidates who secured marks below
⇒ Rs. (20 – x) × crores
100 50% = x.
Q From 3 From 2 no. of candidates who secured marks more
than 50% but less than 60%
140 80x
(20 – x) × = = 80
100 100
Since the number of candidates who secured marks
⇒ 280 – 14x = 8x between 60% and 79% is not known
140 Hence, answer cannot be obtained from any of these.
x =
11 06. (D) From 1 4a2 – 20a + 21 = 0
∴ x = Rs. 12·727 crores ⇒ (2a – 3) (2a – 7) = 0
02. (B) Let the monthly salary of an assistant of the 3 7
⇒ a = or
company be Rs. x 2 2
From 2, 2
2b – 5b + 3 = 0
∴ From 1, monthly salary of a supervisor
⇒ (b – 1) (2b – 3) = 0
= Rs. (x + 12000)
3
And From 2, ⇒ b = 1 or
2
Q x + x + 12000 = 32000 a ≥ b
∴ x = Rs. 10000 07. (B) From 1, 6a2 – 25a + 25 = 0
03. (D) Area of the parallelogram cannot be found. ⇒ (2a – 5) (3a – 5) = 0
Hence answer of the question cannot be found even 5 5
from 1, 2 and 3 together ⇒ a = or
2 3
04. (A) From 1 and 3 the profit earned by B From 2, 15b2 – 16b + 4 = 0
5 ⇒ (3b – 2) (5b – 2) = 0
= × 60000
3 2 2
⇒ b = or
= Rs. 1 lakh 3 5
Hence, only 1 and 3 are sufficient. ∴ a > b

Quantitative Aptitude Test | 323


08. (C) From 1, a2 = 4 17. (D) Reqd. percentage

From 2,
a = ±2
b2 = 9
= (111300 + 115650 )
11295 + 12170
× 100

23465 × 100
⇒ b = ±3 = = 10·34
226950
∴ The relationship between a and b cannot be
1800 × 100
established 18. (A) % of A = = 11·8
15250
09. (A) From 1, 2a2 + 3a + 1 = 0 1100 × 100
% of B = = 8·8
⇒ (2a + 1) (a + 1) = 0 12500
1 1550 × 100
⇒ a = – or – 1 % of C = = 11·07
2 14000
From 2, 12b2 + 7b + 1 = 0 1640 × 100
% of D = = 9·31
17600
⇒ (4b + 1) (3b + 1) = 0
1480 × 100
1 1 % of E = = 9·02
⇒ b = – or – 16400
4 3 1250 × 100
∴ a < b % of F = = 8·62
14500
10. (D) From 1, a2 + 5a + 6 = 0 1400 × 100
and % of G = = 9·33
15000
⇒ (a + 2) (a + 3) = 0
∴ Least percentage is of F.
⇒ a = – 2 or – 3
2 19. (A) Reqd. percentage
From 2, b + 3b + 2 = 0
⇒ (b + 1) (b + 2) = 0 = (15500 + 14800 )
1480 + 1575
× 100
⇒ b = – 1 or – 2 3055 × 100
= = 10 (App.)
∴ a ≤ b 30300
11. (D) 11295
20. (C) Average number = = 1615
108 7
2 3 6 18 109 1944 209952 21. (D) Average number
2 × 3 = 6, 3 × 6 = 18, 6 × 18 = 108, 18 × 108 = 1944, 108 × 1944 = 209952
12500 + 15400 + 13800 + 14000 + 14550
=
12. (B) 5
37 70250
= = 14050
1 3 6 11 20 39 70 5
22. (D) Q V = 38808 ml
×2+1 ×2+0 ×2_1 ×2–2 ×2_3 ×2_4
= 38808 c.c. …(1)
13. (A) 3
11 Q r = h
4
2 13 27 113 561 3369 23581 4r
⇒ h = …(2)
×2+7 ×3_6 ×4+5 ×5_4 ×6+3 ×7_2
3
Volume of cylindrical tank (V)
14. (D) = πr2 h
+5 +9
22 4r
40 388808 = × r2 ×
50 51 47 56 42 65 29 7 3
38808 × 7 ×3
_3 _7 ⇒ r3 =
_ 11
22 × 4
15. (C) ⇒ r3 = 9261
97 ∴ r = 21
3 9 23 99 479 2881 20159 Diameter of base = 2r
×2+3 ×3_4 ×4+5 ×5_6 ×6+7 ×7_8 = 2 × 21
= 42 cm.
16. (A)
2 4 5 8
23. (A) A : B : C = 8 : 14 : 22
13 21 24
2 + 3 = 5, 3 + 5 = 8, 5 + 8 = 13, 8 + 13 = 21, 13 + 21 = 34 = 12 : 21 : 33

Quantitative Aptitude Test | 324


B : C : D = 21 : 33 : 44 36. (A) 37. (A)
∴ A : B : C : D = 12 : 21 : 33 : 44 38. (A) Not in any sector
24. (B) ? = 12 × 958 ÷ 17 24880 × 100
39. (D) Required percentage = %
958 975000
= 12 × = 676·23
17 = 2·5%
~
– 676 40. (C) Total expenditure on education
25. (A) 26. (D) 27. (D) 28. (C) = Rs. 556610 million
29. (A)
Total expenditure on health
8
1 ? = Rs. 34570 million
27 64 125
∴ Required difference = Rs. (56610 – 34570)
(1)3 (2)3 (3)3 (4)3 (5)3 = Rs. 22040 million
30. (D) = Rs. 22040000000
9 4875 × 125
25 ? 4 41. (D) ? = + 88 × 15
16 1 100
(5)2 (4)2 (3)2 (2)2 (1)2 = 6093 + 1320 = 7415
31. (A) 1010
42. (B) ? = + 187 × 20
19660 36
1 6 36 240 1960 ? = 28 + 3740 = 3770
43. (D) ? = 127 × 8 + 6 × 4
× 2 + (2 × 2) × 4 + (4 × 3) × 6 + (6 × 4) × 8 + (8 × 5) × 10 + (10 × 6)
= 1016 + 24 = 1040
32. (A)
32·5 × 100
44. (A) Required percentage = = 650
5
45. (A) Approximate average production
5 + 7·5 + 10 + 17·5 + 25 + 27·5 + 32·5
=
7
125
= = 17·86 = 18 lakh
7
46. (D) Q (17·5 – 10) = (25 – 17·5) = 7·5

47. (B) Per cent increase in production over the previous


year in 1997
7·5 – 5·0
= × 100 = 50%
5·0
Per cent increase in production over the previous year
in 1998
10 – 7·5
= × 100 = 33·33%
7·5
Per cent increase in production over the previous
year in 1999
33. (B) 17·5 – 10
= × 100 = 75%
34. (D) 10
_2 _2 _2 Per cent increase in production over the previous
year in 2000
15 12 17 10 ? 8 25 6 25 – 17·5
= × 100 = 42·86%
17·5
+2 +3 +5 Per cent increase in production over the previous year
35. (C) in 2001
1260 27·5 – 25
= × 100 = 10%
4 6 12 30 90 315 ? 25
Per cent increase in production over the previous
× 1.5 ×2 × 2.5 ×3 × 3.5 ×4 year in 2002

Quantitative Aptitude Test | 325


32·5 – 27·5 53. (B) As per the given condition a committee out of 8
= × 100 = 18·18% men and 6 women can be formed in the following
27·5
manner :
∴ Highest per cent increase in production over the
previous year is in 1999 (1) With 3 men and 3 women the selection may be
done in 8C 3 × 6 C 3 ways
48. (E) 2x2 – 7x + 6 = 0 (2) With 4 men and 2 women, the selection may be
⇒ (x – 2) (2x – 3) = 0 made in 8C 4 × 6 C 2 ways
3 (3) With 5 men and 1 women, the solution may be
⇒ x = 2 or
2 made in 8C 5 × 6 C 1 ways
and, 4y2 = 9 → y (4) With 6 men and 0 woman
3 Therefore, the required number
= ± = (8C 3 × 6 C 3 ) + (8C 4 × 6 C 2 )
2
∴ x ≥ y + (8C 5 × 6 C 1 ) + 8 C 6 6 C 0

49. (E) 4x2 – 4x – 3 = 0 = (


8·7·6 6·5·4
×
1·2·3 1·2·3 ) (
+
8·7·6·5 6·5
×
1·2·3·4 1·2 )
⇒ (2x – 3) (2x + 1) = 0


3 –1
x = or
+ (8·7·6·5·4 6
×
1·2·3·4·5 1
+ ) (8·7·6·5·4·3
1·2·3·4·5·6 )
2 2 = (56 × 20) + (70 × 15) + (56 × 6) + 28 × 1
and 4y2 + 12y + 5 = 0 = 1120 + 1050 + 336 + 28 = 2534
⇒ (2y + 5) (2y + 1) = 0 54. (D) From I and II or III
–5 –1 Work of 1 man = work of 2 women
⇒ y = or
2 2 ∴ 16 men + 8 women = 32 + 8 = 40 women
∴ x ≥ y Now from II or III 40 women will complete 1 work
⇒ (2y + 5) (2y + 1) = 0 in 4 days
–5 –1 55. (D) From 1 and 2
⇒ y = or
2 2
240
∴ x ≥ y Speed of the train = = 24 m/sec
10
50. (A) 4x2 = 49 From 2 and 3
7 240 + 240
⇒ ± Speed of the train = = 24 m/sec
2 20
and 9y2 – 66y + 121 = 0
56. (D) From 1 area of the square
⇒ (3y – 11)2 = 0
1
11 (Diagonal)2
=
y = 2
3
From 2 area of the square = (side)2
∴ x < y
51. (B) x2 + 9x + 14 = 0 From 3 Area of the square = (
Perimeter 2
4 )
⇒ (x + 2) (x + 7) = 0
57. (C) Let the two digit number be 10x + y
⇒ x = – 2 or – 7
From 1 or from 3
and y2 + y – 2 = 0
(10y + x) – (10x + y) = 18
⇒ (y – 2) (y – 1) = 0
⇒ y–x = 2 …(1)
y = – 2 or 1
From (2) x + y = 14 …(2)
∴ x ≤ y
∴ x = 6
52. (A) 9x2 – 18x + 5 = 0
and y = 8
⇒ (3x – 5) (3x – 1) = 0
∴ Number = 68
5 1
x = or
3 3 58. (B) Amount doubles itself in 5 years with S.I.
and 2y2 – 9y + 10 = 0 ∴ If P = Rs. 100
⇒ (2y – 5) (y – 2) = 0 then S.I. = Rs. 100
5 100 × 100
⇒ y = or 2 Rate = = 20% p.a.
2 100 × 5
∴ x < y 59. (C) 60. (D) 61. (B) 62. (A)

Quantitative Aptitude Test | 326


63. (D) Q ? % of 280 + 18% of 550 In 2003 the value of sugar per metric ton
= 143·8 235
= = Rs. 1·566 lakh
? × 280 550 × 18 2 × 75
⇒ + = 143·8 And in 2004 the value of sugar per metric ton
100 100
⇒ 2·8 × ? + 99 = 143·8 110
= = Rs. 1·571 lakh
143·8 – 99 70
? = ∴ It is the highest in the year 1998
2·8
44·8 67. (D) It is lowest in the year 2003
= = 16
2·8 68. (A) 25% of the total production
64. (D) Average production of sugar in years 1998, 1999, 25
2000 and 2001 = × 340
100
15 + 50 + 30 + 35 = 85 thousand metric tons
=
4 And the production of the year 1998 and 2004
= 32·5 thousands metric tons = 15 + 70
And average production of sugar in years 2001,
= 85 thousand metric tons
2002, 2003 and 2004
35 + 65 + 75 + 70 69. (D) Reqd. cost of average sales
=
4 (25 + 45 + 38 + 52 + 47 + 55)
= × 1000 × 1·8
= 61·25 thousands metric tons 6
∴ Required difference = Rs. 78600 lakh
= 61·25 – 32·5 70. (C) Total sale in 2003
= 28·75 thousands metric tons = 43 + 55 + 49 + 63 + 42 + 62
65. (A) In 1999 the per cent increase in value from the = 314 lakh tonnes
previous year And total sale in 2000
50 – 15
= × 100% = 41 + 50 + 43 + 57 + 39 + 64
15
= 294 lakh tonnes
= 233·33%
In 2001 the per cent increase in value from the 314 × 100
∴ Required percentage = %
previous year 294
35 – 30 = 106·80%
= × 100%
30 71. (B) Percentage increase in sales in 2003 from 1998
= 16·66% 314 – 262
In 2003 the per cent increase in value from the = × 100%
262
previous year
= 19·847%
75 – 65
= × 100% = 20%
65
= 15·38% 45 + 39 + 50 + 48 + 53 + 55
72. (D) Average of B =
∴ In the remaining there was decrease 6
66. (D) In 1998 the value of sugar per metric ton = 48·3 lakh tonnes
55 and average sale of C
= = Rs. 1·833 lakh
2 × 15 38 + 40 + 43 + 43 + 46 + 49
=
In 1999 the value of sugar per metric ton 6
80 = 43·16 lakh tonnes
= = Rs. 1·600 lakh
50 ∴ Reqd. ratio = 48·3 : 43·16
In 2000 the value of sugar per metric ton = 29 : 26
50 73. (C) % of decrease in sales in 1999 of E from
= = Rs. 1·666 lakh
30 previous year
In 2001 the value of sugar per metric ton 47 – 46
= × 100 = 2·13%
115 47
= = Rs. 1·642 lakh
2 × 35 % of decrease in sales in 2000 of E from previous
In 2002 the value of sugar per metric ton year
102·5 46 – 39
= = Rs. 1·575 lakh = × 100 = 15·22%
65 46
Quantitative Aptitude Test | 327
% of decrease in sales in 2001 of E from previous % of loss of E in 2000
year 45 – 40
= × 100 = 11·11
39 – 32 45
= × 100 = 17·95%
39 % of profit of F in 2000
% of increase in sales in 2002 of E from previous 32·5 – 25
year = × 100 = 30·00
25
37 – 32 % of profit of G in 2000
= × 100 = 15·63%
32 50 – 45
% of decrease in sales in 2003 of E from previous = × 100 = 11·11
45
year % Highest percentage of profit in 2000 is of the
42 – 37 company A
= × 100 = 13·51%
37
78. (C) Total income of all companies
∴ Max. % decrease is in the year 2001
= 40 + 42·5 + 35 + 50 + 40 + 32·5 + 50
74. (D) Expenditure of companies C and D = Rs. 290 crore
= 40 + 45 And total expenditure of all companies
= Rs. 85 crore = 30 + 32·5 + 45 + 40 + 45 + 25 + 45
and income of companies C and D = Rs. 262·5 crore
= 35 + 50 ∴ Required % profit
= Rs. 85 crore 290 – 262·5
∴ There was no loss or profit = × 100 = 10·5
262·5
75. (C) Income of company A in 1999 79. (D) There are 6 different letters in the word ADJUST
100 × 40 with two vowels (A, U) and four consonants (D, J, S,
= T). The vowels (A, U) put together may be deemed
110
as a single letter. In this condition there are in all (1 +
400
= Rs. crore 4) i.e; 5 letters. The numbers comprising these 5
11
letters are 5 but the vowels (A, U) can be arranged
If the expenditure of company A in 1999 be Rs. x
400 in 2 ways. So putting both the vowels together
–x
11 Number of words = 5 × 2
x × 100
then, 20 =
Number of words when the two vowels do not occur
⇒ x = Rs. 30·30 crore together
76. (B) Expenditure of company G in 1999 = 6 – 5 × 2
100 × 45
= = Rs. 37·5 crore = 720 – 240
100 + 20
= 480
If the income of G in 1999 be Rs. x 80. (C) 3x2 – 12x + 12 < x
x – 37·50 3x2 – 13x + 12 < 0
then 10 = × 100
37·50 (x – 3) (3x – 4) < 0
∴ x = Rs. 41·25 crore
77. (D) % of profit of A in 2000 ( )
(x – 3) x –
4
3
< 0

+ – +
= ( 30 )
40 – 30
× 100

= 33·33 4
–∞ 3+∞
3
% of profit of B in 2000
4
42·50 – 32·50 ≤x≤3
= × 100 3
32·50
= 30·77 81. (D) If the period is not known, we cannot ascertain
the amount invested under scheme A.
% of loss of C in 2000
82. (D)
45 – 35
= × 100 = 22·22 x
45 83. (A) Let the original fraction be y
% of profit of D in 2000
50 – 40 1·4x 7
= × 100 = 25 By question, =
40 2y 16

Quantitative Aptitude Test | 328


⇒ 1·4x × 16 = 2y × 7 92. (D) Reqd. ratio = 550 + 500 + 600 + 650
x 2×7 + 700 + 750) : (450 + 650 + 600
⇒ y = 1·4 × 16 + 700 + 750 + 700) : (500 + 600
x 5
∴ y = 8 + 550 + 750 + 750 + 650)
= 3750 : 3850 : 3800
84. (C) Reqd. number
= 75 : 77 : 76
= 1·53 + 2·03 + 1·87 + 1·95 + 2·32 + 2·36
= 12·06 thousand = 12060 93. (A) Reqd. ratio = (500 + 550) : (650 + 750)
85. (C) Reqd. per cent increase = 1050 : 1400
2·10 – 2·08 = 3:4
= × 100
2·08
94. (D) Total expenditure on accounts deptt.
0·02 × 100
=
2·08 120000000 × 168
=
2 1200
= = 0·96%
2·08 14
= 120000000 ×
2·34 + 2·15 + 2·20 100
86. (B) Reqd. ratio =
2·62 + 2·79 + 2·39 = Rs. 16·8 million
6·69
= = 223 : 260 95. (B) Per employee expenditure on medical
7·80
2·23 13 1
87. (A) Reqd. per cent = × 100 = 87·45 = 120000000 × ×
2·55 100 1200
= 13000
88. (D) In 2000, no. of cars manufactured by all compa-
nies 96. (C) Total expenditure on salary of employees in
= 2·58 + 1·98 + 1·97 + 2·46 + 2·35 + 1·88 marketing deptt.
= 13·22 thousand 30 17
= × 120000000 ×
In 2001, no. of cars manufactured by all companies 100 100
= 2·34 + 2·15 + 2·20 + 2·46 + 2·45 + 1·95 = Rs. 6·12 millions
= 13·55 thousand
97. (D) Amount spent on electricity
In 2002, no. of cars manufactured by all companies
11
= 2·85 + 2·35 + 2·18 + 2·55 + 2·60 + 2·25 = 120000000 ×
100
= 14·78 thousand
= Rs. 13200000
In 2003, no. of cars manufactured by all companies
= 2·87 + 2·62 + 2·25 + 2·62 + 2·79 + 2·39 98. (A) Expenditure on telephone for employees in
= 15·54 thousand computer deptt.
In 2004, no. of cars manufactured by all companies 12 8
= 120000000 × ×
= 2·91 + 2·71 + 2·68 + 2·71 + 2·88 + 2·58 100 100
= 16·47 thousand = Rs. 11·52 lakh
and in 2005, no. of cars manufactured by all compa- 99. (D) Total candidates = 300000
nies
7
= 2·94 + 2·84 + 2·86 + 2·76 + 2·90 + 2·67 Candidates joining in state G = 300000 ×
100
= 16·97 thousand
= 2100
750 × 100
89. (B) Reqd. per cent = No. of candidates passed in state G
3800
= 19·74% 48
= 210 ×
100
= 20%
= 10080
90. (D) Reqd. no. = 3750
No. of male candidates passed in State G
650 × 100
91. (C) Reqd. percentage = 9
(600 + 650 + 500) = × 10080
65000 9 + 11
= = 37·14%
1750 = 4536

Quantitative Aptitude Test | 329


100. (B) No. of candidates passed in State A No. of male candidates passed in State G
15 49 = 4536
= 300000 × ×
100 100 Obviously, the no. of male candidates passed in State
= 22050 C and G in the same
No. of female candidates passed in State A 102. (C) No. of candidates passed in State A
5 = 22050
= × 22050
9 No. of candidates passed in State C
= 12250 = 9720
Similarly, No. of candidates passed in State A and C
No. of total candidates passed in State B = 22050 + 9720
18 61
= 300000 × × = 31770
100 100
No. of candidates joining in State A
= 32940
15
No. of female candidates passed in State B = 300000 ×
100
4
= × 32940 = 45000
10
No. of candidates joining in State C
= 13176
6
No. of total female candidates passed = 300000 ×
100
= 12250 + 13176 = 18000
= 25426 No. of total candidates joining in State A and C
101. (B) No. of candidates passed in State A = 45000 + 18000
15 49 = 63000
= 300000 × ×
100 100 31770
= 22050 Reqd. percentage = × 100
63000
No. of male candidates passed in State A = 50·43
4
= × 22050 103. (A) No. of total candidates passed in State E and D
9
= 9800 = (300000 × 100
12 65
×
100)
No. of candidates passed in State B
+ (300000 ×
100 100)
23 45
18 61 ×
= 300000 × ×
100 100
= 32940 = 23400 + 31050
No. of male candidates passed in State B = 54450
6
= × 32940 104. (D) Male-female ratio of candidates failing in State
10 D has not been given. Therefore the male candidates
= 19764 who joined cannot be ascertained.
No. of candidates passed in State C 105. (D) Number of candidates passed in State F
6 54
= 300000 × × 19 57
100 100 = 300000 × ×
100 100
= 9720 = 32490
No. of male candidates passed in State C Number of female candidates passed in State F
7 8
= × 9720 = × 32490
15 19
= 4536 = 13680

Quantitative Aptitude Test | 330


Miscellaneous Exercise – IV
01. The highest common factor of 09. Which is the biggest of the (C) Rs. 300
70 and 245 is— following fractions ? (D) Data inadequate
(A) 35 (B) 55 3 4
(A) (B) 17. 18 persons can finish a work in
(C) 45 (D) 65 4 5 36 days. How much time will 12
1 5 6 persons take to finish the same
02. The average of the fractions 1 , (C) (D)
2 6 7 work ?
1 1 5
2 , 3 , 4 is—
3 3 6
1
10.
1
7
+ [ (39 + 29) – 29] is equal
7
9

(A) 18 days (B) 24 days
(C) 54 days (D) 72 days
(A) 2 (B) 2 to—
2 18. A and B finish a job in 12 days.
1 1 While A, B and C can finish it in
(C) 3 (D) 4 (A) (B)
7 9 8 days. C alone will finish the
03. ( ) ( )
?
31
×
?
279
=1 (C)
2
9
(D)
3
7
job in—
(A) 14 days (B) 16 days
(A) 31 (B) 93
3 3 2 (C) 20 days (D) 24 days
(C) 217 (D) 8649 11. 5·75 – × 15 + 2 ÷ 1·44 = ?
7 4 35
04. A number exceeds its four 19. The smallest number which when
2 3 subtracted from sum of squares
seventh by 18. What is the (A) (B)
number ? 5 7 of 11 and 13 gives a perfect
(A) 36 4 2 square, is—
(C) (D)
(B) 49 11 9 (A) 1 (B) 4
(C) 63 1 3 4 (C) 5 (D) 9
(D) None of these 12. 12 ×3 ÷1 =?
2 5 5 20. The sum of any seven consecu-
1 1 1 (A) 45 tive whole numbers is always
05. The ratio : : is the same
2 3 5 (B) 81 divisible by—
as— (A) 2 (B) 3
(C) 405
(A) 2 : 3 : 5
(D) None of these (C) 7 (D) 11
(B) 5 : 3 : 2
(C) 15 : 10 : 6 13. ?% of 250 + 25% of 68 = 67 21. In an examination 35% of the
(D) 6 : 10 : 15 total student failed in Hindi. 45%
(A) 10 (B) 15 failed in English and 20% in
3 (C) 20 (D) 25 both. Percentage of total students
⎯√⎯⎯ 91
06. 1– is equal to— passed in both the subjects is—
216
(A)
1
6
14.
⎯√⎯289? = 5451 (A) 10
(C) 30
(B) 20
(D) 40
(A) 108 (B) 324
5
(B) (C) 2916 (D) 6800 22. The least square number exactly
6 divisible by 8, 12, 15 and 20 is—
3 15. In a garden there are 10 rows (A) 900 (B) 1200
⎯⎯91
√ and 12 columns of mango trees.
(C) 1 – (C) 3600 (D) 14400
6 The distance between each tree
(D) None of these is of 2 metres and a distance of 23. If the price of some commodity
one metre is left from all sides of is reduced from Rs. 16 to Rs.
07. The value of (1502)2 – (1498)2 the boundary of the garden. The 12·25 then on purchasing four
is— length of the garden is— such commodities how much
(A) 12000 (B) 16000
(A) 20 metres (B) 22 metres percentage can one save ?
(C) 2256004 (D) 22560
(C) 24 metres (D) 26 metres (A) 15 (B) 49
1
08. 1 + =? 7 4900
1 16. An increase of Rs. 60 in the (C) 23 (D)
1+ monthly salary of Madan made it 16 64
1
1– 50% of the monthly salary of
6 24. The smallest number, which must
6 16 Kamal. What is Madan’s present be added to 1000 to make it a
(A) (B) monthly salary ? perfect square is—
11 11
7 1 (A) Rs. 180 (A) 12 (B) 20
(C) (D)
6 6 (B) Rs. 240 (C) 24 (D) 25

Quantitative Aptitude Test | 331


25. Two consecutive multiples of a
certain number add upto 184.
(C) 9
(D) None of these
(C) Rs. (10000 –P )
10000
2
The number is—
33. If 5 poles are erected at equal ⎯⎯⎯⎯
√ 1 – P2
(A) 4 (B) 8 (D) Rs.
distances between two points 20
(C) 23 (D) 46 100
metres apart. What is the distance
26. The positions of the digits of a between any two poles ? 40. A boy was asked to multiply a
two digit number are interchan- (A) 2 metres (B) 3 metres certain number by 25. He
ged. If the sum of the original multiplied it by 52 and got his
(C) 4 metres (D) 5 metres
number and the number obtained answer more than the correct one
by interchanging the digits is 44. 34. Which of the following has the by 324. The number to be multi-
What is the sum of digits of that fraction in the ascending order ? plied was—
number ? 2 3 4 4 3 2 (A) 12 (B) 15
(A) , , (B) , ,
(A) 2 7 10 13 13 10 7 (C) 25 (D) 52
(B) 4 3 2 4 4 2 3
(C) , , (D) , , 41. The difference between the
(C) 11 10 7 13 13 7 10
simple interest and the compound
(D) Cannot be determined 35. Dhawan bought 10 chairs for Rs. interest at the same rate of
500. He got them repaired and interest on a sum of money at the
27. The least number, which 715 sold them at Rs. 500 per pair. He end of the second year will be
must be multiplied with in order got a profit of Rs. 100 per chair. Rs. 4·00. If the rate of interest is
to get a multiple of 825 is— How much did he spend on the 5% per annum, what is the sum ?
(A) 10 (B) 12 repair of the chairs ? (A) Rs. 1200
(C) 15 (D) 35 (A) Rs. 250 (B) Rs. 500
(B) Rs. 1600
28. The difference of two numbers is (C) Rs. 1000 (D) Rs. 1500
1 (C) Rs. 2000
11 and th of their sum is 9. The 36. Deepak has Rs. 5130 in the form
5 (D) None of these
of 1, 2 and 5 rupee notes. If these
number are— notes be in the ratio 3 : 7 : 8, the 42. If the price of one dozen of
(A) 31, 20 (B) 30, 19 number of five rupee notes he mangoes is Rs. 49·75, what will
(C) 29, 18 (D) 28, 17 has is— be the approximate value of 291
(A) 340 (B) 672 mangoes ?
29. Kavita has one quarter more
money than Nitin. Nitin has two- (C) 720 (D) 768 (A) Rs. 1000 (B) Rs. 1200
third money as of Pravin. If 37. At the election involving two (C) Rs. 1500 (D) Rs. 1800
Pravin has Rs. 876 with him. candidates only 68 votes are 43. The square root of 824464 is—
How much money Kavita has ? declared as invalid. The winning
(A) 686
(A) Rs. 365 (B) Rs. 467·20 candidates scores 52% and wins
by 98 votes. The total number of (B) 868
(C) Rs. 730 (D) Rs. 760
votes polled is— (C) 908
1 (A) 2382
30. th of Nikhil’s money is equal to (D) None of these
4 (B) 2450
1 44. A trader lists his articles 20%
th of Yogesh’s money. If both (C) 2518
6 above C.P. and allows a discount
together have Rs. 600, the diffe- (D) None of these of 10% on cash payment, this
rence between their amounts is— 38. Two different natural numbers gain per cent is—
(A) Rs. 50 (B) Rs. 120 are such that their product is less (A) 5% (B) 6%
(C) Rs. 240 (D) Rs. 360 than their sum. One of the num- (C) 8% (D) 10%
bers must be—
31. A number when divided by 123 (A) 1 45. 10% of 24·2 will be how much
leaves remainder 83. If the same more than 10% of 24·02 ?
(B) 2
number is divided by 41, the (A) 0·02 (B) 0·18
remainder will be— (C) 3
(D) None of these (C) 0·018 (D) 0·002
(A) 0 (B) 1
39. The price of an article was 46. For the theatre the cost of a child
(C) 40 (D) 83
increased by P%. Later the new 1
ticket is of the cost of an adult
32. One fourth of a two digit number price was decreased by P%. If 3
is two less than one-third of the the latest price was Re. 1, the ticket. If the cost of tickets for 3
same number. What is the sum original price was— adults and 3 children is Rs. 60,
of the digits of the number ? (A) Re. l the cost of an adult ticket is—
(A) 6 1 – P2 (A) Rs. 12 (B) Rs. 15
(B) Rs.
(B) 8 100 (C) Rs. 18 (D) Rs. 20

Quantitative Aptitude Test | 332


47. The sum of two numbers is 104 55. Monika deposits Rs. 8000 partly 61. The charges of hired car are Rs.
and their difference is 30. The at 10% and partly at 15% interest 4 per km for the first 60 km, Rs.
difference of their squares is— for one year in a bank. If she 5 km for the next 60 km and Rs.
(A) 74 (B) 2160 gets an interest of Rs. 950 at the 8 for every 5 km for the further
(C) 2320 (D) 3120 end of one year the amount journey. If the balance amount
deposited at 15% is— 1
48. B is twice as fast as A and C is left over with Ajit is less that
(A) Rs. 2000 less than deposited 4
three times as fast as A. If B at 10% what he paid towards the charges
alone can complete a job in 12 of the hired car for travelling 320
(B) Rs. 1250 less than deposited
days, how long will A, B and C km. How much money did he
at 10%
take to complete the same job have initially with him ?
together ? (C) Rs. 500 less than deposited
at 10% (A) Rs. 1032
(A) 3 days
(D) Rs. 1500 less than deposited (B) Rs. 1253
(B) 4 days
at 10% (C) Rs. 1548
(C) 6 days
56. 12 buckets of water fill a tank (D) None of these
(D) None of these
when the capacity of each bucket 62. A 63 cm long wire is to be cut
49. The average of marks obtained is 13·5 litres. How many buckets into two pieces such that one
by Aakash in seven subjects is will be needed to fill the same 2
68. His average in six subjects tank if the capacity of each piece will be as long as the
5
excluding Mathematics is 70. bucket is 9 litres ? other. How many centimetres
How many marks did he get in (A) 8 will the shorter piece be ?
Mathematics ?
(B) 16 (A) 9 (B) 18
(A) 56 (B) 60
(C) 18 (C) 36 (D) 45
(C) 68 (D) 82
(D) None of these 63. A bus goes from A to B at the
50. The L.C.M. of two numbers is rate of 30 kmph and from B to A
2310 and their H.C.F. is 30. If 57. Rs. 1200 amounts to Rs. 1632 in
four years at a certain rate of at the rate of 60 kmph. The
one number is 210, the other average speed of the bus is—
number is— simple interest. If the rate of
interest is increased by 1%, it (A) 40 kmph
(A) 330 (B) 1470 would amount to how much ? (B) 45 kmph
(C) 2100 (D) 16170
(A) Rs. 1635 (B) Rs. 1644 (C) 47·5 kmph
51. A man donated 5% of his income (C) Rs. 1670 (D) Rs. 1680 (D) 52·5 kmph
to a charitable organisation and
deposited 20% of the remainder 58. Jayant started a business 64. A class starts at 10 A.M. and
in a bank. If he now has Rs. 1919 investing Rs. 6000. Six months lasts till 1·27 p.m. Four periods
left, his income is— later Madhu joined him investing are held during this interval.
Rs. 4000. If they make a profit After every period, 5 minutes are
(A) Rs. 2300 (B) Rs. 2500
of Rs. 5200 at the end of the year, given free to the students. The
(C) Rs. 2525 (D) Rs. 2558·60 exact duration of each period
how much should be the share of
52. A sum of Rs. 45 is made up of Madhu ? is—
100 coins of 50 paise and spaise. (A) Rs. 1300 (B) Rs. 1732 (A) 42 minutes
How many of them are 55 paise (B) 48 minutes
coins ? (C) Rs. 3466 (D) Rs. 3900
(C) 51 minutes
(A) 40 (B) 50 59. The compound interest on Rs.
(D) 53 minutes
(C) 75 (D) 80 1
6000 for 1 years at 10% per
2 65. The average of first nine multi-
53. If the area of an equilateral trian- annum, the interest being paid ples of 3 is—
gle is 36⎯√ 3 cm2 the perimeter of half yearly, will be— (A) 12·0 (B) 12·5
the triangle is— (A) Rs. 912·75 (C) 15·0 (D) 18·5
(A) 18 cm (B) 24 cm (B) Rs. 930
(C) 30 cm (D) 36 cm 66. The mean proportional of 0·32
(C) Rs. 932·50 and 0·02 is—
54. A fruit vender has 24 kg of (D) Rs. 945·75
apples. He sells a part of these at (A) 0·08 (B) 0·16
20% gain and the balance at a 60. What number must be added to (C) 0·30 (D) 0·34
loss of 5%. If on the whole he the numbers 3, 7 and 13 so that
67. One third of Vinod’s mark in
earns a profit of 10% the amount they are in a continued propor-
Mathematics exceeds a half of
of apples sold at a loss is— tion ?
his marks in social studies by 30.
(A) 6 kg (B) 4·6 kg (A) 5 (B) 6 If he got 240 marks in the two
(C) 9·6 kg (D) 11·4 kg (C) 8 (D) 9 subjects together how many

Quantitative Aptitude Test | 333


marks did he get in social stu- 74. A person gave Rs. 2500 to his 80. A train covers four successive
dies ? 5 two km streches at speeds of 10
eldest son, of the whole
(A) 40 (B) 60 12 kmph, 20 kmph, 30 kmph and 60
(C) 80 (D) 90 property to the second son and to kmph respectively. Its average
the youngest as much as to the speed over this distance—
68. There are 20 students with an
average height of 125 cm in a first and the second son together. (A) 20 kmph (B) 24 kmph
class. 5 students with an average How much did the youngest son (C) 30 kmph (D) 32 kmph
height of 116 cm leave the class. get ?
81. A sum of money deposited at
What is average height of the (A) Rs. 10000
compound interest amounts to
class now ? (B) Rs. 15000 Rs. 6690 after 3 years and Rs.
(A) 118 cm (B) 120 cm (C) Rs. 20000 10,035 after 6 years. The sum
(C) 128 cm (D) 130 cm (D) Rs. 25000 is—
69. The smallest number which 75. Twenty litres of a mixture (A) Rs. 4400 (B) Rs. 4445
when divided by 10, 15, 20 and contain milk and water in the (C) Rs. 4460 (D) Rs. 4520
35 leaves 6, 11, 16 and 31 as ratio 5 : 3. If 4 litres of this
remainder is— 82. In a competitive examination, a
mixture are replaced by 4 litres student scores 4 marks for every
(A) 416 of milk, the ratio of milk to correct answer and looses 1 mark
(B) 424 water in the new mixture will for every wrong answer. If he
(C) 436 become— attempts all 75 questions and
(D) None of these (A) 2 : 1 (B) 6 : 5 secure 125 marks the number of
70. One third of the boys and one 1 questions he attempts correctly
(C) 7 : 3 (D) 8 : 3
half of the girls of a college par- 8 is—
ticipate in a social work project. (A) 35 (B) 40
76. A milkman procures milk at the
If the number of participating (C) 42 (D) 46
rate of Rs. 4·50 per litre and sells
students is 300 out of which 100 it to his customers at the same 83. The average of three numbers is
are boys. What is the total num- rate. If he makes a profit of 20. If two of the numbers are 16
ber of students in the college ? 1 and 22, the third is—
(A) 500 (B) 600 12 % the quantity of water he
2
(C) 700 (D) 800 (A) 18 (B) 19
mixes for every litre of milk is—
71. A jar full of whisky contains (A) 50 ml (B) 100 ml (C) 20 (D) 22
40% of alcohol. A part of this (C) 125 ml (D) 175 ml 84. A student who secure 20%
whisky is replaced by another marks in an examination fails by
containing 19% alcohol and now 77. A man distributed Rs. 100
30 marks. Another student who
the percentage of alcohol was equally among his friends. If
secure 32% marks gets 42 marks
found to be 26. The quantity of there had been five more friends,
more than those required to pass.
whisky replaced is— each would have received one
Per cent of mark required to pass
1 2 rupee less. How many friends
(A) (B) is—
3 3 had he ?
(A) 20 (B) 25 (A) 20 (B) 25
2 3
(C) (D) (C) 30 (D) 35 (C) 28 (D) 30
5 5
72. 60% of the length of a pole is 78. A car complete a certain journey 85. If the L.C.M. of x and y is z their
painted red, 40% of the rest is in 8 hours. It covers half the H.C.F. is—
painted green and 50% of the distance at 40 kmph and the rest xy
balance is painted blue. The at 60 kmph, the length of the (A) z (B) xyz
remaining unpainted length of journey is— x+y z
the pole is 30 cm. The length of (A) 350 km (B) 384 km (C) z (D) xy
the pole is— (C) 400 km (D) 420 km
(A) 2·5 m (B) 3·2 m 86. Kasim can do a piece of work in
(C) 4·8 m (D) 5·6 m 79. The ratio between a two digit 1
number and the sum of the digits 7 hours and Sunil can finish it
2
73. A train X starts from Meerut at 4 of that number is 7 : 1. If the in 10 hours. If Kasim works at it
p.m. and reaches Ghaziabad at 5 digit in the tenth place is one for 3 hour and Sunil for 4 hours
p.m. while another train Y starts more than the digit in the unit the amount of work left unfini-
from Ghaziabad at 4 p.m. and place. What is the number ? shed is—
reaches Meerut at 5·30 p.m. The
(A) 21 1 2
two trains will cross each other (A) (B)
at— (B) 32 5 5
(A) 4·36 p.m. (B) 4·42 p.m. (C) 43 1 2
(C) (D)
(C) 4·48 p.m. (D) 4·50 p.m. (D) None of these 4 7

Quantitative Aptitude Test | 334


87. Two numbers are in the ratio 92. 15 persons can fill 35 boxes in 7 97. A’s money is to B’s money as
3 : 4 and the product of their days. How many Persons can fill 4 : 5 and B’s money is to C’s
L.C.M. and H.C.F. is 10,800. 65 boxes in 5 days ? money as 2 : 3. If A has Rs. 800
The sum of the numbers is— (A) 13 (B) 39 C has—
(A) 180 (B) 210 (C) 45 (D) 65 (A) Rs. 1000 (B) Rs. 1200
(C) 225 (D) 240 93. A man drives 4 km distance to (C) Rs. 1500 (D) Rs. 2000
88. Pratap bought a radio with 25% go around a rectangular park. If
discount on the original price. the area of the rectangle is 0·75 98. If a man walks at 5 km/hr he
He got Rs. 40 more than the sq km the difference between the reaches the bus stop 5 minutes
original price by selling it at length— too late and if he walks at 6
140% of the price at which he (A) 0·5 km (B) 1 km km/hr he reaches the bus stop 5
bought. At what price did he buy (C) 2·75 km (D) 10·25 km minutes too early. The bus stop
the radio ? is at a distance of—
(A) Rs. 600 (B) Rs. 700 94. The difference between the sim-
(C) Rs. 800 (D) Rs. 900 ple interest and the compound (A) 3 km (B) 5 km
interest earned on a sum of
89. Four different bells ring at money at the end of four years at (C) 6 km (D) 10 km
intervals of 5, 6, 8 and 10 the rate of 10% p.a. is Rs. 256·40.
minutes respectively. If they ring 99. In a division sum, the divisor is
What is the sum ?
together at 4 p.m. they will ring twelve times the quotient and
(A) Rs. 4000 (B) Rs. 4500
together again at— five times the remainder. If the
(C) Rs. 5000 (D) Rs. 6000 remainder be 48, then the divi-
(A) 5·30 p.m. (B) 6·00 p.m.
(C) 7·00 p.m. (D) 8·10 p.m. 95. A sum of Rs. 65000 was divided dend is—
into 3 parts so that they yielded
90. The average of marks obtained (A) 240 (B) 576
the same interest when they were
by Sunil in History and (C) 4800 (D) 4848
lent for 2, 3 and 4 years at 8%
Mathematics is 60%. If he got 90 simple interest at the end of
marks out of 150 in Mathe- 100. A and B can complete a piece of
these periods. The ratio between
matics, how much did he get in work in 8 days. B and C in 12
these parts is—
History out of 100 ? days while C and A in 16 days.
(A) 2 : 3 : 4 (B) 4 : 3 : 2
(A) 1 (B) 6 They work together for 3 days
(C) 8 (D) 12 (C) 6 : 4 : 3 (D) 3 : 4 : 6
when A leaves off. In how many
a b c 96. Rs. 6450 is divided between days more will B and C finish
91. If = = , the value of
3 4 7 Rajan and Suresh so that for the remaining work ?
a+b+c every Rs. 8 that Rajan gets,
is— Suresh Rs. 7, their shares diffe- 1 1
c (A) 1 days (B) 2 days
rence by— 2 4
1 1
(A) (B)
2 7 (A) Rs. 390 (B) Rs. 430 1 1
(C) 7 days (D) 4 days
(C) 2 (D) 7 (C) Rs. 442 (D) Rs. 464 8 2

Answers with Hints


01. (A) 70) 245 (3 4
04. (D) x– x = 18
210 7
35) 70 (2 ⇒ 7x – 4x = 126
70 or x = 42
×
1 1 1
05. (C) : : = 15 : 10 : 6
∴ H.C.F. = 35 2 3 5

02. (C) Average =


1
(32 + 73 + 103 + 296) 3

⎯⎯⎯ (216216– 91)


√ 91 1/3
4 06. (B) 1– =
216
= ( ) = 7224 = 3
9 + 14 + 20 + 29
5×5×5
= (
216)
=(
6 × 6 × 6)
125 1/3 1/3 5
24 =
6
x x
03. (B) Let × = 1
31 279 07. (A) (1502) 2 – (1498) 2 = (1502 – 1498)
Then, x2 = 31 × 279 = (31 × 31 × 3 × 3) (1502 + 1498)
∴ x = 31 × 3 = 93 = 4 × 3000 = 12000

Quantitative Aptitude Test | 335


1 1 20. (C) 1 + 2 + 3 + 4 + 5 + 6 + 7 = 28
08. (B) Given expression = 1 + =1+
1 6 which is divisible by 7
1+ 1+
5 5
21. (D) Failed in Hindi only= (35 – 20) = 15%
6
5 16 Failed in English only= (45 – 20) = 25%
= 1+ = Failed in both
= 20%
11 11
3 Failed in one or both= (15 + 25 + 20)%
09. (D) = 0·75
4 = 60%
4 Passed in both = 40%
= 0·8
5
22. (C) L.C.M. of 8, 12, 15, 20 = 2 × 3 × 2 × 5 × 2
5
= 0·833 ∴ Least square number divisible by 8, 12, 15, 20
6
6 = 2 × 2 × 3 × 3 × 5 × 5 × 2 × 2 = 3600
and = 0·857
7
23. (D) C.P. of 4 commodities = Rs. 64
1
10. (A) Given expression = +
7
7 5 2
– –
9 9 9 [ ] Reduced price of 4 commodities = Rs. 52

1
= +0=
7
1
7
Saving % =
12
64 (
× 100 % )
3
575 3 63 72 100 = 18 %
11. (B) Given expression = – × + × 4
100 7 4 35 144
24. (C) 3 ––
23 27 10 1000 (31
= – +
4 4 7 9
10 3 61 100
= –1+ =
7 7 61
12. (D) Given expression = (252 × 185 × 59) = 25 39
Number to be added = (32) 2 – 1000
x 25 = (1024 – 1000) = 24
13. (C) × 250 + × 68 = 67
100 100
25. (B) 184 = 2 × 2 × 2 × 23
5x
⇒ = (67 – 17) = 50 mx + m (x + 1) = 184
2
So, m [2x + 1] = 184
∴ x = 50 ×
2
5 (
= 20 ) or 2x + 1 = m
184

⎯⎯289x = 5451

x By hit and trial m = 8
14. (B) Let
and x = 11
54 54
Then, = × So the number = 8
289 51 51
∴ x = (
54 54
× × 289 = 324
51 51 ) 26. (B) Let ten’s digit = x and unit’s digit = y
Then, (10x + y) + (10y + x) = 44
15. (C) Lengthwise there are 12 trees ⇒ 11 (x + y) = 44
Total distance between them = (11 × 2) m = 22 m or x+y = 4
∴ Length = (1 + 22 + 1) = 24 m
27. (C) 715 = 5 × 11 × 13
16. (D) Data inadequate and 825 = 5 × 5 × 11 × 3
17. (C) Less persons more days 12 : 18 : : 36 : x The least number is 5 × 3 by which 715 must be
18 × 36 multiplied to obtain a multiple of 825
∴ x = = 54
12 28. (D) Let the number be x and x – 11

18. (D) C’s 1 day’s work = (18 – 121 ) = 241 1


5
(x + x – 11) = 9

∴ C alone can finish the job in 24 days or 2x – 11 = 45


19. (A) (13) 2 + (11)2 = 169 + 121 = 290 or x = 28
∴ Least number to be subtracted = 1 ∴ The numbers are 28, 17.

Quantitative Aptitude Test | 336


x = (30 × 25)
29. (C) N =
2
3
P, K = (
2
3
1 2
P+ × P =
4 3 ) 5P
6
or
= 750
2 5 2 5
Ratio = P : P : P = : : 1 = 4 : 5 : 6 38. (A) Clearly 1x < 1 + x
3 6 3 6
If Pravin has Rs. 6 Kavita has Rs. 5 So, one of the numbers must be 1

If Pravin has Rs. 876, Kavita has Rs.


5
6(× 876 ) 39. (C) Let original price = Rs. x

= Rs. 730 Increased price = (100100+ P) x


N 4
30. (B) = (100 – P) 100 + P
4 6 Reduced price = × x
100 100
⇒ 6N – 4Y = 0
(100) 2 – P 2
or 3N – 2Y = 0 ∴ x = 1
(100) 2
Also N+Y = 600
Solving we get N = 240 only Y = 360 10000
or x =
(10000 – P2 )
∴ Difference = (360 – 240) = 120
31. (B) N = 123, Q = 83 = (41 × 3) Q + (82 + 1) 40. (A) 52x – 25x = 324
= 41 × (3Q + 2) + 1 ⇒ 27x = 324
∴ Required remainder = 1 ⇒ x = 12
32. (A) Let unit’s digit = x and ten’s digit = y
41. (B) Let sum = Rs. P
1 1
(10y + x) – (10y + x) = 2 P×2×5 P
3 4 Then, S.I. = =
100 10
( )
1 1

3 4
(10y + x) = 2
C.I. = P 1 +[( 5
100 ) – P]
2

or 10y + x = 24
40
∴ Number = 24 = P
400
Sum of the digits = 6
41 1
20 P– P = 4
33. (D) Distance between two poles = 400 10
(5 – 1)
= 5 metres or P ( 41

1
400 10 )
= 4

2 3 4 ∴ P = (400 × 4) = Rs. 1600


34. (A) = 0·285, = 0·3 and = 0·307
7 10 13
2 3
∴ Fraction in ascending order are ,
7 10
and
4
13
42. (B) Cost of 291 mangoes = Rs. (49·75
12
× 291)

= Rs. 1200 approx.


35. (C) 2500 – 1000 = 500 + x
43. (C) Do yourself.
or x = 1000
44. (C) Let C.P. = Rs. 100
So, he spent Rs. 1000 on repairs
Then, list price = Rs. 120
36. (C) Let these notes be 3x, 7x and 8x S.P. = 90% of Rs. 120 = Rs. 108
Ratio of their values = 3x × 1 : 7x × 2 : 8x × 5 Gain = 8%
= 3x : 14x : 40x
= 3 : 14 : 40 45. (C) It is more by (100
10
× 24·2
10
100
× 24·02)

Value of 5 rupee notes = Rs. 5130 ×( 40


57 ) = 2·42 – 2·402 = 0·018
1
= Rs. 3600 46. (B) 3x + 3 x = 60
3
3600
Number of these notes = = 720 or x = 15
5
∴ Cost of an adult ticket = Rs. 15
37. (D) Let total votes polled = x, then
52% of x + (52% of x – 98) + 68 = x 47. (D) x + y = 104 and x – y = 30
52 ∴ (x2 – y2) = (x + y) (x – y)
2× x – x = 30
100 = 104 × 30 = 3120

Quantitative Aptitude Test | 337


48. (B) B completes in 12 days, A will take 24 days and 56. (C) Capacity of tank = (12 × 13·5) litres
C will take 8 days = 162 litres
(A + B + C)’s 1 day’s work = ( 1
+
1 1
24 12 8
+ ) Number of new buckets =
162
9 ( ) = 18
6 1
= = 100 × 432
24 4 57. (D) Rate = = 9%
1200 × 4
So, all together will complete the job in 4 days. New rate = 10%
49. (A) Marks in Mathematics = (68 × 7 – 70 × 6) 1200 × 10 × 4
= (476 – 420) = 56 New interest = Rs. ( 100 )
2310 × 30 = Rs. 480
50. (A) Other number = = 330
210 Amount = Rs. 1680
51. (C) Let income = Rs. x 58. (A) Jayant Madhu = (6000 × 12 : 4000 × 6)
Then,
5
100
x+
20
100 (x–
5
100 )
x + 1919 = x = 3:1

x 19x
+ + 1919 = x
(
Madhu’s Share = Rs. 5200 ×
1
4 )
20 100 = Rs. 1300
or 5x + 19x + 191900 = 100x
[6000 × (1 + 1005 ) – 6000]
3
∴ 76x = 191900 59. (D) C.I. = Rs.
191900
= Rs. [6000 × × × – 6000]
or x = 21 21 21
76 20 20 20
= 2525 = Rs. 945·75
52. (D) Let the number of these coins be x and (100 – x) 3+x 7+x
1 100 – x 60. (A) =
x+ = 45 7+x 13 + x
2 4 ⇒ (3 + x) (13 + x) = (7 + x)2
⇒ 2x + 100 – x = 180
or x2 + 16x + 39 = x2 + 14x + 49
or x = 80
or x = 5
∴ Number of 50 paise coins = 80
61. (D) Charges for 320 km
⎯ 3 a2 = 36 √
√ ⎯3
53. (D)
4 = 60 × 4 + 60 × 5 + 8 × (2005)
⇒ a2 = 144
= Rs. (240 + 300 + 320)
or a = 12 cm = Rs. 860
∴ perimeter = 36 cm
54. (C) Let C.P. of apples = Re. 1 per kg ( 1
Balance = Rs. 860 – × 860
4 )
CP = Rs. 245, P = 110% of Rs. 24 = Rs. 26·40 = Rs. (860 – 215) = Rs. 645
Suppose he sold x kg at a loss of 5% Total amount with Ajit
95 120 = Rs. (860 + 645) = Rs. 1505
Then, x+ (24 – x) = 26·40
100 100 2
∴ 95x + 2880 – 120x = 2640 62. (B) x+ x = 63
5
or 25x = 240 ⇒ 7x = 63 × 5
or x = 9·6 kg or x = 45 cm
55. (A) Let amount at 15% be Rs. x
x × 15 × 1 (8000 – x) × 10 × 1
Shorter piece =( )2
5
× 45 cm = 18 cm
+ = 950
2 × 30 × 60
Average speed = (
30 + 60 )
100 100
63. (A) km/hr
or 15x + 8000 – 10x = 95000
5x = 15000 = 40 km/hr.
or x = 3000 64. (B) Total time = (180 + 27) min.
Amount at 15% = Rs. 3000 or = 207 min.
Amount at 10% = Rs. 5000 Free time = 15 min

Quantitative Aptitude Test | 338


Used time = (207 – 15) min Painted green = 16 m
= 192 min Balance = 24 m
192 Painted blue = 12 m
Duration of each period = = 48 min
4 Balance = 12 m
65. (C) Average If balance is 12 m total length = 100 m

=
3 (1 + 2 + 3 + 4 + 5 + 6 + 7 + 8 + 9)
9
= 15 If balance is
30
100
m, total length = (
100 30
×
12 100 )m
= 2·5 m
66. (A) Mean proportion = ⎯√⎯⎯⎯⎯⎯⎯⎯
0·32 × 0·02
73. (A) Suppose total distance = x km
= √⎯⎯⎯⎯⎯
0·0064 = 0·08 Then, X’s speed = x km/hr
M S
67. (B) –
3 2
= 30 and Y’s speed = ()
2x
3
km/hr
⇒ 2M – 3S = 180 Let them meet after y hours
Also, M+S = 240 2x y
Solving 2M – 3S = 180 Then, xy + = x
3
and M+S = 240
we get = 60 or ( )
y 1+
2
3
= 1

68. (C) Average now = (20 × 12515– 5 × 116) cm or


3
y = hr.
5

69. (A) Required number


= 128 cm.
= (
3
5 )
× 60 min.

= (1 cm of 10, 15, 20, 35) – 4 = 36 min.


= 416 So, they cross each other at 4·36 p.m.
70. (C)
and
Let the number of girls
number of boys
= x
= y
74. (B) 2 (2500 + 125 x) = x
or 30000 + 5x = 6x
1
Then, y = 100 or x = 30000
3
or y = 300 x
Share of youngest son =
x 2
Now, + 100 = 300 = Rs. 15000
2
or
∴ Total number of students
x = 400
= (300 + 400) = 700
75. (C) Milk in 20 litres = (20 × 85) litres
71. (B) Alcohol in first mix = 40 litres, water in it = 60 = 12·5 litres
litres.
Let x litres be replaced
Milk in 4 litres = 4 × ( )
5
8
litres

40 = 2·5 litres
Alcohol in (100 – x) litres = (100 – x) litres
100 ∴ Milk in new mix = (12·5 – 2·5 + 4) litres

Alcohol in new x litres = ( )19


100
x
= 14 litres
Water in it = 6 litres
2 19x ∴ Ratio of milk and water in
Total alcohol now (100 – x) +
5 100 new mix = 14 : 6 = 7 : 3
2 19x 76. (C) C.P. = Rs. 45
∴ (100 – x) + = 26
5 100 1
or 4000 – 40x + 19x = 2600 S.P. = 112 % of Rs. 4·50
2
1400 200
or x =
21
=
3 = Rs. ( 225
2 × 100
× 4·50 )
x 200 2
∴ Part replaced = =
100 3 × 100 3
= = Rs. ( ) 20·25
4
72. (A) Let total length = 100 m Rs. 4·50 is the cost of 1 litre
Then, Painted red = 60 m
Balance = 40 m
Rs.
20·25
4
is the cost of( 1
4·50
×
20·25
4 ) 9
= litres
8

Quantitative Aptitude Test | 339


1 ⇒ 5x = 200
∴ Water added to each litre = litre
8 or x = 40
= (18 × 1000) ml = 125 ml. 83. (D)
16 + 22 + x
3
= 20
77. (A) Suppose he had x friends ⇒ 38 + x = 60
100 100 or x = 22
x –x+5 = 1
84. (B) Let total marks = x
⇒ 100 (x + 5) – 100x = x (x + 5)

or
x2 + 5x – 500 – 0
(x + 25) (x – 20) = 0
Then, (
20
100
x + 30 ) = (100
32
x – 42)

3x
Hence x = 20 or = 72
25
72 × 25
78. (B)
x
+
x
2 (40) 2 (60)
= 8 ∴ ( 3 ) = 600
x x
Pass marks = ( x + 30)
⇒ + = 8 20
80 120 100
∴ 3x + 2x = 240 × 8
= ( × 600 + 30) = 150
20
or x = 384 km 100

Pass percentage = ( × 100)% = 25%


79. (A) Let number = 7x 150
and sum of digits = x 600
Let unit digit be y. Then, ten’s digit = x – y Product of numbers xy
85. (A) H.C.F. = = z
Now, x–y–y = 1 Their L.C.M.
or x – 2y = 1
10x – 10y + y = 7x
86. (A) Total work done = (3 × 152 + 4 × 101 × 101 )
= ( + )=
or x = 3y 2 2 4
5 5 5
Solving x – 2y = 1
Work unfinished = (1 – ) =
4 1
and x = 3y
5 5
We get y = 1
and x = 3 87. (B) Let the number be 3x and 4x
∴ Number = 21 Then, their, H.C.F. = x
and their L.C.M. = 12x
80. (A) Total time taken to cover 8 km
∴ 12x × x = 10800
2
( +
2
+
10 20 30 60
2
+
2
=) 24 2
= hours
60 5 or x2 = 900
or x = 30
∴ Average speed = 8 × ( )
5
2
km/hr So, the numbers are 90 and 120
= 20 km/hr The sum of the numbers = 210
88. (A) Suppose he bought the radio for Rs. x
(1 + 100R ) 3
81. (C) P = 6690 If C.P. is Rs. 75 original price = Rs. 100

and P (1 +
100)
R 6
= 10035 If C.P. is Rs. x, original price = Rs. ( )
100
75
×x

On dividing, we get (1 +
100)
R 3
=
10035
6690
= Rs. ()
4x
3
6690 6690 140 4x
∴ P= = x = + 40
100 3
(
1+
R 3
100 ) 10035
6690 or 420x = 400x + 12000
6690 × 6690 or x = 600
= = 4460
10035 89. (B) L.C.M. of 5, 6, 8, 10 2 5, 6, 8, 10
82. (B) Suppose the number of correct answers = x = 2 × 5 × 3 × 4 = 120 5 5, 3, 4, 5
wrong answers = (75 – x) So, they will ring together again 1, 3, 4, 1
∴ (4x – 75 + x) = 125 after 2 hours, i.e., at 6 p.m.

Quantitative Aptitude Test | 340


90. (A) Total marks obtained by Sunil = 60% of
(150 + 100) = 150
96. (B) Rajan’s Share = Rs. (6450 × 158 )
= Rs. 3440
∴ Marks obtained in History = (150 – 90)
Suresh’s Share = Rs. (6450 – 3440)
= 60
= Rs. 3010
a b c Difference = Rs. (3440 – 3010)
91. (C) Let = = = x
3 4 7 = Rs. 430
Then, a = 3x, b = 4x and c = 7x 97. (C) A:B = 4:5
a + b + c 3x + 4x + 7x 14x and B:C = 2:3
∴ = = =2
c 7x 7x A A B 4 2 8
∴ = × = × =
C B C 5 3 15
92. (B) More boxes, More persons, Less days, More
persons Thus A : C = 8 : 15
If A has Rs. 8, C has Rs. 15

35 : 65
5:7 }
: : 15 : x
If A has Rs. 800, C has Rs.
15
( )
× 800 = Rs. 1500
8
65 × 7 × 15
or x = = 39 98. (B) Let required distance = x km
35 × 5
x x 10
93. (B) 2 (x + y) = 4 – =
5 6 60
or x+y = 2 6x – 5x 1
Also, xy = 0·75 or =
30 6
Now, x–y = ⎯√⎯⎯⎯⎯⎯⎯⎯⎯
(x + y)2 – 4xy or 6x = 30
or x = 5 km
= √
⎯⎯⎯⎯⎯⎯⎯⎯
4 – 4 × 0·75 = 1 km
99. (D) Divisor = 12Q = 5R = 5 × 48 = 240
94. (A) Let sum = Rs. x
So Q = 20
x × 4 × 10
S.I. =
100 ()
= Rs.
2x
5
Dividend = (240 × 20 + 48) = 4848

[(
C.I. = x 1 + ) ]
10 4
100
–x 100. (C) 2 (A + B + C)’s 1 day’s work = (18 + 121 + 161 )
13
4641 =
= x 48
10000


4641
x–
2x
= 256·40
(A + B + C)’s 3 day’s work = (3 × 1396)
10000 5 13
=
or 641x = 2564000 32
or
A×2×8
x = 4000
B×3×8 C×4×8
Remaining work = (1 – 1332) = 1932
95. (C) = = =x 1
100 100 100 Q work is done by B and C in 1 day
12
25 25 25
A=
4
x, B =
6
x and C =
8
x ∴
19
32
work will be done by them in (
19
32 )
× 12 days
25 25 25 1
∴ A:B:C = : : =6:4:3 = 7 days.
4 6 8 8

Quantitative Aptitude Test | 341


Miscellaneous Exercise – V
Against each of the questions 2 16. If the cost price of 21 copies of a
(C) 12 km/hr
suggested answers are given. Find 3 book are the same as the selling
out the correct answer and mark it by (D) 14 km/hr price of 18 copies of the book,
putting a tick mark in the place then gain per cent is—
holders. 08. What per cent of 1 kg is 5 gms ? 2 2
(A) ·4% (B) ·5% (A) 14 % (B) 16 %
1. In an election one of the two 7 3
candidates gets 40% votes and (C) ·05% (D) ·005% 1 1
loses by 100 votes. Total number (C) 33 % (D) 23 %
09. Five bells begin to toll together 3 3
of votes is— and toll respectively at intervals
(A) 500 (B) 400 17. The length of a given rectangle
of 6, 7, 8, 9 and 12 seconds. is increased by 20% and the
(C) 600 (D) 1000 After how many seconds will breadth of the rectangle is de-
they toll together again ?
3 3 1 creased by 20%. Then, the new
2. 2 of ÷ + = ? (A) 72 sec. (B) 612 sec.
4 4 4 area—
3 (C) 504 sec. (D) 318 sec. (A) Remains the same
(A)
2 1 3 5 (B) Is increased by 4%
10. The lowest fraction in , , ,
9 2 4 6 (C) Is increased by 5%
(B) 7 2
4 , is— (D) Is decreased by 4%
1 12 5
(C) 18. A 200 metres long train, running
4 1 7
(A) (B) at a speed of 60 km/hr passes a
(D) None of these 2 12 bridge in 1 minute. The length of
5 2 the bridge is—
(2·3)3 – ·027 (C) (D)
3. The value of 6 5 (A) 1200 metres
(2·3)2 + ·69 + ·09
is— 1 3 5 7 9 (B) 900 metres
11. The H.C.F. of , , , , is—
(A) 2 2 4 6 8 10 (C) 800 metres
(B) 2·273 1 1 (D) 600 metres
(A) (B)
(C) 2·327 2 10 19. The sum of the digits of a two
(D) None of these 9 1 digit number is 8. If the digits
(C) (D)
120 120 are reversed the number is de-
4. 0·144 ÷ 0·012 = ?
12. The value of 3755 × 9999 is— creased by 54. The number is—
(A) ·12 (B) 1·2
(A) 37556245 (A) 62 (B) 71
(C) 12 (D) ·012
(B) 38297255 (C) 53 (D) 80
5. In a fort there were provisions
(C) 37546245 20. A mixture contains alcohol and
for 45 days for 150 men. After
10 days 25 men left over. The (D) 34657245 water in the ratio 4 : 3. If 7 litres
food would new last long for— of water is added to the mixture,
13. The value of ⎯
√⎯⎯⎯
(·121) is— the ratio of alcohol and water
(A) 36 days (B) 40 days (A) ·11 becomes 3 : 4. The quantity of
(C) 42 days (D) 50 days (B) 1·1 alcohol in the mixture is—
06. A candidate needs 35% marks to (C) ·347 (A) 10 litres (B) 12 litres
pass. If he gets 96 marks and fails (D) None of these (C) 32 litres (D) 48 litres
by 16 marks, then the maximum
marks are— 561 21. A series discount of 20%, 10% is
14. when reduced to the lowest
748 equivalent to a single discount
(A) 250 (B) 320
terms is— of—
(C) 300 (D) 425
13 3 2
07. A man travels a certain distance (A) (B) (A) 30% (B) 28 %
14 4 3
at the rate of 12 km/hr and 11 23
returns back to the starting point (C) (D) (C) 28% (D) 27%
14 24
at the rate of 15 km/hr. His 22. Bananas are bought at 15 for a
average speed during the whole 15. 133% can be written as— rupee and sold at the rate of 9 for
journey is— (A) 1·33 a rupee. The gain per cent is—
(A) 13·5 km/hr (B) ·133 (A) 30% (B) 60%
1 (C) ·0133 2 1
(B) 13 km/hr (C) 66 % (D) 33 %
3 (D) None of these 3 3

Quantitative Aptitude Test | 342


23. If A : B = 2 : 3 and B : C = 7 : 8, 5 1 1 45. 669, 568, 366, 263, 159—
33. 7 + 23 – 12 = ?
then A : C is— 32 3 8 (A) 468 (B) 263
(A) 9 : 11 35 35 (C) 366 (D) 568
(A) 18 (B) 17
(B) 7 : 12 69 96
46. 6, 3, 3, 4·0, 9, 22·5—
(C) 16 : 21 35 7
(C) 18 (D) 5 (A) 4·0 (B) 3
(D) None of these 96 48
(C) 9 (D) 22·5
24. If 28 men working 8 hours per 3 4
34. 7 of 1386 + of 837 = 27 + (?)2 For Questions 47–50—
day can finish a piece of work in 7 9
10 days. How many hours per (A) 103·6 (B) 103·16 Type A
day 40 men must work to com- (C) 130·16 (D) 203·16
plete the same work in 8 days ? Type B

1 35. 17298 + 13232 – 28989 = ? Type C


(A) 6 hours (B) 6 hours (A) 1541 (B) 1441
2 40
(C) 7 hours (D) 9 hours (C) 1641 (D) 1451 35
25. A man spends 76% of his 36. 664√
⎯⎯⎯⎯⎯⎯⎯⎯
15 + 75·086 = ? 30

Number of Vehicles
income. This income increases (A) 2900 (B) 3500 25
by 20% and he increased his (C) 4000 (D) 2700
expenditure by 15%. This sav- 20
ings are then increased by— 37. 20% of 3375 of 25·003 = ? 15
1 (A) 2800 (B) 2900
(A) 35% (B) 33 % 10
3 (C) 16900 (D) 2700
(C) 40% (D) 33% 5
38. 87300 + 99700 + 29000 = ?
0
26. 70201·002 + ? = 756 × 8 + (A) 218000 (B) 217000 1984 1985 1986 1987 1988 1989 1990
9·007—
(C) 216000 (D) 215000
(A) 65143·995 47. What was the percentage increase
(B) – 64143·995 2 4 2 in production of C type vehicles
39. 237 × 2 ÷ of 3·001 = ?
(C) 64143·995 23 21 3 from 1985 to 1986 ?
(D) 15143·995 (A) 270 (B) 260 (A) 10
(C) 250 (D) 280 (B) 5
27. 140% of ? = 13 × 400 – 780—
(A) 3157 (B) 3157·14 40. 537·07 × 238·06 – 5·56 ÷ 6·006 (C) 20
(C) 3517 (D) 3715 =? (D) None of these
(A) 127805 (B) 21000 48. The number of A type vehicles
28. 11882 ÷ 42 ÷ 7 ÷ ? = 20—
(A) 202 (B) 201 (C) 21100 (D) 21200 produced in 1986 was that per
cent of the number of C type
(C) 203 (D) 102 41. Among the following fractions
vehicles produced in 1988 ?
which one is the largest frac-
29. 820 × 739 ÷ 739 × 20 of 232 = ? 1
tion ? (A) 33 (B) 40
(A) 2804800 (B) 3804800 3
21 22
(C) 5804800 (D) 1804800 (A) (B) (C) 50 (D) 15
22 23
30. 40932 ÷ ? + 20 = ⎯
√⎯⎯⎯⎯⎯⎯⎯
15876 + 39 23 25 49. In how many years was the
(C) (D)
42 42 24 26 production of A type vehicles
(A) 282 (B) 280 less than its average production
145 145 42. 1447, 1422, 1395, 1390, 1366,
42 over the given years ?
(C) 281 (D) 283 1335, 1302—
145 (A) 2 (B) 4
(A) 1422 (B) 1390
7 11 17 (C) 3 (D) 1
31. 2 + 9 ÷ 12 = ? (C) 1366 (D) 1335
9 12 18 50. What was the average number of
43. 27, 28, 37, 60, 111, 192—
(A) 2 B type vehicles produced by the
(A) 60 company over the years ?
(B) 3
(C) 4 (B) 28 (A) 20000
(D) None of these (C) 111 (B) 25000
(D) Series is wrong (C) 15000
32. (0·80·8× 0·8 – 0·9 × 0·9
× 1 – 0·9 × 1 )
=?
44. 39, 38, 34, 24, 9— (D) None of these
(A) 1·4 (B) 1·17 (A) 9 (B) 34 Directions—(Q. 51-52) What
(C) 2 (D) 1·7 (C) 24 (D) 38 approximate value should come in

Quantitative Aptitude Test | 343


place of question mark (?) in each of 47·53 × 47·53 + 2 × 47·53
58. If a man walks to his office at
the following questions ? × 42·47 + 42·4 × 42·47
64. =?
3 38·25 × 38·25 + 138·25
51. 40√
⎯ 2 + 15% of 962 = ? th of his original speed. He
5 × 138·25 – 2 × 38·25
(A) 400 (B) 500 reaches there 10 minutes late. × 138·25
(C) 200 (D) 300 How much will he take to reach (A) 0·81 (B) 8·1
his office if he walks at his ori- (C) 81 (D) 18
1 1
52. 33 % of 33 % of 8979 = ? ginal speed ? What is necessary
3 3 to know the answer of the ques- 65. If the price of an article is
(A) 1000 (B) 1100 tion ? increased by 25% the number
(C) 900 (D) 2200 of articles sold is reduced by
(A) The distance of his office 22%. What will be the effect on
53. If the sum of the digits of a two- (B) His original speed revenue ?
digit number is 9 while their (A) 2·5% increase
product is 14. What is the 59. A shopkeeper sells some toys at
Rs. 250 each. To find out what (B) 2·5% decrease
number ?
per cent profit he makes. Which (C) 1·5% increase
(A) 36 (B) 63 of the following informations is/ (D) 1·5% decrease
(C) 54 (D) 72 are necessary ?
Directions—(Q. 66–67) Three
54. A trade man defrauds by means (A) Number of toys sold of the four parts are exactly equal.
of a false to 10% in buying and (B) Cost price of each toy Which of the parts is not equal to
to the, same extent in selling
60. The total monthly salary of all other three ?
goods. What per cent does he
gain on his outlay by defraud ? the employees in a company is 66. (A) (62 + 2 2 ) ÷ 22 – 17
2 Rs. 75000. To determine what is (B) 116 × 2 + 27 – 266
(A) 21% (B) 22 % the total number of employees in
9 (C) (62 × 9) ÷ 2√
⎯⎯49
that company ? Which of the
2 2 following information given in (D) None of these
(C) 20 % (D) 21 %
9 7 (A) and (B) is/are sufficient ? 67. (A) 20% of 40% of 50
55. If 10 boys and 12 girls together (A) 40 employees get Rs. 1000 (B) 60% of 50
complete a work in 10 days while per month 100
8 boys and 12 girls together can (C) % of 90
(B) 20% of the employees get 3
complete the same work in 12 Rs. 2000 per month
days one boy’s work is equal to (D) (82 – 72 ) + 15
the work of how many girls ? 61. Ram started a business investing (E) None of these
(A) 4 (B) 5 Rs. 45000. Three months after Directions—(Q. 68–69) What
he started, Anil joined him with
(C) 6 (D) 7 approximate value should come in
Rs. 60000. If at the end of the place of question mark (?) in the
Directions—(Q. 56–60) Answer year the total profit in the equation in each of the following
(a) If I alone is sufficient. business is Rs. 26000, what questions ?
would be the share of Anil in the
(b) If II alone is sufficient. profit ? 68. ⎯⎯⎯
√ 441 of 15 ÷ 3 + 4 = ?
(c) I and II together are not (A) Rs. 10000 (A) 107 (B) 113
sufficient.
(B) Rs. 14000 (C) 119 (D) 229
(d) Neither I is needed nor II is
needed. (C) Rs. 13000
69. 2·301 × 4·7 + 15 = ?
(e) I and II together are needed. (D) Rs. 16000
(A) 10 (B) 13
56. 15 article were bought for some Directions—(Q. 62–64) What (C) 17 (D) 29
money and 10 article were sold should come in place of question
for the same amount. What is mark (?) in the equation of each 70. Which number in the following
necessary to know the gain per question ? series is wrong ?
cent ? 574 286 142 72 34 16 7
62. 847·56 + 34·84 – ? = 582·83—
(A) Cost of 15 articles (A) 286 (B) 142
(A) 298·57
(B) Selling price of 10 articles (C) 72 (D) 34
(B) 300·67
57. A man is standing at a place. How (C) 299·47 71. What will be the difference
much time will a bus take to between the simple interest and
(D) None of these
cross him ? What is necessary to compound interest of Rs. 4400
know the answer the question ? 63. 30% of ? = 312 – 216. for 2 years at 5% per annum ?
(A) Speed of the bus (A) 320 (B) 420 (A) Rs. 55 (B) Rs. 507
(B) Length of the bus (C) 380 (D) 1318 (C) Rs. 403 (D) Rs. 44

Quantitative Aptitude Test | 344


72. Total number of mangoes avail- increased by 5 metres, the new (C) 21
able were consumed by 8 fami- length of the field will be 2600 (D) None of these
lies sharing equal mangoes. Had sq. metres. What is the breadth
the mangoes been shared by 10 of the rectangular field ? ⎯⎯⎯
√ 169 ⎯√⎯⎯
441 16
families per family consumption 75. × × =?
(A) 40 m 42 26
would have reduced by 8 man- ⎯⎯225

(B) 60 m
goes. What was the total number 4
(C) 65 m (A)
of mangoes ? 5
(D) Cannot be determined 9
(A) Data inadequate (B)
Directions—(Q. 74–76) In each 15
(B) 320 question what will come in place of 57
(C) 160 question mark ? (C)
29
(D) 240 6 × 7 – 7 × 3 15 5 (D) None of these
74. + ÷ =?
73. The ratio between the length and 1·5 + 1·5 3 3 76. 470 × 40 ÷ 5 + 57 – 3 = ?
breadth of a rectangular field is (A) 56 (A) 3503 (B) 3760
3 : 2. If only the length is (B) 9·33 (C) 1889 (D) 3814

Answers with Hints


01. (A) Out of 100 difference in votes 5 1
08. (B) Fraction = =
= (60 – 40) = 20 1000 200
If difference is 20 total votes = 100 Required percentage = (2001 × 100)% = ·5%
If difference is 100 total votes =
20(
100
× 100 = 500) 09. (C) L.C.M. of 6, 7, 8, 9, 12 is 504.
3 3 1 3 4 1 10. (D) L.C.M. of denominators = 60
02. (B) 2 of ÷ + = × +
4 4 4 2 3 4 1 30 3 45 5 50 7 35 2 24
∴ = , = , = , = , =
1 9 2 60 4 60 6 60 12 60 5 60
= 2+ =
4 4

03. (A) Given expression =


a3
– b3
So, (25) is least.
a2 + ab + b2 11. (D) H.C.F. of 1, 3, 5, 7, 9 is 1 and L.C.M. of given
= a – b = 2·3 – ·3 1
fractions = .
120
= 2
12. (C) 3755 × 9999 = 3755 × (104 – 1)
0·144 144
04. (C) = = 12 = 37550000 – 3755 = 37546245
0·012 12

⎯√⎯⎯ ⎯√⎯⎯⎯⎯
05. (C) Remaining days = 35 Remaining men = 125
Now, 150 men have provisions for 35 days.
13. (C) ⎯⎯⎯
√ ·121 = ( )1210
10000
=
(1210)
100
35 × 150 34·7
125 men will have it for = 42 days = = ·347
125 100
06. (B) 35% of x = 96 + 16 = 112 14. (B) H.C.F. of 561, 748 is 187.
135 3
or × x = 112 Dividing Nr. and Dr. by 187 the fraction is .
100 4
112 × 100 133
or x = = 320 15. (A) 133% = = 1·33
35 100
07. (B) Let that distance be x km 16. (B) Let C.P. of each book be Re. 1.
x x 9x C.P. of 21 books = Rs. 21
Time taken to cover 2x km = + = hrs
12 15 60 ∴ S.P. of 18 books = Rs. 21
2x × 60 21
Average speed = km/hr S.P. of 1 book = Rs.
9x 18
1
= 13 km/hr.
3
Gain on Re. 1 = Rs. (2118 – 1) = Re. 16
Quantitative Aptitude Test | 345
Gain = (16 × 100)% Now, income = Rs. 120

= 16 %
2 New expenditure = Rs. (115
100
× 75) = Rs.
345
4
3
Now saving = Rs. (120 –
4 )
345 135
17. (D) Area = 1 × b = A (say) = Rs.
4
New area = ( 120

100 100
80
b =) 24
25
24
1b = A
25 Increase % in saving = (
35
4 × 25
× 100) = 35%

(
Decrease on A = A – A =
24
25 ) A
25
26. (B)
31. (D)
27. (B)
32. (D)
28. (A) 29. (B) 30. (A)

Decrease % = ( A
25A )
× 100 = 4% 5 1 1
33. (C) 7 + 23 – 12 = ?
32 3 8
18. (C) Distance covered by the train in /min.

=
60 × 1000
60
∴ ? = (7 + 23 – 12) +(325 + 13 – 18)
= 18 + (
96 )
= 1000 metres 15 + n
∴ 200 + (length of bridge) = 1000
or length of bridge = 800 metres. 35 35
= 18 + = 18
96 96
19. (B) Let the tens and units places be x and y respec-
tively. 3 4
34. (B) 7 of 1386 + of 837 = 27 + (?)2
Then, x + y = 8 and (10x + y) – (10y + x) = 54 7 9
or x + y = 8 and x – y = 6 52 4
⇒ 1386 × + 837 × = 27 + (?)2
7 9
∴ x = 7, y = 1, so the number is 71.
∴ (?)2 = 10668 – 27 = 10641
20. (B) Let alcohol and water be 4x and 3x litres respec-
tively. Then ∴ ? = 103·16
4x 3 35. (A) 17298 + 13232 – 28989 = ?
= or x = 3
3x + 7 4 ? = 30530 – 28989
∴ Alcohol = 12 litres = 1541
21. (C) Let C.P. be Rs. 100 36. (D) 664√
⎯⎯15 + 75·086 = ?
S.P. = Rs. ( 90
100 )
× 80 = Rs. 72 ? = 664 × 4 + 75 = 2731
= 2700 (App.)
∴ Discount = Rs. (100 – 72) = 28%
37. (C) 20% of 3375 of 25·003 = ?
So, answer (C) is correct.
22. (C) Let bananas bought (15 × 9) 20
? = 3375 × × 25·003
100
Then, C.P. = Rs. 9 S.P. = Rs. 15
20
Gain % = ( 6
9 )
× 100 % = 3380 ×
100
× 25

= 16900 (App.)
2
= 66 % 38. (C) 87300 + 99700 + 29000 = ?
3
A 2 B 7 ? = 216000
23. (B) = and =
B 3 C 8 2 4 2
39. (B) 237 × 2 ÷ of 3·001 = ?
A A B 2 7 14 7 23 21 3
∴ = × = × = =
C B C 3 8 24 12 2
? = 237 × 2·2 ÷ 3
24. (C) More men less hours per day indirect. 3
Less days more hours per day (indirect) = 521·4 ÷ 2 = 260·7
= 260 (App.)
40 : 28
8 : 10 } :: 8 : x 40. (A) ? = 537·07 × 2378·06 – 5·56 ÷ 6·006
28 × 10 × 8 5·50
∴ x = = 7 hrs. ? = 537 × 238 –
8 × 40 6
25. (A) Let income be Rs. 100. = 127806 – 0·917
Then, expenditure = Rs. 75 saving = Rs. 25 = 127805 (App.)

Quantitative Aptitude Test | 346


21 22 100 100
41. (D) Q = 0·955, = 0·957 52. (A) ? = 8979 × × = 998
22 23 300 300
23 24 = 1000
= 0·958, = 0·960
24 25 53. (D) Let the number be 10x + y
25
= 0·962 ∴ x+y = 9
20
and xy = 14
25
∴ The largest fraction is ·
26 ∴ (x – y)2 = (x + y)2 – 4xy
42. (B) 1395 = (9) 2 – 4 × 14
1447 1422 1390 1366 1335 1302
= 87 – 56 – 25
–25 –27 –29 –31 –33 x – y = 57
Hence, it is clear that instead of 1390, 1395 will ∴ x = 7
come.
and y = 2
43. (A) 62
∴ Number = 72
27 28 37 60 111 192
2x × 100
54. (B) Gain % = where x = 10
+(1)2 +(3)2 +(5)2 +(7)2 +(9)2 100 – x
Hence, it is clear that instead of 60, 62 will come. 2 × 10 × 100 2000
= =
100 – 10 90
44. (C) The order of the given number series is in this
way— 2
= 22 %
9
25
39 38 34 24 9 55. (C) In 10 days the work is completed by 10 boys +
12 girls
–(1)2 –(2)2 –(3)2 –(4)2 ∴ In 1 day the work is completed
Hence, it is clear that instead of 24, 25 will come. = 100 boys + 120 girls
45. (D) 569 Similarly in 12 days the work is completed
669 568 468 366 263 159 = 8 boys + 12 girls
∴ In 1 day work is completed
–100 –101 –102 –103 –104
= 96 boys and 144 girls
Hence, it is clear that instead of 568, 569 will come.
or Work of 4 boys = work of 24 girls
46. (A) 4·5
∴ Work of 1 boy = work of 6 girls.
6 3 3 4 9 22·5
56. (D) Let the cost of 15 articles be Rs. x
× 0·5 ×1 × 1·5 ×2 × 2·5
2
∴ C.P. of 1 article =
Hence, it is clear that instead of 4, 4·5 will come. 15
47. (D) Required percentage of increase x
and S.P. of 1 article =
(15·10) × 100 10
= = 50%
10 x x
– × 100
10 10 15
48. (C) × 100 = 50% ∴ Gain per cent =
20 x
49. (B) In 4 years there is less production in the vehicle 15
that the average production. x × 15
50. (D) Average number of B types vehicles produced = × 100
30 × x
over the years
= 50%
15 + 22·5 + 30 + 22·5 + 15 + 20 + 25
= Hence, without knowing I or II gain % can be
7
calculated.
150
= = 21·428 thousand Distance
7 57. (D) Since time =
Speed
15
51. (C) ? = 40 × 1·4 + 960 × = 200 ∴ Both I and II are needed.
100
Quantitative Aptitude Test | 347
x×t 67. (A) (A) = 20% of 40% of 50 0
58. (D) Time =
y–x 20 40
= × × 50 = 4
x
3 100 100
× 10
yt
5 6×5 (B) = 60% of 50
or Time = = = = 15 min.
x 3 2 60 × 50
1 –y 1 – = = 30
5 100
Neither, I is needed nor II is needed. 100
(C) = × 90
59. (C) If S.P. of each toy is not known, per cent profit 3 × 100
cannot be calculated. Hence, I and II together are not = 30
sufficient.
(D) = (82 – 72 ) + 15
60. (C) I and II together are not sufficient. = (64 – 49) + 15 = 30
61. (C) Investment of Ram for 1 month 68. (A) ? = 15 × 21 ÷ 3 + 4
= 45000 × 12 = 15 × 7 + 4 = 109 = 107
= Rs. 540000 69. (D) ? = 2·301 × 4·7 + 15
and investment of Anil for 1 month = 2·3 × 4·7 + 15
= 60000 × 9 = 10·8 + 15 = 26 = 29
= Rs. 540000 70. (C) 70
∴ Ratio in their investments = 1 : 1 574 286 142 72 34 16 7
1
∴ Share of Anil in the profit = × 26000
2 ÷2–1 ÷2–1 ÷2–1 ÷2–1 ÷2–1 ÷2–1

= Rs. 13000 Hence, there should be 70 in place of 72.


62. (D) ? = 847·56 + 34·84 – 582·83 = 299·57 71. (D)
63. (A) 30% of ? = 312 – 216 72. (B) Let the total number of mangoes be x
30 × ? x x
or = 96 Then, – = 8
100 8 10
96 × 100 5x – 4x
∴ ? = = 320 or = 8
30 40
(47·53)2 + 2 × 47·53 × 42·47 + (42·47)2 x
64. (A) ? = or = 8
(38·25)2 – 2 × 38·25 × 138·25 + (138·25)2 40
(47·53 + 42·47)2 (90) 2 ∴ x = 8 × 40 = 320
= 2 = = 0·81
(38·25 – 138·25) (100) 2 73. (A) Let the length of the field be 3x metres
65. (B) % effect on revenue Its breadth = 2x metres
= Increase % value – decrease % value (3x + 5) × 2x = 2600
Increase % value × decrease % value or 6x2 + 10x – 2600 = 0
=
100
or 3x2 + 5x – 1300 = 0
25 × 22
= 25 – 22 – = 3 – 5·5 = – 2·5 or 3x2 + 65x – 60x – 1300 = 0
100
or x (3x + 65) – 20(3x + 65) = 0
= 2·5% decrease
or (3x + 65) (x – 20) = 0
66. (C) (A) = (62 + 2 2 ) ÷ 22 – 17
∴ x = 20
= 40 ÷ 7 – 17 = –7
∴ Breadth = 2 × 20 = 40 cm
(B) = 116 × 2 + 27 – 266
42 – 21 15 3
= 232 + 27 – 266 74. (D) ? = + ×
3 3 5
= –7 = 7 + 3 = 10
13 21 16
(C) = (62 × 9) ÷ 2√
⎯⎯49 75. (D) ? = × ×
42 26 15
229 1 1 16 4
= 558 ÷ 14 = = × × =
7 2 2 15 15
(D) = 343 ÷ 49 – 14 76. (D) ? = 470 × 8 + 54
= 7 – 14 = –7 = 3760 + 54 = 3814
Quantitative Aptitude Test | 348
Miscellaneous Exercise – VI
01. The length of the longest pole 07. Insert the missing number— (A) 12% (B) 18%
that can be put in a room (25 5, 12, 9, 16, 13, 20…… 2 5
metres × 12 metres × 8 metres) (C) 11 % (D) 10 %
(A) 27 3 7
is—
(B) 23
(A) 25 metres (C) 17 14. ⎯⎯⎯⎯⎯⎯⎯⎯⎯⎯
√ {(65)2 – (16) 2 } = ?
(B) 45 metres (D) None of these (A) 43 (B) 47
(C) 27·7 metres (C) 63 (D) 67
08. If the numerator of a fraction be
(D) 28·8 metres increased by 12% and its de- 15. A watch is bought for Rs. 200
02. Circumference of a circle is 132 nominator decreased by 2% the and sold the same day for Rs.
cm. The area of the circle is— 242 at a credit of 2 years. If the
value of the fraction becomes rate of interest is 10% com-
(A) 792 sq. cm pounded annually then there is—
(B) 1056 sq. cm (67). The original fraction is— (A) A gain of 1%
(C) 1386 sq. cm
(A) ( ) (B) ( )
3 3 (B) A gain of 2%
(D) 924 sq. cm 4 5 (C) Neither gain nor loss
(C) ( ) (D) ( )
03. The diagonal of a square field is 2 2 (D) A loss of 1%
25 metres. The area of the field 3 5
is— 16. What is the S.P. of a 7% stock in
09. A and B enter into partnership which an income of Rs. 250 is
(A) 625 sq. metres with capitals as 4 : 5. At the end derived by investing Rs. 3500,
(B) 312·5 sq. metres
(C) 156·25 sq. metres
of 9 months A withdraws. If the
shares of annual profits be in the brokerage being
1
8() %?
ratio 9 : 10, then money of B
625 1 7
(D) sq. metres remained invested for— (A) Rs. 98 (B) Rs. 97
⎯2
√ (A) 10 months 8 8
04. A takes thrice as long to do a (B) 8 months 1
(C) Rs. 98 (D) Rs. 107
piece of work as B takes. A and (C) 6 months 8
B together can finish a piece of (D) 7 months 17. The population of a town is
work in 15 days. A alone can do decreasing at a uniform rate of

⎯√⎯( )
it in— 47 10% per annum for the last 3
10. The value of is—
(A) 30 days (B) 45 days 5 years. If the present population
(C) 60 days (D) 120 days (A) ·32 of the town is 137700, what it
was 2 years ago ?
05. A monkey ascends a greased (B) 3·17
pole 36 metres high. He ascends (C) 3·06 (A) 152847 (B) 160000
3 metres in first minute and de- (D) None of these (C) 170000 (D) 163657
scends 1 metre in second minute.
13 18. The edge of a cube is increased
He again ascends 3 metres in 11. If of an estate be worth Rs.
third minute and descends 1 15 by 100%. The surface area of the
metre in fourth minute and so 3 cube is increased by—
390, then of it is—
on. In what time he reaches the 5 (A) 100% (B) 200%
top ? (A) Rs. 320 (B) Rs. 270 (C) 300% (D) 400%
(A) 36 minutes (C) Rs. 450 (D) Rs. 324
19. If the time period of a bill is
(B) 33 minutes 12. By selling a radio for Rs. 240 I doubled, then the true discount
5 lose 20%. What per cent shall I
(C) 33 minutes on the bill is—
6 gain by selling it for Rs. 320 ?
2 1 (A) Doubled
2 (A) 6 % (B) 8 %
(D) 34 minutes 3 3 (B) Halved
3
06. A sum of money at S.I. doubles
in 7 years. It will become four
(C) 16 %
2
3
(D) 5% (C) Becomes
3
2 ()
times

times in— 13. The price of sugar increased by (D) None of these
12%. To maintain previous
(A) 14 years (B) 21 years Directions—The following table
budget, the consumption should
(C) 28 years (D) 35 years be reduced by— gives the plan outlay for 1987-88 for

Quantitative Aptitude Test | 349


three states A, B and C under the Study the graph and mark a tick the total pass percentage of both
major heads (in lakhs of rupees)— (√) against the correct answer in each the classes ?
of the following questions— (A) 74% (B) 68%
States A B C Total (C) 72% (D) 77%
Agriculture 1203·64 916·88 378·54 2499·06 29. Some passengers travel in a train.
Rural Development 925·46 741·34 217·62 1884·42 1
rd of the passengers stepped
Irrigation & Flood Control 85·34 72·20 40·36 197·90 3
Energy 2713·57 1015·65 617·42 4346·64 down on the first station and 560
Industries 1056·28 9000·00 136·46 2092·74 passengers entered the train, half
of the total passengers stepped
Education 336·86 432·39 186·54 955·79
down on the second station and
Social Services 100·97 118·42 31·41 250·80
24 more passengers entered in
Communication 81·23 100·20 0·82 182·25 the train and on the third station
Total 6503·35 4297·08 1609·17 12409·60 all 496 passengers stepped down.
How many passengers were trav-
Study the table and mark a tick Y elling in the train in the begin-
(√) against the correct answer in each 25 ning ?
questions given below— (A) 529 (B) 576
20
(C) 626 (D) 625
20. Which area received minimum 15
consideration in state A ? 30. The age of the father 2 years ago
10 was 6 times the age of his son. If
(A) Irrigation and Flood control 5 18 years hence his age will be 2
(B) Communication O
times that of his son, what are
6 7 8 9 10 11 12 X their present ages ?
(C) Social Services A.M. Time → A.M. Noon (A) 34 years, 9 years
(D) Rural Development 24. At what time will the cyclist pass (B) 36 years, 11 years
the man ? (C) 35 years, 7 years
21. The total plan outlay for A and C
(A) 9·20 AM (B) 8·50 AM (D) Data inadequate
is in the ratio—
(C) 7·50 AM (D) 9 AM 31. Which one of the following
(A) 19 : 50 (B) 101 : 25 25. At what distance from the start- products is equal to 2·74 × 0·174
(C) 3 : 2 (D) 65 : 16 ing point will the cyclist pass the × 0·65 ?
man ? (A) 2·74 × 4·74 × 0·650
22. Which area received maximum
consideration in all the states ? (A) 12 km (B) 13·5 km (B) 274 × 0·714 × 0·0065
(C) 12·6 km (D) 15 km (C) 27·4 × 1·74 × 0·650
(A) Agriculture (D) 2·74 × 0·074 × 0·650
26. If the length of the rectangular
(B) Rural Development plot is increased by 50% then 32. Which one of the following
(C) Energy how many per cent should its groups is in descending order ?
(D) Industries breadth be increased so that its 19 16 11
new area is 75% more than its (A) , ,
21 19 14
23. Which is the most appropriate original area ?
11 19 16
statement ? 1 (B) , ,
(A) 20% (B) 17 % 14 21 19
(A) Outlay for B is 6% more 4
19 11 16
than that of C 2 (C) , ,
(C) 16% (D) 16 % 21 14 19
(B) Outlay for B is 160% more 3
11 16 19
than that of C 27. A certain sum of money was (D) , ,
14 19 21
(C) None of these distributed amongst P, R and S
in the ratio 5 : 7 : 9 respectively. 33. How many numbers from the
A man starting at 6 AM walks at If the difference between the following are such that they are
the uniform rate of 6 km/hr resting of distributed money of P and S is divisible by 113 ?
10 minutes at the end of every hour. Rs. 2500. What is that certain 226, 339, 452, 566, 677, 791,
A cyclist, starting from the same sum of money ? 904, 1017 and 1121.
place at 7·30 AM travels in the same (A) Rs. 1312 (A) 3 (B) 4
direction at a uniform rate of 12 (B) Rs. 13525 (C) 5 (D) 6
km/hr. The following graph depicts
(C) Rs. 13125 34. If a number is divided by 119,
the distance covered by the man and
(D) Rs. 23125 the remainder is 19, what will be
the cyclist at various interval of time.
28. In a school there are 40 students the remainder if the same number
Scale— is divided by 17 ?
in class 8th, 80% of them passed
Along OX → 1 div. = 30 mts. and there are 60 students in class (A) 5 (B) 0
Along OY → 1 div. = 2·5 km 9th, 60% of them passed. What is (C) 2 (D) 9

Quantitative Aptitude Test | 350


43. 38% of 25 + 50 = 8 × 19 – 57 = Directions—(Q. 51–54) In each
35. 1524 + 2890 – 40√ ⎯ ? = 52 of 6410. of the following questions and equa-
(A) (B) tion is presented in four parts with =
(A) 2323 (B) 2026 64 × 12 – 673 = 20 × 7 – 15 × 3. sign in between parts. The four parts
(C) 2213 (D) 2116 (C) (D) of the equation are lettered (A) (B)
4 44. 230% of 280 + 56 = 6·5 × 240 (C) and (D). One of these parts in the
36. 1900 + 3 of 1484 – ?% of 1200 equation may be wrong and there by
7 (A)
= 6000. may not yield the same result as the
– 2·5 × 344 = 280% of 280 – 104 other three parts do. Find out the
(A) 100 (B) 115 (B) (C) wrong part. If there is any. If all the
(C) 117 (D) 122 four parts are correctly equated and
= 2816 – 1876 – 240.
2 (D) all of them yield the same result, the
37. 10570 – ? = 18 of 480 – 3960 + answer is (E) i.e., ‘No, error’.
3
45. 60 × 15 + 33 × 32 = 16 × 66 + 30 51. 10% of 0·12 = 12% of 0·1 = 1%
15.
(A) 5600 (B) 5450 (A) (B) (A) (B)
(C) 5555 (D) 1550 × 30 = 49·5 × 40 – 120% of 20 = of (12 × 0·1) = 100% of (120 ×
(C) (C) (D)
Directions—(Q. 38 to 42) In
22 × 60 + 12 × 53. 0·00001). No error
each of the following questions, a
number series is given after the series, (D) (E)
below it a number is given followed 46. 80% of 56 + 240% of 48 = 120% 52. 82 ÷ 43 × 102 – 8 = ⎯
√ 2 × ⎯√⎯32 × 9
by (a) (b) (c) (d). You have to (A) (A) (B)
complete the series starting with the of 120 + 320% of 50 = 62·5% of + 20 = 42 × 4 + 23 × 3 = 102 – 23 .
number given following the squence (B) (C) (D)
of the given series. 160 + 30% of 200 = 50% of 240 No error
38. 3 14 44 112 256 (E)
(C)
35 (a) (b) (c) (d) (e) 3×4 7 2 4
+ 160% of 25. 53. + = + +
Which of the following number
(D) 24 12 2 × 3 3 × 4
will come in place of (d) ? (A) (B)
(A) 368 (B) 768 2 7 2 1 3 4 1
47. + + 2 + 3 + 1 = + 1 + 10 3×4+2×4+2×3
(C) 572 (D) 472 5 5 5 5 5 6 3 =
2×3×4 2×3×4
39. 4 27 160 797 3184 9547 (A) (B) (C)
12 (a) (b) (c) (d) (e) 2 5 6 3 1 2 6 5 1 1 1
2 + = + – + + +6 = + + . No error
Which of the following number 3 6 7 7 7 7 7 7 2 3 4
will come in place of (b) ? (C) (D) (E)
(A) 506 (B) 496 1 2 1 1 54. 2·2 ÷ 0·01 × 0·001 = 2·2 × 0·0001
=1 +6 –4 +5+ .
(C) 498 (D) 360 3 3 3 3 (A) (B)
40. 4 21 120 595 2376 7125 (D) ÷ 0·0001 = 0·022 × ·01 ÷ 0·001 =
7 (a) (b) (c) (d) (e) Directions—(Q. 48 to 50) What (C)
Which of the following number will come in the place of question 2·2 ÷ 0·001 × 0·001. No error
will come in place of (c) ? mark (?) in each of the following (D) (E)
(A) 1775 (B) 1725 questions ? Directions—(Q. 55–60) Study
(C) 1230 (D) 1225 the following graph carefully and
41. 43 78 114 280 548 48. (0·–6 + 0·7– + 0·5– + 0·4–) = ? answer the questions given below—
1 3 Sale of Hardware by the
49·25 (a) (b) (c) (d) (e) (A) 4 (B)
4 9 Computer Industry over the
Which of the following number years
will come in place of (e) ? 4 5000
(C) 2 (D) 2 Domestic
(A) 1296 (B) 1371 9
4200
(C) 1386 (D) 1288 Exports
1·1 × 1·1 × 1·1 – 0·01 4000
42. 17 73 214 1080 3230 × 0·01 × 0·01
14 (a) (b) (c) (d) 49. =? 3200
1·1 × 1·1 – 0·01 × 0·01 3000
Rs. in crores

Which of the following number + 1·1 × 0·01


will come in place of (b) ? (A) 2 (B) 2·09 2000 1600 1800
(A) 178 (B) 183 1400 1200
(C) 2·05 (D) 1·09 1000
(C) 293 (D) 283 1000
900

Directions—(Q. 43 to 47) Three 5 200 400


50. 15% of 3016 + 2148 × +
of the four parts lettered (A) (B) (C) 1074 0
and (D) are exactly equal. You have 1
39 × of 130 = ?
1993-94
1991-92

1994-95
1992-93

1995-96

to find that part which is not equal to 3


the other three. If all the four parts (A) 2152·40 (B) 2253·4
are equal, then the answer is (E). (C) 2253 (D) 3125·5 Years

Quantitative Aptitude Test | 351


55. What was the difference in sale 60. What is the difference in the sub- (A) 1050
of hardware between domestic scription of V scheme between (B) 970
and exports in 1993-94 ? December and January ?
(C) 100
(A) Rs. 1000 crore (A) 80 lacs (B) 8 crore
(B) Rs. 500 crore (C) 240 crore (D) 240 lacs (D) 950
(C) Rs. 1200 crore 61. If the subscription to a scheme is (E) None of these
(D) Rs. 700 crore the criterion of popularity, which 66. 60 121 131 264 284 571 601
56. In which of the following years of the scheme can be termed as 120 (a) (b) (c) (d) (e)
was the percentage increase in most popular over the months ?
(A) V (B) W Which of the following numbers
sale of hardware in domestic
will come in place (d) ?
sector maximum over the pre- (C) X (D) Z
ceding year ? (A) 524
62. In which of the following months
(A) 1992-93 the total subscription to W and X (B) 1011
(B) 1993-94 scheme was equal to subscription (C) 1010
(C) 1994-95 to W scheme in February ?
(D) 514
(D) 1992-93 and 1993-94 (A) September
(B) October (E) None of these
57. What was the difference between (C) November 67. 2 4 9 20 43 90
the total hardware sale in export
(D) December
sector in 1992-93 and 1993-94 3 (a) (b) (c) (d) (e)
together and hardware sale in 63. For which of the following type
of schemes was there continuous Which of the following numbers
domestic sector in 1993-94 ?
decrease over the months ? will come in place of (d) ?
(A) Rs. 300 crore
(A) V (B) W (A) 58
(B) Rs. 200 crore
(C) X (D) Y (B) 99
(C) Rs. 400 crore
(D) Rs. 150 crore 64. What is the percentage increase (C) 48
in the subscription to Z scheme
58. Approximately what was the (D) 59
from January to February ?
percentage increase in the sale of (E) None of these
(A) 130
hardware in domestic sector
(B) 260 68. 200 184 193 157 182 118 167
from 1994-95 to 1995-96 ?
(C) 200 150 (a) (b) (c) (d) (e)
(A) 40 (B) 35
(D) None of these
(C) 25 (D) 30 Which of the following numbers
Directions—(Q. 65–68) In each will come in place of (e) ?
59. What was the difference in the of the following questions a number
average sale of hardware bet- (A) 132
series is given. After the series, a
ween the domestic and exports (B) 115
number is given followed by (a), (b),
sector ? (c), (d) and (e). You have to complete (C) 68
(A) Rs. 900 crore the series starting with the number (D) 117
(B) Rs. 1380 crore given following the sequence of the (E) None of these
(C) Rs. 1560 crore given series. Then answer the given
(D) Rs. 600 crore questions. 69. A second hand VCR when sold
for Rs. 8600 earns a profit.
Directions—(Q. 60–64) Study 65. 80 50 130 100 180 150 230 Which is double the amount of
the following table carefully and 900 (a) (b) (c) (d) (e) loss when the same is sold for
answer the questions given below Which of the following number Rs. 6500. What is the purchase
it— will come in place of (e) ? price of the VCR ?
Subscription to different Schemes of a Mutual Fund (A) Rs. 7600
Company over the Months
(B) Rs. 7100
(Rupee in crores)
(C) Rs. 7200
Scheme (D) Cannot be determined
V W X Y Z Total
Months (E) None of these
September 200 70 30 290 10 600 70. What will come in place of
October 120 130 70 150 290 760 question mark(?) in the follow-
November 45 35 25 125 160 390 ing equation ?
December 160 110 40 115 130 555 48·48 ÷ 1·2 + 0·8 = ?
January 80 90 70 100 140 480 (A) 41 (B) 41·2
February 130 150 30 40 390 740 (C) 24·24 (D) 4·84

Quantitative Aptitude Test | 352


Answers with Hints
1. (D) Length of longest pole
⎯√ 475 = √⎯√⎯⎯⎯(47) ⎯√ 5 = √
⎯⎯⎯
235 15·32
10. (C) × = = 3·06
= √⎯⎯⎯⎯⎯⎯⎯⎯⎯⎯⎯⎯⎯⎯⎯
{(25)2 + (12)2 + (8)2} ⎯(5) √ ⎯5 5 5
= 28·8 metres 13
11. (B) × x = 390
2. (C) 2πr = 132 15
390 × 15
So, r =
132 × 7
2 × 22
= 21 cm or x = ( 13 )
= 450

Area = πr2 = ( 22
7 )
× 21 × 21
3
5
of 450 = 270

= 1386 sq. cms. 12. (A) 80% of C.P. = Rs. 240


Diagonal 240 × 100
3. (B) Area = or C.P. = = Rs. 300
2 80
25 × 25 Now, C.P. = Rs. 300, S.P. = Rs. 320
=
2
= 312·5 sq. metres
Gain % = ( 20
300
× 100 %)
4. (C) The ratio between time taken by A and B to 2
= 6 %
finish work = 3:1 3
1 13. (D) If first expenditure is Rs. 100 then it is increased
Ratio of the rates of doing work = : 1 = 1 : 3
3 to Rs. 112. So, consumption of Rs. 12 should be
1 reduced out of Rs. 112.
(A + B)’s 1 day’s work =
15
1 1 1
∴ Reduction % = ( 12
112
× 100)
∴ A’s 1 day’s work = × =
15 4 60 5
= 10 %
[ divide
1
15 ]
in the ratio 1 : 3
14. (C) ⎯⎯⎯⎯⎯⎯⎯⎯⎯⎯

7
[(65)2 – (16) 2 ] = √
⎯⎯⎯⎯⎯⎯⎯⎯⎯⎯⎯⎯⎯⎯
[(65 – 16) (65 + 16)]
So, A can finish the work in 60 days.
5. (D) It is clear that the monkeys is capable of covering = √
⎯⎯⎯⎯⎯⎯⎯
[(49) (81)] = 7 × 9 = 63
2 metres in 2 minutes. So, in 34 minutes he is able to 15. (C) P.W. of Rs. 242 due 2 years hence
cover 34 metres. Rest of 2 metres he covers in () 2
3
⎡ 242
= Rs. ⎢⎢
10 2⎥
⎤⎥
minute. So, total time taken by him to reach the top
2
( )
⎢⎣ 1 + 100 ⎥⎦
is 34 minutes. 242 × 10 × 10
= Rs. (
11 × 11 )
3
6. (B) Rs. P is S.I. on Rs. P for 7 years.
= Rs. 200
Rs. 3 P is S.I. on Rs. P for (7
P )
× 3P = 21 years.
∴ There is neither gain nor loss.
7. (C) Numbers alternately increase by seven and 16. (B) To obtain an income of Rs. 7 investment
decrease by 3.
So, missing number is 17. ( )
= Rs.
3500
250
× 7 = Rs. 98
3
S.P. = Rs. (98 – ) = Rs. 97
8. (A) . 1 7
4 ∴
8 8
9. (B) Ratio of capital = 4 : 5
17. (C) P (1 –
100)
10 2
= 137700
Let B invested his money for x months.
Ratio of equivalent capital for 1 month 137700 × 10 × 10
or P = = 170000
= 4 × 9 : 5 × x = 36 : 5x 9×9
36 9 18. (C) Area = 6 × L2 = 6A (say)
∴ =
5x 10 New area = 6 × (2L)2 = 24 A

or x = (365 ×× 910) Increase % =


6A(
18A
× 100 % )
= 8 months = 300%

Quantitative Aptitude Test | 353


19. (D) There is no uniform pattern for the true discount ∴ Total pass percentage of both the classes
and the time of the bill. So, (D) is correct. 74
= × 100 = 74%
20. (B) Communication. 100
21. (B) Ratio of outlay for A and C 29. (B) Let the total number of the passengers in the
= (6503·35) : (1609·17) beginning be x then no. of passengers after crossing
= (4·04) : (1) the first station
x
= (404 : 100) = (101 : 25) = x – + 560
3
22. (C) The allocation for energy is maximum in each
state. 2x
= + 560
23. (B) (Outlay for B) – (Outlay for C) 3
= (4297·08 – 1609·17) No. of passengers after crossing the second station
= 2687·91 = (1 2x
2 3 )
+ 560 + 24
∴ Excess of B over A = ( 2687·91
1690·17 )
× 100 %
= (1 2x
)
+ 560 + 24 = 496
2 3
= 160%
x
24. (B) At the point of intersection of the graphs, the or = 192
cyclist passes the man drop a line parallel to OY. It 3
meets OX at a point to indicate 8·50 AM. Hence, the total number of passengers in the beginn-
25. (D) From the point of intersection of the graphs draw ing ⇒ x = 576.
a line parallel to OX to meet OY at a point 30. (D) Let the age of the son two years ago was x years.
respresenting 15 km. Then the age of the father = 6x years
26. (D) Let the original length and breadth of the plot be Hence, present age of the son = x + 2 years
x and y respectively.
Present age of the father = 6x + 2 years
∴ Area of the original plot = xy
(6x + 2 + 18) = 2(x + 2 + 18)
and area after the increase of 75%
or x = 5 years
3 7
= xy + xy = xy ∴ Present age of the father = 6x + 2
4 4

( ) (
x+
x
2
× y+
?
100 )y = xy
7
4 and
= 32 years
present age of the son = x + 2

or (3x
2
×4 1+
?
100) 7
= xy
4 31. (B) 274 × 0·714 × 0·0065
= 7 years.

or (100 + ?) × 6 = 700 19 16 11
32. (A) , , .
2 21 19 14
? = 16 %
3 33. (D) From the given numbers following are the
numbers which are divisible by 113 :
27. (C) Let the share of P, R and S be 5x, 7x and 9x
respectively 226, 339, 452, 791, 904 and 1017.
9x – 5x = Rs. 2500 34. (C) Number = 119 K + 19
= 17 × 7 K + (17 + 2)
x = Rs. 625
= 17(7 K + 1) + 2
Total sum of money = 5x + 7x + 9x
∴ Remainder when the number is divided by 17 is 2.
= 21x = 21 × 625
35. (D) 1524 + 2890 – 40 √
⎯ x = 1282 × 2
= Rs. 13125
or 40√⎯ x = 4414 – 2564
28. (A) Number of students passed in class 8th 1850
= 8% of 40 = 32
or ⎯ x = 40 = 46·25

Number of students passed in class 9th ∴ x = (46) 2 = 2116
x
= 70% of 60 = 42 36. (A) 1900 + 212 × 25 – 1200 × = 6000
100
∴ Total number of students who passed in both the or 1900 + 5300 – 6000 = 12x
classes = 74
1200
Total number of students of both the classes x =
12
= 40 + 60 = 100 = 100

Quantitative Aptitude Test | 354


56 1·1 × 1·1 × 1·1 – 0·01 × 0·01 × 0·01
37. (C) 10570x = 480 × – 3945 49. (D) 0
3 1·1 × 1·1 + 0·01 × 0·01 + 1·1 × 0·01
or x = 10570 + 3945 – 8960 (1·1)3 – (0·01)3
=
(1·1)2 + (0·01)2 + (1·1 × 0·01)
or x = 5555.
= (1·1) – (0·01) = 1·09
38. (B) 3 14 44 112 256 15
50. (A) 3016 × + 2 × 5 + 130 × 13
100
+4×2 +8×2 + 12 × 2 + 16 × 2
3
= 3016 × + 10 + 1690
(A) (B) (C) (D) 20
35 78 172 368 768 = 452·40 + 1700
∴ = 2152·40
39. (B) 4 27 160 797 3184 9547 10
51. (D) (A) = 10% of 0·12 = × 0·12
100
×7–1 ×6–2 ×5–3 ×4–4 ×3–5
= 0·012
(A) (B) 12
(B) = 12% of 0·1 = × 0·1
12 83 496 100

= 0·012
×7–1 ×6 – 2
(C) = 1% of (12 × 0·1)
40. (D) 4 21 120 595 2376 7125 1
= × 1·2 = 0·012
100
×7–7 ×6–6 ×5–5 ×4–4 ×3–3
(D) = 100% of (120 × 0·00001)
4 42 246 1225 100
∴ = × 0·00012 = 0·00012
100
×7–7 ×6–6 ×5–5
52. (C) (A) = 82 ÷ 43 × 102 – 8
41. (D) 43 78 144 280 548 = 68 ÷ 64 × 102 – 8 = 92

×2–8 × 2 – 12 ×2–8 × 2 – 12
(B) = ⎯ 2 × ⎯√⎯32 × 9 + 20

=8 × 9 + 20 = 92
(A) (B) (C) (D) (E) (C) =42 × 4 + 23 × 3 = 64 × 4 + 8 × 3
49·25 90·50 169 330 648 1288
∴ =256 + 24 = 280
×2–8 × 2 – 12 ×2–8 × 2 – 12 ×2–8 (D) =102 – 23 = 100 – 8 = 92
42. (A) 17 73 214 1080 3230 3 × 4 7 1 7 13
53. (C) (A) = + = + =
24 12 2 12 12
×4+5 ×3–5 × 5 + 10 × 3 – 10 2 4 10
(B) = + +
2×3 3×4 2×3×4
(A) (B)
14 61 178 1 1 5 13
= + + =
∴ 3 3 12 12
×4+5 ×3–5 3 × 4 + 2 × 4 + 2 × 3 12 + 8 + 6
(C) = = =1
2×3×4 24
43. (B) Excluding (A) the value of all three parts are
equal. 1 1 1 6 + 4 + 3 13
(D) = + + = =
2 3 4 12 12
44. (B) Excluding (C) the value of all three parts are 22
equal. 54. (B) (A) = 22 ÷ 0·01 × 0·001 = × ·001 = 2·2
·01
45. (A) The value of all the four parts are equal. (B) = 2·2 × 0·001 ÷ 0·0001 = 2·2 × 1 = 2·2
46. (D) Excluding (B) the value of all the three parts are ·022 × 0·01
(C) = 0·022 × 0·01 ÷ 0·001 =
equal. ·001
47. (B) Excluding (B) the value of the three parts are = ·22
equals. (D) = 2·2 ÷ 0·001 × 0·001
6 7 5 4 2·2
48. (D) 0·6 + 0·7 + 0·5 + 0·4 = + + + = × 0·001 = 2·2
9 9 9 9 0·001
22 4 55. (D) Reqd. difference = 1600 – 900
= =2 ·
9 9 = Rs. 700 crore

Quantitative Aptitude Test | 355


56. (C) Reqd. percentage in 1992-93 63. (D)
1400 – 1000 (390 – 140) × 100
= × 100 64. (D) Reqd. percentage =
1000 140
= 40 = 178·57
65. (B) 80 50 130 100 180 150 230
Reqd. percentage in 1993-94
1600 – 1400
= × 100 – 30 + 80 – 30 + 80 – 30 + 80
1400
Similarly,
= 14·29
Reqd. percentage in 1994-95
3200 – 1600
= × 100
1600
= 100 66. (E) 60 121 131 264 284 571 601
Reqd. percentage in 1995-96
×2+1 + 10 ×2+2 + 20 ×2+3 + 30
4200 – 3200
= × 100 Similarly,
3200
= 31·25
Hence, it is maximum in 1994-95.
57. (A) Reqd. difference = 1600 – (400 + 900)
= Rs. 300 crore 67. (D) 2 4 9 20 43 90
58. (D) Reqd. percentage increase
(4200 – 3200) × 100 ×2 ×2+1 ×2+2 ×2+3 ×2+4
=
3200 Similarly,
= 31·25 = 30 (Approx.)
59. (B) Average sale of hardware in domestic sector
1000 + 1400 + 1600 + 3200 + 4200
=
5
= Rs. 2280 crore 68. (D) 200 184 193 157 182 118 167
Average sale of hardware in exports sector
– (4)2 + (3)2 – (6)2 + (5)2 – (8)2 + (7)2
200 + 400 + 900 + 1200 + 1800
=
5 Similarly,
= Rs. 900 crore
∴ Reqd. difference
= 2280 – 900 = Rs. 1380 crore
60. (A) Reqd. difference = 160 – 80 = Rs. 80 lacs
61. (D) Total subscription of scheme V over the months 69. (C) Let the purchased price of the VCR be Rs. x.
= Rs. 735 crore ∴ Profit in I case = Rs. (8600 – x)
Total subscription of scheme W over the months
and loss in II case = Rs. (x – 6500)
= Rs. 585 crore
Total subscription of scheme X over the months ∴ (8600 – x) = 2(x – 6500)
= Rs. 265 crore or 8600 – x = 2x – 13000
Total subscription of scheme Y over the months or 3x = 21600
= Rs. 820 crore
21600
and total subscription of scheme Z over the months ∴ 8x =
3
= Rs. 1120 crore
= Rs. 7200
∴ Scheme Z is most popular.
70. (B) ? = 48·48 ÷ 1·2 + 0·8
62. (D) Subscription to W scheme in February
= Rs. 150 crore 48·48
= + 0·8
1·2
and total subscription to W and X schemes in
December = Rs. (110 + 40) crore = 40·4 + 0·8
= Rs. 150 crore = 41·2

Quantitative Aptitude Test | 356


Miscellaneous Exercise – VII
01. Find out the wrong number in (C) 2·25% decrease the waste pipe is open they can
the following sequence of num- (D) 2·25% increase together fill it in 18 minutes. The
bers waste pipe can empty the full
09. 37·09 × ? = (41 – ·291)— cistern in—
4, 13, 17, 26, 30, 38, 43.
(A) ·11 (B) 1·1 (A) 7 min. (B) 9 min.
(A) 13 (B) 26
(C) 11·1 (D) 9·1 (C) 13 min. (D) 23 min.
(C) 38 (D) 43
10. Which number will replace 17. If twice A is three times B and 5
02. Find out the wrong number in the both the question mark in times B is equal to 6 times C,
sequence
1, 18, 27, 84, 125, 216, 343. (
361 ?
?
=
81 )
? then A : C is—
(A) 4 : 5 (B) 5 : 9
(A) 1 (B) 27 (A) 191 (C) 9 : 5 (D) 5 : 4
(C) 84 (D) 216 (B) 931
18. Suresh and Jagdish start a
03. A is 5 times as old as his son B. (C) 171 business investing Rs. 15000 and
Four years hence the sum of (D) None of these Rs. 22000 respectively. After 4
their ages will be 50 years. B’s
11. The value of the expression months Suresh puts in Rs. 4000
age now is—
1014 × 986 is— more and Jagdish withdraws Rs.
(A) 8 years (B) 6 years 2000. At the end of year total
(C) 7 years (D) 5 years (A) 998924 (B) 999864
profit was Rs. 11500. What is
(C) 999804 (D) 996724 the share of Jagdish in it ?
04. The diameter of a cylindrical
tower is 10 metres and its height 12. (·98 × ·98 – ·98 × 1·52 + ·76 (A) Rs. 6900 (B) Rs. 5300
is 14 metres. The cost of painting × ·76) = ? (C) Rs. 6200 (D) Rs. 4600
the carved surface of the cylinder (A) ·0484 (B) ·2684
19. If 20 typists can type 480 pages
at 75 paise per square metre is— (C) ·3164 (D) ·1562 in 6 hours. How many pages
(A) Rs. 105 (B) Rs. 330 will be typed by 25 typists in 4
(C) Rs. 220 (D) Rs. 440
05. The dimensions of an open box
13.
⎯√⎯⎯
( ) 0·324

(A) ·18
10
=?

(B) 0·1
hours ?
(A) 256 (B) 576
are 52 cms, 40 cms and 26 cms. (C) 900 (D) 400
Its thickness is 1 cm. If 1 cubic (C) ·0018 (D) 2·8
20. A trader allows two successive
cm of the metal used in the box 14. Which of the following is in discount of 20% and 10%. If he
weights 1 gm, then he weight of descending order ? gets Rs. 108 for an article, then
the box is—
3 8 11 25 its marked price is—
(A) 8·48 kg (B) 6·58 kg (A) , , ,
8 15 23 81 (A) Rs. 142·56
(C) 7·28 kg (D) 658 gms 26 11 8 3 (B) Rs. 140·40
(B) , , ,
06. Four-fifth of a number is 10 more 81 23 15 8 (C) Rs. 160
than two-third of the number. 8 11 3 26
(C) , , , (D) Rs. 150
The number is— 15 23 8 81
(A) 55 (B) 65 21. The compound interest on Rs.
3 11 8 26 1
(C) 75 (D) 80 (D) , , , 2800 for 1 years at 10% per
8 23 15 81 2
07. A, B, C hired a Taxi for Rs. 3840 15. Two towns X and Y are some annum compounded annually
and used it separately for 72 distance apart. A man cycle from is—
hours 108 hours an 252 hours X to Y at a speed of 10 km/hr (A) Rs. 441·35
respectively. The amount paid and then back from Y to X at the (B) Rs. 3234
by C is— rate of 15 km/hr. The average (C) Rs. 420
(A) Rs. 960 (B) Rs. 2880 speed during the whole journey (D) Rs. 436·75
(C) Rs. 1920 (D) Rs. 2240 is—
22. What decimal fraction is 20 mm
08. When the price of a TV was (A) 12·5 km of a metre ?
increased by 15% the number of (B) 12 km (A) ·02 (B) ·2
TV’s sold by a company (C) The data is inadequate (C) ·05 (D) ·002
decreases by 15%. What was the (D) 13 km/hr 23. Deepak is twice as old as Vikas
net effect on the sale ?
16. Two taps can separately fill a was 3 years ago when Deepak
(A) No effect was as old as Vikas today. If the
cistern in 10 minutes and 15
(B) 1·5% increase minutes respectively and when difference between their ages

Quantitative Aptitude Test | 357


today be 5 years. What is the 29. In 1994 emissions in India were 3%. It would amount to how
present age of Deepak ? approximately what per cent of much ?
(A) 12 years (B) 16 years the total emissions in all the (A) Rs. 8432
(C) 18 years (D) 14 years given countries together ? (B) Rs. 9920
(A) 8 (B) 10
24. The average age of the husband (C) Rs. 10920
and wife at the time of their (C) 12 (D) 6
(D) None of these
marriage 6 years ago was 28 Directions—(Q. 30 to 32) In
years 6 months. Now the average each of the following question a 34. The inner radius of a same
of the husband, wife and a child question is asked followed by three spherical utensil is 12 cm. This
is 24 years. How old is the child ? informations. You have to decide utensil is filled with the help of
weather the question can be answered small cylindrical bottles. If the
1
(A) 2 years (B) 3 years with any one or two or all the three radius of the base of the bottle be
2
informations. 3
1 cm and height by 4 cm. How
(C) 3 years (D) 4 years 2
2 Answers—
many each bottles are required to
(A) If the informations (I) and (II) fill it ?
25. The length of the diagonal of a are necessary.
square is 4√
⎯ 2 cms. Its area is— (B) If the informations (II) and (III)
(A) 272 (B) 64
(A) 32 cm2 (B) 16 cm2 are necessary. (C) 128 (D) 136
(C) 8 cm2 (D) 36 cm2 (C) If the informations (I), (II) and 35. The area of rhombus is 169 cm2.
Directions—(Q. 26 to 29) Study (III) are necessary. If its one diagonal is double the
the table and answer the questions (D) If the informations (I), (II) and other. What are the length of
given below— (III) together are not sufficient. these diagonals ?
(E) None of these (A) 39 cm, 19·5 cm
Carbon Emission from Fossil
Fuel Burning, 1994 30. Pappu and Munna are the drivers (B) 26 cm, 13 cm
of two different buses. They are (C) 36 cm, 18 cm
Country Total Emiss- Emissions
Emiss- ions Per growth travelling in opposite directions (D) 40 cm, 20 cm
ions Person per and crosses one another in 27
Directions—(Q. 36 to 40) Study
(Mill- (Tonn- person seconds. What is the speed of
ions es) (1990-94) the following graph carefully and
Pappu’s bus ?
Tonnes) (per cent) answer the questions given below it—
Informations—
Russia 597 5·26 –44·0 Registration of New Vehicles
(I) The length of Pappu’s bus
U.S.A. 1289 0·71 13·0 in Delhi
(II) Speed of Munna’s bus
China 954 3·08 24·1 Cars 45
(III) The length of Munna’s bus 45
India 356 2·39 21·6 Total Vehicles
40
U.K. 123 0·24 –23·5 31. A sum of money of Rs. 1600 is 36 36
35
Japan 285 2·62 –0·3 to be distributed among P, Q and 30 28
Number in 000

27
Mexico 90 0·96 7·1 R. How many rupees will be 25 25
21 22
given top ? 20 20
26. Which was the country amongst 17
Informations— 15 15 15
the given countries which has 10
maximum emission per person in (I) R gets Rs. 180 more than P
5
1990 ? (II) Q gets the double amount of 0
(A) India (B) Russia that of R Jan. Feb. Mar. Apr. May June
(III) Q gets Rs. 240 more than 1991
(C) U.S.A. (D) Mexico
the average amount of all the 36. What was the difference between
27. Which country will stand at three the numbers of cars registered in
second position if arranged in January and those registered in
descending order of population 32. A train moving with a uniform
June ?
in 1994 ? speed crosses a standing bus in
20 seconds. What is the speed of (A) 2000
(A) China (B) Japan
the train ? (B) 6000
(C) U.S.A. (D) U.K.
Informations— (C) 500
28. What was the approximate sum
of the population of U.K. and (I) Length of the train (D) None of these
Japan in 1994 ? (II) Length of the bus 37. What was the number of vehicles
(A) 340 million (III) Speed of the bus other than cars registered in
(B) 370 million March 1991 ?
33. Rs. 6200 amounts to Rs. 9176 in
(C) 450 million four years at simple interest. If (A) 5000 (B) 10000
(D) 410 million the interest rate is increased by (C) 40000 (D) 37000

Quantitative Aptitude Test | 358


38. What was the percentage increase 45. When a amount was distributed 50. The difference between the cost
in registration of cars from equally among 12 boys. Each of price of table and chair is Rs.
February to March 1991 ? them got Rs. 60 more than the 1800. Assuming that the per-
(A) 0% (B) 1% amount received by each boy centage profit is same for the
(C) 2% (D) 2·5% when the same amount was table and the chair. What will be
distributed equally to 20 boys. the profit of selling six such
39. What was the increase in regis- What is the amount ? tables and four such chairs ? To
tration of vehicles other than cars (A) Rs. 1800 find the answer, which of the
from January to April 1991 ? following informations given in
(B) Rs. 1440
(A) 5000 (B) 10000 statements (P) and Q is/are
(C) Rs. 1600
(C) 15000 (D) No increase sufficient ?
(D) Data inadequate (P) The cost price of a table is
40. In which of the following month
46. Radha borrowed a certain sum of Rs. 4000.
was the registration of vehicles
simple interest from Laxmi (Q) The percentage profit is 25.
other than cars maximum ?
incurring 6% interest per annum (A) Only (P) alone is sufficient
(A) April (B) June for first 3 years, 8% interest for
(C) May (D) March (B) Only (Q) alone is sufficient
the period next 5 years. If he
paid Rs. 10080 as interest for a (C) Either (P) or (Q) is sufficient
41. The serial number of each player
of a team of volleyball of 8 total period of 8 years. What (D) Both (P) and (Q) together
players is according the marks sum had he borrowed ? are not sufficient
obtained in their display. The (A) Rs. 18000 (E) Both P and Q are needed.
best player got 85 marks. If he (B) Rs. 14000 Directions—(Q. 51–55) What
had gotten 92 marks, then the (C) Rs. 20160 approximate value should come in
average marks obtained by the place of question mark (?) in the
whole team would be 84. Find (D) None of these
equation in each of the following
the total marks obtained by the 47. The ratio between the present questions ?
whole team. ages of Omkar and Ram Kishan
51. 4242 × 42 = ?
(A) 666 is 4 : 5. Seven years ago, the ratio
of their ages was 3 : 4. What is (A) 168100 (B) 178100
(B) 672
the present age of Ram Krishan ? (C) 179100 (D) 135700
(C) 588
(D) None of these (A) 28 years 52. 777 × 404 = ?
(B) 35 years (A) 313000 (B) 313500
42. Ram was asked to multiply a
number by 31 by mistake he (C) 21 years (C) 213900 (D) 314000
muliplied that number by 13 and (D) Data inadequate 53. 8746 + 5827 + 47 = ?
so the answer calculated by him (A) 14500 (B) 13600
48. The length of a plot of land is
was 342 less than the correct
four times its breadth. A play- (C) 14600 (D) 14700
answer. What was that number ?
ground measuring 1200 sq. 54. 310% of 2800 = ?
(A) 191 (B) 17 metres occupies one-third of the
(A) 8650 (B) 8400
(C) 19 (D) 18 total area of the plot. What is the
length of the plot in metres ? (C) 8500 (D) 10500
43. Praveen spends 30% of his
monthly income on filling the (A) 90 55. 7500 × 0·7 = ?
1 (B) 30 (A) 5100 (B) 10714
petrol in the car he spends th of
4 (C) 20 (C) 5200 (D) 10500
the remaining income on house Directions—(Q. 56–70) In each
(D) None of these
rent and he spends his remaining question, what will come in place of
income on children’s education 49. Arti purchased three sarees with question mark ?
and food. If he spends Rs. 3000 a discount of 15% each on the
56. 14·14 × 2·40 + 6·064 = ?
on petrol, what sum of money listed price. She sold one saree
does he spend on house rent ? earning 20% profit and another (A) 33·396 (B) 40·000
saree earning 15% on the price (C) 34·064 (D) 34·000
(A) Rs. 1375 (B) Rs. 1750
she had purchased. If the listed 57. 40% of 120 = ?% of 96
(C) Rs. 3400 (D) Rs. 1500 price of each saree is Rs. 1000 (A) 24
44. In the following number of series how much profit did she earn by (B) 36
a wrong number is given. Find selling two sarees ? (C) 48
out the wrong number. (A) Rs. 397·50 (D) None of these
3, 4, 10, 30, 136, 685, 4116 (B) Rs. 1700 58. 7333 + ? – 4548 = 3602
(A) 685 (B) 30 (C) Rs. 200 (A) 707 (B) 807
(C) 4 (D) 136 (D) None of these (C) 827 (D) 817

Quantitative Aptitude Test | 359


59. 5678 + 3569 + 918 = ? 1 1 3 2 (C) 60
67. 4 + 2 ÷ × 4 = ?
(A) 10165 (B) 10255 2 4 4 3 (D) Data inadequate
(C) 11965 (D) 10145 (A) 9 73. Machine A can print 100000
(B) 7 books in 8 hours, machine B can
60. 9865 – 5134 +786 = ? 2 print the same number of books
(A) 4617 (B) 5517 (C) 1
7 in 10 hours and machine C can
(C) 4731 (D) 5607 (D) None of these print the same number of books
in 12 hours. All the three
1 68. 209 × 33 + 21 = ? machines began to work at 9·00
61. 5 × 0·20 + 0·80 – 0·65 = ?
4 (A) 6897 AM. At 11 AM the machine A
(A) 1·85 (B) 1·20 (B) 6928 stops to work while the other
(C) 1·65 (D) 1·8375 (C) 6908 two continued their work. At
what time approximately the
(D) None of these
2 1 2 work will be complete ?
62. 12 – 5 ÷ 6 = ?
3 3 3 69. ⎯ ? + 720 ÷ 12 = 8 × 10 – 16
√ (A) 12·00 Noon
(A) 8 (A) 4 (B) 12·30 PM
1 (B) 12 (C) 1·00 PM
(B) 6
3 (C) 16 (D) 11·30 AM
2 (D) None of these
(C) 5 74. Two students fought a college
3 70. 286·3154 + 87·596 + 673·49 = ? election. The winning student
(D) None of these (A) 1047·315 got 60% of the total votes and
63. 2548 ÷ 26 + 13 = ? (B) 1046·9604 won the election by 144 votes.
(C) 1046·424 What was the total number of
1
(A) 65 (B) 111 votes polled ?
3 (D) None of these
(A) 720 (B) 360
(C) 85 (D) 98 71. 765 chairs are to be arranged in
(C) 480 (D) 240
columns in such a way that there
64. 5794 + 8326 + 785 = ? should be as many chairs in each 75. In a factory there are some
(A) 15085 (B) 14905 columns. In order to follow this supervisors and some labourers.
(C) 14815 (D) 14995 arrangement how many chairs On their silver jubilee function, 2
should be removed ? shirts to each labour and one
65. 195·84 ÷ 12·24 + 4·02 = ? (A) 6 (B) 36 shirt and one pant to each super-
(A) 12·00 (B) 16·00 (C) 19 (D) 27 visor are distributed. If in all 220
shirts and 20 pants are dis-
(C) 20·02 (D) 12·02 72. 20 buckets each of capacity 13·5 tributed, what is the total number
litres completely fills the tank. of workmen in the factory ?
24 × 3 – 18 × 3
66. =? How many buckets each of
8 × 7 – 23·5 × 2 (A) 100
capacity 9 litres will completely
(A) 2 (B) 1 fill the tank ? (B) 110
1 (A) 30 (C) 120
(C) (D) 6
2 (B) 32 (D) Cannot be determined

Answers with Hints


1 . (C) The numbers successively increase by 9 and 4
respectively. So, 38 is wrong. It must be 39. ∴ Cost of painting = Rs. (440 × 34)
2. (C) The number must be 13 , 23 , 33 , 43 , 53 , 63 , 73 . = Rs. 330
So, instead of 4 3 = 64 it is 84. 5. (B) Volume of metal
3. (C) Let B’s age be x. Then A’s age = 5x = (52 × 40 × 26 – 50 × 38 × 25) cu. cm.
= (54080 – 47500) cu. cm.
Q (x + 4) + (5x + 4) = 50
= 6580 cu. cm.
or x = 7
∴ Weight of metal = 6·58 kg
4. (B) Area of the curved surface 4 2
6. (C) x – x = 10
22 5 3
= 2πh = 2 × × 5 × 14
7 12x – 10x
or = 10
= 440 sq. m. 15
Quantitative Aptitude Test | 360
or 2x = 150 23. (B) Let the age of Vikas 3 years ago be x years.
or x = 75 Deepak’s age today = 2x years
7. (D) The amounts paid by them are in the ratio 72 : Vikas’s age today = (x + 3) years
108 : 252 i.e., 2 : 3 : 7. ∴ 2x – (x + 3) = 500x = 8
3840 × 7
∴ C’s share = Rs. ( 12 ) So, Deepak’s age today = 16 years
24. (B) The total age of husband and wife 6 years ago
= Rs. 2240
8. (D) Let the S.P. of a T.V. be Rs. x and number sold
be y.
( 1
)
= 28 × 2 years
2
= 57 years
Then, total sale = Rs. (xy)
The total age of husband and wife now
Total sale after increase in price = (57 + 12) years = 69 years
= ( )( )
115
100
x
85
100
y Total age of husband, wife and child now
= (24 × 3) years = 72 years
= (1·15 × 0·85)xy
Age of the child = (72 – 69) years = 3 years
= 0·9775xy 1
25. (B) Area = × (diagonal)2
∴ Decrease in sale = (·0225
1 )
× 100 % 2
1
= 2·25% = × 4√
2
( ) ⎯2
9. (B) 10. (C) 11. (C) 12. (A) 13. (A) = 16 cm2
14. (C) 15. (B) 16. (B) 26. (B)
17. (C) 2A = 3B and 5B = 6C 27. (A) On arranging the population in descending order
A 3 B 6 we get 1289, 954, 597, 356, 285, 123, 90.
∴ = and =
B 2 C 5 ∴ China stands at the second position.
A A B 3 6 9
So, = × = × = . 28. (D) The sum of the population of U.K. and Japan in
C B C 2 5 5
1994 = 123 + 285
18. (C) Ratio of their shares
= 408
= (15000 × 4 + 19000 × 8) :
= 410 millions (app.)
(22000 × 4 + 20000 × 8)
29. (B) Total emission of all the given countries in 1994
= 21000 : 24800 = 53 : 62
= 597 + 1289 + 954 + 356
11500 × 62
∴ Jagdish’s share = Rs. ( 115 ) + 123 + 285 + 90
= 3694 millions
= Rs. 6200
356
∴ Required percentage = × 100 = 9·6
20 : 25
19. (D) 6 : 4 } :: 480 : x 3694
30. (C) In order to determine the speed of Pappu’s bus
25 × 4 × 480 we should know the relative speed as well as the
∴ x = = 400
20 × 6 length of both the buses.
20. (D) Let the market price be Rs. 100 ∴ All the three statements are necessary.
Price after 1st discount = Rs. 80 31. (A) Let P’s share be Rs. x
Price after 2nd discount = Rs. (80 – 8) = Rs. 72 Then, R’s share = 180 + x
∴ 72 : 100 = 108 : x Then, Q’s share = 360 + 2x
100 × 108 4x + 540 + 240
or x = 360 + 2x =
72 3
= 150 2x = 180
So, the marked price = Rs. 150 x = 90
21. (B) Amount = 2800 × 1 + (
10
100
1+
5
100 )( ) Hence, all the three statements are necessary to
answer the question.
= Rs. 3234 32. (A) Speed of the train

22. (A) Required fraction = (1 × 10020 × 10) = ·02 = ( Length of the train + Length of the bus
20 )m/sec.

Quantitative Aptitude Test | 361


P×R×T 44. (B)
33. (B) S.I. =
100 33
6200 × R × 4 3 4 10 30 136 685 4116
9176 – 6200 =
100
or R = 12 ×1+1 ×2+2 ×3+3 ×4+4 ×5+5 ×6+6

New rate = 12 + 3 = 15 ∴ The wrong number is 30.


6200 × 15 × 4
New amount = 6200 + 45. (A) Let the amount be Rs. x.
100
x x
= Rs. 9920 ∴ – = 60
12 20
34. (C) Volume of the utensil
5x – 3x
(
=
2
3 )
π × 12 × 12 × 12 cm3 or
60
= 60

= 1152 π cm3 60 × 60
or x =
2
( 3 3
Volume of 1 bottle = π × × π4 cm3
2 2 ) = Rs. 1800
= 9 π cm3 46. (D) Let the sum borrowed be Rs. x.
1152 π x × 6 × 3 x × 8 × 2 x + 12 × 3
∴ Number of bottle = = 128 ∴ S.I. = + +
9π 100 100 100
35. (B) Let the length of diagonal be 2x cm 18x 16x 36x
∴ The length of 2nd diagonal = x cm or 10080 = + +
100 100 100
1 70x
∴ × x × 2x = 169 cm or 10080 =
2 100
or x = 13 cm 10080 × 100
∴ Length of 1st and 2nd diagonal is 26 cm and 13 ∴ x = = Rs. 14400
70
cm respectively. 47. (B) Let the present age of Omkar and Ram Kishan be
36. (D) 28000 – 21000 = 7000 4x years and 5x years respectively.
37. (B) 25000 – 15000 = 10000 4x – 7 3
15 – 15 0 ∴ =
38. (A) × 100 = × 100 = 0% 5x –7 4
15 15
or 16x – 28 = 15x – 21
39. (B) January = 27000 – 21000 = 6000
April = 36000 – 20000 = 16000 or x = 7
∴ Increase = 16000 – 6000 = 10000 ∴ Present age of Ram Kishan
40. (C) 36000 – 17000 = 19000 = May = 5 × 7 = 35 years
41. (D) Let the total marks obtained by the whole team 48. (D) Let the breadth of the plot be x m
be x
x + 92 – 85 ∴ Ist length = 4x m
then = 84
8 1
∴ × 4x × x = 1200
∴ x = 84 × 8 – 7 = 665 3
42. (C) Let the required number be x 1200 × 3
or x2 = = 900
then, 31x – 13x = 342 4
or 18x = 342 ∴ x = 30
or x = 19 ∴ Length of the plot = 4 × 30 = 120 metres
43. (B) Praveen spends 30% of his income that is Rs. 49. (D) Discount on 1st saree = 15% of 1000 = Rs. 150
3000 on petrol. ∴ Cost price of the 1st saree
Hence, Praveen’s income is 100%
= 1000 – 150 = Rs. 850
= Rs. 10000
Money left after he has spend it on petrol ∴ Cost price of 2nd saree = Rs. 850
= 10000 – 3000 = Rs. 7000 Profit on 1st saree = 20% of 850 = Rs. 170
∴ Money spent on house rent and profit on 2nd saree = 15% of 850
1 = Rs. 127·50
= th of 7000
4 Total profit on two sarees = 170 + 127·50
= Rs. 1750 = Rs. 297·50

Quantitative Aptitude Test | 362


50. (E) Cost price of one table = Rs. 4000 71. (B)
(from statement P) 27
2 765
∴ Cost price of six tables = 4000 × 6 = Rs. 24000
4
∴ Cost price of one chair = 4000 – 1800
47 365
= Rs. 2200 329
∴ Cost price of four chairs = 2200 × 4 = Rs. 8800 36
∴ Total cost of six tables and four chairs
∴ No. of chairs to be removed = 36.
= 24000 + 8800 72. (A) No. of buckets each of capacity 13·5 litre
= Rs. 32800 required to fill the tank = 20
Percentage profit on each is 25 (from statement Q) ∴ No. of buckets each of capacity 1 litre required to
fill the tank = 20 × 13·5
32800 × 25
∴ Total profit = ∴ No. of buckets each of capacity 9 litre required to
100
20 × 13·5
= Rs. 8200 fill the tank = = 30
9
∴ To find the answer both the statements P and Q 73. (C) Total work of all the three machines for 1 hour
are needed. 1 1 1 37
51. (B) 52. (D) 53. (C) 54. (A) 55. (C) = + + =
8 10 12 120
56. (B) 57. (D) 58. (D) 59. (A) 60. (B) ∴ Total work of all the three machines for 2 hours
61. (B) 37 37
= 2× =
2 1 2 120 60
62. (D) 12 – 5 ÷ 6 = ?
3 3 3 37 23
Remaining work = 1 – =
38 16 20 60 60
? = – ÷
3 3 3 Total work B and C for 1 hour
38 16 3 1 1 11
= – × = + =
3 3 20 10 12 60
38 4 190 – 12 ∴ Time taken by B and C machines to complete the
= – =
3 5 15 23 11 23
remaining work = ÷ = hours
178 13 60 60 11
= = 11
15 15 = 2 hours (approx.)
63. (B) 64. (B) 65. (C) 74. (A) Let the total number of votes polled be x.
24 × 3 – 18 × 3 ∴ No. of votes obtained by winner
66. (A) = ?
8 × 7 – 23·5 × 2
60x 3x
3(24 – 18) 18 = =
? = = 100 5
56 – 47 9
∴ No. of votes obtained by other
= 2
3x 2x
1 1 3 2 = x– =
67. (D) 4 + 2 ÷ × 4 = ? 5 5
2 4 4 3
3x 2x
9 9 4 14 ∴ – = 144
? = × × × 5 5
2 4 3 3
x
9 9 + 28 37 or = 144
= + 14 = = 5
2 2 2
∴ x = 144 × 5
1
= 18 = 720
2
68. (D) 75. (C) Let the number of supervisor be x and the number
of labourers be y.
69. (C) ⎯ ? + 720 ÷ 12
√ = 8 × 10 – 16
x + 2y = 220
⎯ ? + 60
√ = 80 – 16 and x = 20
⎯ ? + 60
√ = 64 On solving we get y = 100
⎯?
√= 64 – 60 = 4 ∴ Total number of workmen in the factory
? = (4) 2 = 16 = x + y = 20 + 100
70. (D) 286·3154 + 87·596 + 673·49 = 1047·4014. = 120.

Quantitative Aptitude Test | 363


Miscellaneous Exercise – VIII
01. The sum of two digits of a num- 10. A can do a piece of work in 20 (C) Binding, Canvassing, Designing
ber is 9. If 9 is subtracted from days, B can do it in 25 days. etc. 30%
the number, then the digits are They work together for 5 days (D) Miscellaneous 10%
reversed. What is the number ? and then B goes away. In how
(E) Royalty 15%.
(A) 36 (B) 45 many days will A finish the
(C) 54 (D) 72 work ?
1 D E
02. An article is sold for Rs. 240. If (A) 17 days (B) 11 days
2 10% 15%
the profit is one-fourth of the cost
price, what is the cost price ? 4
(C) 8 days (D) 10 days
7 A
(A) Rs. 180 (B) Rs. 164 C 20%
(C) Rs. 192 (D) Rs. 196 11. A lead pipe is 35 cm long its 30%
external diameter is 2·4 cm and
03. Lemons are bought at 5 for a its thickness is 2 mm. If 1 cubic B
rupee and sold at 8 for three cm of lead weighs 5 gms. The 25%
rupees. What is gain or loss per weight of the pipe is—
cent in the transaction ?
(A) 220 gms (B) 242 gms
1 1
(A) 37 % (B) 57 % (C) 420 gms (D) 484 gms Look at the diagram carefully
2 2
12. A company declares a dividend and answer questions.
1
(C) 6% (D) 87 % of 12% on Rs. 100 shares. A 16. The production of A during the
2
man buys such shares and gets given period is less than that of
04. A can run 1 km in 3 min. 10 sec. 15% on his investment. At what B by—
and B in 3 min. 20 sec. By what price he bought the shares ? (A) 6·3% (B) 5·9%
distance can A beat B ? (A) Rs. 125 (B) Rs. 85 (C) 7·2% (D) 4·8%
(A) 36 metres (B) 50 metres (C) Rs. 80 (D) Rs. 76
(C) 40 metres (D) 60 metres 17. What is the angle of pie-diagram
⎛ ·86 × ·86 × ·86 – ·14 ⎞ showing the expenditure incurred
05. The average score of a cricketer ⎜ × ·14 × ·14⎟
for 10 matches is 38·9 runs. If the 13. ⎜⎜ ·86 × ·86 + ·86 × ·14⎟⎟ = ?
on paying the royalty ?
average for the first 6 matches is ⎜⎝ + ·14 × ·14⎟⎠
(A) 24° (B) 48°
41, what is the average for last 4 (C) 54° (D) 15°
matches ? (A) ·72 (B) ·1
18. The marked price of a book is
(A) 36·25 (B) 34·25 (C) ·32 (D) ·26 20% more than the C.P. If the
(C) 35·75 (D) 32·85 14. The cost making an article is marked price of the book be Rs.
1 divided between materials, labour 30. What is the cost of paper used
06. =? and overheads in the ratio 3 : 4 : in a single copy of the book ?
⎯3
√ 1. If the materials cost Rs. 11·25. (A) Rs. 6 (B) Rs. 25
(A) 0·632 (B) ·517 The cost of article is— (C) Rs. 4·50 (D) Rs. 6·50
(C) ·527 (D) ·577 (A) Rs. 33·75 (B) Rs. 45
19. Which two expenditures together
(C) Rs. 9·80 (D) Rs. 30
⎯⎯⎯⎯
√ 1008 will form an angle of 108° at the
07. =?
15. A man can row 5 km/hr in still centre of the pie-diagram ?
⎯7

water. If the river is running at 1 (A) A & E (B) B & E
(A) 12·75 (B) 11·68 km/hr, it takes him 1 hour to row (C) A & D (D) D & E
(C) 12 (D) 13·26 to a place and back. How far is
— the place ? 20. If the difference between two
08. 3· 57 in fractional form is— expenditures be represented by
(A) 2·5 km (B) 2·4 km
357 354 18° in the pie-diagram these
(A) (B) (C) 3 km (D) 3·6 km
99 99 expenditures are—
357 354 Directions—(Q. 16 to 19) The (A) B & E
(C) (D) following pie-diagram shows the
90 90 (B) A & C
expenditure incurred on the prepa-
09. The greatest number of 4 digits, ration of a book by a publisher under (C) B & D
which is a perfect square, is— various heads— (D) None of these
(A) 9981 (B) 9891 (A) Paper 20% Directions—Examine the fol-
(C) 9902 (D) 9801 (B) Printing 35% lowing graph carefully and answer

Quantitative Aptitude Test | 364


questions 21 and 22 based on the 27. A park square in shape was a 3 34. The length of a rectangular plot
information given in it. metre wide road inside it running is sixty per cent more than its
along its sides. The area occu- breadth. If the difference bet-
40 40 pied by the road in 1764 square ween the length and breadth of
1985-86
35 1986-87 metres what is the perimeter that rectangle is 24 cms, what is
32 32 along the outer edge of the road ? the area of that rectangle ?
30 1987-88
27 (A) 576 metres (A) 2400 sq. cm.
25 24
22 (B) 640 metres (B) 2560 sq. cm.
20 20 20
16 (C) 600 metres (C) 2480 sq. cm.
15 15
12 12 12 (D) Data inadequate (D) Data inadequate
10 10 10
28. Four of five parts lettered (A), 35. When the numerator of a certain
5 (B), (C), (D) and (E) are exactly fraction is increased by 2 and
0 equal. Which of the parts is not denominator by 1, its value
A B C D E equal to the other four ? The 1
letter of that part is the answer— changes to , but when the
21. What is the average production 2
(in lakh bales) of these states 2 numerator is increased by 3 and
(A) 4 of 140 + 334 =
during 1987-88 ? 5 denominator by 5, its value then
(A) 15·6 (B) 24·8 (B) 95% of 500 + 50% of 900 = 2
equals to . What is the original
(C) 1860 + 380 – 1290 = 5
(C) 20·4 (D) 26·5
fraction ?
1
22. How many states showing below (D) 2 of 330 + 180 = 1 1
average production in 1985-86 3 (A) (B)
6 4
showed above average produc- (E) 6·5 × 120 + 2·5 × 80 – 30
tion in 1986-87 ? 1 1
29. In the following number series a (C) (D)
3 5
(A) 4 (B) 3 wrong number is given. Find out
(C) 2 (D) 1 that wrong number. 36. How many different combina-
10, 11, 24, 75, 303, 1525, 9156 tions of four letters beginnings
23. Which of the following is a true with E can be formed from the
(A) 24 (B) 1525
statements ? letters of the word equation ?
(C) 75 (D) 303
(A) State (A) has produced (A) 630 (B) 360
maximum cotton during the 30. Three-fifth of two-third of one-
(C) 210 (D) 336
given period sixth of a number is 40, what is
(B) There was no consistent thirty per cent of that number ? 37. What will be the ratio of simple
progress shown by C (A) 180 (B) 240 interest earned by a certain
(C) 480 (D) 160 amount at the same rate of
(C) States A and B showed a interest for 6 years and that for 9
steady progress in the production 31. What approximate value should years ?
of cotton during the given period come in place of the question (A) 2 : 3
(D) There is no downfall in the mark (?) in the following equa-
tion ? (B) 1 : 4
production shown by State B
137% of 6984 + 2·35 of 140·79 (C) 1 : 3
24. The production of A during the
+ 1138·65 = ? (D) Data inadequate
given period is less than that of
B by— (A) 10800 (B) 11000 3
(C) 9800 (D) 11500 38. The product of two fractions is
(A) 6·3% (B) 5·9% 5
32. What should come in place of 5
(C) 7·2% (D) 4·8% and the quotient is . If the
question mark (?) in the follow- 12
25. Admission to a course is ing equation ? 1
increased by 15% every year. If denominator of one is of the
5·8 × 2·5 + 0·6 × 6·75 + 139·25 3
the number of students in this other’s numerator, which of the
=?
course in 1988 is 1600. What is following is that pair of frac-
the expected number of students (A) 157·30 (B) 160·30
tions ?
in 1990 ? (C) 157·50 (D) 158·40
1 9
(A) 2080 (B) 2116 33. If 20 men complete a work in 56 (A) ,
3 5
(C) 2356 (D) 1960 days by working 10 hours a day.
1 6
How many days will 40 men (B) ,
26. The surface area of a cube is 216 take to complete the same work 2 7
cm2. What is its volume ? by working 8 hours a day ? 1 6
(C) ,
(A) 1296 cm3 (B) 648 cm3 (A) 30 (B) 25 2 5
(C) 864 cm3 (D) 216 cm3 (C) 48 (D) 35 (D) Data inadequate

Quantitative Aptitude Test | 365


39. Four of the five parts lettered (A), 45. P can complete a work in 15 ence between the expenditure of
(B), (C), (D) and (E) in the days. Q can complete the same these two years ?
following equations are exactly work in 10 days. P started work- (A) 1 lakh
equal. Which of the part is not ing and after 5 days Q joined (B) 2 lakhs
equal to the other four ? The him. How many days more will (C) 3 lakhs
letter of that part is the answer— they take to complete the re-
maining work ? (D) None of these
(A) 9387 – 7284 + 1125 =
(B) 286 × 18 – 1680 – 240 = (A) 1 day (B) 2 days 50. If the income in 1993 was Rs. 14
lakhs. What was the expenditure
(C) 1640 × 8·5 – 90 × 140 + (C) 3 days (D) 4 days
in that year ?
1888 = 46. The sum of two numbers is three (A) 9·8 lakhs
(D) 1864 + 1058 + 306 = times the smaller number and the (B) 9·1 lakhs
(E) 150% of 10801510 = 108 = difference between them is equal
(C) 8·4 lakhs
to the smaller number. What is
40. At what price should shopkeeper difinitely the bigger number ? (D) Data inadequate
mark a radio that costs him Rs.
(A) 28 51. What was the percentage increase
1200 in order that he may offer a
(B) 36 in income from 1992 to 1993 ?
discount of 20% on the marked
price and still make a profit of (A) 30
(C) 39
25% ? (B) 75
(D) Data inadequate
(A) Rs. 1675 (B) Rs. 1875 (C) 70
Directions—(Q. 47–51) Study (D) Data inadequate
(C) Rs. 2025 (D) Rs. 1900 the following graph carefully and
41. A box contains two white balls, answer the questions given below— Directions—(Q. 52 to 62) Find
three black balls and four red the approximate value which should
balls. In how many ways can Ratio of Expenditure to Income replace the question mark (?) in each
three balls be drawn from the of a Company over the years of the following question.
box of atleast one black ball is to 0.8 52. 6·595 × 1084 + 2568·34 –
be included in the draw ? 1708·34 = ?
(A) 64 (B) 32 0.7 (A) 6000 (B) 12000
(C) 48 (D) 96 0.6 (C) 10000 (D) 8000
42. In the following number series a 0.5 53. Four-fifth of three-eight of a
wrong number is given. Find out number is 24. What is 250 per
that wrong number. 0.4 cent of that number ?
11 207 36 157 57 138 74 (A) 100 (B) 160
0.3
(A) 138 (B) 157 (C) 120 (D) 200
0.2
(C) 36 (D) 207 54. The average of 17 numbers is
0.1 45. The average of first 9 of
43. Jayesh purchased a machine for these numbers is 51 and the last
Rs. 80000 and sent Rs. 5000 to 0 9 of these numbers is 36. What is
repair and Rs. 1000 on transport the ninth number ?
and sold it with 25 per cent 47. If the total expenditure in 1994
and 1995 together was 28 lakhs. (A) 14
profit. At what price did he sell
the machine ? What was the total income in (B) 16
these two years ? (C) 22
(A) Rs. 117500
(A) 42 lakhs (D) None of these
(B) Rs. 107500
(C) Rs. 106250 (B) 36 lakhs 55. Four of five parts lettered (A),
(D) Rs. 105100 (C) 38 lakhs (B), (C), (D) and (E) are exactly
(D) Data inadequate equal. Which of the parts is not
44. The sum of the digits of a two equal to the other four ? The
digit number is one-fourth of the 48. In which of the following years letter of that part is the answer.
number and the difference bet- was the expenditure minimum (A) 16·80 × 4·50 + 44 =
ween the digits is one-third of with reference to the income in
the sum of the digits. What is the given years ? (B) 1600 ÷ 40 + 16 × 2·5 =
number ? (A) 1996 (B) 1997 (C) 5·5 × 8·4 + 34·6 =
(A) 36 (C) 1992 (D) 1995 (D) 1620 ÷ 20 – 1 =
(B) 48 (E) 1856·95 – 1680 – 65 – 96·3 =
49. If the expenditure in 1995 was
(C) 54 26 lakhs and income in 1996 was 56. Arun borrowed a sum of money
(D) Data inadequate 50 lakhs. What was the differ- from Jayant at the rate of 8

Quantitative Aptitude Test | 366


p.c.p.a. simple interest of the mark (?) in the following ques- Domestic Consumption
first four years. 10 p.c.p.a. for tion ? Exports
the next six years and 12 p.c.p.a. 159% of 6531·8 + 5·5 × 1015·2 =
for the period beyond ten years. 40
? = 5964·9
If he pays a total of Rs. 12160 as (A) 10000 (B) 10900 35
interest only at the end of 15
(C) 11000 (D) 10600 30
years, how much money did he 25
borrow ? Directions—(Q. 62 to 66) In 25 22·5
(A) Rs. 8000 each of the following question a
20 20
number series is given. After the 20
(B) Rs. 10000 14
series below it a number is given 15
(C) Rs. 12000 followed by a, b, c, d and e. You 10 12·5
have to complete the series starting 7·5
(D) Rs. 9000 5
with the number given following the
57. Four of the five parts lettered sequence of the given series. Then 0
(A), (B), (C), (D) and (E) are answer the question given below it. 1988 1989 1990 1991 1992 1993 1994
exactly equal. Which of the parts 62. 11 15 38 126 Years
is not equal to the other four ? 7 a b c d e
The letter of the part is the 67. In which of the following years
Which of the following will
answer. was the total of the exports and
come in place of (c) ?
(a + b) (a – 2b) a2 – b (a + 2b) domestic consumption highest
= (A) 102 (B) 30 among the given years ?
A B (C) 2140 (D) 80 (A) 1991 (B) 1993
(a + b) (a – b) 63. 2 3 8 27 (C) 1994 (D) 1992
a2 b2 – ab – 3b2 + ab 5 (a) (b) (c) (d) (e) 68. The difference between the
= =
C D domestic consumption between
Which of the following will
come in place of (e) ? 1990 and 1991 was exactly equal
(a – b)2 b (a + 3b)
= to the difference in exports bet-
E (A) 184 (B) 6 ween which of the following
(C) 925 (D) 45 pairs of years ?
58. The average age of 24 students
and the class teacher is 16 years. (E) 14 (A) 1991 and 1992
If the class teacher’s age is (B) 1989 and 1991
64. 2 3 9 40·5
excluded the average reduces by (C) 1988 and 1990
one year. What is the age of the 4 (a) (b) (c) (d) (e) (D) 1991 and 1993
class teacher ? Which of the following will
69. If there was 40 per cent increase
come in place of (b) ?
(A) 50 years in the domestic consumption
(A) 486 (B) 81 from 1994 to 1995. What would
(B) 45 years
(C) 3645 (D) 18 have been the difference between
(C) 40 years the domestic consumption and
(E) 6
(D) Data inadequate exports in 1995 ?
65. 12 28 64 140 (A) 5000 tonnes
59. What should come in place of 37 (a) (b) (c) (d) (e) (B) 7000 tonnes
the question mark (?) in the
following equation ? Which of the following will (C) 8500 tonnes
come in place of (e) ? (D) Data inadequate
5679 + 1438 – 2015 = ?
(A) 1412 (B) 164 70. In which of the following years
(A) 5192 (B) 5012 was the percentage fluctuation
(C) 696 (D) 78
(C) 5102 (D) 5002 (increase/decrease) in exports
(E) 340
the maximum from the previous
60. The ratio between the length and 66. 5 12 60 340 year ?
breadth of rectangular plot is (A) 1989 (B) 1991
7 (a) (b) (c) (d) (e)
7 : 5. If the perimeter of the plot
Which of the following will (C) 1994 (D) 1992
is 144 metres. What is its area ?
come in place of (d) ? 71. What was the difference between
(A) 1320 sq. metres
(A) 172 (B) 3222 the average domestic consump-
(B) 1260 sq. metres tion and the average exports of
(C) 1012 (D) 20164
(C) 1280 sq. metres the given years ?
(E) 28 (A) 2000 tonnes
(D) 1380 sq. metres
Directions—(Q. 67 to 71) Study (B) 3000 tonnes
61. What approximate value should the following graph carefully and (C) 2500 tonnes
come in place of the question answer the questions given above— (D) None of these

Quantitative Aptitude Test | 367


Directions—(Q. 72–75) Study the following table carefully and answer one-fourth of 176. What is the
the question given below— middle number ?
Subjects (A) 8
Student Sub. History Science Geography English Hindi Maths Total (B) 10
Total Marks 150 200 150 200 100 200 1000 (C) 6
(D) Data inadequate
A 75 110 90 140 75 170 660
B 105 130 75 130 85 140 660 82. The ratio between the per cent
C 95 105 80 150 90 160 680 age of P and Q is 5 : 8. After
D 85 115 95 125 65 135 620 four years the ratio between their
age will be 2 : 3. What is Q’s age
E 115 135 110 145 70 125 700
at present ?
F 120 160 96 110 55 145 686
(A) 36 years
72. What was the difference between 38 × 12 – 39 × 8 (B) 20 years
the percentage of marks obtained =
C (C) 24 years
by student B in History and the 1648 – 938 – 566 (D) None of these
percentage of marks obtained by =
D
student C in Hindi ? 83. Two-fifth of thirty per cent of
(A) 25 (B) 20 1 one-fourth of a number is 15.
6 of 140 – 2·5 × 306·4
(C) 35 (D) 30 2 What is 20 per cent of that
=
E number ?
73. The marks obtained by student A
in Maths were how many times 78. Assuming A, B and C are diffe- (A) 90 (B) 150
the percentage of marks obtained rent single digit numerical values (C) 100 (D) 120
by student F in Science ? other than. What is already used 84. The price of four tables and
(A) 2·5 (B) 4·125 in following equation ? What seven chairs is Rs. 12090.
(C) 1·125 (D) 21·25 number C definitely cannot be ? Approximately what will be the
74. What was the difference between 8A2 + 3B5 + C4 = 1271 price of such twelve tables and
the percentage of marks obtained (A) 7 twenty one such chairs ?
by student C in English and (B) 9 (A) Rs. 32000
average percentage of marks of (B) Rs. 46000
(C) Either 7 or 9
all the six subjects ? (C) Rs. 38000
1 (D) 6
(A) 82 (B) 38 (D) Rs. 36000
3 79. The difference between the digits
(C) 7 (D) 14 85. If the price of 253 pencils is
of a two digit number is one-
Rs. 4263·05. What will be the
75. In how many of the given sub- ninth of the difference between
approximate value of 39 such
jects did students D get marks the original number and the num-
pencils ?
more than seventy per cent ? ber obtained by interchanging
the position of the digits. What is (A) Rs. 650 (B) Rs. 550
(A) None (B) One
definitely the sum of the digits of (C) Rs. 450 (D) Rs. 700
(C) Two (D) Three
that number ? 86. The perimeter of a rectangle is
76. Approximately what was the
(A) 5 equal to the perimeter of a right
average percentage of marks
(B) 14 angle triangle of height 12 cm. If
obtained by the six students in
the base of the triangle is equal
English ? (C) 12 to the breadth of the rectangle.
(A) 67 What is the length of the
(D) Data inadequate
(B) 72 rectangle ?
(C) 80 80. When 35 per cent of a number is (A) 18 cms
(D) Data inadequate added to another number the
second number increases by its (B) 24 cms
77. Four of the five parts lettered 20 per cent. What is the ratio (C) 22 cms
(A), (B), (C), (D) and (E) are between the second number and (D) Data inadequate
exactly equal. Which of the parts the first number ?
is not equal to the other four ? 87. What should come in place of
The letter of that part is the (A) 4 : 7 the question mark (?) in the
answer. (B) 7 : 4 following equation ?
1 2
40% of 160 + of 240 (C) 8 : 5 18 of 150·8 + ? = 8697·32 –
3 5
(D) Data inadequate 3058·16
A
120% of 1200 81. The sum of three consecutive (A) 2764·44
=
B even numbers is 14 less than (B) 2864·34

Quantitative Aptitude Test | 368


(C) 1864·44 answer the question given below. (in 96. What was the percentage drop in
(D) None of these lakh tonnes) sales from 1992 to 1993 ?
Amount of production and sales (A) 1 (B) 10
88. The squared value of the dia-
gonal of a rectangle (64 + x 2 ) sq. by a company over the years (C) 20 (D) 15
cm. where x is less than 8 cm. Production 97. The ratio between the ages of
18 18
What is the length of that Sales 17 Rati and Madhuri is 4 : 5 and the
16
rectangle ? 16 ratio between the ages of
14 Madhuri and Kajri is 5 : 6. If the
(A) 6 cms 14
12 12 sum of their ages is 60 years,
(B) 10 cms 12 11
10 10 what is the age of Kajri ?
(C) 8 cms 9 9
8 8 (A) 20 years (B) 16 years
(D) Data inadequate 7
6 (C) 28 years (D) 24 years
89. Jaydeep purchased 25 kg of rice
at the rate of Rs. 16·50 per kg
4 1 1
98. of boys and of girls of a class
and 35 kg of rice at the rate of 2 5 4
Rs. 24·50 per kg. He mixed the 0 participated in swimming while
two and sold the mixture. 1991 1992 1993 1994 1995 1996 2 3
of boys and of girls partici-
Approximately at what price per 3 5
92. What was the difference between
kg did he sell the mixture to pated in sports. If total number
the sales of 1992 and 1995 ?
make 25 per cent profit ? of students in the class was 65,
(A) 10 lakh tonnes how many girls participated in
(A) Rs. 26·50 (B) Rs. 27·50 (B) 50 thousand tonnes sports ?
(C) Rs. 28·50 (D) Rs. 30·00 (C) 1 lakh tonnes (A) 12
90. In the following number series a (D) 75 thousand tonnes (B) 4
wrong number is given. Find out
93. The total sales of 1991 and 1992 (C) 16
that wrong number.
together were approximately (D) Data inadequate
8, 14, 26, 48, 98, 194, 386 what percentage of the sales in
(A) 194 (B) 98 1994 ? 1
99. If of a number is 12. What
(C) 14 (D) 48 14
(A) 140 (B) 120
will be 150% of the same
91. What approximate value should (C) 160 (D) 130 number ?
come in place of the question 94. What was the approximate per- (A) 180
mark (?) in the following equa- centage increase in production
tion ? (B) 252
from 1993 to 1994 ?
3 (C) 420
3 of 157·85 + 39% of 1847 = ? (A) 6 (B) 45
5 (C) 35 (D) 40 (D) Data inadequate
– 447·30
95. The percentage of sales to pro- 100. What should be added to 20272
(A) 1200 (B) 1500 so that the result may be divisible
duction was maximum in which
(C) 1600 (D) 1800 of the following years ? by 13 ?
Directions—(Q. 92–96) Study (A) 1992 (B) 1994 (A) 8 (B) 9
the following graph carefully and (C) 1996 (D) 1991 (C) 11 (D) 13

Answers with Hints


01. (C) Let the ten’s digit be x and the unit digit by y. 3. (D) Suppose (8 × 5) i.e., 40 lemons be bought.
Then, x + y = 9 and 10x + y – 9 = 10y + x Then, C.P. = Rs. 8 and S.P. = Rs.
3
8 ( )
× 40
Solving, x + y = 9 and x – y = 1
= Rs. 15
We get x = 5 and y = 4
The no. 10x + y = 10 × 5 + 4 = 54.
∴ Gain % =
7
8 (
× 100 % )
1
2. (C) Let the C.P. be Rs. x = 87 %
2
1 4. (B) A beats B by 10 sec.
Then, x + x = 240
4
Distance covered by B in 10 sec. (1000
200
× 10)
or x = (2405 × 4) = Rs. 192 = 50 metres

Quantitative Aptitude Test | 369


5. (B) (6 × 42) + 4 × x = (38·9 × 10) 18. (B) Let the C.P. of the book be Rs. x.
or x = 34·25. Then 120 : 100 = 30 : x
1 1 ⎯√ 3 = √
⎯ 3 = 1·732 = 0·577 100 × 30
6. (D) = × ∴ x = = Rs. 25
120
⎯3
√ ⎯3 3
⎯3 √
√ 3
Thus, C.P. of the book = Rs. 25

7. (C)
⎯⎯⎯⎯


1008
7
=
⎯⎯⎯7 ) = ⎯√⎯⎯144 = 12.
√ ( 1008 ∴ Cost of paper used = (20% of Rs. 25) = Rs. 5.

19. (C) 108° = ( 108


360
× 100 %)
57 354
8. (B) 3· 57 = 3 =
99 99 = 30%
9. (D) Clearly [(99)2 – 9999] = 198 So, A and D together will form an angle of 108°.
∴ Required number = (9999 – 198) = 9801
20. (D) 18° = ( 18
)
× 100 %
10. (B) Work done in 5 days = 5 ( )
1
+
20 25
1
=
9
20 = 5%.
36

Remaining work = 1 – ( ) 9
20
=
11
20
21. (B) Average production in 1987-88
40 + 32 + 22 + 20 + 10
= = 24·8 lakh bales.
This work will be finished by A in( ) 11
20
× 20 5
22. (D) Average production in 1985-86 is 15·6.
= 11 days Average production in 1986-87 is 20·4.
11. (B) Volume of lead = πh (R2 – 12 ) States showing below average production in 1986-87
22 are A, C and E.
= × 35 × [(1·2)2 – (1) 2 ]
7 States showing above average production in 1986-87
= 48·4 cu. cm. are A and B.
∴ Weight of lead = (48·4 × 5) gms = 242 gms So, the required type of states is A only.
12. (C) Let the price of Rs. 100 share be Rs. x. 23. (D) Production by A is 79 lakh bales while
production by B during this period is 84 lakh bales.
Then income on Rs. x = Rs. 12. So (A) is false. Statements (B) and (C) are clearly
Income on an investment of Rs. 100 false.

= ( 12
x × 100 %) Also, (D) is clearly true.


1200
24. (B) Required percentage = ( 5
84 )
× 100 %
x = 15 = 5·9%.
or x = 80 25. (D) 6a2 = 226
a3 – b3
13. (A) Given expression = 2 = (a – b) or a2 = 36
(a + ab + b2)
or a = 6
= (0·86 – 0·14) = 0·72
∴ Volume of the cube = (6) 3 cm3 = 216 cm3
14. (D) Ratio of materials and total cost = 3 : 8
26. (B) Required number of students
∴ 3 : 8 :: 11·25 : x
8 × 11·25
or x =
3
= 1600 × 1 + ( )
15 2
100

= (1600 × × ) = 2116
23 23
= Rs. 30
20 20
15. (B) Man’s rate down stream = 6 km/hr
27. (C)
Man’s rate upstream = 4 km/hr
Let the distance be x km.
x x (x – 6) x
Then + = 100 x = 2·4 km
6 4

16. (B) Required percentage =


5
84 (
× 100 % ) (x – 6)
x
= 5·9%.
∴ 2 × (x × 3) + 2 × (x – 6) × 3 = 1764
17. (C) Required angle = ( 15
100 )
× 360° = 54°. or 6x + 6x – 36 = 1764

Quantitative Aptitude Test | 370


or 12x = 1800 3x
or = 24
1800 5
∴ x = = 150
12 5
∴ x = 24 × = 40 cm
∴ Perimeter = 4 × x 3
= 4 × 150 8x 8 × 40
and = = 64 cm
= 600 5 5
2 ∴ Area = 64 × 40 = 2560 cm2
28. (B) (A) = 4 of 140 = 334 x
5 35. (D) Let the fraction be y.
22
= × 140 + 334 = 950 x+2
5 1
∴ =
(B) = 95% of 500 + 50% of 900 y+1 2
95 × 500 50 + 900 or 2x + 4 = y + 1
= +
100 100 or 2x – y = 3 …(i)
= 475 + 450 = 925 x+3 2
and =
(C) = 1860 + 380 – 1290 = 950 y+5 5
1 or 5x + 15 = 2y + 10
(D) = 2 of 330 + 180
3 or 5x – 2y = –5 …(ii)
7
= × 330 + 180 On solving equations (i) and (ii), we get
3
x = 1
= 770 + 180 = 950
and y = 5
(E) = 6·5 × 120 + 2·5 × 80 – 30
= 780 + 200 – 30 = 950 1
∴ Reqd. fraction is = .
5
29. (D)
36. (C) Reqd. number of combinations
7
= 7p3 = = 7 × 6 × 5 = 210
∴ The wrong number is 303. 4
30. (A) Let the number be x. p ×6×r
37. (A) S.I. for years =
3 2 1 100
∴ of of of x = 40
5 3 6 p ×9×r
and S.I. for a years =
or x = 40 × 15 = 600 100
30 p ×6×r 100
∴ 30% of 600 = × 600 = 180 ∴ Reqd. ratio = × =2:3
100 100 p ×9×r
31. (B) ? = 137% of 6984 + 2·35 of 140·79 + 1138·65 38. (C)
= 9568·08 + 330·8565 + 1138·65 39. (D) (A) = 9387 – 7284 + 1125 = 3228
= 11037·5865 = 11000 (Approx.) (B) = 286 × 18 – 1680 – 240 = 3228
32. (C) ? = 5·8 × 2·5 + 0·6 × 6·75 + 139·25 (C) = 1640 × 8·5 – 90 × 140 + 1888 = 3228
= 14·5 + 4·05 + 139·25 = 157·5 (D) = 1864 + 1058 + 306 = 3228
(E) = 150% of 1080 + 1510 + 108 = 3238
33. (D) If 20 men working 10 hours a day complete a
work in 56 days. 40. (B) C.P. of the radio = Rs. 1200
∴ 1 man working 1 hour a day complete a work and Profit = 25%
= 56 × 20 × 10
∴ 40 men working 8 hours a day complete a work
S.P. = 1200 (
100 + 25
100 )
56 × 20 × 10 = Rs. 1500
= = 35 days
40 × 8 If M.P. be Rs. 100 then, S.P. after discount
34. (B) Let the breadth of the plot be x metres. = 100 – 20 = Rs. 80
(100 + 60) 8 If Rs. 80 is S.P. then M.P. = Rs. 100
∴ Length = x × = x metres
100 5 100 × 1500
∴ If Rs. 1500 S.P. the, M.P. =
8x 80
∴ – x = 24
5 = Rs. 1875

Quantitative Aptitude Test | 371


41. (A) The reqd. number of ways 48. (C) The expenditure was minimum with the income
= 3 C 1 × 6C 2 + 3 C 2 × 6C 1 + 3 C 3 in 1992.
= 3 × 15 + 3 × 6 + 1 49. (D) Expenditure in 1995 = 26 lakhs
= 45 + 18 + 1 = 64 Expenditure in 1996 = (50 × 0·55) lakhs
= 27·5 lakhs
42. (D)
∴ Required difference = (27·5 – 26) lakhs
= 1·5 lakhs
50. (A) Income in 1993 = 14 lakhs
Therefore, the number 207 is wrong and it should be Expenditure
replaced by 180. = 0·7
Income
43. (B) Total cost of machine ∴ Required expenditure = 14 × 0·7 = 9·8 lakhs
= Rs. 80000 + Rs. 5000 + Rs. 1000 51. (D) Since in the graph only ratio of expenditure to
= Rs. 86000 income is given so the data is inadequate to answer
∴ S.P. of machine the question.
= Rs. 86000 × 1·25 52. (D) 6·595 × 1084 + 2568·34 – 1708·34 = ?
= Rs. 107500 7148·98 + 2568·34 – 1708·34 = ?
44. (D) Let the number be 10x + y. ? = 7148·98 + 2568·34 – 1708·34
∴ According to question, ? = 7150 + 2568 – 1708
(10x + y) 10x – y = 9718 – 1708 = 8010 = 8000
(10x + y) = = 53. (D) Let the number be x.
4 3
30x + 3y + 40x – 4y 4 3
or 10x + y = of of x = 24
12 5 8
70x – y 4 3
10x + y = or × × x = 24
12 5 8
120x + 12y = 70x – y or 3x = 24 × 10
50x + 13y = 0 24 × 10
x = = 80
3
1
45. (D) P’s one day work = ∴ 250% of x = 250% of 80
15
1 250 × 80
Q’s one day work = = = 200
10 100
5 1 54. (D) Total sum of 17 numbers = 45 × 17 = 765
∴ P’s five day work = =
15 3 Total sum of Ist 9 numbers = 9 × 51 = 459
1 2 Total sum of last 9 numbers = 9 × 36 = 324
∴ Remaining work = 1 – =
3 3 ∴ Required number = (459 + 324) – 765
1 1 = 783 – 765 = 18
∴ (P + Q)’s one day work = +
15 10 55. (C) (A) 16·80 × 4·50 + 4·4 = 80
2+3 5 1 (B) 1600 ÷ 40 + 16 × 2·5 = 80
= = =
30 30 6 (C) 5·5 × 8·4 + 34·6 = 80·8
∴ Remaining work can be completed by both in (D) 16·20 ÷ 20 – 1 = 80
2 6
× = 4 days (E) 1856·95 – 1680·65 – 96·3 = 80
3 1
Therefore, the required answer is part C.
46. (D) Let the smaller number be x and the bigger
56. (A) Let the principal amount be Rs. P.
number be y, then
P × 8 × 4 P × 10 × 6 P × 12 × 5
x + y = 3x + + = 12160
100 100 100
y–x = x
32P 60P 60P
Therefore, y = 3x + + = 12160
100 100 100
and so the value of x and cannot be calculated. Hence, 152 P = 1216000
the data is inadequate.
1216000
47. (D) Since the expenditure of each year that is 1994 P =
152
and 1995 is not given separately. Hence, their income
cannot be calculated as the data is inadequate. = Rs. 8000

Quantitative Aptitude Test | 372


57. (D) (A) (a + b) (a – 2b) = a2 – ab – 2b2 63. (C) 2 3 8 27
(B) a2 – b(a + 2b) = a2 – ab – 2b2
×1+1 ×2+2 ×3+3
(C) a2 + b2 – ab – 3b2 = a2 – ab – 2b2
Similarly,
(D) (a + b) (a – b) + ab = a2 – b2 + ab
a b c d e
(E) (a – b) 2 – b(a – 3b) = a2 – ab – 2b2 5 6 14 45 184 925
Therefore, the required answer is part D.
×1+1 ×2+2 ×3+3 ×4+4 ×5+5
58. (C) Total age of 24 students and the class teacher
Therefore, the number 925 will come in place of (e).
= 16 × 25 = 400 years
64. (D) 2 3 9 40·5
Total age of 24 students = 24 × (16 – 1)
× 1·5 ×3 × 4·5
= 24 × 15 = 360 years
∴ Age of the class teacher = 400 – 360 Similarly,
= 40 years a b c d e
4 6 18 81 486 3645
59. (C) ? = 5679 + 1438 – 2015
? = 7117 – 2015 × 1·5 ×3 × 4·5 ×6 × 7·5

? = 5102 Therefore, the number 18 will come in place of (b).


60. (B) Let the length and breadth of the rectangular plot 65. (A) 12 28 64 140
be 7x m and 5x m respectively.
×2+4 ×2+8 × 2 + 12
Perimeter = 2(l + b)
144 = 2(7x + 5x) Similarly,
a b c d e
144 = 24x 37 78 164 340 696 1412
x = 6 metres
×2+4 ×2+8 × 2 + 12 × 2 + 16 × 2 + 20
Length = 7x
Therefore, the number 1412 will come in place of
= 7 × 6 = 42 metres (e).
Breadth = 5x 66. (B) 5 12 60 340
= 5 × 6 = 30 metres
× 4 – 8 × 4 + 12 × 6 – 20
Reqd. area = (42 × 30) sq. m.
Similarly,
= 1260 sq. m2
a b c d
61. (A) 159% of 6531·8 + 5·5 × 1015·2 = ? + 5964·9 7 20 92 532 3222
159 × 6531·8
+ 5·5 × 1015·2 = ? + 5964·9 × 4 – 8 × 4 + 12 × 6 – 20 × 6 + 30
100
10385·562 + 5583·6 = ? + 5964·9 Therefore, the number 3222 will come in place of (d).
67. (B) Total of the exports and domestic consumption
15969·162 = ? + 5964·9
In 1988 = (7·5 + 15) thousand tonnes
? = 15969·162 – 5964·9
= 22·5 thousand tonnes
= 1004·262 = 10000 In 1989 = (10 + 15) thousand tonnes
62. (A) 11 15 38 126 = 25 thousand tonnes
In 1990 = (12·5 + 17·5) thousand tonnes
×1+4 ×2+8 × 3 + 12
= 30 thousand tonnes
Similarly, In 1991 = (20 + 22·5) thousand tonnes
a b c d e = 42·5 thousand tonnes
7 11 30 102 424 2140 In 1992 = (20 + 30) thousand tonnes
= 50 thousand tonnes
×1+4 ×2+8 × 3 + 12 × 4 + 16 × 5 + 20
In 1993 = (25 + 35) thousand tonnes
Therefore, the number 102 will come in place of (c). = 60 thousand tonnes

Quantitative Aptitude Test | 373


In 1994 = (15 + 20) thousand tonnes Average percentage of marks obtained by C in all the
= 35 thousand tonnes 680
six subjects = × 100 = 68%
1000
Clearly the exports and domestic consumption is
highest in 1993. ∴ Required difference = 75 – 68 = 7
68. (A) Domestic consumption in 1990 75. (A) Percentage of marks obtained by student D
= 12·5 thousand tonnes 85
In History = × 100 = 56·66%
Domestic consumption in 1991 150
= 22·5 thousand tonnes 115
In Science = × 100 = 57·5%
200
∴ Difference = (22·5 – 12·5) thousand tonnes
95
= 10 thousand tonnes In Geography = × 100 = 47·5%
100
Export in 1991 = 20 thousand tonnes
125
Export in 1992 = 30 thousand tonnes In English = × 100 = 62·5%
200
∴ Difference = (30 – 20) thousand tonnes 65
= 10 thousand tonnes In Hindi = × 100 = 65%
100
Therefore, the required answer is 1991 and 1992. 135
In Maths = × 100 = 67·5%
69. (D) Since the domestic consumption and export in 200
1995 is not given hence the data is inadequate to Hence, it is clear that student D got more than 70%
answer the question. in none of the subjects.
70. (C) Fluctuations in Exports from 76. (A) Average percentage of marks obtained by six
Year Previous Year students in English

1989
(Increase/Decrease)
10 – 15 = – 5
= ( 6 )
70 + 65 + 75 + 62·5 + 72·5 + 55
%

1990 17·5 – 10 = 7·5 400


= = 67%
6
1991 20·0 – 17·5 = 2·5
1
1992 30 – 20·0 = 10 77. (B) (A) 40% of 160 + of 240 = 144
3
1993 35 – 30 = 5
(B) 120% of 1200 = 1440
1994 15 – 35 = –20
(C) 38 × 12 – 39 × 8 = 144
Clearly, the fluctuation in exports is maximum in
1994 from previous year. (D) 1648 – 938 – 566 = 144
71. (D) Average domestic consumption 1
(E) 6 of 140 – 2·5 × 306·4 = 144
7·5 + 10 + 17·5 + 20 + 30 + 35 + 15 2
=
7 Therefore, the required answer is part B.
142·5 78. (A)
= = 20·35 thousand tonnes
7 8 A 2 + 3 B 5 + C 4 = 1271
∴ Difference = (20·35 – 17·5) thousand tonnes
= 2·85 thousand tonnes = 2850 tonnes
Step. 1 : 2 + 5 + 4 = 11
72. (B) Percentage of marks obtained by student B in
105 Step 2 : 8 + 3 = 11
History = × 100 = 70%
150 From above it is clear that (A + B + C) must be equal
Percentage of marks obtained by student C in Hindi to 16. Since the digits 1234567 and 8 are already
used in equation. Therefore, only single digit number
90 which can be used for AB and C is 6 and 9. Hence in
= × 100 = 90%
100 place of C we cannot use the number 7.
∴ Required difference = (90 – 70) = 20 79. (D) Let the two digits number be xy that is 10x + y.
73. (D) Percentage of marks obtained by student F in
1
160 10x – y = [10x + y – 10y – x]
Science = × 100 = 80% 9
200
1
170 or 10x – y = [10x + y – 10y – x]
Required answer = = 21·25 9
80
1
74. (C) Percentage of marks obtained by student C in or 10x – y = [9x – 9y]
9
150
English = × 100 = 75% or 10x – y = x – y
200

Quantitative Aptitude Test | 374


Since the number of equations formed is one and the 12 T + 21 C = Rs. 36270
number of unknown quantities are two, therefore we = Rs. 36000
can’t calculate the required answer. Hence, data is

80.
inadequate.
(B) Let the first number be x and the second number
85. (A) Required value =
253 (
4263·05
× 39 )
= Rs. 657·15
be y.
~
– Rs. 650
0·35x + y = 1·2y
or 0·35x = 1·2y – y 86. (D) 2x + 2y = 12 + y + √ ⎯⎯⎯⎯⎯⎯⎯
(12) 2 + y2
or 0·35x = 0·20y The number of equation formed is one and the
number of unknown quantities are two. Therefore,
0·35 35 7 we cannot calculate the required answer. Hence, data
y = = =
0·20 20 4 is inadequate.
Thus, y : x = 7 : 4. 2
87. (D) 18 of 150·8 + ? = 8697·32 – 3058·16
81. (B) Let the three consecutive even numbers be 5
x (x + 2) and (x + 4). 92
× 150·8 + ? = 8697·32 – 3058·16
176 5
x+x+2+x+4 = – 14
4 2774·72 + ? = 5639·16
3x + 6 = 44 – 14 ? = 5639·16 – 2774·72
3x = 30 – 6 = 2864·44
24 88. (C) Length and breadth of a rectangle are different
x = =8
3 from one another length is always more than the
The middle number is x + 2 that is breadth. Since the square of diagonal
= (length) 2 + (breadth)2
8 + 2 = 10.
∴ (length) + (breadth) = (64 + x2)
2 2
P 5
82. (D) = = (82 + x2)
Q 8
⇒ 8P = 5Q …(i) So, length will be 8 cm.
P+4 2 89. (A) Cost price per kg of mixture
=
Q+4 3 25 × 16·50 + 35 × 24·50
=
3 P + 12 = 2 Q + 8 (25 + 35)
3P–2Q = –4 …(ii) = Rs. 21·16
After putting the value of P from equation (i) and (ii) For 25% profit the selling price of mixture
we get = Rs. 21·16 × 1·25 = Rs. 26·45
3( )5Q
8
–2Q = –4
90. (D)
= Rs. 26·50
×2–2
×2–2
×2–2
15
Q–2Q = –4 50
8 8 14 26 48 98 194 386
or 15 Q – 16 Q = – 32
– Q = – 32 ×2–2 ×2–2 ×2–2
Q = 32 years ∴ The wrong term is 48.
83. (C) Let the number be x. 3
91. (D) 3 of 157·85 + 39% of 1847 = ? – 447·30
2 1 5
of 30% of of x = 15
5 4 18 39 × 1847
× 157·85 + = ? – 447·30
2 30 1 5 100
or × × × x = 15
5 100 4 1288·59 = ? – 447·30
or 3 x = 1500 ? = 1288·59 – 447·30
∴ x = 500 = 1735·89
20 × 500 = 1800
∴ 20% of x =
100 92. (C) Sales in 1992 = 10 lakh tonnes
= 100 Sales in 1995 = 9 lakh tonnes
84. (D) 4 T + 7 C = Rs. 12090 ∴ Required difference = (10 – 9) lakh tonnes
∴ 3 (4 T + 7 C) = Rs. 3 × 12090 = 1 lakh ton

Quantitative Aptitude Test | 375


93. (A) Sales in 1991 = 7 lakh tonnes The percentage of sales to production in 1996
Sales in 1992 = 10 lakh tonnes 17
× 100 = 94
∴ Combined sales of 1991 and 1992 18
= (7 + 10) lakh tonnes Hence, the percentage of sales to production was
= 17 lakh tonnes maximum in 1996.
Sales in 1994 = 12 lakh tonnes 96. (B) Required percentage drop
17 (10 – 9)
∴ Required percentage = × 100 = × 100 = 10%
12 10
= 141·66 = 140% 97. (D) Ratio between the ages of Rati, Madhuri and
94. (B) Production in 1993 = 11 lakh tonnes Kajri = 4:5:6
Production in 1994 = 16 lakh tonnes ∴ Sum of ratios = 4 + 5 + 6 = 15
∴ Required percentage = ( 11 )
16 – 11
× 100 ∴ Age of Kajri =
6
15
× 60
= 45·45% –~ 45% = 24 years
95. (C) The percentage of sales to production in 1991 98. (D)
7 99. (B) Let the number be x.
× 100 = 87·5
8 1
The percentage of sales to production in 1992 ∴ of x = 12
14
10 or x = 12 × 14 = 168
× 100 = 83·3
12 168 × 150
The percentage of sales to production in 1993 ∴ 150% of 168 =
100
9 = 252
× 100 = 81·8
11 100. (D) If 20272 is divided by 13 the remainder is 5.
The percentage of sales to production in 1995 Hence, from the number 20272 if (13 – 5) i.e., 8 is
9 added then the result will be completely divisible by
× 100 = 64·2
14 13.

⎯⎯⎯⎯⎯⎯⎯⎯⎯⎯⎯⎯

Quantitative Aptitude Test | 376

You might also like